Vous êtes sur la page 1sur 1364

JC-2 Examination Papers

2013

Physics
College H1 H2
Yishun Junior College P1 P2 P1 P2 P3
Victoria Junior College P1 P2 P1 P2 P3
Tampines Junior College P1 P2 P1 P2 P3
Temasek Junior College P1 P2 P1 P2 P3
Serangoon Junior College P1 P2 P1 P2 P3
St. Andrews Junior College P1 P2 P1 P2 P3
River Valley High School P1 P2 P1 P2 P3
Raffles Institution P1 P2 P3
Pioneer Junior College P1 P2 P1 P2 P3
Nanyang Junior College P1 P2 P1 P2 P3
Candidates Name . CTG .

YISHUN JUNIOR COLLEGE


JC 2 Preliminary Examinations 2013

PHYSICS 9646/1
th
HIGHER 2 Thursday 29 August 2013
1 hour 15 minutes
Paper 1 Multiple Choice

Additional Materials: Multiple Choice Answer Sheet

YISHUN JUNIOR COLLEGE YISHUN JUNIOR COLLEGE YISHUN JUNIOR COLLEGE YISHUN JUNIOR COLLEGE YISHUN JUNIOR COLLEGE
YISHUN JUNIOR COLLEGE YISHUN JUNIOR COLLEGE YISHUN JUNIOR COLLEGE YISHUN JUNIOR COLLEGE YISHUN JUNIOR COLLEGE
YISHUN JUNIOR COLLEGE YISHUN JUNIOR COLLEGE YISHUN JUNIOR COLLEGE YISHUN JUNIOR COLLEGE YISHUN JUNIOR COLLEGE
YISHUN JUNIOR COLLEGE YISHUN JUNIOR COLLEGE YISHUN JUNIOR COLLEGE YISHUN JUNIOR COLLEGE YISHUN JUNIOR COLLEGE
YISHUN JUNIOR COLLEGE YISHUN JUNIOR COLLEGE YISHUN JUNIOR COLLEGE YISHUN JUNIOR COLLEGE YISHUN JUNIOR COLLEGE
YISHUN JUNIOR COLLEGE YISHUN JUNIOR COLLEGE YISHUN JUNIOR COLLEGE YISHUN JUNIOR COLLEGE YISHUN JUNIOR COLLEGE
YISHUN JUNIOR COLLEGE YISHUN JUNIOR COLLEGE YISHUN JUNIOR COLLEGE YISHUN JUNIOR COLLEGE YISHUN JUNIOR COLLEGE
YISHUN JUNIOR COLLEGE YISHUN JUNIOR COLLEGE YISHUN JUNIOR COLLEGE YISHUN JUNIOR COLLEGE YISHUN JUNIOR COLLEGE
YISHUN JUNIOR COLLEGE YISHUN JUNIOR COLLEGE YISHUN JUNIOR COLLEGE YISHUN JUNIOR COLLEGE YISHUN JUNIOR COLLEGE
YISHUN JUNIOR COLLEGE YISHUN JUNIOR COLLEGE YISHUN JUNIOR COLLEGE YISHUN JUNIOR COLLEGE YISHUN JUNIOR COLLEGE
YISHUN JUNIOR COLLEGE YISHUN JUNIOR COLLEGE YISHUN JUNIOR COLLEGE YISHUN JUNIOR COLLEGE YISHUN JUNIOR COLLEGE

READ THESE INSTRUCTIONS FIRST

Do not open this booklet until you are told to do so.

Write your name and CTG in the spaces provided on this cover page and on the Answer Sheet.

Shade your NRIC in the space provided on the Answer Sheet.

There are forty questions in this paper. Answer all questions. For each question there are four
possible answers A, B, C and D.
Choose the one you consider correct and record your choice in soft pencil on the separate
Answer Sheet.

Read the instructions on the Answer Sheet very carefully.

Each correct answer will score one mark. A mark will not be deducted for a wrong answer.
Any rough working should be done in this booklet.

This question paper consists of 17 printed pages


2

Data

speed of light in free space, c = 3.00 108 m s1


permeability of free space, o = 4 107 H m1
permittivity of free space, o = 8.85 1012 F m1
(1/(36)) 109 F m1
elementary charge, e = 1.60 1019 C
the Planck constant, h = 6.63 1034 J s
unified atomic mass constant, u = 1.66 1027 kg
rest mass of electron, me = 9.11 1031 kg
rest mass of proton, mp = 1.67 1027 kg
molar gas constant, R = 8.31 J K1 mol1
the Avogadro constant, NA = 6.02 1023 mol1
the Boltzmann constant, k = 1.38 1023 J K1
gravitational constant, G = 6.67 1011 N m2 kg2
acceleration of free fall, g = 9.81 m s2

Formulae
1 2
uniformly accelerated motion, s = ut + at
2
v2 = u2 + 2as
work done on/by a gas, W = p V
hydrostatic pressure, p = gh
gravitational potential, Gm
=
r
Displacement of particle in s.h.m. x = xo sin t
velocity of particle in s.h.m., v = vo cos t
= ( x o2 x 2 )
resistors in series, R = R1 + R2+.
resistors in parallel, 1 1 1
= + + ........
R R1 R 2
electric potential, Q
V =
4 o r
alternating current/voltage, x = xo sin t
transmission coefficient T = exp(2kd)
8 2 m(U E )
where k =
h2
radioactive decay, x = xo exp(t)
0.693
decay constant, = t1
2

9646 / YJC / 2013 / Preliminary Examinations / Paper 1


3

1 A thunderstorm drops 2 cm of rain on a basketball court of area 400 m2. Estimate the
number of raindrops that fell during the storm.

A 103

B 106

C 109

D 1015

2 What is the equivalent speed of a beam of light travelling at 3.00 108 m s1?

A 0.108 Tm h1

B 1.08 Tm h1

C 10.8 Tm h1

D 108 Tm h1

3 A student measures the length, L and breadth, B of a rectangular room using a measuring
tape. The readings are L = (5.5 0.2) m and B = (9.8 0.2) m.

Which of the following gives the percentage error of the perimeter of the room?

A 1%

B 2%

C 3%

D 4%

4 Light of intensity, I falls on a surface area A. The energy absorbed by the surface is E. I
is constant for a time duration t. Given the formula E = IAt, which of the following gives
the base units of I?

A Js

B J s1 m2

C kg m s1

D kg s3

9646 / YJC / 2013 / Preliminary Examinations / Paper 1


4

5 A criminals car was racing along a highway at 20 m s1. As it rounded a bend, it passed
by a parked police car. 5.0 s later, the police car started to accelerate at 7.5 m s2
whereas the criminals car also started accelerating at 5.0 m s2. How much time did the
police car take, from the moment it started moving, to overtake the criminals car?

A 4.0 s B 8.9 s

C 11 s D 20 s

6 A steel sphere is held just below the surface of a deep tank of water and released. Which
one of the following best illustrates the relationship between the acceleration a, and the
displacement s, of the sphere? [Take g as 9.8 m s2]

A B
9.8 9.8
a / m s2

a / m s2
0 0
s s

C D
9.8 9.8
a / m s2
a / m s2

0 0
s s

7 A trolley of mass 0.50 kg moves with a certain acceleration down a runway which is
inclined to the horizontal at 15. If the angle of inclination is increased to 20, the
acceleration of the trolley would be doubled. Find the average frictional force, assumed to
be the same in both cases, acting on the trolley.

A 0.55 N B 0.82 N

C 0.86 N D 1.0 N

9646 / YJC / 2013 / Preliminary Examinations / Paper 1


5

8 A block of mass 5.0 kg is placed on the inclined surface of the wedge shown below. All
contact surfaces are assumed to be smooth. What is the magnitude of the horizontal force
F exerted on the wedge so that it remains stationary while the block is accelerating down
the inclined surface?

30 cm
F
40 cm

A 0N B 17 N

C 24 N D 31 N

9 The resultant of two forces F1 and F2 acting at a point can have a minimum value of 1 N
and a maximum value of 7 N. When the two forces act at right angles to each other, the
magnitude of their resultant is

A 3N B 5N

C 6N D 8N

10 P, Q and R are three identical spheres moving with the same


speed along a smooth horizontal track. They undergo head-on P X
elastic collisions with spheres X, Y and Z respectively which are
initially at rest.
Q Y
After collision, P continues to move in its original direction, Q
becomes stationary while R reverses its motion. After collision, R Z
which sphere (X, Y or Z) has the greatest magnitude of
momentum and kinetic energy?

Momentum Kinetic Energy

A Y Z

B Y Y

C Z X

D Z Y

9646 / YJC / 2013 / Preliminary Examinations / Paper 1


6

11 Two books A and B are placed on a horizontal table surface as shown. A horizontal force
F is applied to A but the system remains stationary. Which of the following statements is
incorrect?

F A

A The frictional force acting on B by A is towards the right.

B The system would also remain stationary if F is applied to B instead.

C The frictional force acting on B by the table surface is greater than F.

D All frictional forces acting on/within the system have equal magnitudes.

12 The figure shows two blocks A and B, each of mass m, connected by two
light springs to a fixed support. Each spring has a force constant k. What
is the total extension of the system when it is at static equilibrium?

A mg/(2k)
A
B 3mg/(2k)

C 2mg/k

D 3mg/k B

13 In the figure below, X and Y are blocks of mass 1 kg and 2 kg respectively. S is a spring
balance of negligible mass and P is a smooth pulley fixed at the top of two smooth
inclined planes. What is the reading of S when X is held stationary by an external force?
P
S

X Y

30 30

A 5N B 10 N

C 15 N D 20 N

9646 / YJC / 2013 / Preliminary Examinations / Paper 1


7

14 Ag golfer badly
y misjudgess a putt, sen nding the ball
b only one e-quarter of
o the distannce to the
hole. The origiinal putt gavve the ball an
a initial speed of u, allong a straig
ght line on the
t grass.
If th
he force of resistance due to the grass is co onstant, what would be e the minimmum initial
spe eed needed d to get the ball to the hole
h from itss original po
osition?

d
d

A 2u B 3u

C 4u D 8u

15 Wh wn has the greatest gravitational field strength on its


hich of the following planets show
surrface?

A B
R R

M 2
2M

C D

2R 2R

M 2M
M

16 A ssatellite is shifted
s from a stable orrbit to anoth hich is higher. Which one
her orbit wh o of the
follo
owing quan ntities increa
ases?

A Gravita
ational force
e

B Gravita
ational potential energyy

C Linear speed

D c energy
Kinetic

9646 / YJC / 2013 / Preliminary


P Exa
aminations / Pap
per 1
8

17 A particle P performs simple harmonic motion between points X and Y which are 4.0 cm
apart. The time taken to move from X to Y is 0.80 s. What is the maximum speed of
particle P?

A 5.0 102 m s1

B 7.9 102 m s1

C 16 102 m s1

D 34 102 m s1

18 A pendulum is constructed from a fixed length of light thread and a spherical, polystyrene
bob of low density. It is forced to oscillate in air at different frequencies f. The following
diagram shows how the amplitude of its oscillation varies with f.
amplitude

f
f
If the experiment is repeated in a partial vacuum, which graph best represents the
variation with f of the amplitude?

A amplitude B amplitude

ao ao

f f
fo fo

C D
amplitude amplitude

ao ao

f f
fo fo

9646 / YJC / 2013 / Preliminary Examinations / Paper 1


9

19 Thee displacem ment-time equation


e of an oscillating system is given by
b x = 1.2 ccos(2.0 t)
whe ere x is in metres
m and t is in seconds. What is the total energy
e he system if its mass
of th
is 1.5
1 kg?

A 1.1 J B 3.0 J

C 4.3 J D 18 J

20 Ad nterference experiment is set up as


double-slit in a shown.

Frin
nges are foormed on the
t screen.. The dista en successsive bright fringes is
ance betwee
fou
und to be 3 mm.
m

Tw
wo changes are then made to the experimenta
e ment. The double slit iss replaced
al arrangem
by another wh hich has hallf the spacin
ng. The scrreen is mov
ved so that its distancee from the
dou
uble-slit is tw
wice as gre
eat.

Wh
hat is now th
he distance between successive bright
b fringe
es?

A 1 mm B 3 mm

C 6 mm D 12 mm
m

21 Addiffraction grating
g with N lines perr metre
is used to deflect light of various v
wavvelengths .

The e graph sho ows the relation betwe een the


defflection anggles for diffferent values of
in tthe nth orde
er interferen
nce pattern.

Wh
hat is the gradient of the graph?

N
A Nn B
n

n 1
C D
N Nn

9646 / YJC / 2013 / Preliminary


P Exa
aminations / Pap
per 1
10

22 A proton is released from rest at point Q, where the potential is 0 V. Which statement
describes the subsequent motion of the proton?
100 V 0V +100 V

P Q R

A Moves towards P with a steady speed

B Moves towards P with increasing speed

C Moves towards R with a steady speed

D Moves towards R with increasing speed

23 Which set of equipotential lines best corresponds to the following electric field pattern?

A B
0V 10 V 20 V 30 V 40 V 50 V 0 V 10 V 20 V 30 V 40 V 50 V

C D
50 V 40 V 30 V 20 V 10 V 0 V 50 V 40 V 30 V 20 V 10 V 0V

9646 / YJC / 2013 / Preliminary Examinations / Paper 1


11

24 A ccell is conne
ected to a variable resistor as show
wn.

Thee variation of the voltm ng V with ammeter rea


meter readin ading I is as
s shown in the graph
bellow.

hat are the values


Wh v of th
he e.m.f. of the
t cell and
d its internall resistance?

E.m.f / V Internal resistance


r /
A 2.0 1.0

B 1.5 1.0

C 1.5 2.0

D 1.0 2.0

9646 / YJC / 2013 / Preliminary


P Exa
aminations / Pap
per 1
12

25 Thee diagram shows


s a light-depende
ent resistor (LDR) and a thermisto
or forming a potential
divider.

Under which set


s of conditions will th
he potentiall difference across the
e thermistorr have the
eatest value
gre e?

Illumina
ation Temperature

A low low
w

B high low
w

C low hig
gh

D high hig
gh

26 In tthe potentio
ometer circu
uit shown be alance lengtth, l was found to be to
elow, the ba oo short.

Driver Cell

l
P Q

l ca
an be increa
ased by

A adding a resistor in series witth the driverr cell.

B conneccting a prote
ective resisttor in seriess with the ga
alvanomete
er.

C changing the drive


er cell to one with large
er e.m.f.

D replacing the slide


e wire PQ with
w one thatt has higher resistance
e.
9646 / YJC / 2013 / Preliminary
P Exa
aminations / Pap
per 1
13

27 A sstraight, horizontal, cu
urrent-carrying wire liess at right angles
a to a horizontal magnetic
field d exerts a vertical forrce of 16 m
d. The field mN on the wire. The wire is the en rotated
throough 60 ass shown in the diagram m below. Th he flux dens sity of the magnetic
m fie
eld is also
halved.

60

w
wire wire

Wh
hat is the ma
agnitude off the magne
etic force accting on the wire?

A 4.0 mN
N B N
8.0 mN

C 9.2 mN
N D 16 mN
N

28 ong straigh
A lo ht wire XY lies in the same
s planee as a squa are loop of wire PQRS
S which is
e to move. The sides PS and QR
free t XY. The wire and loop carry
R are initiallly parallel to
steady currentts as shown
n in the diag
gram below.

Wh
hat is the efffect (if any) on the loop
p PQRS?

A It will move
m ds the long wire.
toward

B It will move
m away from
f the lon
ng wire.

C It will ro
otate about an axis parrallel to XY

D It will not be affectted.

9646 / YJC / 2013 / Preliminary


P Exa
aminations / Pap
per 1
14

29 The e diagram below


b show
ws an electtron travellin
ng horizonttally into a region of uniform
u B-
field
d.


direction off B-field
initial path
p of electtron

In o
order for thee electron to
t remain undeflected within the region
r of B--field, the diirection of
d that needs to be app
extternal E-field plied within the
t same reegion is

A upward
ds B downw
wards

C into the
e plane D out of the plane

30 A uniform
u mag gnetic field of flux dens es normallyy through a plane area A. In this
sity B passe
plane lies a coil
c of eightt turns of wire,
w each of area A. Calculatte the mag gnetic flux
linkkage for the
e coil.

A 0.25 BA
A B BA

C 2.0 BA D 8.0 BA
A

31 Thee figures be
elow involve
e a cylindric
cal magnet and
a a loop ofo copper wire.
w The pla
ane of the
wire
e loop is peerpendicula
ar to the axis. The state
e of motion of the mag
gnet and wiire loop is
indicated in the diagram below.
b

I:

Loop with
h its radius decreasing
d

II:

Wh
hich setup would
w produ
uce an inducced e.m.f in
n the loop?

A I only B II only

C I and III D None

9646 / YJC / 2013 / Preliminary


P Exa
aminations / Pap
per 1
15

32 In the diagram shown, the average power dissipated across a 2.0 resistor is 50 W.

50
2000 turns
2.0
turns

What is the r.m.s. potential difference across the primary coil of the ideal transformer?

A 20 V B 40 V

C 200 V D 400 V

33 The figure below shows the circuit diagram for a half-wave rectifier. The a.c. supply to the
rectifier is rated as 50 Hz , 6.0 V r.m.s. What is the average power dissipated in the load
resistor of resistance R = 2.0 ?

50 Hz R
6.0 Vr.m.s

A 9.0 W B 18 W

C 25 W D 36 W

34 A particular collision between an electron and an atom leads to excitation of the atom
without ionization. Which of the following statements is incorrect?

A An orbital electron gains energy.

B An electron is emitted from the nucleus.

C The energy transferred appears later as electromagnetic radiation.

D The colliding electron can transfer part of its energy or all of its energy to the atom.

9646 / YJC / 2013 / Preliminary Examinations / Paper 1


16

35 The transition of electrons between three consecutive energy levels in a particular atom
gives rise to three spectral lines. The shortest and longest wavelengths of those spectral
lines are 1 and 2 respectively. The wavelength of the other spectral line is

A (1 + 2)/2

B 1 2

C (12)/( 1 + 2)

D (1/1 1/2)1

36 An X-ray tube emits X-rays with a minimum wavelength of 3.55 1011 m. Estimate the
potential difference between the cathode and the anode (target) in the X-ray tube.

A 25000 V B 30000 V

C 35000V D 40000 V

37 Which of the following statements concerning a laser system is false?

A An external energy source is needed to create population inversion.

B The laser beam produced is coherent and of a single wavelength.

C By changing the reflective coefficient of the partially reflecting mirror, the intensity
of the laser beam can be varied.

D High-end laser systems can produce collimated beams, i.e. beams that would not
spread at all.

38 Which of the following statements about the energy gap of an intrinsic semiconductor is
incorrect?

A The energy gap is the energy separation between the bottom of the conduction
band and the top of the valence band.

B The energy gap usually carries a magnitude of 1019 J.

C The energy gap can vary between different elements under Group IV of the
periodic table.

D The energy gap can be reduced by introducing dopant atoms.

9646 / YJC / 2013 / Preliminary Examinations / Paper 1


17

39 Which of the following statement best describes the electrical conduction process of a
semiconductor?

A The movement of holes makes up the conduction process.

B The movement of electrons makes up the conduction process.

C The resistivity of the semiconductor decreases with increasing temperature.

D The resistivity of the semiconductor increases with increasing light intensity.

40 The graph below shows how the logarithm of the activity A of a radioactive isotope varies
with time t. What is the half-life of the isotope?

ln (A/Bq)
5

0
400 t/s

A 200 s B 55 s

C 24 s D 0.42 s

End of Paper

9646 / YJC / 2013 / Preliminary Examinations / Paper 1


18

9646 / YJC / 2013 / Preliminary Examinations / Paper 1


2013YJCH2PHPrelimPaper1KeyAnswers

1 C 11 C 21 A 31 A
2 B 12 D 22 B 32 D
3 C 13 B 23 B 33 A
4 D 14 A 24 A 34 B
5 D 15 B 25 B 35 D

6 C 16 B 26 A 36 C
7 C 17 B 27 A 37 D
8 C 18 C 28 A 38 D
9 B 19 C 29 B 39 C
10 D 20 D 30 C 40 B
Candidates Name .. CTG ..

YISHUN JUNIOR COLLEGE


JC 2 Preliminary Examinations 2013

PHYSICS 9646/2
rd
HIGHER 2 Friday 23 August 2013

Paper 2 Structured Questions 1 hour 45 minutes


Candidates answer on the Question Paper.
No Additional Materials are required.

YISHUN JUNIOR COLLEGE YISHUN JUNIOR COLLEGE YISHUN JUNIOR COLLEGE YISHUN JUNIOR COLLEGE YISHUN JUNIOR COLLEGE
YISHUN JUNIOR COLLEGE YISHUN JUNIOR COLLEGE YISHUN JUNIOR COLLEGE YISHUN JUNIOR COLLEGE YISHUN JUNIOR COLLEGE
YISHUN JUNIOR COLLEGE YISHUN JUNIOR COLLEGE YISHUN JUNIOR COLLEGE YISHUN JUNIOR COLLEGE YISHUN JUNIOR COLLEGE
YISHUN JUNIOR COLLEGE YISHUN JUNIOR COLLEGE YISHUN JUNIOR COLLEGE YISHUN JUNIOR COLLEGE YISHUN JUNIOR COLLEGE
YISHUN JUNIOR COLLEGE YISHUN JUNIOR COLLEGE YISHUN JUNIOR COLLEGE YISHUN JUNIOR COLLEGE YISHUN JUNIOR COLLEGE
YISHUN JUNIOR COLLEGE YISHUN JUNIOR COLLEGE YISHUN JUNIOR COLLEGE YISHUN JUNIOR COLLEGE YISHUN JUNIOR COLLEGE
YISHUN JUNIOR COLLEGE YISHUN JUNIOR COLLEGE YISHUN JUNIOR COLLEGE YISHUN JUNIOR COLLEGE YISHUN JUNIOR COLLEGE
YISHUN JUNIOR COLLEGE YISHUN JUNIOR COLLEGE YISHUN JUNIOR COLLEGE YISHUN JUNIOR COLLEGE YISHUN JUNIOR COLLEGE
YISHUN JUNIOR COLLEGE YISHUN JUNIOR COLLEGE YISHUN JUNIOR COLLEGE YISHUN JUNIOR COLLEGE YISHUN JUNIOR COLLEGE
YISHUN JUNIOR COLLEGE YISHUN JUNIOR COLLEGE YISHUN JUNIOR COLLEGE YISHUN JUNIOR COLLEGE YISHUN JUNIOR COLLEGE
YISHUN JUNIOR COLLEGE YISHUN JUNIOR COLLEGE YISHUN JUNIOR COLLEGE YISHUN JUNIOR COLLEGE YISHUN JUNIOR COLLEGE

READ THESE INSTRUCTIONS FIRST

Write your name and CTG in the spaces provided on this


cover page. For Examiners Use
Write in dark blue or black pen on both sides of the paper.
You may use a soft pencil for any diagrams, graphs or rough 1 /8
working.
2 /7
Do not use staples, paper clips, highlighters, glue or
correction fluid. 3 /6
Answer all questions. 4 /7

Write your answers in the spaces provided on the question 5 /5


paper.
For numerical answers, all working should be shown clearly. 6 /9

The number of marks is given in brackets [ ] at the end of 7 /18


each question or part question. 8 /12

Penalty

Total /72

This question paper consists of 16 printed pages


2

Data

speed of light in free space, c = 3.00 108 m s1


permeability of free space, o = 4 107 H m1
permittivity of free space, o = 8.85 1012 F m1
(1/(36)) 109 F m1
elementary charge, e = 1.60 1019 C
the Planck constant, h = 6.63 1034 J s
unified atomic mass constant, u = 1.66 1027 kg
rest mass of electron, me = 9.11 1031 kg
rest mass of proton, mp = 1.67 1027 kg
molar gas constant, R = 8.31 J K1 mol1
the Avogadro constant, NA = 6.02 1023 mol1
the Boltzmann constant, k = 1.38 1023 J K1
gravitational constant, G = 6.67 1011 N m2 kg2
acceleration of free fall, g = 9.81 m s2

Formulae
1 2
uniformly accelerated motion, s = ut + at
2
v2 = u2 + 2as
work done on/by a gas, W = pV
hydrostatic pressure, p = gh
gravitational potential, Gm
=
r
Displacement of particle in s.h.m. x = xo sin t
velocity of particle in s.h.m., v = vo cos t
=
( x o2 x 2 )
resistors in series, R = R1 + R2+.
resistors in parallel, 1 1 1
= + + ........
R R1 R 2
electric potential, Q
V = 4 o r
alternating current/voltage, x = xo sin t
transmission coefficient T = exp(2kd)
8 2 m(U E )
where k =
h2
radioactive decay, x = xo exp(t)
0.693
decay constant, = t1
2

9646 / YJC / 2013 / Preliminary Examinations / Paper 2


[Turn over
3

1 (a) A student is hoisting a flag attached to a rope at constant speed up a pole of height
6.5 m above the ground. The flag of mass 0.60 kg was initially at rest on his shoulders
1.0 m above the ground.

(i) Calculate the increase in potential energy of the flag.

potential energy = .........................J [1]

(ii) Estimate the work done by the student in hoisting the flag to the top of the
flagpole. The total resistive force is 1.5 N and is taken to be constant. You may
assume the flag rises at constant speed.

work done =...........................J [2]

(b) The student releases the flag at the same constant speed down the flagpole. He
catches hold of the flag on his shoulders. Estimate the work done by the student,
assuming the total resistive force is the same as in (a).

work done =..........................J [2]

(c) The time taken for the flag to rise up and down the flagpole is 8.0 s. Calculate the
average power.

average power =...........................W [1]

9646 / YJC / 2013 / Preliminary Examinations / Paper 2


[Turn over
4

(d) Discuss whether the instantaneous power is constant throughout the rising and falling
of the flag. [2]

2 A vertical spring supports a mass, as shown in Fig. 2.1.

Fig. 2.1

The mass is displaced vertically then released. The variation with time t of the
displacement y from its mean position is shown in Fig. 2.2.

Fig. 2.2

9646 / YJC / 2013 / Preliminary Examinations / Paper 2


[Turn over
5

A student claims that the oscillatory motion of the mass may be represented by the
equation
y = yo sint

(a) Give two reasons why the use of this equation is inappropriate. [2]

(b) Determine the value of . [2]

= ..................... rad s1

(c) The mass is a lump of plasticine. It is now flattened to increase its surface area.
When suspended from the spring, the large surface area is positioned horizontally.
The plasticine is now displaced downwards by 1.5 cm and then released. On
Fig. 2.2, sketch a graph to show the subsequent oscillations of the plasticine. [3]

3 Fig 3.1 shows an electron and a positron (a positively-charged electron) shot out
back-to-back with equal initial speeds, u. Initially, the two particles have a
separation ri. The particles eventually just reach infinity.

u u

ri

Fig. 3.1

9646 / YJC / 2013 / Preliminary Examinations / Paper 2


[Turn over
6

(a) If the mass of the electron is m and charge of electron is e, write down an expression for

(i) the initial kinetic energy of the electron,

expression: [1]

(ii) the initial potential energy of the electron and positron system.

expression: [1]

(b) State the value of the final kinetic energy of each particle. [1]

final kinetic energy of electron = . J

final kinetic energy of positron = . J

(c) Given that ri = 100 fm, by energy considerations or otherwise, determine the minimum
value of u each particle must have in order to escape from each other.

minimum value of u = m s1 [3]

9646 / YJC / 2013 / Preliminary Examinations / Paper 2


[Turn over
7

4 (a) A circuit is set up to measure the resistance R of a metal wire. The potential difference V
across the wire and the current in the wire are to be measured.
(i) Draw, in the space below, a circuit diagram of the apparatus that could be used to
make these measurements. [3]

(ii) Readings for V and the corresponding current are obtained. The variation with V of
is shown in Fig. 4.1.

Fig. 4.1

Explain how Fig. 4.1 indicates that the readings are subject to
1. systematic error,

...............................................................................................................................

....................................................................................................................... [1]

2. random error,

...............................................................................................................................

....................................................................................................................... [1]
9646 / YJC / 2013 / Preliminary Examinations / Paper 2
[Turn over
8

(i) Use the data from Fig. 4.1 to determine R. Explain your working.

R =.................... [2]

5 (a) X-rays are used to aid medical diagnosis. The X-ray image is formed because denser
materials, such as bones, absorb more X-ray photons, thus causing variation in the
intensity of the image.

Absorption of X-rays causes photoelectric effect. However, the effect is only observed
when the frequency of the radiation is above a threshold frequency. Explain why this is
so. [2]

(b) X-rays of a frequency of 1.70 1018 Hz can be used to form an image of a bone. The
energy required to free a tightly-bound electron from a calcium atom in bone is
9.61 1016 J.
Determine the maximum kinetic energy with which one of these electrons is emitted from
the calcium atom.

maximum kinetic energy = . J [3]

9646 / YJC / 2013 / Preliminary Examinations / Paper 2


[Turn over
9

6 Nuclear power stations use nuclear fission to provide energy for electricity generation.
When a nucleus of U-235 absorbs a neutron, it becomes unstable and undergoes fission,
producing two lighter nuclei and more neutrons. A typical reaction is

92 U + 0 n 56 Ba+ 36 Kr + neutrons
235 1 141 92

The neutrons released can produce more fission reactions if they are captured by other
U-235 nuclei. Thus a reactor in a power station has to contain neutron-absorbing
materials to control the chain reaction.

(a) State and explain how many neutrons are produced in the reaction above. [2]

....................................................

(b) Explain why a nuclear fission reaction such as the one above releases energy. [2]

....................................................

238
An isotope 92 U is also present within the fuel rods of a nuclear reactor. It absorbs
239
neutrons and heavier nuclei are thus produced. 94 Pu (plutonium) is one of these products
239
and is particularly hazardous. 94 Pu has a half-life of 24 400 years and decays by alpha
emission.

(c) A quantity of plutonium is extracted from the fuel rods. Calculate the fraction of
these 239
94 Pu nuclei which remain after one thousand years.

fraction = [3]
9646 / YJC / 2013 / Preliminary Examinations / Paper 2
[Turn over
10

239
(d) Hence, state the percentage decrease in activity due to 94 Pu over this period of
time. [1]

percentage decrease = ..%

(e) In practice, when tracking the decay of a sample of pure 239


94 Pu , it is found that the
activity of the sample will not decrease as much as that in (d) when it is a
thousand years old. Suggest a reason for this. [1]

7 The Geiger-Nuttall theory of -particle emission establishes the relationship between the
half-life of the -particle emitter to the energy E of the -particle. One form of this
relationship is
166
L= 53.5
E 1/ 2

L is a number calculated from the half-life of the emitting nuclide and E is measured in
MeV. Values of E and L for different nuclides are given in Fig. 7.1. (The uncertainties in
the values are not shown.)

1
Nuclide E / MeV L 1/ 2
/ MeV 1 / 2
E
238
U 4.20 17.15 0.488

236
U 4.49 14.87 0.472

234
U 4.82 12.89 0.455

228
Th 5.42 7.78
.
208
Rn 6.14 3.16 0.404

2.75
212
Po 7.39 0.368

Fig. 7.1

9646 / YJC / 2013 / Preliminary Examinations / Paper 2


[Turn over
11

(a) Suggest why a nuclide would emit an -particle. [2]

...................................................................................................................................

...................................................................................................................................

...................................................................................................................................

1
(b) Complete Fig. 7.1 by calculating, using the value of E provided, the value of for the
E 1/ 2
nuclide Th-228. Give your answer to three significant figures.

1
value of 1/ 2
= MeV1/2 [1]
E

1
(c) The graph of Fig. 7.2 below shows the variation with of L.
E 1/ 2

Fig. 7.2

9646 / YJC / 2013 / Preliminary Examinations / Paper 2


[Turn over
12

(i) Identify the data point for the nuclide Rn-208. Label this point R. [1]
(ii) On the graph, mark the point for the nuclide Th-228. Label this point T. [1]
(iii) Draw the best-fit straight-line for all the data points. [1]

(d) (i) Determine the gradient of the line you have drawn in (c)(iii).

gradient = [2]

(ii) Without taking into consideration any uncertainty in the data, state two reasons
why the graph does not agree with the stated relationship for the Geiger-Nuttall
theory.
[2]

..

..

(e) On Fig. 7.2, draw the line that would be expected if the relationship for the Geiger-Nuttall
theory was correct. Label it G. [2]

(f) The uncertainty in the measurement of E for U-238 is 0.03 MeV. Deduce whether this
1
uncertainty is consistent with quoting the value of to three significant figures. [3]
E 1/ 2

9646 / YJC / 2013 / Preliminary Examinations / Paper 2


[Turn over
13

(g) Fig. 7.3 below shows how the potential energy Ep of an -particle varies with distance r
along a line from the centre of a nucleus where Ro is the nuclear radius. Using this graph,
suggest why nuclei that emit high energy -particles have short half-lives. [3]

Ep

0 r
. Ro

Fig. 7.3

9646 / YJC / 2013 / Preliminary Examinations / Paper 2


[Turn over
14

8 A manufacturer of Christmas tree lights wishes to use a low voltage power supply to
operate a set of lights, consisting of a number of identical 12 V, 6 W lamps wired in
parallel as shown in Fig. 8.1.

Fig. 8.1

The e.m.f. of the supply is fixed at 12 V. The supply has an internal resistance of 5 which limits
the maximum current that may be drawn.

Design a laboratory experiment to investigate how the electrical power received by each lamp
varies with the number of lamps which are placed in parallel with the supply. You may assume
that the following apparatus is available, together with any other standard equipment which may
be found in a school science laboratory.

12 V power supply unit


Oscilloscope
Selection of resistors
Selection of low voltage lamps
Stopwatch
Micrometer screw gauge

Your answer should contain a diagram showing how the chosen equipment would be arranged,
together with details of

(a) the procedure to be followed,


(b) how the power delivered would be measured,
(c) the ranges of any meters used, and
(d) any precautions you would take which may improve the accuracy of your experiment. [12]

Diagram

9646 / YJC / 2013 / Preliminary Examinations / Paper 2


[Turn over
15

..

..

..

..

..

..

..

..

..

..

..

..

..

..

..

..

..

..

..

..

..

..

..

..

..

..

..

..

..
9646 / YJC / 2013 / Preliminary Examinations / Paper 2
[Turn over
16

..

..

..

..

..

..

..

..

..

..

..

..

..

..

..

..

..

..

..

..

[12]

--- End of paper ---

9646 / YJC / 2013 / Preliminary Examinations / Paper 2


[Turn over
1 (a
a)(i)
0.60)(9.81)(5.5)= 32J
mgh = (0
(ii)
T = mg + f and WD D = Td
W = (mg+ff)5.5
WD
= (5.9 +1
1.5)(5.5)
= 41J

(b
b)
T + f = mg g and WD = Td cos
WD = (mg-f)(5.5 5) (-1)
= (5..9-1.5) ( 5.5
5) (-1)
= -24 J
(c
c) .

pow
wer = (41-2
24)/8
=2.1W
W

(d
d) No. Th
he power is
s greater on
n the way u
up and sma
aller on the
e way down
n

2 (a) amplitude
a not
n constant, graph is
s -ve cosin
ne (not sine
e)

(b) T = 0.60 s

= 2/T = 10.5 rad/s

(c) smaller amp


plitude , sa
ame period
d , appropriate decrea
ase in amplitude

3 (a) (i))

(ii)

(b)

(c)
10010155m

5.0107

4(a) (i) metal wire


e in series with
w powerr supply an
nd ammeter
voltmeter in parallel with
w metal wire
rheostat in
n series witth power supply or po
otential div
vider arrangement
or variable
e power supply

(ii) 1.no
on-zero inttercept on graph

2. scatter of re
eadings ab
bout the bes
st fit line

(iii)
correction for zero errror observ
ved
use of V an
nd correcteed valuess from grap
ph,

e = V/ = 22 [e.g. 2.2
resistance 2 / 0.1]

5 (a) Energy of the photon ortional to the frequency (E = hff) of the radiation.
ns is propo
Photon energy must be higher than the work
w functio
on of the material
m to liberate
n.
an electron

(b) hf = + mu2
max x Ek = mu u2 max = hf
Ek = (6.63 10 0-34 1.70 1018) - 9.6
61 10-16
= 1..127 10 - 9.61 10
-15
5
1 -16

= 1..66 10-16 J

6(a) 3 ne um of reacttants mass number must


eutrons. Su m be sa
ame as sum
m of
prod
ducts masss number..

(b) Total binding energies of the products


p a greater than total binding energies
are
ctants, hen
of the reac nce actual mass of products
p is
s slightly less
l (by m
m) than
mass of reactants, By cons servation of
o mass-E Energy, th here would d be a
conversionn of the ma ass into en
nergy given quation E = (m)c2, where
n by the eq w c
is the spee
ed of light.
(c) N/N0 = et
= ln2/t = 2.84 105 years
t = 2.84 10-2
N/N0 = e(2.4 10 2) = 0.97

(d) Percentage of plutonium decayed = 100 97 = 3%

Since activity directly proportional to undecayed plutonium, thus 3%

(e) The daughter nuclei from the decayed Pu could also be radioactive, hence
increasing the total activity.

7a The nuclide is unstable as it has too many neutrons and protons

b 0.430

ci R - correct point identified

ii (T is plotted correct to 1/2 square)

iii (straight-line with acceptable fit)

di 171;

d ii x-intercept should be 0.32, not 0.39

or y-intercept should be -53.5, not -66.0

gradient should be 166, not 171

e straight-line with gradient = 166;

having x-intercept of 0.32;

f 1/4.17 = 0.490 and 1/4.23 = 0.486

error in answer is 0.002;

uncertainty is in third place of decimals so 3 significant digits here is


acceptable

Alternative:

Let y = E0.5.

Thus, fractional error in y = 0.5 (fractional error in E).

Thus, y = = 0.002 MeV0.5


g High energy barrier width shorter

transmission coefficient greater

shorter average waiting time before tunnelling

shorter half-life [0]


8.

Independent Variable: Number of lamps placed in parallel to supply

Dependent Variable: Electrical received by each lamp

Control Variable(s): fixed resistance (besides parallel lamps) in circuit, emf of power
source, resistance of lamps.

Variables should be quantifiable. Cannot define as Type of lamps or Material of


wires

[1/2]Diagram:

Includes voltmeter across parallel resistors.

Optional to include an ammeter in series with one lamp. (needed if formula used
needs I)

Addition of switch to basic diagram

Indication of how to include (N) number of lamps in parallel for subsequent


repetitions.

Circuit diagram should not be drawn haphazardly (proper connecting lines rather than
artistic dashes). Proper circuit symbols should be used.

[4/5]Procedure:

Specification of using voltmeter to measure (and record) p.d. across lamps.

Specification of using ammeter to measure (and record) current through lamp.


(Alternatively, to determine resistance using ohmmeter/digital
multimeter/separate circuit and calculate via V/I)

Calculation of power (with formula P = VI supplied)

Vary number of (identical) lamps upon repetition to obtain more sets of data for
power and N

Plot lg P vs lg N (with appropriate equation relationship proposed) to find


unknowns in the proposed equation.

[1/2]Safety:

Switch open to prevent overheating of lamps

Gloves to handle lamps

Do not write as overheating of wires. Usually insignificant.

[1/2]Reliability:

Range of ammeter (0 3A)/voltmeter(0 15V)/Ohmmeter(0-30) used

Switch open to prevent heating of lamps that changes their resistance and/or
prevent drain to power source.

Optional to include ammeter to each of N lamps to find average value of


current.
Candidates Name . CTG .

YISHUN JUNIOR COLLEGE


JC 2 Preliminary Examinations 2013
PHYSICS 9646 / 03
HIGHER 2 26 August 2013

Paper 3 Structured Questions 2 hours

Candidates answer on the question paper.


No additional materials are required.
YISHUN JUNIOR COLLEGE YISHUN JUNIOR COLLEGE YISHUN JUNIOR COLLEGE YISHUN JUNIOR COLLEGE YISHUN JUNIOR COLLEGE
YISHUN JUNIOR COLLEGE YISHUN JUNIOR COLLEGE YISHUN JUNIOR COLLEGE YISHUN JUNIOR COLLEGE YISHUN JUNIOR COLLEGE
YISHUN JUNIOR COLLEGE YISHUN JUNIOR COLLEGE YISHUN JUNIOR COLLEGE YISHUN JUNIOR COLLEGE YISHUN JUNIOR COLLEGE
YISHUN JUNIOR COLLEGE YISHUN JUNIOR COLLEGE YISHUN JUNIOR COLLEGE YISHUN JUNIOR COLLEGE YISHUN JUNIOR COLLEGE
YISHUN JUNIOR COLLEGE YISHUN JUNIOR COLLEGE YISHUN JUNIOR COLLEGE YISHUN JUNIOR COLLEGE YISHUN JUNIOR COLLEGE
YISHUN JUNIOR COLLEGE YISHUN JUNIOR COLLEGE YISHUN JUNIOR COLLEGE YISHUN JUNIOR COLLEGE YISHUN JUNIOR COLLEGE
YISHUN JUNIOR COLLEGE YISHUN JUNIOR COLLEGE YISHUN JUNIOR COLLEGE YISHUN JUNIOR COLLEGE YISHUN JUNIOR COLLEGE
YISHUN JUNIOR COLLEGE YISHUN JUNIOR COLLEGE YISHUN JUNIOR COLLEGE YISHUN JUNIOR COLLEGE YISHUN JUNIOR COLLEGE
YISHUN JUNIOR COLLEGE YISHUN JUNIOR COLLEGE YISHUN JUNIOR COLLEGE YISHUN JUNIOR COLLEGE YISHUN JUNIOR COLLEGE
YISHUN JUNIOR COLLEGE YISHUN JUNIOR COLLEGE YISHUN JUNIOR COLLEGE YISHUN JUNIOR COLLEGE YISHUN JUNIOR COLLEGE
YISHUN JUNIOR COLLEGE YISHUN JUNIOR COLLEGE YISHUN JUNIOR COLLEGE YISHUN JUNIOR COLLEGE YISHUN JUNIOR COLLEGE

INSTRUCTIONS TO CANDIDATES
Write your name and CTG in the spaces at the For Examiners Use
top of this page. Section A
Answer all questions in Section A and two
questions in Section B. 1 /8
Write your answers in the spaces provided on
the question paper. 2 /12
For numerical answers, all working should be 3 /11
shown clearly.
4 /9

INFORMATION FOR CANDIDATES Section B


The number of marks is given in brackets [ ] at 5 /20
the end of each question or part question.
6 /20
7 /20
Penalty

Total /80

This question paper consists of 21 printed pages


2
Data

speed of light in free space, c = 3.00 108 m s1


permeability of free space, o = 4 107 H m1
permittivity of free space, o = 8.85 1012 F m1
(1/(36)) 109 F m1
elementary charge, e = 1.60 1019 C
the Planck constant, h = 6.63 1034 J s
unified atomic mass constant, u = 1.66 1027 kg
rest mass of electron, me = 9.11 1031 kg
rest mass of proton, mp = 1.67 1027 kg
molar gas constant, R = 8.31 J K1 mol1
the Avogadro constant, NA = 6.02 1023 mol1
the Boltzmann constant, k = 1.38 1023 J K1
gravitational constant, G = 6.67 1011 N m2 kg2
acceleration of free fall, g = 9.81 m s2

Formulae
1 2
uniformly accelerated motion, s = ut + at
2
v2 = u2 + 2as
work done on/by a gas, W = pV
hydrostatic pressure, p = gh
gravitational potential, Gm
=
r
Displacement of particle in s.h.m. x = xo sin t
velocity of particle in s.h.m., v = vo cos t
= ?? ( x o2 x 2 )
resistors in series, R = R1 + R2+.
resistors in parallel, 1 1 1
= + + ........
R R1 R 2
electric potential, Q
V = 4 o r
alternating current/voltage, x = xo sin t
transmission coefficient T = exp(2kd)
8 2 m(U E )
where k =
h2
radioactive decay, x = xo exp(t)
0.693
decay constant, = t1
2

9646 / YJC / 2013 / Preliminary Examinations / Paper 3


3
Section A
Answer all the questions in this section.

1 Domestic washing machines such as that in Fig. 1.1


often incorporate washing, rinsing, spinning and
drying. This question is about the spinning action.

Fig. 1.1

(a) The inner drum of a washing machine into which clothes are placed has large
holes around it. Suggest how, when clothes are being spin-dried, water is
removed from the clothes through the holes. [3]

(b) One of the spin speeds of a model of washing machine was listed as 1000 rpm
(revolutions per minute). Calculate the maximum contact force that could be
exerted on a single wet blanket of mass 1.50 kg being spin-dried in the machine.
The radius of the spinning drum is 22.5 cm. [3]

Maximum contact force = N

(c) If clothes are unevenly distributed in the machine, it vibrates slightly as it rotates.
The outer drum within which the spinning drum rotates is attached to the rest of
the framework of the washing machine by springs. Suggest briefly the purpose of
these springs. [2]

.
9646 / YJC / 2013 / Preliminary Examinations / Paper 3
4

2 (a) Explain what is meant by gravitational potential energy. [1]

(b) The minimum velocity required by a body to escape completely from the surface
of the Earth is known as the escape velocity, ve.

(i) Using energy considerations, show that ve = where R is the radius of


the Earth. [3]

(ii) Given that the radius of the Earth is 6400 km, determine the escape velocity
for a rocket launched vertically upwards from the surface of the earth. [2]

Escape velocity = m s-1 [2]

(iii) If the rocket is launched off from the surface of the Earth with initial speed
equal to half the escape speed, describe the possible motion of the rocket.
[2]

(iv) Suppose that an asteroid with zero speed at an infinitely far distance from
the Earth now falls directly towards the Earths surface. Suggest a value for
the speed with which it will strike the Earths surface. [1]

9646 / YJC / 2013 / Preliminary Examinations / Paper 3


5
(v) Fig 2.1 shows the variation of potential energy of the rocket, U with distance
r, from the surface of the Earth. Sketch the variation of the kinetic energy, K
of the rocket with distance r, from the surface of the Earth. [1]

Energy, E

r
U

Fig. 2.1

(c) (i) One theory of atmospheric evolution suggests that the Earth originally had
an atmosphere rich in hydrogen but that, as a result of a major thermal event
in which the temperature rose to about 6000 K, the hydrogen concentration
then fell to a very low level.

Making reference to the mean molecular speeds and escape velocity of


Earth from (b)(ii), explain how this increase of temperature could have led to
a substantial loss of hydrogen. [1]
[The mean molecular speed of hydrogen atoms at 6000 K is about 1.2 x 104
m s-1]

(ii) Suggest why the moon has no atmosphere. [1]

9646 / YJC / 2013 / Preliminary Examinations / Paper 3


6
3(a) Write down a word equation for the first law of thermodynamics. [2]

...

(b) A fixed mass of ideal gas undergoes changes of pressure p and volume V as
shown in Fig. 3.1, starting at L and ending at M. The temperature of the gas at L
is 110 K.
p / 106 Pa

A M
2.0
Fig. 3.1

1.0 B
L

1.0 3.0
V / 103 m3

Calculate the temperature of the gas at state M.

Temperature =.......................K [1]

(c) The internal energy of the gas at state L is 1500 J. Show that the increase in
internal energy from state L to M is 7500 J.

[2]

9646 / YJC / 2013 / Preliminary Examinations / Paper 3


7
(d) The gas changes state from L to M via two different paths:
Path 1: LAM
Path 2: LBM

(i) Calculate the ratio of the work done by the gas via path 1 to that via path 2.

Ratio = ....[2]

(ii) Calculate the ratio of thermal energy required for path 1 to that required for
path 2.

Ratio = . [2]

(iii) With reference to (i) and (ii), suggest and explain which process is more
profitable. [2]

.....

.....

.....

4(a) Describe briefly the diffraction of monochromatic light as it passes through a


diffraction grating. [1]

.................................................................................................................................

..................................................................................................................................

..................................................................................................................................

9646 / YJC / 2013 / Preliminary Examinations / Paper 3


8
(b) White light is incident on a diffraction grating, as shown in Fig. 4.1.

Fig. 4.1

The diffraction pattern formed on the screen has white light, called zero order,
and coloured spectra in other orders.
(i) Explain
1. why the light is white at the zero order position, [1]

..................................................................................................................................

..................................................................................................................................

2. the difference in the positions of red and blue lights in the first-order
spectrum. [2]

..................................................................................................................................

..................................................................................................................................

..................................................................................................................................

(ii) Light of wavelength 625 nm produces a second-order maximum at an angle


of 61.0 to the incident direction.
Determine the number of lines per metre of the diffraction grating.

number of lines = ......................................... m1 [2]

(iii) Calculate the wavelength of another part of the visible spectrum that gives a
maximum for a different order at the same angle as in (ii).

wavelength = .... nm [3]


9646 / YJC / 2013 / Preliminary Examinations / Paper 3
9

Section B
Answer two questions in this section.

5 A stunt rider was engaged by a company to execute some dare devil stunts for a
promotional event.

(a) (i) For the first show, the stunt rider was required to cross over a series of
boats arranged across a channel that is 75 m wide. The land masses on
both sides of the channel have an elevation difference of 10 m as shown
in Fig. 5.1. Given that he is able to launch at a speed of 100 km h1,
determine the possible values of angle that the launching ramp should
be set up so that he just manages to reach the opposite land mass. [4]

ramp
10 m
ramp
Land channel width = 75 m
Land

boats

Fig. 5.1

Possible values of = .

(ii) Suggest why a second ramp was also set up at the lower land mass for the
rider to land. [1]

..................................................................................................................................

..................................................................................................................................

9646 / YJC / 2013 / Preliminary Examinations / Paper 3


10
(b) The stunt rider was next tasked to ride along the walls of a circular room of radius
5.0 m as shown in Fig. 5.2. Given that the frictional force f between the wheels
and the wall is directly proportional to the normal contact force NC, given by the
relation f = 0.45NC, calculate the speed the rider needs so as to stay at a height
of 2.0 m above the ground. [3]
(You may assume the rider and cycle together can be represented by a point
mass)
5.0 m

Fig. 5.2
2.0 m

Speed = m s1

(c) In another segment of the event, the stunt rider was required to ride into a barrier
at a constant speed of 19.5 m s1 and be thrown over a cliff. A safety net was
placed beyond the barrier as shown in Fig. 5.3 so that he can land safely.

barrier

safety net

Fig. 5.3

(i) If the 150 kg motorcycle comes to an immediate stop after collision with the
barrier, explain whether the principle of conservation of momentum can be
used to calculate for the speed at which the stunt rider gets thrown over the
barrier. [2]

..................................................................................................................................

..................................................................................................................................

..................................................................................................................................

9646 / YJC / 2013 / Preliminary Examinations / Paper 3


11

(ii) The stunt rider gets thrown horizontally off the cliff at time t = 0 and makes
contact with the safety net at time t = to.
1. Ignoring air resistance, plot the variation of kinetic energy and potential
energy with time on Fig. 5.4 and Fig. 5.5 respectively. Label them P. [4]

Kinetic Energy

Fig. 5.4

to time

Potential Energy

Fig. 5.5

time
to

2. If air resistance is not ignored, plot the variations of kinetic energy and
potential energy onto Fig. 5.4 and Fig. 5.5 respectively. Label them Q.
[4]

9646 / YJC / 2013 / Preliminary Examinations / Paper 3


12
(iii) Assume the motorcycle pivots about the front wheel during the collision and
the stunt rider is projected over the cliff directly downwards. Discuss the
considerations for the features of the safety net that should be installed to
break his fall. [2]

..................................................................................................................................

..................................................................................................................................

..................................................................................................................................

9646 / YJC / 2013 / Preliminary Examinations / Paper 3


13

6 Bats emit sound waves of high frequency and receive the reflected waves
(echoes) to locate objects ahead. This process is called echolocation, which is
illustrated in Fig. 6.1.

Fig. 6.1

(a) Sound waves are longitudinal in nature.


Describe, in terms of molecular movement, what is meant by a longitudinal wave.
[2]

..................................................................................................................................

..................................................................................................................................

..................................................................................................................................

(b) Sound waves emitted by the bat travel at 340 m s1. Their typical frequency
range is 20 kHz to 80 kHz.
Calculate the range of wavelengths for this frequency range.

range of wavelengths = ................................................. [2]

9646 / YJC / 2013 / Preliminary Examinations / Paper 3


14
(c) Bats emit two waveforms, wave B and wave P, which superpose to form wave E
as shown in Fig. 6.2.
Wave B (shown in Fig. 6.2) gives information about the surrounding
background.
Wave P (not shown in Fig. 6.2) enables the bat to detect insects (prey).

Fig. 6.2

(i) Use the principle of superposition to determine the displacement of wave P


at times corresponding to points L, M and N on the time axis.
Write the displacement values in the spaces provided. [2]

displacement of wave P at L = ....................... units

displacement of wave P at M = ....................... units

displacement of wave P at N = ....................... units

(ii) Hence draw the waveform for wave P on Fig. 6.2. [2]

9646 / YJC / 2013 / Preliminary Examinations / Paper 3


15
(iii) For wave E of frequency of 20 kHz, determine the duration LN. [2]

Duration LN = . s

(d) A phenomenon known as the Doppler effect can be used to determine the speed
of a wave via changes in its frequency.
The change in frequency, f, shown by wave P when it is reflected by an insect
travelling with velocity v is given approximately by the formula

where c represents the speed of sound waves emitted by the bat.

(i) Wave P has a frequency of 50.80 kHz. Given that its apparent frequency
after reflection is 50.35 kHz, calculate the speed of the insect.

insects speed = ........................................ m s1 [2]

(ii) Suggest the direction of the velocity of the insect relative to that of the bat.
[1]

..................................................................................................................................

..................................................................................................................................

(iii) The bat best discriminates small insect prey when the wavelength of the
reflected wave P is similar in size to the insect.
State the wave phenomenon that is being demonstrated in this situation. [1]

..................................................................................................................................

(e) The bats waves are strongly attenuated in air. Fig. 6.3 is a graph of intensity I
against range x in air for the high frequency waves. The attenuation coefficient is
.

9646 / YJC / 2013 / Preliminary Examinations / Paper 3


16

Fig. 6.3
(i) Suggest
1. what is meant by attenuation here, and

.[1]

2. a reason why it occurs.

.[1]

(ii) Use the graph and the differential equation to determine a value
2
for the attenuation coefficient, when I = 8.4 W m .

= ......................... [3]
(iii) State the units of .

Units of = ......................... [1]

9646 / YJC / 2013 / Preliminary Examinations / Paper 3


17
7(a) In an experiment, a proton travelling with a horizontal speed u of 2.19 107 m s1
enters a square region containing a magnetic field of uniform flux density 0.40 T
and directed into the plane of the paper as shown in Fig. 7.1.

20.0 cm

x x x x
10.0 cm
x x x x
proton
x x x x 20.0 cm
u = 2.19 107 m s1
x x x x
10.0 cm
x x x x

magnetic field
Fig. 7.1

(i) Calculate the radius of the path of the proton in the magnetic field. [2]

Radius = m

(ii) Hence or otherwise, sketch the path of the proton within the magnetic field.
Label the path P. [1]

9646 / YJC / 2013 / Preliminary Examinations / Paper 3


18
(b) An electron of the same initial speed as the proton enters the same magnetic
field as shown in Fig. 7.2.
20.0 cm

x x x x
10.0 cm
x x x x
electron
x x x x 20.0 cm
7 1
u = 2.19 10 m s
x x x x
10.0 cm
x x x x

magnetic field
Fig. 7.2

The semi-circle path of the electron is shown in Fig. 7.2 above.

(i) Describe two differences between the paths of electron and proton above. [2]

..................................................................................................................................

..................................................................................................................................

..................................................................................................................................

(ii) Explain your answers to (b)(i). [4]

..................................................................................................................................

..................................................................................................................................

..................................................................................................................................

..................................................................................................................................

..................................................................................................................................

..................................................................................................................................

(c) (i) State Faradays Law of electromagnetic induction. [1]

..................................................................................................................................

..................................................................................................................................

..................................................................................................................................

9646 / YJC / 2013 / Preliminary Examinations / Paper 3


19
(ii) State Lenzs Law. [1]

..................................................................................................................................

..................................................................................................................................

..................................................................................................................................

(d) A coil is placed in a region of magnetic field which is directed into the plane of the
paper as shown in Fig. 7.3. The magnitude of the flux density of the magnetic
field is decreasing with respect to time.

x x x x magnetic field

x x x x
coil
x x x x

x x x x

x x x x

Fig. 7.3

(i) State the direction of induced current within the coil (if any). [1]

...

(ii) Explain your answer to (d) (i). [3]

..................................................................................................................................

..................................................................................................................................

..................................................................................................................................

...

..................................................................................................................................

9646 / YJC / 2013 / Preliminary Examinations / Paper 3


20

(e) A magnet of length, x = 0.050 m and magnetic flux density, B of 0.0100 T is


dropped vertically from rest with the North Pole facing downwards. It is dropped
from a reference line which is at a vertical height, h of 0.400 m above the top of a
long, suspended solenoid of length, L = 0.300 m and of uniform cross-sectional
area as shown in Fig. 7.4. (Ignore the effect of air resistance)

S
x = 0.050 m magnet

N reference line

h = 0.400 m

long solenoid of length,


L = 0.300 m

ground

Fig. 7.4

9646 / YJC / 2013 / Preliminary Examinations / Paper 3


21
(i) Given that at time t = 0.0 s, the magnet is at its reference point when
dropped, sketch the induced e.m.f, E against time, t graph on Fig. 7.5 until
the magnet exits the solenoid. Label this graph A. [2]

E/V

t/s

Fig. 7.5

(ii) The solenoid is now lowered such that the vertical height between the
reference point and the top of the solenoid, h, is increased to 0.800 m with
all other variables kept constant. Sketch the new induced e.m.f, E against
time, t graph on Fig. 7.5. Label this graph B. [2]

(iii) Starting from the situation described in part (e), if the magnet is released
from rest from its reference point with the polarity of the magnet being
changed (South Pole facing downwards), with all other variables kept
constant, sketch another induced e.m.f, E against time, t graph on Fig. 7.5.
Label this graph C. [1]

End of Paper

9646 / YJC / 2013 / Preliminary Examinations / Paper 3


Paper 3

1a Inner drum provides the centripetal force in the form of normal contact force as it is
the only predominant force (besides weight) acting on the wet clothes. .

At the holes, there is nothing to provide the centripetal force.

The fabric will undergo circular motion together with the spinning drum, poking
through a little at the holes. Water will exit from the inner drum tangentially through
the holes.

OR Water did not stay on clothes due to absence of normal force at the holes, thus
the water leaves through the holes

1b Consider at the bottom of the spin (result in largest N)

= 1000*(2)/60 = 104.72 rad s1

F = mr2, thus N mg = (1.5)(0.225)(104.72)2 = 3701.1


N = 3701.1 + (1.5*9.81) = 3715.8 N = 3720 N (3 s.f)

1c They allow the spinning drum to vibrate a little within the rest of the framework

rather than pass on the vibrations through the framework to the surroundings.

OR Springs cause damping/ Absorb/reduce impact force

which reduces the amplitude of vibration/ prevent from vibrating too much/ smooth
spinning

2a Gravitational PE of a mass at a point is the work done on a point mass in


bringing it from infinity to that point.

2bi By energy considerations,

final energy of the body Ef = 0

initial energy of body Ei = KE + GPE

Since ,
2bii 12x104ms11
=1.1

2biii With half th he escape speed, the e rocket will not reach
h infinity but can onlyy rise to
R/3) above the earths surface be
certain finitte height (R efore cominng to a stop
p. It will
then return to Earth

2biv ed as the esscape veloccity = 1.12 x 104 m s-1


Same spee

2bv

Enerrgy , E

r
U

Fig. 2.1

2ci (The averagge kinetic energy


e of the
e hydrogen gas molecules is direcctly proportional to
the thermoddynamic tem mperature.)) Since the mean molecular speedd of hydroge en is
greater than the escappe speed, thhe hydrogen n gas must have escap
ped from Ea arth.

2cii The escape


e velocity off the moon is very low..

3ai Increase in internal energy of a syystem is the eat supplied to the gas and the
e sum of he
work done on the gas.
w [2]

3aii Yes, energyy is conservved as the heat


Y h supplie
ed to the ga
as is converrted into inccrease in
internal ene
ergy and/or work by exxpanding.

3b (pV/T)L =(pV/T)M

TM = 660
0K

3c U T
(U/T)L = (U/T)M

UM = (1500/110)x 660

=9000J

U = 9000 - 1500 = 7500J

3di work done = area under the graph

path 1 : W = (2x2)x106x10-3= 4000J

path 2: W = (1x2) x106x10-3= 2000J

ratio = 2

3dii U = Q+W

path 1: 7500=Q+(-4000)

Q= 11500J

path 2: 7500=Q+(-2000)

Q= 9500J

ratio = 1.2

3diii path 1 is more profitable as the work done by gas is twice that of path 2 whereas the
heat supplied is only 1.2 times more than path 2.

4a (a) waves pass through the gaps / slits in the grating and spread into the space
beyond/geometric shadow

4bi 1. each wavelength travels the same path difference or arrive in phase

hence produce a maximum

2. to obtain a maximum, the path difference must be or phase difference = 360/ 2


rad

of red and blue are different

hence maxima at different angles / positions

4bii n = d sin

N = sin 61 / (2 625 109) = 7.0 105

4biii n = 2 625 is a constant (1250)

n = 1 = 1250 nm (outside visible)


n = 3 = 417 nm (in visible)

n = 4 = 312.5 nm (outside visible)

Correct wavelength values

Thus, = 417 nm (correct choice i.e. within visible region)

5ai Horizontally, 75 = (27.8cos)t

Vertically, -10 = (27.8sin)t + 0.5(-9.81)t2

Solving, -10 = (27.8sin)(75/27.8cos) + 0.5(-9.81)(75/27.8cos)2

-10 = 75tan -(4.905)(2.698)2(1/cos2)

-10 = 75tan - (4.905)(7.278)(1+tan2)

35.70 tan2 - 75 tan + 25.70 = 0

tan = 1.670 or 0.4312


= 59.1 or 23.4

5aii Because the ramp will lower the vertical (upwards) impact force needed to reduce the
vertical component of velocity to zero, making it safer.

5b 0.45Nc Vertically, F = 0, thus 0.45 Nc = mg Nc = mg/0.45

Horizontally, F = mv2/r, thus Nc = mv2/r

Solving, mg/0.45= mv2/r


Nc
v = 10 m s1

mg
5ci No. The principle only applies to a closed system i.e. a system that has no net
external force acting on it. In this case, the barrier wall exerts an external force onto
the motorcycle.

Or

No. The impact occurring between barrier and the motorcycle

Rider flies off at the same speed


5cii 1. and 2.
Kinetic Energy

P
Fig. 1.4
Q

P:reversegraphofGPEsincetotal
conversionofGPEtoKE.Shouldbea
curvebecauseGPEvarieslike
displacementtimegraph.Timeto
endsameasGPE.

Q:NotadirectreverseofGPE
becausethereisafurtherreduction
inVx.ReductioninKEcomparedtoP
cannotbemorethantheinitialKE
(whichisduetothevx)
to to time

Potential Energy

Fig. 1.5 Q

P
P:ShouldbeacurvebecauseGPE
varieslikedisplacementtimegraph.
TimetoendsameasKE.

Q:Thereshouldbeastraightportion
representingthatterminalvelocity
hasbeenreached.Shouldalwaysbe
abovecurvePbecausedisplacement
isalwayslesserthaninP. time
to to

5ciii The safety net should allow a relatively long break-fall time to prevent
large impact force acting on him throughout the contact.

The safety net should be able to extend safely rather than break at the
Any2points limits of its extension before the rider totally comes to rest.

Have springs to help absorb the high KE by conversion to EPE


The safety net should be installed high enough to allow enough
depression to avoid insufficient space of depression and allow impact
with the ground even before the riders fall is stopped.
6a Direction of oscillation or displacement (of molecules) is parallel to
the direction of travel or direction in which energy is transferred
Do not accept molecules move left and right

6b 340 = 20 103
= 17 mm
17 mm to 4.25 mm

6ci Displacement at L is 0.8 (units)


Displacement at M is 0.0 (units) Allow 0.1
Displacement at N is 1.0 (units)

6cii Correct curve through points


1.5 complete waves drawn

6ciii T = (20 103)1 = 5.0 105 s


Thus tLN = 0.5 T = 2.5 105 s

6di Value of f = 0.45 kHz and f = 50.80 KHz


0.45/50.80 = 2v/340
Insects speed, v = 1.51 m s1
6dii The velocity of the insect is in the same direction as bats i.e. it is in front and moving
away from the bat
6diii Diffraction

6ei1. Intensity decreases as wave travels further through the air


6ei2. Medium absorbs some of the wave energy

6eii Correct calculation of the gradient at I = 8.4 W m2 on graph


Substitution in differential equation
Final answer = 0.64

Alternative:

ln I = x + c

Since I = 16 when x =0, thus c = ln (16) = 2.77

Thus, ln (8.4) = (1) + 2.77 = 0.64 m1

6eiii Units of : m1
7ai FB = FC

Bqv = mv2/r

r = mv/Bq = (1.67 10-27)(2.19 107) /(0.40)(1.60 10-19)

= 0.571 m

7aii Correct shape and direction (must exit at correct side)


20.0 cm

x x x x
10.0 cm

proton
x x x x 20.0 cm
7 1
u = 2.19 10 m s
10.0 cm
x x x x

magnetic field

7bi Upon entry of the region of magnetic field, the direction of deflection of
electron is in the downwards direction whereas the direction of deflection of
proton is in the upwards direction.

The electron is deflected to a larger extent as compared to the proton or the


radius of circular motion of the electron is smaller compared to the proton.

7bii Since proton (positive charged) and electron (negatively charged) are
oppositely charged, the direction of magnetic force exerted on proton is
upwards and on electron is downwards. Hence, direction of magnetic force is
also in the opposite direction

From Fig.3, mass of the proton is about 1830 times of the mass of electron

From (b)(i), r m since (B,q and v) are constant for both electron and proton.
As a result, radius of circular motion for proton is also about 1830 times that of
the electron (radius of electron = 3.12 10-4 m < 57.1 cm) which explains the
smaller extent of deflection for proton.

7ci The magnitude of the induced e.m.f in a circuit is directly proportional to the
rate of change in magnetic flux linkage through the circuit or rate of cutting of
magnetic flux.

7cii The direction of the induced e.m.f. is such that the current it causes to flow
opposes the change in magnetic flux which is producing it.
7di Clockwise direction

7dii Coil experiences a decreasing inwards magnetic flux passing through it.

By Lenzs Law, an inwards magnetic flux will be induced by the coil.

By Right Hand Grip Rule, the induced e.m.f and current will be in the clockwise
direction.

7ei Larger e.m.f


induced when
magnet is exiting
the coil as
compared to
entering the coil.
E.m.f.s are of
opposite sign

Area under graph


must be
approximately the
same because
change in
magnetic flux
linkage is the
same in both
cases.

7eii

Both peaks shift to the


right of graph A and
magnitude of induced
e.m.f greater than graph
A.

Larger e.m.f induced


when magnet is exiting
the coil as compared to
entering the coil and
area under graph must
be approximately the
same because change in
magnetic flux linkage is
the same in both cases.
7eiii

Reflection of graph A about the time axis.

Candidates Name . CTG .

YISHUN JUNIOR COLLEGE


JC 2 Preliminary Examinations 2013

PHYSICS 8866/1
th
HIGHER 1 Thursday 29 August 2013
2.00 pm 3.00 pm
Paper 1 Multiple Choice 1 hour
Additional Materials: Multiple Choice Answer Sheet
YISHUN JUNIOR COLLEGE YISHUN JUNIOR COLLEGE YISHUN JUNIOR COLLEGE YISHUN JUNIOR COLLEGE YISHUN JUNIOR COLLEGE
YISHUN JUNIOR COLLEGE YISHUN JUNIOR COLLEGE YISHUN JUNIOR COLLEGE YISHUN JUNIOR COLLEGE YISHUN JUNIOR COLLEGE
YISHUN JUNIOR COLLEGE YISHUN JUNIOR COLLEGE YISHUN JUNIOR COLLEGE YISHUN JUNIOR COLLEGE YISHUN JUNIOR COLLEGE
YISHUN JUNIOR COLLEGE YISHUN JUNIOR COLLEGE YISHUN JUNIOR COLLEGE YISHUN JUNIOR COLLEGE YISHUN JUNIOR COLLEGE
YISHUN JUNIOR COLLEGE YISHUN JUNIOR COLLEGE YISHUN JUNIOR COLLEGE YISHUN JUNIOR COLLEGE YISHUN JUNIOR COLLEGE
YISHUN JUNIOR COLLEGE YISHUN JUNIOR COLLEGE YISHUN JUNIOR COLLEGE YISHUN JUNIOR COLLEGE YISHUN JUNIOR COLLEGE
YISHUN JUNIOR COLLEGE YISHUN JUNIOR COLLEGE YISHUN JUNIOR COLLEGE YISHUN JUNIOR COLLEGE YISHUN JUNIOR COLLEGE
YISHUN JUNIOR COLLEGE YISHUN JUNIOR COLLEGE YISHUN JUNIOR COLLEGE YISHUN JUNIOR COLLEGE YISHUN JUNIOR COLLEGE
YISHUN JUNIOR COLLEGE YISHUN JUNIOR COLLEGE YISHUN JUNIOR COLLEGE YISHUN JUNIOR COLLEGE YISHUN JUNIOR COLLEGE
YISHUN JUNIOR COLLEGE YISHUN JUNIOR COLLEGE YISHUN JUNIOR COLLEGE YISHUN JUNIOR COLLEGE YISHUN JUNIOR COLLEGE
YISHUN JUNIOR COLLEGE YISHUN JUNIOR COLLEGE YISHUN JUNIOR COLLEGE YISHUN JUNIOR COLLEGE YISHUN JUNIOR COLLEGE

READ THESE INSTRUCTIONS FIRST

Do not open this booklet until you are told to do so.

Write your name and CTG in the spaces provided on this cover page and on the Answer Sheet.

Shade your NRIC number in the space provided on the Answer Sheet.

There are thirty questions in this paper. Answer all questions. For each question there are four
possible answers A, B, C and D.
Choose the one you consider correct and record your choice in soft pencil on the separate
Answer Sheet.

Read the instructions on the Answer Sheet very carefully.

Each correct answer will score one mark. A mark will not be deducted for a wrong answer.
Any rough working should be done in this booklet.

This question paper consists of 14 printed pages.


2

Data

speed of light in free space, c = 3.00 108 m s1

elementary charge, e = 1.60 1019 C

the Planck constant, h = 6.63 1034 J s

unified atomic mass constant, u = 1.66 1027 kg

rest mass of electron, me = 9.11 1031 kg

rest mass of proton, mp = 1.67 1027 kg

acceleration of free fall, g = 9.81 m s2

Formulae
1 2
uniformly accelerated motion, s = ut + at
2
v2 = u2 + 2as

work done on/by a gas, W = pV

hydrostatic pressure, p = gh

resistors in series, R = R1 + R2+.

resistors in parallel, 1 1 1
= + + ........
R R1 R 2

8866 / YJC / 2013 / Preliminary Examinations / Paper 1


3

1 The period T varies with distance x according to the equation


1
T2 = p + q ( )
x
where p and q are constants. Which of the following gives the base units of p and q?

[p] [q]

A s2 s2 m1
B s2 m s2
C m s2 m
D ms ms

2 A student recorded some readings together with their uncertainties to find the density of a
solid. Which of the following measurements contributes the greatest percentage

m b

=

a
c
uncertainty to the density? The formula to calculate density of solid, .

A a = (7.89 0.05) mm
B b = (3.24 0.05) mm

C c = (4.6 0.2) cm
D m = (4.8 0.5) x103 kg

3 A vertical force F is applied to a body of mass m on a smooth plane inclined at an angle


to the horizontal, as shown.

The magnitude of the resultant force acting on the body is

A F cos mg sin
B F cos + mg sin

C F sin + mg sin
D F sin mg sin

8866 / YJC / 2013 / Preliminary Examinations / Paper 1


4

4 Two cars, initially next to each other and at rest, accelerate in the same direction at
different uniform rates. After 5.0 s, they are 16 m apart.

If they continue to accelerate at the same rate, how far apart will they be 10 s after they
started accelerating?

A 16 m B 32 m C 48 m D 64 m

5 A particle was projected with a velocity v at an inclination of to the horizontal and landed
3
at the same level. The ratio of maximum height to range was . Neglecting the effect
4
of air resistance, what is the value of the angle ?

A 30o B 45o C 60o D 75o

6 Spring A and Spring B with spring constants kA = 8.0 N cm-1 and kB = 12 N cm-1
respectively are connected in series and a mass of 2.0 kg is attached to the end of the
springs as shown.

Spring A

Spring B

2.0 kg

Given that the system above is in equilibrium and the elastic limit of both springs has not
been exceeded, what is the total extension of the system?

A 0.10 cm
B 0.42 cm

C 0.98 cm
D 4.09 cm

7 A trolley of mass 0.50 kg moves with a certain acceleration down a runway which is
inclined to the horizontal at 15. If the angle of inclination is increased to 20, the
acceleration of the trolley would be doubled. Find the average frictional force, assumed to
be the same in both cases, acting on the trolley.

A 0.55 N B 0.82 N

C 0.86 N D 1.0 N

8866 / YJC / 2013 / Preliminary Examinations / Paper 1


5

8 The resultant of two forces F1 and F2 acting at a point can have a minimum value of 1.0 N
and a maximum value of 7.0 N. When the two forces act at right angles to each other, the
magnitude of their resultant is

A 3.0 N B 5.0 N

C 6.0 N D 8.0 N

9 In the diagram below, a body S of weight W hangs vertically by a thread tied at Q to the
string PQR. Given that the system is in equilibrium, calculate the magnitude of the tension
of the string PQ.

P Q 30

A W sin 60
B W cos 60
C W tan 30
D W tan 60

10 Two vertical cords, as shown, support a heavy uniform beam of length l, with tensions T1
and T2.

T1 T2

l l

T1
What is the ratio ?
T2

A 0.33
B 0.50
C 2.0
D 3.0

8866 / YJC / 2013 / Preliminary Examinations / Paper 1


6

11 The graph shows the relationship between load F and extension x for a certain spring

F/N
6.0

4.0

2.0

0 x/m
0 0.010 0.020 0.030 0.040 0.050

A load of 3.0 N is placed on the spring.


What additional elastic potential energy will be stored in the spring if it is then extended a
further 0.010 m?

A 0.0050 J
B 0.030 J
C 0.035 J
D 0.080 J

12 A stone of mass 0.60 kg is projected horizontally at a speed of 4.0 m s1 from the top of a
wall, 5.0 m above the surrounding ground. Just before it hits the ground, its speed is
7.0 m s1. How much energy is lost as the stone falls through the air?

A 9.9 J
B 15 J
C 20 J
D 29 J

13 In 1.0 minute, a ski elevator lifts four skiers vertically at a constant speed to a height of
150 m. The average mass of each skier is 65 kg. What is the average power provided by
the motor in pulling up the elevator?

A 1.59 kW
B 6.38 kW
C 38.3 kW
D 95.7 kW

8866 / YJC / 2013 / Preliminary Examinations / Paper 1


7

14 A body of mass 50 kg is initially at rest at the base of the slope which is inclined at an
angle 30 to the horizontal. A force is applied to the body and it moves up the slope with
an acceleration of 2.0 m s2. The frictional force between the slope and the body is
constant at 100 N. What is the work done on the body by the applied force after 6.0 s?
 
  A 3600 J
B 7200 J
C 12.4 kJ
D 16.0 kJ

15 A small light bulb, with a 10 W rating, is operating at normal brightness in a large dark
room. Taking the bulb to be a point source, the intensity of light at a distance of 5.0 m
from the bulb is

A 0.032 W m2 B 0.13 W m2 C 0.16 W m2 D 0.32 W m2

16 A transverse progressive wave travels along a rope towards the right hand side. The
graph shows the variation of displacement with distance along the rope, at a certain time.

Displacement
right

Q
P Distance
0

What are the movements of point P and Q at the instant shown?

Movement of P Movement of Q

A downwards upwards

B downwards downwards
C upwards downwards
D stationary upwards

17 A sound wave of frequency 1100 Hz is travelling in air at the speed of 330 m s1. What is
the phase difference between two points on the wave 1.0 m apart in the direction of the
wave?

A 0.60 rad
B 0.67 rad
C 0.80 rad
D 1.3 rad

8866 / YJC / 2013 / Preliminary Examinations / Paper 1


8

18 A sstudent setss up the appparatus ass shown in the figure below


b to de
emonstrate a two-slit
inte
erference paattern on th
he screen.
Top
T view

L
Light Single Double
ource slit
so slits Screen
n

Whhen yellow light of wa avelength 600


6 nm is used, fringges of separation 3.00 mm are
obsserved. What will be th
he fringe sep
paration wh
hen blue ligh
ht of 400 nm
m is used?

A 1.5 mm
m
B 2.0 mm
m
C 3.0 mm
m
D 4.5 mm
m

19 An air column n in a glass tube is open at one end


e and clo osed at the other by a movable
piston. A 512 Hz tuning fo ork is held at the open
n end. Reso onance is he
eard when the
t piston
is 8
83 cm from m the open end.
e hdrawn succh that the air column becomes
The piston is with
longer and the e next resonance is ob btained. Whhat is the neew distance
e between the
t piston
andd the open n end? Takke the spee ed of sound d in air to be 340 m s1 and ig gnore end
corrrections in your calcula
ations.




 pistton

 tuning
 A 116 cm
m fork
B 133 cm
m
C 150 cm
m
D 166 cm
m

20 Fivve identical resistors of 10 are connected a


as shown be
elow.

10
A B

10 10 10

10
D C

Wh
hat is the resistance be
etween the points
p AB?

A 10 B 6.3 C 5.0 D 2.0


8866 / YJC / 2013 / Preliminary
P Exa
aminations / Pap
per 1
9

21 Consider the cuboid


c cond
ducting matterial shown
n below.

X
2a
Y
a
4a

Whhen a voltage is applied between face X and


a its opposing face,, the curren
nt flowing
thro
ough the material
m he same voltage is applied betw
is IX. When th ween face Y and its
oppposing face
e, the currennt is IY.
IX
Wh
hat is the ratio ?
IY

A 4.0 B 2.0 C 1.0 D 0.25

22 The ows two bulbs P and Q marked 4.0


e circuit sho 4 V, 1.0 W and 10 V, 0.50 W resspectively
con
nnected in parallel
p to a battery of e.m.f
e 12 V.

4.0 V P Q 10 V
12 V 1.0 W 0.50 W

R1 R2

Asssuming thatt bulb P and


d Q are ope
erating norm
mally, which
h of the follo
owing statem
ments is
true
e?
A The po
ower dissipa
ated in R2 iss greater tha
an that in R1.
B The cu
urrent, I, in the
t main cirrcuit is 1.5 A.
A
C The po
otential diffe
erence across R 2 is gre
eater than that in R1.
D The cu
urrent follow
wing through
h R1 is greatter than tha
at in R2.

8866 / YJC / 2013 / Preliminary


P Exa
aminations / Pap
per 1
10

23 1 resisto
A 10 or and a thermistor arre connecte
ed in series to a batterry of e.m.f 5.0
5 V and
gligible internal resistance.
neg

10

5.0 V

The ows the current-voltage


e graph sho e characteriistics of the resistor an
nd of the the
ermistor.

Wh
hat is the cu
urrent in the
e circuit?

A 0.10 A B 0.20 A C 0.30 A D 0.40 A

24 In the
t circuit shown below w, all the fou
ur resistors have the sa
ame resista
ance. The re
esistance
of the
t voltmete er is infinite.

+16 V

0V

The
e potential difference
d b
between the
e input term
minals is 16 V.
V

Wh
hat is the reading on th
he voltmeterr?

A zero B 4.0 V C 6.0 V D 8.0 V

8866 / YJC / 2013 / Preliminary


P Exa
aminations / Pap
per 1
11

25 A straight, horizontal, current-carrying wire lies at right angle to a horizontal magnetic field
as shown below, on the left. The field exerts a vertical force of 16 mN on the wire.
Subsequently, the wire is rotated, in its horizontal plane through 60 as shown below, on
the right. The flux density of the magnetic field is halved.

60 magnetic field
magnetic field

wire wire

Top View

What is the magnitude of the magnetic force acting on the wire as shown on the right?

A 4.0 mN
B 7.0 mN
C 9.2 mN
D 16 mN

26 A long straight wire XY lies in the same plane as a square loop of wire PQRS which is
free to move. The sides PS and QR are initially parallel to XY. The wire and loop carry
steady curents as shown in the diagram below.

X
P Q
P

S R
Y

What is the effect on the loop PQRS?

A It will rotate about an axis parallel to XY.


B It will move away from the long wire.
C It will move towards the long wire.
D It will not be affected.

8866 / YJC / 2013 / Preliminary Examinations / Paper 1


12

27 Ahhorseshoe magnet
m restts on a top-pan balancce with a wirre situated between
b the
e poles of
the
e magnet.

Witth no curren ding on the balance is 142.0 g. With a curren


nt in the wirre, the read nt of 2.0 A
in tthe wire in the
t direction n XY, the reeading on thhe balance changes to 144.6 g. What W is the
rea
ading on the e balance, when
w there is a currentt of 3.0 A in the wire in the directio
on YX?

A 138.1 g
B 140.7 g
C 145.9 g
D 148.5 g

28 Whhen a clean n metal in a vacuum is irradiated ochromatic ultraviolet radiation,


d with mono
ele
ectrons are emitted. Which
W w double if the intensity of the ultraviolet
of the following will
rad
diation is do
oubled?

A The ma
aximum spe
eed of the electrons.
e
B The ma
aximum kin
netic energyy of the elecctrons.
C The rate of emission of the electrons.
D The wo
ork function
n of the metal surface.

29 A cclean plate, made of metal


m with a work functio e is illuminated with ultraviolet
on of 2.36 eV,
rad
diation of wavelength h 370 nm. What is the maxiimum energy of the e emitted
phootoelectronss.

A 1.00 eV
V
B 3.36 eV
V
C 5.38 eV
V
D 5.72 eV
V

8866 / YJC / 2013 / Preliminary


P Exa
aminations / Pap
per 1
13

30 When a parallel beam of white light passes through a metal vapour, dark lines appear in
the spectrum of the emergent light. This is principally because energy is absorbed and

A is not re-radiated at all.


B is re-radiated as infra-red.
C is re-radiated gradually over a long period of time.
D is re-radiated uniformly in all directions.

End of Paper

8866 / YJC / 2013 / Preliminary Examinations / Paper 1


14

BLANK PAGE

8866 / YJC / 2013 / Preliminary Examinations / Paper 1


Candidates Name . CTG .

YISHUN JUNIOR COLLEGE


JC 2 Preliminary Examinations 2013

PHYSICS 8866/2
rd
HIGHER 1 Friday 23 August 2013
2.30 pm 4.30 pm
Paper 2 Structured Questions 2 hours
Candidates answer on the Question Paper.
No Additional Materials are required.

YISHUN JUNIOR COLLEGE YISHUN JUNIOR COLLEGE YISHUN JUNIOR COLLEGE YISHUN JUNIOR COLLEGE YISHUN JUNIOR COLLEGE
YISHUN JUNIOR COLLEGE YISHUN JUNIOR COLLEGE YISHUN JUNIOR COLLEGE YISHUN JUNIOR COLLEGE YISHUN JUNIOR COLLEGE
YISHUN JUNIOR COLLEGE YISHUN JUNIOR COLLEGE YISHUN JUNIOR COLLEGE YISHUN JUNIOR COLLEGE YISHUN JUNIOR COLLEGE
YISHUN JUNIOR COLLEGE YISHUN JUNIOR COLLEGE YISHUN JUNIOR COLLEGE YISHUN JUNIOR COLLEGE YISHUN JUNIOR COLLEGE
YISHUN JUNIOR COLLEGE YISHUN JUNIOR COLLEGE YISHUN JUNIOR COLLEGE YISHUN JUNIOR COLLEGE YISHUN JUNIOR COLLEGE
YISHUN JUNIOR COLLEGE YISHUN JUNIOR COLLEGE YISHUN JUNIOR COLLEGE YISHUN JUNIOR COLLEGE YISHUN JUNIOR COLLEGE
YISHUN JUNIOR COLLEGE YISHUN JUNIOR COLLEGE YISHUN JUNIOR COLLEGE YISHUN JUNIOR COLLEGE YISHUN JUNIOR COLLEGE
YISHUN JUNIOR COLLEGE YISHUN JUNIOR COLLEGE YISHUN JUNIOR COLLEGE YISHUN JUNIOR COLLEGE YISHUN JUNIOR COLLEGE
YISHUN JUNIOR COLLEGE YISHUN JUNIOR COLLEGE YISHUN JUNIOR COLLEGE YISHUN JUNIOR COLLEGE YISHUN JUNIOR COLLEGE
YISHUN JUNIOR COLLEGE YISHUN JUNIOR COLLEGE YISHUN JUNIOR COLLEGE YISHUN JUNIOR COLLEGE YISHUN JUNIOR COLLEGE
YISHUN JUNIOR COLLEGE YISHUN JUNIOR COLLEGE YISHUN JUNIOR COLLEGE YISHUN JUNIOR COLLEGE YISHUN JUNIOR COLLEGE
YISHUN JUNIOR COLLEGE YISHUN JUNIOR COLLEGE YISHUN JUNIOR COLLEGE YISHUN JUNIOR COLLEGE YISHUN JUNIOR COLLEGE
For Examiners Use
YISHUN JUNIOR COLLEGE YISHUN JUNIOR COLLEGE YISHUN JUNIOR COLLEGE YISHUN JUNIOR COLLEGE YISHUN JUNIOR COLLEGE
READCOLLEGE
YISHUN JUNIOR THESEYISHUN
INSTRUCTIONS
JUNIOR COLLEGE FIRST
YISHUN JUNIOR COLLEGE YISHUN JUNIOR COLLEGE YISHUN JUNIOR COLLEGE
YISHUN JUNIOR COLLEGE YISHUN JUNIOR COLLEGE YISHUN JUNIOR COLLEGE YISHUN JUNIOR COLLEGE YISHUN JUNIOR COLLEGE
Write your
YISHUN JUNIOR name
COLLEGE and CTG
YISHUN JUNIORin COLLEGE
the spaces provided
YISHUN JUNIORon this YISHUN JUNIOR COLLEGE
COLLEGE Section A JUNIOR COLLEGE
YISHUN
cover page.
YISHUN JUNIOR COLLEGE YISHUN JUNIOR COLLEGE YISHUN JUNIOR COLLEGE YISHUN JUNIOR COLLEGE YISHUN JUNIOR COLLEGE
Write in
YISHUN JUNIOR dark blue
COLLEGE or black
YISHUN pen
JUNIOR on bothYISHUN
COLLEGE sidesJUNIOR
of the paper. 1 YISHUN JUNIOR /6
COLLEGE YISHUN JUNIOR COLLEGE COLLEGE
You may
YISHUN JUNIOR use aYISHUN
COLLEGE soft pencil forCOLLEGE
JUNIOR any diagrams,
YISHUN graphs or roughYISHUN JUNIOR COLLEGE YISHUN JUNIOR COLLEGE
JUNIOR COLLEGE
working. 2 /8
Do not use staples, paper clips, highlighters, glue or
correction fluid. 3 /9

Section A 4 /5
Answer all questions. 5 /8
Section B 6 /4
Answer any two questions.
Section B
Write your answers in the spaces provided on the question
paper. 7 /20
For numerical answers, all working should be shown clearly.
8 /20
The number of marks is given in brackets [ ] at
the end of each question or part question. 9 /20
Penalty
Total /80

This question paper consists of 21 printed pages.


2
For
Examiners
Data Use

speed of light in free space, c = 3.00 108 m s1

elementary charge, e = 1.60 1019 C

the Planck constant, h = 6.63 1034 J s

unified atomic mass constant, u = 1.66 1027 kg

rest mass of electron, me = 9.11 1031 kg

rest mass of proton, mp = 1.67 1027 kg

acceleration of free fall, g = 9.81 m s2

Formulae
1 2
uniformly accelerated motion, s = ut + at
2
v2 = u2 + 2as

work done on/by a gas, W = pV

hydrostatic pressure, p = gh

resistors in series, R = R1 + R2+.

resistors in parallel, 1 1 1
= ........
R R1 R 2

8866 / YJC / 2013 / Preliminary Examinations / Paper 2


[Turn over
3
For
Examiners
Section A Use

Answer all the questions in this section.

1 A skydiver steps outside an air-craft, falls freely for 2.0 s and then opens his parachute.
About 4.0 s after he opens his parachute, he starts to fall at a constant velocity for a
further 50 s before he reaches the ground safely.

(a) Neglecting air resistance during the first 2.0 s of free fall, determine

(i) the distance of fall before he opens the parachute

Distance of fall = m [1]

(ii) the value of velocity just before he opens the parachute

Velocity = m s1 [1]

(b) Sketch a free body diagram of the sky-diver at the 5th second after he steps out.
Label the forces clearly. [1]

(c) Sketch a graph on Fig. 1.1 to show how the velocity, v, of the skydiver varies with
time, t, from the instance he steps outside the aircraft to the time he reached the
ground safely. [3]

v / m s1

t/s

Fig. 1.1

8866 / YJC / 2013 / Preliminary Examinations / Paper 2


[Turn over
4
For
Examiners
2 (a) State the principle of conservation of momentum. Use

..

..[1]

(b) A particle A of mass 0.10 kg, moving with velocity 2.0 m s1 in the direction shown
in Fig. 2.1 makes a head-on collision with a particle B of mass 0.20 kg that is
originally at rest. After the collision, particle A moves off with velocity 0.50 m s1 in
the opposite direction.

Before After

2.0 m s1 0.50 m s1
V

A B A B

Fig. 2.1

(i) Calculate the velocity of particle B after collision.

Velocity =...................m s1 [2]

(ii) Calculate the loss in kinetic energy of particle A.

Loss in kinetic energy = ...................................J [2]

8866 / YJC / 2013 / Preliminary Examinations / Paper 2


[Turn over
5
For
Examiners
(iii) Calculate the gain in kinetic energy of particle B. Use

Gain in kinetic energy =............................... J [1]

(iv) Deduce, from (ii) and (iii), whether the collision is elastic or inelastic.

....................................................................................................................................

.............................................................................................................................. [2]

3 (a) Two sources of sound waves S1 and S2 are placed a distance of 3.0 m apart at
either end of a narrow pipe. Both sources are emitting waves of wavelength 1.2 m
and of similar amplitude, which travel along the pipe. Draw in the diagram Fig. 3.1
to show how the amplitude of the resultant wave will vary along the line S1S2.

S1 S2

3.0 m

Fig. 3.1
[3]

(b) Water waves of wavelength 2.0 m are produced by two generators S1 and S2 as
shown in Fig. 3.2. Each generator, when operated by itself, produces waves which
have an amplitude A at P, which is 3.0 m from S1 and 4.0 m from S2. The intensity
of the wave at P due to S1 or S2 alone is I.

S1 S2
3.0 m
4.0 m

P
Fig. 3.2

8866 / YJC / 2013 / Preliminary Examinations / Paper 2


[Turn over
6
For
Examiners
(i) Calculate the path difference of the two waves arriving at point P. Use

Path difference =............................m [1]

(ii) Assuming that both generators are operating in phase, what is the phase
difference of the two waves arriving at P?

Phase difference =......................rad [1]

(iii) Calculate the intensity of the wave at point P in terms of I when both
generators are operating

1. in phase.

Intensity = ....................... [2]


2. in antiphase.

Intensity = .......................... [2]


8866 / YJC / 2013 / Preliminary Examinations / Paper 2
[Turn over
7
For
Examiners
Use
4 Fig. 4.1 shows the I/V characteristic of a particular type of semiconductor diode.

Fig. 4.1

(a) With reference to Fig. 4.1,

(i) Explain whether the diode obeys Ohms law.

.[1]

(ii) Calculate the percentage change in the resistance of the diode as the
potential difference across it increases from 0.65 V to 0.70 V.

Percentage change = % [3]

8866 / YJC / 2013 / Preliminary Examinations / Paper 2


[Turn over
8
For
Examiners
(b) On Fig. 4.2, sketch the variation of resistance of the diode, R with the potential Use
difference, V across it.

[1]

R/

0.65 V V/ V Fig. 4.2

5 Two wires X and Y, are carrying current I1, and I2 in the upwards and downwards direction
respectively. The two wires are separated by a distance r as shown in Fig. 5.1. Current I1
o I1
causes a magnetic field of flux density of at Y where o is the permeability of free
2r
7
space = 4 10 H m1.

X Y

I1 I2

Fig. 5.1

(a) Define magnetic flux density of a magnetic field.

.[2]

8866 / YJC / 2013 / Preliminary Examinations / Paper 2


[Turn over
9
For
Examiners
(b) Show that the magnetic force per unit length of wire which I1 causes on wire Y is given by Use
o I1 I 2
the expression: .
2r

[2]

(c) State and explain the direction of the magnetic force exerted on wire Y.

..............................................................................................................................................

[4]

6 Two energy levels in a hydrogen atom are 3.40 eV and 1.51 eV.

(a) Calculate the energy of a photon emitted when an electron transits between these two
levels.

Energy = ..J [2]

(b) Calculate the wavelength of the emitted photon.

Wavelength = ..m [1]

(c) State the region of electromagnetic spectrum this photon corresponds to.

................................................................................................................................................

..[1]
8866 / YJC / 2013 / Preliminary Examinations / Paper 2
[Turn over
10
For
Examiners
Section B Use

Answer two of the questions in this section.

7 (a) (i) State Newtons Second Law.

[2]

(ii) A plump line is hung from the roof of a vehicle as shown in Fig. 7.1. When the
vehicle is travelling at constant velocity to the right, the bob is stationary as seen by
an observer in the vehicle. Draw in Fig. 7.2, the position of the bob when the vehicle
begins to accelerate uniformly to the right. Explain your answer.

Fig. 7.1 Fig. 7.2

[3]

(b) A luggage of weight 200 N has dimensions H = 0.80 m and D = 0.50 m as shown in
Fig. 7.3. It has a rigid handle of length L = 0.30 m and two rollers mounted at the base.

handle

L= 0.30 m

H = 0.80 m rollers
0.80m
Fig. 7.3
D = 0.50 m

8866 / YJC / 2013 / Preliminary Examinations / Paper 2


[Turn over
11
For
Examiners
(i) Draw labelled forces acting on the luggage at rest in Fig. 7.4. Use

Fig. 7.4

[2]

(ii) The owner exerts a horizontal force, T acting to the right on the top of the handle.
One roller just loses contact with the ground, while the luggage stays in static
equilibrium.

1 Mark X on the roller that just loses contact with the ground in Fig. 7.5. [1]

Fig. 7.5

2 Calculate the horizontal force T on the luggage in static equilibrium.

Horizontal force T = ....................N [2]

8866 / YJC / 2013 / Preliminary Examinations / Paper 2


[Turn over
12
For
Examiners
3 Calculate the force exerted on the wheel by the ground. Use

Force on the wheel by ground : Magnitude ....................... N

Direction : ......................................[3]

(c) The owner applies a horizontal force T = 20 N that moves the luggage at a constant
velocity v = 0.60 m s-1 with both wheels in contact with the floor.

(i) Calculate the work done by the man in moving the luggage along a straight path of
distance 100 m.

Work done =.....................................J [2]

(ii) The work done by the horizontal force T does not change the kinetic energy of the
luggage. Explain.

.[1]

(d) The owner continues to walk in a straight path at the speed of 0.60 m s-1 up a slope of
angle of inclination 8.0. Assume all other conditions are unchanged.

(i) Calculate the additional work done to push the luggage up the slope a distance of
100 m.

Additional work done =...............................J [2]


8866 / YJC / 2013 / Preliminary Examinations / Paper 2
[Turn over
13
For
Examiners
(ii) Calculate the potential energy gained in each second by the luggage moving at a Use
velocity of 0.60 m s-1.

Potential energy gained per second =...............................W [2]

8866 / YJC / 2013 / Preliminary Examinations / Paper 2


[Turn over
14
For
Examiners
8 (a) A filament lamp with a resistance of 4.0 and operating at 12 V is turned on for 1.0 hour. Use

(i) Calculate the current in the lamp

Current = A [1]

(ii) Calculate the amount of electrical energy transformed into heat and light energy
by the lamp in 1 hour.

Amount of energy = J [2]

(iii) State and explain microscopically, the effect on the resistance of the filament lamp
if it is operated at 24 V.

.[3]

(b) Fig 8.1 shows a cell connected to an external circuit. The cell has an emf of 1.3 V and
internal resistance of 0.80 .

Fig. 8.1

8866 / YJC / 2013 / Preliminary Examinations / Paper 2


[Turn over
15
For
Examiners
(i) Define the e.m.f of a supply. Use

..

[1]

Assuming that the voltmeter has an infinite resistance, calculate

(ii) the current, I, in the cell.

Current = A [3]

(iii) the voltmeter reading.

Voltmeter reading = .. V [2]

(iv) the percentage of the cells power lost as internal heating within the cell.

Percentage loss = .. % [3]


8866 / YJC / 2013 / Preliminary Examinations / Paper 2
[Turn over
16
For
Examiners
(c) Fig. 8.2 shows a light-dependent resistor (LDR) connected in a potential divider circuit. Use

R1
R1

R2

Fig. 8.2

The resistance of the LDR is R1 and the resistance of the fixed resistor is R2. The supply
has e.m.f E and negligible internal resistance. The current from the supply is I. The
voltmeter has an infinite resistance.

(i) Show that the voltmeter reading V is given by the relation.

R2
V E
R1 R 2

[3]

8866 / YJC / 2013 / Preliminary Examinations / Paper 2


[Turn over
17
For
Examiners
The supply has e.m.f of 5.0 V and the resistor R2 has a resistance of 470 . The light Use
from the filament lamp is guided onto the LDR by the paper tube. The LDR has a
resistance of 120 k when it is in the dark and 150 when in light.

Determine the voltmeter reading when:

(ii) the lamp is not switched on.

Voltmeter reading = .. V [1]

(iii) the lamp is switched on.

Voltmeter reading = . V [1]

8866 / YJC / 2013 / Preliminary Examinations / Paper 2


[Turn over
18
For
Examiners
9 Fig. 9.1 shows the interference pattern produced by an electron Youngs double slit Use
experiment. In this experiment, the electrons were sent into a double-slit apparatus with
an effective slit separation of 200 nm. The detector screen was 79.0 cm from the double
slits. The dots indicate where an electron has struck the detector. (Note: The image below
has been magnified by a factor of 100).

3.60 cm

Fig.9.1

(a) With the aid of Fig. 9.1, calculate the wavelength of the electron.

Wavelength = .m [2]

(b) Calculate the velocity of the electrons passing through the slits.

Velocity = .m s1 [2]

8866 / YJC / 2013 / Preliminary Examinations / Paper 2


[Turn over
19
For
Examiners
Use
(c) List one aspect of the image (Fig. 9.1) that illustrates the particle nature of electrons.

..............................................................................................................................................

[1]

(d) List one aspect of the image (Fig. 9.1) that illustrates the wave nature of electrons.

...............................................................................................................................................

. [1]

(e) (i) Explain what is meant by a photon.

.............................................................................................................................................

.[2]

(ii) Define work function of metal.

..............................................................................................................................................

.[2]

8866 / YJC / 2013 / Preliminary Examinations / Paper 2


[Turn over
20
For
Examiners
Use
(f) Fig. 9.2 shows a photocell placed next to a filament lamp in a dark room, that may be
used to demonstrate the photoelectric effect.

Fig. 9.2

When a red filter is placed in front of the filament lamp, not a single photoelectron is
emitted from the metal plate P and the sensitive ammeter shows no reading. When the
red filter is replaced with a blue filter, photoelectrons are emitted from P and accelerate
towards the positive collector. When the blue filter is used, the ammeter shows a reading
of 9.2 108 A.

(i) Explain why photoelectrons are emitted when the blue filter is used but not when
the red filter is used.

..............................................................................................................................................

.[2]

(ii) The blue filter is placed in front of the filament lamp. Calculate the number of
photoelectrons emitted from P in the time interval of 8.0 s.

Number of photoelectrons = [2]

8866 / YJC / 2013 / Preliminary Examinations / Paper 2


[Turn over
21
For
Examiners
Use
(iii) In a controlled experiment, the intensity of the blue light incident on the photocell
in Fig. 9.2 is doubled. State and explain the effect, if any, on the current measured
by the ammeter.

...................................................................................................................................

...................................................................................................................................

.. [3]

(iv) The polarity of the battery is being reversed. State and explain the effect, if any,
on the current measured by the ammeter.

...................................................................................................................................

...................................................................................................................................

.. [3]

--- End of Paper ---

8866 / YJC / 2013 / Preliminary Examinations / Paper 2


[Turn over
2013 H1 Prelim Exam Paper 1 Suggested Solutions

1) B 2) D 3) C 4) D 5) C 6) D 7) C 8) B 9) D 10) A
11) C 12) C 13) B 14) D 15) A 16) A 17) B 18) B 19) A 20) B
21) D 22) D 23) B 24) A 25) A 26) C 27) A 28) C 29) A 30) D

Paper 2 Suggested Solutions- Section A

1a (i) s = at2 = (9.81) (2)2 = 19.6 m


(ii) v =u + at = (9.81) (2) = 19.6 m s1

b
FD Air resistance

W Weight

(Accept the case where magnitude of FD is less than weight)

v / ms-1

[1] t= 0 to t=2 s, straight line


[1] t= 2 to t =6 s, curve
[1] t=6 to t= 56 s, horizontal line

2 6 56 t/s

(Accept the graph that corresponds to the case where magnitude of FD is less than weight)

2(a) The total momentum of a system is constant , provided no external force acts on it[1]

bi) m(2)=m(-0.5)+2mV[1]
V= 1.25 m s-1[1]

ii) loss in ke = 1/2 0.1(22-0.52) [1]= 0.188J[1]

iii) gain in ke = 1/2 (0.2) 1.252= 0.156J[1]

iv) the loss in ke of A is not equal to gain in ke of B . [1] Hence the ke is not conserved and
collision is inelastic.[1]
page 2

3a)

S1y yS2

3.0m
Fig. 3

The drawing include : 2.5 wavelength [1],Antinode at both ends [1] and same amplitudes for both
waves [1]

b) (i) path difference = 1m


(ii) phase difference = 3.14 or rad
(iii) 1. amplitude = A-A = 0
Intensity = 0
2. amplitude = A+A=2A[1]
Intensity = 4I [1]

4
a) (i) The I-V graph is not a straight line through the origin.[1] Hence the resistance of the
diode is not constant. It therefore does not obey Ohms law.

(ii) At 0.65 V
R= [1]

At 0.70 V

R= [1]

Percentage change = [1]

b)

R/

[1] correct shape and trend

9646/1&2/JC1/YJCMondayTest2
V/ V
page 3

5(i) Magnetic flux density of a magnetic field is defined as the force exerted on a
unit length of a conductor [B1] carrying a unit current placed at right angle to
the field [B1].

(ii) FB = BX I2 L sin [M1]


FB /L = BX I2 sin
o I1
= I2 sin 90 [C1]
2r
o I1 I 2
= [A0]
2r
(iii) Towards the right. [M1]
By Right Hand Grip Rule, the direction of magnetic field due to X at Y is
into the plane of the paper. [B1]
Since the direction of conventional current, I2 is downwards, the direction
of magnetic field at Y is into the plane,
The direction of force exerted on Y is rightwards [B1] by Flemings Left Hand
Rule. [B1]

6(i) Energy of photon = (-1.51) (-3.40) = 1.89 eV [B1]


= 3.02 10-19 J [B1]

(ii) Energy of photon E = hc/


= hc/E = hc/3.02 10-19 = 6.59 10-7 m [A1]

(iii) Visible Light [B1]

9646/1&2/JC1/YJCMondayTest2
page 4

Section B

Q7(a)(i) The rate of change of momentum is directly proportional to the net force and takes place
in the direction of the net force.

Q7(a)(ii)

Fig. 1.1 Fig. 1.2


1 mark for drawing.
The tension has two components , the horizontal component enables the bob to accelerate to the
[1] right. The vertical component balances the weight of the bob. [1]

7bi) R1 and R2 [1], W[1]

R2 R1

W
b)(ii)2 Taking moment about O,
T(1.1)= W(0.25) [1]
T = 45N [1]

f
b(ii)1 X O
[1]
R = W=200N
W
9646/1&2/JC1/YJCMondayTest2
page 5

b)(ii)3 f=T =45N [1]


R=(452+2002)1/2=205N [1]
= tan-1 200/45 = 770 [1] [Diagram to illustrate must be drawn]

(c) (i) work done by man = 20x100[1] =2000J [1]

(ii) work done by man does not increase kinetic energy of luggage as work is done against
friction and is dissipated as heat energy.[1]

(d) (i) additional work done = mgh where h = 100 sin 80=14m
additional work done = 200 x 14 [1] = 2800 J [1]

(ii) potential energy gained in 1 s = mgvsin80= 200x0.60sin80[1]


= 16.7 W[1]

8(a) (i) I = 12/4 = 3.0 A [1]


(ii) Q = It = (3.0) (3600) = 10800 C [1]
Energy = V Q = (12) ( 10800) = 1.3 x 10 5 J [1]

(iii) The resistance of the lamp will be higher than 4.0 because the temperature of the
filament has increased. [1]

As a result,

the vibration of the ions also increases, more collisions between the free electrons
and vibrating ions. [1]
the drift velocity of the electron decreases, manifesting as higher resistance. [1]

(b)(i) The e.m.f of a supply is the amount of energy transferred per unit charge from some forms
into electrical energy when charge is moved round a complete circuit. [1]

(ii) External resistance = R = 0.50 [1]


E = I (R + r)

1.3 = I (0.50 + 0.80) [1]

I = 1.0 A [1]

(iii) E = I (R + r)
1.3 = V + 0.80 [1]

V = 0.50 V [1]

(iv) Power from cell = 1.0 (1.3) = 1.3 W [1]


Power lost due to internal resistance= (1.0)2 (0.80) = 0.80 W [1]

Percentage loss = 0.80/1.3 = 62 % [1]

(c) (i) Total effective resistance = R1 + R2 [1]


E = I (R1 + R2) I = E/ (R1 + R2) [1]
Voltmeter reading = V = I R2 [1]

9646/1&2/JC1/YJCMondayTest2
page 6

(ii)

(iii)

9(a) = ax/D
= (200 10-9)(1.20 10-4)/0.79 [C1]
= 3.04 10-11 m [A1]

(b) = h/mv
3.04 10-11 = h/(9.11 10-31)(v) [C1]
v = 2.39 107 m s-1 [A1]

(c)
Electrons behave like particles when they strike the detector at localized
spot or specific locations [B1].

(d) Electrons behave like waves because they accumulate to produce an


interference pattern [B1].

(e) (i) A photon is a packet of energy. [B1] Its energy, E = hf where h is the
Plancks constant and f is the frequency of photon. [B1]

(ii) Work function of metal is the minimum energy [B1] required to release an electron from
the surface of that metal [B1].

(f) (i) This is because the energy of blue photon is greater than or equal to the
work function of metal plate and hence photoelectrons are emitted. [B1]
On the otherhand, the energy of red photon is lesser than the work function of
metal plate and hence no photoelectrons are being emitted. [B1]

(ii)
Q=It
= 9.2 10-8 (8.0) = 7.36 10-7 C [B1]
Ne = Q
N = Q/e = 7.36 10-7/1.6 10-19 = 4.6 1012 [A1]

(iii) The current will also be doubled. [M1]


This is because as intensity is doubled, the number of photons per unit time reaching
the metal plate is doubled [A1]. As a result, the number of photo-electrons emitted from
metal plate also doubled [A1].

(iv) The current will decrease. [M1]


Some of the emitted photoelectrons will be stopped from reaching the collector [A1] as
they have insufficient kinetic energy [A1] to overcome the attractive force between
metal plate and photoelectrons.

9646/1&2/JC1/YJCMondayTest2
VICTORIA JUNIOR COLLEGE
2013 JC2 PRELIMINARY EXAMINATIONS

PHYSICS 8866/01
Higher 1 27 Sep 2013
FRIDAY
Paper 1 Multiple Choice
8.00 am to 9.00 am
1 Hour
Additional Materials: Multiple Choice Answer Sheet

READ THESE INSTRUCTIONS FIRST

Write in soft pencil.


Do not use staples, paper clips, highlighters, glue or correction fluid.
Write your name, CT group and index number on the Multiple Choice Answer Sheet provided.

There are thirty questions on this paper. Answer all questions. For each question there are four
possible answers A, B, C and D.
Choose the one you consider correct and record your choice in soft pencil on the separate Answer
Sheet.

Read the instructions on the Answer Sheet very carefully.

Each correct answer will score one mark. A mark will not be deducted for a wrong answer.
Any rough working should be done in this booklet.

This document consists of 16 printed pages.

1
Data

speed of light in free space, c = 3.00 108 m s-1

elementary charge, e = 1.60 10-19 C

the Planck constant, h = 6.63 10-34 J s

unified atomic mass constant, u = 1.66 10-27 kg

rest mass of electron, me = 9.11 10-31 kg

rest mass of proton, mp = 1.67 10-27 kg

acceleration of free fall, g = 9.81 m s-2

Formulae

uniformly accelerated motion, s = ut +( ) at2


v2 = u2 + 2as

work done on/by a gas, W = pV

hydrostatic pressure, p = hg

resistors in series, R = R1 + R2 +

resistors in parallel, 1/R = 1/R1 + 1/R2+

2
1. The behavior of many real gases deviates from pV = nRT but can be represented quite
closely over certain ranges of temperature and pressure by an equation of the form


p a V b RT
V n
2


n

in which the values a and b are characteristic of the particular gas. What are the units of
a and b ? (p = gas pressure, V = gas volume, n = number of moles of gas, R = molar gas
constant in J mol-1 K-1 and T = thermodynamic temperature).
a b

A. Pa m-6 mol2 m3 mol-1

B. Pa m6 mol-2 m3 mol-1

C. Pa m6 mol-2 m-3 mol

D. None because they are dimensionless constants

2. A robot navigated by a student moves at an initial velocity of u from point P to point Q and
changes its velocity to v from point Q to point R as shown in the diagram below.

R
Q v

Which of the following vector diagrams correctly represents the change in the velocity v of the
robot?

v
A. B.
v

C. D. v
v

3
3. Four students each made a series of measurements of the acceleration of free fall g. The
table shows the results obtained. Which student obtained a set of results that could be
described as accurate but not precise?

Student results, g / m s-2

A. 10.51 9.65 8.80 10.21

B. 9.81 9.45 9.84 9.88

C. 8.45 8.46 8.50 8.41

D. 9.45 9.21 9.89 9.85

4. The wavelength of red light is found to be 710 nm with an uncertainty of 10 nm.


Calculate the uncertainty for the energy of a red light photon.

A. 1 10 8 J B. 4 10 21 J C. 2 10 25 J D. 8 10 47 J

4
5. A ball is dropped from a certain height from rest. The distances s it moves through with
respect to time are obtained experimentally. Assume air resistance to be negligible.
The following graph is obtained.

/ m1/2

x
x
x
x
x
x
t/s

If air resistance is not negligible and is a constant value, which of the following new
graphs would be obtained?

A. B.
s/m 1/2 s / m1/2

x x
x x
x x
x x
x x
x x
t/s t/s

C. s / m1/2 D.
s / m1/2

x x
x x
x x
x x
x x
x x
t/s t/s

5
6. A ball is kicked at an angle 30o above the horizontal with speed 25.0 m s-1.

-1
25.0 m s

o
30

What is the range R travelled by the ball? Take air resistance to be negligible.

A. 55.2 m
B. 27.6 m
C. 43.4 m
D. 67.0 m

7. A 5.0 kg bowling ball is rolling on a smooth surface towards the north with a speed of
2.0 m s-1. A consistent wind blows from the north-east and exerts a constant force of 1.0 N
on the ball. What is time taken for the balls velocity to be directed towards the west?
A. 7.1 s B. 14.1 s C. 0.71 s D. 0.14 s

8.

2m

3m
weighing
machine

floor of lift
Two rectangular blocks of masses 3m and 2m rest on a weighing machine on the floor of
a lift. When the lift accelerates upwards, what is the relationship between the contact
force F1 between the two blocks and the reading R of the weighing machine?

6
9. A spring of initial length 10.0 cm and spring constant 20 N m-1 is stretched slowly at a
constant speed. Find the work done on the spring when the length of the spring is extended
from 12.5 cm to 18.0 cm.

A. 0.0578 J B. 0.0303 J C. 0.550 J D. 0.116 J

10. A pendulum bob was attached to a string and hung from the roof of a car. When the car
accelerated on a flat road, the string of the bob made an angle 10o from the vertical. Find
the acceleration of the car.

10o

A. 0.49 m s-2 B. 1.73 m s-2 C. 6.49 m s-2 D. 9.67 m s-2

11. In the figure below, the length of the uniform bar is 5.0 m and its weight is 500 N. The
angle between the wire and the bar is 20o.

Wire
Rough
Wall 60o 20o

Bar

Calculate the magnitude of the tension in the wire.


A. 250 N B. 326 N C. 633 N D. 653 N

7
12. An object of mass m slides up an incline at X with a velocity v to stop at point Y.
Assuming a uniform frictional effect, what is the magnitude of the velocity of the mass m
in terms of v when it is half way up the incline XY which is of length L i.e. when it has
1
travelled L up the incline?
2

L
Y
v

A. B.

C. D.

13. A new generation of Honda Civic cars of a hybrid design consists of a fuel engine with a
maximum output of 70 kW and an electric motor with a maximum output of 15 kW.
Driven slowly with only the electric motor running, the car travels at a maximum speed
of 12 m s-1. The total drag D on the car is proportional to the square of its speed v.

When pushed hard, the fuel engine will operate alongside the electric motor. What is the
maximum speed of the car when it is pushed to the output limits of the engines?

A. 28.6 m s-1 B. 25.3 m s-1 C. 21.4 m s-1 D. 68.0 m s-1

8
14. The graph shows the shape at an instant t = 0 of part of a transverse wave travelling along
a string from left to right.

y
A
S

Which of the following correctly gives the displacement-time description for particles
P or S and the phase difference of the motion of S with respect to that of particle P?

Displacement-time relation of P or S Phase difference of S with


respect to P

A. yS = -A sin t

B. yP = A sin t

C. yP = A cos t

D. yS = -A cos t

15. A progressive sinusoidal wave of wavelength is passing transversely along a horizontal


rope. M and N are points on the rope 7/4 apart and the wave is travelling from M to N.
Which of the following correctly describes N at an instant when M is displaced upwards
and is also moving upwards?

displacement of N movement of N

A. upwards downwards

B. upwards upwards

C. downwards upwards

D. upwards stationary

9
16. Two coherent waves having intensities in the ratio of 9:1 interfere. The ratio of the
maximum to minimum intensity is equal to
A. 10:8 B. 9:1 C. 4:1 D. 2:1

17. A long glass tube has a small loudspeaker, connected to a signal generator, placed at one
end. A small microphone that can be moved through the tube is connected to a cathode-
ray oscilloscope (CRO) as shown in figure below.

A set of data is obtained as tabulated below:

frequency/Hz distance of the microphone from the loudspeaker for maximum


amplitude signal/mm
405 82 499 913 1332

Which of the following about the speed of sound and its wavelength can be deduced
from the above set of data?

Wavelength/m Speed of sound/m s-1


A. 0.417 338
B. 0.834 169
C. 1.67 676
D. 0.834 338

10
18. Four pieces of uniform wire of the same material but having different diameters are
connected together as shown below.

W X Y Z A

Which graph best represents the relationship between current and distance between W and
A?

A. Current C. Current

W X Y Z A W X Y Z A
B. D.
Current Current

W X Y Z A W X Y Z A

11
19. The graph below shows the I-V characteristic of a conductor.
I/A

5.0
4.0
3.0
2.0
1.0
V/V
0 1.0 2.0 3.0
Which of the following graphs best represents the variation of power P dissipated in
the same conductor with ?
A. P C. P

I2 I2
B. D.
P P

I2 I2

20. The resistors X, Y and Z in the circuit are of identical resistance.

The battery, of negligible internal resistance, supplies a total power of 24 W. What is


the power dissipated by heating in resistor Z?

A. 4 W B. 6 W C. 8 W D. 12 W
12
21.

1 5

2 6

3 7

4 8

Each resistor in the circuit above is of resistance 1.0 . What is the effective
resistance between pins 4 and 7, assuming all other pins are unused?

A. 1.08 B. 1.83 C. 2.00 D. 3.00

22.

R1

6.0 V R2

4.0 V
R3
3.0 V

The circuit above is used to tap off 4.0 V and 3.0 V from a 6.0 V source. Possible
values for R1, R2 and R3 are respectively

A. 1 k , 2 k and 3 k
B. 2 k , 1 k and 3 k
C. 2 k, 3 k and 1 k
D. 3 k, 1 k and 2 k

13
23. A stationary metal conductor is carrying a current in the direction to the right in the plane
of the paper as shown. It is in a region of uniform magnetic field pointing
perpendicularly into the plane of the page. Which of the following statements is correct?

P
B
Q
A. P has a higher potential than Q.
B. P has a lower potential than Q.
C. P and Q are at the same potential.
D. P is at a positive potential.

24. A conductor suspended by two flexible wires as shown in the figure below has a mass per
unit length of 0.040 kg m-1. What current must exist in the conductor for the tension in
the supporting wires to be zero when the magnetic field is 3.6 T into the page?


conductor B

A. 0.010 A
B. 0.050 A
C. 0.11 A
D. 0.15 A

25. In each diagram, two wire sections are shown, each carrying a constant current I. The
directions of the currents are shown by the arrows. In which diagram will the attractive
force between the wire sections, due to the currents, be zero?
A. B.

C. D.

14
26. Einsteins photoelectric equation is derived on the assumption that

A. electromagnetic radiation is absorbed in quanta of energy proportional to the


frequency of the incident radiation.
B. electrons are associated with wavelengths = h/p where p is momentum.
C. light is emitted only when electrons jump between energy levels.
D. light behaves as a wave.

27. The work function of platinum is twice that of calcium. If the minimum photon energy
required to emit photoelectrons from the surface of platinum is E, then that for the
surface of calcium would be

A. 2E
B. 3E/2
C. E/2
D. impossible to determine from information given.

28.
photocurrent

Potential of
collector electrode
-V0
In a photoelectric effect experiment, energetic radiation is made to fall on a polished
emitter electrode in an evacuated enclosure. A p.d. is applied across the emitter electrode
and a collector electrode. The former is maintained at zero potential. The figure above
shows the variation of photocurrent with the collector potential. No current is observed
when the collector potential is V0 because

A. electrons are no longer being emitted


B. equal numbers of positive and negative charges are emitted
C. electrons which are emitted have insufficient kinetic energy to reach the collector
plate
D. electrons which are emitted have negligible kinetic energy

15
29. An atom in the ground state absorbs 20.00 electronvolts of energy and is ionised by losing
an electron. The ionisation energy is 10.38 eV. What is the speed of the electron after
ionisation?

A. 1.84 106 m s-1


B. 1.91 106 m s-1
C. 2.65 106 m s-1
D. 3.27 106 m s-1

30. Determine an expression for the ratio

Energy of photon of wavelength


Kinetic Energy of electron of de Broglie wavelength

(Take h = Planck constant, c = speed of light in vacuum, m = mass of electron).

mc h 2mc 2h
A. B. C. D.
h mc h mc

16
VICTORIA JUNIOR COLLEGE = 4 10 21 J

SUGGESTED SOLUTIONS TO 2013 Ans: B


H1 P1 PHYSICS PRELIM EXAMS
5. For a ball being dropped in the
a gravitational field of the earth, we have
1. units of p = units of 2
V

n 1 2 g
s gt or s t
units of a = Pa m 6 mol 2 2 2

V The gradient of the s - t graph is


Unit of b = units of
n therefore a measure of the earths
acceleration due to gravity.
= m 3 mol 1
Ans: B
If air resistance exists and is a constant
value, then it will oppose the gravitational
2. Change in velocity =
force acting on the ball when it is falling.
v u v (u )
The gradient of the graph will
Ans: C
consequently be less steep but will still be
a constant value.
3. By calculating the average values,
the results of students A, B and D proved
Ans: B
to be quite accurate. Among them, the
results for student A had the most
deviation. Hence his result is deemed 6. Taking downward as the positive
accurate but not precise. direction, and considering the entire
motion from initial to final points, we have
student Average g / m s-2
A 9.79 u y 25 sin 300 12.5 m s-1
B 9.75
C 8.46 v y 25 sin 300 12.5 m s-1
D 9.60
vy u y
Ans: A a ,
t
Hence time of flight t =
hc vy u y 12.5 (12.5)
4. Energy of photon E 2.55 s
a 9.81
E 0
Range R = v x t 25 cos 30 2.55 55.2 m
E
Ans A

E E

10 (6.63 10 34 )(3.0 108 )
E
710 710 10 9

1
7.

p
pi
45o

Ans: C
pf
Initial momentum = 9. Work done = change in elastic PE
1 2 1 2
= kx2 kx1 , where x is the extension
2 2
From the vector diagram, in m
| | | | W = 0.5 (20) (0.0802 - 0.0252) = 0.0578 J
| |
Ans: A
-1
| | kg m s

By Impulse-Momentum theorem,
10. a 10o
since F is constant. T

time taken = t = 14.1 s.


mg
Ans: B
Consider the pendulum bob.

8. Vertical equilibrium: T cos 10 0 mg....(1)


Horizontal acceleartion:
F1 R
T sin 10 0 ma.....(2)
(2) a
2m a gives tan10 0
5m a (1) g
a 9.81 tan10 1.73 m s-2
0

(2m)g Ans: B

(5m)g 11. Taking moments about point on wall,

clockwise moments = anti-clockwise


Let a = acceleration of system.
R = normal reaction force between lift moments
floor and 5m block.
Apply Newtons 2nd law to the 2m mass: mg 2.5 sin 600 T sin 200 5
( )
T = 633 N
Ans: C
nd
Apply Newtons 2 law to both masses:
( ) ( )
( )( )

2
12. Let the mean frictional force be f. For wave propagates. Its displacement time
the motion X to Y conservation of energy equation is thus given by yP = A cos t.
requires that the loss in kinetic energy =
mechanical work done against friction f +
Ans: C
gain in gravitational potential energy.
15. M and N are essentially apart.
1
Thus, mv2 fL mgh , where L is the The dotted line shows the next wave
2
profile.
slope length.
Sliding up to the mid-point of the slope
XY, the principle of conservation of
energy will now be written as M N
1 2 1 1 1
mv fL mgh mv'2 .
2 2 2 2
Hence,
1 1 1 1 Thus, N would be displaced upwards but
mv'2 mv 2 fL mgh
2 2 2 2 moving downwards.
1 2 1 2 1 2 Ans. A
mv mv mv
2 4 4
16. Intensity of a wave is given by I = kA2.
v
Thus, v' I1 A12 9
2 Hence A1 3 A2
Ans: D I 2 A22 1

13. Given D v 2 and when car is For maximum interference, the resultant
amplitude is A1 + A2 = 4A2 and the
travelling at its maximum speed, the
intensity I max k16A22 .
driving force is equal to D. The power
output, P = Dv. Hence at its maximum For minimum interference, the resultant
amplitude is A1 A2 = 2A2 and the
speed, P v3 . intensity I min k 4A22 .
At motor output of 15 kW, the speed Thus, the ratio of Imax/Imin = 4:1
v1 = 12 m s-1. Hence with both fuel engine Ans. C
and electric motor operating at the limits
of the power output, the maximum speed 17. From the data, the antinode to antinode
of the car, v2 is given by the relation: distances are (499 - 82 = 417 mm),
(913 - 499 = 414 mm) and
70 15 v23 v23 (1332 - 913= 419 mm). The mean
3 3 v2 21.4 m s-1 distance is about 417 mm. Thus, the
15 v1 12
wavelength = 2 x 0.417 = 0.834 m. Thus,
Ans: C the velocity v = f = 405 x 0.834
= 338 m s-1.
3
14. Particles P and S are separated by .
4 Ans. D
Hence the phase difference between them
3 18. Current in all sections is the same since
is . Particle P is at its amplitude they are connected in series.
2
position and is moving downwards as the Ans: A

3
19. For the I-V graph given, the resistance
decreases as the current increases. 22. A
2 R1
Using P = I R, the gradient, representing
the resistance, decreases with increasing 4.0 V
current I. R2
6.0 V
Ans: D
3.0 V
20. Circuit can be rearranged as shown R3
below:
B
Y 2I

I Z
Terminal B is earthed. Then potential
of terminal A is 6.0 V.
I X
The current through all 3 resistors is
the same since the arrangement is a
series connection. Therefore
Let each resistance be R. Then 64 43 2 1 1
R2 R1
24 I 2 R I 2 R (2I ) 2 R 6I 2 R R1 R2 R1 R2 2
Power dissipated in Z is I 2 R = 4 W
Also,
Ans: A 40 43
21. 4 R2 R2 R3
R2 R3 R2
1 5
a R3
or R2
d
2 6

3
b 7
3
f c e
4 8 2
Therefore R1 2 R2 R3
3
Ans: B
Between pins 4 and 7, current
essentially passes through resistors
f, a, b, c, d and e. Note that a, b, c, are 23. By Flemings Left Hand Rule,
connected in parallel. Similarly, d and electrons will be deflected upwards. Q
e are connected in parallel. The parallel will have a higher potential than P.
arrangements are in series with resistor Ans: B
f. The effective resistance is therefore
24. For vertical equilibrium, BIL = mg
Reff Rabc Rde R f I = (m/L)g/B
1 1 = 0.11 A
1
3 2 Ans: C

= 1.83 25. Both wires in options A and B


experience a positive attractive force .
Ans: B
Wires in option D experience a repulsive

4
force. In option C, applying Flemings Hence
Left Hand Rule on the right wire gives an
upward force in the plane of the paper. The 1
9.62(1.6x10 19 ) (9.11x10 31 )v 2
left wire does not experience any magnetic 2
v 1.84 x 10 m s-1
6
field from the right wire and will therefore
Ans: A
not experience any magnetic force. Hence
there will be a zero attractive force hc hc
between the wires in C. E photon 2mc
30. 2 2
K electron p h h
Ans: C
2m 2m 2

26. The energy of a photon is given by


Ans: C
E = hf.

Ans: A

27. The work function of a metal


corresponds to the minimum photon
energy required to eject photoelectrons
from the metal surface. Hence since the
work function of calcium is half that for
platinum, the minimum photon energy to
emit photoelectrons for calcium will be
half that for platinum.
Ans: C

28. When the potential of the collector


plate is negative, electrons will experience
a force of repulsion as they move from
emitter to collector. When the potential of
the collector plate is so negative that even
the most energetic photoelectrons just fail
to reach it, the photocurrent drops to zero.
V0 is called the stopping potential.

Ans: C

29. Energy remaining after the atom has


been ionised is 20.00 10.38 = 9.62 eV.
This energy is given to the escaping
electron as kinetic energy.

5
Name : ________________________________ CT group : _________

VICTORIA JUNIOR COLLEGE


2013 JC2 PRELIMINARY EXAMINATIONS

PHYSICS 8866/02
Higher 1 16 Sep 2013
MONDAY
Paper 2 Structured Questions
2 pm 4 pm
2 Hours
Candidates answer on the Question Paper
No Additional Materials are required.

READ THESE INSTRUCTIONS FIRST For Examiners Use

Write your name and CT group at the top of this page. Section A
Write in dark blue or black pen on both sides of the 1
paper.
You may use a soft pencil for any diagrams, graphs or 2
rough working.
3
Do not use staples, paper clips, highlighters, glue or
correction fluid. 4

Section A 5
Answer all questions. 6
Section B Section B
Answer any two questions.
7
At the end of the examination, fasten all your work 8
securely together.
The number of marks is given in brackets [ ] at the end 9
of each question or part question. Total
(max. 80):

This question set consists of a total of 21 printed pages.

1
Data

speed of light in free space, c = 3.00 108 m s-1

elementary charge, e = 1.60 10-19 C

the Planck constant, h = 6.63 10-34 J s

unified atomic mass constant, u = 1.66 10-27 kg

rest mass of electron, me = 9.11 10-31 kg

rest mass of proton, mp = 1.67 10-27 kg

acceleration of free fall, g = 9.81 m s-2

Formulae

uniformly accelerated motion, s = ut +( ) at2


v2 = u2 + 2as

work done on/by a gas, W = pV

hydrostatic pressure, p = hg

resistors in series, R = R1 + R2 +

resistors in parallel, 1/R = 1/R1 + 1/R2+

2
Section A

Answer all the questions in this section.

1. A student measured the density of a metal cylinder by finding its mass with a balance
reading to 0.1 g and its dimensions with vernier calipers reading to 0.05 mm. He
recorded his readings as follows:

diameter of cylinder = 20.00 mm


length of cylinder = 18.00 mm
mass of cylinder = 51.3 g

From the data above, he calculated the density of the metal cylinder to be
9.072 103 kg m-3.

(a) With the given uncertainties in the readings, calculate the percentage uncertainty in the
value of the density. [2]

Percentage uncertainty = %

(b) Express the density of the metal cylinder together with its uncertainty to an appropriate
number of significant figures. [2]

Density = . . kg m-3

(c) Give an example of a systematic error which could occur in this experiment. Explain
how it could be corrected. [1]

Systematic error: ..........

..........

Correction of error: ..........

..........

3
2. A wildlife conservation scientist spotted a monkey hanging from a tree. It was at a vertical
distance h and a horizontal distance d away from the level of the sleeping dart gun held by
the scientist (see Fig. 2). Overcome with excitement, he forgot about gravity and aimed
his sleeping dart straight at the monkey.

Initial direction of
sleeping dart
h Fig.2

The monkey saw the scientist and panicked and also forgot about the laws of gravity. At
the moment he saw the scientist shoot, it released its grip on the branch and started to fall
freely. Assume that air resistance was negligible.

(a) State the equation relating the vertical distance s1 travelled by the monkey with
respect to acceleration due to gravity g and time t when it was falling. [1]

(b) State the equation relating the vertical distance s2 travelled by the dart with respect
to g, the same time t and the initial vertical velocity uy. [1]

(c) Hence show that the scientist would hit the monkey with the dart after all. [3]

4
3. In an experiment, a student makes use of a d.c. electric motor to hoist a load of 5.0 N
through a height of 80.0 cm in 4.0 s. The motor operates on a potential difference of
6.0 V and draws a current of 1.0 A.

(a) Calculate the useful mechanical power output of the motor. [1]

Power output = . W

(b) Calculate the efficiency of the motor used in this experiment. [2]

Efficiency = .

(c) Explain what contributes to the inefficiency of the motor set up. [2]

..

..

..

..

5
4.

Fig. 4

Light bulbs of fixed resistance 3.0 , and 6.0 , a cell of emf 9.0 V and a switch S are
connected as shown in Fig. 4.

(a) Calculate the current in bulb B. [3]

Current = A

(b) Deduce which bulb is the brightest. Justify your answer. [2]

6
5.

A Conducting rails

24
12 V

0.50 m
switch

B
Fig. 5

A 12 V DC supply is connected in series with a rod AB of length 0.50 m between two


conducting rails (see Fig. 5).The rod has an effective resistance of 24 . The frictionless
connecting rails are infinitely long and have no electrical resistance. The rod lies in the
region of a constant magnetic field of field strength 1.0 T. The direction of the field is
perpendicular to the plane of the page and is into the page. The rod rests on the rails and is
free to move in the plane of the paper. The rod is initially at rest.

(a) Calculate the magnitude and determine the direction of the external horizontal
force required to prevent the rod AB from moving when the switch is closed. [3]

Force required = . N

Direction :

(b) Explain the transformation of energy that occurs in (a) when

(i) the rod is held steady by the external force [1]

..

(ii) the rod is allowed to start moving in the magnetic field from rest. [1]

7
6. Read the following passage about the Earths atmosphere:

The atmosphere of the Earth near its surface can be divided into several layers based on
how the temperature and density vary with altitude.

The troposphere begins at the surface of the Earth. It is mostly heated by transfer of
energy from the Earths surface, so on average the lowest part of the troposphere is
warmest and temperature decreases with altitude. This promotes vertical mixing (hence
the origin of its name in the Greek word , tropos, meaning "turn"). The troposphere
contains roughly 80% of the mass of the atmosphere. The tropopause is the boundary
between the troposphere and the next higher level, the stratosphere.

Within the stratosphere, temperature increases with height due to increased absorption of
ultraviolet radiation by the ozone layer, which restricts turbulence and mixing. While the
temperature may be 60 C at the tropopause, the top of the stratosphere is much warmer,
and may be near freezing. The stratopause is the boundary between the stratosphere and
the next higher level, the mesosphere. The pressure here is 1/1000 that at sea level.

The mesosphere is the layer where most meteors burn up upon entering the atmosphere.
Temperature decreases with height in the mesosphere. The mesopause, the temperature
minimum that marks the top of the mesosphere, has an average temperature around
85 C. At the mesopause, temperatures may drop to 100 C. Due to the cold
temperature of the mesosphere, water vapor is frozen, forming ice clouds.

Fig. 6.1 shows the variation with altitude above sea level of the temperature, absolute
pressure and density of the air in the atmosphere.

Altitude above Temperature, Absolute Pressure, Density,


Sea Level, / oC p / 104 N m-2 / kg m-3
h/m
0 15.00 10.130 1.225
1000 8.50 8.988 1.112
2000 2.00 7.950 1.007
3000 -4.49 7.012 0.9093
4000 -10.98 6.166 0.8194
5000 -17.47 5.405 0.7364
6000 -23.96 4.722 0.6601
7000 -30.45 4.111 0.5900
8000 -36.94 3.565 0.5258
9000 -43.42 3.080 0.4671
10000 -49.90 2.650 0.4135
15000 -56.50 1.211 0.1948
20000 -56.50 0.553 0.08891
25000 -51.60 0.255 0.04008
30000 -46.64 0.120 0.01841
40000 -22.80 0.029 0.003996

8
50000 -25.00 0.008 0.001027
60000 -26.13 0.002 0.0003097
70000 -53.57 0.0005 0.00008283
80000 -74.51 0.0001 0.00001846

Fig. 6.1

(a)
(i) Using information from the passage and Fig. 6.1, write down, in Fig. 6.2, the
ranges of altitudes to which each of the three atmospheric layers belong. [3]

Atmospheric layer Lowest altitude of Highest altitude of


layer / m layer / m

troposphere

stratosphere

mesosphere

Fig. 6.2

(ii) State the atmospheric layer in which the ozone layer is located. [1]

.
(b)
(i) Show that the Ideal Gas equation pV = nRT may be expressed in the form

where V= gas volume


n = number of moles of gas,
= density of the gas,
R = molar gas constant,
T = thermodynamic temperature
and Mm = molar mass of gas. [2]

9
(ii) Using data from Fig. 6.1, plot a suitable graph in Fig. 6.3 to show that the air in
the atmosphere behaves approximately like an ideal gas up to an altitude of about
30,000 m. You may use the grid below for data manipulation. [5]

No.
1
2
3
4
5
6

Fig. 6.3

10
(iii)Using your graph in Fig. 6.3, estimate the average molar mass of the air molecules
within the atmosphere. [2]

Average molar mass = .. kg mol-1

The composition of the atmosphere is shown in the following table.

Gas Percentage by number Molar mass / g mol-1

Nitrogen (N2) 78.1% 28.0

Oxygen (O2) 20.9% 32.0

Argon (Ar) 0.93% 40.0

Others 0.07% Not available

(iv) Using the values from the table, estimate the average molar mass of the molecules
in the Earths atmosphere. [2]

Average molar mass = .. kg mol-1

11
Section B

Answer two of the questions in this section.

7.
(a)
(i) Explain what is meant by an elastic collision. [1]

..

..

(ii) State the condition needed for the momentum of a system to be conserved. [1]

..

..

(b)

m1 m2 m1 m2

u1 u2 v1 v2

Before collision After collision

A particle of mass m1 moving a velocity u1 collides elastically with a second particle


of mass m2 which is moving with a velocity u2 in the same direction as the first
particle. After the collision, the velocities of the two particles are v1 and v2
respectively.

(i) Write down the equation relating the initial velocities u1 and u2 to the final
velocities v1 and v2. [1]

(ii) Write down the equation expressing the conservation of the momentum of the
two-particle system during for the collision. [1]

12
(iii) Using the equations from (b)(i) and (b)(ii), derive an expression for v1 in terms
of u1, u2, m1 and m2. [3]

(iv) Using the equations from (b)(i) and (b)(ii), derive an expression for v2 in terms
of u1, u2, m1 and m2. [1]

(v) If m1 = 2 units, m2 = 3 units, and u2 = u1, find v1 and v2 in terms of u1. [2]

13
(c)

u u v1 v2

Before collision After collision

A deuterium nucleus ( ) and tritium nucleus ( ) moving with the same speed but
in opposite directions suffer a head-on collision with each other.
(i) State Newtons Second Law of Motion. [1]

...

...

...

...

(ii) State Newtons Third Law of Motion. [1]

...

...

...

...

(iii) Explain why the deuterium and tritium nuclei will not come to rest at the same
time during the collision. [3]

...

...

...

...

...

...

14
(iv) On the same set of axes, sketch graphs to show the variation with time of the
velocities of the nuclei over the course of the collision. Label your graphs D and T
to identify the deuterium and tritium nuclei respectively. [3]

(v) Determine the ratio of the initial speed of the deuterium nucleus to that of the
tritium nucleus for the two nuclei to come to rest at the same time during the
collision. [2]

Ratio of initial speeds =

15
8. A stereo system in a large room has two identical loudspeakers, S1 and S2, 1.20 m apart as
shown in Fig. 8.1. The sound from the loudspeakers causes the diaphragm of the eardrum
of an observer at point A to vibrate at the frequency of the sound produced. The cone of
the loudspeakers vibrates with displacement x, where x = a sin 1528t where t is measured
in seconds (s).The speed of the sound in the room is 330 m s-1.

Centre line

Fig. 8.1 (Not to scale)

(a) (i) Calculate the frequency of the sound. [2]

Frequency = .. Hz

(ii) Show that the number of waves between either S1 or S2 and the eardrum is about
34.75. [3]

number of waves =

(iii) Hence, determine the phase difference between the vibrations of the loudspeaker
cone and the eardrum. [2]

Phase difference = . rad

16
The system in Fig. 8.1 is adjusted such that the speakers are emitting the sound
frequency with the same phase.

(iv) Explain what the observer standing on the centre line at point A would detect. [2]

...

...

...

(v) Describe and explain what the observer would hear as he moves along the line at
right angles to the centre line. [3]

...

...

...

...

...

...

...

...

(b)

3.00 m

light 0.600 mm
screen
m

Fig. 8.2 (not drawn to scale)

17
(i) A double slit with slit separation 0.600 mm is situated 3.00 m away from a screen as
shown in Fig 8.2. The double slit is illuminated with coherent light of wavelength
600 nm. Calculate the distance between two adjacent bright fringes formed on the
screen.

[2]

Distance: ______________ m

(ii) Explain the parts played by diffraction and by interference in the production of the
observed fringes. [3]

(c) Two coherent waves of intensities I and 3 I meet in phase at a point. Calculate in
terms of I the intensity of the resultant wave at that point. [3]

Resultant intensity : ____________________

18
9.
(a) Consider a photoelectric experiment involving energetic radiation falling on a
polished emitter plate. Electrons are emitted from the plate with a spread of kinetic
energies up to a certain maximum value.

(i) Describe with the help of a simple diagram how the maximum kinetic energy of
emitted photoelectrons is determined. [4]

(ii) By referring to Einsteins photoelectric equation, suggest how the Planck constant
h can be determined experimentally. [3]

19
(iii) Light of total intensity 1.0 W cm-2 falls on a clean iron sample 1.0 cm2 in area.
The iron sample reflects 96 % of the light and only 3.0 % of the absorbed energy
lies in the ultra-violet region of the spectrum above the threshold frequency.

1. Deduce the intensity of incident radiation that is available for the photoelectric
effect. [2]

Intensity = W cm-2

2. Assuming that all the photons in the ultra-violet region have an effective
wavelength of 250 nm, calculate the number of photons striking the iron
sample per second. [3]

Rate of photon impact = . s-1

(b) The table below shows some of the energy levels for the hydrogen atom. A and F
represent the two extreme energy levels in the atom.

Level Energy/eV
A 0
B -0.278
C -0.378
D -1.51
E -3.40
F -13.6

(i) Calculate the wavelength emitted when the electron falls from state A to state F.
[2]

Wavelength = . m

20
(ii) The result of the Bohr theory for the hydrogen atom can be expressed by

1 1 1
R 2 2
n1 n2

where R is a constant; n1 and n2 are positive integers and n2 > n1 . The equation
provides a way of calculating the wavelength emitted when an electron falls
from level n 2 to level n1 . For the ground state, n1 = 1. The integer n2
corresponds to some higher energy level. Using the result from (i), show that
R 1.09 x 107 m-1. [1]

(iii) Using the expression in (ii), determine the ratio of the wavelengths for Lyman-a
radiation (n = 2 to n = 1) to Balmer-a radiation (n = 3 to n = 2). [3]

Ratio = ..

(iv) Level B corresponds to n 2 = 7. Deduce the total number of spectral lines that will
be emitted when electrons in hydrogen atoms make transitions from level B to
level F. [2]

Total number of lines =

*********** END OF PAPER**********

21
VICTORIA JUNIOR COLLEGE Putting (3) into (2),
sin h
SUGGESTED SOLUTIONS TO 2013
cos u cos t
PHYSICS H1P2 PRELIM EXAMS
u sin t h
4m or u y t h
1(a)
d 2 L in(1) , s1 s2 h dart will hit monkey
m d L
2 as it falls.
m d L
3(a) The useful mechanical power output of
0.1 0.05 0.05 the motor is given by the change in
x100 2 x100
51.3 20.00 18.00 gravitational potential energy per unit time.
= 0.973 mgh 5.0x 0.80
Hence, Po 1.0 W .
t 4.0
0.973
1(b) 9.072 103 kg m 3
100
= 0.09 10 3 kg m 3 ( to 1 s.f.) 3(b) The input power to the motor, Pi = IV
= 6.0 x 1.0 = 6.0 W.
= (9.07 0.09) 10 3 kg m 3 Hence, the efficiency of the motor is
= (Po/Pi) x 100 = 16.7 %
1(c) Systematic error: zero reading of
vernier calipers and/or electronic balance 3(c) The factors contributing to the
Subtract the zero reading from the inefficiency include joule heating loss in the
measurements obtained. armature coil of the motor and mechanical
friction that occurs at the turning parts.
2(a)

1 2 4(a)
s1 gt
2

2(b)

1 2
s2 u y t gt
2

(c) s1 s2 u y t ..(1) The current through bulb A is


E 9.0
IA
h Reff 6.0x3.0
Now tan ..(2) where is angle 3.0
d 6.0 3.0
between initial direction of dart and the
horizontal. = 1.8 A.
Horizontally, d u cos t ..(3)
Now I B 6.0 I C 3.0

1
and I A I B I C AB as well as kinetic energy due to its
motion in the magnetic field.
3.0 1.8
IB IA
6.0 3.0 3.0
6(a) (i)
= 0.60 A
Atmospheric Lowest Highest
layer altitude of altitude of
4(b) The bigger the power dissipation in a layer / m layer / m
bulb, the brighter the bulb. troposphere 0 x
PA I A2 R A (1.8 2 )3.0 9.7 (accept
15000 to
PB (0.60 2 )6.0 2.2 20000)
stratosphere x 40000
PC (1.8 0.60) 2 3.0 4.3 mesosphere 40000 80000

Hence bulb A is the brightest. 6(a)(ii) stratosphere.

5(a) When switch is closed, I = E/R 6(b) (i)


in anticlockwise direction.
By FLHR, F = BIL ( )
E
= B L
R
Since , where M = total mass of the
12
(1.0) (0.50) = 0.25 N to the right. gas,
24
External force has to be applied 0.25 N
to the left to prevent the rod from moving.
( )( )
5(b) (i) The electrical power of the DC
supply is dissipated as heat in the rod AB.
Since density ,
5(b)(ii) The electrical power of the DC
supply is transformed into heat in the rod

2
6(b)(ii)

h/m / oC T/K p / 104 / kg m-3 T /


N m-2 kg m-3 K
1000 8.50 281.65 8.988 1.112 313.2
3000 -4.49 268.66 7.012 0.9093 244.3
5000 -17.47 255.68 5.405 0.7364 188.3
7000 -30.45 242.70 4.111 0.5900 143.2
9000 -43.42 229.73 3.080 0.4671 107.3
15000 -56.50 216.65 1.211 0.1948 42.20

p / 104 N m-2

12

(350, 10.0)
10

8.0

6.0

4.0

2.0

0 100 200 300


-3
T / kg m K

3
6(b)(iii) From the graph, the gradient is 7(b)(iv) Substitute v1 from (iii) into (3):
given by: ( ) ( )

( ) ( )

the average molar mass of the gas 7(b)(v) If m1 = 2 units, m2 = 3 units, and
molecules in the atmosphere is: u2 = u1,
Mm = 0.0291 kg mol-1. ( ) ( )
8(b)(iii) Estimated average molar mass is ( ) ( )

= 28.9 g mol-1 = 0.0289 kg mol-1. Also,


( ) ( )

SECTION B ( ) ( )

7(a)(i) An elastic collision is one in which


the kinetic energy of the system is
conserved.
7(c)(i) The rate of change of momentum of a
body is proportional to the net force acting
7(a)(ii) The net momentum of a system will
on it and occurs in the direction of the force.
be conserved if the net external force acting
on the system is zero.
7(c)(ii) When a body exerts a force on a
second body, the second body will exert a
7(b)(i) Since the collision is elastic,
force on the first body that is equal in
Velocity of approach = velocity of
magnitude but opposite in direction to that
separation
which acts on itself.

7(c)(iii) By Newtons 3rd law, the two nuclei


7(b)(ii)
will exert forces of equal magnitude on each
other.
By Newtons 2nd law, this means that the
momenta of the nuclei will decrease at the
7(b)(iii) From (1),
same rate.
However, the tritium nucleus has a larger
initial momentum due to its greater mass.
Substitute into (2):
This means that the momentum and hence
velocity of the lighter deuterium nucleus
will decrease to zero earlier than that of the
( ) ( ) tritium nucleus.

4
Alternative: The wavelength = v/f = 330/764
= 0.432 m.
The principle of conservation of momentum Hence the number of waves between S1 and
dictates that the total momentum of A or S2 and A = 15.0/0.432 34.75.
interacting particles is constant provided no
net external force acts on the system. 8(a)(iii) The phase difference between the
loudspeaker cone and eardrum vibrations is
Because the deuterium and tritium nuclei x
do not have the same mass, they will given by x 2 .

have a non-zero net momentum when x = (35-34.75) = 0.25
they travel towards each other with the
0.25
same speed. x 2 1.57 rad
2
Consequently, if they come to rest at the
same time upon collision with each other, Alternative:
momentum would not be conserved and this x = (34.75-34) = 0.75
will violate the principle of conservation of 0.75 3
x 2 4.71 rad
momentum. 2

7(c)(iv)
8(a)(iv) The observer at A will detect a loud
v sound.
D This is due to the constructive interference
+0.6u
u
of the two waves from S1 and S2 which are
t coherent and in phase and travel through the
0
same path length before they meet.
u
1.4u
T
8(a)(v) As the observer moves along the line
perpendicular to the central line, whenever
the path difference is an integral number of
wavelengths, he will detect a maximum due
7(c)(v) For the nuclei to come to rest to constructive interference.
simultaneously, they must start off with the When the path difference corresponds to
same momentum according to the reasoning (n + ) wavelengths, where n is an integer,
in (c)(iii). Hence, considering only he will detect a minimum.
magnitude, Hence he will detect alternate loud and soft
sounds due to the changes in the path
difference of the waves.

8(b)(i) The fringe separation is given by


8(a)(i) From equation, = 1528 = 2f, D
x
i.e. f = 764 Hz d

8(a)(ii) From geometry, distance (600x10 9 )(3.00)


x = 3.00 x10-3 m
S1 A S2 A 152 0.62 14.99 15.0 m . 0.600x10 3

5
8(b)(ii) Light in passing through each of the When the sensitive galvanometer just
two slits will first undergo diffraction. registers zero, it means even the
Where two diffracted light waves meet at a most energetic photoelectron just
point on the screen in phase, a bright fringe fails to reach the collector plate. The
would be produced by constructive decelerating p.d. is called the
interference of light. stopping potential Vs.
If the waves meet at a point on the screen in The maximum kinetic energy of
anti-phase, a dark fringe would be produced photoelectrons is converted into
by destructive interference. electric PE. Hence Kmax = Vse.

8(c) Intensity amplitude2 9(a)(ii) Einsteins photoelectric equation:


Let amplitude corresponding to intensity I h
be A1 and that corresponding to intensity 3I hf Vs e or Vs f
e e
2 2
be A2. Hence I kA1 and 3I kA2 The experiment is conducted with a number
Where the two waves meet at a point, the of monochromatic frequencies. For each
resultant amplitude is Ares A1 A2 monochromatic frequency f used, the
stopping potential Vs is determined.
I res kAres k A1 A2
2 2
A graph of Vs against f yields a straight line
2
kA1 2kA1 A2 kA2
2
with a gradient equal to h/e. Hence h can be
found.
I 3I
I 2k
k 3I
k 9(a)(iii)1. Let I0 = 1.0 W cm-2.
= 7.46I Only 4 % of incident light is absorbed.
Hence intensity absorbed is I1 = 0.04I0
9(a) (i) Only 3% of I1 is available for photoelectric
effect. Hence
I2 = 0.03I1 = 0.03(0.04)I0
= 1.2 x 10 -9 W cm-2

9(a)(iii)2. Power of incident light is


P = I2A
Number of photons striking iron sample per
N P P I 2 A
sec is
t hc hc hc



N 1.2x10 9 1.0 250x10 9


t
6.63x10 34 3.0x10 8
1.51 x 109 s-1
The collector is made progressively
more and more negative relative to
the emitter so that photoelectrons
move at a progressively slower rate
towards the collector plate.

6
hc
9(b)(i) E A E F

6.63x10 3.0x108 9.14 x 10-8 m


34

0 13.6x1.6x10 19

1 1 1
9(b)(ii) R 2 2
n1 n2
1 1 1
8
R 2
9.14x10 1
7 -1
R 1.09 x 10 m

9(b)(iii) let the Lyman- and Balmer-


wavelengths be L and B respectively.
1 1 1
R 2 2
n1 n2
1 1 1
Lyman: R 2
L 1 2
1 1 1
Balmer: R 2 2
B 2 3

1 1 94

L 4 9 5 4
36 x
B 1 1 3 36 3
4 4
5
= 0.19
27

1(b)(iv) The number of lines that will be


emitted is 6+5+4+3+2+1
= 21.

7
VICTORIA JUNIOR COLLEGE
2013 JC2 PRELIMINARY EXAMINATIONS

PHYSICS 9646/01
Higher 2
27 Sep 2013
Paper 1 Multiple Choice FRIDAY
8.00-9.15 am
Additional Materials: Multiple Choice Answer Sheet 1 Hour 15 min

READ THESE INSTRUCTIONS FIRST

Write in soft pencil.


Do not use staples, paper clips, highlighters, glue or correction fluid.
Write your name, CT group and shade your index number on the Optical Mark Sheet
provided.

There are forty questions on this paper. Answer all questions. For each question
there are four possible answers A, B, C and D.
Choose the one you consider correct and record your choice in soft pencil on the
separate Answer Sheet.

Read the instructions on the Answer Sheet very carefully.

Each correct answer will score one mark. A mark will not be deducted for a wrong
answer.
Any rough working should be done in this booklet.

This document consists of 19 printed pages.

1
Data

speed of light in free space, c = 3.00 108 m s-1

permeability of free space, o = 4 10-7 H m-1

permittivity of free space, o = 8.85 10-12 F m-1


(1/(36)) 10-9 F m-1

elementary charge, e = 1.60 10-19 C

the Planck constant, h = 6.63 10-34 J s

unified atomic mass constant, u = 1.66 10-27 kg

rest mass of electron, me = 9.11 10-31 kg

rest mass of proton, mp = 1.67 10-27 kg

molar gas constant, R = 8.31 J mol-1 K-1

the Avogadro constant, NA = 6.02 1023 mol-1

the Boltzmann constant, k = 1.38 10-23 J K-1

gravitational constant, G = 6.67 10-11 N m2 kg-2

acceleration of free fall, g = 9.81 m s-2

2
Formulae

uniformly accelerated motion, s = ut + () at2


v2 = u2 + 2as

work done on/by a gas, W = pV

hydrostatic pressure, p = hg

gravitational potential, GM

r

displacement of particle in s.h.m., x = xo sin t

velocity of particle in s.h.m., v vo cos t


( xo2 x 2 )

resistors in series, R = R1 + R2 +

resistors in parallel, 1/R = 1/R1 + 1/R2+

electric potential, V = Q/4or

alternating current/voltage, x = xo sin t

transmission coefficient, T exp(-2kd)


8 2 m(U E )
where k
h2

radioactive decay, x = xo exp(-t)

decay constant, 0.693



t1
2

3
1. The behavior of many real gases deviates from pV = nRT but can be represented
quite closely over certain ranges of temperature and pressure by an equation of the
form


p a V b RT
V n
2


n

in which the values a and b are characteristic of the particular gas. What are the
units of a and b? (The other symbols in the equation carry the usual meanings.)
a b

A. Pa m-6 mol2 m3 mol-1

B. Pa m6 mol-2 m3 mol-1

C. Pa m6 mol-2 m-3 mol

D. None because they are dimensionless constants

2. Four students each made a series of measurements of the acceleration of free fall
g. The table shows the results obtained. Which student obtained a set of results that
could be described as accurate but not precise?

Student results, g / m s-2

A. 10.51 9.65 8.80 10.21

B. 9.81 9.45 9.84 9.88

C. 8.45 8.46 8.50 8.41

D. 9.45 9.21 9.89 9.85

4
3. A linear accelerator sends a charged particle along the axis of a set of coaxial
hollow metal cylinders as shown in the diagram.

track of particle
metal cylinders
The particle travels at constant speed inside each cylinder. The particle crosses
the gaps between the cylinders at equal time intervals, and at each gap its kinetic
energy increases by a fixed amount. Which of the graphs best represents the way
in which v, the velocity of the particle, varies with d, the distance along its track?

A.

B.

C.

D.

5
4. The following graph shows the variation with time of the velocity v of a ball
moving freely and vertically under gravity.

0 t

Which of the following graphs shows the variation with time of the displacement
s of the ball from its initial position?

A. s B. s t
0
t
0

C. s D. s
t t
0 0

6
5. A ball has a mass of 50 g, and is dropped vertically onto a hard ground as shown.

ground

Just before impact with the ground, the ball has a speed of 2.5 m s-1. It bounces
off the ground so that its speed immediately after losing contact with the ground
is 1.5 m s-1. The ball is in contact with the ground for 0.10 s. For the time that the
ball is in contact with the ground, what is the average force that the ground exerts
on the ball?
A. 0.49 N B. 1.3 N C. 2.0 N D. 2.5 N

6. A person stands on the floor of a lift. At which instant is the force between the
floor of the lift and the person a minimum?

A. The lift is travelling downwards at a constant speed.


B. The lift is travelling downwards with an acceleration of 3.5 m s-2.
C. The lift is travelling upwards but decelerating at 4.5 m s-2.
D. The lift is travelling upwards with an acceleration of 4.5 m s-2.

7. A spring of initial length 10.0 cm and spring constant 20 N m-1 is stretched slowly
at a constant speed. Find the work done on the spring when the length of the spring
is extended from 12.5 cm to 18.0 cm.

A. 0.0578 J B. 0.0303 J C. 0.550 J D. 0.116 J

8. A pendulum bob was attached to a string and hung from the roof of a car. When the
car accelerated on a flat road, the string of the bob made an angle 10o with the
vertical. Find the acceleration of the car.

10o

A. 0.49 m s-2 B. 1.73 m s-2 C. 6.49 m s-2 D. 9.67 m s-2

7
9. A new generation of Honda Civic cars of a hybrid design consists of a fuel engine
with a maximum output of 70 kW and an electric motor with a maximum output of
15 kW. Driven slowly with only the electric motor running, the car travels at a
maximum speed of 12 m s-1. The total drag D on the car is proportional to the
square of its speed v.

When pushed hard, the fuel engine will operate alongside the electric motor. What
is the maximum speed of the car when it is pushed to the output limits of the
engines?

A. 28.6 m s-1 B. 25.3 m s-1 C. 21.4 m s-1 D. 68.0 m s-1

10. Satellites A and B of masses mA and mB are placed in circular orbits of radii rA
and rB about the Earth, where the radii are measured from the centre of the Earth to
the respective satellites. What is the ratio of their velocities vA/vB in terms of the
given variables?
A. rA/rB
B. (rB/rA)1/2
C. rB/rA
D. (mArA)/(mBrB)

11. A space station orbits the Earth in a circular orbit. An astronaut on board fires a
control jet such that the radial force away from the Earth is exactly equal to the
gravitational force exerted by the Earth on the space station at that point.
Immediately afterward, which of the following is true of the motion of the space
station?

A. It exhibits many radial oscillations per revolution.


B. It spirals with increasing radius.
C. It moves in a circle with a larger radius.
D. It travels tangentially to its orbit.

8
12. A small 1.0 kg mass is welded to a light, rigid rod and is rotated in a vertical plane
about a fixed point C at a constant angular velocity of 1.0 rad s-1. The radius of
the circular motion is 1.0 m. When the mass is at the top of the rod in the vertical
position shown, what is the magnitude and direction of the force exerted by the
mass on the rod?

C
A. 8.8 N upward
B. 8.8 N downward
C. 10.8 N upward
D. 10.8 N downward

13. A small object tied to a point A via an inextensible string is released from rest at
an angle of 20 o with the vertical. Given that the length of the string is 4.0 m, and
the object is 30 kg, calculate the tension of the string when the object is at the
lowest point.
A

20o 4.0 m

30 kg

A. 35.5 N B. 259 N C. 294 N D. 330 N

14. A dock at Keppel Shipyard has a tidal entrance at which the water is 15 m deep at
noon, when the tide is at its lowest. The water is 35 m deep when the tide is at its
highest, which follows next at 6.00 pm. A ship, needing a depth of 20 m,
requires emergency service and needs to enter the dock as soon as possible that
afternoon. What is the earliest time that it can just clear the dock entrance?

A. 2:00 pm B. 4:20 pm C. 1:00 pm D. 2:10 pm

9
15. A massless spring with a spring constant k is attached to the end of a block of
mass M that is resting on a frictionless horizontal table. The other end of the
spring is fixed to a wall. A bullet of mass m is fired into the block with a speed vo
and comes to rest in the block (assume that this happens almost instantaneously).
Which of the following correctly shows the amplitude of the systems subsequent
motion?

Amplitude of motion

m
A. vo
k ( M m)

M
B. vo
k ( M m)

mvo
C.
k ( M m)

Mvo
D.
k ( M m)

10
16. The graph shows the shape at an instant t = 0 of part of a transverse wave
travelling along a string from left to right.

y
A
S

Which of the following correctly gives the displacement-time description for


particles P or S and the phase difference of the motion of S with respect to that
of particle P?

Displacement-time relation of P or S Phase difference of S with


respect to P

A. yS = -A sin t

B. yP = A sin t

C. yP = A cos t

D. yS = -A cos t

t
17. The oscillation of a particle on a stretched string is y 10 sin at a given
0.02
displacement. Given that t is in second(s) and y in cm, what is the speed of the
wave? The wavelength is 628 cm.

A. 5.00 m s-1 B. 50.0 m s-1 C. 40.0 m s-1 D. 4.00 m s-1

11
18. An ideal gas of mass m undergoes expansion at a constant pressure P as shown by
line B. Which line shows the expansion of a mass m/4 of the same gas, at a
pressure of P/2?

A
Volume V/ m3
8

B
4
C
2
1
D Temperature / oC

19. Which of the following statements is correct?

A. Coherent sources are not needed to produce interference fringes.

B. Two coherent light sources do not always produce bright and totally dark
fringes on a screen.

C. The atoms in a tungsten filament lamp produce coherent light waves.

D. The yellow light from a street lamp is coherent light.

20. Find the resultant resistance RAB across the setup shown below. Each resistance is
R.

A
B

1 1 3 2
A. R B. R C. R D. R
3 4 5 3

12
21. A cylindrical iron rod of diameter D and length L has resistance R measured
across its ends. The iron rod is subsequently stretched in a manner keeping its
volume constant throughout. To what length must it be stretched so that its new
resistance is 4R?

A. 4L B. 1.5 L C. 1.41 L D. 2 L

22. A cell of e.m.f. 10.0 V and of negligible internal resistance is connected to three
resistors of resistances as shown below.
R/2

R/2

A R B
10.0 V
When the circuit segment containing a cell of e.m.f E, shown below, is
connected across AB, the ammeter reading is zero. Find the e.m.f E of the cell.

X Y

A. 5.0 V B. 10.0 V C. 6.0 V D. 4.0 V

13
23. The force F experienced by an electron when placed in an electric field varied
with displacement x as shown in the graph below. Calculate the change in electric
potential energy of the electron when electron was moved from x = 1.0 cm to
x = 2.0 cm in the direction of the force.
F / N

2.0

x / cm
0 2.0

A. 4.0 10-8 J
B. 4.0 10-8 J
C. 2.0 10-8 J
D. 2.0 10-8 J

24. A metal sphere of radius 0.10 m was insulated from its surroundings and given a
large positive charge. A small charge was brought from a distant point to a point
1.00 m from the spheres centre. The work done against the electric field was W
and the force on the small charge in its final position was F. If the small charge
had been moved to 0.50 m from the centre of the sphere, what would have been
the values for the work done and the force ?

work done Force

A. 2W 2F

B. 2W 4F

C. 4W 2F

D. 4W 4F

14
25.

A positively charged particle of mass m and charge q moves with velocity v


perpendicular to a uniform magnetic field of flux density B. Find the period of
its circular motion.

26.

normal

S B Q

I Side view

Plan view

In the zero-gravity environment of a space station, a circular coil was initially


at rest with the normal to its plane pointing vertically upwards as shown
above. The coil is floating freely and can rotate about any axis in all three
dimensions. An electric current flows in the coil in an anti-clockwise direction
when seen from above. A horizontal magnetic field is now suddenly applied
parallel to the plane of coil in the direction P.

In which direction will the normal to the coil finally point, assuming that air
resistance is present to stop any initial oscillations?

A. The normal will point in the direction P.


B. The normal will point in the direction Q.
C. The normal will point in the direction R.
D. The normal will point in the direction S.

15
27.
normal B
B normal

t=0 t = t
A flat coil of N turns, area A and electrical resistance R is placed in a uniform
magnetic field of flux density B with its normal parallel to the field. The coil is
then rotated through 90o in time t . What is the electrical charge that flows in
the coil during this time?
D. zero

28.
ring
E

solenoid I = Io sin t

A ring is levitated in mid-air due to the e.m.f. E induced in it by the fluctuating


magnetic field generated by the sinusoidal current I = Io sin t flowing in the
solenoid directly under it. Which of the following graphs shows the phase
relationship between the I and E ? Assume that the magnetic flux density at the
rings position due to the solenoid is proportional to the current flowing in the
solenoid.

I I
t t
0 0
E E
A. C.

I I
t t
0 0
E
E
B. D.

16
29. In America, the mains electricity supply is 110 volts root-mean-square instead of
240 volts in the UK. As a consequence, kettles take longer to boil in the USA
compared with the UK. A British tourist takes his kettle to the USA and uses it
there. Estimate how many times longer it may take to boil the same mass of water
in the USA compared with the UK

A. 1.2 B. 1.9 C. 2.2 D. 4.8

30. A 20 resistor is connected to an AC power supply with a voltage output that


varies from 4.0 V to 2.0 V at equal time intervals as shown on the graph below.
What is the average heating power dissipated in the resistor?

A. 0.20 W B. 0.50 W C. 0.80 W D. 1.00 W

31. The work function of platinum is twice that of calcium. If the minimum photon
energy required to eject photoelectrons from the surface of platinum is E, then
that for the surface of calcium would be

A. 2E
B. 3E/2
C. E/2
D. impossible to determine from information given

32. Determine an expression for the ratio

(Take h = Planck constant, c = speed of light in vacuum, m = mass of


electron).

m c h 2mc 2h
A. B. C. D.
h m c h m c

17
33. In the hydrogen spectrum, the energy of the nth level is given by
13.6 eV
En .
n2
Determine the ratio of the wavelengths for Lyman- radiation (n = 2 to n = 1) to
Balmer- radiation (n = 3 to n = 2).

A. 5/48 B. 5/27 C. 1/3 D. 3

34. What can be said of the wave function of a particle ?

A. It represents the particle function associated with a wave


B. A big value of 2 indicates a small probability of the particles presence
C. A small value of 2 indicates a big probability of the particles presence
D. A big value of 2 indicates a big probability of the particles presence

35. Which of the following statements about quantum-mechanical tunnelling is false?

A. It is the effect of particles transitioning through a classically forbidden


barrier as in alpha-decay.
B. It is a direct result of the wavelike properties of particles; the wave
associated with a particle "decays" through a barrier such that the
amplitude of the wave on the other side of the barrier is large enough for
there to be a finite probability of finding the particle there.
C. Tunnelling by electrons through a barrier leads to a change in the values of
electron momentum and energy
D. In the scanning tunnelling microscope, the tunnelling current is
proportional to the transmission coefficient.

36. A laser produces light pulses each of energy E and duration t. The beam is circular
and of diameter d. The wavelength of laser light is while the Planck constant
and the speed of light in vacuum are h and c respectively. What is the expression
for the number density (i.e. the number per unit volume) of photons in each
pulse?

4 E E 4E 4 E2
A. B. C. D.
d 2 hc 2t d 2 hc 2t dhc 2t d 2 hc 3t 2

18
238
37. When U is bombarded by slow neutrons it is transformed, absorbing a single
92

neutron and subsequently emitting two particles. The resulting nuclide is


240 240 239 239
A. 93 Np B. Pa
91 C. 94Pu D. 90Th

38. Two deuterium nuclei fuse together to form the Helium-3 nucleus, with the
release of a neutron. The reaction is represented by

2
1 H 12H 23He01n energy

The binding energies per nucleon are:


for 12 H 1.09 MeV
for 23 He 2.54 MeV

How much energy is released in this reaction?


A. 0.36 MeV B. 1.45 MeV C. 3.26 MeV D. 5.44 MeV

39. The graph below shows the number of particles Nt emitted per second by a
radioactive source as a function of time t.

The relationship between Nt and t is

A. Nt = 20 e-(20t/s) B. Nt = 3 e-(0.05t/s) C. Nt =20 e-(0.05t/s) D. Nt = 1000 e(0.05t/s)

40. A radioactive source consists of 6.4 1011 atoms of a nuclide of half-life 2 days.
A second source consists of 8.0 1010 atoms of another nuclide of half-life 3
days. After how many days will the numbers of active atoms in the two sources
be equal?

A. 9 B. 12 C. 15 D. 18

19
VICTORIA JUNIOR COLLEGE 4.
SUGGESTED SOLUTIONS TO 2013
the value of the velocity v at any time
H2 P1 PHYSICS PRELIM EXAMS
on the given graph gives the gradient of
a the s-t graph at that time on the required
1. units of p = units of 2 graph.
V

n For the first portion of the v-t graph, the
units of a = Pa m mol 6 2
value of v is positive but decreases, until it
instantaneously decreases to a negative
V value. Hence, the gradient of the s-t graph
Unit of b = units of
n starts off positive, but decreases slowly,
until a certain point when it
= m 3 mol 1 instantaneously changes to a negative
Ans: B value.

2. By calculating the average values, Continuing with the reasoning, we see that,
the results of students A, B and D proved in the second portion of the v-t graph, the
to be quite accurate. Among them, the value of v is negative and continues to
results for student A had the most decrease to even larger negative values.
deviation. Hence his result is deemed Hence, on the s-t graph, the curve will start
accurate but not precise. off with a negative gradient in the second
portion of the graph, and it will become
student Average g / m s-2 even steeper with a more negative gradient
A 9.79 with time.
B 9.75
C 8.46 At the start of the third portion of the v-t
D 9.60 graph, the value of v changes
instantaneously from a negative value to a
positive value, before decreasing to zero.
Ans: A
On the s-t, the gradient of the curve will
also change instantaneously from a
3. The kinetic energy E is related to negative value to a positive value, before
the speed v via decreasing to zero.
Ans: A
5. Let F be the force ground exerts

on ball. Take upward as positive.
(F mg) = mv/t
Hence, as E increases by fixed amounts, F = (0.050)[1.5-(-2.5)]/0.10]
the speed v will also increase. The increase + (0.050)(9.81)
of v at each succeeding gap gets smaller = 2.5 N
with v plateauing off to the ceiling value of Ans: D
c, the speed of light.
Ans: C

1
W = 0.5 (20) (0.0802 - 0.0252) = 0.0578 J
6.
Ans: A
N
8.
a 10o
a T

mg
mg Consider the pendulum bob.
By N2L, mg N = ma
Vertical equilibrium: T cos 10 0 mg....(1)
For A, a = 0 Horizontal acceleration:
Hence N = mg T sin 10 0 ma.....(2)
For B, N = m(g-a) where a > 0, (2) a
gives tan10 0
(1) g
a = 3.5 m s-2. N = m (g 3.5)
a 9.81 tan10 1.73 m s-2
0

Ans: B

9. Given D v 2 and when car is travelling


at its maximum speed, the driving force is
equal to D. The power output, P = Dv.
N Hence at its maximum speed, P v3 .

a At motor output of 15 kW, the speed


v1 = 12 m s-1. Hence with both fuel engine
and electric motor operating at the limits
of the power output, the maximum speed
of the car, v2 is given by the relation:
mg
For C, N mg = ma 70 15 v23 v23
3 3 v2 21.4 m s-1
N = m(g+a) 15 v1 12

Here a < 0, a = - 4.5 m s-2 Ans: C


Hence N = m (g- 4.5)
10. For circular motion of A around the
For D, N = m(g+4.5)
earth, GMmA/rA2 = mAvA2/rA (1)
For circular motion of B around the earth,
Ans: C GMmB/rB2 = mBvB2/rB (2)
(1)/(2)
vA/vB = (rB/rA)1/2
7. Work done = change in elastic PE =
1 2 1 2
kx2 kx1 , where x is the extension Ans: B
2 2
in m

2
35 m
11. With the gravitational force cancelled
away by the force of the control jet, the
space station effectively moves along the 25 m
tangent to its orbit immediately after firing
and no longer moves in a circle.

Ans: D 15 m
N
12. The tidal displacement from its
M equilibrium depth can be represented by
equation:
Mg
2
x 10 cos( t ) where its period of tidal
12
motion is 12 hours. At 20 m deep,
x = - 5.0 m. Hence
2
5 10 cos( t ) t 2 hours .
12
Consider forces on the mass m,
Therefore the earliest time for the ship to
Mg N = Mr2, where N is the force enter the dock is 2:00 pm.
of rod on mass upward.
1.09.81 N = 1.01.01.02 Ans: A
N = 8.8 N
By N3L, force of mass on rod is 8.8 N
15. Upon impact by the bullet in this
downward.
inelastic collision, the system will move
with a common speed given by the
Ans: B
principle of conservation of momentum:
13. Consider motion of object from the m
point of release to the lowest point. mvo ( M m)v v vo
M m
Loss in GPE of object = gain in KE The maximum compression produced in
mg(4.0 4.0cos20o) = (1/2)mv2 the spring will be the amplitude of
v2 = 4.73 m2 s-2 subsequent oscillation. Hence, by
For circular motion of the object at the conservation of energy principle:
1 2 1 m
lowest position, kA loss in KE ( M m)( vo ) 2
T mg = mv2/r 2 2 M m
T = 330 N mvo
A
k ( M m)
Ans: D
Ans: C
14. The equilibrium tidal depth is 25 m
3
with an amplitude of the tidal swing equal 16. Particles P and S are separated by .
4
to 10 m.
Hence the phase difference between them

3
3 amplitudes to produce totally dark fringes
is Particle P is at its amplitude
2 when they interfere destructively. Not all
position and is moving downwards as the coherent light sources produce light of
wave propagates. Its displacement time equal amplitudes where they interfere
equation is thus given by yP = A cos t. destructively

C Incorrect. The atoms in a tungsten


Ans: C
filament lamp produce light waves
17. From the equation given, angular randomly and the phase relationship is not
2 constant.
frequency = (0.02)-1
T D Incorrect. The yellow light produced
Thus 2f = (0.02)-1 or f = (2 x 0.02)-1 from a street lamp is NOT coherent light
= 7.96 Hz. Now = 628 cm (given). as it is produced from sodium vapour
Thus, the speed of the wave, v = f = 7.96 which emits light randomly.
x 6.28 = 50.0 m s-1.
Ans: B
Ans: B
20. The circuit can be redrawn as shown
m below:
18. PV nRT = RT at temperature T.
Mm

m
= R( 273) where is celsius
Mm A B
temperature

Hence gradient of the V- graph is


mR
gradient (1)
PM m The resistance can then be easily
calculated:
When mass is m/4 and pressure is P/2.
1
1 1 R
m/4 Bottom parallel,
the new graph gradient R R R 2
( P / 2) M m
..(2) R 3
Overall bottom series, R R
2 2
Comparing (1) and (2), the new
graph gradient = half the original graph 1
2 1 3
gradient Overall parallel, R
3R R 5
Ans: C
Ans : C
19. A Incorrect. Coherent sources ARE
needed to produce interference fringes

B Correct. On top of having two coherent


sources, they must also have identical

4
V ( D2 )2 L Qq 1
24. From U , we have U
4 0 r r
L L 2
since Q and q are constants
R , since V is a constant
( )
D 2
V W r
2 Hence new
W rnew
Hence, Wnew 1.00

W 0.50
4 L2 ( 2 L) 2
4R Wnew 2 W
V V

New length is 2L Qq
From F , we have
4 0 r 2
Ans D
1
F since Q and q are
22. Total p.d. across the 3 resistors is r2
10.0 V. constants
2
The resistor of resistance R has p.d. F r
R Hence new
10 5.0 V . F rnew
R R
R Fnew 1.00
2
2 2
The p.d. across AB is hence 10.0 5.0 F 0.50
= 5.0 V . Fnew 4 F
For current to not flow across XY, the emf
of the cell must be E = 5.0 V.
Ans: B
Ans: A
25. For circular motion of the charged
23. particle,
electron
F mv 2
_ Bqv where r = radius of circle.
+ r

x m 2 r
Bq where T is period.
r T
Positive work is done on the electron by
the electric field. This can come about only 2m
because of a decrease in electric potential T
Bq
energy (EPE).
Therefore change in EPE = - work done by Ans: B
electric field in moving from x = 1.0 cm to
x = 2.0 cm
= - area under force-displacement graph
from x = 1.0 cm to x = 2.0 cm
= -(2.0 x 10-6)[(2.0 1.0) x10-2]
= -2.0 x 10-8 J

Ans: D

5
26. But,
P

where is the net charge that flows in


the coil in time t.
S B Q

Using Flemings left hand Rule, the upper | |


half-circle SPQ will experience a force
into the plane of the paper while the lower
half-circle SRQ will experience a force up considering only the magnitude.
out of the plane of the paper. The result is
Initial flux linkage,
a couple causing the normal to the coil to
rotate and point in the direction P.
Final flux linkage,
Ans: A

| |
27. By Faradays Law, the instantaneous
induced e.m.f. is given by:

Ans: B
Over a time interval t, the average 28. The instantaneous magnetic flux
induced e.m.f. is given by: density B produced by the solenoid at the
rings position is given by:

,considering only the magnitude.


where k is a constant.
But,

where is the average current that flows


the magnetic flux linkage of the ring is
in time t.
given by:

6
where C is a constant. 2
Vrms 10
P = 0.50 W
R 20
By Faradays Law, the induced e.m.f. in
the ring is given by: Ans: B

31. The work function of a metal


corresponds to the minimum photon
( ) energy required to eject photoelectrons
from the metal surface. Hence since the
, while work function of calcium is half that for
Ans: A platinum, the minimum photon energy to
29. To boil water, electrical energy is used emit photoelectrons for calcium will be
half that for platinum.
to raise the temperature of the water from,
say, room temperature, to the boiling point Ans: C
of water through a temperature rise hc hc
(assuming negligible loss of heat to the E photon 2mc
surroundings). Hence 32. 2 2
K electron p h h
2m 2m 2
V2
t mc where m is the mass of
R Ans: C
water, and c is the specific heat capacity of
water.
2
V 33. For Lyman-, we have
For America, 1 t1 mc (1)
R hc
2 E 2 E1 ..(1)
V L
For UK, 2 t 2 mc ..(2)
R For Balmer-, we have
2
V1 t1 hc
Hence 1 or E3 E 2 ..(2)
2
V2 t 2 B
t1 V2
2 2 Dividing (2) by (1), we have
240
2 4.8 E3 E 2 L
t 2 V1 110
E 2 E1 B
Ans: D
13.6 13.6

L 32 2 2
30. Rms voltage is
B 13.6 13.6
2 2
T 2 1
Vrms

0
V 2 dt
1 1

T 4 9 5
1 27
1
4 2 x 0.1 2 2 x 0.1 4
= 10 V Ans: B
0.2
Average power dissipated in resistor is

7
34. 2 represents probability density. 39. The equation for activity or number of
It is a measure of the probability of finding emissions per second is given by
a particle in a small region of space. Nt No et , No is the starting amount of
active nuclei present. Taking natural
Ans: D logarithms, the equation becomes
ln Nt t ln No . The gradient of the
35. Quantum-mechanical tunnelling graph gives the decay constant = 3/60
does not lead to a change in wavelength. = 0.05 s-1 while the intercept gives
Hence there will be no change to
ln No = 3 or No = 20. Hence the full
h equation for this activity is Nt =20 e-(0.05t/s).
momentum ( p ) and kinetic energy

p2 Ans. C
(K ).
2m
ln 2
t
Ans: C t1

40. First source: N N 0 e 2

36. Number of photons in a pulse of


E E ln 2
t
energy E is N N1 6.4x10 e 11 2
..(1)
hc hc

ln 2
t
10
N 2 8.0x10 e 3
.(2)
In time t, the pulse travels a distance s = ct
d 2 N1 N 2
Volume of pulse = As = (ct )
4
Hence number density of photons in pulse 11
ln 2
t
ln 2
t

is 6.4x10 e 2
= 8.0x10 e 10 3

N E 4 4 E ln 2 ln 2
2 2 2 6.4x1011
2
t t
3
V hc d ct d hc t 10
e
8.0x10
Ans: A
ln 2 ln 2
2.079 = t
37. By conservation of mass and charge: 2 3

U 01n 210 239


238
92 94 Pu t 18 days

Ans: C Ans: D

38. Total binding energy for the two 12 H


= 2 x 1.09 x 2 = 4.36 MeV. Total binding
energy for 23 He = 3 x 2.54 = 7.62 MeV.

Hence, the energy released = 7.62 - 4.36


= 3.26 MeV.

Ans. C

8
Name : _______________________________ CT group: ____________

VICTORIA JUNIOR COLLEGE


2013 JC2 PRELIMINARY EXAMINATIONS

PHYSICS 9646/02
Higher 2 16 Sep 2013
Paper 2 Structured Questions MONDAY
2 pm 3.45 pm
1 Hour 45 minutes
Candidates answer on the Question Paper.
No Additional Materials are required.

READ THESE INSTRUCTIONS FIRST

Write your name and CT group at the top of this page.


Write in dark blue or black pen on both sides of the paper.
You may use a soft pencil for any diagrams, graphs or rough
working.
Do not use staples, paper clips, highlighters, glue or correction
fluid.

Answer all questions.

At the end of the examination, fasten all your work securely For Examiners Use
together. 1
The number of marks is given in brackets [ ] at the end of each
question or part question. 2

6
7
8
9
Total
(max. 72)

This document consists of 20 printed pages.

1
Data

speed of light in free space, c = 3.00 108 m s-1

permeability of free space, o = 4 10-7 H m-1

permittivity of free space, o = 8.85 10-12 F m-1


(1/(36)) 10-9 F m-1

elementary charge, e = 1.60 10-19 C

the Planck constant, h = 6.63 10-34 J s

unified atomic mass constant, u = 1.66 10-27 kg

rest mass of electron, me = 9.11 10-31 kg

rest mass of proton, mp = 1.67 10-27 kg

molar gas constant, R = 8.31 J mol-1 K-1

the Avogadro constant, NA = 6.02 1023 mol-1

the Boltzmann constant, k = 1.38 10-23 J K-1

gravitational constant, G = 6.67 10-11 N m2 kg-2

acceleration of free fall, g = 9.81 m s-2

2
Formulae

uniformly accelerated motion, s = ut + () at2


v2 = u2 + 2as

work done on/by a gas, W = pV

hydrostatic pressure, p = hg

gravitational potential, GM

r

displacement of particle in s.h.m., x = xo sin t

velocity of particle in s.h.m., v vo cos t


( xo2 x 2 )

resistors in series, R = R1 + R2 +

resistors in parallel, 1/R = 1/R1 + 1/R2+

electric potential, V = Q/4or

alternating current/voltage, x = xo sin t

transmission coefficient, T exp(-2kd)


8 2 m(U E )
where k
h2

radioactive decay, x = xo exp(-t)

decay constant, 0.693



t1
2

3
1. A skilful tennis player hits a ball just before it touches the ground, such that it crosses
over the net at the point where it reaches its greatest height above the ground. (See Fig. 1)

x h

Fig. 1

Given that the top of the net is h = 0.915 m above the ground, and that the ball was
initially x = 2.50 m horizontally away from the base of the net,

(a) calculate the vertical component of the initial velocity of the ball needed for the above
stroke to be successful. [2]

Vertical velocity component = . m s-1

(b) calculate the time taken for the ball to travel from its initial position just above ground
to the top of the net. [1]

Time taken = .. s

4
(c) calculate the horizontal component of the initial velocity of the ball. [1]

Horizontal velocity component = . m s-1

When the ball hits the ground on the other side of the net, it suffers a 30 % reduction in
the vertical component of its velocity.

(d) Sketch a graph to show the variation with time of the vertical component of the balls
velocity, from the initial instant when the ball is hit to the instant when the ball
reaches its highest point on the other side of the net after the first bounce. Include
suitable numerical values of velocity and time in your sketch. Neglect air resistance.
[3]

5
2. In Fig. 2, the length of the uniform bar is 5.0 m and its weight is 500 N. The angle
between the wire and the bar is 20o.

Wire

Rough
Wall

Fig. 2

60o
20o

Bar

(a) In Fig. 2, draw and identify the forces acting on the bar for equilibrium.
[1]
(b) Calculate the magnitude of the tension in the wire. [2]

Tension = .. N

(c) Calculate the magnitude of the force of the wall on the bar and the angle the force
makes with the wall. [3]

Force on wall = .. N

Angle = o

6
3. Fig. 3 shows the Cavendish experiment which consists of two large fixed balls of lead
each of mass M = 100 kg. Two smaller balls of lead each of mass m = 1.00 kg are
attached to the ends of a light rigid rod which is supported horizontally by a very fine
fibre. By measuring how much the fibre gets twisted, one can measure the strength of the
gravitational force between the large and small balls. The centres of the four balls and the
rod lie in the same horizontal plane.

fine fibre

m Fig. 3
M
r
r

m M

The separation between the centres of mass of the large and small ball is r = 0.100 m.
The gravitational force of attraction between one large ball and one small ball separated
by r is 1.00 10-6 N.
(a) Calculate a value of G, the gravitational constant and explain why the gravitational
attraction of the earth need not be taken into consideration in the calculation. [2]

G = .. N m2 kg-2



(b) Explain why two sets of ball are used. [1]
.
.
.

7
(c) From your value of G in part (a), estimate the mean density of the Earth. Take the
radius of the Earth to be 6.37 106 m. [2]

Mean density of Earth = . kg m-3

8
4. A taut copper wire of length 0.80 m conducts a small alternating current of frequency
50 Hz. When a pair of magnets is placed with unlike poles on both sides of the wire, a
stationary wave of 5 segments is set up as shown in Fig. 4.

0.80 m

S Fig. 4

50 Hz
(a) Explain why the wire is observed to vibrate when an alternating current is passed
through it. [2]

.
.
.
.
(b) Calculate the fundamental frequency of the vibrations in the wire. [2]

Fundamental frequency = .. Hz

(c) Describe and explain what will be observed when the frequency of the current is
decreased progressively from 50 Hz to 40 Hz, to 30 Hz and then to 25Hz. [3]

.
.
.
.
.
.
.

9
5. Two ions A and B are separated by a distance of 0.72 nm in a vacuum, as shown in
Fig. 5. Ion A has a charge of 3.2 10 19 C and ion B has a charge of 1.6 10 19 C.

A B

Fig. 5

(a) Calculate the magnitude of the resultant electric field at the midpoint of the straight
line joining A and B. [2]

Electric field = .. N C-1

(b) Without making detailed calculations, draw labelled arrows on Fig. 5 to represent
(i) the electric field EA at the point X due to the charge at A only, [1]
(ii) the electric field EB at the point X due to the charge at B only, [1]
(iii) the resultant electric field E at the point X due to both charges. [1]

(c) Sketch on Fig. 5, lines representing the electric field caused by the two ions in the
region within the rectangle. Include the field line passing through X. [2]

10
6. A student uses a simple circuit (see Fig. 6.1) containing a voltmeter and an ammeter to
investigate the behaviour of different electrical components.

Black
box Fig. 6.1

50

(a) One of the components tested is an ideal diode which displays a zero potential drop
across it when conducting. The diode is part of a black box (see Fig. 6.1) which is
known to contain only the diode and two resistors, but the student cannot see how the
three components are connected. The student tests the 'black box' and obtains the
current-voltage graph shown in Fig. 6.2.

I / mA
20

10

V/V V/V
Fig. 6.2
-1.2 -0.6 0.6 1.2

-10

-20

In the space below draw a possible circuit diagram for the 'black box'. [2]

11
(b) Explain the shape of the graph. [2]

(c) Calculate a value for each of the two resistors. [2]

(d) Calculate the emf of the power supply when the diode is in reverse bias and the
current is 10 mA. [2]

12
7. Consider the electron in the hydrogen atom orbiting around the nucleus consisting of a
single proton. Bohr considered the energy of the atom to be the sum of the kinetic energy
of the electron in its orbit, assumed circular, and the electric potential energy of
interaction between the electron and the nucleus.
e2
(a) Show that the total energy of the atom is given by E where e is the
8 o r
electronic charge, 0 is the permittivity of free space and r is the orbital radius. [2]

(b) Bohr further assumed that the electron has wave properties such that n wavelengths
correspond to the circumference of the electrons circular orbit about the nucleus.
nh
From this assumption, he postulated that mvr where v is the electron velocity, h
2
is the Planck constant and n is a positive integer. By using this postulate and
considering the circular motion of the electron around the nucleus, show that the
n 2 h 2 0
radius of the electrons orbit is given by r where m is the electrons mass.
me 2
[1]

13
(c) The smallest orbital radius is called the Bohr radius a 0 . Calculate a value for a 0 . [1]

a0 = .. m

1
(d) Show that the energy E of the atom is given by E . [1]
n2

14
8. Read the following passage about the Earths atmosphere:

The atmosphere of the Earth near its surface can be divided into several layers based on
the temperature and density variation with altitude.

The troposphere begins at the surface. It is mostly heated by transfer of energy from the
Earths surface, so on average the lowest part of the troposphere is warmest and
temperature decreases with altitude. This promotes vertical mixing (hence the origin of its
name in the Greek word , tropos, meaning "turn"). The troposphere contains
roughly 80% of the mass of the atmosphere. The tropopause is the boundary between the
troposphere and the next higher level, the stratosphere.

Within the stratosphere, temperature increases with height due to increased absorption of
ultraviolet radiation by the ozone layer, which restricts turbulence and mixing. While the
temperature may be 60 C at the tropopause, the top of the stratosphere is much warmer,
and may be near freezing. The stratopause is the boundary between the stratosphere and
the next higher level, the mesosphere. The pressure here is 1/1000 that at sea level.

The mesosphere is the layer where most meteors burn up upon entering the atmosphere.
Temperature decreases with height in the mesosphere. The mesopause, the temperature
minimum that marks the top of the mesosphere, has an average temperature around
85 C. At the mesopause, temperatures may drop to 100 C. Due to the cold
temperature of the mesosphere, water vapor is frozen, forming ice clouds.

Fig. 8.1 shows the variation with altitude above sea level of the temperature, absolute
pressure and density of the air in the atmosphere.

Altitude above Temperature, Absolute Pressure, Density,


Sea Level, / oC p / 104 N m-2 / kg m-3
h/m
0 15.00 10.130 1.225
1000 8.50 8.988 1.112
2000 2.00 7.950 1.007
3000 -4.49 7.012 0.9093
4000 -10.98 6.166 0.8194
5000 -17.47 5.405 0.7364
6000 -23.96 4.722 0.6601
7000 -30.45 4.111 0.5900
8000 -36.94 3.565 0.5258
9000 -43.42 3.080 0.4671
10000 -49.90 2.650 0.4135
15000 -56.50 1.211 0.1948
20000 -56.50 0.553 0.08891
25000 -51.60 0.255 0.04008
30000 -46.64 0.120 0.01841
40000 -22.80 0.029 0.003996

15
50000 -25.00 0.008 0.001027
60000 -26.13 0.002 0.0003097
70000 -53.57 0.0005 0.00008283
80000 -74.51 0.0001 0.00001846

Fig. 8.1

(a)
(i) Using information from the passage and Fig. 8.1, write down, in Fig. 8.2, the
ranges of altitudes to which each of the three atmospheric layers belong. [3]

Atmospheric Lowest altitude of Highest altitude of


layer layer / m layer / m

troposphere

stratosphere

mesosphere

Fig. 8.2

(ii) State the atmospheric layer in which the ozone layer is located. [1]

.
(b)
(i) Show that the Ideal Gas equation pV = nRT may be expressed in the form

where n = number of moles,


= density of the gas,
and Mm = molar mass of gas. [2]

16
(ii) Using data from Fig. 8.1, plot a suitable graph in Fig. 8.3 to show that the air in
the atmosphere behaves approximately like an ideal gas up to an altitude of about
30,000 m. You may use the grid below for data manipulation. [5]

No.
1
2
3
4
5
6

Fig. 8.3

17
(iii)Using your graph in Fig. 8.3, estimate the average molar mass of the air molecules
within the atmosphere. [2]

Average molar mass = .. kg mol-1

The composition of the atmosphere is shown in the following table.

Gas Percentage by number Molar mass / g mol-1

Nitrogen (N2) 78.1% 28.0

Oxygen (O2) 20.9% 32.0

Argon (Ar) 0.93% 40.0

Others 0.07% Not available

(iv) Using the values from the table, estimate the average molar mass of the molecules
in the Earths atmosphere. [2]

Average molar mass = .. kg mol-1

18
9. When unpolarised light is passed through a polaroid called a polariser, it becomes
polarised so that the electric vector in light is constrained to oscillate along a polarising
axis in the polariser.

When this polarised light is next passed through another polaroid called an analyser, the
transmitted intensity I depends on the angle between the polarising axes of both
polariser and analyser.

The transmitted intensity I is thought to depend on angle according to the following


equation:

I a cos b

where a and b are constants.

You are provided with a few square sheets of polaroid and a photovoltaic cell but no
light intensity meter. You may also use any of the other equipment usually found in a
Physics laboratory.

Design an experiment to determine a value for the constant b.

You should draw a labelled diagram to show the arrangement of the apparatus. In your
account, you should pay particular attention to

(a) the identification and control of variables


(b) the procedure to be followed
(c) how the transmitted intensity I and the angle would be determined
(d) any precautions that would be taken to improve the accuracy and safety of the
experiment
[12 marks]

19

20
VICTORIA JUNIOR COLLEGE v / m s-1

SUGGESTED SOLUTIONS TO 2013 H2


P2 PHYSICS PRELIM EXAMS 4.24
1(a) Apply 2.97
to the vertical motion: t/s
0
0.43 0.86 1.16
m s-1.
4.24

1(b) v y u y a y t
2(a)
s
Force of
1(c) Consider horizontal motion: Wall on Bar

Tension
1(d) The vertical component of the velocity
just before the first impact = 4.24 m s-1.
the vertical component of the velocity just
after the first impact Weight

= 0.70 4.24 = 2.97 m s-1.

Using , the time taken to reach


highest point after the first impact is t given 2(b) Taking moments about end of bar in
by contact with wall,
clockwise moments = anti-clockwise
s moments
mg 2.5 sin 60 0 T sin 20 0 5

T = 633 N

1
2(c) 3(c) Let ME and RE be the mass and the
radius of the of Earth respectively.
Force of Wall on
Bar, R On the Earths surface,
2
2 gRE
g = GME/RE or M E
T G
Assuming the earth to be a perfect sphere
633 N with mean density E
20o 2
gRE
(4/3)RE3 E = M E
W = 500 N 30o G
3g
E
4GRE
Horizontally, for equilibrium
T cos 50 0 R sin 3(9.81)

R sin 407 ..(1) 4(1.00 x10 10 ) (6.37 x10 6 )
3680 kg m-3
Vertically, for equilibrium,
T sin 50 0 R cos 500
R cos 15.09 ..(2)
4(a) A current-carrying conductor placed
(1) / (2) gives: perpendicularly in a horizontal magnetic
tan 407 / 15.09 field will experience a transverse force.
87.880 87.9
Since the direction of the current is
alternating, the direction of the magnetic
R = 407 N
force on the wire also alternates at the same
frequency as that of the alternating current.
Hence, forced vibrations occur.
3(a) F = GMm/r2
1.0010-6 = G(100)(1.00)/0.1002
G = 1.0010-10 N m2 kg-2
4(b) For stationary waves on a vibrating
The gravitational attraction of the earth on wire, f nf 0 where f is the frequency of
the bodies need not be taken into account as vibration, n is the number of segments and f0
it acts in a direction perpendicular to the is the fundamental frequency.
direction of the gravitational attraction
between the balls.
50
Hence f 0 = 10 Hz
5
3(b) Only one set of balls will cause the rod
to become vertically oriented and no
meaningful measurements can then be taken.

2
4(c) Stationary waves with distinct segments 6(a)
will be observed when the frequency of the Rs
current is at 40 Hz and 30 Hz.

These 2 frequencies match the natural


overtones of the wire, and hence resonance
Rp
will occur, resulting in the establishment of
well-defined stationary waves.

When the alternating current is at 25 Hz, it


6(b) When the diode is in reverse bias
does not match the natural harmonics of the
(negative p.d. and negative current), the
wire. No stationary waves with distinct
resistance is simply that of the parallel
segments will be observed.
resistor Rp.

When the diode is in forward bias (positive


QA QB
5(a) E res p.d. and positive current), the current passes
4 o r 2 4 o r 2 through both resistors Rp and Rs in parallel.
As two resistors in parallel yield a smaller
3.2 10 19 1.6 10 19 effective resistance than either of the
= + 10 2
4 o (3.6 10 ) 4 o (3.6 10 )
10 2
individual resistances, a greater current
= 3.33 10 NC
10 -1 flows in forward bias.

5(b) & (c) 6(c) In reverse bias,


EA V 1.2
Rp = 120
I 10x10 3
E
X
EB
In forward bias,
1
1 1 1.2

R 20x10 3
p Rs
Rs = 120

6(d) When in reverse bias at 10 mA, the p.d.


across RP is 1.2 V. There is also a p.d. across
the 50 resistor of p.d. IR = (10x10-3) (50)
= 0.5 V.

3
Hence the power supply has emf = 1.2 + 0.5 e2 1
= 1.7 V. 7(d) E or E . Also
8 0 r r

n 2 h 2 0
r or r n 2
1 2 e2 me 2
7(a) E mv ..(1)
2 4 0 r

For circular motion of the electron around 1


Hence E
the nucleus, n2
e2 mv 2
..(2)
4 0 r 2 r
8(a) (i)
Substituting (2) into (1), we have
Atmospheric Lowest Highest
e2 layer altitude of altitude of
E layer / m layer / m
8 0 r
troposphere 0 x
(accept
7(b) Given: 15000 to
20000)
nh nh stratosphere x 40000
mvr v
2 2mr mesosphere 40000 80000

e2 mv 2
Also 8(a)(ii) stratosphere.
4 0 r 2 r
8(b) (i)
2 2
e m nh
Substituting, Since , where M = total mass of the
4 0 r 2
r 2mr
gas,
n 2 h 2 0
rn
me 2 (shown) ( )( )

7(c) The smallest orbital radius corresponds


to n = 1. ( ) or
12 (6.63x10 34 ) 2 (8.85x10 12 )
r
(3.142)(9.11x10 31 )(1.6x10 19 ) 2
Since density ,
5.3 x 10-11 m

4
8(b)(ii)

h/m / oC T/K p / 104 / kg m-3 T /


N m-2 kg m-3 K
1000 8.50 281.65 8.988 1.112 313.2
3000 -4.49 268.66 7.012 0.9093 244.3
5000 -17.47 255.68 5.405 0.7364 188.3
7000 -30.45 242.70 4.111 0.5900 143.2
9000 -43.42 229.73 3.080 0.4671 107.3
15000 -56.50 216.65 1.211 0.1948 42.20

p / 104 N m-2

12

(350, 10.0)
10

8.0

6.0

4.0

2.0

0 100 200 300


-3
T / kg m K

5
8(b)(iii) From the graph, the gradient is used; distance between source and
given by: photovoltaic cell to be constant.

(b) Measure the transmitted intensity I


for a certain value of the angle . Vary
and measure the corresponding value of I for
a total of 6 readings.
the average molar mass of the gas
molecules in the atmosphere is: I a cos b lg I lg a b lg(cos )
Plot a graph of lg I against lg(cos). The
Mm = 0.0291 kg mol-1. gradient should yield the value of b.

8(b)(iv) Estimated average molar mass is


(c) Measurement of angle :
Hold the two polaroids in
position by retort stand
clamps close to each other.
= 28.9 g mol-1 = 0.0289 kg mol-1.
Fix the polariser in position
while enabling the rotation
of the analyser.
9. Diagram of set-up Pass light from the laser
through the polaroids. At a
Photovoltaic cell
certain rotation of the
Laser light
analyser, the micro-ammeter
will register the maximum
A value. This will correspond
to the case of the polarising
Polaroids clamped axes of both polariser and
on retort stands analyser coinciding with
each other.

(a) The independent variable is the angle Using a marker, mark out two
between the polarising axes of the two identical lines, one each on polariser and
polaroids while the dependent variable is the analyser. Draw other lines on the analyser
intensity of transmitted light I. that make 100, 200, 300, etc, relative to the
first line drawn. Angles are measured using
Control of variables: angle at which the a protractor. Rotate the analyser such that
photovoltaic cell faces the analyser must be these other lines coincide with the line on
the same; the same light source must be the polariser to give angles of rotation 100 ,
200 , 300 etc

6
Measurement of light intensity:

The current from the microammeter


is proportional to the light intensity
impinging on the photovoltaic cell.

[Note: An important feature of PV cells is


that the voltage of the cell does not depend
on its size, and remains fairly constant with
changing light intensity. However, the Place the polariser and analyser close
current in a device is almost directly to each other so that the angle can be read
proportional to light intensity and size.] more accurately.

Without light from the laser source, Safety precautions:


record the microammeter reading. This gives
the background current or background light Do not look at laser light directly as
intensity. Switch on the laser light. it can lead to blindness.
Corresponding to each rotation of the
analyser, measure the microammeter current
and subtract the background current from
this reading. The corrected reading will be
proportional to the transmitted light intensity
I.

(d) Precautions to increase accuracy


of experiment: .
Conduct the experiment in a
darkened room to reduce the effect of
ambient light which might not be constant.
Instead of using crude retort stands
to hold the polaroids and then rotate the
analyser, the polaroids could be placed in
holders with proper angle calibrations.

7
Name : ________________________________ CT group : ______________

VICTORIA JUNIOR COLLEGE


2013 JC2 PRELIMINARY EXAMINATIONS

PHYSICS 9646/03
Higher 2 20 Sep 2013
FRIDAY
Paper 3 Longer Structured Questions 2.30 pm 4.30 pm
2 Hours
Candidates answer on the Question Paper.
No Additional Materials are required.

READ THESE INSTRUCTIONS FIRST

Write your name and CT group at the top of this page.


Write in dark blue or black pen on both sides of the paper.
You may use a soft pencil for any diagrams, graphs or rough
working.
Do not use staples, paper clips, highlighters, glue or correction
fluid.

Section A
Answer all questions.

Section B
Answer any two questions. For Examiners Use
You are advised to spend about one hour on each section. 1

At the end of the examination, fasten all your work securely 2


together.
3
The number of marks is given in brackets [ ] at the end of each 4
question or part question.
5

7
8

9
10

Total
This question set consists of a total of 20 printed pages.

1
Data

speed of light in free space, c = 3.00 108 m s-1

permeability of free space, o = 4 10-7 H m-1

permittivity of free space, o = 8.85 10-12 F m-1


(1/(36)) 10-9 F m-1

elementary charge, e = 1.60 10-19 C

the Planck constant, h = 6.63 10-34 J s

unified atomic mass constant, u = 1.66 10-27 kg

rest mass of electron, me = 9.11 10-31 kg

rest mass of proton, mp = 1.67 10-27 kg

molar gas constant, R = 8.31 J mol-1 K-1

the Avogadro constant, NA = 6.02 1023 mol-1

the Boltzmann constant, k = 1.38 10-23 J K-1

gravitational constant, G = 6.67 10-11 N m2 kg-2

acceleration of free fall, g = 9.81 m s-2

2
Formulae

uniformly accelerated motion, s = ut + () at2


v2 = u2 + 2as

work done on/by a gas, W = pV

hydrostatic pressure, p = hg

gravitational potential, GM

r

displacement of particle in s.h.m., x = xo sin t

velocity of particle in s.h.m., v vo cos t


v ( xo2 x 2 )

resistors in series, R = R1 + R2 +

resistors in parallel, 1/R = 1/R1 + 1/R2+

electric potential, V = Q/4or

alternating current/voltage, x = xo sin t

transmission coefficient, T exp(-2kd)


8 2 m(U E )
where k
h2

radioactive decay, x = xo exp(-t)

decay constant, 0.693



t1
2

3
Section A

Answer all the questions in this section.

1. A student was tasked to determine the thickness of a cylindrical glass tube shown in Fig. 1.
He measured the internal and external diameters of the glass tube with a travelling
microscope and recorded his readings as follows:

Internal diameter of glass tube, d = 0.240 mm


External diameter of glass tube, D = 0.365 mm,

Fig. 1
(a) Assuming that his reading of the diameter was subject to an estimated uncertainty of
0.001 mm, calculate the maximum percentage uncertainty in the thickness of the
glass tube. [3]

Maximum percentage uncertainty = %

(b) Express the thickness of the glass together with its associated uncertainty. [2]

Thickness = _________ ___________ m

4
2. A bullet of mass 10 g strikes a stationary block horizontally with a speed of 100 m s-1. The
block rests on a frictionless surface. The time that elapses from the instant the bullet
strikes the block to the instant it just emerges from the other side of the block is 0.030 s.
During this time, the bullets deceleration is 900 m s-2 and the blocks acceleration is
300 m s-2, both with respect to ground.
(a) Calculate the final speed of (i) the bullet and (ii) the block. [2]

Final speed of bullet = .. m s-1


Final speed of block = .. m s-1
(b) Calculate the mass of the block. [2]

Mass = kg
(c) Calculate the loss in kinetic energy of the system consisting of the bullet and block.
[1]

Loss in kinetic energy = . J

5
3. A ball bearing of mass 5.00 g rests on a Hookes spring of elastic constant k = 500 N m-1.
The ball is pushed down 2.0 cm from its equilibrium position and then released as shown
in Fig. 3.1 and Fig. 3.2.

2.0 cm

Fig. 3.1 Fig. 3.2


(a) Calculate the maximum height H to which the ball bearing will rise from its lowest
position . [2]

H = .. cm
(b)
(i) Explain clearly the transformation of energy of the ball bearing when it is
released from the depressed position of the spring until it reaches the
maximum height H. Include discussion of the energy transformation during the
time interval when the ball is rising. [2]

(ii) Sketch the variations of the different energies in (b)(i) as a function of height
starting from the ball bearings lowest depressed position to its highest point.
Label the sketch appropriately. [3]

Energy / J

height risen / m

6
4. A circus performer is riding his motorcycle with uniform speed such that its period is
20.0 s in a horizontal circle of radius 83 m on the inner surface of a cylindrical wall, as
shown in Fig. 4.1.

R
wall
wall
W

Fig. 4.1 Fig. 4.2


The orientation of his motorcycle is shown in Fig. 4.2. Two forces acting on the
motorcycle-man system are the reaction force R acting at an angle with the vertical
and the weight W.

(a) Calculate the angle with the vertical. [3]

= .. o
(b) Sketch a labelled free body diagram of the motorcycle-man system when = 90o.
Hence explain why it is impossible for the system to achieve rotational equilibrium in
this position. [3]

7
5. The p-V diagram in Fig. 5 shows a fixed mass of an ideal gas expanding isothermally
from state A to B.
p / Pa
A
p1

p2 B

3
V1 V2 V / cm

Fig. 5

(a) On Fig. 5, sketch a graph for an adiabatic expansion of the same gas from V1 to V2
from state A. Label the final state as C. [1]

(b) State the First Law of Thermodynamics, and use it to explain why the final pressure at
state C is different from the pressure at state B. [4]

..

..

..

..

..

..

..

..

..

..

..

8
6.
(a) A resistance wire 80.0 cm long has a uniform cross sectional area of 2.5 10-8 m2.
Given that its resistivity is 1.25 10-7 m, calculate the resistance across its ends.
[2]

Resistance =

(b)
(i)

5.0 V 2.0

P
X Y
jockey
3.0 V 1.0

S
A Fig. 6
1.0

The resistance wire XY in (a) is used to set up the circuit as shown in Fig. 6. With
the switch S open, the jockey is moved along the resistance wire until the
galvanometer indicates a null reading. Calculate the balance length XP. [2]

Balance length = .. m

9
(ii) Switch S is now closed and the jockey is disconnected from the wire XY.
Calculate the current in the ammeter. [1]

Current = A

(iii) Hence, or otherwise, calculate the new balance length when the jockey is
connected back to wire XY. [1]

New balance length = . m

10
7. The radioactive nuclide is used in radiotherapy. It has a half-life of 5.27 years
and, at each disintegration, two -rays are emitted, one of energy 1.17 MeV and the other
of energy 1.33 MeV.
(a) This nuclide is prepared by bombarding a suitable target (a stable nuclide) with an
appropriate particle. Possible projectiles are neutrons ( ) and deuterons ( ).
Write down the equations for three separate reactions, involving the target nuclei
, and respectively, and one or the other of the projectiles and ,
by which might be produced. [3]

(b) In a radiotherapy treatment, it is necessary to determine the amount of energy


absorbed from the radiation. A fresh 1.0 g sample of , which may be treated as
a point source, is placed at a position 1.50 m from a patient. Calculate the intensity
of the radiation received by the patient. [3]

Intensity = . W m-2

11
Section B

Answer two questions from this section.

8. A small stone is dropped into a pool of calm water and this sets up ripples as shown in
Fig. 8.1.

A B

Fig. 8.1

A progressive wave is set up from the centre A and it moves outwards passing point B,
which is at 0.90 m from A. A graph showing how the displacement y at A varies with
time t is shown in Fig. 8.2. Another graph, Fig. 8.3, shows how the displacement of the
wave at time t = 0 varies with distance x from point A.

y / cm

t/s

Fig. 8.2

y / cm

A
B x/m

Fig. 8.3

(a) Calculate the speed of the ripple wave. [2]

Speed = m s-1

12
(b) Write down the equation representing the oscillatory motion of point A. [2]

(c) Calculate the phase difference between points A and B. [2]

Phase difference = .. rad

(d) Given that there is no energy loss from the wave travelling from A to B, explain the
wave profile shown in Fig. 8.3. [2]

(e) After an initial vertical displacement, an object floating on the water will bob up and
1 0.10
down with a frequency f given by the expression f , where m is its
2 m
mass in kg. A small seed of mass 2.5 g and capable of floating drops into a pool of
calm water.
(i) Calculate the frequency of the seeds oscillation if it is dropped into the pool of
calm water. [2]

Frequency = Hz

13
(ii) The waves due to the stone in (a) now pass through the region of water where the
seed is floating. Describe and explain how the seed would oscillate, paying
attention to its frequency response. [3]

(iii)
1. Describe and explain how the oscillation of the seed will change with time, if
it is taking in water. [3]

2. Calculate the energy of the seed at the time when it is oscillating with a
maximum amplitude of 1.8 cm. [4]

Energy = J

14
9.

A Bo
v Bo
A
P
Q D
l
2

2 D B
v
C
l

Plan view Isometric view

Fig. 9.1

In the zero-gravity environment in a space station orbiting the Earth, a square


aluminium structure ABCD of side length l is given an initial torque that causes it to
undergo rotational motion about its vertical axis, as shown in Fig. 9.1. The structure has
a break PQ on its top side AD. A horizontal uniform magnetic field of uniform flux
density Bo is then directed across the structure.

(a) Given that the structures angular velocity is , write down an expression for the
linear velocity v of each of its vertical sides. [2]

..
(b) Derive an expression for the e.m.f. induced along each of its vertical sides in terms
of l, , Bo and , where is the angle between the structures normal and the
magnetic field at the instant shown in Fig. 9.1. [2]

(c) State the direction of the e.m.f. induced in side AB at the instant shown in Fig. 9.1.
[1]

..
(d) State which end, P or Q, of the structure across the break, would be at the higher
potential at the instant shown in Fig. 9.1. [1]

..

15
(e) If the angle = 0o at time t = 0, derive an expression for the maximum e.m.f.
induced in the structure. Sketch a labelled graph to show the variation of the induced
e.m.f. in the structure with time. [3]

The break in side AD is now mended such that the structure forms a complete circuit.

(f) Derive an expression for the current I in the structure at the instant shown in
Fig. 9.1, given that the electrical resistance of the structure is R. Give your answer in
terms of Bo, l, and R. [1]

(g) Draw a copy of the plan view of Fig. 9.1 in the space below. On your copy, draw
arrows to show the directions of the forces F acting on sides AB and CD as a result
of the induced current in them. [1]

(h) Derive an expression for the torque on the structure as a result of the forces on its
vertical sides, in terms of Bo, l, , R and . [3]

(i) State the effect of the torque on the motion of the structure. [1]

..

16
(j) On the axes below, sketch a new graph to show the variation of the induced e.m.f. in
the mended structure with time, assuming that angle = 0o at time t = 0. Explain the
shape of your graph. [3]

..

..

..

..

(k) State the energy transformations in the structure over the entire course of its motion.
[2]

..

..

..

..

17
10.

Fig. 10.1

Consider an electron in a one-dimensional box with infinitely high walls. Treat the
electron as a sinusoidal wave. Fig. 10.1 shows possible ways of fitting the wave into
the box of width L.

(a)
(i) Explain why the amplitude of the wave motion must vanish at the walls of the
box. [1]

(ii) Let there be n half-wavelengths in the box, where n = 1, 2, 3, 4 Write


down an expression relating n, and L, where is the wavelength. [1]

18
(iii) Assuming that there is no potential energy, show that the energy of the electron
n2h2
is given by E n where h and m are the Planck constant and mass of the
8mL2
electron respectively. [3]

(iv) The longest-wavelength photon that the electron in the ground state in the box
can absorb is of wavelength 1.0 10-7 m. Calculate a value for the width L of
the box. [4]

L = . m

(b) Electrons of de Broglie wavelength = 8.0 10-12 m hit a gold foil and cause X-rays
to be emitted. A plot of the intensity of emitted X-rays versus their wavelength yields
a smooth curve starting at a minimum wavelength and exhibiting some sharp peaks.

(i) Explain the origin of the smooth curve. [3]

19
(ii) Calculate the minimum wavelength of emitted X-rays. [2]

Minimum wavelength = m

(iii) With increasing wavelengths above this minimum wavelength, a first group of
sharp peaks is seen in the plot of X-ray intensity versus wavelength. State and
explain whether it corresponds to the K series or the L series. [2]

...

..

..

..

(iv) The characteristic X-rays in (iii) are due to incoming electrons kicking out
originally bound electrons from one of the innermost orbits of the gold atom.
Suggest what might be the effect/s of incoming electrons striking the electrons
from one of the outermost orbits of the atom. [2]

..

..

..

..

(v) Explain why the minimum wavelength in (ii) is independent of the nature of the
target material while characteristic wavelengths are dependent on it. [2]

..

..

..

..

..

..

20
VICTORIA JUNIOR COLLEGE 2(b) By conservation of momentum
mau = mbvb + mava where ma and mb are the
SUGGESTED SOLUTIONS TO 2013 mass of the bullet and block respectively.
PHYSICS H2P3 PRELIM EXAMS

1(a)
10(100) = 9mb + 10(73)
mb = 30 g = 0.030 kg

2(c) Loss in KE = Initial KE of bullet


Final KE of bullet and block
= (1/2)(0.010)(100)2 (1/2)(0.030)(9.0)2
(1/2)(0.010)(73)2
= 22 J
t t
d
D 3(a) From conservation of energy, the
elastic energy stored in the spring will be
D d 0.365 0.240 transformed into gravitational potential
t
2 2 energy. Hence:
= 0.0625 mm kx2 = mgH

H = kx2/(mg)
Dd 1 = (500)(0.0202)/(0.005 x 9.81)
t t D d
2 2 = 2.04 m = 204 cm
t = 0.001 mm

t 3(b) (i) As the spring recovers from its


Percentage uncertainty = 100% 2 cm depression, the stored elastic
t
potential energy will be transformed to
0.001 kinetic energy as well as gravitational
100%
0.0625 potential energy.

=1.6 %
The ball bearings kinetic energy will
finally be all transformed to gravitational
1(b) t = ( 0.063 0.001) mm potential energy when the ball bearing
reaches its maximum height H.
= (6.3 0.1) x10-5 m

2(a)(i) Using v = u +at


va = 100 900(0.030) (for bullet)
= 73 m s-1

2(a)(ii) vb = 300(0.030) (for block)


= 9.0 m s-1

1
Energy /J about the centre of mass is unbalanced,
and will cause him to topple.
0.10
epe
gpe
5(a)(i)
ke p/ Pa
A
p1
height
risen /m p2 B
C
0.020 2.04
V1 V2 V/ cm3
Lowest point
5(a)(ii) The First Law of Thermodynamics
states that the increase in internal energy
of the system is equal to the sum of the
4.(a) R can be resolved into its vertical and heat supplied to the system and the work
horizontal components Ry and Rx done on it i.e. U Q W .
respectively. Ry = W = mg for vertical
equilibrium.
For an isothermal expansion, U = 0 since
the temperature is kept constant.
Rx provides the centripetal force and is
equal to mr(2/T)2
For an adiabatic expansion, Q = 0 and
tan = Rx/Ry = mr(2/T)2/mg
W is negative. Hence, there is a decrease
= (83)(2/20.0)2/9.81
in internal energy of the gas.
= 39.9 0

4(b) As the internal energy of the gas decreases,


temperature also decreases.
f Using pV = nRT, at the same volume, since
wall Tc < TB, PC < PB.
N
W

L
f is the frictional force, N is the normal Q6(a) R
A
force of wall on motorcycle, W is the
weight of the motorcycle and man. 1.25 10-7 0.80
4.0
2.5 108

If the motorcyclist is oriented 4.0


6(b)(i) P.D. across XY = 5.0 V
perpendicular to the wall surface, the 4.0 2.0
anticlockwise moment due to friction = 3.33 V

2
3.0 Section B
Balance length = 0.80 = 0.721 m
3.33
8(a) The speed of the wave is given by

v f
3.0 T
6(b)(ii)V = IR, or I = 1.5 A
2.0
0.4
0.32 m s -1
1.25
6(b)(iii) P.D across the XP is now
1.5 x 1.0 = 1.5 V
1.5 8(b) The displacement time relation for
New balance length = 80 36.0 cm 2
3.33 point A is y A 0.020sin( t)
= 0.360 m 1.25
Or y A 0.020sin(5.03t )

Q7(a) The possible reactions include:


63
29Cu01n27
60
Co24He 8(c) A and B are separated by distance =
62
Ni 12H 27
60
Co 24He 2.25 . The phase difference is equivalent
28
x
59
Co 01n 27
60
Co to a separation of 0.25. x 2
27

0.25
x 2 rad 1.57 rad
2
Q7(b) The activity of the 1.00 g sample,
A N
ln 2 1 8(d) As the wave spreads out from a point
x x 6.02x10 23 s -1
(5.27 x365x 24x3600) 60 source, the total energy of the wave will be
4.18x1013 s -1 distributed to and shared by the spreading
ripples.
Power of radiation from source is P = AE
= 4.18 x 1013 (1.17+1.33)(1.6x10-13) W This results in a reduced amplitude in the
wave away from the source.
Hence the intensity of radiation at 1.50 m
is 8(e)(i) The frequency of the oscillation of
P 1 0.10
I the seed =
4r 2 2 2.5x1 0 3

= 1.0 Hz
13 13
4.18 x10 x 2.50x1.6x10

4 (1.50) 2
8(e)(ii) The frequency of the ripples is f
= 0.591 W m-2 =1/T = 1/1.25 = 0.80 Hz. The seed will
be set into forced oscillation at the
frequency of the ripples, which is 0.80 Hz.

3
As the natural frequency of the seed is 9(b) Induced e.m.f. in vertical side is
higher at 1.00 Hz, its amplitude of E = B0lvsin
oscillation is small.

This is because there is no resonance. ( )

8(e)(iii)1. The mass of the seed will


increase causing its natural oscillation
9(c) Vertically downwards.
frequency to decrease.

At a certain critical mass when its natural


9(d) Q will be at a higher potential than P.
frequency has dropped to close to the
ripple frequency of 0.80 Hz, it will be
driven into resonance with increased
amplitude of oscillation. 9(e) When t = 0, = 0o

But as its mass continues to increase with


more water taken in, its forced oscillatory When t = T / 4, = 90o
amplitude will decrease due to its much
smaller natural frequency.
This is the e.m.f. induced in each of the
two vertical sides.
8(e)(iii)2. When the seed is oscillating Hence, the total e.m.f. induced in the
with maximum amplitude, its frequency of structure would be equal to
oscillation will be 0.80 Hz (resonance).

Its soaked mass is now given by:


1 0.10
0.80 m 3.95x10 3 kg .
2 m Hence, the induced e.m.f. at time t is given
by:
The total energy of the seeds oscillatory
motion,
1
E m 2 A 2
2 E

1 2 2
(3.95x10 3 )( ) (1.8x10 2 ) 2
2 1.25
1.62x105 J t
0
T
9(a)

4
9(k) The rotational mechanical energy of
() the ring is converted to electrical energy

and finally into heat energy which is


9(g) v B dissipated to the surroundings.

I
A
10(a)(i) The square of the amplitude of the
wave-function is a measure of the
F probability of locating a particle in a small
F
region of space. As there is no probability
of locating an electron beyond the
infinitely high walls of the box, the
I
amplitude of the wave-function must
D
vanish at the walls.
v

9(h) Torque on ring


10(a)(ii) n L .(1)
= ( ) 2

( ) 10(a)(iii) The kinetic energy of the


p2
electron is given by E
2m

( )
h2 h
E ..(2) (since p )
( ) 2m 2

2L
In (1), . Substituting into (2), we
n
9(i) The torque has the effect of retarding
the rotational motion of the ring. h2 n2 n2 h2
have E
2m( 4 L2 ) 8mL2

9(j) Due to the retarding torque, the


rotational speed of the ring will slow down 10(a)(iv) The longest wavelength of a
with time. Consequently, the induced photon that can be absorbed corresponds to
e.m.f. will also be reduced with time. a transition from n = 1 to n =2 (smallest
energy difference)

h2 hc
Induced e.m.f., E (2 2 12 )
8mL 2
longest

t
0

5
3hlongest 10(b)(iv) One possibility is the electron
L gets ejected from the atom, changing it
8mc into an ion.
3(6.67 x10 34 )(1.0 x10 7 )
If the outer electron is excited but not
8(9.11x10 31 )(3.0x10 8 )
ejected from the atom, it may de-excite but
produce a photon of longer wavelength
3.0 x 10-10 m compared to the K or L series lines.

10(b)(i) When an electron comes close to a


10(b)(v) The minimum X-ray wavelength
positive nucleus, it will veer in its path and
originates from an incoming electron
lose a certain amount of kinetic energy.
losing all its energy in its interaction with
a target nucleus. The wavelength thus
This loss of KE manifests itself as a
depends only its initial KE and therefore
photon of X-ray that is emitted.
only the tube voltage and is independent of
the nature of the target nucleus.
Large numbers of bombarding electrons
lose different amounts of KE in their
Characteristic X-rays are formed from
interactions with nuclei. This leads to a
electron transitions within atoms. Different
large number of wavelengths being
characteristic wavelengths are due to
emitted, resulting in the continuous
different energy levels within atoms.
spectrum.
Hence characteristic wavelengths depend
on the nature of the target atom.
10(b)(ii) From the Duane-Hunt rule, the
entire KE of the bombarding electron gets
******** END ********
converted into the energy of an X-ray
photon.
p2 h2 hc
K
2m 2m 2
min

where is the de Broglie wavelength.


2mc2
min
h
2(9.11x10 31 )(3.0x10 8 )(8.0x10 12 ) 2

6.63x10 34
-11
5.27 x 10 m

10(b)(iii) The first group of sharp peaks


corresponds to the K-series lines.

The energy differences for the formation


of K lines are bigger than those for L-lines,
resulting in shorter wavelengths for the
former.

6
TAMPINES JUNIOR COLLEGE
PRELIMINARY EXAMINATIONS

CANDIDATE
NAME

CIVICS TUTOR
GROUP 1 2 NAME

PHYSICS Tuesday, 24 September 2013


Paper 1 Multiple Choice 1 hour
Additional Material: Multiple Choice Answer Sheet

READ THESE INSTRUCTIONS FIRST

Write in soft pencil.


Do not use staples, paper clips, highlighters, glue or correction fluid.
Write your name and class on the Answer Sheet in the spaces provided.

There are thirty questions on this paper. Answer all questions. For each question there are
four options A, B, C and D.
Choose the one you consider correct and record your choice in soft pencil on the separate
Answer Sheet.

Read the instructions on the Answer Sheet very carefully.

Each correct answer will score one mark. A mark will not be deducted for a wrong answer.

This document consists of 13 printed pages.


2
1 A voltmeter connected across a resistor in a circuit gives readings which have high
precision but low accuracy.

Which of the following best describes the likely error in readings taken with this
voltmeter?

Random Error Systematic Error


A large large
B large small
C small large
D small small

2 Vectors P and Q represent two forces.

In which vector diagram does the vector R represent the difference between P and Q,
such that R = P - Q?

3 A car is travelling at a speed of 15 m s-1 and can be brought to rest with a uniform
retardation in 1.2 s if the brakes are applied. The reaction time of the driver is 0.10 s.
What is the minimum distance of the moving car from a stationary object if a collision is
to be avoided?

A 9.00 m B 9.75 m C 10.5 m D 18.0 m

3
4 A stone is thrown horizontally from the edge of a cliff and falls towards the sea below.
The initial speed of the stone is v.

How far does the stone travel horizontally and vertically in a time interval t after the
stone is thrown?

Neglect air resistance.

horizontally vertically
A 0.5vt 0.5gt2
B 0.5vt gt2
C vt 0.5gt2
D vt gt2

5 Car X travels at a constant speed V along a motorway. At a time t=0, it passes car Y
which immediately accelerates uniformly from rest to speed 2V. Car Y then decelerates
uniformly to rest.
(Time 0Q = time QS.)

Which statement is correct?

A At time Q, car Y overtakes car X.


B At time R, car X overtakes car Y.
C At time Q, car Y has travelled twice as far as car X.
D At time S, car X has travelled twice as far as car Y.

6 The diagram below shows a uniform rod freely pivoted at P. The rod is suspended
horizontally, when a string tied to one end of the rod passes over a smooth pulley and is
attached to a 3.0 kg mass. Determine the mass of the rod.

A 4.8 kg B 3.5 kg C 2.4 kg D 1.7 kg

4
7 Paraffin floats on water. Some paraffin is poured into one arm of a U-tube containing
water.

The diagram below shows a difference of 25 mm in the surface levels of the water and
paraffin in the two arms. The length of the paraffin column in the tube is 125 mm.

A small volume of paraffin is added to the right-hand arm increasing the length of the
paraffin column in the tube to 150 mm.

What is the new difference between the surface levels of the water and paraffin in the
two arms?

A 25 mm B 30 mm C 38 mm D 50 mm

8 The extension x of a particular spring is related to the stretching force as shown in the
graph.

When the extension of the spring is e, the elastic potential energy stored in the spring is
E. What is the increase in the elastic potential energy when the extension is increased
from e to 2e?

A E B 2E C 3E D 4E

5
9 The forces acting on the Earth and a part of the Antarctic ice shelf, which has its centre
of gravity directly above the Earths axis of rotation, are shown. All of the forces are
equal in magnitude.

Which pair of forces are equal because of Newtons 1st Law and which pair are equal
because of Newtons 3rd Law?

1st Law 3rd Law


A F1 and F3 F1 and F2
B F1 and F3 F2 and F4
C F3 and F4 F1 and F2
D F3 and F4 F2 and F4

10 The graph shows how the force acting on a body varies with time.

Assuming that the body is moving in a straight line, by how much does its momentum
change?

A 40 kg ms-1
B 36 kg ms-1
C 20 kg ms-1
D 16 kg ms-1

6
11 A large mass moving at a velocity of 5 m s-1 collides head-on with a small mass moving
at a velocity of 2 m s-1 in the opposite direction.

The collistion is elastic. After the collision, both masses move to the right. The large
mass has a velocity v1 and the small mass has a velocity v2.

Which pair of values v1 and v2 is possible?

V1 V2
A 2 m s-1 5 m s-1
B 3 m s-1 10 m s-1
C 4 m s-1 4 m s-1
D 5 m s-1 12 m s-1

12 A neutron moving with an initial velocity u has a head-on elastic collision with a
stationary proton. After the collision, the velocity of the neutron is v and that of the
proton is w. Taking the masses of the neutron and proton to be equal, which one of the
following statements is wrong?

A Conservation of momentum shows that u = v + w.


B Conservation of energy shows that u2 = v2 + w2
C The momentum and energy equations taken together imply that the speed of the
proton after the collision is the same as that of the neutron before the collision.
D The fact that the collision is elastic implies that the proton and neutron move off
in opposite directions with equal speeds.

13 The gravitational potential energy Ep of a mass near the Earths surface varies with
height h as shown.

What mass is being lifted?

A 1.6 kg B 2.4 kg C 8.2 kg D 16 kg

7
14 An electric motor is required to haul a cage of mass 400 kg up a mine shaft through a
vertical height of 1200 m in 2.0 minutes. 20% of the total power output of the motor is
used in overcoming friction and 80% is used to raising the cage.

What is the total power output of the motor?

A 4.0 kW B 5.0 kW C 39 kW D 49 kW

15 Every second, 6.0 x 1017 electrons and 3.0 x 1017 protons move across a cross-section
of a discharge tube filled with ionised hydrogen gas. The electrons and the protons
move in opposite directions. What is the current flowing through the discharge tube?

A 0.048 A B 0.096 A C 0.14 A D 0.19 A

16 A cell with e.m.f. E and internal resistance r, is connected to a variable resistor R as


shown in Figure (a). Figure (b) shows the variation of the voltmeter reading V with the
ammeter reading A as the resistance of R is varied. Assuming that the voltmeter and
ammeter are ideal, what is the internal resistance r?

A 1.1 B 2.9 C 4.0 D 5.1

17 The wiring in a house must be thick enough to ensure that a fire will not start as a result
of the heating effect produced by current flowing through the wiring. A 2.0 m length of
copper wire must carry a maximum current of 35 A and it is to produce no more than 1.8
W of heat energy per second per metre. What is the minimum diameter of the copper
wire? (resistivity of copper = 1.68 x 10-8 m)

A 1.9 mm B 2.7 mm C 3.8 mm D 5.4 mm

8
18 A battery is connected to 4 resistors as shown in the following circuit diagram. What is
the potential difference across A and B?

A 0V B 3.5 V C 4.5 V D 7.5 V

19 A circuit is needed which switches on a warning lamp when the temperature of a


thermistor is too high.

Which circuit is suitable?

9
20 A wire carrying a current I is placed in a magnetic field of flux density B.

How is the magnitude F of the force acting on the wire related to the angle that it
makes with the field?

A F

B F sin

C F cos

D F 1/sin

21 A positively-charged particle enters a uniform magnetic field.

Which diagram represents the path of the particle in the magnetic field?

10
22 A transverse wave moves to the right.

X and Y are points in the path of the waves.

Which statement about the oscillations at X and at Y is correct?

A The oscillations at X and Y are in phase.


B The frequency of oscillations at X is greater than the frequency of oscillations at
Y.
C The oscillations at X and Y have a phase difference of rad.
D The oscillations at X and Y have the same period.

23 The diagram shows a beam of initially unpolarised light passing through two Polaroid
filters.

The transmitting axes of these filters are initially aligned. The two filters are now rotated
through 360o in opposite directions in their own plane at equal speeds.

How many maxima of intensity occur in the light emerging from the right-hand Polaroid?

A 2 B 3 C 5 D 9

24 A point source of sound emits energy equally in all directions at a constant rate and a
person 8 m from the source listens. After a while, the intensity of the source is doubled.
If the person wishes the sound to seem as loud as before, how far should he be
standing now?

A 2m B 2 2 m C 8 2 m D 2 8 m

25 A stationary sound wave is set up in a laboratory between parallel walls of separation


4.0 m, using a sound generator producing waves of frequency 400 Hz. The speed of
sound is 320 m s-1.

How many antinodes will be present in this stationary wave?

A 5 B 6 C 10 D 11

11
26 A two-source interference experiment is set up as shown.

The source emits light of wavelength 600 nm. The interference pattern on the screen is
shown below.

What is the distance x?

A 3.8 x 10-4 m B 1.9 x 10-3 m C 3.8 x 10-3 m D 1.9 x 10-2 m

27 By blowing across the mouth of a glass bottle, the air column in the bottle can be made
to vibrate and produce a tone. In order to tune the bottle and use it as a musical
instrument, the fundamental frequency of the bottle must be 440 Hz. The bottle is 0.450
m tall and the speed of sound is 340 m s1. How high should the bottle be filled with
water?

A 0.064 m B 0.193 m C 0.257 m D 0.386 m

28 A clean zinc surface is illuminated with monochromatic radiation.

Which condition is necessary for the emission of photoelectrons?

A The frequency of the radiation exceeds a critical value.


B The intensity of the radiation exceeds a critical value.
C The wavelength of the radiation exceeds a critical value.
D The zinc surface is negatively charged.

12
29 Transitions between three energy levels in a particular atom give rise to three spectral
lines. The shortest and the longest wavelengths of these spectral lines are 1 and 2
respectively. The wavelength of the other spectral line is

2 1 12 12
A 2 - 1 B C D
2 1 2 2 1

30 An electron with kinetic energy E has a de Broglie wavelength of . Which of the following
graphs correctly represents the relationship between and E?

-The End-

13
TAMPINES JUNIOR COLLEGE
PRELIMINARY EXAMINATIONS

CANDIDATE
NAME

CIVICS TUTOR
GROUP 1 2 NAME

PHYSICS Tuesday, 3 September 2013


Paper 2 Structured Questions 2 hours

READ THESE INSTRUCTIONS FIRST

Candidates answer on the Question Paper.


Write in dark blue or black pen.
You may use a soft pencil for any diagrams, graphs or rough
working.
Do not use paper clips, highlighters, glue or correction fluid.

For Examiners Use


Section A
Answer all questions. 1 9

2 7
Section B
3 9
Answer any two questions. Circle the questions you have chosen
to do clearly in the table to your right.
4 8

You are advised to spend about one hour on each section. 5 7

6 20
The number of marks is given in brackets [ ] at the end of each
question or part question. 7 20

8 20
The number of marks is given in brackets [ ] at the end of each
question or part question. Total / 80

This document consists of 17 printed pages.

1
2
Section A
Answer all the questions in this section.
1 A boy takes a ride in a hot air balloon. When it is at a height of 90 m above the ground and
rising vertically at a speed of 1.5 m s-1, the boy drops a ball of mass 0.30 kg.

Ignoring air resistance,


(a) state the velocity of the ball just after the boy releases the ball,

velocity = m s1 [1]
(b) show that the time it takes for the ball to reach the ground is 4.4 s.

time = s [1]
(c) determine the velocity of the ball just before it hits the ground.

velocity = m s1 [2]
Air resistance is actually not negligible and is proportional to velocity with a proportionality
constant of 8.9 x 10-2 N s m-1. The ball reaches terminal velocity a short while before hitting
the ground.
(d) determine the terminal velocity of the ball.

terminal velocity = m s1 [1]

3
(e) On the same axes, sketch how the velocity of the ball varies with time from the instant
the boy releases the ball, up to the instant when it reaches the ground:
(i) for the case without air resistance. Label this graph S, and label appropriate values
on the axes.
(ii) for the case with air resistance. Label this graph P. [4]

4
2 (a) State two conditions necessary for the equilibrium of an extended object.

...............................................................................................................................................

........................................................................................................................................... [2]
(b) The figure below shows a set of traffic lights held in equilibrium by two supporting
cables A and B.

(i) Sketch a labelled vector triangle of the forces on the traffic lights.

[2]
(ii) Determine the ratio T1/T2

ratio = [3]
5
3 An explanation of the photoelectric effect includes the terms photon energy and work
function.

(a) Explain what is meant by

(i) a photon,

...............................................................................................................................................

........................................................................................................................................... [1]

(ii) work function.

...............................................................................................................................................

........................................................................................................................................... [1]
(b) In an experiment to investigate the photoelectric effect, a student measures the
wavelength of the light incident on a metal surface and the maximum kinetic energy
Emax of the emitted electrons. The variation with Emax of 1/ is shown in Fig. 3.1 below.

Fig 3.1

(i) The work function of the metal surface is

State an equation, in terms of , and Emax, to represent conservation of energy


for the photoelectric effect. Identify any other symbols you use.

.................................................................................................................................
.................................................................................................................................
..............................................................................................................................[2]

6
(ii) Use only your answer in (i) and Fig. 4.1 (without using the value of Planck
constant), determine

1. the work function of the metal surface,

= ............................................. J [2]
2. a value for the Planck constant.

Planck constant = ........................................... J s [3]

7
4 Fig. 4.1 shows two microwave transmitters A and B 0.20 m apart. The transmitters emit
microwaves of equal amplitude in phase and of wavelength 30 mm. A detector, moved
along the line PQ at a distance of 5.0 m from AB, detects regions of high and low
intensity forming an interference pattern.
P

5.0 m
0.20 m

B Fig. 4.1

(a) Calculate the separation between one region of high intensity and the next along the
line PQ.

separation = m [2]
(b) State the effect, if any, on the position and intensity of the maxima when each of the
following changes is made, separately, to the experiment.
1. The amplitude of the transmitted waves is doubled.

...............................................................................................................................................

...............................................................................................................................................

........................................................................................................................................... [2]
2. The separation between the transmitters is halved.

...............................................................................................................................................

...............................................................................................................................................

........................................................................................................................................... [2]
3. The phase of transmitter A is reversed so that there is now a phase difference of
180 between the waves from A and B.

...............................................................................................................................................

...............................................................................................................................................

........................................................................................................................................... [2]
8
5 A coil of 1500 turns of insulated wire is tightly wound on a non-magnetic tube to make a
solenoid of mean radius 22 mm, as shown in the figure below. The wire itself has radius
0.86 mm and is made of a material of resistivity 1.7 x 10-8 m. The coil is connected to a
supply of e.m.f. and negligible internal resistance.

Calculate
(a) the total length of wire in the coil,

length = m [2]

(b) the total resistance of the coil,

resistance= [3]

(c) the current in the coil

current = A [2]
9
Section B
Answer any two questions in this section.
6 The following data concern a tennis ball at a given instant just before it is struck by a tennis
racket:

horizontal momentum of tennis ball = 2.4 N s,


kinetic energy of tennis ball = 45 J.

(a) Why is it correct to give direction of the momentum but not of the kinetic energy?

...............................................................................................................................................

........................................................................................................................................... [1]
(b) Write down in terms of the mass m and the velocity v of a body, expressions for
(i) the momentum,
(ii) the kinetic energy.

momentum = [1]

kinetic energy = [1]


(c) Use your answer in (b) to help you to calculate the mass and velocity of the tennis ball.

mass = kg [1]

velocity = m s-1 [1]


(d) When the racket hits the ball it exerts a average force of 200 N in a direction opposite to
its momentum, bringing the ball to rest momentarily. Calculate
(i) the time the tennis ball takes to stop,
(ii) the distance the tennis ball travels before stopping.

time = s [2]

distance = m [2]
10
(e) A different average force of 160 N then continue to act on the tennis ball for a further
0.025 s. Calculate
(i) the new momentum of the ball,
(ii) the new velocity of the ball.

momentum = kg m s-1 [2]

velocity = m s-1 [1]

(f) Calculate the increase in kinetic energy of the ball for the whole time that the force is
applied to it and hence deduce the mean power being delivered to the ball while it is in
contact with the racket.

mean power = W [3]

(g) Using Hookes Law, suggest why, in practice, it is impossible for a constant force to be
applied to the ball to decelerate it to zero velocity.

...............................................................................................................................................

........................................................................................................................................... [2]

(h) As the tennis ball travels to the other end of the court, it hits and bounces off the rough
ground. The arrow below gives the direction of the velocity of the ball just before contact.
At the point where the ball is in contact with the ground, draw a free body diagram to
show the forces acting on the ball, together with the action reaction pair for each force
on the ball that is drawn. Label all forces carefully.

[3]

11
7 Fig 8.1 shows a person of mass 65 kg doing a bungee jump.

Fig 8.1
The bungee rope has negligible mass and has an un-stretched length of 60 m. This bungee
rope may be assumed to obey Hookes law. A force of 140 N extends the bungee rope by
2.5 m. The bungee rope is secured to the feet of the person. The person leans over the
bridge and drops vertically downwards towards the river below. The maximum length of the
stretched bungee rope is 110 m. You may assume that air resistance is negligible.

(a) State Hookes law.

...............................................................................................................................................

........................................................................................................................................... [1]

(b) The momentum of the person increases during free fall. Explain whether or not this is a
violation of the principle of conservation of momentum.

...............................................................................................................................................

...............................................................................................................................................

........................................................................................................................................... [2]

(c) (i) Describe the energy changes taking place from the instant the person leaves the
bridge until the bungee rope is fully extended.

...............................................................................................................................................

...............................................................................................................................................

...............................................................................................................................................

........................................................................................................................................... [3]

12
(ii) The unit of energy is joule. Express the joule in terms of SI base units.

units = [1]
(d) Calculate the speed of the person when he has fallen 60 m below the bridge, just before
the bungee rope starts to stretch.

speed = m s-1 [1]

(e) (i) Sketch a graph of the tension F in the bungee rope against its extension e.

F/N

e/m
[1]
(ii) Show that the elastic potential energy in the bungee rope is directly proportional to
the square of its extension.

[2]
(iii) Show that the elastic potential energy in the fully extended bungee rope is about
70kJ.

[2]

13
(iv) Determine the maximum tension in the bungee rope when it is fully stretched.

Tension = N [2]

(v) Determine the magnitude of the acceleration of the person when the bungee rope is
fully stretched.

acceleration = m s-2 [2]

(vi) State the direction of the acceleration of the person when the bungee rope is fully
stretched.

direction = [1]

(f) Explain why it would be extremely dangerous to have a bungee rope that is
much stiffer.

...............................................................................................................................................

........................................................................................................................................... [2]

14
8 A 0.59 m length of zinc wire, with a cross-sectional area of 1 mm2, is shaped into a wire coil
with two turns. The wire coil is suspended in mid-air as shown below. The wire coil and a
component X are connected in series to a DC voltage source of e.m.f. 0.045 V and there is a
current I of 0.40 A flowing through the wire coil. (resistivity of zinc = 59 x 10-9 m )

(a) (i) Define tesla.

...............................................................................................................................................

........................................................................................................................................... [2]

(ii) Sketch the magnetic field produced by the wire coil.

[2]

15
(iii) The magnetic flux density at the centre of a flat circular coil can be computed using
the following formula.

NI
B 0
2r
where 0 4 10 - 7 H m -1
N number of turns
I current flowing through coil
r radius of coil (in metres)

Determine the magnetic flux density at the centre of the wire coil.

magnetic flux density = T [2]


(iv) The wire at segment A exerts a force on the wire at segment B. Explain the origin of
this force.

...............................................................................................................................................

........................................................................................................................................... [2]

(v) The magnetic flux density of a long straight wire can be computed using the following
formula.

0 I
B
2d
where 0 4 10 -7 H m -1
I current flowing through wire
d distance from wire (in metres)

Show that the force referred to in part (iv), is directly proportional to the square of the
current flowing through the wire coil.

[2]
(b) (i) Define potential difference.

...............................................................................................................................................

........................................................................................................................................... [2]
16
(ii) Determine the resistance of the wire coil.

resistance = [2]
(iii) The variation of current through component X with potential difference across
component X is shown in the following graph.

Determine the internal resistance of the DC voltage source.

Internal resistance = [3]


(iv) Determine what percentage of total power supplied is dissipated by the internal
resistance.

percentage = % [3]

The End

17
Mark Scheme

Solutions
1

-1
(a) 1.5 m s upwards. [1]

2
(b) s = ut + at
2
90 = -1.5t + (9.81)t [A1]
2
(9.81)t - 1.5t - 90 = 0

t = 4.4 s

(c) v = u + at

v = -1.5 + 9.81 x 4.4 [C1]


-1
= 41.7 m s [A1]

-2
(d) 0.30 x 9.81 = 8.9 x 10 xv where v is terminal velocity
-1
v = 33 m s [A1]

(e)

S - line [M1]; values [A1]

P- shape of curve [M1]


18
-1
v drops to zero earlier than S; terminal velocity less than 41.7 m s and graph ends beyond 4.4s
[A1]

2(a) State two conditions necessary for the equilibrium of an extended object.

There is no net/resultant force acting on the object.

There is no net/resultant torque about any pivot acting on the object.

[2]

(b) The figure below shows a set of traffic lights held in equilibrium by two supporting cables A and B.

[2]

labeling, Direction, angle must all be


correct. (minus 1 for one wrong)

T2 o
50
W=350N
o
T1 30

(ii) Determine the ratio T1/T2 [3]

o o
resolving: T1sin 30 = T2sin 50 [M1]

Ratio =1.53 [A1] Allow for ecf in T1/T2

3a(i)A photon is a quantum of energy in electromagnetic radiation. [1]

19
(ii) Work function is the minimum energy required for an electron to escape from the surface of a
metal.[1]

b(i) hc/ = + Emax [M1]

h= planck constant

c= speed of light [B1 if both correct]

(ii) when 1/ = 0, = -Emax

working - graph is extended to read the x intercept.[M1]

-19 -19
= 4.0 10 J (allow 0.2 10 J) [A1]

(no mark is given if planck constant value from table is used)

(iii) gradient of graph = 1/hc [C1]

24 24
= 4.80 10 5.06 10 [M1]

8 -34 -34
h = 1/(gradient 3.0 10 ) = = 6.6 10 - 6.9 10 J s [A1]

(only 1 mark if h is correctly obtained by solving using a single point;

2 marks if h is correctly obtained by averaging from 2 calculated values of h)

20
5 (a) A coil of 1500 turns of insulated wire is tightly wound on a non-magnetic tube to make a solenoid of
mean radius 22 mm, as shown in the figure below. The wire itself has radius 0.86 mm and is made of
-8
a material of resistivity 1.7 x 10 m. The coil is connected to a supply of e.m.f. and negligible
internal resistance.

Calculate
(a) the total length of wire in the coil, [2]
-3
Total Length of wire = 2 (22 *10 )*1500 = 207.3m

(b) The total resistance of the coil, [3]

-8 -3 2
Total resistance = (1.7 x 10 )(207.3)/( (0.86*10 ) = 1.52
c} the current in the coil [2]

I=V/R=12/1.52=7.91A

21
6. The following data concern a tennis ball moving horizontally at a given instant just before it
is struck by a tennis racket:

horizontal momentum of tennis ball = 2.4 N s,


kinetic energy of tennis ball = 45 J.

(a) Why is it correct to give direction of the momentum but not of the kinetic energy? [1]
Kinetic energy is a scalar and has no direction while momentum is a vector and has direction.

*Examiners comments: Need to address question. If just mention KE is Scalar and Momentum
is vector lacks details in telling me why the direction should be assigned to Momentum. Must
mention that momentum has a directions while KE does not.
(b) Write down in terms of the mass m and the velocity v of a body, expressions for
(i) the momentum,
(ii) the kinetic energy. [2]

mv
0.5 mv2
Easy almost all got correct.
(c) Use your answer in (b) to help you to calculate the mass and velocity of the tennis ball.[2]
mv=2.4
2
0.5 mv =45

v=37.5 ms-1
m=0.064 kg
Easy almost all got correct.
(d) When the racket hits the ball it exerts a average force of 200 N in a direction opposite to
its momentum, bringing the ball to rest momentarily. Calculate
(i) the time the tennis ball takes to stop,
(ii) the distance the tennis ball travels before stopping. [4]

(i) F=ma, a= 3125 ms-2


at=v-u
(3125)(t)=37.5-0
t=0.012s
Most no issue.
(ii) s=ut +0.5at2
=(37.5)(0.012)+0.5(-3125)(0.012)2
=0.225 m
Few mistakes here as well, those who got it wrong applied s=ut, without realising that in this
case the horizontal velocity is affected by a horizontal acceleration exerted by the racket. (when
there is a net force there is an acceleration.)
(Note: Tennis ball. racket contact time is about 5-6 ms. Max force is about 300N)

22
(e) A different average force of 160 N then continue to act on the tennis ball for a further
0.025 s. Calculate
(i) the new momentum of the ball,
(ii) the new velocity of the ball. [3]

(i) Ft =160(0.025)
p=4.0
pf-pi=4.0
pf=4.0 N s
Quite well done.
(ii) v=62.5 m s-1
Easy Qn
(f) Calculate the increase in kinetic energy of the ball for the whole time that the force is
applied to it and hence deduce the mean power being delivered to the ball while it is in contact
with the racket. [3]

Change in KE = KEfinal -KEinitial


= 80 J [C1]
Mistakes here include squaring the difference between velocities.
Mean power = Increase in KE/ time [C1]
= 80/ (0.037)
= 2162 W [A1]
Time here is the sum of the two times, one for stopping and the other for accelerating.
(g) Using Hookes Law, suggest why, in practice, it is impossible for a constant force to be
applied to the ball to decelerate it to zero velocity. [2]

As the tennis racket hits the ball, the racket strings undergo extension.(M1) The force exerted by the

string increases as the string elongates(A1), according to Hookes Law. Therefore, it is unlikely that a

constant force to be applied.

(h) As the tennis ball travels to the other end of the court, it hits and bounces off the rough
ground. The arrow below gives the direction of the velocity of the ball just before contact.
At the point where the ball is in contact with the ground, draw a free body diagram to
show the forces acting on the ball, together with the action reaction pair for each force
on the ball that is drawn. Label all forces carefully. [3]

Contact Force on
ball

Weight

23 Contact Force on
ground

Gravitational Force of ball


on earth
correct diagram with contact and weight on ball [1] action-reaction pair for ground-ball [1] action-
reaction pair for earth-ball [1]

7. Fig 8.1 shows a person of mass 65 kg doing a bungee jump.

Fig 8.1
The bungee rope has negligible mass and has an un-stretched length of 60 m. This bungee
rope may be assumed to obey Hookes law. A force of 140 N extends the bungee rope by 2.5
m. The bungee rope is secured to the feet of the person. The person leans over the bridge and
drops vertically downwards towards the river below. The maximum length of the stretched
bungee rope is 110 m. You may assume that air resistance is negligible.

(a)State Hookes law. [1]

Hookes law states that the extension is proportional to the stretching force.[B1]

(b)The momentum of the person increases during free fall. Explain whether or not this is a
violation of the principle of conservation of momentum. [2]

The principle of conservation of momentum states that the total momentum of a system is
constant provided there is no net external force.[M1] In the case of a free falling person, weight is
an net external force that causes a change in momentum. [A1]

Few got it right here. You need to identify the system as the person. Hence momentum is not conserved
as weight is an external force.

24
(c)(i) Describe the energy changes taking place from the instant the person leaves the bridge until the
bungee rope is fully extended. [3]

Initially it is at max GPE. During free fall, GPE decreases while kinetic energy (KE) increases [B1]. When
the bungee rope is stretched, GPE decreases while both KE and elastic potential energy (EPE) increase
[B1]. As the rope is stretched beyond the point where tension in the rope equals the weight, both GPE
and KE decrease while EPE increases [B1]. Finally at full extension, it is maximum EPE and zero KE and
minimum GPE.

(ii) The unit of energy is joule. Express the joule in terms of SI base units. [1]

-1 2
Note presentation issues! Do not write KE=kg(ms )

(d) Calculate the speed of the person when he has fallen 60 m below the bridge, just before the bungee
rope starts to stretch. [2]

2 2
v =u +2as
2
v =2(9.81)(60)
-1
v=34.3 m s
2
OR mgh=0.5 mv

(e)(i) Sketch a graph of the tension F in the bungee rope against its extension e.

F/N

e/m

(ii) Show that the elastic potential energy in the bungee rope is directly proportional to the square of
its extension. [2]

EPE= WD in stretching rope

=area under force-extension graph [M1]

Need to mention EPE is equal to work done, which in turn is area under graph.

=1/2 Fe

By Hooke's law F=ke


2
so EPE =1/2 (ke)e=1/2 ke [A1]
2
Hence EPE proportional to e

(iii) Determine the maximum tension in the bungee rope when it is fully stretched. [2]
25
Max tension = ke = 56(50) [M1-identify e=50]

=2800 N [A1]

(iv) Show that the elastic potential energy in the fully extended bungee rope is about 70kJ.[1]

-1
k=F/e=140/2.5= 56 Nm
2 2
EPE = 1/2 ke =1/2(56)(50)

=70 000J [B1 for correct final ans and finding correct k]

(v) Determine the magnitude of the acceleration of the person when the bungee rope is fully stretched. [2]

T-mg=ma

2800-(65)(9.81)=65a[C1]
-2
a=33.3m s [A1]

Most are unable to identify the force acting on the man are both tension and weight.

(vi) State the direction of the acceleration of the person when the bungee rope is fully stretched. [1]

Up[B1]

(e) Explain why it would be extremely dnagerous to have a bungee rope that is much stiffer.[2]

Stiffer rope has larger k, causing the person to stop over a smaller time interval [B1], hence causing a
larger force/acceleration[A1]

8 (a) (i) One tesla is the magnetic flux density in which a conductor one metre long, carrying
a current of one ampere and placed at right angles to the field, experiences a
force of one newton. [2 marks, minus one mark for one mistake.]

(ii)

[1 mark for correct shape, lines outside should be further apart further away]
[1 mark for correct direction]
26
(iii)
0 N I
B
2r
4 10-7 2 0.40
B [M 1]
0.094
B 1.07 10-5 T [A1]
OR
4 10-7 2 0.40
8.9 *10 4 T
-6
2 (10 / )

(iv) The current-carrying wire at segment A generates a magnetic field around itself [M1].
Since the current-carrying wire at segment B is inside (and at right angles to)
this magnetic field[A1], a magnetic force will act on the current-carrying wire at
segment B.

(v)
F B I L sin [M 1]
I
F 0 I L sin
2d
I 2 L sin
F 0
2d
Since 0, L, , and d remain constant when current changes, the force is directly
proportional to the square of the current.[A1]

(b) (i) The potential difference between two points is the amount of electrical energy
converted to other forms of energy per unit charge flow between the points. [2
marks or 0 marks]
Most miss per unit charge

(ii) R = l / A = 59 x 10-9 x 0.59 / (1 x 10-6) [C1]


= 0.035 [A1]

(iii) potential difference across wire coil = 0.035 x 0.40 = 0.014 V


potential difference across component X = 0.025 V (from graph) [M1]
potential difference across internal resistance = 0.045 - 0.014 - 0.025
= 0.006 V [M1]
internal resistance = 0.006 / 0.40 = 0.015 [A1]
Most problems stem from not being able to see that all 3 resistors are in series, or student miss one of the
3 resistors.

27
(iv) power supplied by battery = 0.40 x 0.045 = 0.018 W[C1]
power dissipated by internal resistance = 0.402 x 0.015 = 0.0024 W [C1]
percentage of total power supplied that is dissipated by internal resistance = 100% x
0.0024 / 0.018 = 13.5 % [A1] (Shown via just ratio of pd(no explanation to link
to power), minus 1 mark)

Quite well done

28
TAMPINES JUNIOR COLLEGE
PRELIMINARY EXAMINATIONS

CANDIDATE
NAME

CIVICS TUTOR
GROUP 1 2 NAME

PHYSICS Tuesday, 24 September 2013


Paper 1 Multiple Choice 1 hour 15 minutes
Additional Material: Multiple Choice Answer Sheet

READ THESE INSTRUCTIONS FIRST

Write in soft pencil.


Do not use staples, paper clips, highlighters, glue or correction fluid.
Write your name and class on the Answer Sheet in the spaces provided.

There are forty questions on this paper. Answer all questions. For each question there are
four options A, B, C and D.
Choose the one you consider correct and record your choice in soft pencil on the separate
Answer Sheet.

Read the instructions on the Answer Sheet very carefully.

Each correct answer will score one mark. A mark will not be deducted for a wrong answer.

This document consists of 15 printed pages.


2
1 A voltmeter connected across a resistor in a circuit gives readings which have high
precision but low accuracy. Which of the following best describes the likely error in
readings taken with this voltmeter?

Random Error Systematic Error


A large large
B large small
C small large
D small small

2 In an experiment to find its specific latent heat of vaporisation,


water is vaporised using an immersion heater as shown.

Three sources of error in this experiment are:

Error 1: water splashing out of the container;


Error 2: vapour condensing on the handle of the heater and
dripping back into the container;
Error 3: Heat is conducted away from the bottom of the
container.

What is the effect of these three experimental errors on the calculated value for the
specific latent heat?

Error 1 Error 2 Error 3


A Decrease Decrease Increase
B Decrease Increase Increase
C Increase Decrease Decrease
D Increase Increase Decrease

3 The graph below shows how the volume of 1 g of water varies between 273 K and
279 K.

volume / cm3

temperature / K
273 275 277 279
Why would a thermometer, based on the variation of the volume of a fixed mass of water,
be unsuitable for measurements in this range?

A The decrease in the volume over this range is greater than its increase.
B The variation of volume is negligible over this range.
C The variation of volume is non-linear over this range.
D The variation of volume is not single-valued over this range.

3
4 Which statement about internal energy is correct?

A The internal energy of a system can be increased without transfer of energy by


heating.
B The internal energy of a system depends only on its temperature.
C When two systems have equal internal energies, they are in thermal equilibrium.
D When work is done on a gas, its internal energy always rises.

5 A car is travelling at a speed of 15 m s-1 and can be brought to rest with a uniform
retardation in 1.2 s if the brakes are applied. The reaction time of the driver is 0.10 s.
What is the minimum distance of the moving car from a stationary object if a collision is to
be avoided?

A 9.00 m B 9.75 m C 10.5 m D 18.0 m

6 The diagram below shows a uniform rod freely pivoted at P. The rod is suspended
horizontally, when a string tied to one end of the rod passes over a smooth pulley and is
attached to a 3.0 kg mass. Determine the mass of the rod.

A 4.8 kg B 3.5 kg C 2.4 kg D 1.7 kg

7 A small beaker P containing four lead shots floats in water in a large beaker Q. If two lead
shots are taken out of P and placed in Q, which of the following statements is not correct?

A The total upthrust on P and the two lead shots in Q remains unchanged.

B The height of water in Q falls.

C The lead shots in Q displace its own volume of water.

D The weight of P with two lead shots is equal to the weight of water displaced by P.

4
8 The forces acting on the Earth and a part of the Antarctic ice shelf, which has its centre
of gravity directly above the Earths axis of rotation, are shown. All of the forces are
equal in magnitude.

Which pair of forces are equal because of Newtons 1st Law and which pair are equal
because of Newtons 3rd Law?

1st Law 3rd Law


A F1 and F3 F1 and F2
B F1 and F3 F2 and F4
C F3 and F4 F1 and F2
D F3 and F4 F2 and F4

9 The graph shows how the force acting on a body varies with time.

Assuming that the body is moving in a straight line, by how much does its momentum
change?

A 40 kg ms-1
B 36 kg ms-1
C 20 kg ms-1
D 16 kg ms-1

5
10 A large mass moving at a velocity of 5 m s-1 collides head-on with a small mass moving
at a velocity of 2 m s-1 in the opposite direction.

The collistion is elastic. After the collision, both masses move to the right. The large
mass has a velocity v1 and the small mass has a velocity v2.

Which pair of values v1 and v2 is possible?

V1 V2
A 2 m s-1 5 m s-1
B 3 m s-1 10 m s-1
C 4 m s-1 4 m s-1
D 5 m s-1 12 m s-1

11 A neutron moving with an initial velocity u has a head-on elastic collision with a
stationary proton. After the collision, the velocity of the neutron is v and that of the
proton is w. Taking the masses of the neutron and proton to be equal, which one of the
following statements is wrong?

A Conservation of momentum shows that u = v + w.

B Conservation of energy shows that u2 = v2 + w2

C The momentum and energy equations taken together imply that the speed of the
proton after the collision is the same as that of the neutron before the collision.

D The fact that the collision is elastic implies that the proton and neutron move off
in opposite directions with equal speeds.

12 The gravitational potential energy Ep of a mass near the Earths surface varies with
height h as shown.

What mass is being lifted?

A 1.6 kg B 2.4 kg C 8.2 kg D 16 kg

6
13
H

A
B

Figure 13.1

The figure 13.1 shows part of the route of a roller-coaster in an amusement park. The cart
descends from H, completes a circular loop A and moves to B. If the cart is to complete the
central circular track safely, what is the minimum speed of the cart at the bottom of the
circular track A? Assume that there is no friction between the cart and the track

A 9.81 m s-1 B 19.6 m s-1 C 22.1 m s-1 D 24.5 m s-1

14 What is the linear speed of a point on the Earths equator as a result of the Earths rotation
about its axis? ( Assume radius of Earth = 6.38 x 106 m)

A 2.16 x 10-3 m s-1


B 4.31 x 10-3 m s-1
C 232 m s-1
D 464 m s-1

15 The radius of the Earths orbit about the Sun is 1.50 x 1011 m. The Earth takes 365 days to
orbit the Sun. What is the mass of the Sun?

A 6.40 x 1029 kg
B 2.01 x 1030 kg
C 1.16 x 1030 kg
D 3.31 x 1029 kg

16 Two positive point charges +Q and one negative point charge -Q are positioned as shown
in the diagram (not drawn to scale). Both +Q and -Q have the same magnitude of charge.
Which option best represents the location where the electric field strength is zero?

7
17 W, X, Y and Z are four points on a straight line as shown in the following diagram.

A positive point charge +Q is placed at W and a negative point charge -Q is placed at X.


Both +Q and -Q have the same magnitude of charge. When -Q is moved from X to Y,
which one of the following statements is false?

A The electric potential energy will increase.


B The magnitude of the electric field strength at Z will increase.
C The electric potential at Z will increase.
D The electric force acting on a positive charge placed at Z will increase.

18 Every second, 6.0 x 1017 electrons and 3.0 x 1017 protons move across a cross-section of
a discharge tube filled with ionised hydrogen gas. The electrons and the protons move in
opposite directions. What is the current flowing through the discharge tube?

A 0.048 A B 0.096 A C 0.14 A D 0.19 A

19 A cell with e.m.f. E and internal resistance r, is connected to a variable resistor R as


shown in Figure (a). Figure (b) shows the variation of the voltmeter reading V with the
ammeter reading A as the resistance of R is varied. Assuming that the voltmeter and
ammeter are ideal, what is the internal resistance r?

A 1.1 B 2.9 C 4.0 D 5.1

20 The wiring in a house must be thick enough to ensure that a fire will not start as a result of
the heating effect produced by current flowing through the wiring. A 2.0 m length of copper
wire must carry a maximum current of 35 A and it is to produce no more than 1.8 W of
heat energy per second per metre. What is the minimum diameter of the copper wire?
(resistivity of copper = 1.68 x 10-8 m)

A 1.9 mm B 2.7 mm C 3.8 mm D 5.4 mm

8
21 A battery is connected to 4 resistors as shown in the following circuit diagram. What is the
potential difference between A and B?

A 0V B 3.5 V C 4.5 V D 7.5 V

22 In the diagram below, the inner wall of a glass dish was lined with a copper ring R and a
copper rod Q was placed exactly in the middle of the dish. Rod Q was connected, via a
switch K, to the positive plate, and ring R was connected to the negative plate of a
battery. The arrangement was placed between the poles of a strong magnet. The dish
was then filled with copper (II) sulphate solution and a small float F added to indicate
any motion of the liquid.

Which sketch indicates correctly the possible motion of the float as seen from above
when switch K was closed?

A B C D

+ + + +

9
23 The diagram shows part of an apparatus in which positive ions pass through slits S1, S2 and S3.

S1
electric field across the plane
S2 of the diagram and magnetic
field out of the plane of the
diagram

S3

Between S2 and S3, they pass through mutually perpendicular magnetic and electric fields, the
intensities of which may be varied.

What is the function of the mutually perpendicular fields between S2 and S3?

A to accelerate the ions to high velocity


B to select ions of a particular charge
C to select ions of a particular mass
D to select ions of a particular velocity

24 A flat, circular, single-turn loop of wire of radius r is placed with its plane at right angles to the
direction of a uniform magnetic field. The magnetic flux density is B and the magnetic flux
through the loop is .

What is the magnetic flux density and the magnetic flux through a loop of radius r in the same
plane?

magnetic flux density magnetic flux through loop


A B
B B
C B
D B

10
25 When a coil is rotated in a magnetic field, the induced e.m.f. E varies with time as
shown below.

Which of the following graphs, drawn to the same scale, would be obtained if the speed
of rotation of the coil is halved?

A B E
E

t t

C E D E

t t

26 An alternating current I/A varies with time t/s according to the equation

I = 5 sin (100t)

What is the mean power developed by the current in a resistive load of resistance 10 ?

A 125 W B 160 W C 250 W D 500 W

11
27 A pendulum is driven by a periodic driving force of frequency f. The variation of
pendulums amplitude with driving frequency was determined for experiments done in a
container of fixed volume of gas, and at two temperatures T 1 and T2. When T2 > T1,
determine which of the following graphs best illustrates how amplitude of the pendulum
changes with the driving frequency. (Viscosity of air increases with temperature).
A B
Amplitude T2 Amplitude
B T1

T1 T2
Driving frequency Driving frequency
C D
Amplitude AmplitudeT1
AC T2 DA

T1 T2
Driving frequency Driving frequency

28 A point source of sound emits energy equally in all directions at a constant rate and a
person 8 m from the source listens. After a while, the intensity of the source is doubled. If
the person wishes the sound to seem as loud as before, how far should he be standing
now?
A 2m B 2 2 m C 8 2 m D 2 8 m

29 By blowing across the mouth of a glass bottle, the air column in the bottle can be made to
vibrate and produce a tone. In order to tune the bottle and use it as a musical instrument, the
fundamental frequency of the bottle must be 440 Hz. The bottle is 0.450 m tall and the speed
of sound is 340 m s1. How high should the bottle be filled with water?

A 0.064 m B 0.193 m C 0.257 m D 0.386 m

30 When light of wavelength 570 nm is incident normally on a plane diffraction grating, the
second-order diffraction images are formed at an angle of 34.8 to the normal of the
grating. What is the number of lines per millimetre of the grating?

A 500 B 1000 C 2000 D 500000

31 Transitions between three energy levels in a particular atom give rise to three spectral
lines. The shortest and the longest wavelengths of these spectral lines are 1 and 2
respectively. The wavelength of the other spectral line is

2 1 12 12
A 2 - 1 B C D
2 1 2 2 1

12
32 The energy bands of three materials are shown below.

Which of the following arranges correctly the materials in increasing order of resistance at
room temperature?

A PQR B RPQ C QRP D QPR

33 Which of the following statements about laser production is correct?

A Stimulated emission is a key process because the photons produced have the same
frequency with a constant phase difference.

B A metastable state is required so that there can be more stimulated emissions than
spontaneous emissions.

C Population inversion is achieved when there are more atoms in the ground state
than in the excited state.

D The purpose of pumping is to trigger the occurrence of stimulated emissions.

34 Which of the following statements about a pn junction is correct?

A At the junction, the n-type semiconductor has a lattice of negative ions.

B When the n-type semiconductor is connected to the positive terminal and the p-type
semiconductor is connected to the negative terminal of a battery, the depletion
region becomes narrower.

C The pn junction is able to conduct current when the electric field set by an external
battery is in the same direction as that existing in the depletion region.

D The electric field that is set up at the junction points from the n type to the p type
semiconductor.

13
35 An electron with kinetic energy E has a de Broglie wavelength of . Which of the following
graphs correctly represents the relationship between and E?

36 A nucleus of the nuclide p


qX has binding energy U and rest mass M.
Given that mp and mn are the rest mass of a proton and a neutron respectively and c is
the speed of light, which of the following gives the value of U?

A ( pmp (q p)mn M )c 2 B (qmp ( p q )mn M )c 2


C (M qmp ( p q )mn )c 2 D (M qmp pmn )c 2

37 Samples of two radioactive nuclides, X and Y, each have equal activity Ao at time t= 0. X
has a half-life of 24 years and Y a half-life of 16 years. The samples are mixed together,
What will the total activity of the mixture at t = 48 years?

A Ao/12

B 3Ao/16

C Ao/4

D 3Ao/8

14
38 Figure 38.1 shows a graph of the natural logarithm of the activity A of a radioactive source
plotted against time/year.

What is the half-life of the source in years?

A 70 B 100 C 200 D 400

14
39 Given that the mass of 7 N is 14.003974u and that the sum of the masses of 11H and
13
C is 14.011179u, it would be reasonable to suppose that the nuclear reaction
6

1
1H + 136C 14
7 N

A can only take place if there is a net supply of energy.

B could not take place at all.

C must involve the emission of a further uncharged particle.

D will result in the emission of energy.

40 The half-life of a certain radioactive material is 3.0 s. How long does it take for its activity to
reduce by 90%?

A 0.46 s B 5.4 s C 10 s D 11 s

-The End-

15
Answers
1 2 3 4 5 6 7 8 9 10
C B D A C B A D C B
11 12 13 14 15 16 17 18 19 20
D A C D B A C C B C
21 22 23 24 25 26 27 28 29 30
B D D A B A B C C A
31 32 33 34 35 36 37 38 39 40
D B B D C B D A D C

16
TAMPINES JUNIOR COLLEGE
PRELIMINARY EXAMINATIONS

CANDIDATE
NAME

CIVICS TUTOR
GROUP 1 2 NAME

PHYSICS Tuesday, 3 September 2013


Paper 2 Structured Questions 1 hour 45 minutes

READ THESE INSTRUCTIONS FIRST

Candidates answer on the Question Paper.


Write in dark blue or black pen.
You may use a soft pencil for any diagrams, graphs or
rough working.
Do not use paper clips, highlighters, glue or correction fluid.

Answer all questions. For Examiners Use

The number of marks is given in brackets [ ] at the end of 1 9


each question or part question.
2 9

3 9

4 9

5 9

6 15

7 12

Total / 72

This document consists of 13 printed pages.


2
1 A boy takes a ride in a hot air balloon. When it is at a height of 90 m
above the ground and rising vertically at a speed of 1.5 m s-1, the
boy drops a ball of mass 0.30 kg.

Ignore air resistance.


(a) State the velocity of the ball just after the boy releases the ball.

velocity = m s1 [1]
(b) Show that the time it takes for the ball to reach the ground is 4.4 s. [1]

(c) Determine the velocity of the ball just before it hits the ground.

velocity = m s1 [2]
Air resistance is actually not negligible and is proportional to velocity with a proportionality
constant of 8.9 x 102 N s m1. The ball reaches terminal velocity a short while before hitting
the ground.
(d) Determine the terminal velocity of the ball.

terminal velocity = m s1 [1]


(e) On the same axes, sketch how the velocity of the ball varies with time from the instant
the boy releases the ball, up to the instant just before it hits the ground:
(i) for the case without air resistance. Label this graph S, and label appropriate values
on the axes.
(ii) for the case with air resistance. Label this graph P. [4]

3
2 (a) Newtons Third Law of Motion can be stated in the form: When object X exerts a force
on object Y, Y exerts a force of the same type that is equal in magnitude and opposite in
direction on X.
Explain what is meant by of the same type in this statement of the law.

...............................................................................................................................................

........................................................................................................................................... [1]
(ii) Object X is a book resting in equilibrium on a table (object Y). Draw labeled force
diagrams to show the forces on X and on Y. Make it clear which forces are equal in
magnitude and opposite in direction. [3]

(iii) Now suppose object X is a book that has fallen onto the table (object Y) and has just
landed on it. For the instant of impact, draw labeled force diagrams to show the
forces on X and Y. Make it clear which forces are equal in magnitude and opposite in
direction, and which forces are different in magnitude from those in (ii). [3]

(b) A horizontal force of 54 N is applied to push two touching blocks of mass 6.0 kg and 2.0
kg along a flat surface. The frictional force between the blocks and the surface is 6.0 N.

Calculate the magnitude of the resultant force on the 6.0 kg mass.

magnitude of resultant force = N [2]

4
3 (a) A battery is connected to an electric circuit which draws power from the battery. Explain
why the potential difference across the terminals of the battery is lower than the e.m.f.

...............................................................................................................................................

...............................................................................................................................................

........................................................................................................................................... [2]
(b) An electric circuit is set up as shown below and wire XY is 100 cm in length. It can be
assumed that both cells have negligible internal resistance.

When switches S1 and S2 are closed and the slider is at Z1, there is no current flowing
through the galvanometer.
(i) Determine the potential difference across XZ1.

potential difference = V [2]


(ii) Determine the length of XZ1.

length = cm [2]
Switch S1 is now opened and switch S2 remains closed. When the slider is at Z2 which is
another point on the wire XY, no current flows through the gavanometer.
(iii) Determine the length of XZ2.

length = cm [2]
(iv) State the current flowing through the 30 resistor.

current = A [1]
5
4 An explanation of the photoelectric effect includes the terms photon energy and work
function.
(a) Explain what is meant by
(i) a photon,

...............................................................................................................................................

........................................................................................................................................... [1]
(ii) work function.

...............................................................................................................................................

........................................................................................................................................... [1]
(b) In an experiment to investigate the photoelectric effect, a student measures the
wavelength of the light incident on a metal surface and the maximum kinetic energy
Emax of the emitted electrons. The variation with Emax of 1/ is shown in Fig. 4.1 below.

Fig 4.1

(i) The work function of the metal surface is


State an equation, in terms of , and Emax, to represent conservation of energy for
the photoelectric effect. Identify any other symbols you use.

...............................................................................................................................................

...............................................................................................................................................

........................................................................................................................................... [2]

6
(ii) Use only your answer in (i) and Fig. 4.1 (without using the value of Planck constant),
determine
1. the work function of the metal surface,

work function = J [2]


2. a value for the Planck constant.

Planck constant = J s [3]

7
5 (a) A possible fusion process which has been suggested for the generation of energy is
represented by the following equation:
2
1H 12H 23He 01n 5.2x 1013 J
Rest mass of proton = 1.67357 x10-27 kg Rest mass of neutron = 1.67496 x10-27 kg
Rest mass of 21H = 3.34454 x10-27 kg Rest mass of 32He = 5.00835 x 10-27 kg

(i) Define binding energy of a nucleus.

...............................................................................................................................................

........................................................................................................................................... [1]
Using the above data,
(ii) explain why 21H will not spontaneously split into a proton and a neutron

...............................................................................................................................................

...............................................................................................................................................

........................................................................................................................................... [2]
(iii) determine the energy released in the nuclear fusion.

energy = J [3]
(b) Three alpha particles with equal energy obtained from the above process are fired
towards the nucleus of a gold atom. Complete the diagram, showing the possible paths
of the alpha particles as they pass by, and then move away from the nucleus. [3]

Figure 5.1

8
6 One particular model of an atom which was developed in the early 1900s suggested that the
core of an atom consists of a massive nucleus. The nucleus is made up of nucleons the
protons (charge of +1.6 1019 C) and neutrons (chargeless) giving its net positive charge.
Charged particles interact via the long-range Coulomb force. In the case of like charges
such as protons, the electrostatic force between them is repulsive and varies inversely with
the square of the distance between them,

Q1Q2
F =k
r2

where Q1 and Q2 = magnitude of charges [in C]


k = 8.99 109 [in F1 m]
r = distance between the charges [in m]

All the nucleons also undergo another force known as the nuclear force. This is an attractive
force which is strong enough to overcome the Coulomb force between protons in the short
range, i.e. comparable to the spacing between nucleons in a nucleus.

As a result of the forces which the nucleons undergo within each nucleus, nuclei have
different extents of stability. Each plot on the graph below represents one nucleus.

number of neutrons (N)

140

120

100

80

60

40
60 65 70 75 80 85 number of protons (Z)

Fig. 6.1

9
(a) The electrostatic force between like charges is a repulsive force.
State one other type of repulsive force.

........................................................................................................................................... [1]
(b) Small nuclei which have less than 40 nucleons have approximately equal numbers of
neutrons (N) and protons (Z). For large nuclei, the values of N and Z vary according to
the equation:
N = mZ
where m is a constant.
(i) Sketch the graph of N = Z in the graph in Fig 6.1. [1]
(ii) Determine an approximate value for m. Show your working clearly.

m = [2]
(c) Using the information provided, explain why the existence of the long-range Coulomb
force and the short-range nuclear force causes instability in large nuclei if the number of
neutrons (N) is equal to the number of protons (Z).

...............................................................................................................................................

...............................................................................................................................................

...............................................................................................................................................

...............................................................................................................................................

...............................................................................................................................................

...............................................................................................................................................

........................................................................................................................................... [2]
(d) Consider 2 protons which are positioned at the furthest possible distance apart in a large
spherical nucleus of radius 7.5 1015 m. Calculate the electrostatic force of repulsion
experienced by one charge due to the other.

F = N [2]
10
(e) Suggest, with explanation, if this value of electrostatic force is higher or lower than the
actual force experienced by the same charge.

...............................................................................................................................................

...............................................................................................................................................

...............................................................................................................................................

........................................................................................................................................... [2]
(f) For large nuclei which are unstable, suggest one way they can become more stable.

...............................................................................................................................................

........................................................................................................................................... [1]
12
(g) is stable but 146C is not. Using information provided in this question, explain whether
6C
14
6C is an alpha emitter or a beta emitter.

...............................................................................................................................................

...............................................................................................................................................

........................................................................................................................................... [2]
(h) In Fig. 6.2, draw a line that passes through the stable nuclides. [2]
100

90

80

70

60

N 50 Fig. 6.2

40

30

20

10

0
0 10 20 30 40 50 60 70 80 90 100

11
7 A current-carrying coil produces a magnetic field.

It is suggested that the strength B of the magnetic field at the centre of a flat circular coil is
inversely proportional to the radius r of the coil.

Design a laboratory experiment that uses a Hall probe to test the relationship between B
and r. You should draw a diagram showing the arrangement or your equipment. In your
account, you should pay particular attention to:
(a) The procedure to be followed
(b) The measurements to be taken
(c) The control of variables
(d) The analysis of the data
(e) The safety precautions to be taken.
[12]
Diagram

12

-The End-

13
Marking Scheme

(a) 1.5 m s-1 upwards. [1]

(b) s = ut + at2
90 = -1.5t + (9.81)t2 [A1]
(9.81)t2 - 1.5t - 90 = 0
t = 4.4 s

(c) v = u + at
v = -1.5 + 9.81 x 4.4 [C1]
= 41.7 m s-1 [A1]

(d) 0.30 x 9.81 = 8.9 x 10-2 x v where v is terminal velocity


v = 33 m s-1 [A1]

(e)

S line from negative to positive [M1]; values [A1]

P- shape of curve from negative to positive [M1]


v drops to zero earlier than S; terminal velocity less than 41.7 m s-1 and graph ends
beyond 4.4s [A1]

14
2 (a) (i) It means the forces are of the same nature like gravitational, electrical, etc.
Incorrect: It means the forces are either contact or non-contact forces.
(ii)

weight of X (gravitational
X
force by Earth on X)

normal force by Y on X

normal force by X on Y

weight of Y ( gravitational
Y
force by Earth on Y)

normal force by floor on Y

Show all 5 forces, indicating which body the force is acting on [M1]
Equal in magnitude and opposite in direction:
weight of X and normal force by Y on X (though not action-reaction pair) [B1]
normal force by Y on X and normal force by X on Y (action-reaction pair) [B1]
(iii)

weight of X (gravitational
X
force by Earth on X)

normal force by Y on X

normal force by X on Y

weight of Y (gravitational
Y
force by Earth on Y)

normal force by floor on Y

Same as in (ii) but with the normal forces larger [M1]


Equal in magnitude and opposite in direction: normal force by Y on X and by X on Y [B1]
Forces different in magnitude from those in (ii): the three normal forces [B1]

(b) 56N - 6N = 8.0 a (C1)


a = 6.0 ms-2
Fnet = 6.0*6.0 ms-2 = 36 N (A1)

15
3 (a) The battery has internal resistance [M1] and the potential difference across the internal
resistance causes the potential difference across the battery terminals to be lower than
the e.m.f. [A1].
(b) (i) current through 100 resistor = 2 / ( 100 + 30 ) = 0.015 A [M1]
potential difference across 100 resistor = 0.015 x 100 = 1.5 V
potential difference across XZ1 = 1.5 V [A1]
(ii) length of XZ1 = 100 x 1.5 / 4 [M1] = 38 cm [A1]
(iii) potential difference across XZ2 = 2.0 V, length of XZ2 = 100 x 2 / 4 [M1] = 50 cm [A1]
(iv) 0 A [A1]

4. a(i)A photon is a quantum of energy in electromagnetic radiation. [1]

(ii) Work function is the minimum energy required for an electron to escape from the
surface of a metal.[1]

Markers comments several students were not careful and stated that this was the energy
needed for electrons to escape, for which no mark is given.

b(i) hc/ = + Emax [M1]

h= planck constant
c= speed of light [B1 if both correct]

(ii) when 1/ = 0, = -Emax

working - graph is extended to read the x intercept.[M1]

= 4.0 10-19 J (allow 0.2 10-19 J) [A1]

(no mark is given if planck constant value from table is used)

(iii) gradient of graph = 1/hc [C1]

= 4.80 1024 5.06 1024 [M1]

h = 1/(gradient 3.0 108 ) = = 6.6 10-34 - 6.9 10-34 J s [A1]

(only 1 mark if h is correctly obtained by solving using a single point;


2 marks if h is correctly obtained by averaging from 2 calculated values of h)

5 a)i) Binding energy is the energy required to split a nucleus into its individual
nucleons.
16
Markers comments Students must ensure that they use the correct term nuclei, nuclide or
nucleons to answer correctly.

(ii) mass of proton+mass of neutron = 1.67357 x10-27+1.67496 x10-27


= 3.34853 x10-27 kg
This is greater than the mass of 21H.

2
Hence, energy is needed to split 1H into proton and neutron and this will not happen
spontaneously.

Correct explanation with numerical values 2 marks


Correct explanation without numerical values 1 mark

(iii)
Total rest mass before= 2x 3.34454 x10-27 kg = 6.68908 x10-27 kg
Total rest mass after = 5.00835 x 10-27 + 1.67496 x10-27
= 6.68331 x 10-27 kg

Difference in rest mass = 5.77 x 10-30 kg [M1]

Energy released = 5.77 x 10-30 x (3 x 108)2 [C1]

= 5.2 x 10-13 J [A1]


Markers comments The most common errors are careless mistakes omitting the factor of
10-27 or not squaring the speed of light after stating the correct equation.

b)

Deflection amount [M1]


Deflection before nucleus [A1]
Direction of deflection [A1]
Markers comments The most common mistake is to draw the deflection occurring when the
alpha particle is about to pass or has passed the nucleus.

6 One particular model of an atom which was developed in the early 1900s suggested that the
core of an atom consists of a massive nucleus. The nucleus is made up of nucleons
17
the protons (charge of +1.6 10-19 C) and neutrons (chargeless) giving its net positive
charge. Charged particles interact via the long-range Coulomb force. In the case of like
charges such as protons, the electrostatic force between them is repulsive and varies
inversely with the square of the distance between them,

Q1Q2
F =k
r2

where Q1 and Q2 = magnitude of charges [C]


9 1
k = 8.99 10 [F m]
r = distance between the charges [m]

All the nucleons also undergo another force known as the nuclear force. This is an
attractive force which is strong enough to overcome the Coulomb force between protons
in the short range, i.e. comparable to the spacing between nucleons in a nucleus.

As a result of the forces which the nucleons undergo within each nucleus, nuclei have
different extents of stability. Each plot on the graph below represents one nucleus.

number of neutrons (N)

140

120

100

80

60

40
60 65 70 75 80 85 number of protons (Z)

Fig. 6.1

(a) The electrostatic force between like charges is a repulsive force.


State two other types of repulsive force.
Magnetic force between like poles;
18
electromagnetic force between 2 long straight conductors carrying currents in
opposite directions.
..
..[1]

Markers comments:
Students need to be specific in the answer that they provide. Some students provided answers
that are vague and did not get any marks for it eg. Magnetic force, electromagnetic force,
magnetic repulsion, two unlike current
(b) Small nuclei which have less than 40 nucleons have approximately equal numbers of
neutrons (N) and protons (Z). For large nuclei, the values of N and Z vary according to
the equation:

where m is a constant.

(i) Sketch the graph of N = Z in the graph above.


[line N = Z] [A1]

Markers comments:
Students who got this wrong did not read the question carefully OR failed to analyse the
graph well. Some students simply drew the best fit line of all the points given in the graph
where N Z. Others drew a diagonal line that splits the grids into two without realising that
the starting points and scale of both axis are different.
(ii) Determine an approximate value for m. Show your working clearly.

[use average; pick first and last point OR pick centre point; m = N/Z] [M1]

get m = 1.4 to 1.5 [A1]

(those who do not average, minus 1)

m = [2]

Markers comments: Most students wrongly (~ 95%) used the gradient to find the value of m
as the y-intercept of the of the best fit of all the pointes given in the graph provided is not
equal to zero.
(c) Using the information provided, explain why the existence of the long-range Coulomb
force and the short-range nuclear force causes instability in large nuclei if the number of
neutrons (N) is equal to the number of protons (Z).
..
The attractive short range nuclear force acts on all nucleons, so in a small
..
nucleus, the protons are bounded together despite the weaker repulsive
..
electrostatic force acting on them. [B1]
..
..
In a large nucleus, the attractive short range nuclear force does not extend to the
..
protons which are at the edge of the nucleus. The repulsive electrostatic force is
..
dominant and causes the nucleus to be unstable. [B1]
..[2]

19
(d) Consider 2 protons which are positioned at 2 furthest possible distance apart in a large
spherical nucleus of radius 7.5 10-15 m. Calculate the electrostatic force of
repulsion experienced by one charge due to the other.

7.2 10-1515 1.5 1014 m


r 22x7.5x10 [1]
19
1.6 10 1.6 1019
F 8.99 109 1.02 N [1]

2
1.5 1014

F = N [2]

(e) Suggest, with explanation, if this value of electrostatic force is higher or lower than the
actual force experienced by the same charge.
This value is smaller because a large nucleus contains many protons. [1]
..
..
A proton at the boundary of the nucleus will experience repulsion from all other
..
protons. [1]
..[2]
Markers comments: Presence of external forces besides electrostatic force is not taken into
consideration for this question. Here, we are looking at the magnitude of the actual electrostatic
force being experienced by the charge. Thus, only distance of separation and electrostatic force
caused by other protons are considered relevant.
(f) For large nuclei which are unstable, suggest one way they can become more stable.
..
Radioactive decay; nuclear fission
..[1]
(g) too many N, should strive for N = Z as this is small nucleus [M1]
so this is a beta-emitter (this will bring down to N=Z=7) [A1]
Markers comments: A number of students do not actually understand the process of beta decay
where a neutron becomes a proton and electron with the electron being emitted.
14 14 -1
CORRECT: 6 C 7 N + e

Wrong: 146 C 126 N + 2n


(h) In Fig. 6.2, draw a line that passes through the stable nuclides.[2]

20
100

90 matches
Fig. 6.1
80
line N = Z for small nuclides (40) [B1]
70 then smooth continue and then matches Fig.
6.1 for larger nuclides [B1]
60

N 50 Fig. 6.2

40 any
continuous
30 interpolation

20 N=Z

10

0
0 10 20 30 40 50 60 70 80 90 100

21
(1 mark)

22
23
TAMPINES JUNIOR COLLEGE
PRELIMINARY EXAMINATIONS

CANDIDATE
NAME

CIVICS TUTOR
GROUP 1 2 NAME

PHYSICS Friday, 20 September 2013

Paper 3 Longer Structured Questions 2 hours

READ THESE INSTRUCTIONS FIRST

Candidates answer on the Question Paper.


Write in dark blue or black pen.
You may use a soft pencil for any diagrams, graphs or
rough working.
Do not use paper clips, highlighters, glue or correction fluid.

Section A For Examiners Use


Answer all questions.
1 8
Section B
2 8
Answer any two questions. Please circle, on the cover
page, the two questions chosen. 3 8

4 8
You are advised to spend about one hour on each section.
5 8
The number of marks is given in brackets [ ] at the end of
each question or part question. 6 20

7 20

8 20

Total / 80

This document consists of 15 printed pages.


2
SECTION A
Answer all questions in this Section.

1 (a) State two quantities that are conserved in any collision where no net external force acts.

........................................................................................................................................... [1]
(b) An ideal gas is contained in a container by means of a piston as shown.

piston
Ideal gas atom

Given that the system is thermally insulated, state, with a reason, whether the
momentum of any one gas atom colliding with the piston is conserved in the collision.

...............................................................................................................................................

...............................................................................................................................................

........................................................................................................................................... [2]
(c) The piston is now moved to the right so that the volume of the gas is reduced.
(i) State what change, if any, occurs in the speed of a gas atom during an elastic
collision with the moving piston.

........................................................................................................................................... [1]
(ii) Use the kinetic theory of gases to explain whether the temperature of the gas
should rise.

...............................................................................................................................................

...............................................................................................................................................

........................................................................................................................................... [2]
(iii) Write down a word equation to represent the conservation of energy for this change
in volume of the gas. [2]

3
2 An oscillating mass-spring system is found to reach a maximum speed of 3.0 cm s1 with a
period of 1.5 s. The mass used was previously measured to be 2.0 kg.
(a) Determine the total energy of this system, assuming negligible spring mass.

energy = J [2]
(b) Within the axes provided below, sketch the velocity-time graph of this oscillation,
labelling all known values where applicable. [1]

1.5 3.0

(c) Shade an area in the sketched graph that represents the amplitude of the oscillation. [1]

(d) Determine this amplitude of oscillation.

amplitude = cm [2]
(e) Hence, sketch the velocity-displacement graph that represents this oscillation in the
space provided below, labelling all known values where applicable. [2]

4
3 The diagram below shows the typical apparatus used for the production of X rays. The
potential difference between the cathode and anode is 90 kV.

The target is made of tungsten. The energy levels of tungsten for the three lowest electron
shells are given below.

(a) Explain why there is a minimum wavelength of X rays produced.

...............................................................................................................................................

........................................................................................................................................... [1]
(b) Determine the shortest wavelength of X rays produced

wavelength = m [2]
(c) Calculate the two shortest characteristic wavelengths that will be observed in the X ray
spectrum produced by the tungsten target.

wavelengths = m and m [3]


(d) Sketch the variation of intensity of X rays produced with wavelength, indicating
appropriate values on your sketch. [2]

5
4 The following diagram shows two large vertical plates A and B. Plate A is connected to earth.
An electron is emitted perpendicularly from plate A with an initial velocity of 4.3 x 106 m s1
towards the right. In the region between the two plates, the electron experiences an electric
force of 1.1 x 1016 N towards the left.

(a) Define the electric potential at a point.

...............................................................................................................................................

........................................................................................................................................... [1]
(b) Determine the potential of plate B.

potential = V [3]
(c) The electron comes to a stop momentarily at point P. Determine the distance between
point P and plate A.

distance = V [2]
(d) Determine the potential at point P.

potential = V [2]

6
5 Strontium-90 is a radioactive nuclide that undergoes random and spontaneous decay.
(a) Explain what is meant by random and spontaneous decay.

...............................................................................................................................................

...............................................................................................................................................

...............................................................................................................................................

...............................................................................................................................................

........................................................................................................................................... [2]
(b) A sample of Strontium-90 has a mass of 2.40 108 g. The average activity of this
sample during a period of 1 hour is found to be 1.26 105 Bq.
(i) Define activity.

...............................................................................................................................................

........................................................................................................................................... [1]
(ii) Calculate the decay constant of Strontium-90.

decay constant = s1 [3]


(iii) Hence, deduce the half-life of Strontium-90

half-life = s [1]
(c) Suggest why the determination of decay constant by measuring the mass and activity of
a sample can be used only for nuclides that have relatively small decay constant.

...............................................................................................................................................

...............................................................................................................................................

........................................................................................................................................... [1]

7
SECTION B
Answer two questions from this Section.

6 (a) Fig. 6.1, which is not drawn to scale, shows how the gravitational potential between the
surface of the Moon and the surface of the Earth varies along the line of centres. At point
P the gravitational potential is at its maximum.
x y

Moon Earth
P
0

- 1.3

- 3.9

- 62.3

Potential / 106 J kg-1 Fig 6.1

The distance between the center of the Moon and the Earth is about 470 000 km. Given
mass of Earth is 6.0 x 1024 kg and mass of Moon=7.4 x 1022 kg.
(i) Define gravitational potential at a point.

...............................................................................................................................................

...............................................................................................................................................

........................................................................................................................................... [2]
(ii) Explain why the gravitational potential at a point due to a mass is always negative.

...............................................................................................................................................

...............................................................................................................................................

........................................................................................................................................... [2]
(iii) State how the resultant gravitational force on a mass at any point between the Earth
and the Moon could be deduced from the diagram.

...............................................................................................................................................

........................................................................................................................................... [1]

8
(iv) Using information from the graph, for a mass of 15 000 kg,
1. Determine the minimum kinetic energy it must be projected from the surface of
the Earth in order to reach the surface of the moon.

kinetic energy = J [2]


2. Determine the speed of the mass as it hits the surface of the moon if it is
projected with the minimum kinetic energy in (iii)1.

speed = m s1 [2]
(b) The Earth has radius 6.4 x 106 m and mass 6.0 x 1024 kg. A satellite has mass 850 kg
and orbit at a height of 0.8 x 106 m above Earths surface.
(i) Determine the linear speed of the satellite.

linear speed = m s1 [3]


(ii) Determine whether the satellite is in a geostationary orbit. [4]

(c) The total energy of a satellite orbiting around Earth is given by the equation
ET = GMm / 2R
where M is the mass of Earth, m is the mass of satellite and R is the radius of orbit.
The satellite in part (b) gradually loses energy due to small resistive forces.
Explain why many such satellites eventually burn up in the Earths atmosphere.

...............................................................................................................................................

...............................................................................................................................................

...............................................................................................................................................

...............................................................................................................................................

........................................................................................................................................... [4]

9
7 (a) Fig. 7.1 shows the diagram of a circuit. Part of the circuit QRST is placed in a region of
uniform field B of 0.530 T that is directed in the positive x-direction. The battery supplies
a current I of 2.50 A. The wire experiences a force as a result. The length of wire TS, SR
and RQ are each 2.50 cm. TS is along y-direction, RQ is along x-direction, and SR
makes an angle of 45 with the x-direction.
The region of the magnetic field is represented by the shaded region.
y R Q P

S
B

Fig. 7.1
U V

(i) Define tesla.

...............................................................................................................................................

...............................................................................................................................................

........................................................................................................................................... [1]
(ii) State and explain the direction of the resultant magnetic force acting on the wire TSRQ.

...............................................................................................................................................

...............................................................................................................................................

...............................................................................................................................................

...............................................................................................................................................

........................................................................................................................................... [3]
(iii) Calculate the resultant magnetic force acting on the wire TSRQ.

force = N [2]
(iv) It is possible that there is no resultant force acting on the wire if the field points in one
particular direction. Draw the field in this direction in Fig. 7.1 and label it P. [1]

10
(b) Fig. 7.2 below shows the top view of a current balance where the rectangular wire loop
PQRS pivoted at AB is in equilibrium. It is connected in series with a battery of 300 g
and an e.m.f. of 2.0 V. Part of the wire loop is placed inside a solenoid. The mass of the
loop can be taken to be negligible.

P Q
I
Fig. 7.2

S R
B

(i) Draw the magnetic field lines in the solenoid. [2]


(ii) The length of the side PS is 6.0 cm and SB = 0.6 SR. Given that the magnetic flux
density in the solenoid is 0.40 T and the wire has no resistance, calculate the internal
resistance of the battery.

internal resistance = [3]


(c) A student designs a bicycle that uses electromagnetic braking instead of braking by
friction. Two pairs of electromagnets are attached to the brakes of front and back wheels
of the bicycle. When the brakes are activated, magnetic north poles face the wheels.

2.0 cm S

4.0 cm

Fig. 7.3 aluminium


wheel frame

11
(i) The diagram below shows the top view of the cross section of the braking system.
Sketch the magnetic field lines in the aluminium wheel frame. [2]
S
N

Activated
electromagnets

N
S aluminium wheel
Fig. 7.4 frame

(ii) Using energy consideration, explain how the above setup acts as a braking system.

...............................................................................................................................................

...............................................................................................................................................

...............................................................................................................................................

...............................................................................................................................................

...............................................................................................................................................

........................................................................................................................................... [3]
(iii) An experiment is conducted before fitting the electromagnet to the bicycle. To test
the effectiveness of the braking system, the student places the electromagnet at a
fixed position, with the field produced pointing into the plane of the paper. The
surface of the magnet facing the wheel has a dimension of 4.0 cm by 2.0 cm. A wire
loop is moved at a speed of 10.0 m s-1 towards the magnet.

B
10.0 m s-1
2.0 cm

Fig. 7.5
4.0 cm
Assuming that the uniform magnetic flux density B is 0.70 T, calculate the e.m.f
induced in the wire as it cuts the field.

internal resistance = [2]


(iv) Suggest, using your answer from (c)(iii), why the braking system is an effective one.

...............................................................................................................................................

........................................................................................................................................... [1]

12
8 (a) Explain what is meant by the diffraction of a wave.

...............................................................................................................................................

........................................................................................................................................... [1]
(b) The behaviour of water waves is being investigated in a ripple tank.
(i) Sketch diagrams, one for each case, to illustrate the diffraction of water waves
through a narrow gap and a wide gap [3]

narrow gap wide gap

(ii) Two source interference of water waves, using double-slit arrangement, may not be
observed when the slits are wide. However, narrowing the slits with the same
previous separation produces obvious interference patterns. Suggest why.

...............................................................................................................................................

...............................................................................................................................................

...............................................................................................................................................

...............................................................................................................................................

...............................................................................................................................................

...............................................................................................................................................

........................................................................................................................................... [3]
(c) Almost all data about stars are determined from measurements involving
electromagnetic waves.
(i) Monochromatic light from the hydrogen spectrum is observed using a diffraction
grating having 300.0 lines per millimetre. The first order diffraction pattern occurs at
an angle of 8.385. Determine the wavelength of the light.

wavelength = m [2]
13
(ii) When the hydrogen spectrum in light from a distant star is observed by an
astronomer, the corresponding angle is 8.388 instead. Calculate the percentage
change which has occurred in the wavelength.

percentage change = % [2]

(iii) Suggest why the change above is called a red shift.

........................................................................................................................................... [1]
(iv) The astronomer would like to change the method in (ii) to increase the difference
between the two angles mentioned above. Suggest two ways the method may be
changed.

...............................................................................................................................................

........................................................................................................................................... [2]
(d) Fig. 8.1 shows two microwave transmitters A and B 0.20 m apart. The transmitters
emit microwaves of equal amplitude in phase and of wavelength 30 mm. A detector,
moved along the line PQ at a distance of 5.0 m from AB, detects regions of high and low
intensity forming an interference pattern.

5.0 m
0.20 m

B Fig. 8.1

(i) Calculate the separation between one region of high intensity and the next along
the line PQ.

separation = m [2]
14
(ii) State the effect, if any, on the position and intensity of the maxima when each of the
following changes is made, separately, to the experiment.

1. The amplitude of the transmitted waves is doubled.

...............................................................................................................................................

...............................................................................................................................................

........................................................................................................................................... [2]

2. The separation between the transmitters is halved.

...............................................................................................................................................

...............................................................................................................................................

........................................................................................................................................... [2]

The End

15
Mark Scheme:
1 (a) Linear momentum, total energy [1 each = 0.5]
(b) Not conserved because velocity (at least direction) of gas atom changed upon collision
OR piston exerts a force on the gas atom [1]
hence momentum must also have changed. [1]
(c) (i) speed increases. [1]
(ii) When the piston collides with the gas molecules, it provides them a net gain in KE [1]
Since ideal gas, u = total KE alone (now that PE = 0) has increased, = 3/2Nk T
T must have increased. [1, formula not required, only need proportionality]
(iii) increase in kinetic energy (= increase in internal energy ) = no heat gain (insulated)
+ work done on gas by piston (volume decreases) [2 or 0; if change then 1 mark only]

2 (a) T.E. = max K.E. = 0.5 x 2 x (0.03)2 [1]


= 9.0 x 10-4 J [1]
(b) Correct sinusoidal graph, axes labelled, values of max v to be inserted, 2 cycles in 3.0 s
[1 for all correct items]
(c) shade only area subtended by of a waveform [1]
(d) v = xo 0.030 = (2/1.5) x0 [M1 with subs],
xo = 0.72 cm [A1]
(e) elliptical shape [M1]
Both + and - xo and v labelled with values [A1]

3(a) There is a minimum wavelength of X rays produced when the accelerated electron
loses all its energy at once upon hitting the target and emits one single photon.

(b) eV = hc/o
1.6 x 10-19 x 90000 = 6.63 x 10-34 x 3.00 x 108/o [M1]
o = 1.4 x 10-11 m [A1]

(c) hc/1 = -10.2 - (- 69.5) = 59.3 keV [M1]


1 = 6.63 x 10-34 x 3.00 x 108 /59.3 x 103 x 1.6 x 10-19 = 2.1 x 10-11 m [A1]
2 = 6.63 x 10-34 x 3.00 x 108 /67.0 x 103 x 1.6 x 10-19 = 1.9 x 10-11 m [A1]

(d)
intensity

/10-11 m
1.4 1.9 2.1

- continuous spectrum with 2 peaks;


- peak at 2.1 higher than the peak at 1.9;
- minimum of 1.4
[-1 for any mistake]

16
4 (a) The electrical potential at a point is defined as the work done per unit charge by an
external force in bringing a positive charge from infinity to that point. [B1]

(b) mg = 9.11 x 10-31 x 9.81 = 8.9 x 10-30 N [M1]


The gravitational force is very small compared to the electric force. [A1]

(c)
FEq
V
F q
d
V
1.1 10-16 1.6 1019 [M 1]
0.1
V 69 V [M 1]
Therefore potential of plate B is - 69 V. [A1]

(d) Work done by electric force on electron = 1.1 x 10-16 d cos 180 = - 1.1 x 10-16 d
Loss in KE of electron = 0.5 x 9.11 x 10-31 x ( 4.3 x 106 )2
1.1 x 10-16 d = 0.5 x 9.11 x 10-31 x ( 4.3 x 106 )2 [M1]
d = 0.077 m [A1]

(e) potential difference between point P and plate A = -69 x 0.077 / 0.10 = -53 V [M1]
potential at point P - potential at point A = - 53 V
potential at point P - 0 = - 53 V
potential at point P = - 53 V [A1]

5)
a) The decay is random because there is no way to predict which nucleus will decay OR
when a nucleus will decay.[b1]The decay is spontaneous because it is not affected by
external physical conditions or trigger [b1]
b) i) Activity of a radioactive material is the number of nuclei that disintegrate per unit or
rate of disintegration of nuclei
ii)N = (mass/molar mass) x Avogadro number
= ( 2.40 x 10-8 /90) x 6.02x 1023
= 1.605 x 1014 [M1]
= A/ N
= (1.26 x 105)/ (1.6053 x 1014) [C1]
= 7.85 x 10-10 s-1 [A1]

ii) t1/2 = ln 2/ = 8.83 x 108 s [A1] ecf

c) activity changes significantly during the period of measurement of activity [A1]


d)
e) The rate at which activity in a radioactive decay changes depends on the half-life of the
radioactive nuclide [B1]. A small decay constant would have meant a longer half-life and
thus a more constant and accurate average value of activity can be used to calculate the
decay constant. [B1]

17
6a) i) It is defined as the work done by an external agent per unit mass in bringing an object
from infinity to that point. [2]

ii) Gravitational potential at infinity is taken to be zero [B1].


Gravitational force is attractive. [B1]
Negative work is done by the agent because the force applied by the external agent and the
displacement of the mass are opposite in direction [B1].
[Any 2]

iii) The resultant force at a point can be found by the product of the gradient of the graph at that
point and the mass of the mass at that point. [B1]
iv) 1. For minimum kinetic energy to reach the Moon, KE at P = 0 J
thus, total energy = -1.3 x 106 x 15000 J [C1]
KEmin = (-1.3 - (-62.3)) x 106 x 15000 J = 9.15 x 1011 J [A1]

2. mv2 = m(3.9 1.3)106


v = 2280 m s-1

bi) gravitational force provides the centripetal force, GMm/R2 = mv2/ R [C1]
v = {(6.67 x 10-11 x 6.0 x 1024)/(7.2 x 106) [M1]
v= 7.5 x 10 3 m s-1 [A1]

bii) T = 2r/v = 2 x7.2 x 106/2280 [C1]

T = 1.69 hrs [M1]

A geostationary satellite has a period of 24 hrs [M1]

Satellite is not in geostationary orbit. [A1]

c) Due to resistive forces, total energy of the satellite decreases. From the equation, this leads
to decrease in the radius of orbit [M1]. (ET becomes more negative, GMm/2R has a larger
magnitude, R becomes smaller).
As radius of orbit decreases, satellite will orbit with a greater linear velocity [M1]. (since KE =
GMm/2R)

Resistive force increases with increase in the velocity of the satellite [A1].
With greater resistive force and higher velocity, the heat energy generated per unit time
increases [A1] causing the satellite to burn up.

7 (a) (i) One tesla is the magnetic flux density in which a conductor one metre long and
carrying a current of one ampere placed at right angles to the field experiences a
force of one newton. [1]
(ii) By Flemings Left Hand Rule [M1]
No force on QR, Force on RS out of paper, Force on ST out of paper [M1]
Therefore resultant force is out of paper. [A1]
(iii)

18
FRS BIL sin
(0.530)(2.50)(0.025) sin 45o
0.0234 N

FST BIL sin


(0.530)(2.50)(0.025) sin 90o
0.0331N [M1 for correct FRS and FST]

Fnet 0.0234 0.0331 0.0565N


[A1]

(iv) magnetic field must be parallel to RS.

Either

or

(b) (i) parallel and uniform [B1] towards the left [B1]
(ii)
CWM ACWM
FB ( LSB ) W ( LBR )
( BILPS )( LSB ) mgLBR
(0.40)( I )(0.060)(0.6 L) (0.300)(9.81)(0.4 L) [M 1]
I 81.8 A [M 1]
E IR
2.0 (81.8) R [A1]
R 0.0244

19
(c) (i) shape of magnetic field [B1]. direction of magnetic field. [B1]

(ii) As the wheel frame cuts the magnetic flux, there is a change in flux linkage. [B1]
(By Faradays law), this produces an induced e.m.f. and an induced current in the
wheel frame. [B1]
Hence, mechanical energy is transformed into electrical energy. [B1]
(iii)
d d ( NBA cos ) dBA (0.7 0.02 0.04 0.7 0)
E [M1]
dt dt dt 0.04 / 10
0.14V [A1]
(iv) Though e.m.f. induced is small, the resistance of the wheel frame is small and hence
can result in large electrical power generated. [B1]

8 (a) Spreading of a wave as it passes through openings and around obstacles [1]
(b) (i) For both,
same wavelength in and out [1]
Individually,
Circular pattern for narrow gap [1], straight with curve at edges for wide gap [1]
(ii) When slits are wide, there is little spreading (diffraction) of water waves
and hence 2 individual streams of waves cannot superpose to form interference patterns. [1]
When slits are narrow, significant spreading allows both waves to act as coherent sources [1]
Overlap and coherence allows constructive and destructive interference to form at various
places [1]
(c) (i) d sin = n [1 correct substitution, esp d]
-7
= 4.861 x 10 m [1, using 4 s.f is crucial]
(ii) d sin = n [0 correct substitution, esp d]
-7
new = 4.863 x 10 m [1]
percentage change = (change/original) x 100%[0 for correct number substitution]
= 0.04% [1, allow difference in answer due to s.f. except 3 s.f.]
Called red shift because the observed wavelength has increased from original towards red side
of spectrum. [1]
nd
(iii) 2 or higher order can be used because correspondingly larger angle deviation [1]
Use grating with even higher line density [1]
(d)

20
21
TEMASEK JUNIOR COLLEGE
2013 Preliminary Examination
Higher 1

PHYSICS 8866/01
26 September 2013
Paper 1 Multiple Choice
1 hour

Additional Materials: Multiple Choice Answer Sheet

READ THESE INSTRUCTIONS FIRST

Write in soft pencil.


Do not use staples, paper clips, highlighters, glue or correction fluid.
Write your name and Civics group on the Answer Sheet in the spaces provided.

There are thirty questions in this paper. Answer all questions. For each question there are
four possible answers, A, B, C and D.
Choose the one you consider correct and record your choice in soft pencil on the separate
Answer Sheet.

Read the instructions on the Answer Sheet very carefully.

Each correct answer will score one mark. A mark will not be deducted for a wrong answer.
Any rough working should be done in this booklet.

This booklet consists of 14 printed pages.


[Turn over
2

Data

speed of light in free space, c = 3.00 x 108 m s-1

elementary charge, e = 1.60 x 10-19 C

the Planck constant, h = 6.63 x 10-34 J s

unified atomic mass constant, u = 1.66 x 10-27 kg

rest mass of electron, me = 9.11 x 10-31 kg

rest mass of proton, mp = 1.67 x 10-27 kg

acceleration of free fall, g = 9.81 m s-2

Formulae

uniformly accelerated motion, s = ut + at2

v2 = u2 + 2as

work done on/by a gas, W = pV

hydrostatic pressure, p = gh

resistors in series, R = R1 + R2 +

resistors in parallel, 1/R = 1/R1 + 1/R2 +


3

1 The resistance of an electrical component is measured. The following meter readings


are obtained.

What is the resistance?

A 2.5 k B 2.7 k C 2500 k D 2700 k

2 The radius of the Earth is 6.4 x 103 km and the volume of a grain of sand is
approximately 1 mm3.
What is the order of magnitude of the number of grains of sand that can fit in the volume
of the Earth?

A 1012 B 1018 C 1024 D 1030

3 A man pushing a mop across a floor causes it to undergo two displacements. The first
has a magnitude of 150 cm and makes an angle of 120 with the positive x-axis. The
resultant displacement has a magnitude of 140 cm and is directed at an angle of 30.0
to the positive x-axis. (Both angles are measured anti-clockwise with respect to the
positive x-axis.)

What are the magnitude and direction of the second displacement?

magnitude / cm angle to the positive x-axis


A 290 90

B 290 90
C 205 17
D 205 17

4 Two objects A and B fall from rest from different heights. A has half the mass of B. B
takes twice as long as A to reach the ground.
Neglecting air resistance, what is the ratio of the height fallen by A to that by B?

A 1:4 B 1:2 C 1: 2 D 4:1


4

5 The motion of a moving object can be described by the following acceleration-time


graph.
acceleration

time
0 t1
The change in velocity of the object from time t = 0 to t = t1 is

A the gradient of the tangent to the curve at t1.


B the area under the graph from time t = 0 to t = t1.
C the product of time t1 and the corresponding value of acceleration.
D the slope of the line OA.

6 Two different rifles are fired horizontally at a bulls-eye from the same distance of 150 m
away. The muzzle speeds of the two bullets are 600 m s1 and 1200 m s1. The barrels
of the rifles are pointed directly at the center of the bulls-eye, but the bullets strike the
target below the centre. The separation between the two holes made in the bulls-eye is
A 0 cm B 23 cm C 38 cm D 46 cm

7 A man stands on a weighing scale in a stationary lift. If sometime later, the reading on
the scale increases, the elevator could be moving with

A deceleration downward.
B increasing momentum downward.
C decreasing velocity upward.
D net force downward.
5

8 The figure below shows the variation with time t of the force F during the collision of a
58 g superball with a wall. The initial velocity of the ball is 34 m s -1 perpendicular to the
wall. The ball rebounds directly back with the same speed, also perpendicular to the
wall.

Fmax

t / ms
0 2 4 6

What is the maximum magnitude of the force Fmax on the ball by the wall during the
collision?

A 0.49 N B 490 N C 0.99 N D 990 N

9 Two masses make an elastic collision as shown below.

4.0 kg 3.0 kg

5.0 m s-1 2.0 m s-1

What are their velocities after the collision?

4.0 kg mass 3.0 kg mass


A 1.6 m s-1 to the left 4.6 m s-1 to the right
B 1.6 m s-1 to the right 4.6 m s-1 to the right
C 2.4 m s-1 to the left 5.4 m s-1 to the right
D 2.4 m s-1 to the right 5.4 m s-1 to the right
6

10 The man in the diagram below is lying down on a uniform board supported by a
weighing scale and a pivot. The weighing scale and the pivot are equidistant from each
end. The mans height is 1.8 m and his mass is 110 kg. The mass of the board is 6.0 kg
and the scale reads 580 N.

1.8 m

The distance from the pivot to his centre of gravity is

A 0.88 m B 0.92 m C 0.97 m D 1.0 m

11 A mass is suspended by a cord from a ring which is attached by two further cords to the
ceiling and the wall as shown in the diagram. The cord from the ceiling makes an angle
of less than 45o with the vertical.

The tensions in the three cords are labeled R, S and T. How do they compare in
magnitude?

A S>T>R
B S>R>T
C R>T>S
D R>S>T
7

12 A solid steel sphere is held just below the surface of water and released. Its terminal
velocity v is measured. Assume that upthrust is negligible and the drag force on the
sphere is proportional to its velocity. If the experiment were repeated in places with
different values of g (the acceleration of free fall), which of the following would most
closely resemble a graph of v against g?

v A v B

g g
C D
v v

g g

13 A stone of mass 0.40 kg is projected horizontally at a speed of 6.0 m s -1 from the top of
a wall, which is 5.0 m above the ground. When it arrives at the ground, its speed is
10 m s-1.

How much energy has it lost in falling through the air?

A 13 J B 12 J C 6.8 J D 0.4 J
8

14 A mass m, attached to the end of an unstretched spring, is initially supported by a


platform as shown in Fig. (a). This platform is then removed and the mass falls,
eventually coming to rest at the position shown in Fig. (b).

unstretched
spring

platform

Fig. (a) Fig. (b)


Which of the following correctly relates the changes in energy which may occur during
this process?
A decrease of gravitational potential energy = increase of elastic potential energy

increase of elastic potential energy


B decrease of gravitational potential energy = + energy dissipated as heat

decrease of gravitational potential energy = increase of elastic potential energy


C
+ energy dissipated as heat

decrease of gravitational potential energy = energy dissipated as heat


D
+ increase of elastic potential energy

15 A particle of mass m is sliding down a rough incline. At position P, its velocity is v1 while
at position Q, a vertical distance h down the incline, its velocity is v2.

v1
h Q

v2

What is the expression for the energy lost due to friction in going from position P to Q?
A mgh B mgh - m (v22 v12)
C mgh + m (v22- v12) D m (v22 v12)
9

16 The diagram shows the displacement-time graphs for two waveforms X and Y.

displacement
X Y

0 1 2 3 4 5 6 7 8
time / ms

What is the phase difference between X and Y?

A 0.8 ms C /4 radians
B 1.0 ms D /2 radians

17 The diagram shows a cathode-ray oscilloscope display of an electromagnetic wave.

The time base setting is 0.20 s cm1.

What is the wavelength of the electromagnetic wave?

A 2.0 cm B 60 m C 120 m D 600 m

18 If two waves of the same frequency are superposed in phase, the total energy carried is
proportional to
A the sum of the energies carried by the separate waves.
B the difference of the energies carried by the separate waves.
C the sum of the square of the two amplitudes.
D the square of the sum of the two amplitudes.
10

19 The diagram represents a stationary wave formed by the superposition of sound waves
from a loudspeaker and their reflection from a metal sheet (not shown).

W, X, Y and Z are four points on the line through the centre of this wave.

Which statement about this stationary wave is correct?

A A displacement antinode is formed at the surface of the metal sheet.

B A node is a quarter of a wavelength from an adjacent antinode.

C The oscillations at X are in phase with those at Y.

D The particles of the waves oscillate at right angles to the line WZ.

20 In a Youngs double slit experiment, the ratio of the intensities between the bright and
the dark fringes is 9.

What is the ratio of the amplitudes of the waves from the two slits?

A 2 B 3 C 4 D 9

21 In the diagram below, side X of a conductor has twice the diameter as compared to side
Y.

IX IY
Y
X

What is the ratio of the current IX to IY?


A 0.5 B 1
C 2 D 4
11

22 A student is given a circuit consisting of 2 ideal diodes, an ohmic resistor and a filament
lamp as shown below. The variable power supply can provide both positive and
negative voltages across PQ.

P Q

Which one of the following graphs best represents the I-V characteristics across PQ?
A I B I

V V

C I D
I

V V

23 The circuit below shows a network of resistors each of resistance 1.0 .

What is the effective resistance between the points A and B?

A 2.7 B 2.0 C 1.7 D 0.63


12

24 Four circuits P, Q, R and S are shown below. In each circuit, the cell has the same
e.m.f. E. Each of the lamps has a resistance of 2 .

E 1 E 1

P Q

E 3 E 3

R S
Which of the following gives the combinations in the order of decreasing brightness?
A QPRS B SRPQ
C QSPR D RPSQ
25 Four resistors are connected as shown in the circuit below. R1 and R4 each has a
resistance of 2r, while R2 and R3 each has a resistance of r.

R1 R3

E V

R2 R4

The reading on the voltmeter will be increased by adding another resistor of value r in
parallel with
A the voltmeter B R1
C R2 D R4
13

26 Two long straight and parallel wires carrying currents in opposite directions separate
the surrounding space into three regions 1, 2 and 3.

Region 1 I1

Region 2
I2

Region 3

In which region(s) can there be a neutral point (that is, a point of zero magnetic field)?
A Region 2 only.
B Either region 1 or region 3 but not both.
C Both regions 1 and 3.
D There are no neutral points.

27 A long straight wire XY lies in the same plane as a square loop of wire PQRS which is
free to move. The sides PS and QR are initially parallel to XY.

Q R

P S
X Y

Both the wire and loop carry equal sinusoidal alternating currents. The currents in XY
and side QR differ in phase by rad.
Which one of the following graphs shows the variation with time t of the resultant force
of attraction F between the wire and the loop?

A B
F F

0 t 0 t

C D
F F

(zero at all times)


0 t 0 t
14

28 In a photoelectric experiment, the potential difference V that must be maintained


between the illuminated surface and the collector so as just to prevent any electrons
from reaching the collector is determined for different frequencies f of the incident
illumination. The graph below is obtained.

V1

f
f0 f1
The Planck constant is given by

V1 V1 eV1 f1 f0
A B C D
f1 f1 f0 f1 f0 eV1

29 What is the de Broglie wavelength of a particle of mass m and kinetic energy E?

h 2 h 2mE
A B h C D
2mE mE mE h

30 In the diagram below, E1 to E6 represent some of the energy levels of an electron in an


atom.
E6 0.38 eV
E5 0.54 eV
E4 0.85 eV

E3 1.5 eV

E2 3.4 eV

E1 13.6 eV

Which one of the following transitions produces a photon of wavelength in the ultra-
violet region of the electromagnetic spectrum?

A E2 E1 B E3 E2 C E4 E3 D E6 E5
TEMASEK JUNIOR COLLEGE
2013 Preliminary Examination
Higher 1
CANDIDATE
NAME

CIVICS INDEX
GROUP NUMBER

PHYSICS 8866/02
Paper 2 Structured Questions 16 September 2013
2 hours
Candidates answer on the Question Paper.
No Additional Materials are required.

READ THESE INSTRUCTIONS FIRST

Write your Civics group, index number and name on all the work you hand in.
Write in dark blue or black pen on both sides of the paper.
You may use a soft pencil for any diagrams, graphs or rough working.
Do not use staples, paper clips, highlighters, glue or correction fluid.

Section A
Answer all questions.

Section B For Examiners Use


Answer any two questions.
1
At the end of the examination, fasten all your work securely
together and circle the questions you have answered in Section B 2
in the grid provided.
3
The number of marks is given in brackets [ ] at the end of each 4
question or part question.
5
6
7
sf
Total

This booklet consists of 20 printed pages.


2

Data

speed of light in free space, c = 3.00 x 108 m s-1

elementary charge, e = 1.60 x 10-19 C

the Planck constant, h = 6.63 x 10-34 J s

unified atomic mass constant, u = 1.66 x 10-27 kg

rest mass of electron, me = 9.11 x 10-31 kg

rest mass of proton, mp = 1.67 x 10-27 kg

acceleration of free fall, g = 9.81 m s-2

Formulae

uniformly accelerated motion, s = ut + at2

v2 = u2 + 2as

work done on/by a gas, W = pV

hydrostatic pressure, p = gh

resistors in series, R = R1 + R2 +

resistors in parallel, 1/R = 1/R1 + 1/R2 +


3

Section A

Answer all the questions in this section.

1 The Young Modulus E of a material is given by the equation

FL
E
Ae

where F = force exerted on the material,


L = original length of the material,
A = cross-sectional area of the material,
e = extension of the material.

In performing an experiment to determine the Young Modulus of steel, a student made the
following measurements of a cylindrical steel wire:

diameter of wire = 1.15 0.05 mm


length of wire = 2.00 0.01 m

He measured the extension of the wire as the force was varied. He then plotted a graph of
the force against the extension. He deduced that there were random errors in his data as
there was some scatter of the experimental data. The best-fit line drawn was observed to
pass through the origin.

The slope of the graph of force against extension was found to be 95 5 N mm-1.

(a) What is meant by a random error?

[1]

(b) How does the drawing of the best fit line reduce random errors?

[1]

(c) The values of the extension of the steel wire are found to be small. Suggest a suitable
instrument for the measurement of the extension.

[1]
4

(d) State and explain, with clear workings, which was the most important uncertainty in
the students measurements.

[3]

(e) The student calculated the Young Modulus of steel and expressed it as
E = 1.8293 x 1011 N m-2.
Express E, together with its associated uncertainty, to an appropriate number of
significant figures.

E= N m-2 [3]
5

2 A simplified drawing of a suspension bridge is shown in Fig. 2.1. The bridge AB itself has a
weight W and is supported at its edges A and B as well as by cables at C and D. The length
of the bridge is 3d where AC = CD = DB = d. A tie joining the top of the pillar to the ground at
E and another at F hold the pillars in a vertical position.

Fig. 2.1
(a) State the conditions necessary for a body to be in equilibrium.

[2]

(b) (i) If the reaction on the bridge at each of the supports A and B is W vertically
4
upwards, find the tension, in terms of W, in the tie.

tension in the tie = [4]

(ii) State and explain the advantage of using a cable and a tie to hold the pillar.

[2]
6

3 (a) Helmholtz coils are a pair of flat circular coils, having equal numbers of turns and
equal diameters, arranged on a common axis and connected in series, as shown in
Fig. 3.1.

Fig. 3.1

The magnetic field at the centre of the coils is uniform. Fig. 3.2 shows the top view of
the two coils with the directions of the currents flowing through them indicated.

X X

Fig. 3.2

(i) Sketch the pattern of the magnetic field around the Helmholtz coils on Fig. 3.2.
Indicate clearly the direction of the field in your diagram. [3]

(ii) Explain why no magnetic force is experienced by a straight current-carrying


conductor placed along the axis of the coils.

[2]
7

(b) Two long straight parallel wires A and B are separated by a distance d. Each carries a
current I in the same direction as shown in Fig. 3.3.

A B
d

Fig. 3.3

(i) Explain the forces which exist between the two wires. Indicate the directions of
the forces on both wires.

[3]

(ii) One particular overhead powerline consists of 2 parallel cables with a separation
of 6.0 m. The current in each cable is 200 A. Calculate the force per unit length
on each cable. (The magnetic field strength B at a distance d due to a long
straight wire carrying a current I is given by I .)
o
B
2 d
.

force per unit length = N m-1 [3]


8

4 This question is about power dissipation in a resistor and the internal resistance of a battery.

In the circuit shown in Fig. 4.1, the variable resistor can be adjusted to have known values of
resistance R. The battery has an e.m.f. E and an unknown internal resistance r.

Fig. 4.1

The table below shows the recorded value I of the current in the circuit for different values of
R. The last column gives the calculated value of the power P dissipated in the variable
resistor.

R/ (10%) I/A (0.01 A) P/W

1.0 1.20 1.4

2.0 1.00 2.0

3.0 0.86 2.2

4.0 0.77 2.3

6.0 0.60 2.2

8.0 0.50

10.0 0.43

(a) (i) State the equation which relates the power P, the current I and the resistance R of
a resistor.

[1]

(ii) Complete the last two rows of the table by calculating the power dissipated in the
variable resistor.
[1]
9

(iii) If each value of R is accurate to 10 % and the uncertainty of the ammeter is


0.01 A, determine the absolute uncertainty in the value of P when R = 10.0 .

absolute uncertainty of P = W [3]

(b) (i) On the grid in Fig 4.2, plot the variation with resistance R of the power P.

Fig 4.2 [3]

(ii) It can be shown that the power dissipated in the external resistor is a maximum
when the value of its resistance R is equal to the value of the internal resistance r
of the battery, that is R = r. Use this information and your graph to find the value of
r.

r= [1]
10

(iii) PR
The battery generates total power PT. Show that the fraction is given by
PT
PR R
=
PT Rr

[1]
(iv) PR
Use the equation in (b)(iii) to deduce the fraction when R is
PT
1. equal to r,

fraction = [1]

2. very much larger than r.

fraction = [1]
11

Section B

Answer two questions in this section.

5 (a) A ball bearing, initially at rest, slides down a smooth slope, inclined at an angle of 37o to
the horizontal as shown in Fig. 5.1. It travels 5.0 m along the slope before falling freely
off the edge E.

ball bearing

37o
E

ground G

Fig. 5.1

(i) Calculate the speed of the ball when it reaches the edge E of the slope.

speed = m s-1 [2]

(ii) The time taken for the ball to reach the ground G after leaving the edge of the
slope is 0.85 s. Show that the height H of the edge of the slope above the ground
is 7.5 m.

[2]
12

(iii) Taking reference from edge E, sketch on Fig. 5.2 and Fig. 5.3 the variation with
time of the horizontal displacement x and the vertical displacement y of the ball
during its flight from E to G. Label the graphs P and Q respectively. (Indicate
appropriate numerical values but no further calculation is required.)

x/m

t/s
0

Fig. 5.2

y/m

0 t/s

Fig. 5.3 [3]

(iv) Sketch on Fig. 5.2 and Fig. 5.3 the corresponding variation with time of the
horizontal and vertical displacements of the ball, if air resistance is not negligible.
Label the graphs R and S respectively. [2]
13

(b) Fig. 5.4 shows two masses M1 and M2 connected by a light, very long and inextensible
string which passes over a fixed, smooth and light pulley.

pulley

M1 = 7.0 kg

M2 = 5.0 kg

Fig. 5.4

(i) Draw free-body diagrams showing the forces acting on each of the two masses.

[2]

(ii) The system is released from rest and the 7.0 kg mass reaches the ground after 3.0 s.
Calculate

1.
Calculate
the acceleration of the masses while the string remains taut.

acceleration = m s-2 [3]

2. the tension in the string before the 7.0 kg mass strikes the ground.

tension = N [1]
14

(iii) The 5.0 kg mass is now disconnected from the pulley while the 7.0 kg mass is
connected via the long string to a drum of radius 0.10 m, as shown in Fig. 5.5.
The drum is rotated by an electric motor at a steady rate of 10.0 revolutions per
minute.
drum pulley

0.10 m
10.0 rev min-1

7.0 kg

Fig. 5.5

1. Calculate the useful work done in raising the 7.0 kg when the drum rotates by one
revolution.

useful work done = J [2]

2. Assuming that 20 % of the work done by the motor is lost, calculate the power
developed by the motor.

power developed = W [3]


15

6 (a) Fig 6.1 shows a pipe which is open at one end and closed at the other.

Fig 6.1
(i) Explain how standing waves are formed in the pipe.

[2]

(ii) In Fig 6.1, draw lines to represent the waveforms of the fundamental resonant
note for the pipe, labeling the positions of the node(s) and antinode(s) with the
letters N and A respectively.
[2]

(iii) The length of the pipe is 0.317 m.

1. Calculate the frequency of the fundamental note of the pipe.


(speed of sound in air = 325 m s1)

frequency = Hz [2]

2. Calculate the next resonant frequency of the pipe and its corresponding
wavelength.



frequency = Hz

wavelength = m [2]
16

(b) Two coherent light wavetrains having the same plane of polarization meet at a point.
State two conditions that must be fulfilled before totally destructive interference can
occur.

1.

2.

[2]

(c) Fig. 6.2 shows an experiment to demonstrate interference effects with microwaves. A
transmitter, producing microwaves of wavelength, is placed in front of two slits
separated by a distance a. A receiver is used to detect the strength of the resultant
wave at different points along the line YZ which is at a distance D in front of the slits.

Fig. 6.2

(i) Explain, in terms of the path difference between the wavetrains emerging from
the slits S1 and S2, why a series of interference maxima are produced along the
line YZ.

[2]

(ii) Predict how the distance x between neighbouring maxima on the line YZ would
change if the distance a was doubled while the distance D was halved.

[1]
17

(iii) Explain why it is necessary to use a barrier with two slits rather than two
separate transmitters.

[1]

(iv) In another experiment using the apparatus in Fig. 6.2, a student notices that the
distances between the maxima are not equal. Suggest a reason for this
difference.

[1]

(v) Describe how you could test whether the microwaves leaving the transmitter
were plane polarised.

[2]

(vi) The microwave transmitter is now placed in front of a plane reflector as shown in
Fig. 6.3 and stationary waves are set up in the space between them.

transmitter

detector reflector

Fig. 6.3

A detector is moved between the transmitter and the reflector at a constant speed
of 10 mm s1. The frequency of detection of minima is 1.5 Hz.

Determine the frequency of the microwave oscillator.

frequency = Hz [3]
18

7 In a photoelectric emission experiment as shown in Fig. 7.1, ultraviolet radiation of


wavelength 254 nm and intensity 210 W m-2, was incident on a silver surface in an evacuated
tube, so that an area of 12 mm2 was illuminated. Photoelectrons were collected at an
adjacent electrode at a rate of 2.7 x 1013 per second.

UV light

e-

V
Fig. 7.1

(a) (i) Calculate the energy of an incident photon.

energy = J [2]

(ii) Calculate the rate of incidence of photons on the silver surface.

rate of incidence = s-1 [2]

(iii) Calculate the photoelectric current recorded by the ammeter.

current = A [2]
19

(b) (i) State what is meant by stopping potential.

[1]

(ii) If light of higher wavelength was used to illuminate the silver surface, would the
stopping potential increase or decrease? Explain your answer.

[3]

(iii) Sketch on the same axes to show the variation of the photoelectric current I with
the potential difference V between the electrodes for
1. illumination of wavelength 254 nm, and
2. illumination of a higher wavelength.

[3]
20

(c) Fig. 7.2 shows the lowest five energy levels for a hydrogen atom.

E5 _________________________ - 0.38 eV
E4 _________________________ - 0.54 eV

E3 _________________________ - 0.85 eV

E2 _________________________ - 3.39 eV

E1 _________________________ - 13.6 eV

Fig. 7.2

(i) Explain why such energy levels lead to a line spectrum.

[2]

(ii) Hydrogen atoms in a low pressure vapour lamp are excited from the ground state
when they are bombarded by electrons. These electrons are accelerated from
rest by a potential difference V. A line corresponding to a wavelength of 436 nm is
observed in the line spectrum.

1. Deduce the energy levels involved in the transition to produce this


wavelength.

[3]

2. State the minimum value of V used. Explain your answer.

[2]
21
TJC 2013 PHYSICS H1 PRELIMINARY EXAM SOLUTIONS

PAPER 1

1 2 3 4 5 6 7 8 9 10 11 12 13 14 15
C D C A B B A D D B B D C B B
16 17 18 19 20 21 22 23 24 25 26 27 28 29 30
D C D B A B D D A C B A C A A

PAPER 2

Section A

1 (a) An error is random if repeating the measurement under the same


conditions yields readings with error of different magnitude and sign. [1]

(b) Drawing the best fit line averages out the positive and the negative errors
about the best fit line and hence minimize the random errors. [1]
(c) Travelling microscope. (Do not accept micrometer screwguage or
vernier caliper) [1]

(d)
A d 0.05
2 2 0.086
A d 1.15
l 0.01
0.005
l 2.00

F
e 5 0.053
F 95
e
Measurement of diameter contributes the highest fractional uncertainty for A,
hence it is most significant. [3]

(e)

E A l

F
e
E A l F
e
E
0.086 0.005 0.053
1.8293 1011
E 0.3 1011 Pa

E = (1.8 0.3)x1011 Pa [3]


2 (a) Resultant force acting on the body in any direction is zero. [1]
Resultant moment on the body about any axis is zero. [1]

(b) (i) The net force acting vertically on the bridge is zero.
where T is the tension in the cable [1]

[1]

The net force acting horizontally on the pillar is zero.


where TT is the tension in the tie [1]

[1]

(ii) The vertical component of the tensions in the cable and tie pin the pillar
to the ground. Hence it will not be easily pushed to either side by any [1]
sideway force.
The tensions in the tie and cable provide moments about the base of the
pillar which tend to cancel out, keeping the pillar upright. [1]

3 (a) (i)

[3]

(ii) Current in the straight conductor is parallel to the magnetic field of coil
at the centre. Therefore, according to Flemmings left-hand rule, no force
is exerted on the conductor. [2]

(b) (i)
FBA FAB

A B
d
The magnetic field density due to current in wire A acts downward at wire
B according to Right Hand Grip Rule. It interacts with the current in wire B
to give a force FAB acting to the left according to Flemings Left Hand
Rule. Likewise a force FBA acts to the right on wire A by wire B. They form
an action-reaction pair. [3]

(ii) F = BIL

2
F o I 4 x 10 7 (200) 2
1.33 x 10 3 N m-1 [3]
L 2 d 2 (6.0)
4 (a) (i) P = I2R [1]

(ii) last 2 values of P = 2.0 W, 1.8 W. both correct [1]


(iii) P/P = 2 I/I + R/R [1]
P = [2 (0.01/0.43) + 0.1] 1.85 [1]
= 0.3 W [1]

(b) (i) data points plotted correctly [2] -1 for any wrong plot
best fit curve [1]

(ii) From graph, r = 4.0 (Accept 3.5 < r < 4.5 ) [1]

I 2R R
(iii) fraction = 2
= [1]
I (R r ) Rr
P
(iv) 1. when R =r , R =1/2 =0.5 [1]
PT
P
2. when R>>r, R 1 [1]
PT
Section B

5 (a) (i) v2 = u2 + 2as


= 0 + 2 9.81 sin 37o 5.0 [1]
v = 7.68 m s-1 [1]

(i) s = ut + at2 [1]


= 7.68 sin 37o 0.85 + 9.81 0.852 [1]
= 7.5 m

(ii) & (iv)


x/m y/m
7.5
P Q
S
R

0 t/s 0 t/s
0.85 0.85

Fig. 5.2 Fig. 5.3


2 marks - correct shape of P and Q with appropriate numerical values indicated
1 mark - gradient of y versus t graph 0 at t = 0
1 mark - correct shape of both R and S
1 mark - showing R extends beyond 0.85 s and Q & S end at 7.5 m

(b) (i) [2]

T T

M2 g M1 g

(ii)
By Newton's 2nd Law,

for mass M1, M1g T = M1a -----------------(1) [1]


for mass M2, T M2g = M2a -----------------(2) [1]
Solving, a = 1.64 m s-2 [1]
T= 57.2 N [1]
(iii) 1.
Displacement s = 2 r = 2 x 0.10 = 0.628 m [1]

Useful work done = Mgh = 7.0 x 9.81 x 0.628


= 43.1 J [1]

2.

Time taken for one revolution


= 60 / 10.0
= 6.0 s [1]

= 1 0.20 = 0.80

Mgh 43.1
Puseful = 7.2 [1]
t 6.0

Puseful
Pmotor = 9.0W [1]
0.80

6 (a) (i) Reflection of the incoming sound wave by the closed end of the pipe
produces an identical wave (of same amplitude, wavelength) moving in
the opposite direction. [1] When the two waves superpose, they give a
standing wave. [1]

(ii)

N A

[1] for correct waveform, [1] for labeling of N and A

(iii) 1. For the fundamental note, = 4 L = 4 0.317 = 1.27 m [1]


Frequency, f = v / = 325 / 1.27 = 256 Hz [1]

2. = 4/3 L = 0.423 m [1]


f3 = v / = 325/0.423 = 768 OR
f3 = 3 f1 = 3 256 = 768 Hz [1]
(b) (Totally destructive interference means that the resultant wave has zero
amplitude or waves cancel each other.)
Waves must meet rad out of phase. [1]
Waves must have equal amplitude. [1]

(c) (i) Wavetrains from S1 and S2 are coherent and superpose at points along [1]
YZ.
When path difference is an integral multiple of , the waves are in phase,
constructive interference takes place to give a series of maxima. [1]

(iii) It decreased to one quarter of the original x (since x = D/a) [1]


(iii) So that the wavetrains are coherent. [1]

(iv) The line YZ is not parallel to the slits [1]


or the slits not normal to the (incident) microwaves

(v) Place a metal grid or polaroid in front of the transmitter and rotate [1]
through 90o OR rotate grid/transmitter/detector through 90o.
If this causes minimal/zero signal at some angles, the wave is plane [1]
polarized.

(vi) Distance between two nodes = speed of detector / frequency of detection [1]
= 10 / 1.5 = 6.7 mm

Hence, wavelength = 13 mm [1]


8
f = c/ / 0.013 = 2.3 x 1011 Hz. [1]

7 (a) (i)
hc
E

6.63 10 34 3.0 10 8

254 10 9
7.83 10 19 J
(ii)
ET N
P ( )hf
t t
N P 210 12 10 6

t hf 7.83 10 19
3.22 1015 s 1
(iii) N'

I e 2.7 1013 1.60 1019
t
4.32 106 A

(b) (i) Stopping potential is the potential difference that must be applied
between the cathode and the anode so that the most energetic
photoelectron emitted just fails to reach the anode. [1]
c
(ii) From eVs h

If light of higher wavelength was used, the energy of the photons would
decrease. Since the work function is unchanged (same metal used),
the kinetic energy of the emitted electrons would decrease. Hence the
stopping potential would decrease. [3]

I
(iii)

1. = 254 nm

2. > 254 nm

V
0
[3]

(c) (i) Since the energy levels are discrete (quantized), electron transitions between
the levels involve absorption or emission of photons of discrete/specific
wavelengths resulting in spectral lines. [2]

hc 6.63x10 -34 x 3.00 x 108


(ii) 1. Energy of photon = = = 4.56 x10-19 J
436x10 9

= 2.85 eV

Since E4 E2 = 2.85 eV, the transition is from E4 to E2. [3]

2. The bombarding electron must have a minimum kinetic energy of


13.06 eV in order for the hydrogen atom to be excited from the
ground state E1 to E4. Only then can a transition E4 to E2 take place.
So the minimum value of V is 13.06 V. [2]
TEMASEK JUNIOR COLLEGE
2013 Preliminary Examination
Higher 2

PHYSICS 9646/01
26 September 2013
Paper 1 Multiple Choice
1 hour 15 minutes

Additional Materials: Multiple Choice Answer Sheet

READ THESE INSTRUCTIONS FIRST

Write in soft pencil.


Do not use staples, paper clips, highlighters, glue or correction fluid.
Write your name and Civics group on the Answer Sheet in the spaces provided.

There are forty questions in this paper. Answer all questions. For each question there are four
possible answers, A, B, C and D.
Choose the one you consider correct and record your choice in soft pencil on the separate
Answer Sheet.

Read the instructions on the Answer Sheet very carefully.

Each correct answer will score one mark. A mark will not be deducted for a wrong answer.
Any rough working should be done in this booklet.

This booklet consists of 17 printed pages.


[Turn over
2

Data

speed of light in free space, c = 3.00 x 108 m s-1

permeability of free space, o = 4 x 10-7 H m-1

permittivity of free space, o = 8.85 x 10-12 Fm-1


(1 / (36 )) x 10-9 Fm-1

elementary charge, e = 1.60 x 10-19 C

the Planck constant, h = 6.63 x 10-34 J s

unified atomic mass constant, u = 1.66 x 10-27 kg

rest mass of electron, me = 9.11 x 10-31 kg

rest mass of proton, mp = 1.67 x 10-27 kg

molar gas constant, R = 8.31 J K-1 mol-1

the Avogadro constant, NA = 6.02 x 1023 mol-1

the Boltzmann constant, k = 1.38 x 10-23 J K-1

gravitational constant, G = 6.67 x 10-11 N m2 kg-2

acceleration of free fall, g = 9.81 m s-2


3

Formulae

uniformly accelerated motion, s = ut + at2

v2 = u2 + 2as

work done on/by a gas, W = pV

hydrostatic pressure, p = gh

gravitational potential, Gm
=
r

displacement of particle in s.h.m., x = xo sin t

velocity of particle in s.h.m., v = vo cos t

=
xo x 2
2

resistors in series, R = R1 + R2 +

resistors in parallel, 1/R = 1/R1 + 1/R2 +

electric potential, V = Q / 4or

alternating current/voltage, x = xo sin t

transmission coefficient, T = exp(-2kd)

8 2 m U E
where k =
h2

radioactive decay, x = xo exp (-t)

decay constant, 0.693


=
t1
2
4

1 The resistance of an electrical component is measured. The following meter readings


are obtained.

What is the resistance?

A 2.5 k B 2.7 k C 2500 k D 2700 k

2 The radius of the Earth is 6.4 x 103 km and the volume of a grain of sand is
approximately 1 mm3.
What is the order of magnitude of the number of grains of sand that can fit in the volume
of the Earth?

A 1012 B 1018 C 1024 D 1030

3 A man pushing a mop across a floor causes it to undergo two displacements. The first
has a magnitude of 150 cm and makes an angle of 120 with the positive x-axis. The
resultant displacement has a magnitude of 140 cm and is directed at an angle of 30.0
to the positive x-axis. (Both angles are measured anti-clockwise with respect to the
positive x-axis.)

What are the magnitude and direction of the second displacement?

magnitude / cm angle to the positive x-axis


A 290 90

B 290 90
C 205 17
D 205 17

4 Two objects A and B fall from rest from different heights. A has half the mass of B. B
takes twice as long as A to reach the ground.
Neglecting air resistance, what is the ratio of the height fallen by A to that by B?

A 1:4 B 1:2 C 1: 2 D 4:1

5 A man stands on a weighing scale in a stationary lift. If sometime later, the reading on
the scale increases, the elevator could be moving with

A deceleration downward.
B increasing momentum downward.
C decreasing velocity upward.
D net force downward.
5

6 The figure below shows the variation with time t of the force F during the collision of a
58 g superball with a wall. The initial velocity of the ball is 34 m s -1 perpendicular to the
wall. The ball rebounds directly back with the same speed, also perpendicular to the
wall.

Fmax

t / ms
0 2 4 6

What is the maximum magnitude of the force Fmax on the ball by the wall during the
collision?

A 0.49 N B 490 N C 0.99 N D 990 N

7 Two masses make an elastic collision as shown below.

4.0 kg 3.0 kg

5.0 m s-1 2.0 m s-1

What are their velocities after the collision?

4.0 kg mass 3.0 kg mass


-1 -1
A 1.6 m s to the left 4.6 m s to the right
-1
B 1.6 m s to the right 4.6 m s-1 to the right
C 2.4 m s-1 to the left 5.4 m s-1 to the right
D 2.4 m s-1 to the right 5.4 m s-1 to the right
6

8 A mass is suspended by a cord from a ring which is attached by two further cords to the
ceiling and the wall as shown in the diagram. The cord from the ceiling makes an angle
of less than 45o with the vertical.

The tensions in the three cords are labeled R, S and T. How do they compare in
magnitude?

A S>T>R
B S>R>T
C R>T>S
D R>S>T

9 A cylindrical block of wood has a cross-sectional area A and weight W. It is tied to a


hanging string and then immersed totally in water. The block experiences pressures pt
and pb at its top and bottom surfaces respectively.
Which of the following expressions is equal to the tension in the string?
A pb pt A
B pb pt A
C W - pb pt A
D W + pb pt A

10 A stone of mass 0.40 kg is projected horizontally at a speed of 6.0 m s -1 from the top of
a wall, which is 5.0 m above the ground. When it arrives at the ground, its speed is
10 m s-1.

How much energy has it lost in falling through the air?

A 13 J B 12 J C 6.8 J D 0.4 J
7

11 A mass m, attached to the end of an unstretched spring, is initially supported by a


platform as shown in Fig. (a). This platform is then removed and the mass falls,
eventually coming to rest at the position shown in Fig. (b).

unstretched
spring

platform

Fig. (a) Fig. (b)


Which of the following correctly relates the changes in energy which may occur during
this process?

A decrease of gravitational potential energy = increase of elastic potential energy

increase of elastic potential energy


B decrease of gravitational potential energy = + energy dissipated as heat

decrease of gravitational potential energy = increase of elastic potential energy


C
+ energy dissipated as heat

decrease of gravitational potential energy = energy dissipated as heat


D
+ increase of elastic potential energy

12 Two objects of masses 5.0 kg and 8.0 kg are fixed on a horizontal circular rotating
platform at 2.0 m and 6.0 m from the centre respectively.

What is the ratio of the net force on the 5.0 kg mass to that on the 8.0 kg mass?

A 5 : 24 B 24 : 5 C 8 : 15 D 15 : 8
8

13 A bob that is tied to a fixed set of brass weights is made to execute circular motion in a
horizontal plane, so that the inelastic string traces out a cone, as shown in the diagram
below. The string is passed through a smooth vertical glass tube so that the length L of
the string from the top of the glass tube to the bob can vary freely as the speed of the
bob changes.

bob

brass
weights
What is the relationship between this length L and the frequency f of the circular
motion?

A L f2 B L f

1 1
C L D L
f f2

14 An astronaut in an artificial satellite orbiting the Earth at a steady speed can be


regarded as weightless because

A the gravitational force acting on him is zero.


B the centripetal force experienced by him is zero.
C the centripetal force is balanced by the gravitational force acting on him.
D his acceleration is the same as that of the satellite.

15 A stationary object of mass m is released at a point P on the line joining the centres of
the two planets A and B. The gravitational potential at several points along the line
joining the centres of the two planets are shown in the diagram below. The object will
accelerate towards one of the planets.

planet A planet B
P

gravitational -62.3 -13.4 -3.9


potential / 106 J kg-1

What is the speed of the object just before it hits the surface of the planet?

A 4.36 x 103 m s-1 B 5.18 x 103 m s-1


C 9.89 x 103 m s-1 D 1.08 x 103 m s-1
9

16 The graphs below show the variation with time t of the velocity v and the acceleration a
of a body when it is oscillating with simple harmonic motion.

What is the value of T ?

2 2
A s B s C s D s
9 9 3 3

17 The graph below shows the way the potential energy of a body varies with its
displacement from a point Z.

Which feature of the graph means that the force on the body is directed towards Z?

A The graph passes through the origin.


B The value of the potential energy is always positive.
C The potential energy increases with displacement from Z.
D The slope of the graph increases with displacement from Z.

18 Which thermodynamic temperature is equivalent to 501.85 oC?

A 775.00 K B 775.01 K C 228.69 K D 228.70 K


10

19 Three insulated gas containers X, Y and Z containing the same ideal gas are connected
by a tube of negligible volume, as shown below.

tap and tube of negligible volume

X Y Z

Each container has an internal volume of 5.0 m3. Initially, the 2 taps are closed and
container X is held at a temperature of 300 K and a pressure of 1.2 x 105 Pa. Container
Y is held at a temperature of 320 K and a pressure of 1.5 x 105 Pa. Container Z is held
at a temperature of 303 K and a pressure of 1.0 x 105 Pa.

Given that the final temperature of the gas is 308 K after the taps are open, determine
the final equilibrium pressure of the gas in the containers.

A 1.09 x 105 Pa B 1.23 x 105 Pa C 1.26 x 105 Pa D 1.54 x 105 Pa

20 A piston is pushed into a cylinder containing an ideal gas such that the pressure of the
gas increases to 1.5 times its initial value while the volume decreases to half its original
value.

Which of the following statements is false?

A The average kinetic energy of the gas molecules decreases.


B Heat is absorbed by the gas.
C Work is done on the gas by its surroundings.
D The internal energy of the gas decreases.

21 The diagram shows a cathode-ray oscilloscope display of an electromagnetic wave.

The time base setting is 0.20 s cm1.

What is the wavelength of the electromagnetic wave?

A 2.0 cm B 60 m C 120 m D 600 m


11

22 The diagram represents a stationary wave formed by the superposition of sound waves
from a loudspeaker and their reflection from a metal sheet (not shown).

W, X, Y and Z are four points on the line through the centre of this wave.

Which statement about this stationary wave is correct?

A A displacement antinode is formed at the surface of the metal sheet.

B A node is a quarter of a wavelength from an adjacent antinode.

C The oscillations at X are in phase with those at Y.

D The particles of the waves oscillate at right angles to the line WZ.

23 In a Youngs double slit experiment, the ratio of the intensities between the bright and
the dark fringes is 9.

What is the ratio of the amplitudes of the waves from the two slits?

A 2 B 3 C 4 D 9

24 A spectrum of the Sun's radiation in the infrared region is produced by a diffraction


grating. Light of a particular wavelength is being studied and the angle between its two
first-order maxima is 50.0 as shown below.

grating

light of unknown 50.0


wavelength

Given that the angle between the two fourth-order maxima of the hydrogen line of
wavelength 656.3 nm is 60.0, what is the wavelength of the light being studied?

A 5.55 x 10-7 m B 2.22 x 10-6 m C 3.11 x 10-6 m D 1.39 x 10-7 m


12

25 An electron moves from point P to point Q in the opposite direction to a uniform electric
field. During this displacement
A the work done by the field is negative and the potential energy of the electron field
system increases.
B the work done by the field is negative and the potential energy of the electron field
system decreases.
C the work done by the field is positive and the potential energy of the electron field
system increases.
D the work done by the field is positive and the potential energy of the electron field
system decreases.

26 A student is given a circuit consisting of 2 ideal diodes, an ohmic resistor and a filament
lamp as shown below. The variable power supply can provide both positive and
negative voltages across PQ.

P Q

Which one of the following graphs best represents the I-V characteristics across PQ?
A I B I

V V

C I D
I

V V
13

27 Four circuits P, Q, R and S are shown below. In each circuit, the cell has the same
e.m.f. E. Each of the lamps has a resistance of 2 .

E 1 E 1

P Q

E 3 E 3

R S
Which of the following gives the combinations in the order of decreasing brightness?
A QPRS B SRPQ
C QSPR D RPSQ

28 In the diagram below, side X of a conductor has twice the diameter as compared to side
Y.

IX IY
Y
X

What is the ratio of the current IX to IY?


A 0.5 B 1
C 2 D 4
14

29 Four resistors are connected as shown in the circuit below. R1 and R4 each has a
resistance of 2r, while R2 and R3 each has a resistance of r.

R1 R3

E V

R2 R4

The reading on the voltmeter will be increased by adding another resistor of value r in
parallel with
A the voltmeter B R1
C R2 D R4

30 A beam of electrons enters a region in which there are magnetic and electric fields
directed at right angles to each other and to the beam direction, as shown below. The
electron beam passes straight through without deflection.

electron
beam
magnetic field

electric field

A second beam of alpha particles travelling at half the speed of the first beam of
electrons is then directed along the same line.

How is this second beam deviated?


A Upwards in the plane of paper
B Downwards in the plane of the paper
C Out of the plane of the paper
D Into the plane of the paper
15

31 A long straight wire XY lies in the same plane as a square loop of wire PQRS which is
free to move. The sides PS and QR are initially parallel to XY.

Q R

P S
X Y

Both the wire and loop carry equal sinusoidal alternating currents. The currents in XY
and side QR differ in phase by rad.
Which one of the following graphs shows the variation with time t of the resultant force
of attraction F between the wire and the loop?

A B
F F

0 t 0 t

C D
F F

(zero at all times)


0 t 0 t

32 A copper disc of area A rotates at angular frequency at the centre of a long solenoid
of length L and having N turns. The solenoid carries a current I. The plane of the disc is
normal to the magnetic flux. The rotation rate is adjusted so that the e.m.f. generated
between the centre of the copper disc and its rim is 10% of the potential difference
across the ends of the solenoid.
(You may assume that the magnetic flux density B at the centre of the solenoid is given
by the expression B = onI.)
Which expression gives the potential difference across the ends of the solenoid?

A 10oNIA
B 1.6oNIA /L
C 0.1oNIA
D 0.016oNIA /L
16

33 A flat circular coil of N turns, each of area A, is connected to a galvanometer of


negligible resistance. The coil is made from a long wire of diameter d. The coil is placed
with its plane normal to a uniform magnetic field B. The flux density of the field is then
steadily reduced to zero in time t. As a result of this operation, a charge flows in the
circuit.
Which two quantities, when increased, have no effect on the amount of charge flowing
in the circuit?
A A, N B B, d
C d, t D N, t

34 The potential difference V1 shown in Fig. (a) applied across a resistor R produces heat
at a mean rate W.

V1 V2

Fig. (a) Fig. (b)

What is the mean rate of production of heat when the potential difference V2 shown in
Fig. (b) is applied across the same resistor?

A W/2 B W C 2W D 2W

35 A generator produces a current of 60 A at a voltage of 120 V. The voltage is stepped up


to 4500 V by an ideal transformer and transmitted through a power line of total
resistance 1.0 .
What is the percentage of the power generated that is lost in the transmission line?

A 0.018 % B 0.024 % C 0.036 % D 0.048%


17

36 In a photoelectric experiment, the potential difference V that must be maintained


between the illuminated surface and the collector so as just to prevent any electrons
from reaching the collector is determined for different frequencies f of the incident
illumination. The graph below is obtained.

V1

f
f0 f1
The Planck constant is given by

V1 V1 eV1 f1 f0
A B C D
f1 f1 f0 f1 f0 eV1

37 The de Broglie wavelength of an electron that is accelerated from rest through a


potential difference of V is given by . The wavelength is proportional to

A V 1 2 B V12 C V D V2

38 A p-n junction is formed when a p-type semiconductor is in contact with an n-type


semiconductor. The most important property of a p-n junction is that it rectifies an
alternating current. Which of the following statements is false?

A During reverse bias condition, the p-type semiconductor is connected to the


negative terminal of an externally applied p.d.

B During reverse bias condition, the width of the depletion region becomes larger as
the charge carriers are drawn away from it.

C During forward bias condition, if the applied p.d. overcomes the junction potential,
electrons will cross steadily from the p-type side to the n-type side while the holes
will cross steadily in the opposite direction.

D During forward bias condition, current increases with p.d.

39 The mass of a beryllium nucleus ( 7 Be ) is 7.01473 u. The mass of a proton is 1.00728 u


4
while the mass of a neutron is 1.00867 u.

What is the binding energy per nucleon of this nucleus?

A 1.6 MeV B 5.4 MeV C 9.4 MeV D 12.5 MeV

40 The probability that a sodium-25 nucleus will decay in a time of one hour is 4.61%.

The time that a sample of sodium-25 will take to decay to one-quarter of its initial
amount is

A 30.0 hours B 16.3 hours C 6.4 hours D 5.4 hours


1

TEMASEK JUNIOR COLLEGE


2013 Preliminary Examination
Higher 2
CANDIDATE
NAME

CIVICS INDEX
GROUP NUMBER

PHYSICS 9646/02
Paper 2 Structured Questions 16 September 2013
1 hour 45 minutes
Candidates answer on the Question Paper.
No Additional Materials are required.

READ THESE INSTRUCTIONS FIRST

Write your Civics group, index number and name on all the work you hand in.
Write in dark blue or black pen on both sides of the paper.
You may use a soft pencil for any diagrams, graphs or rough working.
Do not use staples, paper clips, highlighters, glue or correction fluid.

For Examiners Use


Answer all questions.
1
At the end of the examination, fasten all your work securely 2
together. The number of marks is given in brackets [ ] at the
end of each question or part question. 3
4
5
6
Total
7

This booklet consists of 18 printed pages.


2

Data

speed of light in free space, c = 3.00 x 108 m s-1

permeability of free space, o = 4 x 10-7 H m-1

permittivity of free space, o = 8.85 x 10-12 Fm-1


(1 / (36 )) x 10-9 Fm-1

elementary charge, e = 1.60 x 10-19 C

the Planck constant, h = 6.63 x 10-34 J s

unified atomic mass constant, u = 1.66 x 10-27 kg

rest mass of electron, me = 9.11 x 10-31 kg

rest mass of proton, mp = 1.67 x 10-27 kg

molar gas constant, R = 8.31 J K-1 mol-1

the Avogadro constant, NA = 6.02 x 1023 mol-1

the Boltzmann constant, k = 1.38 x 10-23 J K-1

gravitational constant, G = 6.67 x 10-11 N m2 kg-2

acceleration of free fall, g = 9.81 m s-2


3

Formulae

uniformly accelerated motion, s = ut + at2

v2 = u2 + 2as

work done on/by a gas, W = pV

hydrostatic pressure, p = gh

gravitational potential, Gm
=
r

displacement of particle in s.h.m., x = xo sin t

velocity of particle in s.h.m., v = vo cos t

= x o
2
x2
resistors in series, R = R1 + R2 +

resistors in parallel, 1/R = 1/R1 + 1/R2 +

electric potential, V = Q / 4or

alternating current/voltage, x = xo sin t

transmission coefficient, T = exp(-2kd)

8 2 m U E
where k =
h2

radioactive decay, x = xo exp (-t)

decay constant, 0.693


=
t1
2
4

1 A mass of (24 2) g of ice at 15 C is taken from a freezer and placed in a beaker


containing 200 g of water at 28 C. Data for ice and for water are given in Fig. 1.1.

ice water
specific heat capacity / J kg1K1 2.1 x 103 4.2 x 103
specific latent heat of fusion / J kg1 3.3 x 105 -

Fig. 1.1

(a) Calculate the quantity of thermal energy required to convert the ice at 15 C to water
at 0 C. Express your answer with its associated uncertainty.

energy required = J [4]

(b) Assuming that the beaker has negligible mass, calculate the final temperature of the
water in the beaker.

final temperature = C [3]


5

2 A ball bearing, initially at rest, slides down a smooth slope, inclined at an angle of 37o to the
horizontal as shown in Fig. 2.1. It travels 5.0 m along the slope before falling freely off the
edge E.

ball bearing

37o
E

ground G

Fig. 2.1

(a) Calculate the speed of the ball when it reaches the edge E of the slope.

speed = m s-1 [2]

(b) The time taken for the ball to reach the ground G after leaving the edge of the slope is
0.85 s. Show that the height H of the edge of the slope above the ground is 7.5 m.

[2]
6

(c) Taking reference from edge E, sketch on Fig. 2.2 and Fig. 2.3 the variation with time of
the horizontal displacement x and the vertical displacement y of the ball during its flight
from E to G. Label the graphs P and Q respectively. (Indicate appropriate numerical
values but no further calculation is required.)

x/m

0 t/s

Fig. 2.2

y/m

0 t/s

Fig. 2.3 [3]

(d) Sketch on Fig. 2.2 and Fig. 2.3 the corresponding variation with time of the horizontal
and vertical displacements of the ball, if air resistance is not negligible. Label the
graphs R and S respectively. [2]
7

3 A simplified drawing of a suspension bridge is shown in Fig. 3.1. The uniform bridge AB itself
has a weight W and is supported at its edges A and B as well as by cables at C and D. The
length of the bridge is 3d where AC = CD = DB = d. A tie joining the top of the pillar to the
ground at E and another at F hold the pillars in a vertical position.

Fig. 3.1
(a) State the conditions necessary for a body to be in equilibrium.

[2]

(b) (i) W
If the reaction on the bridge at each of the supports A and B is vertically
4
upwards, find the tension, in terms of W, in the tie.

tension in the tie = [4]


8

(ii) State and explain the advantage of using a cable and a tie to hold the pillar.

[2]

4 (a) Two coherent light wavetrains having the same plane of polarization meet at a point.
State two conditions that must be fulfilled before totally destructive interference can
occur.

1.

2.

[2]

(b) Fig. 4.1 shows an experiment to demonstrate interference effects with microwaves. A
transmitter, producing microwaves of wavelength, is placed in front of two slits
separated by a distance a. A receiver is used to detect the strength of the resultant
wave at different points along the line YZ which is at a distance D in front of the slits.

Fig. 4.1
9

(i) Explain, in terms of the path difference between the wavetrains emerging from
the slits S1 and S2, why a series of interference maxima are produced along the
line YZ.

[2]

(ii) Predict how the distance x between neighbouring maxima on the line YZ would
change if the distance a was doubled while the distance D was halved.

[1]

(iii) Explain why it is necessary to use a barrier with two slits rather than two
separate transmitters.

[1]

(iv) In another experiment using the apparatus in Fig. 4.1, a student notices that the
distances between the maxima are not equal. Suggest a reason for this
difference.

[1]

(v) Describe how you could test whether the microwaves leaving the transmitter
were plane polarised.

[2]
10

(c) The microwave transmitter is now placed in front of a plane reflector as shown in
Fig. 4.2 and stationary waves are set up in the space between them.

transmitter

detector reflector

Fig. 4.2

A detector is moved between the transmitter and the reflector at a constant speed of 10
mm s1. The frequency of detection of minima is 1.5 Hz.

Determine the frequency of the microwave oscillator.

frequency = Hz [3]
11

5 (a) LASER stands for Light Amplification by Stimulated Emission of Radiation. What do
you understand by the terms in italic?

Stimulated Emission:

[1]

Light Amplification:

[2]

(b) Using band theory, distinguish between the conduction properties of metals and
intrinsic semiconductors under the application of an electric field.

[4]
12

6 Radioactive decay is the spontaneous and random emission of radiation from a radioactive
52Te decays by emission to form 53 I . 53 I is not stable, and
source. Radioactive nuclide 131 131 131

131
decays by emission to the stable isotope 54 Xe . The half-life for this second decay is
very much longer than that for the decay of 131 131
52Te . A sample of pure 52Te is prepared at

time t 0 . Fig 6.1 shows the activity A of the sample over a period of 10 hours.

t/hr A/Bq
0 1.00 x 1012
1 1.94 x 1011
2 3.95 x 1010
3 1.08 x 1010
4 2.66 x 109
5 2.27 x 109
6 2.12 x 109
7 2.12 x 109
8 2.12 x 109
9 2.12 x 109
10 2.12 x 109

Fig. 6.1

(a) Explain the meaning of the following terms.

spontaneous:

half-life:

[2]
13

(b) The relation between activity A and time t follows the expression

A A0 e t

where A0 is the initial activity and is a constant.

Data from Fig. 6.1 are used to obtain values for ln A. The variation with time t of ln A is
plotted on the graph of Fig 6.2.

ln (A/ Bq)

27

26

25

24

23

22

21
0 1 2 3 4 5 6 7 8 9 10 t/ hr

Fig. 6.2

(i) Determine ln A for t = 3 hr.

ln A [1]

(ii) On Fig. 6.2,

1. plot the point corresponding to t = 3 hr,


2. draw the line of best fit for all the points. [3]
14

(iii) Explain the shape of the line drawn in (ii).

[2]

(iv) Using the line drawn in (ii), show that the decay constant of 131
Te is
52

approximately 1.5 hr-1 .

[2]

(v) Hence, or otherwise, deduce the initial number of 131


Te isotopes.
52

initial number of 131


Te isotopes =
52
[2]

(vi) Explain why the amount of 131


53 I in the sample first increases and then decreases.

[2]
15

(vii) If we can assume that all the 131


52Te has been converted to
131
I after about 6 hours,
53
131
calculate the half-life of I.
53

[3]
16

7 When light is incident on the front of a photocell, an e.m.f. is generated in the photocell. A
student wishes to investigate the effect of adding various thicknesses of glass in front of a
photocell. This may be carried out in the laboratory by varying the number of identical thin
glass sheets between a light source and the front of the photocell.

It is suggested that the e.m.f. V is related to the number n of glass sheets by the equation
nt
V Voe
where t is the thickness of one sheet, is the absorption coefficient of glass and Vo is the
e.m.f. for n = 0.

You are provided with a photocell. You may also use any of other equipment usually found
in a Physics laboratory.

Design an experiment to determine the value of .

You should draw a labelled diagram to show the arrangement of your apparatus. In your
account you should pay particular attention to

(a) the identification and control of variables,


(b) the equipment you would use,
(c) the procedure to be followed,
(d) how the e.m.f. generated and the thickness of the glass would be determined,
(e) any precautions that would be taken to improve the accuracy and safety of the
experiment. [12]

Diagram
17

18

TJC 2013 PHYSICS H2 PRELIMINARY EXAM SOLUTIONS

PAPER 1

1 2 3 4 5 6 7 8 9 10 11 12 13 14 15 16 17 18 19 20
C D C A A D D B C C B A D D C D C A B B
21 22 23 24 25 26 27 28 29 30 31 32 33 34 35 36 37 38 39 40
C B A B D D A B C B A B D B C C A C B A

PAPER 2

1 (a) Energy required to warm ice = m c = 24 10-3 2.1 103 15 = 756 J


Energy required to melt ice at 0 C = m Lv = 24 10-3 3.3 105 = 7920 J [1]
Total energy E = 8680 J [1]

E = m (c + Lv )

E m 2
0.0833 [1]
E m 24

E = 723 700 J (round off to 1 sig. fig.)


[1]
Therefore, E = (8700 700) J (value of E rounded off to nearest 100).

(b) Energy lost by warm water = 200 10-3 4.2 103 (28 - T) [1]
200 4.2 (28 - T) = 24 4.2 T + 8680 [1]
T = 16 C [1]
2 (a) v2 = u2 + 2as
= 0 + 2 9.81 sin 37o 5.0 [1]
v = 7.68 m s-1 [1]

(b) s = ut + at2 [1]


= 7.68 sin 37o 0.85 + 9.81 0.852 [1]
= 7.5 m

(c)
x/m y/m
(d)
7.5
P Q
S
R

0 t/s 0 t/s
0.85 0.85

Fig. 2.2 Fig. 2.3


2 marks - correct shape of P and Q with appropriate numerical values indicated
1 mark - gradient of y versus t graph 0 at t = 0
1 mark - correct shape of both R and S
1 mark - showing R extends beyond 0.85 s and Q & S end at 7.5 m

3 (a) Resultant force acting on the body in any direction is zero.


Resultant moment on the body about any axis is zero. [2]

(b) (i) The net force acting vertically on the bridge is zero.
where T is the tension in the cable [1]

[1]

The net force acting horizontally on the pillar is zero.


where TT is the tension in the tie [1]

[1]

(ii) The vertical component of the tensions in the cable and tie pin the pillar
to the ground. Hence it will not be easily pushed to either side by any [1]
sideway force.
The tensions in the tie and cable provide moments about the base of the
pillar which tend to cancel out, keeping the pillar upright. [1]
4 (a) (Totally destructive interference means that the resultant wave has zero
amplitude or waves cancel each other.)
Waves must meet rad out of phase. [1]
Waves must have equal amplitude. [1]

(b) (i) Wavetrains from S1 and S2 are coherent and superpose at points along [1]
YZ.
When path difference is an integral multiple of , the waves are in phase,
constructive interference takes place to give a series of maxima. [1]
(iii) It decreased to one quarter of the original x (since x = D/a) [1]
(iii) So that the wavetrains are coherent. [1]

(iv) The line YZ is not parallel to the slits [1]


or the slits not normal to the (incident) microwaves
(v) Place a metal grid or polariser in front of the transmitter and rotate [1]
through 90o OR rotate transmitter/detector through 90o.
If this causes minimal/zero signal at some angles, the wave is plane [1]
polarized.

(c) Distance between two nodes = speed of detector / frequency of detection


= 10 / 1.5 = 6.7 mm [1]
Hence, wavelength = 13 mm [1]
f = c/ 8
/ 13 x 10-3 = 2.3 x 1010 Hz. [1]

5 (a) Stimulated Emission:


Process whereby an incident photon (of energy equal to the energy difference [1]
between the excited state and ground state) causes an excited atom to return to
ground state with the emission of an additional photon.
(The emitted photon has same energy, phase, frequency and travels in the same
direction as the incident photon.)

Light Amplification:
The emitted photons stimulate further emission from other excited atoms. [1]
The reflecting mirrors placed at 2 ends of the laser system help to confine and [1]
enhance this process.
(The resultant beam is an intense coherent monochromatic light.)

(b) - In metals, the conduction band is partially filled, electrons can jump easily into [1]
higher unoccupied energy levels.
- When a potential difference is applied, the electrons can accelerate and [1]
increase their energy. Hence a current flows readily.
- In intrinsic semiconductors, the unfilled conduction band is separated from the [1]
filled valence band by a small energy gap (of about 1 eV). When temperature is
high enough, electron in the valence band can gain energy and cross the small
energy gap into the conduction band, leaving holes.
- The number of free charge carriers available for electrical conduction is smaller [1]
than in the case of metal. When a potential difference is applied, these free
charges carriers accelerate and constitute a smaller current.
6 (a) Spontaneous means that the decay process occurs on its own accord and cannot be
speeded up or slowed down by physical means such as changes in pressure and
temperature and are unaffected by chemical combination. [1]

Half-life of a radioactive element is the average time taken for half the original number
of the radioative nuclei to decay. [1]

ln (A/ Bq)

27

26

25

24

23

22

[3]
21
0 1 2 3 4 5 6 7 8 9 10 t/ hr

(c) (i) ln A = 23.103 [1]

(iii) From 0 to 3 hr, the graph is linear with a negative slope. This is because the
activity is predominantly that of 131 131
52Te as there are only small amounts of 53 I . (As

the activity of 131


53 I nuclei grows, that of
131
Te decreases. Hence the curve from 3 to 6
52
131
hrs.) From 6 to 10 hrs, the line is horizontal due mainly to the activity of 53I which
131
has a very long half-life as compare to 52 Te . [2]

(iv) 27.60 23.40 4.20


gradient 1.50 hr 1 [2]
0.00 2.80 2.80
(v) The equation of the graph is ln A ln A0 t .
Hence, 1.50 hr 1 4.17 10 4 s
1.00 1012
Ao
No 2.40 1015
4.17 10 4

131
(vi) Initially the rate of production of I is much higher than the rate of its decay hence
53
131
the amount increases. Later, most of the 52Te would have decayed hence the rate
131
of production of I drops. The amount will then decrease due to its decay to
53
131
54 Xe . [2]

(vii) A N
ln 2
2.12 109 (2.40 1015 )
t1
2
5
t 1 7.85 10 s
2
[3]
7

SUGGESTED MARK SCHEME:

Defining the problem


Independent variable: n 1
Dependent variable: voltage V
Control of variables: keep distance from light to photocell/ intensity of light constant
(Allow constant voltage across lamp/current through lamp/brightness. Do not allow same 1
lamp/same output)
Method of data collection
Fully-labelled diagram including lamp, glass sheet and photocell in line. Voltmeter connected 1
to photocell. Penalise unworkable photocell circuit.
Method of measuring thickness of glass sheet (micro meter screw-gauge) 1
Take many readings of thickness at different points of glass and find the average thickness 1
Method of measuring voltage (voltmeter)
Method of varying n (vary number of glass) 1
1

Method of analysis
Suggest an appropriate graph (V Voe nt lnV lnVo nt. Plot a graph of ln V against n. 1
Allow lnV against nt)
1
= gradient/t (For graph of ln V against nt, then = gradient)
Safety considerations
Wear gloves to prevent burns from hot source/cut from glass 1
Shield lamp/wear dark glasses/do not look at source directly to prevent eye damage from
bright/intense source
Additional details
Use small distance/high intensity source to gain large reading 2
Clean sheets of glass before use
Check that output of lamp is constant (measure current I through/p.d. V across lamp and
calculate P = VI)
Regularly check Vo in absence of glass
Ensure constant output of lamp with variable resistor or variable power supply
Direction of light is perpendicular to glass sheets/constant orientation
Total 12
PAPER 3

Section A

1 (a) Total momentum is conserved since no external force acts on the system. [1]
total initial momentum of the system = total momentum of the system at t1
mvo + 0 = m(0.5vo) + momentum of nucleus at t1 [1]
momentum of nucleus at t1 = 0.5mvo [1]

(b) (i) Both are moving with the same velocity. [1]

(ii) By conservation of momentum,


total initial momentum of the system = total momentum of the system at t2
mvo = mv + Mv [1]
m
v v0 [1]
m M

(c)

Argument 1
The -particle and the gold nucleus may be considered to be nearly at rest and so the
assumptions are valid.

Argument 2
The speeds of the -particle and the gold nucleus are non-zero (and could be large if
vo is large) and so the assumptions are not valid.
[2]
(d) Argument 1
As the -particle approaches the nucleus, the kinetic energy of the -particle is
converted into electrical potential energy (due to repulsion between like charges).
When they separate, the potential energy is converted back to kinetic energy of the -
particle and the nucleus. The kinetic energy of the system remains constant before
and after the interaction. Hence the interaction can be considered as an elastic
collision.

Argument 2
When the -particle approaches the nucleus, it decelerates and emits electromagnetic
radiation. Therefore the kinetic energy of the system decreases after the interaction.
Hence the interaction is not an elastic collision.

[Statements such as The relative speed of approach is not equal to the relative speed
of separation and so kinetic energy is not conserved. Hence interaction is not elastic.
will not be awarded any marks because the discussion is either not conclusive or
complete.]
[2]
2 (a) The internal energy of a system is a function of its state. The increase in the
internal energy of a system is equal to the sum of the heat absorbed by the
system and the work done on the system. [2]

(b) Net work done by the system can be obtained from the area enclosed by the [1]
cycle abcda.

(c) In one complete cycle, U = 0 (since it returns to the original state) and [1]
the net work done on the gas is negative. [1]
From the first law of thermodynamics,
U = Q + W = 0
Q= -W
Hence Q is positive. [1]
(d) The gas must expand very slowly so that there is time for heat to flow into the [1]
cylinder, to maintain constant temperature.
[1]
The walls of the container must be a good conductor of heat.

3 (a)

(b)

[1]
v

(b) The gravitational field strength at a point is defined as the gravitational force per [1]
unit mass at that point.
(c) GM
g=
r2
6.67 1011 4.0 1030

(2.0 1011 ) 2 [2]
3 2
6.67 10 ms

(c) mv 2
mg =
r
3 v2
6.67 10
1.0 1011
v 2.58 104 ms 1 [2]

(d) v 2

r T
2.58 104 2

1.0 1011 T
7
T 2.4 10 s [2]

4 (a) (i) (For the oil droplet to remain stationary anywhere between the plates, the
electric force must balance the weight.)
The plates are parallel to provide an uniform electric field so that the electric
force is constant. [1]
They are horizontal so that the electric force acts vertically to balance the weight.
[1]

(ii)
q = mg

q = 7.7 10-15 9.81 [1]

q = 4.8 10-19 C [1]

Sign of the charge is negative. [1]

(b) The charge on the electron is 1.6 10-19 C. Any charge must be an integral multiple of
the electronic charge (or charge is quantized) (or 1.6 x 10-19 is the lo\west common
factor) [1]
5 (a) It gives the minimum value of the negative potential that will stop the most
energetic electrons from reaching the electrode (anode/collector) [1]

(b) - At a given intensity, the number of photons that arrive per unit time at the
metal surface is fixed, hence the number of photoelectrons emitted per unit time
is also fixed.
- Above a certain positive/accelerating potential, all the photoelectrons emitted
will be collected at the electrode. Hence, photocurrent current saturates at a
given intensity. [2]

(c) (i)

0.28 x 10-3 = (1.60 x 10-19)

No. of photons = [2]

(ii) (Minimum momentum of photon corresponds to energy that is just enough


to overcome the work function for metal)

[3]
Section B

6 (a) (i) The oscillations of the magnet are lightly damped. The amplitude decreases with
time. [1] As the magnet moves, magnetic flux is cut by the coil. From Faradays
law, an e.m.f. is induced in the coil. [1] Since the circuit is closed, a current flows.
[1]
From Lenzs law, the induced current produces a magnetic force on the magnet
opposing its motion. [1]
(OR From Lenzs law, the electrical energy, which is dissipated in the load as
heat, is derived from the mechanical energy of the magnet.)

(ii) T = 0.60 s (2 d.p.) [1]


2 2
o = = 10.5 rad s-1 [1]
T 0.60

(iii) 6
a = o2x = (10.5) 2 D = 44 D [2]
15
direction of a: downward [1]

(iv) initial total energy Ei = mo2D2


3
at t = 1.2 s, total energy Ef = mo2 ( 15 D)2 [1]

2
E E i E f E D 3
fraction = = 1 f 1 f = 1 - ( 15 )2 = 0.96 [1]
E Ei Ei Di

(v) Work done W = mean force distance moved


For oscillation, distance moved = 15 10-3 m
Average power = W / t = 2.5 15 10-3 / (0.60/4) = 0.25 W [2]

(b) less damping


larger amplitude [1] with period unchanged [1]
(c) (i)

Peaked curve [1]


Peak at o and amplitude
never zero [1]

(ii) Resonance [1]

(iii) Useful [1]


Cooking: microwaves cause water molecules to resonate thereby heating food
MRI: radio waves (in a magnetic field) cause nuclei/proton to resonate
Person on swing: intermittent pushes cause swing to go higher

Problem [1]
Bridges: wind/walking in step causes bridge to resonate
Vehicles: engine vibrations cause panels/mirrors to resonate
Earthquakes: ground vibrating causes buildings to resonate

7 (a) Electromotive force of a cell is the chemical energy converted into electrical
energy per unit charge passing through it.
Potential difference across the cell is the difference between electromotive force
and the amount of electrical energy converted to heat per unit charge passing
through the internal resistance of the cell. [2]

(b) (i) I

V [1]
(ii) - Draw a straight line of gradient = 1/x passing through the origin.
- Determine the value of I and V in which the straight line intersect.
- Power of the filament is given by product of I and V. [3]

(iii) 1.

I = 1.0 A
[3]

2. 9.0 8.0 = I r
r = 1.0 [2]

(c) The magnetic flux density at a point is defined as the force acting per unit current
per unit length of the conductor when the conductor is placed at right angles to
the magnetic field. Unit: Tesla (T) [3]

(d) (i) Direction of magnetic field in the vicinity of B is acting to the right. [1]

(ii) - aqueous sodium chloride conducts electricity and a current flows from B to
A, this current in the presence of a magnetic field produces a magnetic
force on AB.
- direction of the force, given by Flemings Left hand rule, is perpendicular to
both the current and magnetic field and acts into the plane of paper. Hence
wire AB rotates anticlockwise. [3]

(iii) When S2 is closed, the resistance of the circuit is lowered as resistor R is


bypassed. Current flowing in the circuit is increased. Hence the force acting
on AB is increased causing it to rotate at a higher speed. [2]
227
Ac 223 4 [1]
87 Fr 2 He
8 (a) (i)
89

223
Fr 223 0 [1]
87 88 Ra 1 e

(ii) m (227.0278 223.0198 4.0026)u


[1]
5.4 103 u

Kinetic energy released


= m c2
= 5.4 x 10-3 x 1.66 x10-27 x (3.00 x 108)2
= 8.07 x 10-13 J
= 5.04 MeV [2]

(iii) 1 Mass defect represents the loss in mass due to the release of energy in
assembling the atom from the individual protons, neutrons and
electrons.
[2]

2 mass defect = m
= [(87 x 1.0073) + (136 x 1.0087) + (87 x 0.0005) 223.0198] u

Binding energy
= m x c2
= 2.75 x 10-10 J
= 1.72 x 103 MeV [2]

3
Eb Ra
/ MeV x Ac
A x x
- position of Ra higher than
Fr and position of Ac lower Fr
than Fr [1 mark]

- shape of graph correct,


graph cannot start from 0
and labelling of mass A
number at 223 for Ra and I I
Fr and 227 for Ac 223 227
respectively [1 mark]
[2]

(b) (i) Count rate detected by a detector is the number of pulses produced per unit time by
an ionizing radiation and background radiation. [1]
Whereas activity is the number of parent nuclei which decays per unit time. [1]
(ii) 1. 200.8
16
1 1.2 1 1 [2]
fraction =
2 2 65536
2. C 1

C0 65536
1
C 3.7 1010 5.64 105 s 1
65536

C
Count rate per unit area =
4r 2
5.64 105
112s 1cm2
4 20 2 [3]

3. Since the fraction of photons which passes through the lead decreases
with increasing distance from the source, more photons are absorbed in
the region nearer the source.
Hence greater heating effect is likely to be experienced nearer the [2]
source.
TEMASEK JUNIOR COLLEGE
2013 Preliminary Examination
Higher 2
CANDIDATE
NAME

CIVICS INDEX
GROUP NUMBER

PHYSICS 9646/03
Paper 3 Longer Structured Questions 19 September 2013
2 hours
Candidates answer on the Question Paper.
No Additional Materials are required.

READ THESE INSTRUCTIONS FIRST

Write your Civics group, index number and name on all the work you hand in.
Write in dark blue or black pen on both sides of the paper.
You may use a soft pencil for any diagrams, graphs or rough working.
Do not use staples, paper clips, highlighters, glue or correction fluid.

For Examiners Use


Section A
Answer all questions. 1
2
Section B
Answer any two questions. 3
4
You are advised to spend about one hour on each section.
5
At the end of the examination, fasten all your work securely 6
together. The number of marks is given in brackets [ ] at the
7
end of each question or part question.
8
Total

This booklet consists of 23 printed pages.


2

Data

speed of light in free space, c = 3.00 x 108 m s-1

permeability of free space, o = 4 x 10-7 H m-1

permittivity of free space, o = 8.85 x 10-12 Fm-1


(1 / (36 )) x 10-9 Fm-1

elementary charge, e = 1.60 x 10-19 C

the Planck constant, h = 6.63 x 10-34 J s

unified atomic mass constant, u = 1.66 x 10-27 kg

rest mass of electron, me = 9.11 x 10-31 kg

rest mass of proton, mp = 1.67 x 10-27 kg

molar gas constant, R = 8.31 J K-1 mol-1

the Avogadro constant, NA = 6.02 x 1023 mol-1

the Boltzmann constant, k = 1.38 x 10-23 J K-1

gravitational constant, G = 6.67 x 10-11 N m2 kg-2

acceleration of free fall, g = 9.81 m s-2


3

Formulae

uniformly accelerated motion, s = ut + at2

v2 = u2 + 2as

work done on/by a gas, W = pV

hydrostatic pressure, p = gh

gravitational potential, Gm
=
r

displacement of particle in s.h.m., x = xo sin t

velocity of particle in s.h.m., v = vo cos t

= x o
2
x2
resistors in series, R = R1 + R2 +

resistors in parallel, 1/R = 1/R1 + 1/R2 +

electric potential, V = Q / 4or

alternating current/voltage, x = xo sin t

transmission coefficient, T = exp(-2kd)

8 2 m U E
where k =
h2

radioactive decay, x = xo exp (-t)

decay constant, 0.693


=
t1
2
4

Section A

Answer all the questions in this section.

1 An -particle of mass m and initial velocity v0 is directed head-on towards a stationary


heavy nucleus of mass M. Assume that their initial separation is very large. The variation
with time of the velocities of the -particle (solid line) and the nucleus (dashed line), are
shown in Fig. 1.1.

Fig. 1.1

(a) Write down, in terms of m and vo, the expression for the momentum of the nucleus at
time t1. Explain how you deduce this expression.

[3]

(b) The -particle is closest to the nucleus at time t2. The velocity of the -particle at this
time is v.

(i) Describe the motion of the -particle and the nucleus at this time.

[1]
5

(ii) Determine the velocity v of the -particle in terms of m, M and v0.

velocity = [2]

(c) In the Rutherford -particle scattering experiment, -particles with large scattering
angles (~180o) are thought to have approached the heavy gold nucleus head-on. The
closest distance of approach is obtained by using the principle of conservation of
energy, and by making the assumptions that the gold nucleus is stationary and the -
particle comes to rest momentarily at this point.

The masses of the -particle and the gold nucleus are 4 u and 197 u respectively.
Using these values and your result from (b)(ii), discuss whether the assumptions used
to derive the closest distance of approach are valid.

[2]

(d) Suggest, with a reason, whether the interaction between the -particle and the gold
nucleus can be considered as an elastic collision.

[2]
6

2 An ideal gas is contained within a cylinder as shown in Fig. 2.1. The gas undergoes a
cyclical process involving four stages as shown in Fig 2.2. Stages a to b and c to d are
isothermal processes. For stages b to c and d to a, there is no heat exchange with the
surroundings.

Wall of
cylinder
piston

Gas

Fig. 2.1 Fig. 2.2

(a) State the first law of thermodynamics.

[2]

(b) Explain how the net work done by the gas in one complete cycle could be obtained
from Fig. 2.2.

[1]

(c) State whether the heat supplied to the gas in one complete cycle is positive or
negative. Explain how you arrive at your answer.

[3]
7

(d) Suggest and explain how the isothermal process a to b could be achieved in practice.

[2]

3 Fig. 3.1 shows two stars P and Q, each of mass 4.0 x 1030 kg, moving with constant speed v
in a circular orbit of radius 1.0 x 1011 m about their common centre of mass.

Fig. 3.1

You may assume each star to be a uniform sphere with its mass concentrated at the centre of
the sphere.

(a) Define gravitational field strength.

[1]

(b) Draw on Fig. 3.1 the direction of the gravitational field strength which star Q causes at
star P.
[1]
8

(c) Calculate the magnitude of the gravitational field strength which star Q causes at star P.

gravitational field strength = N kg-1 [2]

(d) Calculate the linear speed v of each star in the system.

v= m s-1 [2]

(e) Determine the period of rotation of each star.

period = s [2]
9

4 (a) Two parallel horizontal metal plates P and Q, separated by a distance of 5.4 mm, are
situated in a vacuum as shown in Fig. 4.1.

plate Q

5.4 mm

plate P
Fig. 4.1

The lower plate P is earthed. The potential difference between the plates can be
varied. An oil droplet of mass 7.7 10-15 kg is observed to remain stationary between
the plates when plate Q is at a potential of +850 V.

(i) Suggest and explain why plates P and Q must be parallel and horizontal.

[2]

(ii) Calculate the magnitude of the charge on the oil droplet. State its sign.

charge = C
sign [3]

(b) The procedure in (a) was repeated for three further oil droplets. The magnitude of the
charge on each of the droplets was found to be 3.2 10-19 C, 6.4 10-19 C and 3.2
10-19 C.
Use these data and your answer in (a)(ii) to suggest a value for the charge on the
electron. Explain you answer.

[1]
10

5 A metal surface in an evacuated tube has a work function of 2.05 eV and is illuminated with
monochromatic light causing emission of photoelectrons which are collected at an
electrode.

For a given intensity of light, the variation of the photocurrent I with potential difference V
between the electrodes is shown in Fig. 5.1 below. It is observed that the photocurrent
saturates at 0.28 mA.
I/ mA

0.28

V/ V
-0.50 0

Fig. 5.1

(a) Explain the significance of the value of the x-intercept in Fig. 5.1.

[1]

(b) Explain why photocurrent saturates at a given intensity of light.

[2]
11

(c) Given that 5.0 % of the photons are able to eject electrons, calculate
(i) the number of photons incident on the metal surface per unit time at saturation
current.

number of photons incident = s-1 [2]

(ii) the minimum momentum of the photon that enables photoelectric effect to occur.

minimum momentum = Ns [3]


12

Section B
Answer two questions from this section.

6 A magnet is suspended vertically from a fixed point by means of a spring, as shown in


Fig. 6.1.

Fig. 6.1

One end of the magnet hangs inside a coil of wire. The coil is connected in series with a
resistor R.

(a) The magnet is displaced vertically downward a small distance D and then released.
Fig. 6.2 shows the variation with time t of the vertical displacement d of the magnet
from its equilibrium position.
d

Fig. 6.2
13

(i) State and explain, by reference to the laws of electromagnetic induction, the
nature of the oscillations of the magnet.

[4]

(ii) Calculate the angular frequency o of the oscillations.

o = rad s-1 [2]

(iii) Determine the magnitude, in terms of D, and the direction of the acceleration of
the magnet at t = 0.40 s.

magnitude =

direction [3]
14

(iv) Calculate the fraction of the initial total energy of the magnet-spring system that
is lost after t = 1.2 s.

fraction [2]

(v) The magnet experiences a mean damping force of 2.5 N. Calculate the average
power needed to be supplied to the magnet to keep it oscillating with a constant
amplitude D where D = 15 mm.

power = W [2]

(b) The resistance of the resistor R is increased. The magnet is again displaced a vertical
distance D and released. On Fig. 6.2, sketch the variation with time t of the
displacement d of the magnet. [2]
15

(c) The resistor R in Fig. 6.1 is replaced by a variable-frequency signal generator of


constant r.m.s. output voltage. The angular frequency of the generator is gradually
increased from about 0.7o to about 1.3o, where o is the angular frequency
calculated in (a)(ii).

(i) On the axes of Fig. 6.3, sketch a graph to show the variation with of the
amplitude A of the oscillations of the magnet. [2]

Fig. 6.3

(ii) State the name of the phenomenon illustrated in the graph of Fig. 6.3.

[1]

(iii) Briefly describe one situation where the phenomenon named in (ii) is useful and
one situation where it is a problem or nuisance. For each example identify what
is oscillating and what causes these oscillations.

useful:

problem:

[2]
16

7 (a) With reference to energy conversion, distinguish between electromotive force of a cell
and potential difference across the cell.

[2]

(b) The circuit shown in Fig. 7.1 is used to investigate the variation with potential
difference V of current I in a filament lamp.

Fig. 7.1

(i) Sketch in Fig 7.2 the variation with V of I you would expect to observe.

Fig. 7.2

[1]
17

(ii) Explain how you would use Fig. 7.2 to find the power of this filament lamp having
a resistance of x .

[3]

(iii) In one set-up, R is set to 4.0 . When switch S is open, voltmeter V1 reads
9.0 V. When S is closed, voltmeter V1 reads 8.0 V. Assume that the meters are
ideal and that for such a filament lamp, V varies with I as
V = 4 I2.

Determine

1. the ammeter reading I and voltmeter reading V when switch S is closed,


and

I= A

V= V [3]

2. the internal resistance r of the battery.

r= [2]
18

(c) Define magnetic flux density and state its unit.

[3]

(d) A bowl of concentrated aqueous sodium chloride, which is an electrical conductor,


contains a bar magnet D fixed to its base. One pole of D is above the level of the
solution. A thick copper wire AB, arranged as shown in Fig 7.3, has its end A vertically
above magnet D. The wire AB is pivoted freely at A. Initially, switches S1 and S2 are
open.
S1

R
copper wire S2

Fig. 7.3

A copper plate C is fixed to the side of the bowl and an electrical connection made.
When switch S1 is closed while S2 remains open, the wire AB rotates about D in an
anticlockwise manner, looking downwards at D.

(i) If the north pole of D is the pole above the surface of the solution, sketch on
Fig. 7.3, the magnetic field direction in the vicinity of B.
[1]
19

(ii) Explain the physical principles involved in the rotation.

[3]

(iii) Explain what will be observed if switch S2 is also closed.

[2]
20

8 (a) Three adjacent elements in a long radioactive series are actinium, francium and radium.

actinium (Ac) francium (Fr) radium (Ra)

An atom of 22387 Fr is formed when an atom of Ac decays with the emission of an alpha
particle. An atom of Ra is formed when an atom of Fr decays with the emission of a
beta particle. The masses of the atoms in atomic mass units are

actinium atom 227.0278 u


francium atom 223.0198 u
helium atom 4.0026 u

(i) Write down equations to represent the two nuclear reactions.

[2]

(ii) The decay of the parent actinium atom releases considerable amount of kinetic
energy. Calculate this amount of kinetic energy.

kinetic energy = MeV [3]

(iii) 1. Define mass defect of an atom.

[2]
21

2. The mass of a proton is 1.0073 u, of an electron is 0.0005 u and that


of a neutron is 1.0087 u.

Calculate the binding energy of a francium atom.

binding energy = MeV [2]

3. Eb
Sketch a graph showing how the binding energy per nucleon varies with
A
mass number A, and indicate the approximate position of the actinium (Ac),
francium (Fr) and radium (Ra) nuclei.

[2]

(b) (i) Distinguish between the count rate as measured by a detector, and the activity of
a source.

[2]
22

(ii) A radioactive source of activity 3.7 x 1010 Bq emits -ray photons uniformly in all
directions. In order to shield the source, it is placed in the centre of a hollow lead
sphere, as shown in Fig. 8.1.

Fig. 8.1

The inner and outer radii of the sphere are 0.80 cm and 20 cm respectively.

Lead absorbs -ray photon such that, for every 1.2 cm thickness of lead through which
the photons pass, half the total number of photons entering that thickness of lead are
absorbed.

1. Show that the fraction of the total number of photons emitted by the source which
1
emerge from the lead sphere is .
65536

[2]

2. Hence calculate the count-rate per unit area at the surface of the sphere.

count rate = s-1 cm-2 [3]


23

3. The absorption of the -ray photons produces a heating effect in the lead.
Suggest, with a reason, which region of the sphere is likely to experience the
greatest heating effect due to the absorption of the -ray photons.

[2]
SERANGOON JUNIOR COLLEGE
General Certificate of Education Advanced Level
Higher 1
NAME

CG INDEX NO.

PHYSICS 8866
Preliminary Examination 29th August 2013
Multiple Choice Questions 1 hr
Additional Materials: OMS.

READ THIS INSTRUCTIONS FIRST


Write your name, civics group and index number in the spaces at the top of this page.

Write in dark blue or black pen.


You may use a soft pencil for any diagrams, graphs or rough working.
Do not use staples, paper clips, highlighters, glue or correction fluid.

There are forty questions in this section. Answer all questions. For each question there are four possible
answers A, B, C and D.
Choose the one you consider correct and record your choice in soft pencil on the OMS.

Each correct answer will score one mark. A mark will not be deducted for a wrong answer.
Any rough working should be done in this booklet.

This document consist of 14 printed pages and 2 blank pages


For
2 Examiners
Use
DATA AND FORMULAE
Data

speed of light in free space, c = 3.00 x 108 m s1

elementary charge, e = 1.60 x 1019 C

the Planck constant, h = 6.63 x 1034 Js

unified atomic mass constant, u = 1.66 x 1027 kg

rest mass of electron, me = 9.11 x 1031 kg

rest mass of proton, mp = 1.67 x 1027 kg

acceleration of free fall, g = 9.81 m s2

Formulae

uniformly accelerated motion, s = ut + at2


v2 = u2 + 2as

work done on/by a gas, W = pV

hydrostatic pressure, p = gh

resistors in series, R = R1 + R2 +

resistors in parallel, 1/R = 1/R1 + 1/R2 +

SRJC 2013 8866/Prelim/2013


3

Answer all questions

1 A student made a series of measurements of the diameter d, of a coin using four vernier
callipers A, B, C and D. The table below shows the measurements taken. If the actual
diameter of the coin was 2.49 cm, which vernier calliper produced a set of readings that
could be described as accurate but not precise?

Vernier calliper Readings d / cm


A 2.49 2.46 2.52 2.50
B 2.48 2.58 2.51 2.40
C 2.35 2.37 2.42 2.42
D 2.32 2.37 2.41 2.50

2 The viscous drag force F of a sphere of radius r moving through a fluid with speed v is
given by F = 6rv. What are the S.I. base units of the viscosity of the fluid, ?

A kg-1 m s C kg m-1 s-1

B kg m3 s-1 D kg m-1 s-3

3 The graph below describes the motion of an object which has been thrown upwards in the
gravitational field of the Earth.

What could the physical quantities Y and X represent respectively?

A Displacement, time C Displacement, kinetic energy


B Time, kinetic energy D Time, velocity

SRJC 2013 8866/Prelim/2013 [Turn Over


For
4 Examiners
Use
4 A boy holding a ball is standing on the floor of a lift. The lift then starts accelerating from
rest at 4.9 m s-2 upwards. After 2.0 s, he releases the ball at 1.2 m with respect to the floor
of the lift. At the moment the ball is released, the lift comes to a sudden complete stop.
What is the time taken for the ball to reach the floor of the lift?

A 0.49 s B 1.1 s C 2.1 s D 3.0 s

5 A stone is thrown upwards from the top of a cliff. After reaching its maximum height, it falls
past the cliff-top and into the sea. The graph shows how the vertical velocity v of the stone
varies with time t after being thrown upwards. P and Q are the magnitudes of the areas of the
two triangles.

P
0
t
Q

What is the height of the cliff-top above the sea?

A P B Q C Q+P D QP

SRJC 2013 8866/Prelim/2013


5

6 Two identical cylindrical bar magnets are stored in a light plastic frictionless cylinder of
negligible mass. When the magnets are arranged as shown in the figure below and weighed,
the balance reads W. (The whole system is at rest.)

Light,
frictionless
plastic tube

top-pan
balance

If the mass of each magnet is M, which of the following is correct?

A W = Mg B Mg < W < 2Mg C W = 2Mg D W > 2Mg

7 A 5.00 kg object moves at 15.0 m s1. It collides perfectly inelastically with a 10.0 kg object
which was at rest. How much kinetic energy is lost in the collision?

A 188 J B 375 J C 563 J D 702 J

8 The scale of a certain spring balance reads from 0 to 100 N. When the balance reads 100 N,
the extension in the spring is 0.1 m long. Calculate the strain energy stored in the spring
when the balance reads 70 N. The spring obeys Hookes Law.

A 0.07 J B 2.45 J C 2.55 J D 5.00 J

SRJC 2013 8866/Prelim/2013 [Turn Over

x2
For
6 Examiners
Use
9 A heavy uniform rod of length l is supported by two cords attached to the ceiling as shown.

T1 T2

3 1
l l
4 4

T1
What is the ratio of the tensions in these cords?
T2

sin cos 1 sin 1 cos


A B C D
sin cos 2 sin 2 cos

10 A light uniform beam of length 4 m supports a 20 kg mass as shown in the figure below.

53

60

20 kg

What is the magnitude of the tension in the supporting wire?

A 170 N B 185 N C 222 N D 256 N

SRJC 2013 8866/Prelim/2013


7

11 An ideal gas within a tube was compressed with a piston as shown below without a change
in pressure within the tube.

4.0 x 104 Pa 1.0 x 105 Pa

5.0 cm

The pressure within the tube is 4.0 x 104 Pa and the pressure of the atmosphere outside is
1.0 x 105 Pa. The cross-sectional area of the piston is 2.0 x 10-4 m2.

What is the work done on the gas by the atmosphere, when the piston compresses the gas
by 5.0 cm?

A - 0.6 J B 0.4 J C 0.6 J D 1.0 J

12 An 8 W Light Emitting Diode (LED) and 15 W compact fluorescent (CFL) each generates
approximately the same light output. The LED generates heat of 3.4 Btu per hour
(1 Btu = 1055 J) and the CFL generates 30 Btu per hour.

What is the difference in efficiency of the two lighting device?

A 41 % B 46 % C 54 % D 88 %

13 A coal power station generates an average electric power of 2000 MW. The efficiency of
such power stations is only 33%.

What is the energy wastage in one week?

A 1340 MJ B 4060 MJ C 8.1 x 108 MJ D 2.5 x 109 MJ

14 A siren on top of a tall building is taken to be a point source and radiates sound waves
uniformly in all directions. At a distance 4x, the amplitude of the wave is A.
What is the distance from the siren at the point where the amplitude of the wave is 2A?

A x B 2x C 8x D 16x

SRJC 2013 8866/Prelim/2013 [Turn Over


For
8 Examiners
Use
15 A graphical representation of a progressive longitudinal wave is shown below.

displacement / m

(right)
X Z
0 distance / m

(left)
Y

Which one of the following set of statements is the most appropriate description of the wave
in the figure above?

A Position X of the longitudinal wave at this instant is at high pressure.


B Position Z of the longitudinal wave at this instant is at high pressure.
C Positions X and Z of the longitudinal wave are the nodes of the wave.
D Position Y at this instant is at the longitudinal wave's antinode.

16 Which one of the followings would produce a stable and observable interference pattern?

A Two laser beams from two different sources


B Two filament lamps
C Two speakers fed by two signal generators producing signals with different
frequencies
D Two dippers attached to one oscillator in a ripple tank

17 In a Youngs double slit experiment as shown below, a pattern of equally spaced parallel
interference fringes appears on the screen. The slit width is p while the separation between
the slits is q.

p
screen
q
p

coherent light

Which one of the following changes would cause the separation of interference fringes to be
doubled?

A Slit width p is halved. B Slit width p is doubled.


C Slit separation q is halved. D Slit separation q is doubled.
SRJC 2013 8866/Prelim/2013
9

18 A plane wave of speed v and wavelength is diffracted at a small aperture as shown below.

X
P

After what time will some portion of the wavefront XY reach point P?

3 2 3 4
A B C D
2v v v v

19 A generator with output power P and output voltage V is connected to a factory by cables of
total resistance R.
R
2

generator (P, V) factory

R
2

What is the power input to the factory?

A P
P R
B P ( )
V 2
P R
C P ( )2
V 2
P
D P ( )2 R
V

20 Wire X has resistance R. Another wire Y, of the same material, has double the length and
double the diameter of wire X.

The resistance of wire Y is


R R
A B C 2R D 4R
4 2

SRJC 2013 8866/Prelim/2013 [Turn Over


For
10 Examiners
Use

21 A battery, with an e.m.f E and internal resistance r, is connected to a switch S and two
identical resistors in series. Each resistor has resistance R.

S
R R

Which one of the following statements is correct when the switch S is closed?
A The voltmeter reading is 0.5E when an ideal voltmeter is connected across one
resistor.
The voltmeter reading is E when an ideal voltmeter is connected across two
B
resistors.
The voltmeter reading is E when an ideal voltmeter is connected across the
C
battery.
D The voltmeter reading is less than E when an ideal voltmeter is connected
across the battery.

22 With 4 resistors, each having a resistance of 12 , it is impossible to arrange all 4


resistors to have an effective resistance of

A 9 B 20 C 24 D 30

23 A battery is marked 1.5 V.

What does this mean?

A Each coulomb of charge from the battery supplies 1.5 J of electrical energy to the
whole circuit.

B The battery supplies 1.5 J to an external circuit for each coulomb of charge.

C The potential difference across any component connected to the battery will be 1.5 V.

D There will always be 1.5 V across the battery terminals.

SRJC 2013 8866/Prelim/2013


11

24 A circuit consisting of 4 resistors is shown below.

R1 R2

K
200 100 200

When a wire is connected between J & K, no current flows in the wire.

R2
The ratio of resistance is
R1

A dependent on the value of E B 0.5


C 0.67 D 1.5

25 An electron is instantaneously moving with a velocity v along the plane of the paper. It is
moving in a uniform magnetic field which is perpendicular to the plane of the paper and
going into the page.


electron

v

Which of the following statements is correct?

A The magnitude of momentum of the electron will remain constant.

B The work done by the net force exerted on the electron is a non-zero constant.

C The initial force on the electron is vertically upwards.

D The magnetic force exerted on the electron may not always be perpendicular to the
magnetic field.

SRJC 2013 8866/Prelim/2013 [Turn Over


For
12 Examiners
Use

26 In order to produce a magnet with the polarity as shown,

S N

Which of the following windings and the current flowing in them will give the desired
polarity?

A B

C D

27 In any photoelectric emission experiment, the variables are frequency of the source, power
of the same light source and the material used.

Which of these will affect the maximum kinetic energy of the photoelectrons?

A Frequency of the source only.


B Frequency of the source and power of the light source.
C Frequency of the source and material used.
D Material used only.

SRJC 2013 8866/Prelim/2013


13

28 In a photoelectric experiment, two different metal electrodes were illuminated initially by blue
radiation as shown in the circuit.
radiation source

metal metal
B A
A

e.m.f sorce

There was an initial reading in the ammeter. It was realised that when the radiation source
was changed to a green radiation with the same power rating, the reading in the ammeter
remains unchanged. This may be due to

A the electrons being emitted from metal A never reaching metal B.


B the work function of metal B being much higher than that of metal A.
C the increase in number of electrons ejected from metal A and a decrease in the
number of electrons ejected from metal B.
D the increase in number of electrons ejected from metal B and a decrease in the
number of electrons ejected from metal A.

29 The particles listed below have the same de Broglie wavelength.


Which one must have the lowest velocity?

A particle (Helium nucleus)


B neutron
C - particle (electron)
1H nucleus
3
D

SRJC 2013 8866/Prelim/2013 [Turn Over


For
14 Examiners
Use
30 In a collision between an excited Helium atom and a Neon atom, the Neon atom which is at
the ground state may be excited. The energy levels of Helium and Neon are given below.

20.61 eV 20.66 eV
18.70 eV

Common ground state


(0 eV)
Helium Neon
Which of the following statements is correct?

A Energy cannot be directly transferred by the collision process. Only photons are able to
cause excitation of the Neon atom.
B For the excited Neon atom, it is only possible to emit a photon of energy 18.70 eV.
C For the excited Neon atom, it is only possible to emit a photon of energy 18.70 eV or
20.66 eV.
D For the excited Neon atom, it is only possible to emit a photon of energy 1.96 eV, 18.70
eV or 20.66 eV.

End of Paper

SRJC 2013 8866/Prelim/2013


SERANGOON JUNIOR COLLEGE
General Certificate of Education Advanced Level
Higher 1
NAME

CG INDEX NO.

PHYSICS 8866
Preliminary Examination 29th August 2013
Multiple Choice Questions 1 hr 15 mins
Additional Materials: OMS.

READ THIS INSTRUCTIONS FIRST


Write your name, civics group and index number in the spaces at the top of this page.

Write in dark blue or black pen.


You may use a soft pencil for any diagrams, graphs or rough working.
Do not use staples, paper clips, highlighters, glue or correction fluid.

There are forty questions in this section. Answer all questions. For each question there are four possible
answers A, B, C and D.
Choose the one you consider correct and record your choice in soft pencil on the OMS.

Each
. correct answer will score one mark. A mark will not be deducted for a wrong answer.
Any rough working should be done in this booklet.

This document consist of 32 printed pages and 0 blank page


2

DATA AND FORMULAE


Data
speed of light in free space, c = 3.00 x 108 m s1
permeability of free space, 0 = 4 x 10-7 H m-1
permittivity of free space, 0 = 8.85 x 10-12 F m-1
= (1/(36 )) x 10-9 F m-1
elementary charge, e = 1.60 x 1019 C
the Planck constant, h = 6.63 x 1034 J s
unified atomic mass constant, u = 1.66 x 1027 kg
rest mass of electron, me = 9.11 x 1031 kg
rest mass of proton, mp = 1.67 x 1027 kg
molar gas constant, R = 8.31 J K1 mol1
the Avogadro constant, NA = 6.02 x 1023 mol1
the Boltzmann constant, k = 1.38 x 10-23J K1
gravitational constant, G = 6.67 x 10-11N m2 Kg2

acceleration of free fall, g = 9.81 m s2

SRJC 2013 8866/Prelim/2013


3

Formulae
uniformly accelerated motion, s = ut + at2

v2 = u2 + 2as

work done on/by a gas, W = pV


hydrostatic pressure, p = gh
GM
gravitational potential, = -
r
displacement of particle in s.h.m., x = x0 sin t
velocity of particle in s.h.m., v = v0 cos t

= x02 - x 2
resistors in series, R = R1 + R2 +
resistors in parallel, 1/R = 1/R1 + 1/R2 +
electric potential, V = Q/ 4 0r
alternating current/ voltage, x = x0 sin t
transmission coefficient, T exp(-2kd)

8 2 m(U - E )
where k =
h2
radioactive decay, x = x0 exp(-t)

decay constant, = 0.693


t1
2

SRJC 2013 9646/MYE/2013 [Turn Over


4 For
Examiners
Use
Answer all questions

1 A student made a series of measurements of the diameter d, of a coin using four vernier
callipers A, B, C and D. The table below shows the measurements taken. If the actual
diameter of the coin was 2.49 cm, which vernier calliper produced a set of readings that
could be described as accurate but not precise?

Vernier calliper Readings d / cm


A 2.49 2.46 2.52 2.50
B 2.48 2.58 2.51 2.40
C 2.35 2.37 2.42 2.42
D 2.32 2.37 2.41 2.50

Vernier calliper Mean d /cm Spread / cm


A 2.49 2.52 2.46 = 0.06
B 2.49 2.58 2.40 = 0.18
C 2.39 2.42 2.35 = 0.07
D 2.40 2.50 2.32 = 0.18
Ans: B. Readings from instrument B have the closest
mean to the actual reading (most accurate) and the
biggest spread (least precise).

2 The viscous drag force F of a sphere of radius r moving through a fluid with speed v is
given by F = 6rv. What are the S.I. base units of the viscosity of the fluid, ?

A kg-1 m s C kg m-1 s-1

B kg m3 s-1 D kg m-1 s-3

Ans: C
= F/(6rv)
[] = [F]/[rv]
= kg m s-2 m-1 m-1 s
= kg m-1 s-1

SRJC 2013 9646/MYE/2013


5

3 The graph below describes the motion of an object which has been thrown upwards in the
gravitational field of the Earth.

What could the physical quantities Y and X represent respectively?

A Displacement, time C Displacement, kinetic energy


B Time, kinetic energy D Time, velocity

Ans: A. Y-X has the shape of a displacement-time graph. (It could also
represent the potential energy-time graph, since potential energy is proportional to
displacement.)

4 A boy holding a ball is standing on the floor of a lift. The lift then starts accelerating from
rest at 4.9 m s-2 upwards. After 2.0 s, he releases the ball at 1.2 m with respect to the floor
of the lift. At the moment the ball is released, the lift comes to a sudden complete stop.
What is the time taken for the ball to reach the floor of the lift?

A 0.49 s B 1.1 s C 2.1 s D 3.0 s

Ans: C
After 2.0 s, the ball acquires a velocity of v = u + at
= 0 + (4.9)(2.0)
= 9.8 m s-1 upward

Taking the downward as +ve, and using s = ut + (1/2)at2 and a = g since the
ball the acceleration due to gravity alone.

1.2 = -9.8t + (1/2)gt2


t = 2.1 s

SRJC 2013 9646/MYE/2013 [Turn Over


6 For
Examiners
Use
5 A stone is thrown upwards from the top of a cliff. After reaching its maximum height, it falls
past the cliff-top and into the sea. The graph shows how the vertical velocity v of the stone
varies with time t after being thrown upwards. P and Q are the magnitudes of the areas of the
two triangles.

P
0
t
Q

What is the height of the cliff-top above the sea?

A P B Q C Q+P D QP

Ans: D. Area P represents the maximum height that the stone will reach, taking the
cliff as the starting position. Area Q represents the total height, starting from the
maximum height the stone reaches till it drops to the sea. Therefore, the height of
the cliff-top above the sea is Q P.

6 Two identical cylindrical bar magnets are stored in a light plastic frictionless cylinder of
negligible mass. When the magnets are arranged as shown in the figure below and
weighed, the balance reads W. (The whole system is at rest.)

Light,
frictionless
plastic tube

top-pan
balance

If the mass of each magnet is M, which of the following is correct?


A W = Mg B Mg < W < 2Mg C W = 2Mg D W > 2Mg

SRJC 2013 9646/MYE/2013


7
Ans: C
FBD for top magnet
Fmagnet Mg --(1) Mg

Fmagnet

FBD for bottom magnet


Fmagnet
Fmagnet Mg W --(2)
Subst (1) into (2),
W Mg Mg 2Mg Mg
W

7 A 5.00 kg object moves at 15.0 m s1. It collides perfectly inelastically with a 10.0 kg object
which was at rest. How much kinetic energy is lost in the collision?

A 188 J B 375 J C 563 J D 702 J

Ans: B
v
u1 = 15.0 m s1 u2 = 0 m s1

m1 = 5.00 kg
m2 = 10.0 kg
mf = 15.0 kg

Using conservation of momentum,


m1u1 m2u 2 mf v
5.00(15.0) 10.0(0) 15.0(v )
v 5.00 m s1
1 2 1
m1u1 mf v 2
Loss of kinetic energy = 2 2
1 1
(5.00)(15.0) 2 (15.0)(5.00) 2
2 2
375 J

SRJC 2013 9646/MYE/2013 [Turn Over


8 For
Examiners
Use
8 The scale of a certain spring balance reads from 0 to 100 N. When the balance reads
100 N, the extension in the spring is 0.1 m long. Calculate the strain energy stored in the
spring when the balance reads 70 N. The spring obeys Hookes Law.

A 0.07 J B 2.45 J C 2.55 J D 5.00 J


Ans: B

F1 = kx1
F/N
k = 100 / 0.1
= 1000 Nm1 100

When F2 = kx2 = 70 N, 70
x2 = 70 / 1000
= 0.07 m
A
Energy = Area A (under F-x graph) x/m
0.1
= 70 0.07
= 2.45 J
x2

9 A heavy uniform rod of length l is supported by two cords attached to the ceiling as shown.

T1 T2

3 1
l l
4 4

T1
What is the ratio of the tensions in these cords?
T2

sin cos 1 sin 1 cos


A B C D
sin cos 2 sin 2 cos

Ans: C
Taking moments about the centre of mass of the rod,
1 1 1
T1 sin l T2 sin l l
2 2 4
T1 1 sin

T2 2 sin

SRJC 2013 9646/MYE/2013


9

10 A light uniform beam of length 4 m supports a 20 kg mass as shown in the figure below.

53

60

20 kg

What is the magnitude of the tension in the supporting wire?

A 170 N B 185 N C 222 N D 256 N

Ans : B
The system is in equilibrium, F
Horizontally: 60
Fsin60o =Tsin53o --(1)
Vertically: 20(9.81)
o
53
Tcos53 +Fcos60 =20(9.81)
o
--(2) T

Subst. (1) into (2),


Tsin53o
Tcos53o + o
cos60o =20(9.81)
sin60
T(0.60182+0.46109)=20(9.81)
T=185 N

SRJC 2013 9646/MYE/2013 [Turn Over


10 For
Examiners
Use
11 An ideal gas within a tube was compressed with a piston as shown below without a change
in pressure within the tube.

4.0 x 104 Pa 1.0 x 105 Pa

5.0 cm

The pressure within the tube is 4.0 x 104 Pa and the pressure of the atmosphere outside is
1.0 x 105 Pa. The cross-sectional area of the piston is 2.0 x 10-4 m2.

What is the work done on the gas by the atmosphere, when the piston compresses the gas
by 5.0 cm?

A - 0.6 J B 0.4 J C 0.6 J D 1.0 J

Ans: B
Work done on gas = 4 x 105 x 2 x 10-4 x 5.0 x 10-2 = 0.4 J

12 An 8 W Light Emitting Diode (LED) and 15 W compact fluorescent (CFL) each generates
approximately the same light output. The LED generates heat of 3.4 Btu per hour
(1 Btu = 1055 J) and the CFL generates 30 Btu per hour.

What is the difference in efficiency of the two lighting device?

A 41 % B 46 % C 54 % D 88 %

Ans: B
(8 3600) (3.4 1055)
Efficiency of LED = 100% 87.5%
(8 3600)
(15 3600) (30 1055)
Efficiency of CFL = 100% 41.4%
(15 3600)
Difference = 87.5 % - 41.4 % 46%

13 A coal power station generates an average electric power of 2000 MW. The efficiency of
such power stations is only 33%.

What is the energy wastage in one week?

A 1340 MJ B 4060 MJ C 8.1 x 108 MJ D 2.5 x 109 MJ

SRJC 2013 9646/MYE/2013


11

Ans: D
Power output =2000MW
2000
Power wastage = 67MW
33
2000
Energy wastage = 67 24 3600 7 2.5 109 MJ
33

14 A siren on top of a tall building is taken to be a point source and radiates sound waves
uniformly in all directions. At a distance 4x, the amplitude of the wave is A.
What is the distance from the siren at the point where the amplitude of the wave is 2A?

Ax B 2x C 8x D 16x

Ans: B
I A2
1
I
r2
1
A
r
When the amplitude is doubled, the distance is halved.

15 A graphical representation of a progressive longitudinal wave is shown below.

displacement / m

(right)
X Z
0 distance / m

(left)
Y

Which one of the following set of statements is the most appropriate description of the wave
in the figure above?

A Position X of the longitudinal wave at this instant is at high pressure.


B Position Z of the longitudinal wave at this instant is at high pressure.
C Positions X and Z of the longitudinal wave are the nodes of the wave.
D Position Y at this instant is at the longitudinal wave's antinode.

Ans: A

SRJC 2013 9646/MYE/2013 [Turn Over


12 For
Examiners
Use

16 Which one of the followings would produce a stable and observable interference pattern?

A Two laser beams from two different sources


B Two filament lamps
C Two speakers fed by two signal generators producing signals with different
frequencies
D Two dippers attached to one oscillator in a ripple tank
Ans: D

17 In a Youngs double slit experiment as shown below, a pattern of equally spaced parallel
interference fringes appears on the screen. The slit width is p while the separation between
the slits is q.

p
screen
q
p

coherent light

Which one of the following changes would cause the separation of interference fringes to be
doubled?

A Slit width p is halved.


B Slit width p is doubled.
C Slit separation q is halved.
D Slit separation q is doubled.

Ans: C

D
x where a is slit separation. For x to be doubled, slit separation has to be halved.
a

SRJC 2013 9646/MYE/2013


13

18 A plane wave of speed v and wavelength is diffracted at a small aperture as shown below.

X
P

After what time will some portion of the wavefront XY reach point P?

3 2 3 4
A B C D
2v v v v

Ans: C
distance 3
t=
speed v

19 A generator with output power P and output voltage V is connected to a factory by cables of
total resistance R.
R
2

generator (P, V) factory

R
2

What is the power input to the factory?

A P
P R
B P ( )
V 2
P R
C P ( )2
V 2
P
D P ( )2 R
V

Ans: D
P
Ploss I 2R ( )2 R
V
P
P i P Ploss P ( )2 R
V
SRJC 2013 9646/MYE/2013 [Turn Over
14 For
Examiners
Use

20 Wire X has resistance R. Another wire Y, of the same material, has double the length and
double the diameter of wire X.

The resistance of wire Y is


R R
A B C 2R D 4R
4 2

Ans: B
l
R
r 2
(2l ) l R

(2r ) 2
2 r 2
2

21 A battery, with an e.m.f E and internal resistance r, is connected to a switch S and two
identical resistors in series. Each resistor has resistance R.

S
R R

Which one of the following statements is correct when the switch S is closed?
A The voltmeter reading is 0.5E when an ideal voltmeter is connected across one
resistor.
The voltmeter reading is E when an ideal voltmeter is connected across two
B
resistors.
The voltmeter reading is E when an ideal voltmeter is connected across the
C
battery.
D The voltmeter reading is less than E when an ideal voltmeter is connected
across the battery.

Ans: D
For closed circuits, terminal p.d. is lower when there is internal resistance in the source
(non-ideal).

SRJC 2013 9646/MYE/2013


15

22 With 4 resistors, each having a resistance of 12 , it is impossible to arrange all 4


resistors to have an effective resistance of

A 9 B 20 C 24 D 30

Ans: C

20 9

30

23 A battery is marked 1.5 V.

What does this mean?

A Each coulomb of charge from the battery supplies 1.5 J of electrical energy to the
whole circuit.

B The battery supplies 1.5 J to an external circuit for each coulomb of charge.

C The potential difference across any component connected to the battery will be 1.5
V.

D There will always be 1.5 V across the battery terminals.

Ans: A
Due to the fact that there is internal resistance in the battery,
B ,C & D will not be true.

SRJC 2013 9646/MYE/2013 [Turn Over


16 For
Examiners
Use
24 A circuit consisting of 4 resistors is shown below.

R1 R2

K
200 100 200

When a wire is connected between J & K, no current flows in the wire.

R2
The ratio of resistance is
R1

A dependent on the value of E B 0.5


C 0.67 D 1.5

Ans: D
As no current flows from J to K, Potential of J must be the same as K.
This implies that PD across R1 and PD across the 200 resistance must
be the same.

By potential divider principle


R1 200
E E
R1 R2 200 100 200
200 R2 300 R1
R2
1.5
R1

SRJC 2013 9646/MYE/2013


17

25 An electron is instantaneously moving with a velocity v along the plane of the paper. It is
moving in a uniform magnetic field which is perpendicular to the plane of the paper and
going into the page.


electron

v

Which of the following statements is correct?

A The magnitude of momentum of the electron will remain constant.

B The work done by the net force exerted on the electron is a non-zero constant.

C The initial force on the electron is vertically upwards.

D The magnetic force exerted on the electron may not always be perpendicular to the
magnetic field.

Ans: A
As the force by magnetic field is always perpendicular to the motion of the
electron, the work done by the magnetic force is always zero. There is no
change in the speed of the electron, hence the momentum of the electron
remains constant.
(When using Flemings Left Hand Rule to determine the direction of the
force on the electron, remember that the electron is negatively charged,
hence the direction of current is opposite of direction of motion,

SRJC 2013 9646/MYE/2013 [Turn Over


18 For
Examiners
Use

26 In order to produce a magnet with the polarity as shown,

S N

Which of the following windings and the current flowing in them will give the desired
polarity?

A B

C D

Ans: B
Using right hand grip rule.

27 In any photoelectric emission experiment, the variables are frequency of the source, power
of the same light source and the material used. Which of these will affect the maximum
kinetic energy of the photoelectrons?

A Frequency of the source only.


B Frequency of the source and power of the light source.
C Frequency of the source and material used.
D Material used only.

Ans: C
Photoelectric equation : hf = + KEmax
Frequency of radiation and work function will affect KEmax. ( is dependent on
the material which is irradiated.)

SRJC 2013 9646/MYE/2013


19

28 In a photoelectric experiment, two different metal electrodes were illuminated initially by blue
radiation as shown in the circuit.
radiation source

metal metal
B A
A

e.m.f sorce

There was an initial reading in the ammeter. It was realised that when the radiation source
was changed to a green radiation with the same power rating, the reading in the ammeter
remains unchanged. This may be due to

A the electrons being emitted from metal A never reaching metal B.


B the work function of metal B being much higher than that of metal A.
C the increase in number of electrons ejected from metal A and a decrease in the
number of electrons ejected from metal B.
D the increase in number of electrons ejected from metal B and a decrease in the
number of electrons ejected from metal A.

Ans: B
Higher work function implies the KE of the electrons emitted is lower, which may not have
sufficient energy to reach the opposite electrode at the first place.

29 The particles listed below have the same de Broglie wavelength.


Which one must have the lowest velocity?

A particle (Helium nucleus)


B neutron
C - particle (electron)
3
D 1H nucleus

Ans: A
h h
De Broglies wavelength .
p mv
1
As is constant, v , the option with the largest mass will have the smallest v (velocity)
m

SRJC 2013 9646/MYE/2013 [Turn Over


20 For
Examiners
Use
30 In a collision between an excited Helium atom and a Neon atom, the Neon atom which is at
the ground state may be excited. The energy levels of Helium and Neon are given below.

20.61 eV 20.66 eV
18.70 eV

Common ground state


(0 eV)
Helium Neon
Which of the following statements is correct?

A Energy cannot be directly transferred by the collision process. Only photons are able to
cause excitation of the Neon atom.
B For the excited Neon atom, it is only possible to emit a photon of energy 18.70 eV.
C For the excited Neon atom, it is only possible to emit a photon of energy 18.70 eV or
20.66 eV.
D For the excited Neon atom, it is only possible to emit a photon of energy 1.96 eV, 18.70
eV or 20.66 eV.

Ans: D
Due to the kinetic energy of the atoms during the collision, it is possible for electrons in the
Neon atom to be excited to the 20.66 eV. When electrons de-excite from 20.66 eV, there are
3 possible paths of further de-excitation.

End of Paper

SRJC 2013 9646/MYE/2013


SERANGOON JUNIOR COLLEGE
General Certificate of Education Advanced Level
Higher 1
NAME

CG INDEX NO.

PHYSICS 8866
Preliminary Examination 23rd August 2013
Paper 2 Structured Questions 2 hours

Candidates answer on the Question Paper.

No Additional Materials are required.

READ THIS INSTRUCTIONS FIRST


Write your name, civics group and index number in the spaces at the top of this page.

Write in dark blue or black pen on both sides of the paper. For Examiners Use
You may use a soft pencil for any diagrams, graphs or rough working.
Do not use staples, paper clips, highlighters, glue or correction fluid.
Q1 / 4
Section A
Answer all questions. Q2 / 6

Section B Q3 / 6
Answer any two questions.
Q4 / 6
You are advised to spend about an hour on each section.
Q5 / 8
At the end of the examination, fasten all your work securely together.
The number of marks is given in bracket [ ] at the end of each question or Q6 / 10
part question.
. Q7 / 20

Q8 / 20

Q9 / 20
Total
/ 80
marks

This document consist of 24 printed pages and 0 blank page


2

DATA AND FORMULAE


Data

speed of light in free space, c = 3.00 x 108 m s1

elementary charge, e = 1.60 x 1019 C

the Planck constant, h = 6.63 x 1034 J s

unified atomic mass constant, u = 1.66 x 1027 kg

rest mass of electron, me = 9.11 x 1031 kg

rest mass of proton, mp = 1.67 x 1027 kg

acceleration of free fall, g = 9.81 m s2

Formulae

uniformly accelerated motion, s = ut + at2


v2 = u2 + 2as

work done on/by a gas, W = pV

hydrostatic pressure, p = gh

resistors in series, R = R1 + R2 +

resistors in parallel, 1/R = 1/R1 + 1/R2 +

SRJC 2013 8866/Prelim/2013


3 For
Examiners
Use
Section A
Answer all the questions in this section.

l
1 The formula for the period of a simple pendulum is T 2 . Such a pendulum is used to
g
determine g. The measured value of l is (98.2 0.1) cm and that of T is (1.99 0.01) s.
Determine the value of g and its associated uncertainty.

g g = ..... m s-2 [4]

SRJC 2013 8866/Prelim/2013 [Turn Over


4 For
Examiners
Use

2 Two carts of masses 400 g and 1200 g are free to move on a frictionless horizontal table.
Fig. 2.1 shows a spring of spring constant 500 N m-1 placed between the carts and the carts
are tied together by a thread so that the compression of the spring is 3.0 cm. The carts are
initially at rest.

thread
spring

Fig. 2.1

(a) Calculate the elastic potential energy stored in the spring when the compression is
3.0 cm.

elastic potential energy = ..... J [2]

(b) The thread is then cut. Determine the final speeds of the two carts.

final speed of 400 g cart = ..... m s-1

final speed of 1200 g cart = ..... m s-1 [4]

SRJC 2013 8866/Prelim/2013


5 For
Examiners
Use
3 (a) (i) State the principle of moments.

..

..

..

.... [1]

(ii) A pair of pliers consists of 2 halves held together by a pivot. Fig 3.1 shows two
forces acting on one of the halves for a pair of pliers holding a sphere.

30o 8 N

10 cm

4 cm

Fig. 3.1

Calculate the magnitude of the contact force of the sphere on the pliers, F.

F = ..... N [2]

SRJC 2013 8866/Prelim/2013 [Turn Over


6 For
Examiners
Use

(b) (i) The Earth has an atmosphere which consist of a mixture of air. The average
density of air is 1.225 kg m-3 and the atmospheric pressure is 1.0 x 105 Pa at the
Earths surface.

Calculate the height of the atmosphere.

height = m [2]

(ii) Besides the estimates of the density and pressure of air, discuss why the answer
to part (b)(i) may not be accurate.

....

.[1]

SRJC 2013 8866/Prelim/2013


7 For
Examiners
Use
4 Fig. 4.1 shows two coherent electromagnetic wave emitters, M1 and M2 which emit waves in
phase. A detector is placed at X. As the frequency of the emitters gradually increases, the
detected signal shows a series of maxima and minima.

M1 X
detector
9 cm

M2
40 cm

Fig. 4.1

(i) Explain how the maxima and minima can be observed at the same point X.

.... [3]

(ii) Calculate the frequency at which the first minimum intensity is observed at X.

frequency = ... Hz [3]

SRJC 2013 8866/Prelim/2013 [Turn Over


8 For
Examiners
Use

5 (a) Define the term tesla.

[2]

(b) Fig. 5.1 shows a cylindrical conducting rod of length 10.0 cm resting on two
horizontal copper rails. The rails are connected to a 12 V e.m.f source. The region
between has a magnetic field of flux density 0.20 T acting perpendicularly into the
plane of the paper.

rod

x x x x x xx x x
rails x x x x x xx x x 12 V e.m.f
x x x x x xx x x source

Uniform B- Field
(0.20 T)

Fig. 5.1

(i) Sketch in Fig. 5.1 how the e.m.f source must be connected so that the rod
experiences a force pushing it to the left. [1]

(ii) 1. In order to keep the rod just stationary, the plane of the rails must be tilted at an
angle from the horizontal as shown in Fig. 5.2. The magnetic field remains in
the same direction and the frictional force between the rod and rails is negligible.

Draw and label all the forces acting on the rod. [2]

rod

rail

Fig. 5.2

SRJC 2013 8866/Prelim/2013


9 For
Examiners
Use
2. The mass of the rod is 5.0 g, is 30. Determine the total resistance of the rod
for this set up. The rails and conducting wires are of negligible resistance.

total resistance = .. [3]

SRJC 2013 8866/Prelim/2013 [Turn Over


10 For
Examiners
Use

6 The circuit in Fig. 6.1 is used for an experiment to determine Plancks constant.

B V
L
R1
5V
A
R2

Fig. 6.1

A Light-Emitting Diode (LED) that gives off red light of wavelength 695 nm is used for
component L. As contact point at resistor R1 shifts from A to B, the reading across the
voltmeter also varies.

Initially, the red LED light did not light up, it was only when the voltmeter reached a reading
of V0 did it light up. V0 was then recorded. L was then replaced with LEDs that gave off
infrared light of wavelength 1000 nm, yellow light of wavelength 660 nm and green light of
wavelength 630 nm.

The data collected is as shown in table in Fig. 6.2.

LED V0 / V / nm -1 / nm-1

Infrared 0.866 1000

Red 1.367 695

Yellow 1.464 660

Green 1.560 630

Fig. 6.2

(a) Fill in column on -1 in Fig. 6.2. [1]

SRJC 2013 8866/Prelim/2013


11 For
Examiners
Use
(b) Plot a graph of V0 against -1 on the grids in Fig. 6.3. [3]

V0 / V

-1/ nm-1
Fig. 6.3

(c) State the energy which is required for an electron to move across the LED in terms of
elementary charge e and voltage across the LED V0.

.[1]

(d) The energy of each photon corresponds to the energy stated in (c). Show how the
Plancks constant can be determined from the graph in Fig. 6.3. [2]

SRJC 2013 8866/Prelim/2013 [Turn Over


12 For
Examiners
Use

(e) Determine the Plancks constant from the graph in Fig. 6.3.

Plancks constant = [3]

SRJC 2013 8866/Prelim/2013


13 For
Examiners
Use
Section B
Answer two questions from this section.

7 (a) A hot air balloon was rising steadily at a speed of 10.0 m s-1 vertically. Then a constant wind
blew horizontally, causing the hot air balloon to travel 5.0 m s-1 in the same direction as the
wind while it continues to rise steadily. The balloon has a resultant velocity of v at an angle
R
to the horizontal, as shown in Fig. 7.1.

v
R

Fig. 7.1

(i) Calculate the magnitude of the resultant velocity v .


R

v = ..... m s-1 [2]


R

(ii) Sandbag 1 was dropped from the balloon. Calculate the distance between the
balloon and sandbag 1 after 3.0 s. (Assume that the dropping of sandbags did not
affect the velocity of the hot air balloon and that effects of air resistance on the
sandbags were negligible.)

distance = ..... [3]

SRJC 2013 8866/Prelim/2013 [Turn Over


14 For
Examiners
Use

(iii) Sandbag 2 was subsequently dropped from the moving balloon. Sandbag 1 was
dropped at time t = 0 s. Sandbag 2 was dropped 1.0 s later.

1 Calculate the times for sandbags 1 and 2 to reach their respective highest points.

time for sandbag 1 = .... s

time for sandbag 2 = ..... s [2]

2 Calculate the vertical velocities of sandbags 1 and 2 at t = 3.0 s.

vertical velocity for sandbag 1 = ..... m s-1

vertical velocity for sandbag 2 = ..... m s-1 [2]

SRJC 2013 8866/Prelim/2013


15 For
Examiners
Use

3 Considering only the vertical velocities vy, sketch the vy against t graphs of the
two sandbags from t = 0 s to t = 3.0 s in Fig. 7.2. Label your graphs clearly with
appropriate values. [2]

vy / m s-1

t/s

Fig. 7.2

(b) A stone is projected up an inclined plane at an angle of 20o with respect to the inclined
plane as shown in Fig. 7.2. The initial speed of projection is 40.0 m s-1.

40.0 m s-1
20
stone
30

Fig. 7.2

(i) State any assumption made about the acceleration when making use of the kinematic
equations.

.... [1]

SRJC 2013 8866/Prelim/2013 [Turn Over


16 For
Examiners
Use

(ii) Using s = ut + at2, write down two equations relating the horizontal displacement h
and the vertical displacement x with time t.

h = .....

x = ..... [2]

(iii) State an equation relating h and x when the stone hits the inclined plane.

. [1]

(iv) By using equations relating the horizontal and vertical displacements with time or
otherwise, show that the stone will land on the inclined plane at t = 3.22 s. [2]

(v) Determine the speed of the stone on impact with the inclined plane at t = 3.22 s.

speed of stone on impact = . m s-1 [3]


SRJC 2013 8866/Prelim/2013
17 For
Examiners
Use
8 (a) Explain what is meant by

(i) a battery has an electromotive force (e.m.f.) of 2.0 V,

.... [1]

(ii) the potential difference (p.d.) across a resistor is 2.0 V.

.... [1]

(b) In an electrical device, the current in the electron beam between the cathode and anode is
3.0 mA. If the device is switched on for 40 s, determine

(i) the number of electrons leaving the cathode,

number of electrons = . [2]

(ii) the energy supplied to the device if it is connected to a 220 V mains.

energy supplied = J [2]

SRJC 2013 8866/Prelim/2013 [Turn Over


18 For
Examiners
Use

(c) The current-potential difference relationship for two electrical components P and Q is shown
in Fig. 8.1.

current / mA

P
8
Q
7
6
5
4
3
2
1
0
2 4 6 8 10 potential difference / V
Fig. 8.1

(i) Identify

1. component P.

...... [1]

2. component Q.

...... [1]

(ii) P and Q are connected in parallel. The current flowing through P is 6 mA.

1. State the potential difference across Q.

potential difference across Q = V [1]

2. State the current flowing in Q.

current in Q = mA [1]

SRJC 2013 8866/Prelim/2013


19 For
Examiners
Use
3. Determine the effective resistance if P and Q are considered as a single component.

effective resistance = .. [3]

(iii) P and Q are now connected in series with a battery of internal resistance of 2 . The
current flowing through P is 3 mA.

1. Calculate the terminal potential difference across the battery.

terminal potential difference across the battery = V [2]

2. Calculate the rate of heat dissipated in the battery.

rate of heat dissipated by the battery = . W [1]

SRJC 2013 8866/Prelim/2013 [Turn Over


20 For
Examiners
Use

(iv) Component P only exhibits the I-V characteristics as shown in Fig. 8.1 when the
temperature of P is constant.

State and explain the effect on the resistance of component P when the temperature
increases.

[4]

SRJC 2013 8866/Prelim/2013


21 For
Examiners
Use
9 Fig. 9.1 shows a photoemissive cell consisting of the emitter and the collector plate, made of
the same metal connected to a variable d.c supply and a voltmeter and an ammeter.

I
Emitter

B
Radiation
V Variable e.m.f source
A
Collector

Fig. 9.1
2
The emitter of area 0.40 cm is illuminated with monochromatic radiation of wavelength 270 nm
and intensity 200 W m2. The current I in the circuit is measured for various values of the
applied potential difference V between the collector and emitter. The results are shown in
Fig. 9.2.

I /nA

5.0

-1.1 1.6 V /V
Fig. 9.2

(a)(i) 1. When V is negative, state which point of the variable e.m.f source (A or B) is at the
higher potential.

point .. [1]

2. Calculate the rate of incidence of photons on the emitter.

rate of incidence = s-1 [3]

SRJC 2013 8866/Prelim/2013 [Turn Over


22 For
Examiners
Use

3. Calculate the work function of the emitter in eV.

work function = eV [3]

(ii) While keeping the frequency of the radiation constant, the intensity is now halved to
100 W m2.

1. Sketch and label in Fig. 9.2 the graph which will be obtained. [2]

2. Explain the key features of the new graph in relation to the original graph.

[3]

(b) Most street lights in Singapore use Sodium lamp. They give off light of distinct orange-
yellowish colour.

(i) Explain why the lamp gives off a distinct orange-yellowish light.

... [2]

SRJC 2013 8866/Prelim/2013


23 For
Examiners
Use
(ii) Two different setups were used to analyse the light source from the Sodium lamp.
Fig. 9.3 shows the two visible light spectrums P and Q.

Fig. 9.3
1. Identify the type of spectrum P and Q.

P: ...

Q: [1]

2. With the aid of a diagram, describe how spectrum P is produced.

[4]

SRJC 2013 8866/Prelim/2013 [Turn Over


24 For
Examiners
Use

(c) Calculate the energy of the photon emitted by the Sodium atom if the wavelength of the
light emitted is 569 nm.

energy of photon = . J [1]

End of paper

SRJC 2013 8866/Prelim/2013


SERANGOON JUNIOR COLLEGE
General Certificate of Education Advanced Level
Higher 1
NAME

CG INDEX NO.

PHYSICS 8866
Preliminary Examination 23rd August 2013
Paper 2 Structured Questions (ANS) 2 hours

Candidates answer on the Question Paper.

No Additional Materials are required.

READ THIS INSTRUCTIONS FIRST


Write your name, civics group and index number in the spaces at the top of this page.

Write in dark blue or black pen on both sides of the paper. For Examiners Use
You may use a soft pencil for any diagrams, graphs or rough working.
Do not use staples, paper clips, highlighters, glue or correction fluid.
Q1 / 4
Section A
Answer all questions. Q2 / 6

Section B Q3 / 6
Answer any two questions.
Q4 / 6
You are advised to spend about an hour on each section.
Q5 / 8
At the end of the examination, fasten all your work securely together.
The number of marks is given in bracket [ ] at the end of each question or Q6 / 10
part question.
. Q7 / 20

Q8 / 20

Q9 / 20
Total
/ 80
marks

This document consist of 26 printed pages and 0 blank page


2

DATA AND FORMULAE


Data

speed of light in free space, c = 3.00 x 108 ms1

elementary charge, e = 1.60 x 1019 C

the Planck constant, h = 6.63 x 1034 Js

unified atomic mass constant, u = 1.66 x 1027 kg

rest mass of electron, me = 9.11 x 1031 kg

rest mass of proton, mp = 1.67 x 1027 kg

acceleration of free fall, g = 9.81 ms2

Formulae

uniformly accelerated motion, s = ut + at2


v2 = u2 + 2as

work done on/by a gas, W = pV

hydrostatic pressure, p = gh

resistors in series, R = R1 + R2 +

resistors in parallel, 1/R = 1/R1 + 1/R2 +

SRJC 2013 8866/Prelim/2013


3 For
Examiners
Use
Section A
Answer all the questions in this section.

l
1 The formula for the period of a simple pendulum is T 2 . Such a pendulum is used to
g
determine g. The measured value of l is (98.2 0.1) cm and that of T is (1.99 0.01) s.
Determine the value of g and its associated uncertainty.

l
T 2
g
l
g 4 2
T2
0.982
4 2 9.7896
1.992

g l T
2
g l T
0.1 0.01
2
98.2 1.99
0.01107
g 0.01107(9.7896)
0.1 (1 s.f.)

g (9.8 0.1) m s -2

g g = ..... m s-2 [4]

SRJC 2013 8866/Prelim/2013 [Turn Over


4 For
Examiners
Use

2 Two carts of masses 400 g and 1200 g are free to move on a frictionless horizontal table.
Fig. 2.1 shows a spring of spring constant 500 N m-1 placed between the carts and the carts
are tied together by a thread so that the compression of the spring is 3.0 cm. The carts are
initially at rest.

thread
spring

Fig. 2.1

(a) Calculate the elastic potential energy stored in the spring when the compression is
3.0 cm.
1
Strain energy = kx 2
2
1
(500)(0.03) 2
2
0.225 J

elastic energy = ..... J [2]

SRJC 2013 8866/Prelim/2013


5 For
Examiners
Use
(b) The thread is then cut. Determine the final speeds of the two carts.

By conservation of linear momentum,


0 m1v1 m2 v2
0 0.4v1 1.2v2
0 v1 3v2
v1 3v2 (1)

By conservation of energy, strain energy is converted to KE


1 1
0.225 (0.4)v12 (1.2)v22
2 2
2 2
1.125 v1 3v2 (2)

Sub. (1) into (2),


1.125 (3v2 ) 2 3v22
1.125 9v22 3v22
v2 0.3062

v1 3(0.3062)
0.919 m s -1 [Speed =0.919 m s -1 ]

final speed of 400 g cart = ..... m s-1

final speed of 1200 g cart = ..... m s-1 [4]

SRJC 2013 8866/Prelim/2013 [Turn Over


6 For
Examiners
Use

3 (a) (i) State the principle of moments.

..
The principle of moments states that for a system in rotational equilibrium the sum of the
..
clockwise moments about any point must be equal to the sum of anticlockwise moments
about that same point.
..

.... [1]

(ii) A pair of pliers consists of 2 halves held together by a pivot. Fig 3.1 shows two
forces acting on one of the halves for a pair of pliers holding a sphere.

8N
30o

10 cm

4 cm

Fig. 3.1

Calculate the magnitude of the contact force of the sphere on the pliers, F.

Taking moments about the pivot,


ACW moments CW moments
F (4) (8cos 30o )(6)
F 10.4 N
F = ..... N [2]

SRJC 2013 8866/Prelim/2013


7 For
Examiners
Use
(b) (i) The Earth has an atmosphere which consist of a mixture of air. The average
density of air is 1.225 kg m-3 and the atmospheric pressure is 1.0 x 105 Pa at the
Earths surface.

Calculate the height of the atmosphere.

p = hg
1.0 x 105 = h(1.225)(9.81)
h = 8321 m

height = m [2]

(ii) Besides the estimates of the density and pressure of air, discuss why the answer
to part (b)(i) may not be accurate.

....
- gravitational (field strength)/acceleration is also an estimate.

.[1]

SRJC 2013 8866/Prelim/2013 [Turn Over


8 For
Examiners
Use

4 Fig. 4.1 shows two coherent electromagnetic wave emitters, M1 and M2 which emit waves in
phase. A detector is placed at X. As the frequency of the emitters gradually increases, the
detected signal shows a series of maxima and minima.

M1 X
detector
9 cm

M2
40 cm

Fig. 4.1

(i) Explain how the maxima and minima can be observed at the same point X.

.
By adjusting the frequency and hence the wavelength, of the sources
.
Maxima is observed when constructive interference takes place when the path
difference of the two waves is n times of wavelength, where n is integral
.

Minima is observed when destructive interference takes place when the path
.
difference of the two waves is (n+0.5) times of wavelength, where n is integral
.... [3]

(ii) Calculate the frequency at which the first minimum intensity is observed at X.

M 2 X 92 402 41 cm

Path difference = 41 40 = 1 cm [

For first minimum, path difference = 0.5, so = 2 cm

v 3 108
f
2 102
1.5 1010 Hz

frequency = Hz [3]

SRJC 2013 8866/Prelim/2013


9 For
Examiners
Use
5 (a) Define the term tesla.

.
If a conductor carrying a current of 1 ampere is placed at right angles to a uniform
magnetic field of flux density 1 tesla, then the force per unit length on the conductor
.
is 1 newton per metre.
.

[2]

(b) Fig. 5.1 shows a cylindrical conducting rod of length 10.0 cm resting on two
horizontal copper rails. The rails are connected to a 12 V e.m.f source. The region
between has a magnetic field of flux density 0.20 T acted perpendicularly into the
plane of the paper.

rod

x x x x x xx x x
rails x x x x x xx x x 12 V e.m.f
x x x x x xx x x source

Uniform B- Field
(0.20 T)

Fig. 5.1

(i) Sketch in Fig. 5.1 how the e.m.f source must be connected so that the rod
experiences a force pushing it to the left. [1]

(ii) 1. In order to just keep the rod stationary, the plane of the rails must be tilted at an
angle from the horizontal as shown in Fig. 5.2. The magnetic field remains in
the same direction and the frictional force between the rod and rails is negligible.

Draw and label all forces acting on the rod. [2]

mg : weight of rail
FB : Electromagnetic force
rod N
acting on rail due to
current
N : Normal reaction acted by
slope on rail FB
rail
mg

Fig. 5.2

SRJC 2013 8866/Prelim/2013 [Turn Over


10 For
Examiners
Use

2. The mass of the rod is 5.0 g, is 30 Determine the total resistance of the rod for
this set up. The rails and conducting wires are of negligible resistance.

As rod is in equilibrium, sum of forces parallel to the slope is zero.


mg sin FB cos
mg sin B Il cos
sin B Il

cos mg
tan (mg ) tan 30(0.005)(9.81)
I
Bl (0.20)(0.10)
I 1.416 A
V 12
Using R = 8.47
I 1

total resistance = .. [3]

SRJC 2013 8866/Prelim/2013


11 For
Examiners
Use
6 The circuit in Fig. 6.1 is used for an experiment to determine Plancks constant.

B V
L
R1
5V
A
R2

Fig. 6.1

A Light-Emitting Diode (LED) that gives off red light of wavelength 695 nm is used for
component L. As contact point at resistor R1 shifts from A to B, the reading across the
voltmeter also varies.

Initially, the red LED light did not light up, it was only when the voltmeter reached a reading
of V0 did it light up. V0 was then recorded. L was then replaced with LEDs that gave off
infrared light of wavelength 1000 nm, yellow light of wavelength 660 nm and green light of
wavelength 630 nm.

The data collected is as shown in table in Fig. 6.2.

LED V0 / V / nm -1 / nm-1

Infrared 0.866 1000 0.001000

Red 1.367 695 0.00144

Yellow 1.464 660 0.00152

Green 1.560 630 0.00159

Fig. 6.2
-1
(a) Fill in column on in Fig. 6.2. [1]

SRJC 2013 8866/Prelim/2013 [Turn Over


12 For
Examiners
Use

(b) Plot a graph of V0 against -1 on the grids in Fig. 6.3. [3]

V0 / V

1.500

1.400
1.560 1.080 = 0.480

1.300

1.200

1.100

0.0015900 0.0011800 = 0.0004100

1.000

0.900

0.800
0.0010 0.0011 0.0012 0.0013 0.0014 0.0015 0.0016 -1/ nm-1

Fig. 6.3

(c) State the energy which is required for an electron to move across the LED in terms of
elementary charge e and voltage across the LED V0.
eV0
.[1]

(d) The energy of each photon corresponds to the energy stated in (c). Show how the
Plancks constant can be determined from the graph in Fig. 6.3. [2]

hc
E

hc
eV0

hc 1
V0 ( )
e
hc
Hence, is the gradient of the graph.
e

SRJC 2013 8866/Prelim/2013


13 For
Examiners
Use

(e) Determine the Plancks constant from the graph in Fig. 6.3.

hc 0.480

e 0.0004100 109
0.480 1.6 1019
h
0.0004100 109 3 108
6.24 1034

Plancks constant = [3]

SRJC 2013 8866/Prelim/2013 [Turn Over


14 For
Examiners
Use

Section B
Answer two questions from this section.

-1
7 (a) A hot air balloon was rising steadily at a speed of 10.0 m s vertically. Then a constant wind
-1
blew horizontally, causing the hot air balloon to travel 5.0 m s in the same direction as the
wind while it continues to rise steadily. The balloon has a resultant velocity of v at an angle
R
to the horizontal, as shown in Fig. 7.1.

v
R

Fig. 7.1

(i) Calculate the magnitude of the resultant velocity v .


R

vR 10.02 5.02
11.2 m s -1

v = ..... m s-1 [2]


R

(ii) A sandbag was dropped from the balloon. Calculate the distance between the balloon
and sandbag after 3.0 s. (Assume that the dropping of sandbags did not affect the
velocity of the hot air balloon and that effects of air resistance on the sandbags were
negligible.)

SRJC 2013 8866/Prelim/2013


15 For
Examiners
Use
Taking downwards as positive:
For the sandbag,
s y1 ut 1 gt 2
2
(10.0)(3.0) 1 (9.81)(3.0) 2
2
14.145 m [1]

For the rising hotair balloon,


s y 2 ut
(10.0)(3.0)
30.0 m [1]

Therefore distance = 14.145 +30.0 = 44.1 m [1]

distance = ..... [3]

(iii) Two sandbags were subsequently dropped from the moving balloon. Sandbag 1 was
dropped at time t = 0 s. Sandbag 2 was dropped 1.0 s later.

1 Calculate the times for sandbags 1 and 2 to reach their respective highest points.

Time for sandbags 1 and 2 to reach their respective highest points,


v1 u1 gt1
0 10 9.81t1
t1 1.019s

t2 1 1.019 2.019s

time for sandbag 1 = .... s

time for sandbag 2 = ..... s [2]

2 Calculate the vertical velocities of sandbags 1 and 2 after 3.0 s.

SRJC 2013 8866/Prelim/2013 [Turn Over


16 For
Examiners
Use

Final velocity for sandbag 1 at 3s,


v y1 u g (3.0)
10 9.81(3.0)
19.43 m s -1

Final velocity for sandbag 2 at 3s,


v y 2 u g (2.0)
10 9.81(2.0)
9.62 m s -1

vertical velocity for sandbag 1 = ..... m s-1

vertical velocity for sandbag 2 = ..... m s-1 [2]

3 Considering only the vertical velocities vy, sketch the vy against t graphs of the
two sandbags from t = 0 s to 3.0 s. Label your graphs clearly with appropriate
values indicated. [2]

vy / m s-1

1st sandbag
19.43

9.62 2nd sandbag

1.0 1.02 2.02 3.0 t/s

-10.0

SRJC 2013 8866/Prelim/2013


17 For
Examiners
Use

(b) A stone is projected up an inclined plane at an angle of 20o with respect to the inclined plane
as shown in Fig. 7.2. The initial speed of projection is 40.0 m s-1. Use h for the vertical
displacement and x for the horizontal displacement.

40.0 m s-1
20
stone
30o

Fig. 7.2

(i) State any assumption made about the acceleration when making use of the kinematic
equations.
The acceleration is constant.
..
[1]

(ii) Using s = ut + at2, write down two equations relating the horizontal displacement h
and the vertical displacement x with time t.

vertically: Using s = ut + at2


x = 40.0 sin50o t gt2 ---(1)

horizontally: Using s = ut + at2


h = 40.0 cos50o t ---(2)

h = .....

x = ..... [2]

(iii) Write down an equation relating h and x when the stone hits the inclined plane.

o
tan 30 = x /h ---(3)
[1]

(iv)By using equations relating the horizontal and vertical displacements with time or
otherwise, show that the stone will land on the inclined plane at t = 3.22s.
[2]
SRJC 2013 8866/Prelim/2013 [Turn Over
18 For
Examiners
Use

Subst. (1) and (2) into (3):


tan 30o = (40.0 sin50o t g t2) / (40.0cos50o t)
t = 3.22 s (shown)

(v) Determine the speed of the stone on impact with the inclined plane at t = 3.22 s.

[3]

horizontally : Using v = u + at
vx = 40.0 cos50o
= 25.7 m s-1

vertically : Using v = u +at


vy = 40.0 sin50o 9.81(3.22)
= -0.946 m s-1

Therefore, velocity of stone on impact with inclined plane,


v = (vx2 + vy2)1/2
= (25.72 + 0.9462)1/2
= 25.7 m s-1

speed of stone on impact = . m s-1 [3]

SRJC 2013 8866/Prelim/2013


19 For
Examiners
Use
8 (a) Explain what is meant by

(i) a battery has an electromotive force (e.m.f.) of 2.0 V,


.
The electromotive force (e.m.f.) of 2.0 V is the 2.0 J of electrical energy converted from
.
other forms of energy per unit charge delivered round a complete circuit.

.... [1]

(ii) the potential difference (p.d.) across a resistor is 2.0 V.

.
2.0 J of electrical energy changed to some other forms of energy when one coulomb
charge passes from through the resistor.
.

.... [1]

(b) In an electrical device, the current in the electron beam between the cathode and anode is
3.0 mA. If the device is switched on for 40 s, determine

(i) the number of electrons leaving the cathode,

charge Q = I t = 3.0 x 10-3 x 40 = 0.12 C


no. of electrons = 0.12 / (1.6 x 10-19) = 7.5 x 1017

number of electrons = .. [2]

(ii) the energy supplied to the device if it is connected to a 220 V mains.


W = QV
= 0.12 x 220
= 26.4 J

energy supplied = .. J [2]

SRJC 2013 8866/Prelim/2013 [Turn Over


20 For
Examiners
Use

(c) The current-potential difference relationship for two electrical components P and Q is shown
in Fig. 6.1.

current / mA

P
8
Q
7
6
5
4
3
2
1
0
2 4 6 8 10 potential difference / V
Fig. 6.1

(i) Identify

1. Component P
Fixed Resistor / ohmic conductor / metal at constant temperature
...... [1]

2. Component Q
Thermistor
...... [1]

(ii) P and Q are connected in parallel. The current flowing through P is 6 mA.

1. State the potential difference across Q.

6V
potential difference across Q = V [1]

2. State the current flowing in Q.

3 mA current in Q = mA [1]

SRJC 2013 8866/Prelim/2013


21 For
Examiners
Use

3. Determine the effective resistance if P and Q are considered as a single component.


Resistance of P = 6 / 6 x 10-3 = 1000
Resistance of Q = 6 / 3 x 10-3 = 2000
Effective resistance
1000 2000

1000 2000
667

OR
current flowing through the component = 6 + 3 = 9 mA
Effective resistance = 6 / 9 x 10-3
= 667

effective resistance = [3]

(iii) P and Q are now connected in series with a battery of internal resistance of 2 . The
current flowing through P is 3 mA.

1. Calculate the terminal potential difference across the battery.


Terminal p.d. = 3 + 6
=9V

terminal potential difference across the battery.= V [2]

2. Calculate the rate of heat dissipated by the battery.

rate of heat dissipated by the battery = I2R = (3 x 10-3)2 x 2 = 1.80 x 10-5 W

rate of heat dissipated by the battery.= W [1]

(iv) State and explain the effect on the resistance of component P when the temperature
increases.

.
At higher temperatures, the lattice ions vibrate with larger amplitudes. The mobile
electrons collide more frequently with the lattice ions.
.
With more collisions, the current flow is impeded for a given potential difference.
.
As potential difference increases, current increases less than proportionately.
.
Hence, the resistance increases.
.

[4]
SRJC 2013 8866/Prelim/2013 [Turn Over
22 For
Examiners
Use

9 Fig. 9.1 shows a photoemissive cell consisting of the emitter and the collector plate, made of
the same metal connected to a variable d.c supply and a voltmeter and an ammeter.

I
Emitter

B
Radiation
V Variable e.m.f source
A
Collector

Fig. 9.1
2
The emitter of area 0.40 cm is illuminated with monochromatic radiation of wavelength 270 nm
and intensity 200 W m2. The current I in the circuit is measured for various values of the
applied potential difference V between the collector and emitter. The results are shown in
Fig. 9.2.

I /nA
(a)(ii)1.
]

5.0

2.5

-1.1 1.6 V /V
Fig. 9.2

(a)(i) 1. When V is negative, state which point of the variable e.m.f source (A or B) is at the
higher potential.

B
point .. [1]

2. Calculate the rate of incidence of photons on the emitter.

nhf
I
tA
nhc
IA where A is the surface area of incidence
t
n IA 200 0.40 104 270 109

t hc 6.63 1034 3.0 108
n
1.09 1016 s 1
t rate of incidence = s-1 [3]
SRJC 2013 8866/Prelim/2013
23 For
Examiners
Use

3. Calculate the work function of the emitter in eV.

= Ephoton - KEmax
hc
= eVs

(6.63 1034 3.0 108 )
= 9
(1.60 1019 )(1.1)
270 10
= 5.606 x 10-19 J
=3.50 eV

work function = eV [3]

(ii) While keeping the frequency of the radiation constant, the intensity is now halved to
100 W m2.

1. Sketch and label in Fig. 9.2 the graph which will be obtained. [2]

2. Explain the key features of the new graph in relation to the original graph.

.
Stopping potential remains the same
as energy of each photon is only affected by the frequency of
.
radiation. [1]
.
KE max of electron emitted is unchanged.
.
Saturation current is halved
.
Intensity is halved, the rate of incidence of photons on surface is
halved hence rate of emission of electrons is halved.
[3]

(b) Most street lights in Singapore use Sodium lamp. They give off light of distinct orange-
yellowish colour.

(i) Explain why the lamp gives off a distinct orange-yellowish light.
Light is being emitted when energised electrons falls from a higher orbital
.
shell to a lower one. The energy gaps of each element is distinct], for
Sodium, the orange-yellowish light is detected by our eye.
(Note: Other radiation are emitted as well, but may not be detected by eye.)
.

... [2]

SRJC 2013 8866/Prelim/2013 [Turn Over


24 For
Examiners
Use

(ii) Two different setups were used to analyse the light source from the Sodium lamp.
Fig. 9.3 shows the two visible light spectrums P and Q.

Fig. 9.3
1. Identify the type of spectrum P and Q.

P: ...
P: Emission Spectrum
Q: Absorption Spectrum
Q: [1]

2. With the aid of a diagram, describe how spectrum P is produced.

Electric field is used to excite the electrons in the Sodium atom.


When it de-excite, the light is given off .
.
Sodium vapour gives off characteristic orange-yellowish light which is
shown in P.
.
Light passes through the diffraction grating to produce the emission
spectrum.
[4]

(c) Calculate the energy of the photon emitted by the Sodium atom if the wavelength of the
light emitted is 569 nm.

E = hc /
= 6.63 x 10-34 x 3 x 108 / 569 x 10-9
= 3.50 x 10-19 J

energy of photon = . J [1]

End of paper

SRJC 2013 8866/Prelim/2013


SERANGOON JUNIOR COLLEGE
General Certificate of Education Advanced Level
Higher 2
NAME

CG INDEX NO.

PHYSICS 9646
Preliminary Examination 29th August 2013
Multiple Choice Questions 1 hr 15 mins
Additional Materials: OMS.

READ THIS INSTRUCTIONS FIRST


Write your name, civics group and index number in the spaces at the top of this page.

Write in dark blue or black pen.


You may use a soft pencil for any diagrams, graphs or rough working.
Do not use staples, paper clips, highlighters, glue or correction fluid.

There are forty questions in this section. Answer all questions. For each question there are four possible
answers A, B, C and D.
Choose the one you consider correct and record your choice in soft pencil on the OMS.

Each correct answer will score one mark. A mark will not be deducted for a wrong answer.
Any rough working should be done in this booklet.

This document consist of 16 printed pages and 0 blank page


2

DATA AND FORMULAE


Data
speed of light in free space, c = 3.00 x 108 m s1
permeability of free space, 0 = 4 x 10-7 H m-1
permittivity of free space, 0 = 8.85 x 10-12 F m-1
= (1/(36 )) x 10-9 F m-1
elementary charge, e = 1.60 x 1019 C
the Planck constant, h = 6.63 x 1034 J s
unified atomic mass constant, u = 1.66 x 1027 kg
rest mass of electron, me = 9.11 x 1031 kg
rest mass of proton, mp = 1.67 x 1027 kg
molar gas constant, R = 8.31 J K1 mol1
the Avogadro constant, NA = 6.02 x 1023 mol1
the Boltzmann constant, k = 1.38 x 10-23J K1
gravitational constant, G = 6.67 x 10-11N m2 Kg2

acceleration of free fall, g = 9.81 m s2

SRJC 2013 9646/Prelim/2013


3

Formulae
uniformly accelerated motion, s = ut + at2

v2 = u2 + 2as

work done on/by a gas, W = pV


hydrostatic pressure, p = gh
GM
gravitational potential, = -
r
displacement of particle in s.h.m., x = x0 sin t
velocity of particle in s.h.m., v = v0 cos t

= x02 - x 2
resistors in series, R = R1 + R2 +
resistors in parallel, 1/R = 1/R1 + 1/R2 +
electric potential, V = Q/ 4 0r
alternating current/ voltage, x = x0 sin t
transmission coefficient, T exp(-2kd)

8 2 m(U - E )
where k =
h2
radioactive decay, x = x0 exp(-t)

decay constant, = 0.693


t1
2

SRJC 2013 9646/Prelim/2013 [Turn Over


4 For
Examiners
Use
Answer all questions

1 A student made a series of measurements of the diameter d, of a coin using four vernier
callipers A, B, C and D. The table below shows the measurements taken. If the actual
diameter of the coin was 2.49 cm, which vernier calliper produced a set of readings that
could be described as accurate but not precise?

Vernier calliper Readings d / cm


A 2.49 2.46 2.52 2.50
B 2.48 2.58 2.51 2.40
C 2.35 2.37 2.42 2.42
D 2.32 2.37 2.41 2.50

2 The viscous drag force F of a sphere of radius r moving through a fluid with speed v is
given by F = 6rv. What are the S.I. base units of the viscosity of the fluid, ?

A kg-1 m s C kg m-1 s-1

B kg m3 s-1 D kg m-1 s-3

3 The graph below describes the motion of an object which has been thrown upwards in the
gravitational field of the Earth.

What could the physical quantities Y and X represent respectively?

A Displacement, time C Displacement, kinetic energy


B Time, kinetic energy D Time, velocity

SRJC 2013 9646/Prelim/2013


5

4 A boy holding a ball is standing on the floor of a lift. The lift then starts accelerating from
rest at 4.9 m s-2 upwards. After 2.0 s, he releases the ball at 1.2 m with respect to the floor
of the lift. At the moment the ball is released, the lift comes to a sudden complete stop.
What is the time taken for the ball to reach the floor of the lift?

A 0.49 s B 1.1 s C 2.1 s D 3.0 s

5 Two identical cylindrical bar magnets are stored in a light plastic frictionless cylinder of
negligible mass. When the magnets are arranged as shown in the figure below and
weighed, the balance reads W. (The whole system is at rest.)

Light,
frictionless
plastic tube

top-pan
balance

If the mass of each magnet is M, which of the following is correct?


A W = Mg B Mg < W < 2Mg C W = 2Mg D W > 2Mg

6 A 5.00 kg object moves at 15.0 m s1. It collides perfectly inelastically with a 10.0 kg object
which was at rest. How much kinetic energy is lost in the collision?

A 188 J B 375 J C 563 J D 702 J

7 The scale of a certain spring balance reads from 0 to 100 N. When the balance reads 100 N,
the extension in the spring is 0.1 m long. Calculate the strain energy stored in the spring
when the balance reads 70 N. The spring obeys Hookes Law.

A 0.07 J B 2.45 J C 2.55 J D 4.90 J

SRJC 2013 9646/Prelim/2013 [Turn Over


6 For
Examiners
Use
8 A heavy uniform rod of length l is supported by two cords attached to the ceiling as shown.

T1 T2

3 1
l l
4 4

T1
What is the ratio of the tensions in these cords?
T2

sin cos 1 sin 1 cos


A B C D
sin cos 2 sin 2 cos

9 An ideal gas within a tube was compressed with a piston as shown below without a change
in pressure within the tube.

4.0 x 104 Pa 1.0 x 105 Pa

5.0 cm

The pressure within the tube is 4.0 x 104 Pa and the pressure of the atmosphere outside is
1.0 x 105 Pa. The cross-sectional area of the piston is 2.0 x 10-4 m2.

What is the work done on the gas by the atmosphere, when the piston compresses the gas
by 5.0 cm?

A - 0.6 J B 0.4 J C 0.6 J D 1.0 J

10 An 8 W Light Emitting Diode (LED) and 15 W compact fluorescent (CFL) each generates
approximately the same light output. The LED generates heat of 3.4 Btu per hour
(1 Btu = 1055 J) and the CFL generates 30 Btu per hour.

What is the difference in efficiency of the two lighting device?

A 41 % B 46 % C 54 % D 88 %

SRJC 2013 9646/Prelim/2013


7

11 Which of the following statements must correctly describe the movement of an object in a
uniform force field?

A An electron travelling between two plates of different potential with an initial velocity
perpendicular to electric field lines will move in a curved circular path with constant
acceleration.

B An alpha particle moving parallel to magnetic field lines will move in a circular path with
constant acceleration.

C A proton moving perpendicular to a magnetic field will move in a curved circular path
with changing acceleration.

D A ball thrown horizontally in free fall will move in a parabolic path with increasing
acceleration.

12 A ball of 2 g was attached to a 50 cm rod and swung in a vertical circular motion freely. It
reaches a maximum speed of 5 m s-1 at the bottom of the vertical motion.

What is the magnitude of the force exerted by the rod on the ball at the highest point?

A 1.9 x 10-3 N B 8.0 x 10-2 N C 1.0 x 10-1 N D 1.2 x 10-1 N

13 The gravitational potential at various distances from a planet Xs surface is shown in the
table below.

Distance from surface of planet X / 103 km Gravitational potential / J kg-1


10 3.92 x 108
20 2.47 x 108
30 1.80 x 108
40 1.42 x 108

What is the best estimate of the field strength at a distance 20,000 km from the surface of
planet X?

A 7.07 N kg-1 B 8.3 N kg-1 C 10.6 N kg-1 D 12.4 N kg-1

14 Which of the following is a correct description of a geostationary orbit?

Mass of Earth = 6.00 x 1024 kg.

A The moon is a geostationary satellite of Earth.

B A geostationary satellite has an orbital circumference of 2.66 x 108 m.

C A geostationary satellite moves from North pole to South pole in 24 hours.

D A geostationary satellite moves from east to west.

SRJC 2013 9646/Prelim/2013 [Turn Over


8 For
Examiners
Use
15 An object in simple harmonic motion is at a position of 0.20x0 with speed v, where x0 is the
amplitude of the oscillation. The period of oscillation is T.

What is its position when its speed is 0.50v?

A 0.98 x0 B 0.87 x0 C 0.50 x0 D 0.25 x0

16 Which of the following is the most accurate description of a graph reflecting the variation of
the amplitude of a forced oscillation and the driving frequency of the force?

A The amplitude increases when the driving frequency increases.

B Damping will reduce the frequency at which the highest amplitude occurs.

C It is impossible to have the same amplitude with different driving frequencies.

D The displacement increases then decreases when the driving frequency increases.

17 A cup was inverted into a pool of water. It traps 250 cm3 of air within it when it is near the
surface of the water. The cup is then forced to go deeper into the pool while still inverted.

Estimate the volume of the trapped air when it is 2 m deep into the pool.
(Density of water = 1000 kg m-3, atmospheric pressure = 1.0 x 105 Pa)

A 150 cm3 B 200 cm3 C 250 cm3 D 300 cm3

18 The pressure P and density at the core of the sun is estimated to be 1.95 x 1016 Pa and
150 x 103 kg m-3. The mass of a hydrogen atom is 1.67 x 10-27 kg and

P = 1/3 <c2>

where P is the pressure of the ideal gas,


is the density of the ideal gas, and
<c2> is mean square speed of the ideal gas.

If the hydrogen in the core is considered ideal and monatomic, what is the temperature at the
core of the sun?

A 250 000 K B 560 000 K C 1 800 000 K D 16 000 000 K

19 A siren on top of a tall building is taken to be a point source and radiates sound waves
uniformly in all directions. At a distance 4x, the amplitude of the wave is A.

What is the distance from the siren at the point where the amplitude of the wave is 2A?

A x B 2x C 8x D 16x

SRJC 2013 9646/Prelim/2013


9

20 A graphical representation of a progressive longitudinal wave is shown below.

displacement / m

(right)
X Z
0 distance / m

(left)
Y

Which one of the following set of statements is the most appropriate description of the wave
in the figure above?

A Position X of the longitudinal wave at this instant is at high pressure.


B Position Z of the longitudinal wave at this instant is at high pressure.
C Positions X and Z of the longitudinal wave are the nodes of the wave.
D Position Y at this instant is at the longitudinal wave's antinode.

21 Which one of the followings would produce a stable and observable interference pattern?

A Two laser beams from two different sources


B Two filament lamps
C Two speakers fed by two signal generators producing signals with different
frequencies
D Two dippers attached to one oscillator in a ripple tank

22 In a Youngs double slit experiment as shown below, a pattern of equally spaced parallel
interference fringes appears on the screen. The slit width is p while the separation between
the slits is q.

p
screen
q
p

coherent light

Which one of the following changes would cause the separation of interference fringes to be
doubled?

A Slit width p is halved. B Slit width p is doubled.


C Slit separation q is halved. D Slit separation q is doubled.
SRJC 2013 9646/Prelim/2013 [Turn Over
10 For
Examiners
Use
23 Two point charges of +Q and +2Q are lined up in a vertical straight line as shown below. The
distance between them is r.

+Q 1
r
3
X
r

+2Q

What is the electric potential energy of a point charge +3Q when it is placed at position X, a
1
distance r from +Q?
3
3Q 3Q 9Q 2 9Q 2
A B C D
2 0 r 2 0 r 2 2 0 r 2 0 r 2

24 The diagram shows the electric field lines due to two charged parallel plates.

X Y

Which of the following statements must always be true?

A The upper plate is at a positive potential and the lower plate is at a negative
potential.
B A proton at Z experiences a greater force than if it were placed at Y.
C A proton at Z would experience the same force if it were placed at X.
D A proton at Z experiences less force than if it were placed at Y.

25 Wire X has resistance R. Another wire Y, of the same material, has double the length and
double the diameter of wire X.

The resistance of wire Y is


R R
A B C 2R D 4R
4 2

SRJC 2013 9646/Prelim/2013


11

26 A battery, with an e.m.f E and internal resistance r, is connected to a switch S and two
identical resistors in series. Each resistor has resistance R.

S
R R

Which one of the following statements is correct when the switch S is closed?
A The voltmeter reading is 0.5E when an ideal voltmeter is connected across one
resistor.
The voltmeter reading is E when an ideal voltmeter is connected across two
B
resistors.
The voltmeter reading is E when an ideal voltmeter is connected across the
C
battery.
D The voltmeter reading is less than E when an ideal voltmeter is connected
across the battery.

27 With 4 resistors, each having a resistance of 12 , it is impossible to arrange all 4


resistors to have an effective resistance of

A 9 B 20 C 24 D 30

28 The resistance R of an unknown resistor is compared with a fixed 200 resistor using the
circuit as shown in the diagram. The balance length l is 20.0 cm and 50.0 cm when J is
connected to K and L respectively. The length of the wire AB is 100 cm. The two cells E 1
and E 2 have negligible internal resistances.

E1

l
A B
J
K
L
200 R

E2

The resistance of R is

A 80 B 200 C 300 D 500

SRJC 2013 9646/Prelim/2013 [Turn Over


12 For
Examiners
Use
29 An electron is instantaneously moving with a velocity v along the plane of the paper. It is
moving in a uniform magnetic field. The field is perpendicular to the plane of the paper and
going into the page.


electron

v

Which of the following statements is correct?

A The magnitude of momentum of the electron will remain constant.

B The work done by the net force exerted on the electron is a non-zero constant.

C The initial force on the electron is vertically upwards.

D The magnetic force exerted on the electron may not always be perpendicular to the
magnetic field.

30 A system is designed to control the brightness of a light bulb. Coil 1 and coil 2 are not
electrically connected.

Alternating e.m.f source


Light bulb

R1

Soft iron core

coil 1 coil 2

Which combination of actions will result in the dimmest possible light,

A R1 is reduced and the soft iron core is pulled out of coil 2.

B R1 is increased and the soft iron core is pushed into coil 2.

C R1 is increased and the soft iron core is pulled out of coil 2.

D R1 is reduced and the soft iron core is pushed out of coil 2.

SRJC 2013 9646/Prelim/2013


13

31 2 square conducting coils of equal dimensions are moving in or out of the region of a
uniform magnetic field represented by the dotted box below. The magnetic flux density is
directed into the paper. The direction of motion of the coils is indicated by the arrows and
both coils are travelling with the same constant velocity.

B-field (directed into the paper)

1 2

Which of the following statements is false?

A Coil 2 experiences a magnetic force in the same direction as coil 1.


B Coil 1 has an induced current which is opposite to the direction of the induced
current in coil 2.
C Coil 1 has an induced current which is decreasing in magnitude.
D Coil 2 has an induced current which is decreasing in magnitude.

32 The figure below shows the side and top view of a coil of wire rotating in a magnetic uniform
magnetic field.


N S N S
s s s s
Side View Top View
s s
The graph of how the angle and how it varies with time t is shown below.
/

270
s
180
s
90
s
0 0.1 0.2 0.3 t/s
s s s s s

Here are 3 statements regarding the coil.

i the magnitude of the angular velocity is a constant.


ii the maximum magnitude of induced e.m.f occurs at 0.4 s.
iii the maximum magnitude of flux linkage experienced occurs at 0.4 s.

. Which statement(s) is true?

A i B ii C i & ii D all of the above

SRJC 2013 9646/Prelim/2013 [Turn Over


14 For
Examiners
Use
33 A full wave sinusoidal current of root mean square current I dissipates power in a resistor R
at a maximum instantaneous rate of P.

The same sinusoidal current now passes through an ideal diode such that the current is half-
wave rectified before passing through resistor R.

The new values for the root mean square current in the resistor R and the mean power
dissipated are

Root Mean Square current Mean power


I P
A
2 2

I P
B
2 4

I P
C
2 4

I P
D
2 2

34 When a steady direct current of value I passes through a heating element of resistance R,
the time taken to raise water of mass m by 10 K is time T.

When an alternating current with a peak current of value I passes through a heating
element of resistance 2R, the time taken to raise water of mass m by 10 K is

T T
A 2T B T C D
2 4

SRJC 2013 9646/Prelim/2013


15

35 In a photoelectric experiment, two different metal electrodes were illuminated initially by blue
radiation as shown in the circuit.
radiation source

metal metal
B A
A

e.m.f sorce

There was an initial reading in the ammeter. It was realised that when the radiation source
was changed to a green radiation with the same power rating, the reading in the ammeter
remains unchanged. This may be due to

A the electrons being emitted from metal A never reaching metal B.


B the work function of metal B being much higher than that of metal A.
C the increase in number of electrons ejected from metal A and a decrease in the
number of electrons ejected from metal B.
D the increase in number of electrons ejected from metal B and a decrease in the
number of electrons ejected from metal A.

36 Which of the following statements concerning semiconductors is correct?

One of the main features which explains the electrical properties of semiconductors is that

A there are a lot of electrons in the conduction band.


B it has a small energy band gap between the conduction and valence band.
C the conduction band is usually wide for electrons to move freely.
D it has a fully filled valence band.

37 In order to further improve the conductivity of an extrinsic p-type silicon, doping is necessary.

Which of the following statements is correct?

Conductivity can be further improved

A by doping the silicon with a trivalent atom.


B by doping the silicon with a tetravalent atom.
C by doping the silicon with a pentavalent atom.
D by doping the silicon with electrons.

SRJC 2013 9646/Prelim/2013 [Turn Over


16 For
Examiners
Use
38 In a collision between an excited Helium atom and a Neon atom, the Neon atom which is at
the ground state may be excited. The energy levels of Helium and Neon are given below.

20.61 eV 20.66 eV
18.70 eV

Common ground state


(0 eV)
Helium Neon
Which of the following statements is correct?

A Energy cannot be directly transferred by the collision process. Only photons are able to
cause excitation of the Neon atom.
B For the excited Neon atom, it is only possible to emit a photon of energy 18.70 eV.
C For the excited Neon atom, it is only possible to emit a photon of energy 18.70 eV or
20.66 eV.
D For the excited Neon atom, it is only possible to emit a photon of energy 1.96 eV, 18.70
eV or 20.66 eV.

39 The following nuclear equation shows one possible outcome when Uranium-235 is
bombarded by a neutron.

235
92 U+ 01n 97 M
40 Zr + 52Tc +bX where b is an integer.

What is the value of M and the particle X?

M X
A 135 Helium
B 137 neutron
C 139 electron
D 141 photon

40 A student placed 3 radioactive sources into an aluminium container of 10 mm thick. He uses


a Geiger-Muller tube to read the radiation outside the container. The Geiger-Muller counter
register counts from

A gamma radiation only.


B beta particles and gamma radiation only.
C alpha particles, beta particles and gamma radiation.
D none of the above are correct.

End of Paper

SRJC 2013 9646/Prelim/2013


SERANGOON JUNIOR COLLEGE
General Certificate of Education Advanced Level
Higher 2
NAME

CG INDEX NO.

PHYSICS 9646
Preliminary Examination (ANS) 29th August 2013
Multiple Choice Questions 1 hr 15 mins
Additional Materials: OMS.

READ THIS INSTRUCTIONS FIRST


Write your name, civics group and index number in the spaces at the top of this page.

Write in dark blue or black pen.


You may use a soft pencil for any diagrams, graphs or rough working.
Do not use staples, paper clips, highlighters, glue or correction fluid.

There are forty questions in this section. Answer all questions. For each question there are four possible
answers A, B, C and D.
Choose the one you consider correct and record your choice in soft pencil on the OMS.

Each correct answer will score one mark. A mark will not be deducted for a wrong answer.
Any rough working should be done in this booklet.

This document consist of 16 printed pages and 0 blank page


2

DATA AND FORMULAE


Data
speed of light in free space, c = 3.00 x 108 m s1
permeability of free space, 0 = 4 x 10-7 H m-1
permittivity of free space, 0 = 8.85 x 10-12 F m-1
= (1/(36 )) x 10-9 F m-1
elementary charge, e = 1.60 x 1019 C
the Planck constant, h = 6.63 x 1034 J s
unified atomic mass constant, u = 1.66 x 1027 kg
rest mass of electron, me = 9.11 x 1031 kg
rest mass of proton, mp = 1.67 x 1027 kg
molar gas constant, R = 8.31 J K1 mol1
the Avogadro constant, NA = 6.02 x 1023 mol1
the Boltzmann constant, k = 1.38 x 10-23J K1
gravitational constant, G = 6.67 x 10-11N m2 Kg2

acceleration of free fall, g = 9.81 m s2

SRJC 2013 9646/Prelim/2013


3

Formulae
uniformly accelerated motion, s = ut + at2

v2 = u2 + 2as

work done on/by a gas, W = pV


hydrostatic pressure, p = gh
GM
gravitational potential, = -
r
displacement of particle in s.h.m., x = x0 sin t
velocity of particle in s.h.m., v = v0 cos t

= x02 - x 2
resistors in series, R = R1 + R2 +
resistors in parallel, 1/R = 1/R1 + 1/R2 +
electric potential, V = Q/ 4 0r
alternating current/ voltage, x = x0 sin t
transmission coefficient, T exp(-2kd)

8 2 m(U - E )
where k =
h2
radioactive decay, x = x0 exp(-t)

decay constant, = 0.693


t1
2

SRJC 2013 9646/Prelim/2013 [Turn Over


4

Answer all questions

1 A student made a series of measurements of the diameter d, of a coin using four vernier
callipers A, B, C and D. The table below shows the measurements taken. If the actual
diameter of the coin was 2.49 cm, which vernier calliper produced a set of readings that
could be described as accurate but not precise?

Vernier calliper Readings d / cm


A 2.49 2.46 2.52 2.50
B 2.48 2.58 2.51 2.40
C 2.35 2.37 2.42 2.42
D 2.32 2.37 2.41 2.50

Vernier calliper Mean d /cm Spread / cm


A 2.49 2.52 2.46 = 0.06
B 2.49 2.58 2.40 = 0.18
C 2.39 2.42 2.35 = 0.07
D 2.40 2.50 2.32 = 0.18
Ans: B. Readings from instrument B have the closest
mean to the actual reading (most accurate) and the
biggest spread (least precise).

2 The viscous drag force F of a sphere of radius r moving through a fluid with speed v is
given by F = 6rv. What are the S.I. base units of the viscosity of the fluid, ?

A kg-1 m s C kg m-1 s-1

B kg m3 s-1 D kg m-1 s-3

Ans: C
= F/(6rv)
[] = [F]/[rv]
= kg m s-2 m-1 m-1 s
= kg m-1 s-1

SRJC 2013 9646/MYE/2013


5

3 The graph below describes the motion of an object which has been thrown upwards in the
gravitational field of the Earth.

What could the physical quantities Y and X represent respectively?

A Displacement, time C Displacement, kinetic energy


B Time, kinetic energy D Time, velocity

Ans: A. Y-X has the shape of a displacement-time graph. (It could also
represent the potential energy-time graph, since potential energy is proportional
to displacement.)

4 A boy holding a ball is standing on the floor of a lift. The lift then starts accelerating from
rest at 4.9 m s-2 upwards. After 2.0 s, he releases the ball at 1.2 m with respect to the floor
of the lift. At the moment the ball is released, the lift comes to a sudden complete stop.
What is the time taken for the ball to reach the floor of the lift?

A 0.49 s B 1.1 s C 2.1 s D 3.0 s

Ans: C
After 2.0 s, the ball acquires a velocity of v = u + at
= 0 + (4.9)(2.0)
= 9.8 m s-1 upward

Taking the downward as +ve, and using s = ut + (1/2)at2 and a = g since the
ball the acceleration due to gravity alone.

1.2 = -9.8t + (1/2)gt2


t = 2.1 s

SRJC 2013 9646/Prelim/2013 [Turn Over


6

5 Two identical cylindrical bar magnets are stored in a light plastic frictionless cylinder of
negligible mass. When the magnets are arranged as shown in the figure below and
weighed, the balance reads W. (The whole system is at rest.)

Light,
frictionless
plastic tube

top-pan
balance

If the mass of each magnet is M, which of the following is correct?


A W = Mg B Mg < W < 2Mg C W = 2Mg D W > 2Mg

Ans: C.
FBD for top magnet
Fmagnet Mg --(1) Mg

Fmagnet

FBD for bottom magnet


Fmagnet
Fmagnet Mg W --(2)
Subst (1) into (2),
Mg
W Mg Mg 2Mg
W

SRJC 2013 9646/MYE/2013


7

6 A 5.00 kg object moves at 15.0 m s1. It collides perfectly inelastically with a 10.0 kg object
which was at rest. How much kinetic energy is lost in the collision?

A 188 J B 375 J C 563 J D 702 J

Ans: B
v
u1 = 15.0 m s 1 u2 = 0 m s1

m1 = 5.00 kg
m2 = 10.0 kg
mf = 15.0 kg

Using conservation of momentum,


m1u1 m2u 2 mf v
5.00(15.0) 10.0(0) 15.0(v )
v 5.00 m s1
1 2 1
m1u1 mf v 2
Loss of kinetic energy = 2 2
1 1
(5.00)(15.0) 2 (15.0)(5.00) 2
2 2
375 J

7 The scale of a certain spring balance reads from 0 to 100 N. When the balance reads
100 N, the extension in the spring is 0.1 m long. Calculate the strain energy stored in the
spring when the balance reads 70 N. The spring obeys Hookes Law.

A 0.07 J B 2.45 J C 2.55 J D 4.90 J

Ans: B

F1 = kx1
k = 100 / 0.1 F/N
= 1000 Nm1 100
When F2 = kx2 = 70 N,
x2 = 70 / 1000 70
= 0.07 m

Energy = Area A (under F-x graph)


A
= 70 0.07 0.1 x/m
= 2.45 J
x2

SRJC 2013 9646/Prelim/2013 [Turn Over


8

8 A heavy uniform rod of length l is supported by two cords attached to the ceiling as shown.

T1 T2

3 1
l l
4 4

T1
What is the ratio of the tensions in these cords?
T2

sin cos 1 sin 1 cos


A B C D
sin cos 2 sin 2 cos

Ans: C
Taking moments about the centre of mass of the rod,
1 1 1
T1 sin l T2 sin l l
2 2 4
T1 1 sin

T2 2 sin

SRJC 2013 9646/MYE/2013


9

9 An ideal gas within a tube was compressed with a piston as shown below without a change
in pressure within the tube.

4.0 x 104 Pa 1.0 x 105 Pa

5.0 cm

The pressure within the tube is 4.0 x 104 Pa and the pressure of the atmosphere outside is
1.0 x 105 Pa. The cross-sectional area of the piston is 2.0 x 10-4 m2.

What is the work done on the gas by the atmosphere, when the piston compresses the gas
by 5.0 cm?

A - 0.6 J B 0.4 J C 0.6 J D 1.0 J

Ans: B
Wk done on gas = 4.0 x 104 x 5 x 10-2 x 2.0 x 10-4 = 0.4 J

10 An 8 W Light Emitting Diode (LED) and 15 W compact fluorescent (CFL) each generates
approximately the same light output. The LED generates heat of 3.4 Btu per hour
(1 Btu = 1055 J) and the CFL generates 30 Btu per hour.

What is the difference in efficiency of the two lighting device?

A 41 % B 46 % C 54 % D 88 %

Ans: B
(8 3600) (3.4 1055)
Efficiency of LED = 100% 87.5%
(8 3600)
(15 3600) (30 1055)
Efficiency of CFL = 100% 41.4%
(15 3600)
Difference = 87.5 % - 41.4 % 46%

SRJC 2013 9646/Prelim/2013 [Turn Over


10

11 Which of the following statements must correctly describe the movement of an object in a
uniform force field?

A An electron travelling between two plates of different potential with an initial velocity
perpendicular to electric field lines will move in a curved circular path with constant
acceleration.

B An alpha particle moving parallel to magnetic field lines will move in a circular path with
constant acceleration.

C A proton moving perpendicular to a magnetic field will move in a curved circular path
with changing acceleration.

D A ball thrown horizontally in free fall will move in a parabolic path with increasing
acceleration.

Ans: C

12 A ball of 2 g was attached to a 50 cm rod and swung in a vertical circular motion freely. It
reaches a maximum speed of 5 m s-1 at the bottom of the vertical motion.

What is the magnitude of the force exerted by the rod on the ball at the highest point?

A 1.9 x 10-3 N B 8.0 x 10-2 N C 1.0 x 10-1 N D 1.2 x 10-1 N

Ans: A
Conservation of energy:
1 2 1
mv mgh = mu 2
2 2
1 2 1
v (9.81)(1) = (5) 2
2 2
-1
v 2.32 m s
mv 2
T mg
r
(2.32) 2
T 2 103 [ (9.81)] 1.9 103 N
0.50

SRJC 2013 9646/Prelim/2013


11

13 The gravitational potential at various distances from a planet Xs surface is shown in the
table below.

Distance from surface of planet X / 103 km Gravitational potential / J kg-1


10 3.92 x 108
20 2.47 x 108
30 1.80 x 108
40 1.42 x 108

What is the best estimate of the field strength at a distance 20,000 km from the surface of
planet X?

A 7.07 N kg-1 B 8.3 N kg-1 C 10.6 N kg-1 D 12.4 N kg-1

Ans: C
dV (3.92 1.80) 108
g 3
10.6 N kg -1
dr 20000 10

14 Which of the following is a correct description of a geostationary orbit?

Mass of Earth = 6.00 x 1024 kg.

A The moon is a geostationary satellite of Earth.

B A geostationary satellite has an orbital circumference of 2.66 x 108 m.

C A geostationary satellite moves from North pole to South pole in 24 hours.

D A geostationary satellite moves from east to west.

Ans: B

15 An object in simple harmonic motion is at a position of 0.20x0 with speed v, where x0 is the
amplitude of the oscillation. The period of oscillation is T.

What is its position when its speed is 0.50v?

A 0.98 x0 B 0.87 x0 C 0.50 x0 D 0.25 x0


Ans: B
At 0.20 x0 ,
v x02 x 2 x02 (0.20 x0 ) 2 0.96 x02 _(1)
At 0.50v,
0.50v x02 x 2 _(2)
(2) (1),
x02 x 2
0.50
0.96 x02
SRJC 2013 9646/Prelim/2013 [Turn Over
x 0.872 x0
12

16 Which of the following is the most accurate description of a graph reflecting the variation of
the amplitude of a forced oscillation and the driving frequency of the force?

A The amplitude increases when the driving frequency increases.

B Damping will reduce the frequency at which the highest amplitude occurs.

C It is impossible to have the same amplitude with different driving frequencies.

D The displacement increases then decreases when the driving frequency increases.

Ans: B

17 A cup was inverted into a pool of water. It traps 250 cm3 of air within it when it is near the
surface of the water. The cup is then forced to go deeper into the pool while still inverted.

Estimate the volume of the trapped air when it is 2 m deep into the pool.
(Density of water = 1000 kg m-3, atmospheric pressure = 1.0 x 105 Pa)

A 150 cm3
B 200 cm3
C 250 cm3
D 300 cm3
Ans: B
5
= 2 (1000) (9.81) + 1 x 105 = 119620 Pa

Assume no change in temperature of the water,


p1V1 = p2V2
1 x 105 x 250 = 119620 V2
V2 = 209 cm3

18 The pressure P and density at the core of the sun is estimated to be 1.95 x 1016 Pa and
150 x 103 kg m-3. The mass of a hydrogen atom is 1.67 x 10-27 kg and

P = 1/3 <c2>

where P is the pressure of the ideal gas,


is the density of the ideal gas, and
<c2> is mean square speed of the ideal gas.

If the hydrogen in the core is considered ideal and monatomic, what is the temperature at the
core of the sun?

A 250 000 K B 560 000 K C 1 800 000 K D 16 000 000 K


Ans: D
2
P >
16
1.95 x 10 = 1/3 (150 000) <c2>
<c2> = 3.90 x 1011 m2 s-4

1/2 mH <c2> = 3/2 kT


SRJC1/2 (1.67 x 10-27) (3.89 x 1011) = 3/2
2013 (1.38 x 10-23) T
9646/Prelim/2013
6
T = 15.7 x 10 K
13

19 A siren on top of a tall building is taken to be a point source and radiates sound waves
uniformly in all directions. At a distance 4x, the amplitude of the wave is A.
What is the distance from the siren at the point where the amplitude of the wave is 2A?

Ax B 2x C 8x D 16x

Ans: B
I A2
1
I
r2
1
A
r
When the amplitude is doubled, the distance is halved.

20 A graphical representation of a progressive longitudinal wave is shown below.

displacement / m

(right)
X Z
0 distance / m

(left)
Y

Which one of the following set of statements is the most appropriate description of the wave
in the figure above?

A Position X of the longitudinal wave at this instant is at high pressure.


B Position Z of the longitudinal wave at this instant is at high pressure.
C Positions X and Z of the longitudinal wave are the nodes of the wave.
D Position Y at this instant is at the longitudinal wave's antinode.

Answer: A

SRJC 2013 9646/Prelim/2013 [Turn Over


14

21 Which one of the followings would produce a stable and observable interference pattern?

A Two laser beams from two different sources


B Two filament lamps
C Two speakers fed by two signal generators producing signals with different
frequencies
D Two dippers attached to one oscillator in a ripple tank

Ans: D

22 In a Youngs double slit experiment as shown below, a pattern of equally spaced parallel
interference fringes appears on the screen. The slit width is p while the separation between
the slits is q.

p
screen
q
p

coherent light

Which one of the following changes would cause the separation of interference fringes to be
doubled?

A Slit width p is halved.


B Slit width p is doubled.
C Slit separation q is halved.
D Slit separation q is doubled.

Ans: C

D
x where a is slit separation. For x to be doubled, slit separation has to be halved.
a

SRJC 2013 9646/Prelim/2013


15

23 Two point charges of +Q and +2Q are lined up in a vertical straight line as shown below. The
distance between them is r.

+Q 1
r
3
X
r

+2Q

What is the electric potential energy of a point charge +3Q when it is placed at position X, a
1
distance r from +Q?
3
3Q 3Q 9Q 2 9Q 2
A B C D
2 0 r 2 0 r 2 2 0 r 2 0 r 2
Ans: C

Q 2Q 3Q
V= + =
1 2
4 0r 4 0 r 2 0 r
3 3
9Q 2
U = qV =
2 0 r

24 The diagram shows the electric field lines due to two charged parallel plates.

X Y

Which of the following statements must always be true?

A The upper plate is at a positive potential and the lower plate is at a negative
potential.
B A proton at Z experiences a greater force than if it were placed at Y.
C A proton at Z would experience the same force if it were placed at X.
D A proton at Z experiences less force than if it were placed at Y.

Ans: C
The field is uniform, so the force on charged particle in the field is the same.
SRJC 2013 9646/Prelim/2013 [Turn Over
16

25 Wire X has resistance R. Another wire Y, of the same material, has double the length and
double the diameter of wire X.

The resistance of wire Y is


R R
A B C 2R D 4R
4 2

Ans: B
l
R
r 2
(2l ) l R

(2r ) 2
2 r 2
2

26 A battery, with an e.m.f E and internal resistance r, is connected to a switch S and two
identical resistors in series. Each resistor has resistance R.

S
R R

Which one of the following statements is correct when the switch S is closed?
A The voltmeter reading is 0.5E when an ideal voltmeter is connected across one
resistor.
The voltmeter reading is E when an ideal voltmeter is connected across two
B
resistors.
The voltmeter reading is E when an ideal voltmeter is connected across the
C
battery.
D The voltmeter reading is less than E when an ideal voltmeter is connected
across the battery.

Ans: D
For closed circuits, terminal p.d. is lower when there is internal resistance in the source
(non-ideal).

SRJC 2013 9646/Prelim/2013


17

27 With 4 resistors, each having a resistance of 12 , it is impossible to arrange all 4


resistors to have an effective resistance of

A 9 B 20 C 24 D 30

Ans: C

20 9

30

SRJC 2013 9646/Prelim/2013 [Turn Over


18

28 The resistance R of an unknown resistor is compared with a fixed 200 resistor using the
circuit as shown in the diagram. The balance length l is 20.0 cm and 50.0 cm when J is
connected to K and L respectively. The length of the wire AB is 100 cm. The two cells E 1
and E 2 have negligible internal resistances.

E1

l
A B
J
K
L
200 R

E2

The resistance of R is

A 80 B 200 C 300 D 500

Ans: C
When J is connected to K
At Null deflection.
VAJ = V200
LAJ R200 20 200
E1 E2 E1 E2 ------- eq(1)
LAB R200 R 100 200 R
When J is connected to L
At Null deflection.
VAJ = E2
LAJ 50
E1 E2 E1 E2 -------------- eq(2)
LAB 100
Eq(1)/Eq(2)
0.2 200

0.5 200 R
R = 300

SRJC 2013 9646/Prelim/2013


19

29 An electron is instantaneously moving with a velocity v along the plane of the paper. It is
moving in a uniform magnetic field which is perpendicular to the plane of the paper and
going into the page.


electron

v

Which of the following statements is correct?

A The magnitude of momentum of the electron will remain constant.

B The work done by the net force exerted on the electron is a non-zero constant.

C The initial force on the electron is vertically upwards.

D The magnetic force exerted on the electron may not always be perpendicular to the
magnetic field.

Ans: A
As the force by magnetic field is always perpendicular to the motion of the
electron, the work done by the magnetic force is always zero. There is no
change in the speed of the electron, hence the momentum of the electron
remains constant.
(When using Flemings Left Hand Rule to determine the direction of the
force on the electron, remember that the electron is negatively charged,
hence the direction of current is opposite of direction of motion,

SRJC 2013 9646/Prelim/2013 [Turn Over


20

30 A system is designed to control the brightness of a light bulb. Coil 1 and coil 2 are not
electrically connected.

Alternating e.m.f source


Light bulb

R1

Soft iron core

coil 1 coil 2

Which combination of actions will result in the dimmest possible light,

A R1 is reduced and the soft iron core is pulled out of coil 2.

B R1 is increased and the soft iron core is pushed into coil 2.

C R1 is increased and the soft iron core is pulled out of coil 2.

D R1 is reduced and the soft iron core is pushed out of coil 2.

Ans: C
The iron core should be pulled out of coil 2 as the soft iron core
reinforces the B-field created by coil 1 passing into coil 2.
By removing the soft iron core from coil 2, the B-field experienced by coil 2
is reduced, thus the induced e.m.f will also drop in magnitude.
R1 should be increased to reduce the current flowing through coil 1, thus
reducing the strength of the B-field generated by coil 1.

SRJC 2013 9646/Prelim/2013


21

31 2 square conducting coils of equal dimensions are moving in or out of the region of a
uniform magnetic field represented by the dotted box below. The magnetic flux density is
directed into the paper. The direction of motion of the coils is indicated by the arrows and
both coils are travelling with the same constant velocity.

B-field (directed into the paper)

1 2

Which of the following statements is false?

A Coil 2 experiences a magnetic force in the same direction as coil 1.


B Coil 1 has an induced current which is opposite to the direction of the induced
current in coil 2.
C Coil 1 has an induced current which is decreasing in magnitude.
D Coil 2 has an induced current which is decreasing in magnitude.

Ans: D
A is true as the magnetic forces will be towards the left for both coils.
B is true as the induced current in coil 1 is anticlockwise while the induced
current in coil 2 is clockwise.
C is true as the increase in area entering into the B-Field is at a
decreasing rate, hence rate of change of flux linkage is decreasing,
induced e.m.f and hence current is decreasing.

SRJC 2013 9646/Prelim/2013 [Turn Over


22

32 The figure below shows the side and top view of a coil of wire rotating in a magnetic uniform
magnetic field.


N S N S
s s s s
Side View Top View
s s
The graph of how the angle and how it varies with time t is shown below.
/

270
s
180
s
90
s
0 0.1 0.2 0.3 t/s
s s s s s

Here are 3 statements regarding the coil.

i the magnitude of the angular velocity is a constant.

ii the maximum magnitude of induced e.m.f occurs at 0.4 s.

iii the maximum magnitude of flux linkage experienced occurs at 0.4 s.

. Which statement(s) is true?

A i B ii C i & ii D all of the above

Ans: C

i is true. As , from graph it is clear that angular velocity is a
t
constant.
ii is true. At 0.4 s, the angle rotated is 360, the rate of cutting of magnetic
flux linkage is the largest possible.
iii is false. At 0.4 s, the angle rotated is 360, the flux linkage is
instantaneously zero.

SRJC 2013 9646/Prelim/2013


23

33 A full wave sinusoidal current of root mean square current I dissipates power in a resistor R
at a maximum instantaneous rate of P.

The same sinusoidal current now passes through an ideal diode such that the current is half-
wave rectified before passing through resistor R.

The new values for the root mean square current in the resistor R and the mean power
dissipated are

Root Mean Square current Mean power


I P
A
2 2

I P
B
2 4

I P
C
2 4

I P
D
2 2

Ans: B
From information of full wave a.c:
I0
I= where I 0 is peak current.
2
P
Pmean =
2
When a.c is half wave rectified:
I0 I I
Irms = = (as I = 0 )
2 2 2
P
Pmean =
4

SRJC 2013 9646/Prelim/2013 [Turn Over


24

34 When a steady direct current of value I passes through a heating element of resistance R,
the time taken to raise water of mass m by 10 K is time T.

When an alternating current with a peak current of value I passes through a heating
element of resistance 2R, the time taken to raise water of mass m by 10 K is

T T
A 2T B T C D
2 4

Ans: B
mc mc
P => t
t P
For steady d.c
mc
t as P = I2R
I 2R

For a.c with resistance 2R,


mc mc I 2
tac 2 as Pac ( ) 2R
I 2 I R 2
( ) 2R
2
tac =T

SRJC 2013 9646/Prelim/2013


25

35 In a photoelectric experiment, two different metal electrodes were illuminated initially by blue
radiation as shown in the circuit.
radiation source

metal metal
B A
A

e.m.f sorce

There was an initial reading in the ammeter. It was realised that when the radiation source
was changed to a green radiation with the same power rating, the reading in the ammeter
remains unchanged. This may be due to

A the electrons being emitted from metal A never reaching metal B.


B the work function of metal B being much higher than that of metal A.
C the increase in number of electrons ejected from metal A and a decrease in the
number of electrons ejected from metal B.
D the increase in number of electrons ejected from metal B and a decrease in the
number of electrons ejected from metal A.

Ans: B
Higher work function implies the KE of the electrons emitted is lower, which
may not have sufficient energy to reach the opposite electrode at the first
place.

36 Which of the following statements concerning semiconductors is correct?

One of the main features which explains the electrical properties of semiconductors is that

A there are a lot of electrons in the conduction band.

B it has a small energy band gap between the conduction and valence band.

C the conduction band is usually wide for electrons to move freely.

D it has a fully filled valence band.

Ans: B
Fact

SRJC 2013 9646/Prelim/2013 [Turn Over


26

37 In order to further improve the conductivity of an extrinsic p-type silicon, doping is necessary.

Which of the following statements is correct?

Conductivity can be further improved

A by doping the silicon with a trivalent atom.


B by doping the silicon with a tetravalent atom.
C by doping the silicon with a pentavalent atom.
D by doping the silicon with electrons.
Ans: A
A p-type semiconductor has majority of charge carriers which are holes.To produce more
holes, trivalent atoms should be used for doping.
38 In a collision between an excited Helium atom and a Neon atom, the Neon atom which is at
the ground state may be excited. The energy levels of Helium and Neon are given below.

20.61 eV 20.66 eV
18.70 eV

Common ground state


(0 eV)
Helium Neon
Which of the following statements is correct?

A Energy cannot be directly transferred by the collision process. Only photons are able to
cause excitation of the Neon atom.
B For the excited Neon atom, it is only possible to emit a photon of energy 18.70 eV.
C For the excited Neon atom, it is only possible to emit a photon of energy 18.70 eV or
20.66 eV.
D For the excited Neon atom, it is only possible to emit a photon of energy 1.96 eV, 18.70
eV or 20.66 eV.

Ans: D
Due to the kinetic energy of the atoms during the collision, it is possible for electrons in the
Neon atom to be excited to the 20.66 eV. When electrons de-excite from 20.66 eV, there are
3 possible paths of further de-excitation.

SRJC 2013 9646/Prelim/2013


27

39 The following nuclear equation shows one possible outcome when Uranium-235 is
bombarded by a neutron.

235
92 U+ 01n 97 M
40 Zr + 52Tc +bX where b is an integer.

What is the value of M and the particle X?

M X
A 135 Helium
B 137 neutron
C 139 electron
D 141 photon

Ans: B
235
original equation : 92 U + 01n 97
40 Zr + 137
Tc + 2 01n
52

40 A student placed 3 radioactive sources into an aluminium container of 10 mm thick. He uses


a Geiger-Muller tube to read the radiation outside the container. The Geiger-Muller counter
register counts from

A gamma radiation only.

B beta particles and gamma radiation only.

C alpha particles, beta particles and gamma radiation.

D none of the above are correct.

Ans: A
The penetrating properties of the different types of radiation are as follows.
Alpha particles A few cm of air.
Beta particles a few mm of Aluminium (about 4 mm)
Gamma radiation A few cm of lead
Alpha and Beta particles will be stopped by 10 mm of aluminium, only gamma radiation will
be detected.

End of Paper

SRJC 2013 9646/Prelim/2013 [Turn Over


SERANGOON JUNIOR COLLEGE
General Certificate of Education Advanced Level
Higher 2
NAME

CG INDEX NO.

PHYSICS 9646
Preliminary Examination 23rd August 2013
Paper 2 Structured Questions 1 hour 45 min
Candidates answer on the Question Paper.
No Additional Materials are required.

READ THIS INSTRUCTIONS FIRST


Write your name, civics group and index number in the spaces at the top of this page.

Write in dark blue or black pen on both sides of the paper. For Examiners Use
You may use a soft pencil for any diagrams, graphs or rough working.
Do not use staples, paper clips, highlighters, glue or correction fluid.
Q1 / 4
Answer all questions.
Q2 / 6
At the end of the examination, fasten all your work securely together.
The number of marks is given in bracket [ ] at the end of each question or Q3 / 6
part question.
. Q4 / 6

Q5 / 6

Q6 / 5

Q7 / 12

Q8 / 15

Q9 / 12
Total
/ 72
marks

This document consist of 19 printed pages and 1 blank page


2

DATA AND FORMULAE


Data
speed of light in free space, c = 3.00 x 108 m s1
permeability of free space, 0 = 4 x 10-7 H m-1
permittivity of free space, 0 = 8.85 x 10-12 F m-1
= (1/(36 )) x 10-9 F m-1
elementary charge, e = 1.60 x 1019 C
the Planck constant, h = 6.63 x 1034 J s
unified atomic mass constant, u = 1.66 x 1027 kg
rest mass of electron, me = 9.11 x 1031 kg
rest mass of proton, mp = 1.67 x 1027 kg
molar gas constant, R = 8.31 J K1 mol1
the Avogadro constant, NA = 6.02 x 1023 mol1
the Boltzmann constant, k = 1.38 x 10-23J K1
gravitational constant, G = 6.67 x 10-11N m2 Kg2

acceleration of free fall, g = 9.81 m s2

SRJC 2013 9646/PRELIM/2013


3

Formulae
uniformly accelerated motion, s = ut + at2

v2 = u2 + 2as

work done on/by a gas, W = pV


hydrostatic pressure, p = gh
GM
gravitational potential, = -
r
displacement of particle in s.h.m., x = x0 sin t
velocity of particle in s.h.m., v = v0 cos t

= x02 - x 2
resistors in series, R = R1 + R2 +
resistors in parallel, 1/R = 1/R1 + 1/R2 +
electric potential, V = Q/ 4 0r
alternating current/ voltage, x = x0 sin t
transmission coefficient, T exp(-2kd)

8 2 m(U - E )
where k =
h2
radioactive decay, x = x0 exp(-t)

decay constant, = 0.693


t1
2

SRJC 2013 9646/PRELIM/2013 [Turn Over


4 For
Examiners
Use

l
1 The formula for the period of a simple pendulum is T 2 . Such a pendulum is used to
g
determine g. The measured value of l is (98.2 0.1) cm and that of T is (1.99 0.01) s.
Determine the value of g and its associated uncertainty.

g g = ..... m s-2 [4]

SRJC 2013 9646/PRELIM/2013


5 For
Examiners
Use
2 (a) A stone is projected up an inclined plane at an angle of 20o with respect to the inclined
plane as shown in Fig. 2.1. The initial speed of projection is 40.0 m s-1.

40.0 m s-1
20
stone
30o

Fig. 2.1

(i) State any assumption made about acceleration of the stone when making use of
the kinematic equations.

..

.. [1]

(ii) Using h for the vertical displacement and x for the horizontal displacement, write
two equations relating the horizontal and the vertical displacements with time t.

h = ......

x = ........ [2]

(iii) Write an equation relating h and x when the stone hits the inclined plane.

.. [1]

(iv) Calculate the time taken for the stone to land on the inclined plane.

time = ..... s [2]

SRJC 2013 9646/PRELIM/2013 [Turn Over


6 For
Examiners
Use

3 Two carts of masses 400 g and 1200 g are free to move on a frictionless horizontal table.
Fig. 3.1 shows a spring of spring constant 500 N m-1 placed between the carts, and the carts
are tied together by a thread so that the compression of the spring is 3.0 cm. The carts are
initially at rest.

thread
spring

Fig. 3.1

(a) Calculate the elastic potential energy stored in the spring when the compression is
3.0 cm.

elastic potential energy = ..... J [2]

(b) The thread is then cut. Determine the final speeds of the two carts.

final speed of 400 g cart = ..... m s-1

final speed of 1200 g cart = ..... m s-1 [4]

SRJC 2013 9646/PRELIM/2013


7 For
Examiners
Use

4 (a) (i) State the principle of moments.

.....

.....

.....

....... [1]

(ii) A pair of pliers consists of 2 halves held together by a pivot. Fig 4.1 shows two
forces acting on one of the halves for a pair of pliers holding a sphere.

8N
30o

10 cm

4 cm

Fig. 4.1

Calculate the magnitude of the contact force of the sphere on the pliers, F.

F = ..... N [2]

SRJC 2013 9646/PRELIM/2013 [Turn Over


8 For
Examiners
Use

(b) (i) The Earth has an atmosphere which consist of a mixture of air. The average
density of air is 1.225 kg m-3 and the atmospheric pressure is 1.0 x 105 Pa at the
Earths surface.

Calculate the height of the atmosphere.

height = m [2]

(ii) Besides the estimates of the density and pressure of air, discuss why the answer
to part (b)(i) may not be accurate.

.....

... [1]

SRJC 2013 9646/PRELIM/2013


9 For
Examiners
Use

5 The voltage rating used by an appliance designed to work in Singapore is 240 V r.m.s while
the voltage used by an appliance designed to be work in United States of America (USA) is
120 V r.m.s.

In order to use an appliance bought from USA in Singapore, an ideal transformer is used.

(a) Explain the significance of the term ideal relating to the transformer.

[1]

(b) State and explain one feature of the design of a transformer to ensure that it performs
as ideally as possible.

[2]

(c) Calculate the turn ratio of the primary to the secondary coil of the transformer.

turn ratio = ....[1]

SRJC 2013 9646/PRELIM/2013 [Turn Over


10 For
Examiners
Use

(d) A varying input voltage is supplied to the ideal transformer. The graph of the variation
of the input voltage Ein of the ideal transformer with respect to time t is shown in
Fig. 5.1. The peak voltage of the Ein is 240 V while the r.m.s voltage is 138.6 V.

Ein / V
240

1 2 3 4 t/s
-240

Fig. 5.1

With reference to Fig. 5.1 sketch a corresponding graph in Fig. 5.2 to show how Vout
the output voltage to the appliance varies with t. With clear working, label the values of
Vout. [2]

Vout / V

1 2 3 4 t/s

Fig. 5.2

SRJC 2013 9646/PRELIM/2013


11 For
Examiners
Use
6 (a) Describe how it is possible for an electron to tunnel across a potential barrier.

.....

.....

.....

.....

.....

... [3]

(b) State two factors which will affect the probability of whether an electron can tunnel
through a potential barrier and explain how they affect the probability.

.....

.....

.....

.....

... [2]

SRJC 2013 9646/PRELIM/2013 [Turn Over


12 For
Examiners
Use

7 (a) Define binding energy.

.....

....[2]

226 222
(b) 88 Ra is a stationary radioactive isotope which decays to 86 Rn with the release of an
alpha particle. Given:
226
Mass of 88 Ra = 226.025 u
Mass of 24 He = 4.00260 u
222
Mass of 86 Rn = 222.018 u
Mass of proton = 1.00783 u
Mass of neutron = 1.00867 u

226
(i) Calculate the binding energy per nucleon of 88 Ra in MeV (to 4 significant figures).

binding energy per nucleon = ... MeV [2]

(ii) Calculate the energy released in the above decay reaction in MeV (to 4 significant
figures).

energy released = ... MeV [2]

SRJC 2013 9646/PRELIM/2013


13 For
Examiners
Use

(c) Nuclide X of decay constant = 7.85 x 10-10 s-1 is a beta emitter which decays to an
unstable nuclide Y. A sample initially contains N0 number of nuclei of X.

(i) It was found that a fresh sample of X emits 5.25 x 1013 particles in 10 s. Calculate
the time taken by a 50 year-old sample of X to emit the same number of particles.

time taken = ... s [3]

(ii) Without numerical values, sketch in Fig. 7.1, a graph to show how, starting with only
X nuclei, the activity A of X changes with time t. Label the graph X. [1]

0 t

Fig. 7.1

(iii) On the same axes in Fig. 7.1, sketch another graph to suggest how the activity of
the Y nuclei formed from the X nuclei changes with time. Label the graph Y. [2]

SRJC 2013 9646/PRELIM/2013 [Turn Over


14 For
Examiners
Use

8 The circuit in Fig. 8.1 is used for an experiment to determine Plancks constant.

B V
L
R1
5V
A
R2

Fig. 8.1

A Light-Emitting Diode (LED) that gives off red light of wavelength 695 nm is used for
component L. As contact point at resistor R1 shifts from A to B, the reading across the
voltmeter also varies.

Initially, the red LED light did not light up, it was only when the voltmeter reached a reading
of V0 did it light up. V0 was then recorded. L was then replaced with LEDs that gave off
infrared light of wavelength 1000 nm, yellow light of wavelength 660 nm and green light of
wavelength 630 nm.

The data collected is as shown in table in Fig. 8.2.

LED V0 / V / nm -1 / nm-1

Infrared 0.866 1000

Red 1.367 695

Yellow 1.464 660

Green 1.560 630

Fig. 8.2

(a) Fill in column on -1 in Fig. 8.2. [1]

SRJC 2013 9646/PRELIM/2013


15 For
Examiners
Use
(b) Plot a graph of V0 against -1 on the grids in Fig. 8.3. [3]

V0 / V

-1/ nm-1

Fig. 8.3

(c) State the energy which is required for an electron to move across the LED in terms of
elementary charge e and voltage across the LED V0.

[1]

(d) The energy of each photon corresponds to the energy stated in (c). Show how Plancks
constant can be determined from the graph in Fig. 8.3. [2]

SRJC 2013 9646/PRELIM/2013 [Turn Over


16 For
Examiners
Use

(e) Determine Plancks constant from the graph in Fig. 8.3.

Plancks constant = [3]

(f) Suggest two possible reasons for the difference between the calculated Plancks
constant from graph in Fig. 8.3 and the theoretical value for Plancks constant.

....

[2]

(g) When electrons and holes in a P-N Junction recombine, light will be emitted. With this
information, suggest how photons of specific wavelengths are formed in the LED when
the circuit is closed.

[2]

(h) Using the table in Fig. 8.2, determine the energy gap between the conduction and
valence band of the infrared LED.

energy gap = . J [1]

SRJC 2013 9646/PRELIM/2013


17 For
Examiners
Use

9 In manufacturing ships, it is important to determine the frequency which the ship will oscillate
up and down when it is in water. To minimise discomfort to the passengers it carries, the
frequency of the water waves should not match the natural frequency of the oscillation of the
ship.

It is proposed that the relationship between frequency of oscillations and mass m of the
ship may be calculated by

2 = Cmk

where C and k are constants.

You are provided with some 0.50 kg wooden blocks. You may also use any of the other
equipment usually found in a Physics laboratory.

Design an experiment to determine the values of C and k.

You should draw a labelled diagram to show the arrangement of your apparatus. In your
account you should pay particular attention to

(a) the identification and control of variables,


(b) the equipment you would use,
(c) the procedure to be followed,
(d) how the frequency of the wooden block may be determined,
(e) any precautions that would be taken to improve the accuracy and safety of the
experiment.

SRJC 2013 9646/PRELIM/2013 [Turn Over


18 For
Examiners
Use


SRJC 2013 9646/PRELIM/2013
19 For
Examiners
Use

[12]

End of paper
SRJC 2013 9646/PRELIM/2013 [Turn Over
SERANGOON JUNIOR COLLEGE
General Certificate of Education Advanced Level
Higher 2
NAME

CG INDEX NO.

PHYSICS 9646
Preliminary Examination 23th Aug 2013
Paper 2 Structured Questions (ANS) 1 hour 45 min

Candidates answer on the Question Paper.

No Additional Materials are required.

READ THIS INSTRUCTIONS FIRST


Write your name, civics group and index number in the spaces at the top of this page.

Write in dark blue or black pen on both sides of the paper. For Examiners Use
You may use a soft pencil for any diagrams, graphs or rough working.
Do not use staples, paper clips, highlighters, glue or correction fluid.
Q1 / 4
Answer all questions.
Q2 / 6
At the end of the examination, fasten all your work securely together.
The number of marks is given in bracket [ ] at the end of each question or Q3 / 6
part question.
. Q4 / 6

Q5 / 6

Q6 / 5

Q7 / 12

Q8 / 15

Q9 / 12
Total
/ 72
marks

This document consist of 21 printed pages and 1 blank page


2

DATA AND FORMULAE


Data
speed of light in free space, c = 3.00 x 108 m s1
permeability of free space, 0 = 4 x 10-7 H m-1
permittivity of free space, 0 = 8.85 x 10-12 F m-1
= (1/(36 )) x 10-9 F m-1
elementary charge, e = 1.60 x 1019 C
the Planck constant, h = 6.63 x 1034 J s
unified atomic mass constant, u = 1.66 x 1027 kg
rest mass of electron, me = 9.11 x 1031 kg
rest mass of proton, mp = 1.67 x 1027 kg
molar gas constant, R = 8.31 J K1 mol1
the Avogadro constant, NA = 6.02 x 1023 mol1
the Boltzmann constant, k = 1.38 x 10-23J K1
gravitational constant, G = 6.67 x 10-11N m2 Kg2

acceleration of free fall, g = 9.81 m s2

SRJC 2013 9646/PRELIM/2013


3

Formulae
uniformly accelerated motion, s = ut + at2

v2 = u2 + 2as

work done on/by a gas, W = pV


hydrostatic pressure, p = gh
GM
gravitational potential, = -
r
displacement of particle in s.h.m., x = x0 sin t
velocity of particle in s.h.m., v = v0 cos t

= x02 - x 2
resistors in series, R = R1 + R2 +
resistors in parallel, 1/R = 1/R1 + 1/R2 +
electric potential, V = Q/ 4 0r
alternating current/ voltage, x = x0 sin t
transmission coefficient, T exp(-2kd)

8 2 m(U - E )
where k =
h2
radioactive decay, x = x0 exp(-t)

decay constant, = 0.693


t1
2

SRJC 2013 9646/PRELIM/2013 [Turn Over


4 For
Examiners
Use

l
1 The formula for the period of a simple pendulum is T 2 . Such a pendulum is used to
g
determine g. The measured value of l is (98.2 0.1) cm and that of T is (1.99 0.01) s.
Determine the value of g and its associated uncertainty.

l
T 2
g
l
g 4 2
T2
0.982
4 2 9.7896
1.992

g l T
2
g l T
0.1 0.01
2
98.2 1.99
0.01107
g 0.01107(9.7896)
0.1 (1 s.f.)

g (9.8 0.1) m s -2

g g = ..... m s-2 [4]

SRJC 2013 9646/PRELIM/2013


5 For
Examiners
Use
2 (a) A stone is projected up an inclined plane at an angle of 20o with respect to the inclined
plane as shown in Fig. 2.1. The initial speed of projection is 40.0 m s-1.

40.0 m s-1
20
stone
30o

Fig. 2.1

(i) State any assumption made about acceleration of the stone when making use of
the kinematic equations.

The acceleration is constant.


..

. [1]

(ii) Using h for the vertical displacement and x for the horizontal displacement, write
two equations relating the horizontal and the vertical displacements with time t.

vertically: Using s = ut + at2


h = 40.0 sin50o t gt2 ---(1)

horizontally: Using s = ut + at2


x = 40.0 cos50o t ---(2)

h = ......

x = ........ [2]

(iii) Write an equation relating h and x when the stone hits the inclined plane.

tan 30o = h / x ---(3)


[1]

(iv) Calculate the time taken for the stone to land on the inclined plane.

Subst. (1) and (2) into (3):


tan 30o = (40.0 sin50o t g t2) / (40.0cos50o t)
t = 3.22 s

time = ..... s [2]

SRJC 2013 9646/PRELIM/2013 [Turn Over


6 For
Examiners
Use

3 Two carts of masses 400 g and 1200 g are free to move on a frictionless horizontal table. Fig.
3.1 shows a spring of spring constant 500 N m-1 placed between the carts and the carts are
tied together by a thread so that the compression of the spring is 3.0 cm. The carts are
initially at rest.

thread
spring

Fig. 3.1

(a) Calculate the elastic potential energy stored in the spring when the compression is
3.0 cm.

1
Strain energy = kx 2
2
1
(500)(0.03) 2
2
0.225 J

strain energy = ..... J [2]

SRJC 2013 9646/PRELIM/2013


7 For
Examiners
Use
(b) The thread is then cut. Determine the final speeds of the two carts.

By conservation of linear momentum,


Assuming both carts move in the same direction finally,
0 m1v1 m2 v2
0 0.4v1 1.2v2
0 v1 3v2
v1 3v2 (1)

By conservation of energy, strain energy is converted to KE


1 1
0.225 (0.4)v12 (1.2)v22
2 2
2 2
1.125 v1 3v2 (2)

Sub. (1) into (2),


1.125 (3v2 ) 2 3v22
1.125 9v22 3v22
v2 0.3062

v1 3(0.3062)
0.919 m s -1 (Speed = 0.919 m s -1 )

final speed of 400 g cart = ..... m s-1

final speed of 1200 g cart = ..... m s-1 [4]

SRJC 2013 9646/PRELIM/2013 [Turn Over


8 For
Examiners
Use

4 (a) (i) State the principle of moments.

..
The principle of moments states that for a system in equilibrium the sum of the clockwise
..
moments about any point must be equal to the sum of anticlockwise moments about that
same point.
..

.... [1]

(ii) A pair of pliers consists of 2 halves held together by a pivot. Fig 4.1 shows two
forces acting on one of the halves for a pair of pliers holding a sphere.

8N
30o

10 cm

4 cm

Fig. 4.1

Calculate the magnitude of the contact force of the sphere on the pliers, F.

Taking moments about the pivot,


ACW moments CW moments
F (4) (8cos 30o )(6)
F 10.4 N
F = ..... N [2]

SRJC 2013 9646/PRELIM/2013


9 For
Examiners
Use
(b) (i) The Earth has an atmosphere which consist of a mixture of air. The average
density of air is 1.225 kg m-3 and the atmospheric pressure is 1.0 x 105 Pa at the
Earths surface.

Calculate the height of the atmosphere.

p = hg
1.0 x 105 = h(1.225)(9.81)
h = 8321 m

height = m [2]

(ii) Besides the estimates of the density and pressure of air, discuss why the answer
to part (b)(i) may not be accurate.

....
- gravitational (field strength)/acceleration is also an estimate.

.[1]

SRJC 2013 9646/PRELIM/2013 [Turn Over


10 For
Examiners
Use

5 The voltage rating used by an appliance designed to work in Singapore is 240 V (r.m.s) while
the voltage used by an appliance designed to be work in United States of America (USA) is
120 V (r.m.s).

In order to use an appliance bought from USA in Singapore, an ideal transformer is used.

(a) Explain the significance of the term ideal relating to the transformer.

An ideal transformer implies that the power input into the primary is
.
equal to the power output at the secondary. (Or energy input in
primary = energy output in secondary.)
.

[1]

(b) State and explain one feature of the design of a transformer to ensure that it performs
as ideally as possible.

The laminated core reduces the amount of eddy currents induced in


.
the core, thus reducing energy lost as heat in the core.
.
OR
The soft iron core linking the primary to the secondary coil. This
.
ensures that the magnetic field created by the primary coil will link the
secondary coil.
[2]

(c) Calculate the turn ratio of the primary to the secondary coil of the transformer.

VP/VS = NP/NS
=240/120
=2 / 1 => Turn ratio = 2:1

turn ratio = ....[1]

SRJC 2013 9646/PRELIM/2013


11 For
Examiners
Use
(d) A varying input voltage is supplied to the ideal transformer. The graph of the variation
of the input voltage Ein of the ideal transformer with respect to time t is shown in
Fig. 5.1. The peak voltage of the Ein is 240 V while the r.m.s voltage is 138.5 V.

Ein / V
240

1 2 3 4 t/s
-240

Fig. 5.1

With reference to Fig. 5.1 sketch a corresponding graph in Fig. 5.2 to show how Vout
the output voltage to the appliance varies with t. With clear working, label the values of
Vout. [2]

Vrms secondary coil = 138.6 / 2 = 69.3 V

Vout / V
The reflection about t
69 axis is accepted.
.

1 2 3 4 t
-69.3

Fig. 5.2

SRJC 2013 9646/PRELIM/2013 [Turn Over


12 For
Examiners
Use

6 (a) Describe how it is possible for an electron to tunnel across a potential barrier.

The particle can be associated with a wave function


.....
and the probability of finding the particle at a given position is directly
proportional to the square of the modulus of its wave function at that
.....
position.
The wave function is smooth and continuous and therefore associated
.....
with the particle is non-zero in the regions within the barrier and
beyond the barrier,
.....
so there is a non-zero probability of the particle being found beyond
the barrier. This allows the particle to cross the barrier.
.....

... [3]

(b) State two factors which will affect the probability of whether an electron can tunnel
through a potential barrier and explain how they affect the probability.

.....
Barrier height: The higher the barrier height, the lower the probability of
barrier tunnelling of an electron.
.....
Barrier width: The wider the barrier width, the lower the probability of barrier
tunnelling of an electron.
... [2]

SRJC 2013 9646/PRELIM/2013


13 For
Examiners
Use

7 (a) Define binding energy.


.....
Binding energy is the amount of energy required to completely separate a
nucleus into its constituent nucleons such that they are infinitely far apart.
....[2]

226 222
(b) 88 Ra is a stationary radioactive isotope which decays to 86 Rn with the release of an
alpha particle. Given:
226
Mass of 88 Ra = 226.025 u
Mass of 24 He = 4.00260 u
222
Mass of 86 Rn = 222.018 u
Mass of proton = 1.00783 u
Mass of neutron = 1.00867 u

226
(i) Calculate the binding energy per nucleon of 88 Ra in MeV (to 4 significant figures).

Mass defect m 88m p (226 88) mn M Ra


88 1.00783u 138 1.00867u 226.025u
1.8605u

mc 2
Binding energy per nucleon =
226
27 8 2
1.8605(1.66 10 )(3.0 10 )

226
12
1.2299 10 J
7.687 MeV

binding energy per nucleon = ... MeV [2]

(ii) Calculate the energy released in the above decay reaction in MeV (to 4 significant figures).

226 222
88 Ra 86Rn 24He
Energy released
mreac tan ts c 2 mproducts c 2
226.025 222.018 4.00260 (1.66 10 27 )(3 10 8 )2
6.5736 10 13 J
4.109MeV

energy released = ... MeV [2]

SRJC 2013 9646/PRELIM/2013 [Turn Over


14 For
Examiners
Use

(c) Nuclide X of decay constant = 7.85 x 10-10 s-1 is a beta emitter which decays to an
unstable nuclide Y. A sample initially contains N0 number of nuclei of X.
(i) It was found that a fresh sample of X emits 5.25 x 1013 particles in 10 s. Calculate
the time taken by a 50 year-old sample of X to emit the same number of particles.

5.25 1013
A0 5.25 1012 Bq
10
A 5.25 1012 e

7.851010 50365246060

1.52 1012 Bq (rate of decay or particles emitted, for 50 yr-old sample)


To emit 5.25 1013 particles, the time taken,
5.25 1013
t 34.5s
1.52 1012

time taken = ... s [3]

(ii) Without numerical values, sketch in Fig. 7.1, a graph to show how, starting with only
X nuclei, the activity A of X changes with time t. Label the graph X. [1]

A
A

Ao

0 t

Fig. 7.1

(iii) On the same axes in Fig. 7.1, sketch another graph to suggest how the activity of
the Y nuclei formed from the X nuclei changes with time. Label the graph Y. [2]

SRJC 2013 9646/PRELIM/2013


15 For
Examiners
Use
8 The circuit in Fig. 8.1 is used for an experiment to determine Plancks constant.

B V
L
R1
5V
A
R2

Fig. 8.1

A Light-Emitting Diode (LED) that gives off red light of wavelength 695 nm is used for
component L. As contact point at resistor R1 shifts from A to B, the reading across the
voltmeter also varies.

Initially, the red LED light did not light up, it was only when the voltmeter reached a reading
of V0 did it light up. V0 was then recorded. L was then replaced with LEDs that gave off
infrared light of wavelength 1000 nm, yellow light of wavelength 660 nm and green light of
wavelength 630 nm.

The data collected is as shown in table in Fig. 8.2.

LED V0 / V / nm -1 / nm-1

Infrared 0.866 1000 0.001000

Red 1.367 695 0.00144

Yellow 1.464 660 0.00152

Green 1.560 630 0.00159

Fig. 8.2
-1
(a) Fill in column on in Fig. 8.2. [1]

SRJC 2013 9646/PRELIM/2013 [Turn Over


16 For
Examiners
Use

(b) Plot a graph of V0 against -1 on the grids in Fig. 8.3. [3]

V0 / V

1.500

1.400
1.560 1.080 = 0.480

1.300

1.200

1.100

0.0015900 0.0011800 = 0.0004100

1.000

0.900

0.800
0.0010 0.0011 0.0012 0.0013 0.0014 0.0015 0.0016 -1/ nm-1

Fig. 8.3

(c) State the energy which is required for an electron to move across the LED in terms of
elementary charge e and voltage across the LED V0.
eV0
.[1]

(d) The energy of each photon corresponds to the energy stated in (c). Show how the
Plancks constant can be determined from the graph in Fig. 8.3. [2]

hc
E

hc
eV0

hc 1
V0 ( )
e
hc
Hence, is the gradient of the graph.
e

SRJC 2013 9646/PRELIM/2013


17 For
Examiners
Use

(e) Determine the Plancks constant from the graph in Fig. 8.3.

hc 0.480

e 0.0004100 109
0.480 1.6 1019
h
0.0004100 109 3 108
6.24 1034

Plancks constant = [3]

(f) Suggest two possible reasons for the difference between the calculated Plancks
constant from graph in Fig. 8.3 and the theoretical value for Plancks constant.


The voltmeter may not be ideal hence V0 is not accurate.

It is difficult to determine exactly when the LED actually lit up
hence it is difficult to determine V0 exactly.
.

[2]

(g) When electrons and holes in a P-N Junction recombine, light will be emitted. With this
information, suggest how photons of specific wavelengths are formed in the LED when
the circuit is closed.


For a recombination to occur, the electrons de-excite from the conduction
band to the valence band,

thus emitting a photon corresponding to the energy gap.
Thus the wavelength of the photons is of specific values.

[2]

(h) Using the table in Fig. 8.2, determine the energy gap between the conduction and
valence band of the infrared LED.

Band gap = eV0


= 0.866 x 1.6 x 10-19
= 1.39 x 10-19 J

Energy gap = . J [1]

SRJC 2013 9646/PRELIM/2013 [Turn Over


18 For
Examiners
Use

9 In manufacturing ships, it is important to determine the frequency which the ship will oscillate
up and down when it is in water. To minimise discomfort to the passengers it carries, the
frequency of the water waves should not match the natural frequency of the oscillation of the
ship.

It is proposed that the relationship between frequency of oscillations and mass m of the
ship may be calculated by

2 = Cmk

where C and k are constants.

You are provided with some 0.50 kg wooden blocks. You may also use any of the other
equipment usually found in a Physics laboratory.

Design an experiment to determine the values of C and k.

You should draw a labelled diagram to show the arrangement of your apparatus. In your
account you should pay particular attention to

(a) the identification and control of variables,


(b) the equipment you would use,
(c) the procedure to be followed,
(d) how the frequency of the wooden block may be determined,
(e) any precautions that would be taken to improve the accuracy and safety of the
experiment.

SRJC 2013 9646/PRELIM/2013


19 For
Examiners
Use


SRJC 2013 9646/PRELIM/2013 [Turn Over
20 For
Examiners
Use

[12]

End of paper
SRJC 2013 9646/PRELIM/2013
SERANGOON JUNIOR COLLEGE
General Certificate of Education Advanced Level
Higher 2
NAME

CG INDEX NO.

PHYSICS 9646
Preliminary Examination 26th August 2013
Paper 3 Longer Structured Questions 2 hrs

Candidates answer on the Question Paper.

No Additional Materials are required.

READ THIS INSTRUCTIONS FIRST


Write your name, civics group and index number in the spaces at the top of this page.

Write in dark blue or black pen on both sides of the paper. For Examiners Use
You may use a soft pencil for any diagrams, graphs or rough working.
Do not use staples, paper clips, highlighters, glue or correction fluid.
Q1 / 6
Section A
Answer all questions. Q2 / 8

Section B Q3 / 10
Answer any two questions.
Q4 / 8
You are advised to spend about an hour on each section.
Q5 / 8
At the end of the examination, fasten all your work securely together.
The number of marks is given in bracket [ ] at the end of each question or Q6 / 20
part question.
. Q7 / 20
.
Q8 / 20
Total
/ 80
marks

This document consist of 22 printed pages and 2 blank pages


2

DATA AND FORMULAE


Data
speed of light in free space, c = 3.00 x 108 m s1
permeability of free space, 0 = 4 x 10-7 H m-1
permittivity of free space, 0 = 8.85 x 10-12 F m-1
= (1/(36 )) x 10-9 F m-1
elementary charge, e = 1.60 x 1019 C
the Planck constant, h = 6.63 x 1034 J s
unified atomic mass constant, u = 1.66 x 1027 kg
rest mass of electron, me = 9.11 x 1031 kg
rest mass of proton, mp = 1.67 x 1027 kg
molar gas constant, R = 8.31 J K1 mol1
the Avogadro constant, NA = 6.02 x 1023 mol1
the Boltzmann constant, k = 1.38 x 10-23J K1
gravitational constant, G = 6.67 x 10-11N m2 Kg2

acceleration of free fall, g = 9.81 m s2

SRJC 2013 9646/PRELIM/2013


3

Formulae
uniformly accelerated motion, s = ut + at2

v2 = u2 + 2as

work done on/by a gas, W = pV


hydrostatic pressure, p = gh
GM
gravitational potential, = -
r
displacement of particle in s.h.m., x = x0 sin t
velocity of particle in s.h.m., v = v0 cos t

= x02 - x 2
resistors in series, R = R1 + R2 +
resistors in parallel, 1/R = 1/R1 + 1/R2 +
electric potential, V = Q/ 4 0r
alternating current/ voltage, x = x0 sin t
transmission coefficient, T exp(-2kd)

8 2 m(U - E )
where k =
h2
radioactive decay, x = x0 exp(-t)

decay constant, = 0.693


t1
2

SRJC 2013 9646/PRELIM/2013 [Turn Over


4 For
Examiners
Use

Section A
Answer all the questions in this section.

1 A satellite is set to orbit around the Earth. The mass of the Earth is ME, radius of orbit of
satellite is r and period of its orbit around Earth is T.

r3
(a) Show that T = 2 . [2]
GME

(b) Determine the radius of the orbit of a satellite around Earth when it has a period of 2
days. It is known that ME = 5.97 x 1024 kg.

radius = . km [2]

(c) In the calculation in part (b), the effect of the gravitational force of the moon has not
been taken into account. Discuss any effect it has on the period of the orbit of the
satellite if the gravitational force of the moon has been taken into consideration. The
distance from the centre of Earth to the centre of moon is 384 000 km.

... [2]

SRJC 2013 9646/PRELIM/2013


5 For
Examiners
Use
2 In a proposal of a car lifting system, the following model was designed. An ideal gas at a
pressure of 250 kPa and temperature of 380 C enclosed by a gas-tight, frictionless piston.
The 100 kg platform has a circular cross-sectional area of 0.0300 m2 and is prevented from
moving further up by the two stoppers. The model car has a mass of 50 kg. The atmosphere
is at 100 kPa and 30.0 C. The gas cools as heat is transferred to the atmosphere until it is in
thermal equilibrium with the atmosphere.

Model car
Moveable platform
(piston)
stopper stopper

Ideal gas 25.0 cm

Fig. 2.1

(a) (i) Show that the amount of ideal gas in the cylinder is 0.345 mol. [2]

(ii) Show that the piston begins to move down at 390 K. [2]

SRJC 2013 9646/PRELIM/2013 [Turn Over


6 For
Examiners
Use

(b) For each of the following statements below, discuss whether it is correct, stating clearly
your reasons.

(i) The latent heat of fusion of a substance is always more than the specific latent
heat of vaporisation of the substance.

... [2]

(ii) For the same mass, volume and temperature, the pressure exerted by a real gas
is the same as the pressure exerted by an ideal gas.

... [2]

SRJC 2013 9646/PRELIM/2013


7 For
Examiners
Use
3 The current-potential difference relationship for two electrical components P and Q is shown
in Fig. 3.1.

current / mA

P
8
Q
7
6
5
4
3
2
1
0
2 4 6 8 10 potential difference / V
Fig. 3.1

(a) Identify

(i) component P.

...... [1]

(ii) component Q.

...... [1]

(b) P and Q are connected in parallel. The current flowing through P is 6 mA.

(i) State the potential difference across Q.

potential difference across Q = V [1]

(ii) State the current flowing in Q.

current in Q = mA [1]

(iii) Determine the effective resistance if P and Q are considered as a single


component.

effective resistance = [3]

SRJC 2013 9646/PRELIM/2013 [Turn Over


8 For
Examiners
Use

(c) P and Q are now connected in series with a battery of internal resistance of 2 . The
current flowing through P is 3 mA.

(i) Calculate the terminal potential difference across the battery.

terminal potential difference across the battery = .V [2]

(ii) Calculate the rate of heat dissipated in the battery.

rate of heat dissipated by the battery = .W [1]

SRJC 2013 9646/PRELIM/2013


9 For
Examiners
Use
4 Most street lights in Singapore use Sodium lamp. They give off light of distinct orange-
yellowish colour.

(a) Explain why the lamp gives off a distinct orange-yellowish light.

... [2]

(b) Two different setups were used to analyse the light source from the Sodium lamp.
Fig. 4.1 shows the two visible light spectrums P and Q.

Fig. 4.1

(i) Identify the type of spectrum P and Q.

P: ...

Q: [1]

SRJC 2013 9646/PRELIM/2013 [Turn Over


10 For
Examiners
Use

(ii) With the aid of a diagram, describe how spectrum P is produced.

[4]

(c) Calculate the energy of the photon emitted by the Sodium atom if the wavelength of the
light emitted is 569 nm.

energy of photon = . J [1]

SRJC 2013 9646/PRELIM/2013


11 For
Examiners
Use
5 (a) With reference to the way lasers are produced, explain why laser light is highly intense
and monochromatic.

... [4]

(b) In an n-type semiconductor, there are excess electrons forming the sea of electrons,
thus making the n-type semiconductor negatively charged.

(i) State how an n-type semiconductor is doped.

[1]

(ii) State and explain why the above statement is inappropriate.

.[3]

SRJC 2013 9646/PRELIM/2013 [Turn Over


12 For
Examiners
Use

Section B

Answer two questions from this section.


6 (a) Distinguish between stationary and progressive waves.

.... [3]

(b) Describe, with a labelled diagram, an experiment to determine the speed of sound using
stationary waves.

....... [5]
SRJC 2013 9646/PRELIM/2013
13 For
Examiners
Use
(c) Fig. 6.1 shows two coherent electromagnetic wave emitters, M1 and M2 which emit
waves in phase. A detector is placed at X. As the frequency of the emitters gradually
increases, the detected signal shows a series of maxima and minima.

X
M1 X
detector
9 cm

M2
40 cm
B
Fig. 6.1
(i) Explain how the maxima and minima can be observed at the same point X.

.... [3]

(ii) Calculate the frequency at which the first minimum intensity is observed at X.

frequency = Hz [3]

(iii) State if a maximum or minimum intensity will be detected at X when the frequency
is doubled.

..... [1]

SRJC 2013 9646/PRELIM/2013 [Turn Over


14 For
Examiners
Use

(iv) The detector moves along line AB and detects multiple maxima.

1. State the effect on the positions of the maxima when M1 and M2 are emitting
waves that are in anti-phase.

[1]

2. State and explain the effect on the positions of the maxima when M1 is
gradually moved towards M2.

[2]

(v) When more emitters which are in phase with M1 are placed equidistant and along
the line joining M1 and M2, State and explain the effect on the number of maxima
which can be detected when the detector moves along line AB.

[2]

SRJC 2013 9646/PRELIM/2013


15 For
Examiners
Use
7 (a) An electron is placed near two positive charges as shown in Fig. 7.1
+1.6 x 10-19 C
2 mm
3 mm
A B C
Electron x x
u = 0 m s-1
4 mm 3 mm

+1.6 x 10-19 C
Fig. 7.1
(i) Determine the magnitude of the resultant electric force acting on the electron at
Point A.

resultant electric force = N [3]

(ii) Determine the resultant electric force acting at the electron at Point B.

resultant electric force = N [2]

(iii) Hence state and explain whether the motion is simple harmonic.

...

...

...

...

. [3]

SRJC 2013 9646/PRELIM/2013 [Turn Over


16 For
Examiners
Use

(iv) Determine the acceleration of the electron at Point B.

acceleration = .. m s-2 [2]

(v) Determine the speed of the electron at Point C when it moves from Point A to C
due to the resultant electric force acting on it.

speed = .. m s-1 [3]

SRJC 2013 9646/PRELIM/2013


17 For
Examiners
Use
(b) Fig. 7.2 shows a spring-mass system placed on a frictionless slope which has an angle of
from the horizontal. When a block of m was hung, the spring stretched by an extension of e
and the mass remained in equilibrium. The spring was then further extended by x
downwards and released for the mass spring system to oscillate. The spring constant is k.

Fixed
wall

Equilibrium
position
Frictionless
slope

Fig. 7.2

(i) When the block is at rest at the equilibrium position the tension in the spring is T1.
Write the expression for T1 in terms . [1]

(ii) Write the expression for the acceleration a of the block at the lowest point using the
least number of symbols defined in the question. [2]

SRJC 2013 9646/PRELIM/2013 [Turn Over


18 For
Examiners
Use

(iii) Hence, sketch the graph showing the variation of acceleration a with respect to , at
the lowest point, in Fig. 7.3 when the extension of the spring from the equilibrium
position is kept at x cm. [1]


90
Fig. 7.3

(iv) Determine the position from equilibrium when speed of mass is of the maximum
speed during oscillation, in terms of the amplitude x0.

position from equilibrium = . x0 [3]

SRJC 2013 9646/PRELIM/2013


19 For
Examiners
Use
8 (a) Define the terms magnetic flux and the weber.

..

....................................................................................................................................

..................................................................................................................................................

............................................................................................................................................. [2]

(b) Fig.8.1 shows a rectangular conducting loop of resistance 1.2 , width 5 cm, and length
10 cm being pulled horizontally at constant speed v through a region of uniform
magnetic field strength B which has a width of 16 cm. The magnetic field strength is
perpendicular to the plane of the loop and going out of the plane of the paper.

The direction of the motion of the loop is along the plane of the paper and perpendicular
to one side of the loop as shown

At time t = 0 s, the rectangular loop is in the position shown in Fig. 8.1.


16 cm

10 cm
B (out the page)
5 cm v

Fig. 8.1

Fig. 8.2 shows how the induced current I flowing in the loop varies with t.
I / mA

3.0

5.0 8.0 13.0 t/s

-3.0

Fig. 8.2

(i) Deduce the speed v of the conducting loop.

v = .. cm s-1 [1]

SRJC 2013 9646/PRELIM/2013 [Turn Over


20 For
Examiners
Use

(ii) Fig.8.2 shows a period when I is positive and another when the current is negative.
State the direction of negative I with reference to Fig. 8.1.

Direction of negative I is . [1]

(iii) With clear working, sketch and label in Fig. 8.3 the variation of induced e.m.f. E with
time t. [2]

E/V

t/s
5.0 8.0 13.0

Fig. 8.3

(iv) Hence determine the magnitude of the magnetic field strength B.

B = T [2]

SRJC 2013 9646/PRELIM/2013


21 For
Examiners
Use
(v) With clear working, sketch with labels in Fig. 8.4 the variation of magnetic flux
through the loop with time t. [2]

/ Wb

5.0 8.0 13.0


t/s

Fig. 8.4

(vi) In order to move the loop with the constant speed an external horizontal pulling force F
must be acting on the loop. With clear working, sketch in Fig. 8.5 the variation of F with
time t. Take the direction of motion of the loop to be positive. (All forces apart from
magnetic forces are negligible.) [2]

F /N

5.0 8.0 13.0


t/s

Fig. 8.5

SRJC 2013 9646/PRELIM/2013 [Turn Over


22 For
Examiners
Use

(vii) Describe the processes of conversion of energy if any from t = 5.0 s to t = 13.0 s with
clear calculated values.

.............

.............

.............

.............

. [4]

(viii) The direction of the uniform magnetic field B is now changed such that it is directed
into the plane of the page while all other conditions remain the same.

State which graph(s) (Fig. 8.2 to Fig. 8.5) would change and which graph(s) would
remain unchanged.

.............

.............

.............

..... [2]

(ix) The loop is now replaced with a flat metal conducting plate of the same dimensions and
made to move through the uniform magnetic field while all other conditions remain the
same.

Describe and explain the key difference for the external force F required in this scenario.

.............

.............

.............

. [2]

End of paper
SRJC 2013 9646/PRELIM/2013
SERANGOON JUNIOR COLLEGE
General Certificate of Education Advanced Level
Higher 2
NAME

CG INDEX NO.

PHYSICS 9646
Preliminary Examination 26th Aug 2013
Paper 3 Longer Structured Questions (ANS) 2 hrs

Candidates answer on the Question Paper.

No Additional Materials are required.

READ THIS INSTRUCTIONS FIRST


Write your name, civics group and index number in the spaces at the top of this page.

Write in dark blue or black pen on both sides of the paper. For Examiners Use
You may use a soft pencil for any diagrams, graphs or rough working.
Do not use staples, paper clips, highlighters, glue or correction fluid.
Q1 / 6
Section A
Answer all questions. Q2 / 8

Section B Q3 / 10
Answer any two questions.
Q4 / 8
You are advised to spend about an hour on each section.
Q5 / 8
At the end of the examination, fasten all your work securely together.
The number of marks is given in bracket [ ] at the end of each question or Q6 / 20
part question.
. Q7 / 20
.
Q8 / 20
Total
/ 80
marks

This document consist of 23 printed pages and 0 blank page


2

DATA AND FORMULAE


Data
speed of light in free space, c = 3.00 x 108 m s1
permeability of free space, 0 = 4 x 10-7 H m-1
permittivity of free space, 0 = 8.85 x 10-12 F m-1
= (1/(36 )) x 10-9 F m-1
elementary charge, e = 1.60 x 1019 C
the Planck constant, h = 6.63 x 1034 J s
unified atomic mass constant, u = 1.66 x 1027 kg
rest mass of electron, me = 9.11 x 1031 kg
rest mass of proton, mp = 1.67 x 1027 kg
molar gas constant, R = 8.31 J K1 mol1
the Avogadro constant, NA = 6.02 x 1023 mol1
the Boltzmann constant, k = 1.38 x 10-23J K1
gravitational constant, G = 6.67 x 10-11N m2 Kg2

acceleration of free fall, g = 9.81 m s2

SRJC 2013 9646/PRELIM/2013


3

Formulae
uniformly accelerated motion, s = ut + at2

v2 = u2 + 2as

work done on/by a gas, W = pV


hydrostatic pressure, p = gh
GM
gravitational potential, = -
r
displacement of particle in s.h.m., x = x0 sin t
velocity of particle in s.h.m., v = v0 cos t

= x02 - x 2
resistors in series, R = R1 + R2 +
resistors in parallel, 1/R = 1/R1 + 1/R2 +
electric potential, V = Q/ 4 0r
alternating current/ voltage, x = x0 sin t
transmission coefficient, T exp(-2kd)

8 2 m(U - E )
where k =
h2
radioactive decay, x = x0 exp(-t)

decay constant, = 0.693


t1
2

SRJC 2013 9646/PRELIM/2013 [Turn Over


4 For
Examiners
Use

Section A
Answer all the questions in this section.

1 A satellite is set to orbit around the Earth. The mass of the Earth is ME, radius of orbit of
satellite is r and period of its orbit around Earth is T.

r3
(a) Show that T = 2 . [2]
GME

Gravitational force provides centripetal force,


GM E m 2
mr ( ) 2
r T
r3
T 2
GM E

(b) Determine the radius of the orbit of a satellite around Earth when it has a period of 2
days. It is known that ME = 5.97 x 1024 kg.

r3
T 2
GM E
r3
2(24)(3600) 2
G (5.97 1024 )
r 67031 km

radius = . km [2]

(c) In the calculation in part (b), the effect of the gravitational force of the moon has not
been taken into account. Discuss any effect it has on the period of the orbit of the
satellite if the gravitational force of the moon has been taken into consideration. The
distance from the centre of Earth to the centre of moon is 384 000 km.

.
As the gravitational force of the moon on the satellite is negligible
.
compared to that of the Earth due to significant difference between
the distance from Moon to satellite and Earth to satellite[, the effect
on the period is also negligible.
.

... [2]

SRJC 2013 9646/PRELIM/2013


5 For
Examiners
Use
2 In a proposal of a car lifting system, the following model was designed. An ideal gas at a
pressure of 250 kPa and temperature of 380 oC enclosed by a gas-tight, frictionless piston.
The 100 kg platform has a circular cross-sectional area of 0.0300 m2 and is prevented from
moving further up by the two stoppers. The model car has a mass of 50 kg. The atmosphere
is at 100 kPa and 30.0 C. The gas cools as heat is transferred to the atmosphere until it is in
thermal equilibrium with the atmosphere.

Model car
Moveable platform
(piston)
stopper stopper

Ideal gas 25.0 cm

Fig. 2.1

(a) (i) Show that the number of moles of ideal gas in the cylinder is 0.345 mol.

PV = nRT
(250 x 103)(0.25 x 0.030) = n x 8.31 x (380 + 273.15)
n = 0.345 mol

(ii) Show that the piston begins to move down at 390 K.

External pressure due to piston, car and atmosphere


150 9.81
= 100 103
0.0300
= 149050 Pa

Piston begins to move down when external pressure = pressure of gas

PV = nRT,
149050 x (0.25 x 0.030) = 0.345 x 8.31 x T
T = 390 K (shown)

SRJC 2013 9646/PRELIM/2013 [Turn Over


6 For
Examiners
Use

(b) For each of the following statements below, discuss whether it is correct, stating clearly
your reasons.

(i) The specific latent heat of fusion of a substance is always more than the specific
latent heat of vaporisation of the substance.
.
.
For the same substance,
In the vaporisation process, the molecules need to overcome the intermolecular
.
attraction to separate and become independent molecules, while at the same time
doing work in expansion against the atmosphere.
... [2]
In the melting process, the molecules need only break down and escape from the
crystalline structure to have the higher degree of freedom and disorder that
characterises a liquid state. The change in volume is negligible and hence, work
done against the atmosphere is not significant.

Hence, the latent heat of vaporisation (lv) is higher than that of fusion (lf) since no
heat is expended as work done against the surroundings in the case of melting.

(ii) For the same mass, volume and temperature, the pressure exerted by a real gas
is the same as the pressure exerted by an ideal gas.

.
False.
Pressure of a real gas is less than that of an ideal gas for the same mass, volume and
.
temperature. This is because attractive forces exist between the atoms/molecules in a
real gas and not in an ideal gas.
... [2]

SRJC 2013 9646/PRELIM/2013


7 For
Examiners
Use
3 The current-potential difference relationship for two electrical components P and Q is shown
in Fig. 3.1.

current / mA

P
8
Q
7
6
5
4
3
2
1
0
2 4 6 8 10 potential difference / V
Fig. 6.1
(a) Identify

(i) component P.

Fixed Resistor / ohmic conductor / metal at constant temperature


...... [1]

(ii) component Q.
Thermistor
...... [1]

(b) P and Q are connected in parallel. The current flowing through P is 6 mA.

(i) State the potential difference across Q.

6V

potential difference across Q = V [1]

(ii) State the current flowing in Q.

3 mA
current in Q = mA [1]

(iii) Determine the effective resistance if P and Q are considered as a single


component.
Resistance of P = 6 / 6 x 10-3 = 1000
Resistance of Q = 6 / 3 x 10-3 = 2000
Effective resistance
1000 2000
667
1000 2000
OR
current flowing through the component = 6 + 3 = 9 mA
effective resistance = [3]
Effective resistance = 6 / 9 x 10-3
= 667
SRJC 2013 9646/PRELIM/2013 [Turn Over
8 For
Examiners
Use

(c) P and Q are now connected in series with a battery of internal resistance of 2 . The
current flowing through P is 3 mA.

(i) Calculate the terminal potential difference across the battery.


Terminal p.d. = 3 + 6
=9V

terminal potential difference across the battery = V [2]

(ii) Calculate the rate of heat dissipated in the battery.

rate of heat dissipated by the battery


= I2R
= (3 x 10-3)2 x 2 = 1.80 x 10-5 W

rate of heat dissipated by the battery = W [1]

SRJC 2013 9646/PRELIM/2013


9 For
Examiners
Use
4 Most street lights in Singapore use Sodium lamp. They give off light of distinct orange-
yellowish colour.

(a) Explain why the lamp gives off a distinct orange-yellowish light.
Light is being emitted when energised electrons falls from a higher orbital
.
shell to a lower one. The energy gaps of each element is distinct , for
Sodium, the orange-yellowish light is detected by our eye.
.
(Note: Other radiation are emitted as well, but may not be detected by eye.)

... [2]

(b) Two different setups were used to analyse the light source from the Sodium lamp. Fig.
4.1 shows the two visible light spectrums P and Q.

Fig. 4.1

(i) Identify the type of spectrum P and Q.

.
P is a emission spectrum of Sodium
Q is absorption spectrum of Sodium
[1]

(ii) With the aid of a diagram, describe how spectrum P is produced.

In a discharge tube, an electric field is used to excite the electrons in


the Sodium atom.
.
When the atoms de-excite, light(photons) is given off [1].
Sodium vapour gives off characteristic orange-yellowish light which is
.
shown in P.
Light passes through the diffraction grating to produce the emission
.
spectrum.

[4]

SRJC 2013 9646/PRELIM/2013 [Turn Over


10 For
Examiners
Use

(c) Calculate the energy of the photon emitted by the Sodium atom if the wavelength of the
light emitted is 569 nm.

E = hc /
= 6.63 x 10-34 x 3 x 108 / 569 x 10-9
= 3.50 x 10-19 J

energy of photon = . J [1]

5 (a) With reference to the way lasers are produced, explain why laser light is highly intense
and monochromatic.

.
Population inversion can be achieved when an atom has a metastable
state, which makes it possible for the Laser to be intense. When most of
the excited electrons in the metastable level de-excite at the same time,
.
an intense beam can be produce, this is couple by the partial mirror which
helps the beam to be built up.
.
It is monochromatic as Laser effect makes use of stimulated emission.
When a photon of energy matching the energy gap of an excited electron
.
pass by the electron, it will de-excite thus emitted a photon of the same
energy as the incoming photon.
... [4]

(b) In an n-type semiconductor, there are excess electrons forming the sea of electrons,
thus making the n-type semiconductor negatively charged.

(i) State how an n-type semiconductor is doped.

A neutral pentavalent atom (group V) with 5 valance electrons are doped


.
with an intrinsic semiconductor which is also neutral .
[1]

(ii) State and explain why the above statement is inappropriate.

.
The semiconductor is neutral. There are no excess electrons. There are no
de-localised electrons to form the seas of electrons like that formed in
.
metals.

.[3]

SRJC 2013 9646/PRELIM/2013


11 For
Examiners
Use
Section B
Answer two questions from this section.
6 (a) Distinguish between stationary and progressive waves.

Stationary Wave Progressive Wave 1 mark for


Does not advance. Advances in the each
Wave profile direction of energy comparison
transfer of the wave. [1]

The energy is stored Energy is transferred in


Energy of as the kinetic and the direction of wave
wave potential energy of the propagation.
vibrating particles.
Depends on position Same for all particles in
along the wave the wave regardless of
Amplitude of
oscillation of Particles at the position (assuming no
individual antinodes oscillate with energy loss).
particles maximum amplitude
Particles at the nodes
do not oscillate
Twice the distance Distance between any 2
between 2 adjacent consecutive points on
Wavelength nodes/ antinodes. the wave with the same
Equal to the wavelength phase.
of the component
waves.
All particles in an Wave particles have
Phase of
internodal loop vibrate different phases (0 to
wave
particles in a in phase. Particles in 2) within a wavelength.
wavelength neighboring loops are in
anti-phase.

.... [3]

SRJC 2013 9646/PRELIM/2013 [Turn Over


12 For
Examiners
Use

(b) Describe, with a labelled diagram, an experiment to determine the speed of sound using
stationary waves.

Method 1

Source (e.g. loudspeaker


connected to signal
generator)

reflector
Detector (e.g.
signal microphone)
generator

CRO

Labelled diagram with a signal generator, loudspeaker, CRO, microphone, reflector [1


.
mark]
.
A wave of known frequency is generated by a source, e.g. a loudspeaker connected
to a signal generator.
.

Shift the microphone until it registers the highest / lowest signal


.
The wavelength of the wave, = 2d, where d is the distance between two adjacent
.
nodes (or two adjacent antinodes).

.
Using v= f to calculate the speed of sound

.......
Method 2 [5]
Tube Tuning fork

water

Labelled diagram with a tube with 2 open ends, tuning fork and a container of water

Use a tuning fork with known frequency.

Shift the tube upwards until the first loud sound is heard

The wavelength of the wave, = 4L where where L is the length of the tube above
water surface.

Using v= f to calculate the speed of sound

SRJC 2013 9646/PRELIM/2013


13 For
Examiners
Method 3 Use

loudspeaker
Tube / pipe

Signal generator

Labelled diagram with a signal generator, loudspeaker, tube / pipe

Slowly increase the frequency of signal generator until the first loud sound is heard.

Record of frequency of the sound from signal generator.

= 2L where L is the length of the tube / pipe.

Using v= f to calculate the speed of sound

If the tube / pipe is closed in one end, then = 4L

(c) Fig. 6.1 shows two electromagnetic wave emitters, M1 and M2 which emit waves in
phase. A detector is placed at X. As the frequency of the emitters gradually increases,
the detected signal shows a series of maxima and minima.

X
M1 X
detector
9 cm

M2
40 cm
B
Fig. 6.1
(i) Explain how the maxima and minima can be observed at the same point X.

.
By adjusting the frequency and hence the wavelength, of the sources
.
Maxima is observed when constructive interference takes place when the path
difference of the two waves is n times of wavelength, where n is integral
.

Minima is observed when destructive interference takes place when the path
.
difference of the two waves is (n+0.5) times of wavelength, where n is integral
.... [3]

SRJC 2013 9646/PRELIM/2013 [Turn Over


14 For
Examiners
Use

(ii) Calculate the frequency at which the first minimum intensity is observed at X.

M 2 X 92 402 41 cm

Path difference = 41 40 = 1 cm

For first minimum, path difference = 0.5, so = 2 cm

3 108
v
f
2 102
1.5 1010 Hz

frequency = Hz [3]

(iii) State if a maximum or minimum intensity will be detected at X when the frequency
is doubled.

A maximum intensity will be detected. (As the path difference is now 1 )


..... [1]

(iv) The detector moves along line AB and detects multiple maxima. State and
explain the effect on the positions of the maxima when

1. State the effect on the positions of the maxima when M1 and M2 are emitting
waves that are in anti-phase.

.
Maxima is observed when the path difference of the two waves is
(n+0.5) times of wavelength, where n is integral.
.

Minima is observed when the path difference of the two waves is n times
.
of wavelength, where n is integral.
.
Or the locations of maximas will now be minimas and vice versa
[1]

2. State and explain the effect on the positions of the maxima when M1 is
gradually moved towards M2.

.
The distance between locations where maxima are observed will get
larger.
.

D
.
Using the approximation that x , when M1 goes closer to M2, a
a
decreases, thus x, the distance between maxima will increase.
.

[2]
SRJC 2013 9646/PRELIM/2013
15 For
Examiners
Use

(v) When more emitters which are in phase with M1 are placed equidistant and along
the line joining M1 and M2, state and explain the effect on the number of maxima
which can be detected when the detector moves along line AB.

.
There should be far fewer maxima detected.
.
With more sources present, it is more difficult for a location to meet the
.
criterion that path difference of all sources must be n for constructive
interference.
.

[2]

SRJC 2013 9646/PRELIM/2013 [Turn Over


16 For
Examiners
Use

7 (a) An electron is placed near two positive charges as shown in Fig. 7.1.
+1.6 x 10-19 C
2 mm
3 mm
A B C
Electron x x
u = 0 m s-1
4 mm 3 mm

+1.6 x 10-19 C
Fig. 7.1
(i) Determine the magnitude of the resultant electric force acting on the electron at
Point A.

Qq (1.6 1019 )(1.6 1019 )


F1.6 9.208 1024 N
4 0 r 2
4 0 (5 10 )
3 2

4
Resultant force =2( F1.6 cos ) 2( F1.6 )( )
5
23
1.473 10 N

resultant electric force at Point A = N [3]

(ii) Determine the resultant electric force acting at the electron at Point B.

Qq (1.6 1019 )(1.6 1019 )


F1.6 1.7711023 N
4 0 r 2
4 0 (3.605 10 )
3 2

2
Resultant force =2( F1.6 cos ) 2( F1.6 )( )
3.605
1.97 1023 N

resultant electric force at Point B = N [2]

(iii) Hence state and explain whether the motion is simple harmonic.
For SHM, acceleration is directly proportional to the distance from equilibrium.
...
In this case, the resultant force (and therefore acceleration) which causes the
motion is not directly proportional to the distance from equilibrium, as seen from
...
the fact that the force is not halved when the distance from equilibrium has been
halved.
Hence motion is not SHM.
...

...

. [3]

SRJC 2013 9646/PRELIM/2013


17 For
Examiners
Use
(iv) Determine the acceleration of the electron at Point B.

Resultant force = ma
1.97 10-23 = 9.1110-31a
a = 2.16 107 m s -2 [1]

acceleration = .. m s-2 [2]

(v) Determine the speed of the electron at Point C when it moves from Point A to C
due to the resultant electric force acting on it.
Total E at A = EPE at A + KE at A
Qq
2( )+0
4 0 r
1.6 1019 1.6 1019
= - 2( )
4 0 5 103
Total E at A =Total E at B = EPE at B + KE at B
1.6 1019 1.6 1019 1.6 1019 1.6 1019 1 2
-2( ) -2( )+ mv
4 0 5 103 4 0 3 103 2
1 2
mv =6.138 1026 J
2
v =367 m s -1

speed = .. m s-1 [3]

SRJC 2013 9646/PRELIM/2013 [Turn Over


18 For
Examiners
Use

(b) Fig. 7.2 shows a spring-mass system placed on a slope which has an angle of from the
horizontal. When a block of m was hung, the spring stretched by an extension of e and the
mass remained in equilibrium. The spring was then further extended by x downwards and
released for the mass spring system to oscillate. The spring constant is k.

Fixed
wall
Equilibrium
position
Frictionless
slope

Fig. 7.2

(i) When the block is at rest at the equilibrium position the tension in the spring is T1. Write the
expression for T1 in terms . [1]
As object is in equilibrium,
T1 = mgsin

(ii) Write the expression for the acceleration a of the block at the lowest point using the least
number of symbols defined in the question. [2]
By N2L, T - mgsin = ma
As mgsin = T1,
T T1 = ma
k(x+e) ke = ma
a = kx / m
(iii) Hence, sketch the graph showing the variation of acceleration a with respect to , at the
lowest point, in Fig. 7.3 when the extension of the spring from the equilibrium position is
kept at x cm. [1]


90
Fig. 7.3
SRJC 2013 9646/PRELIM/2013
19 For
Examiners
Use

(iv) Determine the position from equilibrium when speed of mass is of the maximum speed
during oscillation, in terms of the amplitude x0.

v x02 x 2
0.25v0 x02 x 2
[2]
v0 x02
1 x
1 ( )2
16 x0
15
x x0 0.968 x0 [1]
16

position from equilibrium = . x0 [3]

SRJC 2013 9646/PRELIM/2013 [Turn Over


20 For
Examiners
Use

8 (a) Define the terms magnetic flux and the weber.

..
The magnetic flux through a plane surface is the product of the flux density normal
to the surface and the area of the surface.
.........................................................................................................................
One weber is the magnetic flux through an area of one squared metre, when the
magnetic flux density perpendicular to the area is one tesla.
.......................................................................................................................................

....................................................................................................................................[2]

(b) Fig.8.1 shows a rectangular conducting loop of resistance 1.2 , width 5 cm, and
length 10 cm being pulled horizontally at constant speed v through a region of
uniform magnetic field strength B which has a width of 16 cm. The magnetic field
strength is perpendicular to the plane of the loop and going out of the plane of the
paper

The direction of the motion of the loop is along the plane of the paper and
perpendicular to one side of the loop as shown

At time t = 0 s, the rectangular loop is in the position shown in Fig. 8.1.

16 cm

10 cm
B (out the page)
5 cm v

Fig.8.1

Fig. 8.2 shows how the induced current I flowing in the loop varies with t.
I / mA

3.0

5.0 8.0 13.0 t/s

-3.0

Fig. 8.2

(i) Deduce the speed v of the conducting loop.


In 5.0 s, the loop enters completely into the
v = 10/5 = 2.0 cm s-1

v = .. cm s-1 [1]
SRJC 2013 9646/PRELIM/2013
21 For
Examiners
Use
(ii) Fig.8.2 shows a period when I is positive and another when the current is negative.
State the direction of negative I with reference to Fig. 8.1.

Anticlockwise
Direction of negative I is . [1]

(iii) With clear workings, sketch and label in Fig. 8.3 the variation of induced e.m.f. E with
time t. [2]

E = IR = 3.0 x 10-3 x 1.2 = 3.6 x 10-3 V

E/V
3.6x10-3

5.0 8.0 13.0


t/s

-3.6x10-3

Fig. 8.3

(iv) Hence determine the magnitude of the magnetic field strength B.

E = BLv
B = E / Lv = 3.6 x 10-3 / [(5 x 10-2)(2 x 10-2)] [1]
= 3.6 T [1]

B = T [2]

(v) With clear workings, sketch with labels in Fig. 8.4 the variation of magnetic flux
through the loop with time t. [2]

Max flux = BA = 3.6 (5 x 10-2)(10 x 10-2)


= 0.018 Wb
/ Wb

5.0 8.0 13.0


0.018
t/s

Fig. 8.4

SRJC 2013 9646/PRELIM/2013 [Turn Over


22 For
Examiners
Use

(vi) In order to move the loop with the constant speed an external horizontal pulling force F
must be acting on the loop. With clear workings, sketch in Fig. 8.5 the variation of F with
time t. Take the direction of motion of the loop to be positive. (All forces apart from
magnetic forces are negligible.) [2]

F = BIL = 3.6 (3 x 10-3)(5 x 10-2)


= 5.4 x 10-4 N [1]

F /N

5.4 x 10-4

5.0 8.0 13.0


t/s

Fig. 8.5

(vii) Describe the processes of conversion of energy if any from t = 5.0 s to t = 13.0 s with
clear calculated values.

Energy dissipated from t = 8 s to t = 13.0 s


= VIt = (3.6x10-3) x (3.0x10-3) x (5) [1]
= 5.4x10-5 J [1]

.............
From t= 5 s to t = 8 s, there is no conversion of energy taking place. [1]
.............
From t = 8 s to t = 13.0 s, the work done by F of 5.4x10-5 J is converted into
electrical energy (which is dissipated as heat.) [1]
.............

.............

. [4]

(viii) The direction of the uniform magnetic field B is now changed such that it is directed
into the plane of the page while all other conditions remain the same.

State which graph(s) (Fig. 8.2 to Fig. 8.5) would change and which graph(s) would
remain unchanged.

.............
The direction of the induced currents will be in the opposite directions, implying
that Fig. 8.2 to Fig. 8.4 will all change direction.
.............
Fig. 8.5 will remain unchanged (Lenzs Law will explain this.)
.............

..... [2]

SRJC 2013 9646/PRELIM/2013


23 For
Examiners
Use
(ix) The loop is now replaced with a flat metal conducting plate of the same dimensions and
made to move through the uniform magnetic field while all other conditions remain the
same. Describe and explain the key difference for the external force F required in this
scenario.

.............
F can never be zero throughout time t=0 s to t = 13 s.
When the plate moves entirely within a magnetic field, induced currents can be
.............
generated in separate loops within different parts of the plate.
.............

. [2]

End of paper

SRJC 2013 9646/PRELIM/2013 [Turn Over


1

ST. ANDREWS JUNIOR COLLEGE


PHYSICS 8866/1 (Higher 1)

Friday 20th September 2013 1 hour

Preliminary Examination Paper 1

Instructions to candidates:

1. Write in soft pencil on the OMS.


2. Do not use staples, paper clips, highlighters, glue or correction fluid.
3. Write your name, Civic Group and index number on the separate Optical Mark Sheet (OMS).

There are thirty questions in this paper. Answer all the questions. For each question there are four
possible answers A, B, C, D.
Choose the one you consider correct and record your choice in soft pencil on the separate Optical
Mark Sheet (OMS).

Each correct answer will score one mark. A mark will not be deducted for a wrong answer.

Instructions for using the Optical Mark Sheet (OMS)

Class no.
Write your
numbers here

Register no.

1. Fill in your class number (e.g. 12S27 = "27", 12A01 = 01) in the first two rows.
2. Fill in your class register number in the next two rows. (e.g. register number 1 is filled in as
"01").
3. Write your class and register numbers into the column on the left (ie. 2715)

The Question Paper consists of 14 printed pages (including this cover sheet).

SAJC 2013 Prelims/8866/1 [Turn Over


2

ST. ANDREWS JUNIOR COLLEGE


PHYSICS 8866/1 (Higher 1)

Friday 20th September 2013

Preliminary Examination Paper 1

Data
speed of light in free space, c = 3.00 x 108 m s-1
elementary charge e = 1.60 1019 C
the Planck constant h = 6.63 1034 Js
unified atomic mass constant u = 1.66 1027 kg
rest mass of electron me = 9.11 1031 kg
rest mass of proton mp = 1.67 1027 kg
acceleration of free fall g = 9.81 m s-2

Formulae
uniformly accelerated motion, s = u t + a t2
v2 = u2 + 2 a s
work done on/by a gas, W = p V
hydrostatic pressure, p = gh
resistors in series R = R1 + R2 + ...
resistors in parallel 1/R = 1/R1 + 1/R2 + ...

SAJC 2013 Prelims/8866/1 [Turn Over


3

1 The drag force F experienced by a steel sphere of radius r dropping at speed v through a
liquid is given by

F = arv

where a is a constant.

What would be a suitable SI unit for a?

A N s-1

B N m2 s-1

C kg m-1 s-1

D kg m-1 s-3

2 An ammeter has a zero error. This fault will affect

A neither the precision nor the accuracy of the readings.

B only the precision of the readings.

C only the accuracy of the readings.

D both the precision and the accuracy of the readings.

3 Two quantities X and Y are related by the equation :

X = Y -2.

If the percentage uncertainty of X is 6 %, what is the percentage uncertainty of Y ?

A 2%

B 3%

C 12%

D 18%

4 If a car can be brought to rest from 15 ms-1 in a distance of 12 m, what would be the braking
distance if it is traveling at 30 ms-1? Assuming braking force and road conditions are the same.

A 18 m

B 24 m

C 36 m

D 48 m

SAJC 2013 Prelims/8866/1 [Turn Over


4

5 A velocity-time graph of a journey is shown in the diagram below.

Which is the corresponding acceleration-time graph for the journey?

SAJC 2013 Prelims/8866/1 [Turn Over


5

6 The acceleration-time graph of an object moving in a straight line is as shown.

The object started its motion from rest.

At which point is the body moving with the largest speed?

7 An object of mass 4.0 kg is initially moving with 3.0 m s-1 in the x-direction. A force of 8.0 N
acts in the y-direction for 2.0 s. What is the final velocity of the object?

4.0 kg
3.0 m s-1

8.0 N

A 5.0 ms-1 53o above x-axis

B 7.0 ms-1 53o above x-axis

C 7.0 ms-1 along Y-axis

D 5.0 ms-1 38o above x-axis

8 Two identical objects, each of mass m, are traveling at equal speed v and in opposite direction
along a smooth horizontal surface. They collide head-on and coalesce. Which of the following
statements is true?

A The impulse delivered to each of the object during the interval of the collision is mv.

B The total kinetic energy of the two objects after collision is always greater than zero.

C The total momentum of the system consisting of the two objects is not conserved.

D Each object will rebound with speed lower than v in opposite directions after the collision.

SAJC 2013 Prelims/8866/1 [Turn Over


6

9 Two spheres, A and B, are moving towards each other at speeds u1 and u2
respectively and make a head-on elastic collision. After the collision, A and B move
off with speeds v1 and v2 respectively, as shown.

before collision A B
u1 u2

after collision A B
v1 v2

What is the correct expression that equates the relative speed of approach to the
relative speed of separation?

A u1 + u2 = v1 + v2 B u1 + u2 = v2 v1
C u2 u1 = v2 v1 D u1 u2 = v2 v1

10 A full barrel of weight W is being rolled up a ramp.

The force P is required to hold the barrel at rest on the ramp. Friction between the barrel and
the ramp stops the barrel from slipping.

Which arrow represents the resultant force the ramp exerts on the barrel?

SAJC 2013 Prelims/8866/1 [Turn Over


7

11 A non-uniform object is balanced on a knife-edge. The part of the object on the left hand side
of the knife-edge is P while the other is Q. If this object broken into part P and Q, what is the
ratio : mass of P : mass of Q ? (Assume that P and Q are individually uniform).

4x 2x

P Q

A 2:1

B 1:2

C 4:1

D 1:4

12 Which one of the following objects is in equilibrium?

A A stone trapped in the tread of a rotating tyre

B An air molecule as a sound wave passes through the air

C A steel ball falling at constant speed through oil

D An electron moving through a metal under the action of a potential difference

13 A 50 kg block is released from rest at a height of 5.00 m above the ground. It then travels a
distance of 10.0 m along a curved slope to the ground as shown in the figure below. The final
speed of the block at the end of the slope is 4.90 m s-1 because a constant resistive force acts
on it during descent.

What is the resistive force acting on the block?

A 185 N

B 600 N

C 2450 N

D 22500 N

SAJC 2013 Prelims/8866/1 [Turn Over


8

14 An engine takes in an amount E of thermal energy and, as a result, does an amount W of


useful work. An amount H of thermal energy is ejected. The law of conservation of energy and
the efficiency of the engine are given by which of the following?

Law of conservation of Efficiency


energy
A E=W+H W
W
B E=W+H
E
W
C E+H=W
H
W
D E+H=W
E-H

15 A motor using electrical energy at the rate of 400 W raises a block of weight 120 N. If the block
moves 8.0 m vertically in 4.0 s, the efficiency of the motor is

A 60%

B 48 %

C 30%

D 24%

16 The frequency of a certain wave is 600 Hz and its speed is 330 m s-1. What is the phase
difference between the motions of two points on the wave 0.275 m apart?

A 0

B rad
4

C rad
2
D rad

17 A point source emits 60.0 W of sound. A small microphone of area 0.75 cm 2 detects the sound
at 5.0 m from the source. What is the power detected by the microphone?

A 1.4 10-5 W

B 1.4 10-4 W

C 1.9 10-2 W

D 1.9 10-1 W

SAJC 2013 Prelims/8866/1 [Turn Over


9

18 In a Youngs double slit experiment, when light of a wavelength of 4.5 x 10 -7 m is shone


through the slits which are separated by 1.4 mm, a fringe pattern with a fringe separation of
7.0 x 10-4 m is obtained. What would be the fringe separation if light of wavelength 6.5 x 10 -7 m
is shone through slits which are separated by 0.50 mm?

A 1.7 x 10-4 m

B 3.6 x 10-4 m

C 1.4 x 10-3 m

D 2.8 x 10-3 m

19 A piece of glass tubing with both ends open is closed at one end by covering it with a sheet
of metal as shown in the diagram below. The fundamental frequency is found to be 280 Hz.
If the metal sheet is now removed, what is the new fundamental frequency of the resulting
open tubing? Treat the speed of wave to be constant.

Metal sheet

Closed pipe

A 140 Hz

B 280 Hz

C 420 Hz

D 560 Hz

SAJC 2013 Prelims/8866/1 [Turn Over


10

20 Two loudspeakers are placed 1.2 m apart on two corners of a rectangular floor as shown
below. Both speakers are operating in phase and are giving out a steady frequency of 400 Hz.
Point X is 1.6 m from one speaker. The speed of sound can be taken as 320 ms-1

Which of the following is correct about point X?

1.6 m
X

1.2 m

A It has maximum intensity

B It has intensity between minimum and maximum.

C It has minimum intensity

D It has intensity that varies from minimum to maximum periodically

21 An electron travels around the circuit shown in the diagram. The cell has negligible internal
resistance.

At which point in the circuit does the electron have its maximum electrical potential energy?

SAJC 2013 Prelims/8866/1 [Turn Over


11

22 A circuit containing four resistors is connected across a 12V supply as shown.

What are the resistances R1 and R2?

R1 / R2 /
A 1.5 4.5
B 2.0 2.0
C 2.0 6.0
D 4.5 4.5

23 In the circuit shown below, the cell has negligible internal resistance.

Which of the following is correct?

A I1 = 2I2

B I1 = 2I3

C I2 = 2I3

D I3 = 2I1

SAJC 2013 Prelims/8866/1 [Turn Over


12

24 A wire carrying a current I is placed in the magnetic field of flux density B, and = 40.

What is the angle between the electromagnetic force F and flux density B?

A 0

B 40

C 50

D 90

25 Three parallel conductors, carrying currents, pass vertically through the three corners of an
equilateral triangle XYZ. It is required to produce a resultant magnetic field at O in the direction
shown. What must be the directions of the currents?

O
Resultant
magnetic field

X Z

Into the page Out of the page

A X Y and Z

B Z X and Y

C X and Z Y

D X, Y and Z None

SAJC 2013 Prelims/8866/1 [Turn Over


13

26 A particle, moving through a certain region of space, experiences a non-zero magnetic force.
Which of the following is possible?

A A magnetic field exists in that region and changes the speed of the particle.

B A magnetic field exists in this region and the particles velocity is parallel to the
magnetic field.

C A magnetic field exists in this region and the particle is moving at right angle to the
magnetic field.

D A magnetic field exists in this region and the particle is moving in the direction opposite
to the magnetic field.

27 In a photoelectric experiment, ultraviolet radiation is illuminated on a clean metal housed in an


evacuated chamber and electrons are emitted. If the intensity of the radiation is reduced by
half, which of the following will be reduced by approximately the same factor?

1. The energy of each photon


2. The number of incoming photons per unit time.
3. The photoelectric current.
4. The maximum kinetic energy of the emitted electrons.

A 1, 2, 3

B 1, 4

C 2, 3

D All

28 If the de Broglie wavelength of a free proton with a kinetic energy of 4eV is , what
would be the de Broglie wavelength of this proton if its kinetic energy is 64eV ?

A /16

B /4

C 4

D 16

SAJC 2013 Prelims/8866/1 [Turn Over


14

29 When a parallel beam of white light passes through a metal vapour, dark lines appear
in the spectrum of the emergent light. This is principally because energy is absorbed
and

A is re-radiated as dark colours.

B is re-radiated as ultra-violet.

C is re-radiated uniformly in all directions.

D is re-radiated gradually over a long period of time.

30 The diagram (drawn approximately to scale) shows some of the energy levels of an atom.
Transition P results in the emission of a photon of wavelength 4 x 10-7 m.

A
B

P C

Which one of the transitions A, B, C or D could result in the emission of a photon of


wavelength 8 x 10-7 m ?

SAJC 2013 Prelims/8866/1 [Turn


END OF Over
PAPER
Solutions for 2013 H1 Physics Prelims 2013 MCQ Solutions

1 Answer : C

2 Answer : C

3 Answer: B

4. Answer: D

5 Answer: A

6 Answer: B

7 Answer: A

8 Answer: A

9 Answer: A

10 Answer : C

11 Answer: B

12 Answer : C

13 Answer : A

14 Answer : B
15 Answer : A

16 Answer: D

17 Answer: A

18 Answer: D

19 Answer: D

20 Answer: C

21 Answer: A

22 Answer: C

23 Answer: C
24 Answer: D

25 Answer: C

26 Answer: C

27 Answer: C

28 Answer: B

29 Answer: C

30 Answer: B
ST. ANDREWS JUNIOR COLLEGE
PHYSICS 8866/2 (Higher 1)

Tuesday 3rd September 2013 2 hours

Preliminary Exams Paper 2


Instructions to candidates:

1. Write your name, civics group and class register number in the spaces provided
below.

2. Answer all the questions in the spaces provided in the question paper.

For Examiners Use:


Section A Marks

Name: ____________________ 2
Civics Group: ______________
3
Class Register No: ___________

Section B Marks

Total /80
The Question Paper consists of 25 printed pages (including this cover sheet).
2

ST. ANDREWS JUNIOR COLLEGE


PHYSICS 8866/2 (Higher 1)

Tuesday 3rd September 2013 2 hours

Preliminary Exams Paper 2

Data
speed of light in free space, c = 3.00 x 108 m s-1
elementary charge e = 1.60 1019 C
the Planck constant h = 6.63 1034 Js
unified atomic mass constant u = 1.66 1027 kg
rest mass of electron me = 9.11 1031 kg
rest mass of proton mp = 1.67 1027 kg
acceleration of free fall g = 9.81 m s-2

Formulae
uniformly accelerated motion, s = u t + a t2
v2 = u2 + 2 a s
work done on/by a gas, W = p V
hydrostatic pressure, p = gh
resistors in series R = R1 + R2 + ...
resistors in parallel 1/R = 1/R1 + 1/R2 + ...

SAJC 2013
Prelims/8866 [Turn Over
3

Section A ( Answer ALL questions)

1 (a) An experiment uses the following equation to find the resistivity of a


copper wire.
4l
R=
d2

where d is the diameter and l is the length of the copper wire and R is its resistance.
Numerical values from one experiment were
d = (2.1 0.1) mm
l = (20.000 0.001) m
R = (0.10 0.01) .

(i) Calculate the resistivity of the copper together with its uncertainty. Show the
method you used to determine the uncertainty.

resistivity = .. m [3]

(ii) Explain what the new resistance of the wire would be if it is stretched uniformly
to twice its original length.

....

....

.... [2]

SAJC 2013
Prelims/8866 [Turn Over
4

2 Sand falls vertically onto a horizontal conveyor belt at a rate of 60 kg s-1, as shown in
Fig. 2.1 below.

sand
60 kg s-1
2.0 m s-1

Fig. 2.1

The conveyor belt that is driven by an engine, moves with speed 2.0 m s-1.

When the sand hits the conveyor belt, its horizontal speed is zero.

(a) Name the force F that accelerates the sand to the speed of the conveyor belt.

.[1]

(b) Determine the magnitude of the force F.

force = .. N [2]

SAJC 2013
Prelims/8866 [Turn Over
5

(c) Calculate the additional power P required to move the conveyor belt at constant
speed.

power = W [1]

(d) Determine the rate of change of kinetic energy of the sand.

rate of change of kinetic energy = W [1]

(e) Explain why P and the rate of change of kinetic energy are not equal.

.............

..

..

.....[2]

SAJC 2013
Prelims/8866 [Turn Over
6

3 (a) Define magnetic flux density.

......[1]

(b) A current balance circuit can be used to determine an unknown magnetic flux density.
A current I passes through a pivoted rectangular wire frame, A, which is initially
balanced. The magnet with an unknown magnetic field strength B is then placed near
the middle of one of the sides of the wire frame, with its magnetic field acting
perpendicular to the side of the wire frame and this causes the wire frame to tilt.
Different riders are placed until a rider of mass m is placed which helps the frame
regains its balance as shown in Fig. 4.1 below.

rider I
A pivots MAGNET r
I

p q
Fig. 3.1

(i) State and explain the polarity of the magnet nearest to the wire frame.

...

.....[2]

(ii) Deduce an expression for magnetic field strength B of the magnet, in terms of
p, q, r, m, I and g, where g is the acceleration of free fall. [2]

SAJC 2013
Prelims/8866 [Turn Over
7

(iii) The current magnet is now replaced by a magnet that has double its magnetic
flux density. Suggest a method in which the frame can be balanced without
changing the rider or the frame.

...

.....[1]

(c) A twin core electrical cable made up of two wires as shown in Fig 3.2, is used to supply
current of 0.50 A to a household lamp. The distance between the centres of the wires
is 0.50 cm.

0.50 cm

Electrical
insulation
Wires

Fig. 3.2 Cross-sectional View

(i) Given that the magnetic flux density for a long wire carrying current
0 I
is B , where d is the distance between the two wires, and I is the current
2d
in the wire, calculate the force per unit length between the wires.

(o, permeability of free space = 4 x 10-7 H m-1)

force per unit length = . N m-1[2]


SAJC 2013
Prelims/8866 [Turn Over
8

4 (a) Explain what is meant by photoelectric emission.

[1]

(b) In a photoelectric experiment, a parallel beam of monochromatic radiation is incident


upon a metal surface of area 1.0 10-4 m2 in a vacuum tube. The metal has a work
function of 2.06 eV. The photocurrent against voltage graph is shown in Fig. 4.1
below.
I / A

0.13

-0.70 0 V/V
Fig. 4.1

(i) Calculate the maximum kinetic energy of the photoelectrons emitted.

maximum kinetic energy = J [1]

(ii) Hence, determine the frequency of radiation incident on the metal surface.

frequency = . .Hz [2]

SAJC 2013
Prelims/8866 [Turn Over
9

(iii) If one photoelectron is emitted for every 8000 photons incident on the
metal, calculate the intensity of the radiation incident on the metal surface.

intensity = ...W m-2 [3]

(iv) Sketch on Fig. 4.1 the graph you expect to obtain if the intensity and the
frequency of the radiation are halved. [1]

(v) Suggest a reason for the sloping section of the graph in Fig. 4.1 for negative
values of p.d.

[1]

SAJC 2013
Prelims/8866 [Turn Over
10

5 Dangers associated with exposure to radiation have been recognised for many years. Prolong
exposure to gamma, , radiation can lead to serious cell damages which can eventually lead
to death. As a result of these hazards, measures have been adopted to reduce exposure to
radiation to as low a level as possible. One such measure is to shield individuals from
radioactive sources using radiation absorbing materials.

Experiments have been carried out to investigate the effectiveness of materials as absorbers
of -ray photons. One possible experiment is illustrated in Fig. 5.1.

Fig. 5.1

The count-rate Cx of -ray photons is measured for various thicknesses x of the absorber,
together with the count-rate Co for no absorber. Fig. 5.2 shows the variation with thickness x of
the ratio Cx/Co for lead.

Fig. 5.2

SAJC 2013
Prelims/8866 [Turn Over
11

(a) (i) In the experiment, suggest why it is necessary to have a parallel beam of -
radiation.

...

...[1]

(ii) Explain the evidence provided in Fig. 5.2 for the fact that, theretically, complete
shielding is not possible.

...[2]

(b) Fig. 5.2 indicates that there may be an exponential decrease of the ratio Cx/Co with
thickness x. In order to test this suggestion, a graph of ln(Cx/Co) against x is plotted.
This is shown in Fig. 5.3.

Fig. 5.3

SAJC 2013
Prelims/8866 [Turn Over
12

(i) Show that Fig. 5.3 indicates a relationship of the form

Cx= Co e-x,

where is a constant. [2]

(ii) The constant is known as the linear absorption coefficient. Use


Fig. 5.3 to calculate a value of for lead.

= .. cm-1 [2]

SAJC 2013
Prelims/8866 [Turn Over
13

(c) The linear absorption coefficient has been found to depend on photon energy and on
the absorbing material itself. For -ray photons of one energy, is different for different
materials.

In order to assess absorption of -ray photons in matter such that the material of the
absorber does not have to be specified, a quantity known as the mass absorption m is
calculated. m is given by the expression

m = /

where is the density of the absorbing material.

Values of for 2.75 MeV photons and of for different materials are given in Fig. 5.4.

Fig. 5.4

(i) give a consistent unit for m,

unit for m= .[1]

(ii) use your answer to (b)(ii) to complete the table of values for lead. [1]

(d) Concrete is a common building material which is sometimes used for shielding. The
linear absorption coefficient for 2.75 MeV photons in concrete is approximately 0.09
cm-1.

(i) By reference to Fig. 5.2, calculate the approximately thickness of concrete


which would provide the same level of shielding, for 2.75 MeV photons, as
thickness of 4.0 cm of lead.

thickness = cm [2]
SAJC 2013
Prelims/8866 [Turn Over
14

Section B ( Answer any 2 of the 3 questions)

6 A speed-time graph for a train travelling between two stations is shown as Fig 6.1

20
Speed
/ms-1
15

10

0 5 10 15 20 25 30 35
Fig 6.1 Time /s

Acceleration
/m s-2

Time /s
Fig 6.2

(a) On Fig 6.2, draw the corresponding acceleration-time graph. Put numerical values on the
acceleration and time axis. [3]

SAJC 2013
Prelims/8866 [Turn Over
15

(b) Calculate the distance travelled between the two stations.

Distance = [2]

(c) A student claims that Fig 6.1 could also be the velocity-time graph. State the
requirement for the travel of the train such that the students claim can be valid.

......

.. [1]

(d) The train is divided into 3 cabins; the engine cabin and the 2 passenger cabins,
connected by chains between them. Each cabin is 5 x 103 kg. The average friction on
each cabin is 600 N. During the initial acceleration of the train from rest,

(i) Compute the force required by the engine cabin.

Force = N [2]

(ii) Compute the tension between engine cabin and the passenger cabin.

Tension = . N [2]

SAJC 2013
Prelims/8866 [Turn Over
16

(e) A Daredevil attempts to leap across the Singapore River with his new Subaru WRX as
shown in the Fig 6.3. He started his car at 50 m behind ramp, and accelerated
constantly throughout the whole motion until he leaves the ramp. His agent calculated
the whole motion, and advised the Daredevil to accelerate at a rate of 5 ms -2. As a
physics student, you are tasked to find out if 5 ms-2 is sufficient for the Daredevil to
leap across the Singapore River to reach the ramp of the same height at the opposite
bank.

Fig 6.3
(i) Show that the length of the slope on the ramp, y, is 10 m. [1]

(ii) Show that the speed of the car as it leaves the ramp is 24.5 m s-1 [2]

(iii) Show, with calculations, if the car is able to reach the ramp across the Singapore
River.

...

[4]
SAJC 2013
Prelims/8866 [Turn Over
17

(iv) The same agent also claims that;

While the car is in the air, the engine is not able to apply any force, so the car
will not gain any momentum at all.

State the principle of conservation of momentum and use it to validate the


agents claim.

[3]

SAJC 2013
Prelims/8866 [Turn Over
18

7 (a) Describe what is meant by a polarized wave and hence, explain why sound
cannot be polarized.

.....

.....

.... [2]

(b) The variation with distance x of the displacement y of a transverse wave travelling from
left to right at a particular instant is shown in Fig. 7.1.

Fig. 7.1

(i) The speed of the wave is 40 m s-1. Calculate the period of the wave.

Period of the wave = s [2]

(ii) Hence, sketch on Fig.7.1 the profile of the wave 5 ms later, showing the
positions of particles A & B. [2]

SAJC 2013
Prelims/8866 [Turn Over
19

(c) A wire with a mass per unit length of 1.6 g m-1 is attached to a fixed block A and pulled
by a spring balance as shown in Fig 7.2. The stretched piece of wire is 1.8 m long and
its centre is placed between the poles of a large magnet. An alternating current of
frequency 50 Hz is connected across the wire. This means that the current changes
the direction of flow 50 times per second

Fig. 7.2
(i) When a stationary wave of a particular mode is set up in the wire, the distance
between successive nodes is 0.55 m. Calculate the speed of the waves in the
wire.

Speed of the waves = m s-1 [2]

(ii) The natural frequency f of vibration of the wire in its fundamental mode can be
found using the formula

where l is the length of the wire, T the tension in the wire and its mass per unit
length.

1. Using the formula above, show that the unit of frequency is Hz or s-1.
[2]

SAJC 2013
Prelims/8866 [Turn Over
20

2. The spring balance is pulled with an increasing force. Show that the wire
will oscillate in its fundamental mode when the tension in the wire is 52
N.

.....

.... [2]

(iii) Deduce and explain what will happen if the alternating current is switched to
direct current.

......

......

...... [2]

SAJC 2013
Prelims/8866 [Turn Over
21

(d) The frequency used in a microwave oven is 2.5 GHz. Microwaves from a source reach
point P in a piece of meat directly and by reflection from the walls of the oven. The
direct path is shown and also one of the reflected paths. As shown in Fig. 7.3, SP = 16
cm and SQ = QP = 17 cm.

Fig 7.3
(i) Calculate the path difference between the two microwaves reaching P and
express this path difference as a multiple of the wavelength .

Path Difference = [2]

(ii) In practice the reflection at Q adds the equivalent of half a wavelength to the
path SQP. Determine whether the superposition at P will therefore produce a
maximum or a minimum superposition.

......

......

...... [2]

(iii) Suggest why microwave ovens have rotating turntables.

......

......

...... [2]

SAJC 2013
Prelims/8866 [Turn Over
22

8. (a) Electromotive force (e.m.f) and potential difference (p.d.) may both have the volt as a
unit

(i) Define the volt

..

[1]

(ii) Explain what is meant by this expression A cell has an e.m.f of 12 V.

..

[1]

(b) A heating element has an I V characteristics as shown below in Fig 8

I/A

0 10 20 30 40 50
V/V

Fig 8

SAJC 2013
Prelims/8866 [Turn Over
23

(i) By drawing an appropriate line through the origin on the graph or otherwise,

1. Estimate the largest possible value of resistance of the element

Resistance = [1]

2. Estimate the potential difference which should be applied to the element if it is


to have a resistance of 6

Potential Difference = V [2]

(ii) Calculate the power of this element when it has this resistance of 6 .

Power = W [2]

(iii) A cell of e.m.f. 12.0 V and internal resistance is connected to this heating element. A
voltmeter of infinite resistance is connected in parallel with the heating element. An
ammeter of zero resistance is also connected in series to the heating element.

1. Draw a diagram of such a circuit. Include in the circuit a switch so that the
voltmeter may be used to measure either the e.m.f. E of the cell or the terminal
p.d.

[3]
SAJC 2013
Prelims/8866 [Turn Over
24

2. When the switch is closed for a period of 4.0 minutes, this ammeter shows a
reading of 3.0 A. Calculate the charge which flows past the ammeter and hence
the number of electrons during this time.

Charge = W [1]

Number of Electrons = W [2]

3. While the ammeter shows a constant reading of 3.0 A, the resistance of the
heating element remains constant. Using Fig. 8, estimate the resistance of the
heating element and hence calculate the internal resistance of the battery.

Resistance of heating element = [2]

Internal resistance = [2]

(iv) The heating element is made up of many resistance wires. Such a resistance wire of
length L and cross sectional area A has a resistance R. The length of this wire is held
between two supports. During the manufacture of a typical heating element, the length
of these wires increases when the two supports moves apart. Both the volume and
resistivity still remain constant.

1. Show that the resistance of this wire is directly proportional to the square of its
length. [2]

SAJC 2013
Prelims/8866 [Turn Over
25

2. Suggest a reason why, for the same resistance, manufacturers do not use a
heating element with resistance wires of very small cross-sectional area.

..

[1]

SAJC 2013
Prelims/8866 [Turn
End Over
of Paper
1

Solutions for 2013 H1 Prelims Physics H1 Paper 2

Rd2 (0.10)(0.0021)2
1 (a) (i) = 4l = 4(20)
= 1.732 x10-8 m
R 2d l
= R + d + l

0.01 2(0.1) 0.001
= 0.10 + 2.1 + [1]
20.000
= 0.19529
= (0.19529)(1.732 x 10-8)
= 0.338 x10-8 m [1]

= 1.7 x 10-8 0.3 x 10-8 m [1]

(ii) When the length is stretched to twice its original length, its cross-sectional area
is now halved. [1]

As resistance is directly proportional to its length and inversely proportional to is


cross-sectional area, the resistance is now
four times its original value. [1]

2 (a) friction [1]

m
(b) F = rate of change of momentum = t v [1]
= 60 2.0 = 120 N [1]

(c) P = Fv = 120 2.0 = 240 W [1]


m
(d) K= = t v2 = x 60 x 4 = 120 W [1]

(e) the sand on the conveyor belt must slip to be accelerated;


in slipping kinetic energy is dissipated / lost as internal energy /
heat in the sand and conveyor belt;

OR

there is friction between the sand and conveyor belt;


therefore kinetic energy is dissipated / lost as internal energy /
heat in the sand and conveyor belt; [2]

Award zero for bald statement energy is lost as heat.

SAJC 2013
Prelims/8866 [Turn Over
2

3 (a) Magnetic flux density at a point is the force per unit length of conductor per
unit current carried, placed at that point at right angles to the field. [1]

(b) (i)
rider I
A pivots MAGNET r
I

p q

F
Using Flemings left hand rule, magnetic flux density provided by the magnet is to the left.
[1]

Thus the polarity of the magnet nearest to the side is a north pole.

(iii) Clockwise moments = Anti-clockwise moments


Fq = mgp [1]
BIrq = mgp
B = mgp / Irq [1]

(iv) Choose one of below:

Reducing value of I by half

OR

Shift the pivots towards the magnet until the ratio p/q is increased by twice.

[1]

(c) (i) The magnetic flux density experience by one conductor due to the
0 I
other is calculated by B .
2 d
0 I 4 x107 x0.50
B
Thus 2 d 2 x0.50 x102 [1]
5
2.0 x10 T

F
BI
l
Therefore 2.0 x10 5 x0.50
1.0 x10 5 Nm 1 [1]

SAJC 2013
Prelims/8866 [Turn Over
3

4 (a) It is the phenomenon whereby electrons from a metal are emitted when
em radiation of sufficiently high frequency is incident on the metal.
[1]

(b) (i) From the graph, stopping potential is 0.70 V.

Hence maximum kinetic energy of the photoelectrons


= e Vs = 1.12 10-19 J [1]

(ii) hf = + Ek [1]

f = { (2.06 x 1.6 x 10-19) + 1.12 10-19 } 6.63 x 10-34


= 6.66 x 1014 Hz [1]

This is a 2 or 0 marks question.

(iii) For the photoelectrons:


dQ d ( Ne) dN
I e {dN/dt = N/t}
dt dt dt

dN I 0.13 106
19
= 8.15 x 1011 [1]
dt e 1.6 10
{Value of I = saturation current }

Number of photons incident per second:


dN photon 0.13 106
19
8000 = 6.5 x 1015 [1]
dt 1.6 10

Intensity
E Nhf dN photons hf
=
tA tA dt A
0.13 106 6.63 1034 6.66 1014
19
8000
1.6 10 1.0 104
28.7 W m-2
[1]
14
6.66 x10
(iv) Energy of photon at lower freq = hf 2 = 6.63 x 10-34 x
2
= 2.21 x 10-19 J
Since work function = 2.06 x 1.6 x 10-19 = 3.30 x 10-19 J > hf2,
emission cannot take place, zero current. [1]

(v) It denotes the fact that the electrons are emitted with a range of KE.
[1]

SAJC 2013
Prelims/8866 [Turn Over
4

5 (a) (i) This is to ensure that every photon travels the same distance through
the absorber. [1]

(ii) The curve of Fig. 5.2 does not touch the x-axis, even for large values of x.
[1]

This shows that even for large values of x, a small number of photons will still
pass through. [1]

(b) (i) The graph is of the form,


y = - mx
C
ln x = - mx [1]
Co
Cx
= e-mx
Co
Cx = Co e-mx where m = [1]

(ii) - = gradient of graph [1]


4.0 0
= 9.0 0
= 0.444 cm-1 [1]

(c)
material /cm-1 / g cm-3 m/
lead 0.444 11.3 0.039 [1]

(i) Unit of m = cm-1


g cm-3
= cm2 g-1 [1]

m
(ii) m =

0.444
=
11.3
= 0.039

SAJC 2013
Prelims/8866 [Turn Over
5

Cx
(d) (i) From Fig. 5.2, when x = 4.0 cm, = 0.16 . [1]
Co

Cx
Now, = 0.16 = e-x for concrete
Co
0.16 = e-0.09x
-0.09x = ln 0.16
x = 20 cm [1]

SAJC 2013
Prelims/8866 [Turn Over
6

6 (a) 3 Straight lines including 0 from t =0 to t =31s [1]


2.33 ms-2 t =0 to t =6 s [1]
1.56 ms-2 t =31 to t =40 s [1]
(b) Distance travelled = area of speed-time graph
= ( ) [1]
= 455 m [1]
(c) The train must be travelling in a straight line. [1]
(d) (i) Considering all three cabins,
F 3(600) = 3(5 x 103) 2.33 [1]
F = 3.68 x 104 N [1]
(ii) Considering the engine cabin,
F 600 T = 5 x 103 x 2.33 [1]
T = 2.45 x 104 N [1]
5
(e) (i) sin 30 [1]
y
y 10 m (shown)

(ii) s = 60, u = 0, v = ? and a = 5


v 2 u 2 2as
v 2 0 2 2560 [1]
v = 24.5 ms-1 (shown) [1]

(iii) taking up as +ve, sy=0,u=25.5 sin 30, a = -9.81


1
s ut at 2
2
0 25.5 sin 30t 9.81t 2
1
[1]
2
t = 2.50 s [1]
ux = 24.5 cos 30o
sx = 24.5 cos 30o x 2.50 = 57.4 m [1]

Since 57.4m is less than 80m, the Daredevil is not able to reach the ramp at the
opposite end. [1]

(iv) When the bodies in a system interact, the total momentum is conserved if there
is no net external force acting on them. [2]
When it is in the air, there is still gravitational force acting, which is a net force
on it [1]

SAJC 2013
Prelims/8866 [Turn Over
7

7(a) A polarized wave is a (transverse) wave where the vibrations are only in one direction
in a plane perpendicular to the direction of energy transfer. [1]

Sound cannot be polarized because it is a longitudinal wave where the particles


oscillate parallel to the direction of energy transfer. [1]

7(b)(i) = 0.8 m [1]


v = f = /T
T = 0.8/40 = 0.02 s [1]

7(b)(ii) Graph is sketched shifted to the right by phase difference of /2 (1/4


wavelength) [1]

Positions of A & B drawn correctly [1]

7(c)(i) = 2 X Distance between successive nodes = 2 X 0.55 = 1.1 m [1]

v = f = 50 X 1.1 = 55 m s-1 [1]

7(c)(ii) 1. [f] = [(kg m s-2 / kg m-1)] / m [2]


= s-1

7(c)(ii) 2.


( )
[1]

Since the frequency of the alternating current (driving the oscillation) of 50 Hz is the
same as the natural frequency (of the fundamental mode of vibration) of 50 Hz, the
wire will oscillate in its fundamental mode. [1]

7(c)(iii) The wire will not oscillate and be deflected to one side [1]

Because the magnetic force acting on the current-carrying wire will be in only
one direction, instead of alternating. [1]

7(d)(i) = v/f = 3 X 108 / 2.5 X 109 = 0.12 m [1]

Path Difference = (0.17+0.17) 0.16 = 0.18 m = 1.5 [1]

7(d)(ii) Since the equivalent path difference is 1.5+0.5 = 2 , [1]

The microwaves arrive in phase and hence, the superposition at P will be


maximum [1]

7(d)(iii) There are regions of destructive interference which has zero permanent
intensity. OR regions of constructive interference which has a large
intensity. The food has to be rotated so the food can be cook uniformly. [2]

SAJC 2013
Prelims/8866 [Turn Over
8

8(a)(i) The volt is the potential difference between two points in a circuit if one joule of [1]
electrical energy is converted to other forms of energy when one coulomb of
charge passes between the two points.

(ii)
The cell will convert 12 J of non-electrical energy converted to electrical energy [1]
when 1 coulomb of charge is driven through it..
(b)(i)1. [1]
Resistance = 46.0/5.8 = 7.93
(Accept readings of (46.0 0.5) / (5.8 0.1) = 7.71 to 8.16
2.
V = 30 V [1] for correct
working;
By drawing a straight from the origin with a gradient of 1/6, the point of [1] mark for
intersection (30, 5) will give the corresponding I and V for a resistance of 6 . correct answer
Or any other valid workings

(ii)
Power = IV = (5)(30) [1] for correct
working;
= 150 W [1] mark for
correct answer
(iii)1.
[1] mark each
for correct
A position of
voltmeter,
ammeter and
switch)
[3]
V

2. [1]
Q = It = (3)(4 x 60) = 720 C
Using Q = ne,
No of electrons = Q/e = 720/(1.6 x 10-19) = 4.50 x 1021 [1]
[1]
3. [1]
From the graph, V = 10 V when I = 3.0 A.
Resistance of the element = 10/3 = 3.33 [1]

Using E = I (R+r) [1]


[1]
12.0 = 3 (3.33 + r)
r = 0.667
(iv)1. [1]
l x x 2 [1]
R (where x is the new length)
A (V ) V
x
2. [1]
The heat dissipated will melt/ break the wire easily.

SAJC 2013
Prelims/8866 [Turn Over
1

ST. ANDREWS JUNIOR COLLEGE


PHYSICS 9646/1 (Higher 2)

Friday 20th September 2013 1 hour 15 mins

Preliminary Examinations Paper 1

Instructions to candidates:

1. Write in soft pencil on the separate Optical Mark Sheet (OMS).


2. Do not use staples, paper clips, highlighters, glue or correction fluid.
3. Write your name, Civic Group and index number on OMS.

There are forty questions in this paper. Answer all the questions. For each question there are four
possible answers A, B, C, D.

Choose the one you consider correct and record your choice in soft pencil on the separate Optical
Mark Sheet (OMS).

Each correct answer will score one mark. A mark will not be deducted for a wrong answer.

Instructions for using the Optical Mark Sheet (OMS)

Class no.
Write your
numbers here

Register no.

1. Fill in your class number (e.g. 12S07 = "07", 12S23 = 23) in the first two rows.
2. Fill in your class register number in the next two rows. (e.g. register number 6 is filled in as
"06").
3. Write your class and register numbers into the column on the left (ie. 0706)

The Question Paper consists of 21 printed pages (including this cover sheet).

SAJC 2013 Prelims/9646/1 [Turn Over


2

ST. ANDREWS JUNIOR COLLEGE


PHYSICS 9646/1 (Higher 2)

Friday 20th September 2013 1 hour 15 mins

Preliminary Examinations Paper 1

Data

speed of light in free space , c = 3.00 x 108 m s-1

permeability of free space, o = 4 x 10-7 H m-1

permittivity of free space, o = 8.85 x 10-12 F m-1

= (1 / (36)) x 10-9 F m-1

elementary charge, e = 1.60 x 10-19 C

the Planck constant, h = 6.63 x 10-34 J s

unified atomic mass constant, u = 1.66 x 10-27 kg

rest mass of electron, me = 9.11 x 10-31 kg

rest mass of proton, mp = 1.67 x 10-27 kg

molar gas constant, R = 8.31 J K-1 mol-1

the Avogadro constant, NA = 6.02 x 1023 mol-1

the Boltzmann constant, k = 1.38 x 10-23 J K-1

gravitational constant, G = 6.67 x 10-11 N m2 kg-2

acceleration of free fall, g = 9.81 m s-2

SAJC 2013 Prelims/9646/1 [Turn Over


3

Formulae

uniformly accelerated motion, s = ut + at2

v2 = u2 + 2as

work done on/by a gas, W = p V

hydrostatic pressure, p = gh

gravitational potential, = -Gm/r

displacement of particle in s.h.m., x = x0 sin t

velocity of particle in s.h.m., v = v0 cos t

= ( x 02 x 2 )
Mean kinetic energy of a molecule of an ideal gas,

3
E = 2 kT

resistors in series, R = R1 + R2 +

resistors in parallel, 1 / R = 1 / R1 + 1 / R2 +

Q
electric potential, V =
4 0 r

alternating current/voltage, x = x0 sin t

transmission coefficient, T exp(-2kd)

8 2m(U E)
where k =
h2

radioactive decay, x = x0 exp (-t)

0.693
decay constant, =
t1
2

SAJC 2013 Prelims/9646/1 [Turn Over


4

1 The drag force F experienced by a steel sphere of radius r dropping at speed v through a
liquid is given by

F = arv

where a is a constant.

What would be a suitable SI unit for a?

A N s-1

B N m2 s-1

C kg m-1 s-1

D kg m-1 s-3

2 An ammeter has a zero error. This fault will affect

A neither the precision nor the accuracy of the readings.

B only the precision of the readings.

C only the accuracy of the readings.

D both the precision and the accuracy of the readings.

SAJC 2013 Prelims/9646/1 [Turn Over


5

3 A velocity-time graph of a journey is shown in the diagram below.

Which is the corresponding acceleration-time graph for the journey?

SAJC 2013 Prelims/9646/1 [Turn Over


6

4 The acceleration-time graph of an object moving in a straight line is as shown.

The object started its motion from rest.

At which point is the body moving with the largest speed?

5 Two identical objects, each of mass m, are traveling at equal speed v and in opposite direction
along a smooth horizontal surface. They collide head-on and coalesce. Which of the following
statements is true?

A The impulse delivered to each of the object during the interval of the collision is mv.

B The total kinetic energy of the two objects after collision is always greater than zero.

C The total momentum of the system consisting of the two objects is not conserved.

D Each object will rebound with speed lower than v in opposite directions after the collision.

6 Two spheres, A and B, are moving towards each other at speeds u1 and u2
respectively and make a head-on elastic collision. After the collision, A and B move
off with speeds v1 and v2 respectively, as shown.

before collision A B
u1 u2

after collision A B
v1 v2

What is the correct expression that equates the relative speed of approach to the
relative speed of separation?
A u1 + u2 = v1 + v2

B u1 + u2 = v2 v1

C u2 u1 = v2 v1

D u1 u2 = v2 v1

SAJC 2013 Prelims/9646/1 [Turn Over


7

7 A full barrel of weight W is being rolled up a ramp.

The force P is required to hold the barrel at rest on the ramp. Friction between the barrel and
the ramp stops the barrel from slipping.

Which arrow represents the resultant force the ramp exerts on the barrel?

8 Which one of the following objects is in equilibrium?

A A stone trapped in the tread of a rotating tyre

B An air molecule as a sound wave passes through the air

C A steel ball falling at constant speed through oil

D An electron moving through a metal under the action of a potential difference

9 A 50 kg block is released from rest at a height of 5.00 m above the ground. It then travels a
distance of 10.0 m along a curved slope to the ground as shown in the figure below. The final
speed of the block at the end of the slope is 4.90 m s-1 because a constant resistive force acts
on it during descent.

What is the resistive force acting on the block?

A 185 N B 600 N C 2450 N D 22500 N

SAJC 2013 Prelims/9646/1 [Turn Over


8

10 An engine takes in an amount E of thermal energy and, as a result, does an amount W of


useful work. An amount H of thermal energy is ejected. The law of conservation of energy and
the efficiency of the engine are given by which of the following?

Law of conservation of Efficiency


energy
A E=W+H W
W
B E=W+H
E
W
C E+H=W
H
W
D E+H=W
E-H

11 A pendulum bob of mass 0.55 kg is supported by a string so that the radius of its path is 0.600
m. The pendulum is raised from its lowest point to one side so that it gains a vertical height of
0.300 m, and released from rest. What is the tension in the string at the instant when the
pendulum bob moves pass the lowest point?

A 5.39 N

B 5.41 N

C 10.8 N

D 12.6 N

SAJC 2013 Prelims/9646/1 [Turn Over


9

12 A pendulum bob is tied to a string and set to swing in a horizontal circle as shown

Which diagram shows the forces acting on the pendulum bob at the position shown?

Tension

Centripetal
Centripetal
A B force
force
Weight
Weight

Tension Tension

C Centripetal D
force

Weight Weight

13 Which of the following statement about two satellites of masses X and Y in the geostationary
orbit around the Earth is always false?

A The gravitational potential energy of each satellite is different.

B The kinetic energy of each satellite is different.

C The magnitude of the centripetal acceleration of each satellite is different.

D The magnitude of the angular velocity of each satellite is the same.

SAJC 2013 Prelims/9646/1 [Turn Over


10

14 Two satellites A and B orbit the Earth in circular orbits, the radius of satellite As orbit is 4
times that of satellite B. If the orbital period of satellite A is T, what is the orbital period of
satellite B?

T
A 8

T
B 4

C 4T

D 8T

15 A m ass o f gas en clo sed in a cylin d er b y a p ist o n is h eat ed g en t ly. At t h e sam e


t im e, t h e p ist o n is m o ved so t h at t h e p r essu r e r em ain s co n st an t .

As a r esu lt o f t h is, w h at w ill not o ccu r ?

A Th e aver ag e velo cit y o f t h e m o lecu les w ill in cr ease.

B Th e m ean sep ar at io n o f t h e m o lecu les w ill in cr ease.

C Th e m o lecu les w ill t r avel g r eat er d ist an ces b et w een co llisio n s.

D Th e n u m b er o f co llisio n s p er seco n d o f t h e m o lecu les o n t h e p ist o n w ill


in cr ease.

SAJC 2013 Prelims/9646/1 [Turn Over


11

16 Each b o x sh o w s id en t ical m o lecu les o f a gas r ep r esen t ed b y cir cles w it h ar r o w s


t o sh o w t h e
d ir ect io n o f t r avel an d t h e sp eed o f t h e m o lecu le. A lo n g er ar r o w r ep r esen t s a
h ig h er sp eed .
Wh ich b o x co n t ain s a g as o f t h e h ig h est d en sit y an d t h e lo w est t em p er at u r e?

17 Th e d iag r am sh o w s a velo cit y -t im e g r ap h f o r a m ass m o vin g u p an d d o w n o n


t h e en d o f a sp r in g .

Wh ich p o in t r ep r esen t s t h e velo cit y o f t h e m ass w h en at t h e lo w est p o in t o f it s


m o t io n ?

SAJC 2013 Prelims/9646/1 [Turn Over


12

18 A p ar t icle m o ves in t h e m an n er sh o w n b y t h e velo cit y -t im e g r ap h .


Po in t Q r ef er s t o a p o in t in it s m o t io n .

Wh ich r o w o f t h e t ab le is co r r ect ?

t im es f o r m axim um acceler at io n at
d isp lacem en t / s p o in t Q / m s-2
A 2.5 12.5 2

B 5 15 2
C
2.5 12.5 0
D
5 15 0

19 The frequency of a certain wave is 600 Hz and its speed is 330 m s-1. What is the phase
difference between the motions of two points on the wave 0.275 m apart?

A 0

B rad
4

SAJC 2013 Prelims/9646/1 [Turn Over


13


C rad
2
D rad

20 A point source emits 60.0 W of sound. A small microphone of area 0.75 cm 2 detects the sound
at 5.0 m from the source. What is the power detected by the microphone?

A 1.4 10-5 W

B 1.4 10-4 W

C 1.9 10-2 W

D 1.9 10-1 W

21 In a Youngs double slit experiment, when light of a wavelength of 4.5 x 10 -7 m is shone


through the slits which are separated by 1.4 mm, a fringe pattern with a fringe separation of
7.0 x 10-4 m is obtained. What would be the fringe separation if light of wavelength 6.5 x 10 -7 m
is shone through slits which are separated by 0.50 mm?

A 1.7 x 10-4 m

B 3.6 x 10-4 m

C 1.4 x 10-3 m

D 2.8 x 10-3 m

22 A suspension bridge is to be built across a valley where it is known that the wind can gust at
5 s intervals. It is estimated that the speed of transverse waves along the span of the bridge
would be 400 m s-1.

For what length of the bridge would there be greatest danger of resonance at the bridges
fundamental frequency?

A 40 m

B 80 m

C 1000 m

D 2000 m

23 An electron travels around the circuit shown in the diagram. The cell has negligible internal
resistance.

At which point in the circuit does the electron have its maximum electrical potential energy?

SAJC 2013 Prelims/9646/1 [Turn Over


14

24 A circuit containing four resistors is connected across a 12V supply as shown.

What are the resistances R1 and R2?

R1 / R2 /
A 1.5 4.5
B 2.0 2.0
C 2.0 6.0
D 4.5 4.5

25 In the circuit shown below, the cell has negligible internal resistance.

SAJC 2013 Prelims/9646/1 [Turn Over


15

Which of the following is correct?

A I1 = 2I2

B I1 = 2I3

C I2 = 2I3

D I3 = 2I1

26 Four point charges are arranged at the corners of a square as shown in the figure below.
-2Q +Q

+Q -2Q

What is the value of the electric field strength E and the electric potential V at the centre of the
square?

E V

A zero less than zero

B zero more than zero

C less than zero zero

D more than zero more than zero

27 Two large metal plates are separated by a distance of 5.0 mm. The lower plate is at a potential
of - 5.0 V.

SAJC 2013 Prelims/9646/1


5 mm [Turn Over
16

-5 V

What potential should be applied to the upper plate so that an electric field of 2000 V m-1
pointing upwards is created in the space between the plate?

A - 15 V

B - 10 V

C 5V

D 10 V

28 A wire carrying a current I is placed in the magnetic field of flux density B, and = 40.

What is the angle between the electromagnetic force F and flux density B?

A 0

B 40

C 50

D 90

29 A charged particle is moving in a region where there is a uniform magnetic field. The path of
the particle begins at P and is as shown.

SAJC 2013 Prelims/9646/1 [Turn Over


17

What can be deduced about the charge on the particle, its speed and the direction of the
magnetic field?

Charge Speed Direction of Magnetic field

A Negative Decreasing Into the plane

B Negative Increasing Into the plane

C Positive Decreasing Into the plane

D Positive Increasing Out of the plane

30 A flat circular coil has 120 turns and an area of 0.070 m2. It is placed perpendicularly to a
magnetic field. The flux density of the magnetic field is changed steadily from 80 mT to
- 80 mT over a period of 4.0 s. What is the e.m.f. induced in the coil during this time?

A 1.4 mV

B 4.48 mV

C 168 mV

D 336 mV

31 A circular metal disc is rotating perpendicularly in a magnetic field as shown.

SAJC 2013 Prelims/9646/1 [Turn Over


18

The induced e.m.f. between the axle and the edge of the disc is 12.0 V. If the diameter of the
disc is halved, what is the induced e.m.f. between the axle and the edge of the disc?

A 3.0 V

B 6.0 V

C 12.0 V

D 24.0 V

32 An alternating sinusoidal voltage of amplitude 12 V and frequency of 50 Hz is connected is


series with a diode and a resistor. The power output in the resistor is 9.0 W. If the amplitude,
frequency and resistance are all doubled, the power output in the resistor is

A 4.5 W

B 9.0 W

C 18.0 W

D 36.0 W
33 The secondary coil of an ideal transformer delivers an r.m.s. current of 2.5 A to a load
resistance of 8.0 . If the ratio of primary turns to secondary turns is 20:1, what is the r.m.s
current in the primary coil?

A 0.120 A

B 0.125 A

C 40 A

D 50 A

34 When a parallel beam of white light passes through a metal vapour, dark lines appear
in the spectrum of the emergent light. This is principally because energy is absorbed
and

A is re-radiated as dark colours.

B is re-radiated as ultra-violet.

C is re-radiated uniformly in all directions.

D is re-radiated gradually over a long period of time.

SAJC 2013 Prelims/9646/1 [Turn Over


19

35 The diagram below shows the wave function of a particle tunnelling through a potential energy
barrier from the left. It is observed that the amplitude of the wave function decreases upon
passing through the barrier, but that the wavelength remains constant.

What deductions can be made from these two observations?

potential barrier

Amplitude of the wave Wavelength remains constant


function decreases

A Mass of the particle Momentum of the particle remains


reduces upon passing unchanged after passing through the
through barrier. barrier.

B Amplitude of oscillation Energy of the particle remains


reduces upon passing unchanged after passing through the

SAJC 2013 Prelims/9646/1 [Turn Over


20

through barrier. barrier.

C Mass of the particle Momentum of the particle remains


increases upon passing unchanged after passing through the
through barrier. barrier.

D Reduced probability of Energy of the particle remains


finding the particle after unchanged after passing through the
the barrier. barrier.

36 The uncertainty in the position of an electron is 50 nm. What is the minimum uncertainty in its
velocity?
A 1.2 103 m s1
B 2.3 103 m s1
C 7.3 103 m s1
D 1.1 104 m s1

37
Electron energy

E6

conduction band
E5
E4

E3
E2 valence band

E1

The energy level diagram above shows the energy bands in a semiconductor material. The
shaded areas represent filled energy levels, while the unshaded areas represent empty
energy levels not occupied by electrons. Which of the following statements is true?

A The minimum energy which an electron in the valence band must absorb
before it can reach the conduction band is E4 E3.

B The minimum energy which an electron in the valence band must absorb
before it can reach the conduction band is E5 E3.

C The minimum energy which an electron in the valence band must absorb before it

SAJC 2013 Prelims/9646/1 [Turn Over


21

can reach the conduction band is E5 E2.

D The material is at a temperature of 0 K.

38 Why is laser light monochromatic?

A The atoms in the laser medium are in a state of population inversion.

B The excited atoms in the laser medium are in a metastable state.

C Photons which trigger off stimulated emission produce more photons of the same
energy.

D The photons produced by stimulated emission are reflected back by the use of
reflecting mirrors in the laser system.

39 Radioactive atoms of P disintegrate with a half-life of T to give atoms of Q which are stable. If
the initial ratio of number of P atoms to number of Q atoms is 1:2, find the ratio after a period
of 2T.

A 1:7

B 1:8

C 1:11

D 1:12

40 A radioactive nucleus decays to a daughter nucleus which is an isotope of the original


nucleus. What are the radioactive products emitted?

A one alpha and four beta particles

B four alpha and one beta particles

C two alpha and one beta particles

D one alpha and two beta particles

SAJC 2013 Prelims/9646/1 [Turn Over


1

ST. ANDREWS JUNIOR COLLEGE


PHYSICS 9646/2 (Higher 2)

Tuesday 3rd September 2013 1 hour 45 mins

Preliminary Examinations Paper 2


Instructions to candidates:

1. Write your name, civics group and class register number in the spaces provided
below.

2. Answer all the questions in the spaces provided in the question paper.

For Examiners Use:


Marks

1
Name: ____________________

Civics Group: ______________ 2

Class Register No: ___________


3

The Question Paper consists of 24 printed


9
pages (including this cover sheet).

Total /72
SAJC 2013
Prelims/9646 [Turn Over
2

ST. ANDREWS JUNIOR COLLEGE


PHYSICS 9646/2 (Higher 2)

Tuesday 3rd September 2013 1 hour 45 mins

Preliminary Examinations Paper 2

Data

speed of light in free space , c = 3.00 x 108 m s-1

permeability of free space, o = 4 x 10-7 H m-1

permittivity of free space, o = 8.85 x 10-12 F m-1

(1 / (36)) x 10-9 F m-1

elementary charge, e = 1.60 x 10-19 C

the Planck constant, h = 6.63 x 10-34 J s

unified atomic mass constant, u = 1.66 x 10-27 kg

rest mass of electron, me = 9.11 x 10-31 kg

rest mass of proton, mp = 1.67 x 10-27 kg

molar gas constant, R = 8.31 J K-1 mol-1

the Avogadro constant, NA = 6.02 x 1023 mol-1

the Boltzmann constant, k = 1.38 x 10-23 J K-1

gravitational constant, G = 6.67 x 10-11 N m2 kg-2

acceleration of free fall, g = 9.81 m s-2

SAJC 2013
Prelims/9646 [Turn Over
3

Formulae

uniformly accelerated motion, s = u t + a t2

v2 = u2 + 2 a s

work done on/by a gas, W = p V

hydrostatic pressure, p =gh

Gm
gravitational potential, =
r

displacement of particle in s.h.m., x = x0 sin t

velocity of particle in s.h.m., v = v0 cos t

= ( x 02 x 2 )
Mean kinetic energy of a molecule of an ideal gas,

E = 3/2 kT

resistors in series, R = R1 + R2 +

resistors in parallel, 1 / R = 1 / R1 + 1 / R2 +

Q
electric potential, V =
4 0 r

alternating current/voltage, x = x0 sin t

transmission coefficient, T exp (-2 k d)

8 2m(U E)
where k =
h2

radioactive decay, x = x0 exp (- t)

0.693
decay constant, =
t1
2

SAJC 2013
Prelims/9646 [Turn Over
4

1 (a) An experiment uses the following equation to find the resistivity of a


copper wire.
4l
R=
d2

where d is the diameter and l is the length of the copper wire and R is its resistance.
Numerical values from one experiment were
d = (2.1 0.1) mm
l = (20.000 0.001) m
R = (0.10 0.01) .

(i) Calculate the resistivity of the copper together with its uncertainty. Show the
method you used to determine the uncertainty.

resistivity = .. m [3]

(ii) Explain what the new resistance of the wire would be if it is stretched uniformly
to twice its original length.

....

....

.... [2]

SAJC 2013
Prelims/9646 [Turn Over
5

2 Sand falls vertically onto a horizontal conveyor belt at a rate of 60 kg s-1, as shown in
Fig. 2.1 below.

sand
60 kg s-1
2.0 m s-1

Fig. 2.1

The conveyor belt that is driven by an engine, moves with speed 2.0 m s-1.

When the sand hits the conveyor belt, its horizontal speed is zero.

(a) Name the force F that accelerates the sand to the speed of the conveyor belt.

.[1]

(b) Determine the magnitude of the force F.

force = .. N [2]

SAJC 2013
Prelims/9646 [Turn Over
6

(c) Calculate the additional power P required to move the conveyor belt at constant
speed.

power = W [1]

(d) Determine the rate of change of kinetic energy of the sand.

rate of change of kinetic energy = W [1]

(e) Explain why P and the rate of change of kinetic energy are not equal.

.............

..

..

.....[2]

SAJC 2013
Prelims/9646 [Turn Over
7

3 (a) The equation


pV = constant T

relates the pressure p and volume V of a gas to its Kelvin (thermodynamic)


temperature T.

State two conditions for the equation to be valid.

1. ...............

..........................

2. ...................

,..............[2]

(b) Explain what is meant by the internal energy of a substance.

..........................

......................

,..............[1]

(c) State and explain, in molecular terms, the change in internal energy of some water as
it evaporates at constant temperature.

.........

.......................

.......................

,..............[2]

SAJC 2013
Prelims/9646 [Turn Over
8

4 (a) Define magnetic flux density.

......[1]

(b) A current balance circuit can be used to determine an unknown magnetic flux density.
A current I passes through a pivoted rectangular wire frame, A, which is initially
balanced. The magnet with an unknown magnetic field strength B is then placed near
the middle of one of the sides of the wire frame, with its magnetic field acting
perpendicular to the side of the wire frame and this causes the wire frame to tilt.
Different riders are placed until a rider of mass m is placed which helps the frame
regains its balance as shown in Fig. 4.1 below.

rider I
A pivots MAGNET r
I

p q
Fig. 4.1

(i) State and explain the polarity of the magnet nearest to the wire frame.

...

.....[2]

(ii) Deduce an expression for magnetic field strength B of the magnet, in terms of
p, q, r, m, I and g, where g is the acceleration of free fall. [2]

SAJC 2013
Prelims/9646 [Turn Over
9

(iii) The current magnet is now replaced by a magnet that has double its magnetic
flux density. Suggest a method in which the frame can be balanced without
changing the rider or the frame.

...

.....[1]

(c) A twin core electrical cable made up of two wires as shown in Fig 4.2, is used to supply
current of 0.50 A to a household lamp. The distance between the centres of the wires
is 0.50 cm.

0.50 cm

Electrical
insulation
Wires

Fig. 4.2 Cross-sectional View

(i) Given that the magnetic flux density for a long wire carrying current
0 I
is B , where d is the distance between the two wires, and I is the current
2d
in the wire, calculate the force per unit length between the wires.

force per unit length = . N m-1[2]

SAJC 2013
Prelims/9646 [Turn Over
10

5 (a) State Lenzs Law

...

...

.[1]

(b) A simple transformer with a soft-iron core is illustrated in Fig. 5.1.

Fig. 5.1

(i) Explain why the core is

1. made of iron,

......

..[1]

2. laminated.

..[2]

SAJC 2013
Prelims/9646 [Turn Over
11

(ii) An e.m.f. is induced in the secondary coil of the transformer. Explain how a
current in the primary coil gives rise to this induced e.m.f.

..

......

...

...

..[3]

SAJC 2013
Prelims/9646 [Turn Over
12

6 (a) Explain what is meant by photoelectric emission.

[1]

(b) In a photoelectric experiment, a parallel beam of monochromatic radiation is incident


upon a metal surface of area 1.0 10-4 m2 in a vacuum tube. The metal has a work
function of 2.06 eV. The photocurrent against voltage graph is shown in Fig. 6.1
below.
I / A

0.13

-0.70 0 V/V
Fig. 6.1

(i) Calculate the maximum kinetic energy of the photoelectrons emitted.

maximum kinetic energy = J [1]

(ii) Hence, determine the frequency of radiation incident on the metal surface.

frequency = . .Hz [2]

SAJC 2013
Prelims/9646 [Turn Over
13

(iii) If one photoelectron is emitted for every 8000 photons incident on the
metal, calculate the intensity of the radiation incident on the metal surface.

intensity = ...W m-2 [3]

(iv) Sketch on Fig. 6.1 the graph you expect to obtain if the intensity and the
frequency of the radiation are halved. [1]

(v) Suggest a reason for the sloping section of the graph in Fig. 6.1 for negative
values of p.d.

[1]

SAJC 2013
Prelims/9646 [Turn Over
14

7 (a) Define the decay constant of a radioactive isotope.

..

........[1]

(b) Geothermal energy originates as energy released in the radioactive decay of the
uranium isotope 238
92 U deep inside the Earth. Each nucleus that decays releases
4.2 MeV of energy.

238
[half-life of 92 U = 4.5 109 years, 1 year = 3.1 x 107 s]

238
(i) Calculate the number of atoms in 1 kg of the uranium isotope 92 U.

238
number of 92 U atoms = ..[1]

238
(ii) Calculate the activity of 1 kg of the uranium isotope 92 U.

activity of 1 kg of the uranium = s-1 [2]

SAJC 2013
Prelims/9646 [Turn Over
15

(iii) Calculate the energy released each second by the activity of 1 kg of the
uranium isotope 238
92 U .

energy released each second = J [1]

(c) In hospitals, certain medical procedures are designed to utilize radioactivity in a way
that is advantageous and important from a health perspective that satisfy the desired
diagnostic or treatment objective.

Give 2 suggestions why, in the treatment for thyroid cancer, it is more advantageous to
use radio-nuclides of short half-lives

1. .

..

....[1]

2. .

..

....[1]

SAJC 2013
Prelims/9646 [Turn Over
16

8 Dangers associated with exposure to radiation have been recognised for many years. As a
result of these hazards, measures have been adopted to reduce exposure to radiation to as
low a level as possible. One such measure is to shield individuals from radioactive sources
using radiation absorbing materials.

Experiments have been carried out to investigate the effectiveness of materials as absorbers
of -ray photons. One possible experiment is illustrated in Fig. 8.1.

Fig. 8.1

The count-rate Cx of -ray photons is measured for various thicknesses x of the absorber,
together with the count-rate Co for no absorber. Fig. 8.2 shows the variation with thickness x of
the ratio Cx/Co for lead.

Fig. 8.2

SAJC 2013
Prelims/9646 [Turn Over
17

(a) (i) In the experiment, suggest why it is necessary to have a parallel beam of -
radiation.

...

...[1]

(ii) Explain the evidence provided in Fig. 8.2 for the fact that, theretically, complete
shielding is not possible.

...[2]

(b) Fig. 8.2 indicates that there may be an exponential decrease of the ratio Cx/Co with
thickness x. In order to test this suggestion, a graph of ln(Cx/Co) against x is plotted.
This is shown in Fig. 8.3.

Fig. 8.3

SAJC 2013
Prelims/9646 [Turn Over
18

(i) Show that Fig. 8.3 indicates a relationship of the form

Cx= Co e-x,

where is a constant. [2]

(ii) The constant is known as the linear absorption coefficient. Use


Fig. 8.3 to calculate a value of for lead.

= .. cm-1 [2]

SAJC 2013
Prelims/9646 [Turn Over
19

(c) The linear absorption coefficient has been found to depend on photon energy and on
the absorbing material itself. For -ray photons of one energy, is different for different
materials.

In order to assess absorption of -ray photons in matter such that the material of the
absorber does not have to be specified, a quantity known as the mass absorption m is
calculated. m is given by the expression

m = /

where is the density of the absorbing material.

Values of for 2.75 MeV photons and of for different materials are given in Fig. 8.4.

Fig. 8.4

(i) give a consistent unit for m,

unit for m= .[1]

(ii) use your answer to (b)(ii) to complete the table of values for lead. [1]

(d) Concrete is a common building material which is sometimes used for shielding. The
linear absorption coefficient for 2.75 MeV photons in concrete is approximately 0.09
cm-1.

(i) By reference to Fig. 8.2, calculate the approximately thickness of concrete


which would provide the same level of shielding, for 2.75 MeV photons, as
thickness of 4.0 cm of lead.

thickness = cm [2]
SAJC 2013
Prelims/9646 [Turn Over
20

(ii) Comment on your answer to (i), making a suggestion as to why concrete may
be used, in preference to lead, where radioactive sources of high activity are to
be shielded.

..

...

..

..[1]

SAJC 2013
Prelims/9646 [Turn Over
21

9 It is suggested that a spiral metal spring will undergo a small contraction if an electric
current is passed through it.

With all other variables kept constant, the relation between the amount of contraction c between
two successive loops of the spring when a current I is passed through the spring is

c = k In
where k and n are constants.

Design an experiment to determine the constant n. You should include a diagram showing the
arrangement of the apparatus and describe the procedure to be followed.

Mention briefly some of the difficulties that you may expect to encounter.

When giving your account you should pay particular attention to the following points:

(i) the stiffness of the spring that is to be used,

(ii) the magnitude of the current to be passed through the spring,

(iii) the method of measuring the contraction of the spring (given that it is likely to
be quite small).

[12]

Diagram:

SAJC 2013
Prelims/9646 [Turn Over
22

SAJC 2013
Prelims/9646 [Turn Over
23

SAJC 2013
Prelims/9646 [Turn Over
24

SAJC 2013
Prelims/9646 [TurnOF
END Over
PAPER
1

ST. ANDREWS JUNIOR COLLEGE


PHYSICS 9646/3 (Higher 2)

Tuesday 17th September 2013 2 hours

Preliminary Examinations Paper 3


Instructions to candidates:

1. Write your name, civics group and class register number in the spaces provided
below.

2. Answer all questions in Section A and only two questions in Section B.

For Examiners Use:


Marks
Name: ____________________
Section A
40
Civics Group: ______________
Section B
Class Register No: ___________ 40

Total 80

The Question Paper consists of 27 printed pages (including this cover sheet).

SAJC 2013
Prelims/9646 [Turn Over
2

ST. ANDREWS JUNIOR COLLEGE


PHYSICS 9745/3 (Higher 2)

Tuesday 17th September 2013 2 hours

Preliminary Examinations Paper 3

Data

speed of light in free space , c = 3.00 x 108 m s-1

permeability of free space, o = 4 x 10-7 H m-1

permittivity of free space, o = 8.85 x 10-12 F m-1

(1 / (36)) x 10-9 F m-1

elementary charge, e = 1.60 x 10-19 C

the Planck constant, h = 6.63 x 10-34 J s

unified atomic mass constant, u = 1.66 x 10-27 kg

rest mass of electron, me = 9.11 x 10-31 kg

rest mass of proton, mp = 1.67 x 10-27 kg

molar gas constant, R = 8.31 J K-1 mol-1

the Avogadro constant, NA = 6.02 x 1023 mol-1

the Boltzmann constant, k = 1.38 x 10-23 J K-1

gravitational constant, G = 6.67 x 10-11 N m2 kg-2

acceleration of free fall, g = 9.81 m s-2

SAJC 2013
Prelims/9646 [Turn Over
3

Formulae

uniformly accelerated motion, s = u t + a t2

v2 = u2 + 2 a s

work done on/by a gas, W = p V

hydrostatic pressure, p =gh

Gm
gravitational potential, =
r

displacement of particle in s.h.m., x = x0 sin t

velocity of particle in s.h.m., v = v0 cos t

= ( x 02 x 2 )

Mean kinetic energy of a molecule of an ideal gas,

E = 3/2 kT

resistors in series, R = R1 + R2 +

resistors in parallel, 1 / R = 1 / R1 + 1 / R2 +

Q
electric potential, V =
4 0 r

alternating current/voltage, x = x0 sin t

transmission coefficient, T exp (-2 k d)

8 2m(U E)
where k =
h2

radioactive decay, x = x0 exp (- t)

0.693
decay constant, =
t1
2

SAJC 2013
Prelims/9646 [Turn Over
4

Section A

1 (a) Newtons law of gravitation applies to point masses

(i) State Newtons law of gravitation.

....

[1]

(ii) Explain why, although the planets and the Sun are not point masses, the law
also applies to planets orbiting the Sun.

....

...[1]

(b) The orbit of the Earth around the Sun can be assumed to be circular with a radius of
1.49 x 108 km. The period of the orbit is 365 days.

(i) Calculate the angular speed of the Earth in its orbit around the Sun.

angular speed = . . [1]

(ii) Calculate the mass of the Sun.

mass of Sun = kg [2]

SAJC 2013
Prelims/9646 [Turn Over
5

(c) The actual orbit taken by the Earth is elliptical in shape with a distance of 1.47 x 1011 m
to 1.52 x 1011 m from the Sun. Calculate the change in kinetic energy of the Earth from
the largest to the shortest distance from the Sun. The mass of the Earth is taken to be
6.0 x 1024 kg.

change in kinetic energy = J [2]

SAJC 2013
Prelims/9646 [Turn Over
6

2 A tube, closed at one end, has a constant area of cross-section A. Some lead shot is placed
in the tube so that the tube floats vertically in a liquid of density , as shown in Fig. 2.1.

Fig. 2.1

The total mass of the tube and its contents is M.


When the tube is given a small vertical displacement and then released, the vertical
acceleration a of the tube is related to its vertical displacement y by the expression

Ag
a=- M

where g is the acceleration of free fall.

(a) Define simple harmonic motion.

.................

.................

.............[2]

SAJC 2013
Prelims/9646 [Turn Over
7

(b) Fig. 2.2 shows the variation with time t of the vertical displacement y of the tube in
another liquid.

Fig. 2.2

(i) The tube has an external diameter of 2.4 cm and is floating in a liquid of density
Ag
950 kg m3. Assuming the equation a = - M holds, calculate the mass of the
tube and its contents.

mass = ..................................... kg [2]

(ii) State what feature of Fig. 2.2 indicates that the oscillations are damped.

......

.[1]

(iii) On the same figure Fig. 2.2, sketch a graph to show the variation with time t of
the acceleration of the tube. [1]

SAJC 2013
Prelims/9646 [Turn Over
8

(c) A f r ict io n less t r o lley o f m ass m is h eld o n a h o r izo n t al su r f ace b y m ean s o f


t w o sim ilar
sp r in g s, each o f sp r in g co n st an t k. Th e sp r in g s ar e at t ach ed t o f ixed
p o in t s as
illu st r at ed in Fig. 2.3.

Fig. 2.3

Wh en t h e t r o lley is in eq u ilib r iu m , t h e ext en sio n o f each sp r in g is e.

Th e t r o lley is t h en d isp laced a sm all d ist an ce x t o t h e r ig h t alo n g t h e axis


o f t h e sp r in g s.

Bo t h sp r in g s r em ain ext en d ed .

(i) Sh o w t h at t h e m agn it u d e F o f t h e r est o r in g f o r ce act in g o n t h e


t r o lley is g iven b y
F = 2kx.

[2]

(ii) Th e t r o lley is t h en r eleased . Sh o w t h at t h e acceler at io n a o f t h e


t r o lley is g iven b y
2kx
a=- m

SAJC 2013
Prelims/9646 [Turn Over
9

[1]

3 Two small spherical charged particles P and Q may be assumed to be fixed point charges
located at their centres. The particles are in a vacuum separated by 5.0 cm.

An electron is moved along the line joining the two charges, as illustrated in Fig. 3.1.

x
P Q

Fig. 3.1

The variation with the displacement of electron from P, x of the electric potential energy Ep of
the electron is shown in Fig 3.2.

Ep / eV

x / 10-2 m

Fig. 3.2

(a) State and explain the sign of charge P and Q.

SAJC 2013
Prelims/9646 [Turn Over
10

[2]
(b) Calculate the magnitude of the force acting on the electron when it is at the point x =
3.0 cm.

Force = ..N [2]

(c) Calculate the work done to remove the electron from the point x = 3.0 cm to infinity.

Work Done = J [1]

(d) Sketch the electric field pattern in the space around P and Q. [2]

P Q

SAJC 2013
Prelims/9646 [Turn Over
11

(e) State the effect on the shape of the graph of doubling the charge on particle Q.

...

...

..[1]

SAJC 2013
Prelims/9646 [Turn Over
12

4 (a) An atom has two energy levels with a transition wavelength of 632.8 nm.

There are 7.0 1020 atoms pumped into the upper state with 4.0 1020 atoms in the
lower state. Assuming that the emission of a single laser pulse stops when the two
energy levels have equal numbers of atoms, calculate the energy that could be
released in a single laser pulse.

energy = .J [2]

(b) Explain how band theory accounts for the relative conducting ability of a metal
and intrinsic semiconductor at room temperature.

..

..

..

..

..

..

..

..

..

...

...

...[4]

SAJC 2013
Prelims/9646 [Turn Over
13

(c) Explain using band theory why electrical resistance of an intrinsic semiconductor
material decreases as its temperature rises.

...

...

...

...

...

....

.[4]

SAJC 2013
Prelims/9646 [Turn Over
14

5 (a) Classic experiments on -particles scattering were performed by Rutherford, Geiger


and Marsden. State the experimental observations obtained from such experiments
which suggest that

(i) the nucleus is small, and

.....

..[1]

(ii) the nucleus is massive and positively charged.

....

.[1]

(b) Fig. 5.1 shows a graph of the binding energy per nucleon against nucleon number.
State which nuclear reaction (fusion or fission) uranium-235 tends to undergo and
explain why energy can be released in such a reaction.

binding energy per nucleon

nucleon (mass)
Fe-56 U-235 number

Fig. 5.1

..

..

..

........[2]

SAJC 2013
Prelims/9646 [Turn Over
15

(c) A reaction which takes place in the core of a nuclear reactor is described by the
following equation :

91
235
92 U 01n 142
56 Ba 36 Kr 3 01 n 174.4 MeV

Mass of one nucleus of 92 U = 235.0439 u


235

Mass of one nucleus of 56 Ba = 141.9164 u


142

Mass of one nucleus of 36Kr = 90.9234 u


91

1 u (atomic mass unit) = 1.660 x 10-27 kg, which corresponds to 934 MeV.

Calculate the mass of a neutron in atomic mass unit.

mass of a neutron = u [2]

SAJC 2013
Prelims/9646 [Turn Over
16

Section B

Answer any 2 questions from this section.

6 (a) Fig. 6.1 shows the displacement-time graph of a moving object from a point P.

displacement /m

time /s
0 A B C D E

Fig. 6.1

A student describes the state of motion of the object for time interval A to B as follows:

The object is moving away from point P. It is slowing down and is decelerating
towards P.

(i) Comment and explain the validity of the students description.

...

...

...

...

................. [3]

SAJC 2013
Prelims/9646 [Turn Over
17

(ii) Sketch the corresponding velocity-time graph of the object for the whole
journey. [2]

velocity /m s-1

0
A B C D E time /s

(b) A tennis ball, released from rest, falls vertically to the floor and bounces up. Explain
how Newtons Third Law and the Principle of Conservation of Momentum apply in this
situation.

....

.....

[4]

SAJC 2013
Prelims/9646 [Turn Over
18

(c) A 20-kg projectile is fired at an angle of 600 above the horizontal with a velocity of
400 m s-1, from the ground. At the highest point of its trajectory, the projectile explodes
into two fragments of equal mass. Assume air resistance is negligible.

(i) Explain why the velocity of the projectile at the highest point of the trajectory,
just before the explosion, is 200 m s-1 in the horizontal direction.

...

...

................. [1]

(ii) After the explosion one of the fragments falls vertically with zero initial speed.
Calculate the speed of the other fragment just after the explosion.

speed = m s-1 [2]

(iii) Both fragments eventually land on the ground, at the same level as the firing
position. If the horizontal distance travelled by the projectile just before the
explosion is 7100 m, determine the positions of both fragments when they land on
the ground relative to the firing position.

position of 1st fragment = ... m;

position of 2nd fragment =.m [4]

SAJC 2013
Prelims/9646 [Turn Over
19

(d) A light spring is attached between two masses m1 and m2 resting on a frictionless floor.
A force of 50 N is applied on m1 as shown in Fig. 6.2.

Given: m1 = 2.0 kg, m2 = 4.0 kg, and the spring constant is 12 N cm-1.

F
m1 m2

Fig. 6.2

(i) Calculate the acceleration of the masses.

acceleration = m s-2 [1]

(ii) Calculate the compression of the spring.

compression = cm [3]

SAJC 2013
Prelims/9646 [Turn Over
20

7 (a) Explain what is meant by the diffraction of a wave.

.....

.....

.[2]

(b) A long tube, fitted with a tap, is filled with water. A tuning fork is sounded above the top
of the tube as the water is allowed to run out of the tube, as shown in Fig. 7.1.

Fig. 7.1 Fig. 7.2

A loud sound is first heard when the water level is as shown in Fig. 7.1, and then again
when the water level is as shown in Fig. 7.2.

Fig. 7.1 illustrates the stationary wave produced in the tube.

On Fig. 7.2,

(i) sketch the form of the stationary wave set up in the tube, [1]

(ii) mark, with the letter N, the positions of any nodes of the stationary wave. [1]

SAJC 2013
Prelims/9646 [Turn Over
21

(iii) The frequency of the fork is 512 Hz and the difference in the height of the water
level for the two positions where a loud sound is heard is 32.4 cm.

Calculate the speed of sound in the tube.

speed = m s1 [2]

(iv) The length of the column of air in the tube in Fig. 7.1 is 15.7 cm.
Suggest where the antinode of the stationary wave produced in the tube in
Fig. 7.1 is likely to be found.

....

....

...[2]

SAJC 2013
Prelims/9646 [Turn Over
22

(c) Light reflected from the surface of smooth water may be described as a polarised
transverse wave. Thus, polarised sunglasses have been popular for years with boaters
and fishermen who need to reduce reflected glare from the water surrounding them.

By reference to the direction of propagation of energy, explain what is meant by

(i) 1. a transverse wave,

......

..[1]

2. polarisation.

....

.[1]

(ii) Suggest how polarised sunglasses can be used to reduce glare from the sun
reflecting off water surfaces.

.....

.[1]

SAJC 2013
Prelims/9646 [Turn Over
23

(d) A glass tube, closed at one end, has fine dust sprinkled along its length. A sound
source is placed near the open end of the tube, as shown in Fig. 7.3.

39.0 cm

Fig. 7.3

The frequency of the sound emitted by the source is varied and, at one frequency, the
dust forms small heaps in the tube.

(i) Explain, by reference to the properties of stationary waves, why the heaps of
dust are formed.

....

....

....

.[3]

(ii) One frequency at which heaps are formed is 2.14 kHz.

The distance between six heaps, as shown in Fig. 7.3, is 39.0 cm.

Calculate the speed of sound in the tube.

speed = .........................................m s1 [2]

SAJC 2013
Prelims/9646 [Turn Over
24

(iii) The wave in the tube is a stationary wave. Explain the formation of a stationary
wave. Hence explain what is meant by the speed calculated in (d)(ii).

......

.....

.....

.....

.....

.....

.[3]

(iv) Explain why a node is found at the closed end of the tube.

.....

.....

.[1]

SAJC 2013
Prelims/9646 [Turn Over
25

8 (a) A cell of electromotive force (e.m.f.) E and internal resistance r is connected in


series with a resistor R, as shown in Fig 8.1 below.

EE rr

RR

Fig 8.1

The cell supplies 8.1 103 J of energy when 5.8 103 C of charge moves completely
round the circuit. The current in the circuit is constant.

(i) Show that E is 1.4 V. [1]

(ii) The resistor R has resistance 6.0 . The potential difference between its
terminals is 1.2 V. Determine the internal resistance r of the cell.

r = [2]

(iii) Calculate the total energy transfer in the resistor R.

energy transfer = . J [2]

SAJC 2013
Prelims/9646 [Turn Over
26

(iv) Describe, in terms of a simple model of electrical conduction, the mechanism by


which the energy transfer in the resistor R takes place.

..

..[3]

(b) The graph Fig. 8.2 below shows the V-I characteristic for two 12 V filament lamps A
and B.

Potential
Potential
Difference Lamp
lampAA Lamp
lampBB
/ V difference 1212
/V

0
0 0.5
0.5 1.0
1.0

Current
current // A
A

Fig. 8.2

(i) Based on the features of Fig. 8.2, explain why these lamps do not obey Ohms
law.

.....

.....

.....

.....[2]

SAJC 2013
Prelims/9646 [Turn Over
27

(ii) State and explain which lamp has the greater power dissipation for a potential
difference of 12 V.

..

..

.....[2]

(c) The two lamps in (b) are now connected in series with a 12 V battery as shown in
Fig. 8.3 below.
12VVbattery
12 battery

Lamp A
lamp A Lamp
lamp BB

Fig. 8.3

(i) State how the current in lamp A compares with that in lamp B.

....

....[1]

(ii) Use the V-I characteristics of the lamps in Fig. 8.2 to deduce the total current
from the battery.

current = . A [2]

SAJC 2013
Prelims/9646 [Turn Over
28

(iii) State and explain which lamp will be brighter under its operating conditions.

....

...

..[2]

(d) Six resistors are connected to an emf source and ammeter as shown in Fig. 8.4 below.
2 k X 5 k

10 k

6 k 15 k
B C
Y

A
8 k

emf source

Fig. 8.4

State and explain which of the six resistors can be removed and yet the ammeter
reading will remain unchanged.

...

...

...

...

...

...[3]

SAJC 2013
Prelims/9646 [Turn
END Over
OF PAPER
H2 Physics 2013 Preliminary Exam

Paper 1

Q1 Q2 Q3 Q4 Q5 Q6 Q7 Q8 Q9 Q10
C C A B A A C C A B

Q11 Q12 Q13 Q14 Q15 Q16 Q17 Q18 Q19 Q20
C D C A D D D D D A

Q21 Q22 Q23 Q24 Q25 Q26 Q27 Q28 Q29 Q30
D C A C C A A D A D

Q31 Q32 Q33 Q34 Q35 Q36 Q37 Q38 Q39 Q40
A C B C D A C C C D

Paper 2

Rd2 (0.10)(0.0021)2
1 (a) (i) = = 4(20)
4l
= 1.732 x10-8 m
R 2d l
= R + d +
l
0.01 2(0.1) 0.001
= 0.10 + 2.1 + 20.000 [1]
= 0.19529
= (0.19529)(1.732 x 10-8)
= 0.338 x10-8 m [1]

= 1.7 x 10-8 0.3 x 10-8 m [1]


or 17 x 10-9 3 x 10-9 m

(ii) When the length is stretched to twice its original length, its cross-
sectional area is now halved. [1]
As resistance is directly proportional to its length and inversely
proportional to is cross-sectional area, the resistance is now
four times its original value. [1]

2 (a) Friction [1]

dm
(b) F = rate of change of momentum = dt v [1]

= 60 2.0 = 120 N [1]

(c) P = Fv = 120 2.0 = 240 W [1]


m 2
(d) K= = v = x 60 x 4 = 120 W [1]
t

(e) There is friction between the sand and conveyor belt; therefore part of the
work done by belt is converted into internal energy (or heat) of the sand and
conveyor belt; [2]

3 (a) f ixed m ass/ am o un t o f g as


[1]

id eal g as [1]

(b) su m o f r an d o m kin et ic an d p o t en t ial en er g ies o f t h e


at o m s/m o lecu les o f t h e su b st an ce
[1]

(c) p o t en t ial en er g y in cr eases b ecau se sep ar at io n in cr eases


[1]

kin et ic en er g y u n ch an g ed b ecau se t em p er at u r e u n ch an g ed
[1]

so in t er n al en er g y in cr eases

4 (a) Magnetic flux density at a point is the force per unit length of conductor per

unit current carried, placed at that point at right angles to the field. [1]

(b) (i)

rider I
A r
pivots MAGNET
I

p q

Using Flemings left hand rule, magnetic flux density provided by the
magnet is to the left. [1]

Thus the polarity of the magnet nearest to the side is a north pole. [1]

(ii) Taking moments about the pivot,


Clockwise moments = Anti-clockwise moments

Fq = mgp [1]

BIrq = mgp
mgq
B = [1]
Irq

(iii) Reducing value of I by half; OR

Shift the pivots towards the magnet until the ratio p/q is increased to
twice its original value. [1]

(c) (i) The magnetic flux density experience by one conductor due to the

0 I
other is calculated by B .
2 d

0 I 4 x107 x0.50
B
Thus 2 d 2 x0.50 x102
2.0 x105 T

[1]

F
BI
l
Therefore 2.0 x10 5 x0.50
1.0 x10 5 Nm 1 [1]

5 (a) Induced e.m.f./current produces effect, or acts in such a direction to

oppose the change (in magnetic flux linkage) causing it. [1]

(b) (i) 1. to increase flux linkage


OR, to increase permeability of the medium
OR, to concentrate the magnetic field lines (not flux)
OR, to strengthen the magnetic flux density/flux [1]

2. To reduce eddy current [1]


Thus will reduce energy/heat loss [1]

(ii) Alternating current/voltage gives rise to changing flux in core [1]


Flux links the secondary coil [1]
By Faradays Law, changing flux induces emf in secondary coil. [1]
6 (a) It is the phenomenon whereby electrons from a metal are emitted when
em radiation of sufficiently high frequency is incident on the metal.
[1]

(b) (i) From the graph, stopping potential is 0.70 V (not - 0.70V).

Hence, maximum kinetic energy of the photoelectrons

= e Vs = 1.12 10-19 J [1]

(ii) hf = + Ek

f = { (2.06 x 1.6 x 10-19) + 1.12 10-19 } 6.63 x 10-34 [1]

= 6.66 x 1014 Hz [1]

(iii) For the photoelectrons:

dQ d ( Ne) dN
I e {dN/dt = N/t}
dt dt dt

dN I 0.13 106
19
= 8.15 x 1011 [1]
dt e 1.6 10

{Value of I = saturation current }

Number of photons incident per second:

dN photon 0.13 106


8000 = 6.5 x 1015 [1]
dt 1.6 1019

Intensity
E Nhf dN photons hf
=
tA tA dt A
0.13 106 6.63 1034 6.66 1014
19
8000
1.6 10 1.0 104
28.7 W m-2

(iv) Energy of photon at lower frequency


6.66 x1014
= hf2 = 6.63 x 10-34 x = 2.21 x 10-19 J
2
Since work function = 2.06 x 1.6 x 10-19 = 3.30 x 10-19 J > hf2,
emission cannot take place, zero current. [1]

(v) It denotes the fact that the electrons are emitted with a range of KE.[1]
7 (a) The decay constant of a radioactive isotope is the fraction of the total

number of nuclei that decay per unit time


OR,
probability of decay of a nucleus per unit time. [1]

1
(b) (i) Number of nuclei in 1 kg : N x 6.02 x 10 23 2.53 x 10 24 [1]
0.238

ln 2
(ii) Activity A = N A xN
t 1/2

ln 2
A 9 7
x 2.53 x 10 24 [1]
4.5 x 10 x 3.1 x 10

A = 1.26 x 107 s-1 [1]

(iii) Energy released each second

= Activity x energy per nucleus


= 1.26 x 107 x (4.2 x 106 x 1.6 x 10-19)

= 8.47 x 10-6 J [1]

(c) Any 2 of the following 3, one mark per point.

With short half-life, decay constant is higher. Hence for the same
activity (i.e treatment dosage), only a small number of radioactive
nuclei are needed thus reduces cost.
Since a small amount of radioactive nuclei are used and the half life is
short, they stay in the body for a shorter period of time thus reducing
the harmful effects (of ionizing radiation).
For the same N the activity will be higher hence treatment time can be
reduced.
8 (a) (i) This is to ensure that every photon travels the same distance through

the absorber. [1]

(ii) The curve of Fig. 8.2 does not touch the x-axis, even for large values
of x. [1]

This shows that even for large values of x, a small number of


photons will still pass through. [1]

(b) (i) The graph is of the form,

y = - mx

C
ln x = - mx [1]
Co

Cx
= e-mx
Co

Cx = Co e-mx where m = [1]

(ii) - = gradient of graph [1]

4.0 0
= 9.0 0

= 0.444 cm-1 [1]

(c)

material /cm-1 / g cm-3 m/

Lead 0.444 11.3 0.039 [1]

-1
(i) Unit of m = cm
g cm-3

= cm2 g-1 [1]

m
(ii) m =

0.444
=
11.3

= 0.039
Cx
(d) (i) From Fig. 8.2, when x = 4.0 cm, = 0.16 . [1]
Co

Cx
Now, = 0.16 = e-x for concrete
Co

0.16 = e-0.09x

-0.09x = ln 0.16

x = 20 cm [1]

(ii) Concrete is cheaper than lead.

Moreover, it is easier to mold concrete into any shape when it is needed for
radiation shielding.
9 Diagram
switch

Variable
clamp Resistor

Retort spring
stand Power supply
A

mercury bath

[Drawn a well-labelled diagram showing how the spring is setup, together with an external
circuit connection. At least one end of the spring should be free to move.] [2]
Procedure

1. Set up the experiment as shown in the figure above. One end of the spring is
fixed to a retort stand whereas the other end is free to move in a mercury
bath.
2. Adjust the variable resistor to adjust the current and close the switch.
[Stated how the magnitude of the current is to be adjusted either by using a
power supply with a variable resistor or a variable power supply unit.] [1]
3. Record the current passing through the spring from the ammeter.

[Stated how the measurement of current is made using an ammeter.] [1]

4. With the switch open, the vertical distance between two loops of the spring
can be determined using a travelling microscope. Record this as d0.
The travelling microscope is used to determine the vertical distance between
the same two loops when the switch is closed. Record this as d 1.

Suggested possible ways:


Shining a source of parallel light rays and measuring the distance between two
successive loops from the shadow cast (on a vertical surface) using vernier
callipers, or
Using a travelling microscope to measure the distance between two successive
loops.] [1]
5. Calculate the contraction as follows:

Contraction, c = d0, distance (without current) d1, distance (with current)

[Stated how the contraction can be calculated.] [1]

6. Vary the current through the spring by lowering the resistance setting of the variable
resistor and obtain different sets of readings for the current and the contraction.

Analysis
7. Plot a graph of lg c against lg I to determine n from the gradient of the graph.
[1]
Answer to part (i):
Consideration of the stiffness of the spring mentioning that using a spring with a
small spring constant (less stiff) can give a large contraction. [1]

Answer to part (ii):

Do a preliminary adjustment to take into consideration of the magnitude of the current


using a larger current can result in measurable contraction. [1]

Difficulties Expected to Encounter (any one). [1]

The spring may get too hot if the current passing through it is too large/too
long./or use of gloves
The connecting wires may interfere with the contraction.

Safety / Good Features (any two). [2]

A long spring will give a larger contraction


A mercury bath is used to reduce interference {instead of using a crocodile clip to
connect the lower end of the spring to the rest of the circuit}.
Where a light source is used, the light source should be directed perpendicularly
at the vertical spring, not at any other angle.
Wait for oscillation to stabilize before taking readings
Take reading only at top or bottom of the spring
Take average readings at different loops
Paper 3

1 (a) (i) The (mutual) gravitational force of attraction between two point
masses is proportional to the product of their masses & inversely
proportional to the square of their separation. [1]

(ii) Separation much greater than size of Sun & /planet. [1]

2 2
(b) (i) = T = 365 x 24 x 3600 = 1.99 x10-7 rad s-1 [1]

(ii) The gravitation force provides the centripetal force (for orbit)
GMm
= mr2 [1]
r2
r32
M = G
(1.49 x 1011)3(1.99 x10-7)2
= 6.67x 10-11
= 1.96 x 1030 kg [1]

(c) Change in KE = Change in GPE


GMm GMm
= - r - r [1]
f i
1 1
= -(6.67 x 10-11)(1.96 x 1030)(6x1024)( 1.52x1011 - 1.47x1011 )

= 1.76 x 1032 J [1]

2 (a) It is a motion where the objects acceleration is proportional to displacement


from a fixed point/equilibrium position and [1]
directed towards that fixed point. . [1]

(b) (i) T = 0.60 s [1]


Ag
2 = M
2 ((1.2 10-2)2)(950)(9.81)
(0.6 )2 = M
M = 0.0384 kg [1]

(ii) Decreasing peak height/amplitude [1]

(iii) A reflection of the y-t graph about the t-axis, amplitude must be
decreasing. [1]
(c) (i) The restoring force, F, is a resultant force and the system is not in
equilibrium.

forces in springs are k(e + x) and k(e - x) [1]


resultant = Fleft spring Fright spring
= k(e + x) k(e - x) [1]
= 2kx (shown)

2kx
(ii) F = ma => a=-m

Explain the -ve sign >>


the spring in the opposite direction of the displacement will be
extended more and therefore larger tension and therefore net force in
that direction [1]

3 (a) P is negatively charged and Q is positively charged. [1]


Qpq
Potential energy, Ep = 4 r . Since of the charge of electron is negative, and
o

the graph shows that Ep is positive when the electron is near to P, the
potential of P must also be negative near P. [1]

(b) Gradient of graph (x = 3.0 cm) = 0.8 eV/cm [1]


Magnitude of F = gradient of potential energy graph

Convert the unit to SI unit, F = 1.28 x 10-17 N [1]

(c) Since at that point the potential energy is zero, the work done is zero. [1]

(d)

P is larger in magnitude, with more field lines


(as the field strength is not symmetrical, turning point nearer to Q)
[1]
Arrows are pointing correctly. [1]
(e) Values of potential near Q will be more negative for the same x. [1]

OR, the zero potential energy point will shift towards P, occur at a smaller x

OR, the value of the gradient near Q will become steeper.

4 (a) The largest number of atoms that can undergo stimulated emission is
= 1.5 x 1020 [1]

(1.5 x 1020)(hc)
Total energy in a single pulse of laser = 632.8 10-9
= 47.1 J [1]

(b)

For a metal, when an electric field is applied, electrons in the partially-filled conduction
band can very easily gain energy from the field to jump to unfilled energy states since
they are nearby. [1]

The ease at which these electrons may move to a nearby unfilled/ unoccupied energy
state, plus the fact that there is a high number density of free electrons make metals
very good electrical conductors. [1]

For intrinsic semiconductor, there is an energy gap between the VB & CB. [1]

At room temperature, some electrons from VB gain sufficient thermal energy to be


promoted into the CB, leaving behind holes in the VB. [1]

(c) When temperature is low, few electrons in the VB have sufficient energy to jump
across the energy gap to get into the CB. When temperature rises, more electrons
in the VB receive sufficient thermal energy to enter into the CB leaving holes in the
VB [1]

Electrons in the CB (free electrons) & holes in the VB are mobile charge carriers &
can contribute to current [1]

Increasing the number of charge carriers means lower resistance. [1]

As temperature rises lattice vibrations increases but this effect is outweighed by the
increase in the number of mobile charge carriers. [1]
5 (a) (i) The observation which suggest that the nucleus is small is that most
of the -particles pass through the gold sample undeflected. [1]

(ii) A small number of the -particles were repelled so strongly that they
bounced back or were deflected through large angles. [1]

(b) U-235 undergoes nuclear fission, because during fission of the heavier
nuclides, with lower binding energy per nucleon are separated and
recombined to form more stable nuclides with smaller nucleon number and
thus higher binding energy per nucleon. Hence energy is released in the
process.
[2]

174.4
(c) The energy released is due to a mass difference of
934 = 0.1867 u.

Mass diff = total mass before reaction total mass after reaction [1]
0.1867 u = (235.0439 + mn) (232.8398 + 3mn)
Mn = 1.0087 u [1]

6 (a) (i) The statement is invalid because


Object is moving towards P as displacement from P is decreasing. [1]
It is speeding up as (magnitude of) the gradient which denotes speed,
is increasing with time [1]
It is accelerating towards P as its speed is increasing towards P. [1]

(ii)
velocity/m s-1 Either concave/convex
curve or straight

time/s
0 A B C D E

2 marks for correct graph

(b) By Newtons law of gravitation, the tennis ball exerts a gravitational force
upward on the floor & by Newtons 3rd law the floor exerts an equal and
opposite force acting downward on the ball. [1]

At the point of impact, the ball exerts a repulsive force on the floor downward,
and the floor exerts an equal and opposite force upward on the ball.
[1]
When the ball is released from rest, the total momentum of the system of ball
and the earth is zero. When the ball is falling down, it gains downward
momentum. The floor is gaining upward momentum; and so the total
momentum of the system is still zero and conserved. After the collision, the
ball moves upwards with decreasing momentum in the upward direction. The
floor will move downwards also with decreasing momentum so that the total
momentum of the system is zero and conserved.
At a certain instant during impact where both have zero velocity, the total
momentum of the system is zero and conserved. [2]

(c) (i) Net force is provided by the gravitational force which acts on the
projectile, vertically downwards.
Thus the horizontal component of the velocity of the projectile remains
a constant.
Thus vx = ux = u cos = 400 cos 600 = 200 m s-1 [1]

(ii) By principle of conservation of momentum in x-direction,


M M
M ux = v1,x + v2 ,x
2 2
Given that v1,x = 0,
M M
M (200) = (0) + v2,x [1]
2 2

v2,x = 400 m s-1 [1]

(iii) Since the first fragment falls vertically downward, its displacement from
the starting position is 7100 m also. [1]

The second fragment takes the same time to fall to the ground as it
takes to rise.

Time taken to rise is found from Sx = uxt


ie 7 100 = 200 t
t = 35.5 s [1]

Horizontal distance travelled by second fragment as it falls x2 = v2,xt


= 400 (35.5) = 14 200 m [1]

Hence position of 2nd fragment relative to firing position = 7 100 + x2


= 7100 + 14200 = 21000 m [1]

(d) (i) F = (m1 + m2 ) a


ie 50 = (2.0 + 4.0) a
a = 8.3 m s-2 [1]
(ii) Consider m1: F FS = m1a
FS = 50 2.0 (8.33) = 33.33 N [1]

FS = kx [1]
33.33 = 12 x
x = 2.8 cm [1]
Or,
Consider m2 :Fs = m2a = 4x 8.33 = 33.3 N
FS = kx
33.33 = 12 x
x = 2.8 cm

7 (a) Diffraction of a wave refers to its spreading [bending] after it passes through
an opening (gap or round an obstacle) and moves into the shadow region.[2]

(b) (i) correct shape drawn [1]

(ii) Both nodes marked correctly [1]

(iii) = 0.324 m [1]


v = f
= 512 2 0.324
= 332 m s1 [1]

(iv) = 16.2 cm [1]


either antinode is 0.5 cm above top of tube
OR antinode is 16.2 cm above water surface [1]
(c) (i) 1. vibrations normal to direction of energy propagation [1]
2. vibrations only in one direction in a plane [1]

(ii) direction of polarisation (in sunglasses) is perpendicular to the


polarisation of the reflected light [1]

(d) (i) at (displacement) antinodes / where there are no heaps, wave has
maximum amplitude (of vibration) [1]
at (displacement) nodes/where there are heaps, amplitude of vibration
is zero/minimum [1]
dust is pushed to / settles at (displacement) nodes [1]

(ii) 2.5 = 39 cm [1]


v = f
v = 2.14 103 15.6 10-2
= 334 m s-1 (allow 330, not 340) [1]

(iii) St at io n ar y w ave f o r m ed b y in t er f er en ce / su p er p o sit io n /


o ver lap o f [1]

w ave t r avellin g d o w n t u b e an d it s r ef lect io n OR


t w o w aves o f sam e (t yp e an d ) f r eq uen cy t r avellin g in
o p p o sit e d ir ect io n s
[1]

sp eed is t h e sp eed o f t h e in cid en t / r ef lect ed w aves


[1]

(iv) At the point of reflection, the incident and reflected waves have a
phase difference of radians/anti-phase. [1]

OR

At the closed end, it is a rigid wall and the air molecule next to it
cannot vibrate/move.
W 8.1 x 103
8 (a) (i) E= = 1.4 V [1]
Q 5.8 x 103

(ii) Using E = IR + Ir, => E = V + Ir


=> 1.4 = 1.2 + Ir [1]
Ir = 0.2
Using V = IR => 1.2 = I (6)
=> I = 0.2
Hence, from the first equation, r = 1.0 [1]

(iii) Energy transfer is proportional to the resistance [1]


6
Energy transfer = 6 + 1 8.1 103 = 6.94 x 103 J [1]

(iv) The charge carriers (mobile electrons with KE) collides


with the lattice ions [1]
this collisions result in increase in the vibrational energy of
the lattice [1]
and the higher energy translate to higher temperature [1]
(or work done on the resistor)

(b) (i) Ohms law : The current in a component is proportional to the


potential difference across it provided physical conditions (eg
temperature) stay constant. [1]

As the graphs are not straight lines, the current is not proportional to
the potential difference as temperature changes [1]

(ii) Lamp B must have greater power dissipation since P = IV


and for a similar V of 12 V, Lamp B has a higher current. [2]

(c) (i) Current lamp A equals the current in lamp B [1]

(ii) Find I such That VA + VB = 12 V [1]


From Fig. 8.2, current = 0.4 A [1]

(iii) For constant (similar) current, Power is proportional to potential


difference. ( ie P = IV ) [1]
Lamp A will be brighter as there is higher power dissipation [1]

(d) There will always be no current through the branch XY as potential at X =


potential at Y [1]
R2k R6k
because VBX = VBY and VCX = VCY due to R = R [1]
5k 15k
Thus, the 10 k resistor will have no effect on the overall effective resistance
of the circuit and the current through the ammeter will be unchanged even
when this resistor is removed. [1]
RIVER VALLEY HIGH SCHOOL
YEAR 6 PRELIMINARY EXAMINATION

H1 PHYSICS 8866
PAPER 1
27 SEP 2013
1 HOUR

CANDIDATE
NAME

CENTRE INDEX
NUMBER S NUMBER

CLASS 6
INSTRUCTIONS TO CANDIDATES

DO NOT OPEN THIS BOOKLET UNTIL YOU ARE TOLD TO DO SO.

Read these notes carefully.


Write your name, class and index number in the spaces above.

There are thirty questions in this paper. Answer all questions. For each question, there are four
possible answers, A, B, C and D.
Choose the one you consider correct and record your choice in soft pencil on the separate
Answer Sheet.

Read the instructions on the Answer Sheet very carefully.

Each correct answer will score one mark. A mark will not be deducted for a wrong answer. Any
rough working should be done on the Question Paper.

The total number of marks for this paper is 30.

____________________________________________________________________________
This Question Paper consists of 15 printed pages.

River Valley High School Pg 1 of 15 Year 6 H1 Physics 8866


Preliminary Examination 2013
Data

speed of light in free space, c = 3.00 108 m s1

elementary charge, e = 1.60 1019 C

the Planck constant, h = 6.63 1034 J s

unified atomic mass constant, u = 1.66 1027 kg

rest mass of electron, me = 9.11 1031 kg

rest mass of proton, mp = 1.67 1027 kg

acceleration of free fall, g = 9.81 m s2

Formulae

1 2
uniformly accelerated motion, s ut at
2
v 2 u 2 2as

work done on/by a gas, W pV

hydrostatic pressure, p gh

resistors in series, R R1 R2

resistors in parallel, 1/ R 1/ R1 1/ R2

Q
electric potential, V
4 O r

River Valley High School Pg 2 of 15 Year 6 H1 Physics 8866


Preliminary Examination 2013
For each question there are four possible answers, A, B, C and D. Choose the one you
consider to be correct.

1 The following physical quantities can be either positive or negative.

s: displacement of a particle along a straight line


: temperature on a celcius scale
q: electric charge
V: readings on a digital voltmeter

Which of these quantities are vectors?

A s only
B , V
C s, q, V
D s, , q, V

2 When a beam of light is incident on a surface, it delivers energy to the surface. The
intensity of the beam is defined as the energy delivered per unit area per unit time.

What is the unit of intensity, expressed in SI base units?

A kg s2 B kg s3 C kg m2 s1 D kg m2 s3

3 A small stone is thrown horizontally towards a vertical wall 1.20 m away. It hits the wall
0.80 m below its initial level. Neglecting air resistance, at what speed does the stone hit
the wall?

stone
0.80 m

1.20 m wall

A 3.0 m s1
B 4.0 m s1
C 5.0 m s1
D 6.0 m s1

River Valley High School Pg 3 of 15 Year 6 H1 Physics 8866


Preliminary Examination 2013
4 A small object is projected at an angle of 45 to the horizontal with an initial kinetic
energy E. Neglecting air resistance, what is its kinetic energy when it is halfway up?

A 0
B E/2
C 3E/4
D E/ 2

5 A ball is thrown vertical upwards at time t0. It reaches maximum height at time t1, and
returns to the throwers hand at time t2. Taking upward direction as positive, which of the
following graph correctly represents its motion?

River Valley High School Pg 4 of 15 Year 6 H1 Physics 8866


Preliminary Examination 2013
6 A resupply aircraft is flying at 360 km h1, at an angle of 15 below the horizontal. The
cargo door opens and a supply package drops out. Assuming negligible air resistance,
how much time passes before the packages speed doubles?

A 7.9 s B 10.4 s C 15.2 s D 54.8 s

7 A body of mass 1.0 kg is moving at 10 m s1. A force now acts in its direction of motion,
varying with time as shown.

What is most likely the momentum of the body after 5.0 s?

A 15.0 kg m s1 B 20.5 kg m s1 C 22.5 kg m s1 D 25.0 kg m s1

8 Two identical particles, each of mass m, are travelling in the same direction with
respective speeds v1 and v2. They collide and stick together, moving off with a common
speed. What is the final kinetic energy of the system right after the collision?

1
A
4

m v 12 v 22
1
B
4

m v1 v 2
2

1
C
2

m v 12 v 22
1
D
2

m v1 v 2
2

River Valley High School Pg 5 of 15 Year 6 H1 Physics 8866


Preliminary Examination 2013
9 A rod, made up of 2 uniform portions, A and B, is held in place by a cable under tension,
T. The cable is at an angle of 30 to the horizontal. Given that portions A and B have
masses of 20.0 kg and 30.0 kg respectively, and are of identical length, L, what is the
tension T in the cable?

A 150 N B 190 N C 230 N D 270 N

10 A block, resting on the rough surface of a ramp, is connected to a ball, resting on the
frictionless surface of the same ramp, using a taut but inextensible string. The ball has a
mass of 10.0 kg. The frictional force acting on the block is 30 N. If the ball accelerates
uniformly at 1.0 m s2 down the slope, what is the mass of the block?

A 3.8 kg B 5.0 kg C 6.7 kg D 7.4 kg

River Valley High School Pg 6 of 15 Year 6 H1 Physics 8866


Preliminary Examination 2013
11 A particle at P is acted on by 5 forces as shown. What are the resultant forces, F x and Fy,
along the x- and y-directions?

Fx Fy
A 6.0 N 0.8 N
B 10.8 N 11.2 N
C 0.8 N 6.0 N
D 6.0 N 8.0 N

12 An object of weight 50 N has an initial velocity of 10 m s1 projected at an inclined slope


of 30 and come to rest at a vertical height of 4 m.

What is the average friction force acting on the object?


A 6.9 N
B 14 N
C 27 N
D 55 N

13 A small metal sphere of mass m is moving through a viscous liquid. When it reaches a
constant downward velocity v, which of the following describes the changes with time, t in
the kinetic energy and gravitational potential energy of the sphere?

kinetic energy gravitational potential energy


A constant and equal to mv2 decreases at a rate of mgv
B constant and equal to mv2 decreases at a rate of (mgv mv2/t)
C increases at a rate of mgv decreases at a rate of mgv
D increases at a rate of mgv decreases at a rate of ( mv2/t mgv)

River Valley High School Pg 7 of 15 Year 6 H1 Physics 8866


Preliminary Examination 2013
14 A gas undergoes a process A B C.

What is the work done by the gas in this process?

A PB(V2 V1) B V2(PB PA) C V1(PB PA) D PA(V2 V1)

15 A beam of unpolarised light with amplitude A and intensity I is passed through two optical
polarisers. The first polarisers transmission axis is oriented at 60 to the vertical, while
the second polarisers transmission axis is oriented at 45 to the horizontal.

What is the intensity of the light at P and amplitude of the light at Q?

Intensity of light at P Amplitude of light at Q

A 1 1
I A cos15
4 2

B 1 1
I A sin15
4 2

C 1 1
I A cos15
2 2

D 1 1
I A sin15
2 2

River Valley High School Pg 8 of 15 Year 6 H1 Physics 8866


Preliminary Examination 2013
16 A point source of sound emits energy equally in all directions at a constant rate and a
person 9.0 m from the source listens. After a while, the intensity of the source is halved.
If the person wishes the sound to seem as loud as before, how far should he be now
from the source?

A 3.4 m
B 4.5 m
C 5.5 m
D 6.4 m

17 A progressive wave is traveling from left to right.

Which of the points on the wave are in phase?


A P and Q B Q and R C Q and S D Q and T

18 Waves from a source S reach P through two possible paths, X and Y.

Given that the resultant wave amplitude at P is a minimum, what is the phase difference
between the two waves from S?

A 2n 1 rad
B n rad
C 1
n rad
2
D 1
2n rad
2

River Valley High School Pg 9 of 15 Year 6 H1 Physics 8866


Preliminary Examination 2013
19 Plane waves of wavelength in a ripple tank travel towards a straight barrier parallel to
the wave fronts. There are 2 gaps of identical width, spaced d apart. Which of the
following and d will produce the narrowest-spaced interference patterns at a screen
distance D away?

/ cm d / cm

A 1.3 2.3

B 1.3 5.2

C 2.6 5.2

D 2.6 2.6

20 Which of the following statements is true about the characteristics of a stationary wave?

A Particles at the antinodes have minimum potential energy.


B Particles at the nodal positions have maximum kinetic energy.
C Separation between a node and the adjacent antinode is half a wavelength.
D Amplitude of vibrations varies from a minimum at the node to a maximum at the
antinode.

River Valley High School Pg 10 of 15 Year 6 H1 Physics 8866


Preliminary Examination 2013
21 The graph shows the current-voltage (I-V) characteristic of an electrical component.
I
I

I1

I0

0
0 V0 V1 V
What is the resistance of the component at potential difference V1 and how does the
resistance change, if at all, when the potential difference increases from V0 to V1?

resistance at V1 Resistance change from V0 to V1


V1 V0 no change
A
II1 -II0
1 0

V1 V0 decreases
B
I1I - II0
1 0

V1 no change
C
II1
1

V1 decreases
D
I1
I1

River Valley High School Pg 11 of 15 Year 6 H1 Physics 8866


Preliminary Examination 2013
22 A battery of e.m.f E and internal resistance r delivers a current I through a resistance R

r
I
A

R is set at two different values and the corresponding currents I, measured using an
ammeter of negligible resistance, are shown in the table below.

R/ I/A
3.0 1.14
1.0 2.67

What is the value of the e.m.f. E ?


A 3.0 V B 3.4 V C 4.0 V D 6.1 V

A battery of e.m.f. 10.0 V and negligible internal resistance is connected to four resistors,
23
each of resistance R, and a variable resistor T, as shown below.

R
R

R
T R
10.0 V
V

RR

R
R

The resistance of T changes from 0 to 5R.

What is the change in the reading of the ideal voltmeter?

A zero B 2.14 V C 3.33 V D 3.57 V

River Valley High School Pg 12 of 15 Year 6 H1 Physics 8866


Preliminary Examination 2013
24 The diagram below shows a circuit with a uniform wire XY of length 1.0 m and resistance
1.0 connected in series with a cell of e.m.f. 3.0 V and internal resistance 0.50 . The
circuit is used to measure the e.m.f. generated by a photocell

3.0 V
0.50

X Y

0.20 V
photocell

When the e.m.f generated by the photocell is 0.20 V and the galvanometer gives a zero
reading, the position of the jockey will be

A 10 cm away from X.
B 20 cm away from X.
C 80 cm away from X.
D 90 cm away from X.

25 A variable resistor is connected in a circuit. For one value of resistance, the readings of
ammeter and voltmeter are 0.25 A and 1.00 V, while for another value of resistance the
readings are 0.30 A and 0.90 V, correspondingly.

What are the values of the e.m.f. and internal resistance of the e.m.f. source?
A 1.8 V; 1.0 B 2.0 V; 0.5 C 1.5 V; 2.0 D 3.0 V; 2.0

River Valley High School Pg 13 of 15 Year 6 H1 Physics 8866


Preliminary Examination 2013
26 A rigidly-clamped straight horizontal current-carrying wire is held midway between the
poles of a magnet on a top pan balance. The wire is perpendicular to the magnetic field.

110 g

The balance, which was zeroed before the switch was closed, reads 110 g after the
switch is closed. The current is reversed and doubled.

What will be the final reading on the balance?

A 220 g B 110 g C zero D 110 g

27

A square coil WXYZ is suspended with its plane vertical from a string. The coil with N
turns carrying a current I, is in a uniform horizontal magnetic field B which makes an
angle with the plane of the coil.

If the area of the coil is given by A, what is the couple experienced by the coil?

A BIAN
sin
B BIAN
cos
C BIAN sin
D BIAN cos

River Valley High School Pg 14 of 15 Year 6 H1 Physics 8866


Preliminary Examination 2013
28

Three identical infinitely long wires carrying current I, with directions as indicated are
arranged equally spaced apart along a circle. The three currents contribute to the
magnetic field at the centre of the circle. What could be the most likely direction of the
resultant magnetic field at the centre?

29 An electron is accelerated from rest through a potential difference of 54 V, gaining kinetic


energy of value E.
Which of the following shows the wavelength of the accelerated electron and that of a
photon of energy E ?

electrons wavelength / m photons wavelength / m


A 2.3 10 8
400 109
B 1.7 1010 1.7 1010
C 2.3 108 2.3 108
D 1.7 1010 2.3 108

30 Diagram below shows three energy levels of a hydrogen atom, with the values of the first
and third levels given.

n=3 -1.51 eV

n=1 -13.6 eV

A hydrogen atom absorbs a photon of wavelength such that the electron in the ground
state is brought to an excited level of n = 3.
What is the maximum wavelength of a photon that can cause ionisation (electron
completely removed from the atom) of a hydrogen atom in the ground state?

A 8 / 9
B 9 / 8
C 3 / 2
D 2 / 3

END OF PAPER

River Valley High School Pg 15 of 15 Year 6 H1 Physics 8866


Preliminary Examination 2013
EOY 2013

Y6 H1 Physics

1 Only displacement is a vector quantity.

Ans: A

2 [Intensity] = -

Ans: B

3 vertically: v2 = u2 + 2 as
= 0 2(9.81)(-0.8)
Vy = 3.9618

Vertically : S = ut + a t2
t 0.40 8 55
Horizontally, s = u t
. ux = 2.97 = vx

Therefore v = sqrt(vx2+vy2) = 4.9522 = 5.0

Ans: C

Distracter: option A : consider only vx = 2.97 = 3.0


Distracter: option B : consider only vy = 3.96 = 4.0
Distracter: option D : no reasoning, just follow natural sequence of the other
options.

4 Given Ek = m u2 = E at bottom;
assume Ep = 0 at bottom; hence total energy = E

At top; Ep = mgH;
Ek = m (ucos45)2 = ( mu2 ) = E
Since total energy = E; therefore Ep at top = E

At halfway; Ep = mg (H/2) = E
Ek = E E = E

Ans: C

Distracter: option A : to sieve out the dice thrower


Distracter: option B : obvious distractor; halfway
Distracter: option D : cos 45 = 1/ 2

5 At t1 and t3 gradient of the graph have same magnitudes. This mean that the upward
velocity at t1 and downward velocity at t3. At t2, gradient equals to zero, meaning the
ball is momentarily at rest at its maximum height.

Ans: A

6 360 km h-1 100 m s-1


Initial horizontal vel. = final horizontal vel. = 100 cos15 = 96.59 m s-1
Initial vertical vel. = 25.89 m s-1

River Valley High School Pg 1 of 6 Year 6 H2 Physics 9646


P 2013
Final vel. = 2 x 100 m s-1
Final vertical vel. = (2002 96.592)1/2 = 175.13 m s-1
Using vy = uy + ayt 175.13 = 25.89 + 9.81(t) t = 15.2 s

Ans: C

7 mv 5 2 5 3
Favg 1v f 10 v f 22.5
t 2 2
Final momentum = 22.5 kg m s-1
However since 2.0 5.0 s is non-linear decrease of force, final momentum will be
less than 22.5 ks m s-1, but greater than 15.0 ks m s-1, which is given by when t is
until 2.0s.

Ans: B

By C.O.M.

mv1 + mv2 = (m + m)v

v = (v1 + v2)/2

Final KE of system is (m + m)v2 = m(v1 + v2)2

Ans: B

2L sin60T cos 30 L cos 60 209.81


9
2
3L
cos 60 309.81 2L cos 60T sin30
2
2 sin60T cos 30 2 cos 60T sin30 269.775
269.775
T 270 N
1
Ans: D

10 Considering the block:


T m9.81sin30 30 m1
Considering the ball:
Fnet 101 109.81cos 45 T
m = 4.9732 kg

River Valley High School Pg 2 of 6 Year 6 H2 Physics 9646


P 2013
Ans: B

11 Resolving horizontally,

Fx = + 10 5cos40 6cos60 4 = 0.83 N

Resolving vertically,

Fy = + 8 + 5sin40 6sin60 = 6.02 N

Ans: C

12 Total Initial Energy = KE = 254.8J


Total Final Energy = GPE = 200J
Change in Energy = Loss to Friction
54.8 = F x d
d = 4/sin30
F = 6.855N = 6.9 N

Ans: A

13 Sphere reached terminal velocity = constant Velocity = constant KE.


GPE decreases at mgh/t = mgv.

Ans: A

P dV PB V2 V1
14 V2
WD = V1

Ans: A

15 After passing 60,


Pamplitude = A Pintensity = kA2 = I

After passing 45,


Qamplitude = A sin15

Ans: B

16 Intensity = Power/Area
P
Ia
4 9 2
1
P
2
Ia
1
4r
2

2
81
r 6.36 m
2

Ans: D

River Valley High School Pg 3 of 6 Year 6 H2 Physics 9646


P 2013
17 Particles are considered to be in pha e when they execute the a e otion at the
same time. To check that two points are in phase we check that they have the same
velocity at the same displacement from the equilibrium position.

Ans: C

18 Since the two wave trains from S start in phase, and resultant wave at P is
minimum, this implies that the path difference / , / , 5/ or that the pha e
difference , , 5 ( n + 1) rad where n 0, 1, ,

Ans: A

19 x = (1.3)(D)/(5.2) = 0.25D

Ans: B

20 Ans: D

21 Ratio is the ratio of the pd and current.


The ratio of this Pd and Current changes from V0 to V1.

Ans:D

22 E = I(R) + I(r)
E = 1.14 (3) + 1.14 (r)
E = 2.67 (1) + 2.67 (r)
Si ultaneou Que t ion.

E = 3.98 V
Ans: C

23 When T = 0, V = 0 V
Same potential at the 2 points of the circuit.

When T = 5R, difference in V = 5 - 10/7 V


Pd = 3.57 V

Ans: D

24 Pd across XY = 1/1.5 x 3 = 2 V
Pd across X and balance pt = 0.2 V
0.2/2 x 100 cm = 10 cm

Ans: A

25 Let e.m.f. of the source = E and its internal resistance = r


For V = 1.00 V, I = 0.25 A,
Re i t ance of R V/I 4

For V = 0.90 V, I = 0.30 A,


Re i t ance of R V/I
Therefore for R 4, I E/(r + R) 0.25 = E/(r +4)

For R , I E/(r + R) 0.3 = E/(r + 3)

River Valley High School Pg 4 of 6 Year 6 H2 Physics 9646


P 2013
Solvin both equation , r ,E 1.5 V

Ans: C

26 Newton rd Law states that the upwards magnetic force on the wire would be equal
and opposite to the downwards magnetic force on the magnet on the top pan
balance (equivalent to a weight of 110 g).
Hence when current is reversed, and doubled (FB = BIL), the reading on the balance
would be 220 g.

Ans: A

27
X
X
W
Y
W

Let F be the force acting on the vertical limb WZ or XY.

Let WZ = XY = a and WX = ZY = b
Then F = BIaN
Referring to plan view of coil, the couple produced is C = Fbcos
Thus C = (BIaN)b cos = BIabN cos = BIAN cos

Ans: D

28

Ans: B

29 Electron: qV = 8.64 10-18; Ek = p2/ 2m = (h/)2 / 2m


Hence 1.7 1010

Photon: E = hc/ . . 108

Ans: C

River Valley High School Pg 5 of 6 Year 6 H2 Physics 9646


P 2013
Distracter: electron uses E = hc/ as well.
Photon uses same value as electron.

30 E = hc /
(13.6 1.51) = hc / .. (1)

For ionization, need at least 13.6 eV


13.6 eV = hc / 2 .. ( )

Eqn 1/ 2 : 12.09 / 13.6 = 2 /

2 = 0.88897
Hence for ionization , wavelength needed is 0.88897 or less.

Hence option B and C are out.

8 / 9 = 0.88888
2 / 3 = 0.66666

Both A and D can cause ionization, but question asked for the maximum.

Ans: A

River Valley High School Pg 6 of 6 Year 6 H2 Physics 9646


P 2013
RIVER VALLEY HIGH SCHOOL
YEAR 6 PRELIMINARY EXAMINATION

H1 PHYSICS 8866
PAPER 2
25 SEP 2013
2 HOURS
CANDIDATE
NAME

CENTRE INDEX
NUMBER S NUMBER

CLASS 6
INSTRUCTIONS TO CANDIDATES
DO NOT OPEN THIS BOOKLET UNTIL YOU ARE TOLD TO DO SO.
Read these notes carefully.
Write your name, centre and index number and class in the spaces above.

Section A and Section B FOR EXAMINERS USE


Candidates answer on the Question Paper. Section A
Write in dark blue or black pen. 1 /8
You may use a soft pencil for any diagrams, graphs or rough
working. 2 /8
Do not use paper clips, highlighters, glue or correction fluid. 3 /8
Section A 4 /8
Answer all questions.
It is recommended that you spend about 1 hour on this 5 /8
section. Section B

Section B 6 /20
Answer any two out of the three questions. 7 /20
It is recommended that you spend about 1 hour on this
section. 8 /20
Deduction
The number of marks is given in brackets [ ] at the end of
each question or part question. TOTAL /80
____________________________________________________________________________
This paper consists of 19 printed pages.

River Valley High School Pg 1 of 19 Year 6 H1 Physics 8866


Preliminary Examination 2013
Data

speed of light in free space, c = 3.00 108 m s1

elementary charge, e = 1.60 1019 C

the Planck constant, h = 6.63 1034 J s

unified atomic mass constant, u = 1.66 1027 kg

rest mass of electron, me = 9.11 1031 kg

rest mass of proton, mp = 1.67 1027 kg

acceleration of free fall, g = 9.81 m s2

Formulae

1 2
uniformly accelerated motion, s ut at
2
v 2 u 2 2as

work done on/by a gas, W pV

hydrostatic pressure, p gh

resistors in series, R R1 R2

resistors in parallel, 1/ R 1/ R1 1/ R2

Q
electric potential, V
4 O r

River Valley High School Pg 2 of 19 Year 6 H1 Physics 8866


Preliminary Examination 2013
Section A

Answer all questions.

1 (a) Define random error and systematic error.

[2]

(b) A double-pulley laboratory experiment is conducted which obeys the following


equation,

T cos2 W T / cos2

where T and T/ are experimentally obtained tensions, and W is the slotted mass
provided by the laboratory. and are measured and recorded as
(41 2) degrees and (35 2) degrees, respectively, while T/ is measured to be
(7.05 0.02) N. W provided by the laboratory is indicated to be (5.00 0.01) N.

Determine the absolute uncertainty of T.

absolute uncertainty of T = . s [4]

River Valley High School Pg 3 of 19 Year 6 H1 Physics 8866


Preliminary Examination 2013
(c) A student derived the parabolic path of a projectile motion to be described by the
following equation,

2
sX 1 sX
sY u sin g
u cos 2 u cos

where sX and sY are the horizontal and vertical displacements respectively, u is the
initial velocity, is the angle of elevation from the horizontal, and g is the gravitational
acceleration.

Show that the equation is homogeneous.

[2]

2 The force that one neutral atom exerts on the other in a di-atomic molecule, varied with
respect to distance, is given by Fig. 2.1.

Fig. 2.1

River Valley High School Pg 4 of 19 Year 6 H1 Physics 8866


Preliminary Examination 2013
(a) (i) Estimate, with justification, the work done in moving one of the atoms from a
distance of 0.6 nm to 0.5 nm from the other atom.

..

..

..

.. [2]

(ii) Estimate, with justification, the work done in separating the two atoms in the
di-atomic molecule, from an equilibrium position to infinite distance apart.

..

..

..

.. [2]

(b) A stream of such neutral atoms is stripped of their outermost electron. This stream of
particles is then passed right between two oppositely charged plates, as shown in
Fig. 2.2.

Fig. 2.2

(i) Describe how the electric field affects the path of the particles.

..

..

.. [2]

River Valley High School Pg 5 of 19 Year 6 H1 Physics 8866


Preliminary Examination 2013
After exiting the electric field, the particles enter perpendicularly into a uniform
magnetic field region, as shown in Fig. 2.3, where the magnetic field is turned on
only when the entire stream of particles has entered this region.

Fig. 2.3

(ii) Describe how the uniform magnetic field region affects the motion of the stream
of particles.

..

..

.. [2]

3 (a) Define electromotive force (e.m.f.).

. [1]

(b) The following electrical circuit of an ideal e.m.f. source and 3 identical bulbs each of
5-ohms resistance is set up, with the symbols depicting their respective standard
components.

Fig. 3.1

River Valley High School Pg 6 of 19 Year 6 H1 Physics 8866


Preliminary Examination 2013
(i) The variable resistor is set at 100 , and then the switch is closed. Determine the
potential difference between the points A and B.

potential difference between A and B = V [2]

(ii) Determine the maximum value of the variable resistor to be set such that the
power dissipated by any of the 3 identical bulbs is at most 0.538 W.

maximum resistance = [3]

(iii) With reference to the scenario in part (i), state and explain the effect on the
brightness of the 3 bulbs, if any, if the e.m.f. source used is non-ideal.

..

..

..

.. [2]

4 (a) Write down the equation defining magnetic flux density in terms of the force, F, it
produces on a long straight conductor of length L carrying a current I at an angle to
the field. In the space provided, draw and label a clear diagram to illustrate the
direction of the force relative to the current and magnetic field.

[3]

River Valley High School Pg 7 of 19 Year 6 H1 Physics 8866


Preliminary Examination 2013
(b)

Fig. 4.1

A small square coil of N turns has sides of length d and is mounted so that it can pivot
freely about a horizontal axis XY through its centre, as shown in Fig. 4.1. the coil is
positioned between the poles of a magnet which produces a uniform magnetic field of
flux density B. The coil is maintained in a vertical plane by moving a rider of mass M,
M
along a horizontal beam of mass per unit length d. When a current I flows through
8
the coil, equilibrium is restored by placing the rider a distance W along the beam from
the coil.
(i) Show that B is given by the expression

d
Mg w
B 16
16NId 2

[3]

(ii) M
Discuss the effect(s) if the horizontal beam of mass per unit length d is
8
replaced by a massless one.

..

..

..

.. [2]

River Valley High School Pg 8 of 19 Year 6 H1 Physics 8866


Preliminary Examination 2013
5 In a photoelectric effect experiment, the maximum kinetic energy Ek,max of the ejected
photoelectrons is measured for various wavelengths, of the incident light.

Fig. 5.1 shows the variation of Ek,max as a function of .

Ek,max / eV

20.0

15.0

10.0

5.0

0 / nm
50 100 150 200 250

Fig. 5.1

(a) Briefly describe how the maximum kinetic energy of the ejected electrons may be measured
experimentally.

..

..

..

. [2]

(b) Explain what is meant by the work function of a metal.

..

..

. [1]

River Valley High School Pg 9 of 19 Year 6 H1 Physics 8866


Preliminary Examination 2013
(c) Using Fig. 5.1, calculate the threshold frequency and the work function of the metal used.

threshold frequency = .. Hz

work function = . eV [3]

(d) Data from experiments like this are often graphed showing maximum kinetic energy as a
function of 1/ instead of that in Fig. 5.1.

Sketch a clearly labeled graph to show how Ek,max would vary with 1/.

Ek,max

0 1/

[2]

River Valley High School Pg 10 of 19 Year 6 H1 Physics 8866


Preliminary Examination 2013
Section B

Answer two out of the three questions.

6 A howitzer shown in Fig. 6.1 has a mass of 5600 kg.

barrel

rear extensible legs


Fig. 6.1

(a) A military truck is towing an unloaded howitzer at a uniform speed of 30.0 km h1. The
howitzer is connected to the rear of the truck by a horizontal tow-bar, which can sustain a
maximum force of 40.0 kN. Total resistive forces acting on the howitzer are 10.0 kN.
Calculate the shortest time the howitzer can be brought to rest safely.

time = .. s [2]

(b) The howitzer is now loaded with an ammunition round of mass 43.0 kg. Before firing, the
two rear extensible legs of the howitzer are secured to the ground. When fired, the round
has an exit velocity of 563 m s1.

(i) Explain why there is a need to secure the rear legs of the howitzer to the ground.

.. [1]

River Valley High School Pg 11 of 19 Year 6 H1 Physics 8866


Preliminary Examination 2013
(ii) The barrel of the howitzer can be elevated to a maximum of 63 above the horizontal.
Calculate the horizontal recoil speed of the howitzer if it is not secured to the ground
as it fires a round at an angle of 25 to the horizontal.

speed = m s1 [3]

(iii)

Fig. 6.2

The two rear extensible legs can be assumed to undergo only compressive force
whenever the artillery is fired. Given that the recoil force acting on each leg is
110 kN, shown in Fig. 6.2, determine the spring constant of each leg, if each leg is
compressed by 0.50 cm.

spring constant = N m1 [2]

(iv) Sketch a compression (x / m) against force (F / N) diagram to depict the scenario in


part (iii). Extrapolate your sketch to beyond the limit of proportionality. Include all
important values in your diagram.

[3]

River Valley High School Pg 12 of 19 Year 6 H1 Physics 8866


Preliminary Examination 2013
(c) When the howitzer is fired, rapid expansion of gases produced pushes the round out. The
ammunition round has the same diameter, 15.5 cm, as the interior of the barrel. The
effective length of the barrel is 3.56 m.

(i) Show that the average force acting at the rear of the ammunition round when it exits
the barrel, 0.10 s after firing, is 242 kN.

[1]

(ii) Given that the average frictional force within the barrel is 20.0 kN, calculate the
average work done in pushing the round out of the barrel.

work done = J [3]

(d) The manufacturer advised that the furthest target the howitzer can strike is 14.6 km away.

(i) Using the data given, determine the furthest target the howitzer can hit.

furthest distance = m [3]

(ii) Suggest a reason and explain why the distance calculated in (d)(i) contradicts the
value provided by the manufacturer.

[2]

River Valley High School Pg 13 of 19 Year 6 H1 Physics 8866


Preliminary Examination 2013
7 The first microwave oven was invented by Percy Spencer after World War II, based on the
fundamentals of radar technology developed during the war. Microwave ovens heat foods that
have high water content, quickly. If the presence of water is negligible in the microwaved item, the
item rarely gets hot. The operating frequencies of microwave ovens are usually within the range of
915 MHz to 2.45 GHz, respectively.

Fig. 7.1

(a) State three conditions for the formation of stationary waves.

...

[3]

(b) During operation, stationary waves are formed within the chamber. Given that, for a
particular brand of microwave oven, the frequency generated is 2.00 GHz and the
cooking chamber is 45 cm wide.

(i) Determine the number of antinodes under the given conditions, assuming
displacement nodes are found at both ends of the cooking chamber.

number of antinodes = .
[2]

River Valley High School Pg 14 of 19 Year 6 H1 Physics 8866


Preliminary Examination 2013
(ii) Sketch the stationary waves in the given diagram below. Label the positions of
displacement nodes and antinodes.

[2]

(iii) In order to investigate the wavelength of microwaves, using laboratory equipment and
apparatus, suggest an experimental procedure to determine the wavelength, assuming
you do not know the frequency of the electromagnetic waves. Include relevant diagram
of the setup.

[3]

River Valley High School Pg 15 of 19 Year 6 H1 Physics 8866


Preliminary Examination 2013
(c) (i) When food placed on a porcelain plate is heated in the microwave oven, the food
becomes very hot, while the plate remains relatively cooler. Discuss how a microwave
oven cooks the food that is placed in the cooking chamber.

[2]

(ii) Suggest and explain the highest temperature the food could reach by this cooking
method of microwaving.

[2]

(d) A microwave source emits progressive transverse waves which pass through a diffraction
grating.

(i) Explain what is meant by a progressive transverse wave.

[3]

(ii) A certain order of diffraction for 2.00 GHz is superimposed on 2.45 GHz microwaves of
the next order, where the angle of diffraction is 67. These diffractions are also the
largest order of diffraction for the respective frequencies.

Determine the number of lines per metre in the grating. Leave your answer to 2
significant figures.

number of lines per metre = m1 [3]

River Valley High School Pg 16 of 19 Year 6 H1 Physics 8866


Preliminary Examination 2013
8

Fig. 8.1

Fig. 8.1 shows a simplified diagram of the 5 lowest energy levels of the outermost electron in the
sodium atom.

(a) (i) Considering transitions between only these levels, determine the number of spectral
emission lines that might be produced by transitions among these levels.

[1]

(ii) State the transition which produces the second longest wavelength. Give your answer
in terms of level numbers. Name the region of the electromagnetic spectrum which it
belong to?

transition = ..

region = .. [3]

(b) (i) State the number of absorption lines which might be detected.

[1]

(ii) Comment on the answers you have given in (a)(i) and (b)(i).

... [2]

River Valley High School Pg 17 of 19 Year 6 H1 Physics 8866


Preliminary Examination 2013
(c) Cool sodium vapour at low pressure is bombarded with electrons of kinetic energy E.

State and explain which transitions you would expect to observe if E has the value

(i) 3.1 x 1019 J,

(ii) 4.3 x 1019 J,

(iii) 5.7 x 1019 J.

[6]

(d) In fact, level 2 consists of a pair of closely spaced levels. Transitions from them to another
level give rise to the sodium D-lines of wavelengths 589.0 nm and 589.6 nm. Calculate the
energy difference between the two levels of the closely spaced pair and identify the other
level involved.

energy difference = .. J

level = .. [2]

River Valley High School Pg 18 of 19 Year 6 H1 Physics 8866


Preliminary Examination 2013
(e) These line spectra correspond to specific photons of certain wavelength.

(i) Explain what is meant by a photon.

.............. [3]

(ii) Suggest two experiments that provide the evidence for the particulate and wave nature
of light.

.............. [2]

End of paper

River Valley High School Pg 19 of 19 Year 6 H1 Physics 8866


Preliminary Examination 2013
1 (a) Random errors are seen as a scatter of readings about a mean value. B1

Systematic errors are when all measurements are either too high (or too low) by a
certain amount compared to the actual value. B1

(b) 4.99 7.03 cos 2 33


Tmin 8.26219 8.18715 0.07504
cos 2 39 cos 2 39
5.01 7.07 cos 2 37
T max 9.36662 8.43066 0.93596
cos 2 43 cos 2 43
T Tmax Tmin 0.43046 0.4
Absolute uncertainty to be left as 1 s.f.

(c) 2
sX 1 sX
sY u sin g
u cos 2 u cos
2
m m

m m s 1

m s 2
1

1
M1
m s m s
m m2

m m s 1
m s 2
1
2 2 M1
m s m s
A0
m m m (shown)

2 (a) (i) Zero. The force between the two atoms is negligible between this apart. B1

(ii) Negative implies that the two di-atomic molecules experience attractive B1
force.

(iii) Positive implies that the force is repulsive. This could be due to the fact B1
that the positively charged protons in the nucleus of the atoms now get
very close to each other.
q1q 2 B1
Using the relation, F , the force increases rapidly.
4 0 x 2

(b) (i) The neutral atoms once stripped of their outermost electrons become
positively charged.
These positively charged particles will be attracted towards the negatively B1
charged plate/electrode.
Describing a parabolic path. B1

(ii) As the direction and the entire stream of positive particles behave like the B1
electric current direction,
the presence of the uniform magnetic field will exert an induced magnetic B1
force perpendicular to the current and the magnetic field.

3 (a) the energy transferred per unit charge from some form into electrical energy when B1
charge is moved round a complete circuit

(b) (i) Effective resistance of bulbs = (1/10 + 1/5)1 = 3 1/3 ohms


Current flowing through 100 ohms = 9/(100 + 3 1/3) = 0.0871 A M1
p.d. drop across 100 ohms = 8.7097 V (p.d. across the two bulbs = 0.2903
V) p.d. across A and B = 8.7097 + (0.2903/2) = 8.85 V A1

River Valley High School Pg 1 of 7 Year 6 H1 Physics 8866


Preliminary Examinations 2013
p.d. across A and C is simply 8.71 V A1

(ii) Let unknown resistance to be Z


(Z/(Z + 3 1/3)) x 9 = p.d. across AC M1
p.d. across any of the two bulbs in parallel = (p.d. across AC / 2)
therefore, highest power dissipation has to come from the single bulb =
V2/R = 0.538 W V = 1.64 V M1
(p.d. across AC p.d. across unknown Z = 7.36 V)
9Z = (Z + 3 1/3)7.36 15.0 ohms A1

(iii) With non-zero internal resistance for the e.m.f., p.d. across AC will be B1
smaller.
As a result of the smaller p.d., power dissipation of all bulbs will be B1
reduced.

4 (a) F = BILsin M1

M1

M1

(b) (i) 2(B)(I)(d)(d/2) + (2Mg/8)(d) M1


= (Mg)(W) + (3Mg/8)(1.5d) M1

d
Mg w
Successfully show that B
16 M1
2
16NId

(ii) If the beam is massless, from the equation B1


2(B)(I)(d)(d/2) + (2Mg/8)(d)
= (Mg)(W) + (3Mg/8)(1.5d)
the rider would have to be positioned further from the horizontal axis XY B1

River Valley High School Pg 2 of 7 Year 6 H1 Physics 8866


Preliminary Examinations 2013
5 (a) B1
apply negative potential at the collector, slowly increase negative value until
photo-current drops to zero. This potential is the stopping potential Vs.

Use Ek, max = e Vs B1

(b) min energy to minimum energy required to eject an electron from the surface of a B1
metal.

(c)
(at least two sets of values and find average)

hc / = Ek, max + M1

.. = 4.86 eV or 5.29 eV or 4.93 eV.

average 5.0 eV A1

threshold freq = / h 1.2 x 1015 Hz A1

(d) Ek, max

1/
1/0

[1 mk for correct straight line; 1 mk for correct label]

6 (a) v = 8.33 m s1
Fnet = Fmax = F friction
56008.33 M1
Fmax = 40.0 kN 10.0 kN
t t
t = 0.933 s
A1

(b) (i) To minimize the distance the howitzer moves backwards, as a result of B1
the recoil.

(ii) By conservation of momentum,


mhowitzervhowitzer + mroundvround = 0 C1
vround,horizontal = 563cos25 = 510.25 m s1 M1
(5600)(vhowitzer) = (43)(510.25)
vhowitzer = 3.92 m s1 (backwards) A1

River Valley High School Pg 3 of 7 Year 6 H1 Physics 8866


Preliminary Examinations 2013
(iii) 220 kN = 2F = 2kx = 2(k)(0.00500) M1
k = 22 x 106 N m1 A1

(iv)

(c) (i) 43563 M1


Favg = 242 kN
t 0.1

(ii) Work done by gas = Favg x 3.56 = 861520 J M1


Work done against friction = 20.0 kN x 3.56 = 71200 J M1
Average work done on pushing round out = 790 kJ A1

(d) (i) Max vertical velocity = 563sin45 = 398.1 m s1 M1


Time to reach max height, v = u + at 40.58 s
Round-trip time = 81.16 s M1
s = ut s = (563cos45)(81.16) = 32.3 km A1

(ii) The distance calculated does not consider air resistance, which if included B1
in the calculations, would require the consideration of a reducing
horizontal velocity as well as vertical velocity. B1

7 (a) two progressive waves of equal amplitude B1


equal frequency and speed B1
travelling from opposite directions and meet B1

(b) (i) v = f

3 x 108 = (2 x 109) = 0.15 m


M1
when standing waves generated, distance between two adjacent
nodes/antinodes equal to half a wavelength.

6 antinodes A1

River Valley High School Pg 4 of 7 Year 6 H1 Physics 8866


Preliminary Examinations 2013
(ii) B1

N N
A A

B1

(iii)

B1
relevant diagram given

microwave generator produces mcirowave through an emitter. These


waves are reflected back by the reflector and the two waves superpose to B1
give a stationary wave.

a detector is placed between the emitter and the reflector to detect the
nodes and antinodes of the stationary wave. detector is connected to a
cathode-ray oscilloscope (CRO) with the time-base turned off.

By moving the microphone slowly backward and forward, a vertical trace


can be seen on the screen of the CRO which varies from a minimum
length to a maximum length corresponding to the nodes and antinodes.

Measuring the length between successive nodes or antinodes of the


stationary wave will give /2. From this measurement, the wavelength of B1
the sound waves can be obtained.

(c) (i) food contains high water content.


energy of microwave transfers to water molecules in food B1
water molecules vibrate more energetically as a result of energy transfer
from the microwave, transferring energy to the rest of the food as thermal B1
energy

(ii) food contains water and boiling point of water is 100C B1


so highest temperature reached would be 100C B1

River Valley High School Pg 5 of 7 Year 6 H1 Physics 8866


Preliminary Examinations 2013
(d) (i) A progressive transverse wave is a wave in which the energy transfer is in B1
the direction of wave motion while displacements of the particles in the B1
wave are at right angles to the direction of transfer of the energy of the B1
wave.

(ii) dsin=n

2.00 GHz 0.15 m wavelength


2.45 GHz 0.122 m wavelength

dsin=(n)(0.15) M1
dsin=(n+1)(0.122)

0.15n = 0.122n + 0.122

n = 4.35 M1

largest order of diffraction, assume d(sin90) = 4.35(0.15)

average number of lines per metre = 1/d = 1.53 A1

8 (a) (i)

10 emission transitions B1

(ii) Longest wavelength lowest energy photon C1


2nd longest wavelength 2nd lowest energy photon

5 4, 0.21 x 1019 J
4 3, 0.69 x 1019 J A1

1.45 x 106 m microwave (accept radio wave) B1

River Valley High School Pg 6 of 7 Year 6 H1 Physics 8866


Preliminary Examinations 2013
(b) (i)

4 absorption transitions B1

(ii) Emission spectrum, transition can take place from any state of higher B1
energy level to a state of lower energy level. Each transition will release a
quanta of energy.
However, absorption spectrum, transition can only take place from the
ground state to a state of higher energy level, as the lifetimes of excited B1
atoms at the higher energy levels are too short for an incoming photon of
matching energy to interact with these atoms to further excite them to a
state of even higher energy.

(c) (i) Cool sodium vapour most atoms at ground state B0


If E = 3.1 x 1019, it does not match the minimum absorption transition of
3.38 x 1019 J. No absorption transition observed. B1

(ii) 4.3 x 1019 J is sufficient to bring about absorption from ground state to B1
level 2, a dark line would be observed at wavelength 296 nm, ultraviolet B1
range.

(iii) 5.7 x 1019 J is sufficient to bring about absorption from either ground state B1
to level 2 or ground state to level 3, a dark line would be observed at B1
either wavelength 296 nm, ultraviolet range, or 196 nm, deep ultraviolet B1
range.

(d) c
To produce 589.0 nm E h 1.701 10 19 J
589 nm
c
To produce 589.6 nm E h 1.699 10 19 J
589.6 nm
A1
The energy difference is 0.002 x 1019 J.
A1
Only transition between level 2 and 3 give an energy difference close to the
above calculated values.

(e) (i) A photon is a quanta of energy of electromagnetic radiation, defined by , B1


defined by E = hf B1
B1

(ii) Particulate nature: Photoelectric effect experiment B1


Wave nature: Youngs double slit experiment B1

River Valley High School Pg 7 of 7 Year 6 H1 Physics 8866


Preliminary Examinations 2013
RIVER VALLEY HIGH SCHOOL
YEAR 6 PRELIMINARY EXAMINATION

H2 PHYSICS 9646
PAPER 1
27 SEP 2013
1 HOUR 15 MIN

CANDIDATE
NAME

CENTRE INDEX
NUMBER S NUMBER

CLASS 6
INSTRUCTIONS TO CANDIDATES

DO NOT OPEN THIS BOOKLET UNTIL YOU ARE TOLD TO DO SO.

Read these notes carefully.


Write your name, class and index number in the spaces above.

There are forty questions in this paper. Answer all questions. For each question, there are four
possible answers, A, B, C and D.
Choose the one you consider correct and record your choice in soft pencil on the separate
Answer Sheet.

Read the instructions on the Answer Sheet very carefully.

Each correct answer will score one mark. A mark will not be deducted for a wrong answer. Any
rough working should be done on the Question Paper.

The total number of marks for this paper is 40.

____________________________________________________________________________
This Question Paper consists of 20 printed pages.

River Valley High School Pg 1 of 20 Year 6 H2 Physics 9646


Preliminary Examination 2013
Data

speed of light in free space, c = 3.00 108 m s1

permeability of free space, O = 4 107 H m1

permittivity of free space, O = 8.85 1012 F m1

(1/(36 )) 109 F m1

elementary charge, e = 1.60 1019 C

the Planck constant, h = 6.63 1034 J s

unified atomic mass constant, u = 1.66 1027 kg

rest mass of electron, me = 9.11 1031 kg

rest mass of proton, mp = 1.67 1027 kg

molar gas constant, R = 8.31 J K1 mol1

the Avogadro constant, NA = 6.02 1023 mol1

the Boltzmann constant, k = 1.38 1023 J K1

gravitational constant, G = 6.67 1011 N m2 kg2

acceleration of free fall, g = 9.81 m s2

River Valley High School Pg 2 of 20 Year 6 H2 Physics 9646


Preliminary Examination 2013
Formulae

1 2
uniformly accelerated motion, s ut at
2
v 2 u 2 2as

work done on/by a gas, W pV

hydrostatic pressure, p gh

GM
gravitational potential,
r

displacement of particle in s.h.m., x xO sin t

velocity of particle in s.h.m., v vO cos t

x 2
O x2
3
mean kinetic energy of a molecule of an ideal gas, E kT
2

resistors in series, R R1 R2

resistors in parallel, 1/ R 1/ R1 1/ R2

Q
electric potential, V
4 O r

alternating current/voltage, x xO sin t

transmission coefficient, T exp 2kd

8 2 m U E
where k =
h2

radioactive decay, x xO exp t

0.693

decay constant, t1
2

River Valley High School Pg 3 of 20 Year 6 H2 Physics 9646


Preliminary Examination 2013
For each question, there are four possible answers, A, B, C and D. Choose the one you
consider to be correct.

1 The following physical quantities can be either positive or negative.

s: displacement of a particle along a straight line


: temperature on a Celsius scale
q: electric charge
V: readings on a digital voltmeter

Which of these quantities are vectors?

A s only
B , V
C s, q, V
D s, , q, V

2 When a beam of light is incident on a surface, it delivers energy to the surface.

What is the unit of intensity, expressed in SI base units?

A kg s2 B kg s3 C kg m2 s1 D kg m2 s3

3 A small stone is thrown horizontally towards a vertical wall 1.20 m away. It hits the wall
0.80 m below its initial level. Neglecting air resistance, at what speed does the stone hit
the wall?

stone
0.80 m

1.20 m wall

A 3.0 m s1
B 4.0 m s1
C 5.0 m s1
D 6.0 m s1

River Valley High School Pg 4 of 20 Year 6 H2 Physics 9646


Preliminary Examination 2013
4 A small object is projected at an angle of 45 to the horizontal with an initial kinetic
energy E. Neglecting air resistance, what is its kinetic energy when it is halfway up?

A 0
B E/2
C 3E / 4

D E/ 2

5 A resupply aircraft is flying at 360 km h1, at an angle of 15 below the horizontal. The
cargo door opens and a supply package drops out. Assuming negligible air resistance,
how much time passes before the packages speed doubles?

A 7.9 s B 10.4 s C 15.2 s D 54.8 s

6 A body of mass 1.0 kg is moving at 10 m s1. A force now acts in its direction of motion,
varying with time as shown below.

What is most likely the momentum of the body after 5.0 s?

A 15.0 kg m s1 B 20.5 kg m s1 C 22.5 kg m s1 D 25.0 kg m s1

River Valley High School Pg 5 of 20 Year 6 H2 Physics 9646


Preliminary Examination 2013
7 A rod, made up of 2 uniform portions, A and B, is held in place by a cable under tension,
T. The cable is at an angle of 30 to the horizontal. Given that portions A and B have
masses of 20.0 kg and 30.0 kg respectively, and are of identical length, L, what is the
tension T in the cable?

A 150 N B 190 N C 230 N D 270 N

8 A block, resting on the rough surface of a ramp, is connected to a ball, resting on the
frictionless surface of the same ramp, using a taut but inextensible string. The ball has a
mass of 10.0 kg. The frictional force acting on the block is 30 N. If the ball accelerates
uniformly at 1.0 m s2 down the slope, what is the mass of the block?

A 3.8 kg B 5.0 kg C 6.7 kg D 7.4 kg

9 An object of weight 50 N has an initial velocity of 10 m s1 projected up an inclined slope


of 30 and comes to rest at a vertical height of 4 m.

What is the average frictional force acting on the object?


A 6.9 N
B 14 N
C 27 N
D 55 N

River Valley High School Pg 6 of 20 Year 6 H2 Physics 9646


Preliminary Examination 2013
10 A small metal sphere of mass m is moving through a viscous liquid. When it reaches a
constant downward velocity v, which of the following describes the changes with time, t in
the kinetic energy and gravitational potential energy of the sphere?

kinetic energy gravitational potential energy


A constant and equal to mv2 decreases at a rate of mgv
B constant and equal to mv2 decreases at a rate of (mgv mv2/t)
C increases at a rate of mgv decreases at a rate of mgv
D increases at a rate of mgv decreases at a rate of ( mv2/t mgv)

11 A mass on the end of a string is set in motion so that it describes a circle in a horizontal
plane at a constant speed. The angle made by the string to the vertical is 40.

What is the ratio of the resultant force to the weight of the mass?

40

A 0.012
B 0.643
C 0.766
D 0.839

River Valley High School Pg 7 of 20 Year 6 H2 Physics 9646


Preliminary Examination 2013
12 Three identical masses are tied together using strings and made to rotate around a pin
on a smooth horizontal table as shown in the figure. The three masses remain in a
straight line as they rotate.

What is the ratio of the tension in string 1 : string 2 : string 3?

pin
string 1
r
plan view of setup
r string 2
r
string 3

A 1:2:3
B 3:2:1
C 3:5:6
D 6:5:3

13 Two satellites A and B of the same mass are moving in circular orbits round the Earth.
The radius of As orbit is r and that of Bs orbit is 2r respectively with their total energy EA
and EB respectively. Which of the following descriptions about EA and EB is correct?

A EA > 0 and EB = EA

B EA > 0 and EB = 2EA

C EA < 0 and EB = EA

D EA < 0 and EB = 2EA

14 Which statement about a geostationary orbit is false?

A A geostationary orbit must be directly above the equator.


B There is only one possible radius for a geostationary orbit.
C All satellites in a geostationary orbit must have the same speed.
D All satellites in a geostationary orbit must have the same kinetic energy.

River Valley High School Pg 8 of 20 Year 6 H2 Physics 9646


Preliminary Examination 2013
15 A camera was used to photograph the bob of a simple pendulum moving in front of a
horizontal scale. The extreme positions of the bob were at the 500 mm and 600 mm
marks. The photograph showed that while the shutter was open, the bob moved from the
550 mm mark to the 575 mm mark.

If the period of the pendulum was 2.0 s, how long did the shutter remain open?

A 0.17 s B 0.25 s C 0.33 s D 0.50 s

16 A particle of mass 150 g moves with simple harmonic motion with an amplitude of
2.5 cm. At a displacement of 1.0 cm from the equilibrium point, its kinetic energy is
0.62 J.

What is its natural frequency of oscillation?

A 18 Hz B 20 Hz C 22 Hz D 24 Hz

17 An insulated cylinder is fitted with a perfectly fitting piston and a heater. There is
negligible friction between the cylinder and the piston. The cylinder contains a fixed mass
of helium gas. Assume the gas is ideal.

The following two experiments are performed.

Experiment 1: The heater provides 150 J of energy with the piston fixed in position. The
temperature rise of the gas is found to be 29 K.

Experiment 2: The heater provides 150 J of energy with the piston free to move so that
the gas expands at constant pressure. The temperature rise of the gas is now 18 K.

Given U is the change in internal energy, W is the work done on the system and Q is
the heating of the system.

Which of the following is correct?

Experiment U / J W/J Q/J


A 1 , more than decrease in experiment 2 0

B 2 + + +

C 1 +, less than increase in experiment 2 0 +

D 2 + +
River Valley High School Pg 9 of 20 Year 6 H2 Physics 9646
Preliminary Examination 2013
18 The graph shows the p-V diagram for one cycle of a pump used for aerating water in an
aquarium.

What is net work done on the air in the pump?


A 0J
B 0.10 J
C 0.28 J
D 0.38 J

River Valley High School Pg 10 of 20 Year 6 H2 Physics 9646


Preliminary Examination 2013
19 A beam of unpolarised light with amplitude A and intensity I is passed through two optical
polarisers. The first polarisers transmission axis is oriented at 60 to the vertical, while
the second polarisers transmission axis is oriented at 45 to the horizontal.

What is the intensity of the light at P and amplitude of the light at Q?

Intensity of light at P Amplitude of light at Q

A 1 1
I A cos15
4 2

B 1 1
I A sin15
4 2

C 1 1
I A cos15
2 2

D 1 1
I A sin15
2 2

20 A point source of sound emits energy equally in all directions at a constant rate and a
person 9.0 m from the source listens. After a while, the intensity of the source is halved.
If the person wishes the sound to seem as loud as before, how far should he be now
from the source?

A 3.4 m
B 4.5 m
C 5.5 m
D 6.4 m

River Valley High School Pg 11 of 20 Year 6 H2 Physics 9646


Preliminary Examination 2013
21 Monochromatic light is incident normally on a diffraction grating and the second order
diffraction is observed at an angle of 67.0 from the central maximum.

Which of the following statements about the diffraction pattern produced by the grating is
true?

A The first order diffraction is observed at 27.4.


B The first order diffraction is observed at 33.5.
C The diffraction pattern has 4 intensity maxima formed.
D The third order diffraction is observable but of low intensity.

22 Plane waves of wavelength in a ripple tank travel towards a straight barrier parallel to
the wave fronts. There are 2 gaps of identical width, spaced d apart. Which of the
following and d will produce the narrowest-spaced interference patterns at a screen
distance D away?

/ cm d / cm

A 1.3 2.3

B 1.3 5.2

C 2.6 5.2

D 2.6 2.6

23 An electron travelling at constant speed enters a uniform electric field at right angles to
the field. While the electron is in the field it accelerates in a direction which is
A in the same direction as the electric field.
B in the opposite direction to the electric field.
C in the same direction as the motion of the electron.
D in the opposite direction to the motion of the electron.

24 A uniform electric field of strength 10 V m1 acts downwards as shown below.


P Q

A charge of 4 C is moved from P to Q and then from Q to R. If the distance PQ is 2 m


and QR is 3 m, what is the change in potential energy of the charge when it is moved
from P to R?

A 40 J B 50 J C 120 J D 200 J
River Valley High School Pg 12 of 20 Year 6 H2 Physics 9646
Preliminary Examination 2013
25 The graph shows the current-voltage (I-V) characteristic of an electrical component.
I
I

I1

I0

0
0 V0 V1 V

What is the resistance of the component at potential difference V1 and how does the
resistance change, if at all, when the potential difference increases from V0 to V1?

resistance at V1 resistance change from V0 to V1


V1 V0 no change
A
II11 - II00

V1 V0 decreases
B
I1I - II0
1 0

V1 no change
C
II1
1

V1 decreases
D
II1
1

River Valley High School Pg 13 of 20 Year 6 H2 Physics 9646


Preliminary Examination 2013
26 A battery of e.m.f E and internal resistance r delivers a current I through a resistance R.

r
I
A

R is set at two different values and the corresponding currents I, measured using an
ammeter of negligible resistance, are shown in the table below.

R/ I/A
3.0 1.14
1.0 2.67

What is the value of the e.m.f. E ?


A 3.0 V B 3.4 V C 4.0 V D 6.1 V

27 A battery of e.m.f. 10.0 V and negligible internal resistance is connected to four resistors,
each of resistance R, and a variable resistor T, as shown below.

R
R

R
T R
10.0 V
V

RR

R
R

The resistance of T changes from 0 to 5R.

What is the change in the reading of the ideal voltmeter?

A zero B 2.14 V C 3.33 V D 3.57 V

River Valley High School Pg 14 of 20 Year 6 H2 Physics 9646


Preliminary Examination 2013
28 The diagram below shows a circuit with a uniform wire XY of length 1.0 m and resistance
1.0 connected in series with a cell of e.m.f. 3.0 V and internal resistance 0.50 . The
circuit is used to measure the e.m.f. generated by a photocell

3.0 V
0.50

X Y

0.20 V
photocell

When the e.m.f generated by the photocell is 0.20 V and the galvanometer gives a zero
reading, the position of the jockey will be

A 10 cm away from X.
B 20 cm away from X.
C 80 cm away from X.
D 90 cm away from X.

29 A rigidly-clamped straight horizontal current-carrying wire is held midway between the


poles of a magnet on a top pan balance. The wire is perpendicular to the magnetic field.

110 g

The balance, which was zeroed before the switch was closed, reads 110 g after the
switch is closed. The current is reversed and doubled.

What will be the final reading on the balance?

A 220 g B 110 g C zero D 110 g

River Valley High School Pg 15 of 20 Year 6 H2 Physics 9646


Preliminary Examination 2013
30 A particle of mass m, carrying charge q and travelling with speed v, enters a magnetic
field of flux density B at right angles, describing a circular path with radius r.

Which one of the following changes would produce an increase in v, the speed of the
particle?
A Increase in B.
B Increase in m.
C Decrease in r.
D None of the above.

31 The magnetic flux through a coil of N turns is increased uniformly from zero to a
maximum value in a time t. An e.m.f. E is induced across the coil.

What is the maximum value of the magnetic flux through the coil?
A Et/N B N/(Et) C NEt D E/(Nt)

32 A rectangular coil is rotated in a uniform magnetic field with the axle at right angles to the
field direction. When in the position shown, the angle between the direction of the
magnetic field and the normal to the plane of the coil is .

The coil has 50 turns and an area of 1.9 103 m2. The flux density of the magnetic field
is 2.8 102 T. What is the flux linkage for the coil when is 35?
A zero
B 1.5 103 Wb turn
C 2.2 103 Wb turn
D 2.7 103 Wb turn

33 A 230 V, 60 W lamp is connected to the output terminals of a transformer which has a


200 turn primary coil and a 2000 turn secondary coil. The primary coil is connected to an
a.c. source with a variable output potential difference. The lamp lights at its normal
brightness when the primary coil is supplied with an alternating current of 2.7 A.

What is the percentage efficiency of the transformer?


A 97 % B 93 % C 7% D 3%

River Valley High School Pg 16 of 20 Year 6 H2 Physics 9646


Preliminary Examination 2013
34 An ammeter uses the heating effect of a current to produce the deflection of the pointer.
The reading, which is proportional to the heating, is X when a direct current I flows
through the meter. When the ammeter is inserted in a circuit, in which an alternating
current of r.m.s. value I flows, the reading is
A X/2, because the constantly changing current produces a constantly changing
heating effect which averages to one half that of the direct current.
B X, because it measures r.m.s. current which gives the same heating effect as the
direct current.
C 2 X , because it measures the peak current which is 2 times the direct current
D X 2 , because it measures the peak current which is 2 times the direct current.

35 An electron is accelerated from rest through a potential difference of 54 V, gaining kinetic


energy of value E.
Which of the following shows the wavelength of the accelerated electron and that of a
photon of energy E?
electrons wavelength / m photons wavelength / m
A 2.3 108
400 109
B 2.3 108 2.3 108
C 1.7 1010 2.3 108
D 1.7 1010 1.7 1010

36 The diagram below shows three energy levels of a hydrogen atom, with the values of the
first and third levels given.

n=3 1.51 eV

n=1 13.6 eV

A hydrogen atom absorbs a photon of wavelength such that the electron in the ground
state is brought to an excited level of n = 3.
What is the maximum wavelength of a photon that can cause ionisation (electron
completely removed from the atom) of a hydrogen atom in the ground state?

A 8 / 9
B 9 / 8
C 3 / 2
D 2 / 3

River Valley High School Pg 17 of 20 Year 6 H2 Physics 9646


Preliminary Examination 2013
37 The diagram shows a portion of the energy levels of the helium (He) and neon (Ne)
atoms in a red He-Ne laser.

Which of the following statements is not necessarily true?

A The energy of E3 has to be close to the value of 20.61 eV.


B The energy transition from E2 to E1 emits a photon in the infra-red range.
C The energy transition from E3 to E2 emits a photon of wavelength close to
632.8 nm.
D Collision is used to stimulate the neon atoms in order to achieve population
inversion.

38 Which of the statements is correct for an n-type semiconductor?

A There are excess holes in the valence band.


B There are excess holes in the conduction band.
C There are excess electrons in the valence band.
D There are excess electrons in the conduction band.

River Valley High School Pg 18 of 20 Year 6 H2 Physics 9646


Preliminary Examination 2013
39 A radioactive sample emits both alpha and beta particles. The figure below shows the
path of the beta particle in the region of a crossed magnetic and electric field.

region with B-field acting


into paper, E-field acting
beta particle downwards.

Which of the following shows the path of the alpha particle in the same magnetic and
electric field?

alpha particle
A

B alpha particle

C alpha particle

alpha particle
D

River Valley High School Pg 19 of 20 Year 6 H2 Physics 9646


Preliminary Examination 2013
40 The graph shows how the logarithm of activity A varies with time t of a radioactive
isotope.

ln (A / Bq)

5.0

400 t / hours

What is the half-life in hours and initial activity at time t = 0 s of the isotope?

Half-life / h Initial activity / Bq


A 55 150
B 55 5.0
C 200 2.5
D 200 5.0

END OF PAPER

River Valley High School Pg 20 of 20 Year 6 H2 Physics 9646


Preliminary Examination 2013
EOY 2013

Y6 Physics

1 Only displacement is a vector quantity.

Ans: A

2 [Intensity] = -

Ans: B

3 vertically: v2 = u2 + 2 as
= 0 2(9.81)(-0.8)
Vy = 3.9618

Vertically : S = ut + a t2
t 0.40 8 55
Horizontally, s = u t
. ux = 2.97 = vx

Therefore v = sqrt(vx2+vy2) = 4.9522 = 5.0

Ans: C

Distracter: option A : consider only vx = 2.97 = 3.0


Distracter: option B : consider only vy = 3.96 = 4.0
Distracter: option D : no reasoning, just follow natural sequence of the other
options.

4 Given Ek = m u2 = E at bottom;
assume Ep = 0 at bottom; hence total energy = E

At top; Ep = mgH;
Ek = m (ucos45)2 = ( mu2 ) = E
Since total energy = E; therefore Ep at top = E

At halfway; Ep = mg (H/2) = E
Ek = E E = E

Ans: C

Distracter: option A : to sieve out the dice thrower


Distracter: option B : obvious distractor; halfway
Distracter: option D : cos 45 = 1/ 2

5 360 km h-1 100 m s-1


Initial horizontal vel. = final horizontal vel. = 100 cos15 = 96.59 m s-1
Initial vertical vel. = 25.89 m s-1
Final vel. = 2 x 100 m s-1
Final vertical vel. = (2002 96.592)1/2 = 175.13 m s-1
Using vy = uy + ayt 175.13 = 25.89 + 9.81(t) t = 15.2 s

Ans: C

River Valley High School Pg 1 of 7 Year 6 H2 Physics 9646


P 2013
6 mv 5 2 5 3
Favg 1v f 10 v f 22.5
t 2 2
Final momentum = 22.5 kg m s-1
However since 2.0 5.0 s is non-linear decrease of force, final momentum will be
less than 22.5 ks m s-1, but greater than 15.0 ks m s-1, which is given by when t is
until 2.0s.

Ans: B

2L sin60T cos 30 L cos 60 209.81


7
2
3L
cos 60 309.81 2L cos 60T sin30
2
2 sin60T cos 30 2 cos 60T sin30 269.775
269.775
T 270 N
1
Ans: D

8 Considering the block:


T m9.81sin30 30 m1
Considering the ball:
Fnet 101 109.81cos 45 T
m = 4.9732 kg

Ans: B

9 Total Initial Energy = KE = 254.8J


Total Final Energy = GPE = 200J
Change in Energy = Loss to Friction
54.8 = F x d
d = 4/sin30
F = 6.855N = 6.9 N

Ans: A

10 Sphere reached terminal velocity = constant Velocity = constant KE.


GPE decreases at mgh/t = mgv.

Ans: A

11 Vertical Forces:
T co W
Horizontal Forces:
T i n Fc = Resultant Force
Re u ltant Force/ W tan tan 40 0.8 9

Ans: D

12 T3 2(3r)
T2 2( r) + 2(3r)
T1 2(r) + 2( r) + 2(3r)

River Valley High School Pg 2 of 7 Year 6 H2 Physics 9646


P 2013
Ratio = 6:5:3

Ans: D

13 Object in Orbits have negative total energy.


Total Energy (GPE and KE) = - GMm/2r
EB = - GMm/2(2r)
EA = - GMm/2(r)

Ans: C

14 Kepler Law :
T2 varies r3
And eo t ationary orbit ean pe riod of 4 hour . So r i fixed.
r is fixed and w is fixed so velocity is fixed.
But KE depends on the mass of the object so it is not fixed.

Ans: D

15 5 50 i nt
t/T /6
t = 1/6 s

Ans: A

16 The velocity of the oscillating particle, v = (xo2 x2)


Its k.e., Ek = m v2 = m 2 (xo2 x2)
i.e. 0.62 = m 2 (xo2 x2)
= (150 x 10-3)(2f)2(0.0252 0.0102)
Thus, f = 20 Hz

Ans: B

17 Options B, C and D are true for Q = +ve, since heater supplies heat to both
experiments 1 and 2

Options C and D are true for W = 0 J and W = -p(V) = -ve, since there is no change
in volume for experiment 1, while in experiment 2, the change in volume is = +ve.

Option C is deemed incorrect for U +, le than increa e in experi ent , i nce


the change in internal energy is dependent on the change in temperature which, in
experiment 1, should be of a greater increase than in experiment 2

Ans: D

18 Net work done on the air in the pump = area enclosed by the curve
1 big square = (0.1 105)(1.0 106) = 0.01 J
roughly 10 big squares = 0.10 J

Ans: B

19 After passing 60,


Pamplitude = A Pintensity = kA2 = I

After passing 45,

River Valley High School Pg 3 of 7 Year 6 H2 Physics 9646


P 2013
Qamplitude = A sin15

Ans: B

20 Intensity = Power/Area
P
Ia
4 9 2
1
P
2
Ia
1
4r
2

2
81
r 6.36 m
2

Ans: D

21 d i n n d i n67 /d 0.460
-1
therefore 1st = sin ((1)(0.460)) = 27.4
for n , i n > 1 error

Ans: A

22 x = (1.3)(D)/(5.2) = 0.25D

Ans: B

23 acceleration occurs in the direction of the resultant force which would be the
direction of the electric force on the electron.

Ans: B

24 No change in potential when charge is moved from P to Q, hence no work done.


When charge is moved from Q to R,
work done = q V = q E d (since E = V/d) = 4 10 3 = 120 J

Ans: C

25 Ratio is the ratio of the pd and current.


The ratio of this Pd and Current changes from V0 to V1.

Ans:D

26 E = I(R) + I(r)
E = 1.14 (3) + 1.14 (r)
E = 2.67 (1) + 2.67 (r)
Si ultaneou Que t ion.

E = 3.98 V

River Valley High School Pg 4 of 7 Year 6 H2 Physics 9646


P 2013
Ans: C

27 When T = 0, V = 0 V
Same potential at the 2 points of the circuit.

When T = 5R, difference in V = 5 - 10/7 V


Pd = 3.57 V

Ans: D

28 Pd across XY = 1/1.5 x 3 = 2 V
Pd across X and balance pt = 0.2 V
0.2/2 x 100 cm = 10 cm

Ans: A

29 Newton rd Law states that the upwards magnetic force on the wire would be equal
and opposite to the downwards magnetic force on the magnet on the top pan
balance (equivalent to a weight of 110 g).
Hence when current is reversed, and doubled (FB = BIL), the reading on the balance
would be 220 g.

Ans: A

30 The magnetic force on a moving charge in a B-field is perpendicular to its velocity


and hence could not change its magnitude, only its direction. Option A and B would
only affect the radius of curvature but not velocity. Option C cannot be controlled
directly as it is dependent on the other factors.

Ans: D

31 induced E = rate of change of magnetic flux linkage


E = N/t
= Et/N

Ans: A

32 N = 50 (2.8 102) (1.9 103) cos 35 = 2.2 103 Wb turn

Ans: C

33 However, required primary V = 23 V (given turns ratio = 2000/200 = 10)


NS VS

NP VP
outputpower 60
Hence, efficiency = 0.97
VP I P 23 2.7
Ans: A

34 The power delivered to a resistor by the rms current of an ac supply is equivalent to


the power delivered by the dc current of a dc supply.

Ans: B

35 Electron: qV = 8.64 10-18; Ek = p2/ 2m = (h/)2 / 2m

River Valley High School Pg 5 of 7 Year 6 H2 Physics 9646


P 2013
Hence 1.7 1010

Photon: E = hc/ . . 108

Ans: C

Distracter: electron uses E = hc/ as well.


Photon uses same value as electron.

36 E = hc /
(13.6 1.51) = hc / .. (1)

For ionization, need at least 13.6 eV


13.6 eV = hc / 2 .. ( )

Eqn 1/ 2 : 12.09 / 13.6 = 2 /

2 = 0.88897
Hence for ionization , wavelength needed is 0.88897 or less.

Hence option B and C are out.

8 / 9 = 0.88888
2 / 3 = 0.66666

Both A and D can cause ionization, but question asked for the maximum.

Ans: A

37 Option A is true, He atom collides Ne atom and transfer same amount of energy
Option C is true. (20.61 18.7) x 1.6 x 10-19 h (c/) ~ 650 n
Option D is true. Given in context of question.
E2 E1, 18.7 eV
If E 1.91 eV e i t red , to e i t infra-red , E < 1.91 eV

Ans: B

38

n type e ico nductor with donor ato ener y level

Ans: D

39
alpha travels much slower than beta, hence FB = Bqv smaller than FE = qE for the
same E and B as for the beta particle.
Ans: C

River Valley High School Pg 6 of 7 Year 6 H2 Physics 9646


P 2013
40 A = Ao e - t
ln A = ln Ao - t

gradient = -
hence . 0.01 5
t1/2 = ln 2 / = 55 h

ln Ao 5 .. Ao .148

Ans: A

Distracter A: half life : half of 400; A0 = initial value on graph = 5


Distracter B: half of value on graph
Distracter C: A0 = initial value on graph = 5

River Valley High School Pg 7 of 7 Year 6 H2 Physics 9646


P 2013
RIVER VALLEY HIGH SCHOOL
YEAR 6 PRELIMINARY EXAMINATION

H2 PHYSICS 9646
PAPER 2 (SECTION A)
19 SEP 2013
1 HOUR 45 MIN
CANDIDATE
NAME

CENTRE INDEX
NUMBER S NUMBER

CLASS 6
INSTRUCTIONS TO CANDIDATES
DO NOT OPEN THIS BOOKLET UNTIL YOU ARE TOLD TO DO SO.
Read these notes carefully.
Write your name, centre and index number and class in the spaces above.

Section A and Section B FOR EXAMINERS USE


Candidates answer on the Question Paper. Section A
Write in dark blue or black pen. 1 / 8
You may use a soft pencil for any diagrams, graphs or rough
working. 2 / 9
Do not use paper clips, highlighters, glue or correction fluid.
3 / 10
Answer all questions.
4 / 9
5 / 5
6 / 19
Deduction

TOTAL / 60

The number of marks is given in brackets [ ] at the end of each question or part question.
____________________________________________________________________________
This paper consists of 21 printed pages.

River Valley High School Page 1 of 21 Year 6 H2 Physics 9646


Preliminary Examination 2013
Data

speed of light in free space, c = 3.00 108 m s1

permeability of free space, O = 4 107 H m1

permittivity of free space, O = 8.85 1012 F m1

(1/(36 )) 109 F m1

elementary charge, e = 1.60 1019 C

the Planck constant, h = 6.63 1034 J s

unified atomic mass constant, u = 1.66 1027 kg

rest mass of electron, me = 9.11 1031 kg

rest mass of proton, mp = 1.67 1027 kg

molar gas constant, R = 8.31 J K1 mol1

the Avogadro constant, NA = 6.02 1023 mol1

the Boltzmann constant, k = 1.38 1023 J K1

gravitational constant, G = 6.67 1011 N m2 kg2

acceleration of free fall, g = 9.81 m s2

River Valley High School Page 2 of 21 Year 6 H2 Physics 9646


Preliminary Examination 2013
Formulae

1 2
uniformly accelerated motion, s ut at
2
v 2 u 2 2as

work done on/by a gas, W pV

hydrostatic pressure, p gh

GM
gravitational potential,
r

displacement of particle in s.h.m., x xO sin t

velocity of particle in s.h.m., v vO cos t

x 2
O x2
3
mean kinetic energy of a molecule of an ideal gas, E kT
2

resistors in series, R R1 R2

resistors in parallel, 1/ R 1/ R1 1/ R2

Q
electric potential, V
4 O r

alternating current/voltage, x xO sin t

transmission coefficient, T exp 2kd

8 2 m U E
where k =
h2

radioactive decay, x xO exp t

0.693

decay constant, t1
2

River Valley High School Page 3 of 21 Year 6 H2 Physics 9646


Preliminary Examination 2013
Section A

Answer all questions.

1 A simple pendulum may be used to determine a value for the acceleration of free fall g.
Measurements are made of the length L of the pendulum and the period T of oscillation.

The values obtained, with their uncertainties, are as shown.


T = (1.90 0.04) s
L = (890 1) mm

(a) Calculate the percentage uncertainty in the measurement of

(i) the period T,

percentage uncertainty = . % [1]

(ii) the length L.

percentage uncertainty = . % [1]

(b) The relationship between T, L and g is given by

Using your answers in (a), calculate the percentage uncertainty in the value of g.

percentage uncertainty = . % [1]

(c) The values of L and T are used to calculate a value of g as 9.733 m s2.

(i) By reference to the measurements of L and T, suggest why it would not be


correct to quote the value of g as 9.733 m s2.

... [1]

River Valley High School Page 4 of 21 Year 6 H2 Physics 9646


Preliminary Examination 2013
(ii) Use your answer in (b) to determine the absolute uncertainty in g.

Hence state the value of g, with its uncertainty, to an appropriate number of


significant figures.

g = m s2 [2]

(d) Explain what is meant by random error and suggest how it may be reduced in
determining the value of g.

..

..

..

. [2]

2 (a) Define the terms moment of a force and torque of a couple.

... [2]

River Valley High School Page 5 of 21 Year 6 H2 Physics 9646


Preliminary Examination 2013
(b) A uniform T-shaped structure, of mass per unit length 1.0 kg m1, is resting against a
smooth wall and a rough floor, as shown in Fig. 2.1. Portion PQ, positioned
perpendicularly to RS at its midpoint, is 1.5 times the length of RS. RS is given to be
2.0 m long.

Fig. 2.1

Determine the magnitude of the resultant force the floor acts on the T-structure.

resultant force = N [3]

River Valley High School Page 6 of 21 Year 6 H2 Physics 9646


Preliminary Examination 2013
(c) Now the T-structure is placed on an elastic cable of spring constant 1 MN m1 with the
structure still pressing against the smooth wall, as shown in Fig. 2.2. Point R is
positioned at the middle of the cable. The T-structure causes the cable to sag, causing
a total extension of 0.010 m, making an angle to the horizontal. The entire system is
suspended above the floor.

Fig. 2.2

Determine the angle if the system is in equilibrium.

= [3]

(d) Explain why the value of is unlikely to be zero.

.. [1]

River Valley High School Page 7 of 21 Year 6 H2 Physics 9646


Preliminary Examination 2013
3 (a) When a force is applied to a vertical spring-mass system, Fig. 3.1 shows the
subsequent displacement-time graph. The system is performing simple harmonic
motion.
m
The period of oscillation of a loaded spring is T 2 where m is the mass on the
k
spring and k is the spring constant.

displacement / m

0.10

1.00 2.00 time / s

0.10

Fig. 3.1

Fig. 3.2 shows how the total potential energy of the spring-mass system varies with
displacement.
potential energy / J

200

100

displacement / m

0.20 0 0.20

Fig. 3.2
(i) Calculate the spring constant k, of the system.

k = . N m1 [2]

River Valley High School Page 8 of 21 Year 6 H2 Physics 9646


Preliminary Examination 2013
(ii) Show that when an identical spring is added in parallel and displaced with the
same force, shown in Fig. 3.3, the effective spring constant is 2k.

force

Fig. 3.3

[2]

(b) Define resonance and name one circumstance in which resonance is useful

..

..

..

..

. [2]

(c) The spring in (a) is installed on each of the wheels of a motor car, which is used to test
a speed bump system by driving at different speeds over a series of equally spaced
bumps.

When the motor car is driven at a particular speed over the speed bumps, the
amplitude of vibration of the car is maximum.

Given that the separation of the bumps is 20 m and the combined mass of the car and
driver is 1000 kg, calculate the speed of the car.

speed of the car = . m s1 [2]

River Valley High School Page 9 of 21 Year 6 H2 Physics 9646


Preliminary Examination 2013
(d) Suggest 2 reasons why resonance is seldom experienced under normal driving
conditions.

..

..

..

. [2]

4 (a) Define potential difference and the volt.

..

..

..

. [2]

(b) A cell of e.m.f. E and internal resistance r is connected to a resistor of resistance R, as


shown in Fig. 4.1.
E r

V
Fig. 4.1

A voltmeter of infinite resistance is connected in parallel with the resistor.

(i) Include a switch in the circuit in Fig. 4.1 so that the voltmeter may be used to
measure the e.m.f. E of the cell or the terminal potential difference V.
Explain when the voltmeter will measure E or V.

... [3]

River Valley High School Page 10 of 21 Year 6 H2 Physics 9646


Preliminary Examination 2013
(ii) State a relationship between E, V, r and the current I in the circuit.

... [1]

(c) Y

G
P R2
A
B
R1
X

Fig. 4.2

In the circuit shown in Fig. 4.2, cell A has a constant e.m.f. of 2.0 V and negligible
internal resistance. Wire XY is 100 cm long with a resistance of 5.0 . Cell B has an
e.m.f. of 1.5 V and an internal resistance of 0.80 . Resistors R1 and R2 are 1.0 and
.0 respectively.

Calculate the length XP required to produce zero current in the


galvanometer G.

length = . m [3]

River Valley High School Page 11 of 21 Year 6 H2 Physics 9646


Preliminary Examination 2013
5 When an electric current flows through a thin metallic conductor in a magnetic field, the
moving charges will accumulate at the sides of the conductor. The voltage measured across
both sides is known as the Hall voltage.

Fig. 5.1 shows a metallic conductor of thickness d and width w, placed perpendicularly to a
magnetic field, B.

Fig. 5.1

(a) (i) An electric current, I, has been passed through the metallic conductor in the
direction shown until a steady voltage reading, VH, is obtained.

Indicate on Fig. 5.1 the polarity of the respective edges of the conductor. [1]

(ii) Explain how your answer in (i) is obtained.

...

.. [1]

(b) Show that the Hall voltage, VH, obtained is given by

IB
VH
ned

where n is the number of mobile charges per unit volume and e is the electronic
charge.

[3]

River Valley High School Page 12 of 21 Year 6 H2 Physics 9646


Preliminary Examination 2013
6 To make space travel more economical, new methods of powering spacecraft are needed.

Magnetic levitation
One proposal is to use electrical energy to accelerate the spacecraft on a track that uses
magnetic levitation (lift). Magnetic levitation would make the spacecraft float above the track
so that friction would be negligible.

Fig. 6.1 shows the principle of magnetic levitation using a superconducting magnet and a
conducting sheet track.

Fig. 6.1

As the superconducting magnet moves, eddy currents are induced in the track which provide
the lift for as long as the magnet is moving. This method of reducing frictional forces has
problems that have to be overcome in practical systems. Although there is no power loss in
the magnet, power is lost due to the electrical resistance in the track. There is also a
magnetic drag force due to induced currents and a drag force due to air resistance acting on
the spacecraft.

(a) (i) Suggest why there is no power loss in the superconducting magnet used in the
magnetic levitation system.

.. [1]

(ii) Use the law(s) of electromagnetic induction to explain why magnetic drag
forces occur in the magnetic levitation system.

.. [2]

(iii) With reference to your answer in (a)(ii), explain what will happen when the
spacecraft accelerates.

.. [1]

River Valley High School Page 13 of 21 Year 6 H2 Physics 9646


Preliminary Examination 2013
Lightcraft
Another method suggests making a lightcraft which is powered by a pulsed infrared laser
beam from a laser that is mounted on the Earth. The principle of this spacecraft is shown in
Fig. 6.2.

Fig. 6.2

The laser beam is directed at the lightcraft. Air from the front of the craft is directed to an
engine which is essentially a cylindrical vessel fitted with an exhaust nozzle. The energy in
the laser beam is reflected on to the air in the engine which rapidly heats to a temperature
of about 40 000 K. This causes the air to expand explosively and the air that is propelled
backwards provides the thrust.

An experimental lightcraft has been propelled using a carbon dioxide laser


(wavelength = 1060 nm) of mean power 10 kW pulsing 28 times a second.

Designers are now working on a 1 MW laser which it is hoped will put a 1 kg practical
lightcraft with 1 kg of hydrogen into space. The laser will be used to propel the lightcraft
vertically to five times the speed of sound (= 340 m s1) during the first 30 km of the flight.
The lightcraft will then use the hydrogen for propellant.

(b) (i) Determine the energy in a single pulse from the laser used when launching the
experimental lightcraft.

energy = .. J [1]

River Valley High School Page 14 of 21 Year 6 H2 Physics 9646


Preliminary Examination 2013
(ii) Calculate the number of photons in one pulse of this laser beam.

number of photons = .. [3]

(c) (i) Calculate the average acceleration that is expected during the first 30 km of
flight for the practical lightcraft.

average acceleration = .. m s2 [2]

(ii) Show that the time taken to cover the first 30 km of flight is about 35 s.

[1]

River Valley High School Page 15 of 21 Year 6 H2 Physics 9646


Preliminary Examination 2013
(d) (i) Calculate the total energy gained by the practical lightcraft during the first
30 km of flight.

total energy gained = .. J [3]

(ii) Calculate the expected efficiency of this method of propulsion.

efficiency = .. [3]

(e) Assume that in the engine, the air is an ideal gas with a temperature of 300 K and a
pressure of 1.0 105 Pa and that the heating takes place at constant volume. Estimate
the pressure of the air after heating by the laser.

pressure = .. Pa [2]

END OF PAPER 2 SECTION A

River Valley High School Page 16 of 21 Year 6 H2 Physics 9646


Preliminary Examination 2013
RIVER VALLEY HIGH SCHOOL
YEAR 6 PRELIMINARY EXAMINATION

H2 PHYSICS 9646
PAPER 2 (SECTION B)
19 SEP 2013
CANDIDATE
NAME

CENTRE INDEX
NUMBER S NUMBER

CLASS 6
INSTRUCTIONS TO CANDIDATES
DO NOT OPEN THIS BOOKLET UNTIL YOU ARE TOLD TO DO SO.
Read these notes carefully.
Write your name, centre and index number and class in the spaces above.

FOR EXAMINERS USE


Section B

Planning Question 7 /12

____________________________________________________________________________
This section consists of 5 printed pages.

River Valley High School Page 17 of 21 Year 6 H2 Physics 9646


Preliminary Examination 2013
7 The needle of an orientation compass shown below deflects according to the magnetic field it
is placed in. The angle of deflection depends on the direction and the magnitude of the
surrounding magnetic field strength.

The magnetic field strength B at a distance from a long straight wire is thought to depend on
the current I passing through it. The relation between the magnetic field strength at that point
and the current I may be written in the form
B = a In
where a and n are constants.

You are provided with a long straight wire and an orientation compass. You may use any other
equipment usually found in a Physics laboratory.

Note that both teslameter (or gaussmeter) and Hall probe are not available.

Design an experiment to determine the value of n.

You should draw a labelled diagram to show the arrangement of your apparatus.
In your account you should pay particular attention to

(a) identification and control of variables,


(b) the equipment you would use,
(c) the procedure to be followed,
(d) how the constant n could be determined with the effect of the Earths magnetic field taken
into consideration, and
(e) any precautions that would be taken to improve the accuracy and safety of the
experiment.

River Valley High School Page 18 of 21 Year 6 H2 Physics 9646


Preliminary Examination 2013
Diagram For
Examiners
use

River Valley High School Page 19 of 21 Year 6 H2 Physics 9646


Preliminary Examination 2013
For
Examiners
use

River Valley High School Page 20 of 21 Year 6 H2 Physics 9646


Preliminary Examination 2013
For
Examiners
use

END OF PAPER 2 SECTION B

River Valley High School Page 21 of 21 Year 6 H2 Physics 9646


Preliminary Examination 2013
2013 Preliminary Examinations
Y6 H2 Physics Paper 2 Suggested Solutions

1 (a) (i) 2.1 % (2.s.f.) A1

(ii) 0.11 % (2 s.f.) A1

(b) 2 (a)(i) + (a)(ii) = 4.3 % (2 s.f.) A1

(c) (i) The value has more significant figures than the data provided. B1
Or
Calculated uncertainty of g renders more than 1 decimal place
meaningless.

(ii) Uncertainty of g = 0.4 (1 s.f.) C1

g = (9.7 0.4) m s2 (same d.p.) A1

(d) An error is random if repeating the measurement under the same conditions
yields readings with error of different magnitude and sign. B1
Or
Measured readings are scattered about the mean value with an error of equal
chance of being negative or positive.

It may be reduced by plotting a graph and drawing a line of best fit for the points. B1
Or
Take repeated measurements of the period and take average for a large number
of oscillations.

2 (a) The moment of a force F about an axis is the product of that force and the B1
perpendicular distance from the line of action of the force to the axis.

The torque of a couple is the product of the force and the perpendicular distance B1
between the forces.

(b) (3 9.81)(0.5 cos45) + Nw(2 sin45) = (2 9.81)(1)(cos45) M1


Nw = 2.4525 N friction of rough floor, Normal reaction of floor = weight of PQ + RS M1
Nresultant = 49.1 N A1

(c) 2Tsin = normal reaction of floor = 49.05 N M1


Using Hookes Law, T = kx, 2(1 106)(0.005)sin = 49.05 M1
= 0.28 A1

(d) From the equation, 2Tsin = weight of PQRS, for 0, T , which is B1


impossible.

2
3 (a) (i) Total Energy, ET = k xo M1
2
30 = k (0.10)
1
k = 6000 N m A1

(ii) F = kx
For a parallel springs system, Force acting on each spring is F/2. M1

River Valley High School Pg 1 of 5 Year 6 H2 Physics 9646


Preliminary Examinations 2013
Extension = x/2

For the whole System, Force = F and Extension = x/2. M1


k// = 2k

(b) Resonance occurs when the driving frequency to a system match the natural B1
frequency of that system.
B1
Occurs in most musical instruments.

(c) When a particular speed of the motorcycle reached gives a larger amplitude of
vibration the natural frequency of the cars suspension system is reached

1 k
To achieve resonance, f = f0 =
2 msystem
k = 4 6000 = 24000
M1
1 24000
f0 = 0.78 Hz
2 1000 A1
1
v = Distance / time = 1/0.78 20 = 15.6 m s

(d) Damping, B2
rarely are there evenly spaced bumps,
usually there is variation in mass and speed. Either 2

4 (a) Potential difference between 2 points of a circuit is defined as the amount of M1


electrical energy converted per unit charge to other forms when a unit charge
moves from one point to the other.

The potential difference between 2 points in a circuit is one volt if one joule of M1
electrical energy is converted to other forms when one coulomb of charge
moves from one point to the other.

(b) (i)
E r

VV
Fig. 4.1
switch
R

Add a switch at the correct position B1


Switch on: to find terminal PD B1
Switch off: to find e.m.f B1

(b) (ii) E = Ir + V A1

(c) Vxy = 1.6667 V C1

River Valley High School Pg 2 of 5 Year 6 H2 Physics 9646


Preliminary Examinations 2013
VR2 = 1.07 V C1

1.07/1.66667 1 m = 0.642 m A1

5 (a) (i) (+) to the right of conductor, () to the left of conductor B1

(ii) Using FLHR, induced magnetic force act on the electrons, resulting in B1
polarity indicated in (a)(i)

(b) eE = Bev E = Bv M1
Q Ne N eV exA M1
I n nevA nevwd
time time V time time
BIw BI M1
VH = Ew = Bvw =
newd ned

6 (a) (i) Superconductors have no resistance B1


so no heating occurs / no I2R losses.

(ii) The superconducting magnet moving in one direction produces changing B1


magnetic flux which induces emfs (Faradays Law) and hence currents in
magnet and track.
By Lenzs Law, the induced currents in the track flow such that they oppose B1
the change in magnetic flux (or the change causing them), thus incurring a
magnetic drag force in the opposite direction.

(iii) When the spacecraft accelerates, there is an increase in rate of change of B1


magnetic flux and hence an increase in magnitude of induced currents
which leads to increased magnetic drag.

(b) (i) 10000/28 = 357 J A1

(ii)
energy of 1 photon
6.63 10 3.00 10 = 1.8764 10
34 8
19
J
C1
1060 10 9
19
(b)(i) / (1.8764 10 ) = 1.903325 1021 = 1.90 1021 photons C1
ecf on (b)(i) A1

(c) (i) v2 = u2 + 2as C1


a = 48 m s2 A1

(ii) v = u + at 1700 = 48 t M1
no ecf on a as t can be determined without a.

(d) (i) EP = mgh or GMm(1/r1 1/r2) = 588 600 J C1


EK = mv2 = 2 890 000 J C1
3.4786 MJ = 3.48 MJ A1

(ii) energy supplied = 1 MW 35 s = 35 MJ C1


efficiency = (d)(i) / 3500 000 = 9.9 % A1
ecf on (d)(i)

River Valley High School Pg 3 of 5 Year 6 H2 Physics 9646


Preliminary Examinations 2013
(e) P / T = constant C1
P = (1 105)(40000/300) = 1.33333 107 = 13 MPa A1

River Valley High School Pg 4 of 5 Year 6 H2 Physics 9646


Preliminary Examinations 2013
Q7
Diagram: switch
long vertical straight wire Clear diagram with
labels.
rheostat
Essential items:
battery,
ammeter A compass rheostat (or variable
power supply),
long straight wire,
ammeter,
battery compass

B-field of wire
BEarth parallel to plane of
Bresultant compass

Bwire [2]
Top view

1. Place wire through a horizontal flat platform. Clamp wire so that it is vertical. Set-up [1]
2. Place compass on platform.

3. With no current flowing in wire, align compass so that it points in direction of the
o
Earths magnetic field with bearing 0 . Draw a line in the North-South direction
on horizontal plane, intersecting the wire (dotted line shown in diagram).

4. Place compass on the dotted line drawn at a fixed distance from wire. Preliminary readings
[1]
5. Preliminary readings : Allow current to flow in wire. Adjust the rheostat to vary
the current and obtain a range of suitable values of current such that the angle
of deflection varies significantly. Discussion of
consideration of
6. The angle of deflection (or bearing) as indicated by the compass needle gives Earths B-field
the angle the resultant B-field makes with the Earths magnetic field. [1]

7. tan = Bwire / BEarth hence Bwire = k tan (where k = BEarth) Dependent


variable(angle )
n
8. Given B wire = a I [1]
ln (k tan) = lnk + ln(tan ) = ln a + n ln I Independent variable
ln (tan ) = (ln a lnk )+ n ln I (current)
[1]
9. Take about 6 readings of varying current (independent variable) and record the Eqn manipulation [1]
corresponding angle of deflection, or bearing (dependent variable) of the
compass needle.
Repeat readings [1]
10. Tabulate current I (measured by ammeter), , tan , ln (tan ) and ln I.

11. Plot ln (tan ) against ln I Analysis [1]


12. If straight line graph is obtained, the gradient of the graph will give the value of n.

Reliability : Any two relevant


1. The distance between the compass and the wire must be kept constant discussions [2]
throughout experiment .
2. Check that there are no magnetic material nearby that will affect the experiment.
Safety:
1. As electrical source is used, avoid using wet hands to handle equipment.

River Valley High School Pg 5 of 5 Year 6 H2 Physics 9646


Preliminary Examinations 2013
RIVER VALLEY HIGH SCHOOL
YEAR 6 PRELIMINARY EXAMINATION

H2 PHYSICS 9646
PAPER 3
25 SEP 2013
2 HOURS
CANDIDATE
NAME

CENTRE INDEX
S
NUMBER NUMBER

CLASS 6
FOR EXAMINERS USE
Section A
INSTRUCTIONS TO CANDIDATES
DO NOT OPEN THIS BOOKLET UNTIL YOU ARE
Q1 /8
TOLD TO DO SO.
Q2 /9
Read these notes carefully.
Write your name, centre and index number and class in the Q3 /8
spaces above.
Q4 /6
Candidates answer on the Question Paper.
Q5 /9
Write in dark blue or black pen. You may use a soft pencil for Section B
any diagrams, graphs or rough working. Do not use staples,
Q6 / 20
paper clips, highlighters, glue or correction fluid.

Section A
Q7 / 20
Answer all questions.
Q8 / 20
Section B Deduction
Answer any two questions.
TOTAL / 80
You are advised to spend about one hour on each section.
__________________________________________________________________________
This question paper consists of 23 printed pages.
The number
River Valley High of marks is given in brackets [ ] Pg
School at 1the end of
of 23 Year 6 H2 Physics 9646
Preliminary Examination 2013
each question or part question.
Data

speed of light in free space, c = 3.00 108 m s1

permeability of free space, O = 4 107 H m1

permittivity of free space, O = 8.85 1012 F m1

(1/(36 )) 109 F m1

elementary charge, e = 1.60 1019 C

the Planck constant, h = 6.63 1034 J s

unified atomic mass constant, u = 1.66 1027 kg

rest mass of electron, me = 9.11 1031 kg

rest mass of proton, mp = 1.67 1027 kg

molar gas constant, R = 8.31 J K1 mol1

the Avogadro constant, NA = 6.02 1023 mol1

the Boltzmann constant, k = 1.38 1023 J K1

gravitational constant, G = 6.67 1011 N m2 kg2

acceleration of free fall, g = 9.81 m s2

River Valley High School Pg 2 of 23 Year 6 H2 Physics 9646


Preliminary Examination 2013
Formulae

1 2
uniformly accelerated motion, s ut at
2
v 2 u 2 2as

work done on/by a gas, W pV

hydrostatic pressure, p gh

GM
gravitational potential,
r

displacement of particle in s.h.m., x xO sin t

velocity of particle in s.h.m., v vO cos t

x 2
O x2
3
mean kinetic energy of a molecule of an ideal gas, E kT
2

resistors in series, R R1 R2

resistors in parallel, 1/ R 1/ R1 1/ R2

Q
electric potential, V
4 O r

alternating current/voltage, x xO sin t

transmission coefficient, T exp 2kd

8 2 m U E
where k =
h2

radioactive decay, x xO exp t

0.693

decay constant, t1
2

River Valley High School Pg 3 of 23 Year 6 H2 Physics 9646


Preliminary Examination 2013
Section A

Answer all the questions in this section.

1 A howitzer shown in Fig. 1.1 has a mass of 5600 kg.

barrel

rear extensible legs


Fig. 1.1

(a) A military truck is towing an unloaded howitzer at a uniform speed of 30.0 km h1. The
howitzer is connected to the rear of the truck by a horizontal tow-bar, which can sustain a
maximum force of 40.0 kN. Total resistive forces acting on the howitzer are 10.0 kN.
Calculate the shortest time the howitzer can be brought to rest safely.

time = .. s [2]

River Valley High School Pg 4 of 23 Year 6 H2 Physics 9646


Preliminary Examination 2013
(b) The howitzer is now loaded with an ammunition round of mass 43.0 kg. Before firing, the
two rear extensible legs of the howitzer are secured to the ground. When fired, the round
has an exit velocity of 563 m s1.

(i) Explain why there is a need to secure the rear legs of the howitzer to the ground.

..

.. [1]

(ii) The barrel of the howitzer can be elevated to an angle above the horizontal.
Calculate the horizontal recoil speed of the howitzer if it is not secured to the
ground as it fires a round at an angle of 25 to the horizontal.

speed = m s1 [3]

(iii)

Fig. 1.2

The two rear extensible legs can be assumed to undergo only compressive force
whenever the artillery is fired. Given that the recoil force acting on each leg is
110 kN, as shown in Fig. 1.2, determine the spring constant of each of the
extensible legs, if each leg is compressed by 0.50 cm.

spring constant = .. N m1 [2]

River Valley High School Pg 5 of 23 Year 6 H2 Physics 9646


Preliminary Examination 2013
2 Fig. 2.1 shows an arrangement that is used to measure the density of sand. The air in the
cylinder is forced into the flask and the air pressure is measured by the pressure gauge P
after the compression.

The process is then repeated with a sample of sand present in the flask.

final position
of piston

Fig. 2.1

(a) The final pressure of the air in the flask is 150 kPa when the process is carried out with
25 g of sand in the flask.

Calculate the change in volume of air and hence the density of this amount of sand.

Make the following assumptions in your calculations:


air is an ideal gas,
the compression is achieved at constant temperature,
the same amount of air is used in both experiments,
the volume of the tubes and pressure gauge is negligible
and the sand is not disturbed in the process.

Given volume of the empty flask = 2.50 104 m3


final pressure of the air in the flask and cylinder without sand = 140 kPa.

change in volume = .... m3

density of sand = .. kg m3 [4]


River Valley High School Pg 6 of 23 Year 6 H2 Physics 9646
Preliminary Examination 2013
Fig. 2.2 shows a graph of the variation of pressure with volume of the air in the cylinder for a
short interval.

1.5

1.4

1.3
5
pressure / 10 Pa

1.2

1.1

1.0
1.4 1.5 1.6 1.7 1.8
4 3
volume / 10 m

Fig. 2.2

(b) (i) Use the data from Fig. 2.2 to show that the process is in fact not isothermal.

[2]

(ii) Use Fig. 2.2 to estimate the work done on the gas for the change in volume
from 1.80 104 m3 to 1.47 104 m3.

work done on gas = .. J [3]

River Valley High School Pg 7 of 23 Year 6 H2 Physics 9646


Preliminary Examination 2013
3 A cyclotron is a type of particle accelerator in dee 1 dee 2
which charged particles accelerate outwards
from the centre along a spiral path.

The particles are made to move in a spiral


trajectory by a static magnetic field and
accelerated by a rapidly varying electric field.

Fig. 3.1 shows the plan view of a cyclotron in


which protons are emitted in between the target
D-shaped electrodes (dees). The protons are
deflected into a circular path by the application of
a magnetic field. a.c.

proton
source

D-shaped
electrodes
(dees)

Fig. 3.1

(a) (i) State the direction of the magnetic field in the region of the dees such that it will
deflect the proton beam in the direction shown in Fig. 3.1.

.. [1]

(ii) Derive an expression for the time T for a proton to make a complete circular
path in this magnetic field in terms of the proton mass m, the magnetic flux
density B acting on the proton and the proton charge +e.

[3]

River Valley High School Pg 8 of 23 Year 6 H2 Physics 9646


Preliminary Examination 2013
(b) In addition to the magnetic field, an alternating electric field is applied only between
the dees such that the proton is accelerated towards whichever dee that is negatively
charged.

(i) Describe how each field (magnetic and electric) affect the motion of the proton.

.. [2]

(ii) Sketch the speed-time graph of a proton from its ejection from the proton
source to after crossing the gap between the dees the third time.

speed

time

[2]

River Valley High School Pg 9 of 23 Year 6 H2 Physics 9646


Preliminary Examination 2013
4 A sinusoidal alternating current source of frequency 50 Hz, is connected to a resistor of
resistance 2.0 k, a diode and an oscilloscope, as shown in Fig. 4.1. The r.m.s. current
through the resistor is 5.0 mA. Assume the oscilloscope has infinite resistance.

50 Hz R 2.0 k oscilloscope

Fig. 4.1

(a) Calculate the peak value of the voltage across R.

peak voltage= .. V [3]

(b) Fig. 4.2 represents the screen of the oscilloscope, each square being 1 cm 1 cm. The
time base of the oscilloscope is set at 5 ms cm1 and the voltage sensitivity is
5.0 V cm1.

Fig. 4.2

Draw on Fig. 4.2 the signal one would expect to see on the oscilloscope. [3]

River Valley High School Pg 10 of 23 Year 6 H2 Physics 9646


Preliminary Examination 2013
5 In a photoelectric effect experiment, the maximum kinetic energy Ek,max of the ejected
photoelectrons is measured for various wavelengths, of the incident light.

Fig. 5.1 shows the variation of Ek,max as a function of .

Ek,max / eV

20.0

15.0

10.0

5.0

0 / nm
50 100 150 200 250

Fig. 5.1

(a) Briefly describe how the maximum kinetic energy of the ejected electrons may be measured
experimentally.

. [2]

(b) Explain what is meant by the work function of a metal.

..

..

.. [1]

River Valley High School Pg 11 of 23 Year 6 H2 Physics 9646


Preliminary Examination 2013
(c) Using Fig. 5.1, calculate the threshold frequency and the work function of the metal used.

threshold frequency = .. Hz

work function = . eV [3]

(d) Data from experiments like this are often graphed showing maximum kinetic energy as a
function of 1/ instead of that in Fig. 5.1.

Sketch a clearly labelled graph to show how Ek,max would vary with 1/.

Ek,max

0 1/

[2]

(e) State one advantage of graphing the data in (d) rather than that in Fig. 5.1.

.. [1]

River Valley High School Pg 12 of 23 Year 6 H2 Physics 9646


Preliminary Examination 2013
Section B

Answer two questions in this section.

6 (a) (i) Explain the difference between gravitational field strength and acceleration of
free fall.

[2]

(ii) Assuming the Earth to be a perfect sphere, explain qualitatively, with the help of
a diagram, the difference in the weight of a person when measured at the poles
and at the equator.

[3]

(b) Assuming the Earth to be a sphere of radius 6.4 x 106 m, estimate the mass of the
Earth.

mass of Earth = . kg [2]


River Valley High School Pg 13 of 23 Year 6 H2 Physics 9646
Preliminary Examination 2013
(c) A space capsule is travelling between the Earth and the Moon. Calculate the distance
from the Earth when the resultant gravitational field acting on it is zero. Consider only
the gravitational force of the Earth and the Moon.

Given that the mass of the Moon is 7.4 x 1022 kg and distance between centres of the
Earth and the Moon is 3.8 x 108 m.

distance = m [3]

(d) The speed of escape from the Earth is the speed with which a body should be
projected from the surface of the Earth in order to reach an infinite distance and is
equal to 1.1 x 104 m s 1.

Given the ratio of the radius of the Earth to the radius of the Moon is 3.7, estimate the
speed of escape from the Moon. Distance from Earth = . kg [2]

speed of escape = m s1 [3]

River Valley High School Pg 14 of 23 Year 6 H2 Physics 9646


Preliminary Examination 2013
(e) A satellite orbits the Earth of mass M in a circular path of radius r with period T.

(i) Draw a labelled free body diagram of the force(s) acting on the satellite and
state the acceleration of the satellite in terms of r and T.

Earth

satellite

acceleration = .. m s2 [2]

(ii) Hence show that


4 2 3
T2 r .
GM

[2]

(iii) Calculate the radius of the orbit of a geostationary satellite.

radius = .. m [2]

(iv) State the radius of another geostationary satellite of twice the mass orbiting the
Earth.

radius = .. m [1]

River Valley High School Pg 15 of 23 Year 6 H2 Physics 9646


Preliminary Examination 2013
7 The first microwave oven was invented by Percy Spencer after World War II, based on the
fundamentals of radar technology developed during the war. Microwave ovens heat foods that
have high water content, quickly. If the presence of water is negligible in the microwaved item,
the item rarely gets hot. The operating frequencies of microwave ovens are usually within the
range of 915 MHz and 2.45 GHz, respectively.

Fig. 7

(a) State three conditions for the formation of stationary waves.

..

..

..

..

..

..

.. [3]

(b) During operation, stationary waves are formed within the chamber. Given that, for a
particular brand of microwave oven, the frequency generated is 2.00 GHz and the
cooking chamber is 45 cm wide.

(i) Determine the number of antinodes under the given conditions, assuming
displacement nodes are found at both ends of the cooking chamber.

number of antinodes = . [2]

River Valley High School Pg 16 of 23 Year 6 H2 Physics 9646


Preliminary Examination 2013
(ii) Sketch the stationary waves in the given diagram below. Label the positions of
displacement nodes and antinodes.

microwave
oven
[2]

(iii) In order to investigate the wavelength of microwaves, using laboratory equipment


and apparatus, suggest an experimental procedure to determine the wavelength,
assuming you do not know the frequency of the electromagnetic waves. Include
relevant diagram of the setup.

[3]

River Valley High School Pg 17 of 23 Year 6 H2 Physics 9646


Preliminary Examination 2013
(c) (i) When food placed on a porcelain plate is heated in the microwave oven, the food
becomes very hot, while the plate remains relatively cooler. Discuss how a
microwave oven cooks the food that is placed in the cooking chamber.

[2]

(ii) Suggest and explain the highest temperature the food could reach by this cooking
method of microwaving.

[2]

(d) A microwave source emits progressive transverse waves which pass through a diffraction
grating.

(i) Explain what is meant by a progressive transverse wave.

[3]

River Valley High School Pg 18 of 23 Year 6 H2 Physics 9646


Preliminary Examination 2013
(ii) A certain order of diffraction for 2.00 GHz is superimposed on 2.45 GHz
microwaves of the next order, where the angle of diffraction is 67. These
diffractions are also the largest order of diffraction for the respective frequencies.

Determine the number of lines per metre in the grating. Leave your answer to 2
significant figures.

number of lines per metre = m1 [3]

River Valley High School Pg 19 of 23 Year 6 H2 Physics 9646


Preliminary Examination 2013
8 (a) Both nuclear fission and alpha decay involve nuclear reactions that release energy.

State one other similarity and one difference between the two nuclear reactions.

..

..

..

..

.. [2]

(b) In a nuclear reactor, a possible reaction which can take place is described by the
following equation:

235
92 U 01n 142
56 Ba Kr 3 01n 174.4 MeV
91
36

The binding energy per nucleon and mass of one nuclide of U-235 is 7.59 MeV and
3.902 1025 kg respectively.

(i) Explain what is meant by the binding energy of a U-235 nuclide.

....

... [1]

(ii) With reference to the nuclear reaction, explain why Ba-142 and Kr-91 nuclides
have greater binding energies per nucleon than that of U-235 nuclide.

..

..

..

..

... [2]

River Valley High School Pg 20 of 23 Year 6 H2 Physics 9646


Preliminary Examination 2013
(iii) The fuel rods in the reactor contain 1.0 104 kg of U-235 nuclide. Calculate the
total energy released from the complete fission of all the U-235 nuclei in the fuel
rods.

energy = MeV [2]

(iv) If the mean power output of the reactor is 500 MW and the efficiency of
conversion of nuclear energy to electrical energy is 40%, estimate the time for
which the fuel rods can be used.

time = years [2]

(v) Neutrons are released during fission. Suggest why the fuel rods are usually
replaced well before the time estimated in (iv) has elapsed.

..

..

[1]

River Valley High School Pg 21 of 23 Year 6 H2 Physics 9646


Preliminary Examination 2013
(c) A certain model of a smoke detector shown in Fig. 8.1 has a small radioactive source
inside the chamber of the detector. During normal operation, the radioactive source
emits ionising particles and a certain ionisation current is maintained inside the
chamber. In the event of a fire, when smoke particles enter the chamber, they attach
themselves to the ions and neutralise them. The detector senses a drop in current and
sets off an alarm.

Fig. 8.1

(i) The manufacturer claims that the radioactive source in the smoke detector
presents no hazard to health in normal use. Comment on this claim and briefly
explain which type of radioactive source is used in the smoke detector.

..

..

..

..

..

..

[3]

(ii) When the radioactive source in the detector is placed close to a Geiger-Mller
tube, the count rate measured is 2000 s1. The average number of ions produced
by each radiation particle is 5.0 104 and each ion carries a charge of
1.6 1019 C.

Estimate the maximum ionisation current in the smoke detector.

current = A [2]
River Valley High School Pg 22 of 23 Year 6 H2 Physics 9646
Preliminary Examination 2013
(iii) Suggest why the ionisation current in the smoke detector is significantly less than
this maximum value.

[1]

(iv) The smoke detector should be disposed of when its maximum ionisation current
drops below 5.0 1012 A. The manufacturer claims that the life of the detector is
10 years.

1. Define half-life of a radioactive source.

... ..

[1]

2. Estimate the half-life of the radioactive source used in the detector.

half-life = years [3]

END OF PAPER

River Valley High School Pg 23 of 23 Year 6 H2 Physics 9646


Preliminary Examination 2013
RVHS H2 Physics Prelim 2013

Paper 3 Soln

1 (a) v = 8.33 m s1
Fnet = Fmax = F friction
56008.33 M1
Fmax = 40.0 kN 10.0 kN
t t
t = 0.933 s
A1

(b) (i) To minimize the distance the howitzer moves backwards, as a result of B1
the recoil.

(ii) By conservation of momentum,


mhowitzervhowitzer + mroundvround = 0 C1
vround,horizontal = 563cos25 = 510.25 m s1 M1
(5600)(vhowitzer) = (43)(510.25)
vhowitzer = 3.92 m s1 (backwards) A1

(iii) 220 kN = 2F = 2kx = 2(k)(0.00500) M1


k = 2.2 x 107 N m1 A1

2 (a) p1V1 = p2V2 = nRT C1


(140)(2.50) = 150V2
V2 = 2.33333 C1
change in volume = 2.50 2.33333 = 1.66667 105 C1
density of sand = 0.025 / (1.66667 105) = 1500 kg m3 A1

(b) (i) Read at least 2 data points correctly from graph and use them to show M1
that pV is not constant and
since the amount of air is assumed to be the same, the temperature must M1
be different. p1V1 = p2V2 = nRT
e.g.
p1V1= (1.0)(1.8) = 1.8
p2V2 = (1.5)(1.47) = 2.205
p3V3 = (1.05)(1.75) = 1.8375

(ii) Area under p-V graph = work done on gas C1


Correct method to determine area e.g. counting squares, approximation M1
to trapezium (neither of the scales start from zero)
3.8 to 4.2 J A1

River Valley High School Pg 1 of 9 Year 6H2 Physics 9646


Preliminary Examinations2013
3 (a) (i) Into the plane of the paper. B1

(ii) Magnetic force provides centripetal force. M1


Bev = mv2/r
Be = mv/r
v = 2r/T M1
Be = r/T)/r
T = 2m/Be M1

(b) (i) B-field reverses the direction of the proton (increasing the path length) B1
(accept change in direction)
E-field increases the speed/kinetic energy of the proton B1
(accept accelerates the motion)

(ii) time interval for E-field portion getting shorter (speed increases) B1
time interval for B-field portion constant (period independent of v) (T/2)
increase in speed is getting smaller for E-field portion B1
speed is constant for B-field portion

speed

time
T/2 T/2 T/2

4 (a) mean P = (peak P) for half-rectified wave C1


Irms2R= (Io2R )
Irms = Io
peak current, Io = 2 (5 103) C1
peak voltage across R, Vo = IoR = 2 (5 103)(2.0 103) = 20 V A1

(b) timebase: f = 50 Hz, T = 20 ms 4 squares M1


voltage gain: Vo = 20 V 4 squares M1
shape: half-rectified wave M1

5 (a) apply negative potential at the collector, slowly increase negative value until B1
photo-current drops to zero. This potential is the stopping potential Vs.
Use Ek, max = e Vs

B1
(b) min energy to minimum energy required to eject an electron from the surface of a B1
metal.

River Valley High School Pg 2 of 9 Year 6H2 Physics 9646


Preliminary Examinations2013
(c) (at least two sets of values and find average)
hc / = Ek, max +
.. = 4.86 eV or 5.29 eV or 4.93 eV.
average 5.0 eV M1

threshold freq = / h 1.2 x 1015 Hz


A1
Ek, max

A1
(d)

1/
1/0

[1 mk for correct straight line; 1 mk for correct label]

(e) Best-fit straight line allows for random error to be reduced more OR A1
Work-function can be obtained directly from the negative of the vertical intercept.

River Valley High School Pg 3 of 9 Year 6H2 Physics 9646


Preliminary Examinations2013
6 (a) (i) acceleration of free fall: the acceleration of a body towards the surface of B1
the Earth.
gravitational field strength: the force per unit mass acting on a small mass
placed at a point in the gravitational field. B1
Accept:
Gravitational field strength is independent of the circular motion that an
B1
object makes with earth whereas acceleration of free fall includes that
circular motion effect.

(ii)
North Pole Equator

Fg
N

Fg

Fc = 0 Fc not = 0

Free Body diagram (include Normal reaction Force, Fg) B1


Correctly identify that N = Weight.

At different locations, due to the different radius with which object is


rotating, angular velocity is constant, ac (as a result, Fc ) is not constant. B1

Fc is max at the equator and is 0 at the poles.


Fg N = Fc (Allow Representation Vector form)
Fg is constant assuming that earth is spherical in shape. Bigger Fc -> B1
Smaller N.
(b) g = G m/r2 M1

m = g r2 / G

m = 9.81(6.4x106)2 /6.67 x10-11 =6.0 x1024 A1


(c) At the null point, g resultant = 0
i.e. gE = gM M1

GME/rE2 = GMM/rM2

6.0 x 1024 / x2 = 7.4 x 1022 / (3.8 x108 x)2 M1

x = 3.4 x 108 m A1

River Valley High School Pg 4 of 9 Year 6H2 Physics 9646


Preliminary Examinations2013
(d) Speed of escape=> minimum speed to provide enough KE to offset the
negative GPE on Moon.

Conservation of Energy => Total Initial Energy = Total Final Energy M1

1/2mv2 = GM/r M1

(VM / VE)2 = (MM/ME) x (rE/rM)

VM = 2300 - 2400 m s-1 A1


(e) (i)

Since there is only one force acting on the body.


VM = 2500 m s-1 [A1]

M1
Gravitation
Force

A1
a = w2r = 42r/T2

(ii)

Fc = Fg

mac = GMm/r2 [M1] M1

ac = GM/r2

= 42/T2r ..[M1] M1

4 2 3
T2 r
GM
(iii)

Correct Substitution, T, M, G M1

r = 4.1 x107 m A1

(iv)

No Change A1

River Valley High School Pg 5 of 9 Year 6H2 Physics 9646


Preliminary Examinations2013
7 (a) two progressive waves of equal amplitude B1
equal frequency and speed B1
travelling from opposite directions and meet B1

(b) (i) v = f

3 x 108 = (2 x 109) = 0.15 m


M1
when standing waves generated, distance between two adjacent
nodes/antinodes equal to half a wavelength.

6 antinodes A1

(ii)

N N
A A

(iii)

B1
relevant diagram given

Microwave generator produces mcirowave through an emitter. These


waves are reflected back by the reflector and the two waves superpose to B1
give a stationary wave.

A microwave detector is placed between the emitter and the reflector to


detect the nodes and antinodes of the stationary wave. Detector is
connected to a cathode-ray oscilloscope (CRO) with the time-base turned
off.

By moving the microphone slowly backward and forward, a vertical trace

River Valley High School Pg 6 of 9 Year 6H2 Physics 9646


Preliminary Examinations2013
can be seen on the screen of the CRO which varies from a minimum
length to a maximum length corresponding to the nodes and antinodes.

Measuring the length between successive nodes or antinodes of the B1


stationary wave will give /2. From this measurement, the wavelength of
the sound waves can be obtained.

(c) (i) food contains high water content.


energy of microwave transfers to water molecules in food B1
water molecules vibrate more energetically as a result of energy transfer
from the microwave, transferring energy to the rest of the food as thermal B1
energy

(ii) food contains water and boiling point of water is 100C B1


so highest temperature reached would be 100C B1

(d) (i) A progressive transverse waveis a wave in which the energy transfer is in B1
the direction of wave motion while displacements of the particles in the B1
wave are at right angles to the direction of transfer of the energy of the B1
wave.

(ii) dsin=n

2.00 GHz 0.15 m wavelength


2.45 GHz 0.122 m wavelength

dsin=(n)(0.15) M1
dsin=(n+1)(0.122)

0.15n = 0.122n + 0.122

n = 4.35 M1

largest order of diffraction, assume d(sin90) = 4.35(0.15)

average number of lines per metre = 1/d = 1.53 A1

River Valley High School Pg 7 of 9 Year 6H2 Physics 9646


Preliminary Examinations2013
8 (a) Similarity: both involves unstable nuclides. OR both involves a loss in
mass.
Difference: [A1]
Fisson: from heavy nucleus into two lighter nuclei of approximately
equal masses.
Alpha decay: product includes alpha particle and another much [A1]
heavier nucleus OR
Fisson: Mostly induced. Alpha decay: Spontaneous

(b) (i) work done needed to separate one nuclide of U-235 into its constituent [A1]
neutrons and protons.

(ii) Fission involves splitting of a unstable heavy nuclide into two lighter stable
nuclides, [B1]
and binding energy per nucleon gives an indication of its stability, hence
Ba-142 and Kr-91 nuclides have greater binding energies per nucleon than
that of U-235 nuclide. [B1]
(iii) each reaction releases 174.4 MeV (given)
1 10 4
total number of reactions =
3.902 10 25
1 10 4
total energy released = 174.4 MeV
3.902 10 25
= 4.47 1030 MeV

[M1]

[A1]
(iv) Power = energy / time
4.47 1030 106 1.6 1019
hence 0.4 500 106 [M1]
t
t = 5.72 108 s 18 years

[A1]
(v) As nuclear fuel undergoes fission, many new substances are
continually formed which can absorb the neutrons that are needed
to continue the chain reaction for fission. For this reason, the fuel [A1]
rods must be periodically replaced before all the nuclear fuel they
contain has been used up.

(c) (i) Claim valid, source emits alpha particles


low penetrating power (or short range), placed on ceiling hence far [B1]
from people
Alpha although highly ionising, it is only harmful if ingested, which is [B1]
normally not possible during normal use

[B1]
(ii) In 1 second, 2000 ions produced, hence total charge in 1 s
= (2000)(5104)(1.610-19) [M1]
= 1.610-11 A

River Valley High School Pg 8 of 9 Year 6H2 Physics 9646


Preliminary Examinations2013
[A1]
(iii) Dust particles in air may neutralise the ions, hence lower the current [A1]
detected.
(iv) 1) half-life : average time taken for the activity of the nuclide to fall to half its
original value.
OR average time taken for half the original number of radioactive [A1]
nuclei to decay.

(2) after 10 years, I = 510-12 A (given)

Q = It

Nq = It

5 1012 -1
N/t=I/q= s ( = no. of ions produced per sec)
1.6 1019
[M1]
since each radiation particle produces 5.0 10 ions (which
4

give rise to the current)

hence number of radiation particle per second is

5 10 12 [M1]
C= = 625 s-1
(5 10 )(1.6 10 )
4 19

[A1]
ln 2
t
C = Co e
t1 / 2

ln 2
10
625 = 2000 e
t1 / 2

solving t1/2 = 5.96 years

River Valley High School Pg 9 of 9 Year 6H2 Physics 9646


Preliminary Examinations2013
RAFFLES INSTITUTION
2013 Preliminary Examination

PHYSICS 9646 / 01
Higher 2

Paper 1 Multiple Choice

30 September 2013
1 hour 15 minutes

Additional Materials: OMR form


Soft clean eraser
Soft pencil (type B or HB is recommended)

READ THESE INSTRUCTIONS FIRST

Do not open this booklet until you are told to do so.

Fill in your particulars on the OMR form.

There are forty questions on this paper. Answer all questions. For each question
there are four possible answers A, B, C and D. Choose the one you consider correct
and record your choice in soft pencil on the OMR form.

Read the instructions on the OMR form very carefully.

Each correct answer will score one mark. A mark will not be deducted for a wrong
answer.

Any rough working should be done in this booklet.

This booklet consists of 21 printed pages including the cover page.


2

Data

speed of light in free space, c = 3.00 108 m s1

permeability of free space, 0 = 4 107 H m1

permittivity of free space, 0 = 8.85 1012 F m1


(1/(36)) 109 F m1

elementary charge, e = 1.60 1019 C

the Planck constant, h = 6.63 1034 J s

unified atomic mass constant, u = 1.66 1027 kg

rest mass of electron, me = 9.11 1031 kg

rest mass of proton, mp = 1.67 1027 kg

molar gas constant, R = 8.31 J K1 mol1

the Avogadro constant, NA = 6.02 1023 mol1

the Boltzmann constant, k = 1.38 1023 J K1

gravitational constant, G = 6.67 1011 N m2 kg2

acceleration of free fall, g = 9.81 m s2


3

Formulae

uniformly accelerated motion, s ut 21 at 2

v 2 u 2 2as

work done on/by a gas, W pV

hydrostatic pressure, p gh

Gm
gravitational potential,
r

displacement of particle in s.h.m., x x0 sin t

velocity of particle in s.h.m., v v 0 cos t

v x2
0 x2
mean kinetic energy of a
E 32 kT
molecule of an ideal gas,

resistors in series, R R1 R2 ...

resistors in parallel, 1 R 1 R1 1 R2 ...

Q
electric potential, V
4 0 r

alternating current/voltage, x x0 sin t

transmission coefficient, T exp 2kd

8 2 m U E
where k
h2

radioactive decay, x x0 exp t

0.693
decay constant,
t1
2
4

1 A car is travelling along the expressway under smooth traffic conditions. What is the
order of magnitude of its kinetic energy, in joules?

A 103 B 104 C 105 D 106

2 A quantity X varies with temperature as shown.

0
0 100 oC

is determined from the corresponding values of X by using this graph.


X is measured with a percentage uncertainty of 1% of its value at all temperatures.

Which statement about the uncertainty in is correct?

A The percentage uncertainty in is least near 0 oC.


B The percentage uncertainty in is least near 100 oC.
C The actual uncertainty in is least near 0 oC.
D The actual uncertainty in is least near 100 oC.

3 Two forces F1 and F2 act with an angle between them.


F2


F1

Which combination could produce a resultant force of magnitude 1 N?

F1 / N F2 / N
A 1 1 less than 90o
B 1 1 more than 90o but less than 180o
C 2 1 less than 90o
D 2 1 more than 90o but less than 180o
5

4 Water-bombing planes are deployed to help fight fires in the hotspots of Riau,
Indonesia. One such plane flies horizontally over a fire at a speed of 30 m s1 and drops
a giant water bomb to help extinguish the fire. It flies at a height of 200 m, as shown
below.

30 m s1

200 m

d
d

How far before the flame (distance d) must the plane release the water bomb?

A 0m B 96 m C 135 m D 192 m

5 The variation of the force F with time t acting on a body of mass 200 g is as shown in
the graph below.

F/N

t / ms

Given that the velocity of the body at t = 75 ms is 15 m s1, what is the velocity of the
body at t = 150 ms?

A 24 m s1 B 30 m s1 C 39 m s1 D 55 m s1
6

6 A fast moving neutron with an initial velocity u makes a head-on elastic collision with a
stationary proton. After the collision, the velocity of the neutron is v and that of the
proton is w.
Taking the masses of the neutron and proton to be equal, which one of the following
statements is incorrect?

A The fact that the collision is elastic implies that u + v = w.


B By considering kinetic energies of the particles, it can be shown that u2 = v2 + w2.
C The speed of the proton after the collision is the same as that of the neutron before
the collision.
D The proton and the neutron move off in opposite directions with equal speeds.

7 A stationary body floats in water.

Which statement about the forces acting on the body is correct?

A The gravitational force is equal to the viscous force.


B The gravitational force is greater than the upthrust.
C The upthrust is zero.
D The viscous force is zero.

8 An object is pushed from rest in a straight line by a variable force F along a rough
ground. The ground exerts a constant frictional force of 5 N throughout the motion of
the object. The variation with displacement x of the force F is shown below.

F/N

K
5

L M
x/m
0 10
The magnitude of the kinetic energy of the object when it has travelled 10 m is the area

A K B K+L C K+L+M D L+M


7

9 In the diagram below, a 50.0 g ball at point X is projected up a smooth slope with a
velocity of 4.20 m s1, where it encounters a light spring after moving a distance of
0.200 m. The spring has a spring constant of 120 N m1.

60o
X

What is the maximum compression of the spring?

A 1.21 mm B 5.93 mm C 7.36 cm D 7.70 cm

10 An aircraft flies along a horizontal circle of radius 10.0 km with a constant speed of
155 m s1.

wing of the
aircraft

What is the angle between its wings and the horizontal?

A 13.8 B 22.6 C 67.4 D 76.2

11 The bob of a simple pendulum is released from rest at P. The mass of the bob is m and
the length of the pendulum is L.

60o
P

What is the tension in the string when the pendulum bob is at position Q (where the
string is vertical)?

A 0.50 mg B 0.87 mg C 1.3 mg D 2.0 mg


8

12 Two stationary particles of masses M1 and M2 are a distance d apart. A third particle,
lying on the line joining the particles, experiences no resultant gravitational force.

What is the distance of this particle from M1?

M
A d 2
M1

M
B d 1
M2

M1
C d
M1 M2

M1
D d
M M
1 2

13 The diagrams below show the variation with time t of the displacement x and the
velocity v of a body when it is oscillating with simple harmonic motion.

x / cm
1.2

0 t/s
T
1.2

v / cm s1
6.0

0 t/s
T
6.0

What is the value of T ?

2
A B C 5 D 10
5 5
9

14 A spring-mass system is subjected to a periodic driving force of constant amplitude and


of variable frequency f. The graph below shows how the amplitude of the spring-mass
system varies with f.

amplitude

f0 f
Which of the following graphs best represents how the amplitude of the spring-mass
system varies with f if the mass is decreased?

A B

amplitude amplitude

f0 f f0 f

C D

amplitude amplitude

f0 f0 f
f

15 The specific heat capacity of ice is 2090 J kg1 K1 and its specific latent heat of fusion
is 334 kJ kg1. The density of ice is 917 kg m3.

How much heat is required to melt 1000 m3 of ice at a temperature of 20C?

A 38.6 MJ B 345 MJ C 38.6 GJ D 345 GJ


10

16 A heat pump takes a fixed mass of ideal gas through a cycle from X Y Z X as
shown.

pressure
X
p0

p0
Z Y
volume
V0 2V0 4V0

Which of the following statements is incorrect?

A The mean kinetic energy of the gas molecules is the same at X and Y.
B The gas loses heat to its environment as it goes from Y to Z.
C Overall, there is net heat supplied to the gas.
D The internal energy of the gas decreases when it goes from Z to X.
11

17 The graph below shows the variation with distance x of the displacement y of a
transverse wave at a particular instant of time.
y / cm

2.0

0 x/m
0.2 0.4
2.0

A second transverse wave has the same amplitude, but has double the speed and
double the frequency of the first wave.
For the second wave, which of the graphs below shows the variation with distance x of
its displacement y?
y / cm

2.0
A
0 x/m
0.2 0.4
2.0

y / cm

2.0

B 0 0.4 x/m
0.2
2.0

y / cm

2.0

C 0 x/m
0.2 0.4
2.0

y / cm
4.0

x/m
D 0
0.2 0.4

4.0
12

18 Ripples on the surface of a pond spread out in circles from the point of an initial
disturbance.

Assume that the energy of the wave is spread over the entire circumference of the
ripple. The amplitude A at a distance r from the initial disturbance is proportional to

1 1 1
A B C D r
r r r2

19 A student uses a loudspeaker of adjustable frequency to measure the depth of a water


well. He detected two successive resonances at 52.0 Hz and 60.0 Hz. The speed of
sound in air is 343 m s1.

What is the depth of the well? [Ignore end corrections.]

A 5.35 m B 10.7 m C 21.4 m D 42.9 m

20 A grating with 250 lines per mm is used to view an incandescent lamp.

What is the number of complete visible spectra?

A 5 B 10 C 11 D 20

21 W, X, Y and Z are four points on a square as shown.

W X

Z Y

A point charge +Q is fixed at W. Another point charge Q is moved from X to Y.

Which of the following statements is true?

A The electric potential at Z will increase.


B The electric potential energy of the system will decrease.
C The attractive force between the two charges will increase.
D The magnitude of the electric field strength at Z will increase.
13

22 An electron, placed at a point P inside a uniform electric field, experiences a force of


magnitude F. The horizontal lines below are equipotential lines and V1 > V4.

x
V1
P
V2
y
V3
V4
Q
The electron is moved from point P to point Q.

What is the change in the potential energy of the electron?

A Gain of Fx
B Gain of Fy
C Loss of Fx
D Loss of Fy

23 Five resistors of resistance R are connected as shown.

R
E F

R
R R

H G
R

Between which two points is the resistance of the combination a maximum?

A E and F
B E and G
C F and G
D F and H
14

24 A 6.0 V battery of negligible internal resistance is connected to one 1.0 k resistor, two
4.0 k resistors and a 5.0 k variable resistor as shown.

5.0 k 4.0 k

6.0 V
X Y

1.0 k 4.0 k

VXY is the potential of X with respect to Y.

What are the limits of VXY?

A 2.0 V and 3.0 V


B 2.0 V and 3.0 V
C 0 V and 6.0 V
D 6.0 V and 0 V

25 A beam of ions, each of charge 3.20 1019 C and travelling at a speed of


6.0 104 m s1, enters a region where a uniform magnetic field of flux density
2.4 102 T acts normally to the original direction of the beam, as shown below.

beam of ions magnetic field of flux


density 2.4 102 T
6.0 104 m s1
acting normally into
0.195 m paper

When the ions are in the magnetic field, they travel in a circular path of radius 0.195 m.

What is the nucleon number of the nuclei of the ions in the beam?

A 4 B 8 C 15 D 30
15

26 The diagram shows four parallel wires W, X, Y and Z that carry currents of different
magnitudes in the different directions.

i 2i i i

a a a
W X Y Z

Which wire has the largest resultant force acting on it?

A W B X C Y D Z

27 A small magnet that is suspended by a thread is placed near the end of a solenoid
carrying a direct current i.

soft iron core


S N

i i

What happens to the magnet as the soft iron core is inserted into the solenoid?

A The magnet will swing to the left.


B The magnet will swing to the right.
C The magnet will start to rotate.
D Nothing happens.
16

28 A roller coaster train moves in the Northward direction in a region where the Earths
magnetic field is horizontal. It loop-the-loop in a vertical circle as shown. The axle PQ
of the front wheels of the train is a conductor and is perpendicular to the Earths
magnetic field.

North
Y

P Train enters Train leaves


the loop the loop

Q
X

Top view Side view

At which point, X or Y, of the loop will the induced e.m.f. in the axle PQ be larger, and
correspondingly, which end of PQ has a higher potential?

larger e.m.f. higher potential


A X P
B X Q
C Y P
D Y Q

29 The diagram below shows a metal disc of diameter 2.3 cm situated in a uniform
magnetic field of flux density 3.6 105 T. The plane of the disc is perpendicular to the
magnetic field which is directed into the plane of the diagram. The disc is rotated about
an axis through its centre at 1500 revolutions per minute.

rotation region of uniform


magnetic field of flux
disc density 3.6 105 T

What is the magnitude of the induced e.m.f. between the centre of the disc and its rim?

A 0V B 3.74 107 V C 2.35 106 V D 2.24 105 V


17

30 The screen of a cathode ray oscilloscope is fitted with a grid ruled in cm. When a
sinusoidal potential difference is applied to the Y-input, the resulting trace is shown in
the diagram below. The Y-sensitivity is set at 2.00 V cm1 and the time-base is
switched off.

1 cm

1 cm

What is the root-mean-square value of the applied p.d.?

A 2.83 V B 4.00 V C 5.66 V D 8.00 V

31 An ideal transformer has 100 turns in its primary coil and 500 turns in its secondary
coil. The power supplied is 2000 W and the alternating voltage in the primary coil and
secondary coils are Vp and 2000 V respectively.
Which of the following gives the correct value of voltage Vp and current Ip in the primary
coil?

Vp / V Ip / A
A 400 1.0
B 400 5.0
C 10 000 1.0
D 10 000 5.0

32 The diagram shows a generator with supply cables linking to the factories. The
generator produces an electrical power of 2000 kW, 240 kV. The supply cables have a
total resistance of 1.5 k.

Generator
2000 kW supply cables Factories
240 kV

What is the power loss in the supply cables?

A 38.4 kW B 104 kW C 208 kW D 2.00 103 kW


18

33 The diagram shows five energy levels of an atom. Five possible transitions between the
levels are indicated. Each transition produces a photon of definite energy and
frequency.

Which spectrum corresponds most closely to the transitions shown?

increasing frequency

34 When electrons of energy E are incident on a pair of narrow slits 0.0600 m apart, the
bright bands in the interference pattern are separated by 0.400 mm on a screen
20.0 cm from the slits.

What is the value of E ?

A 0.136 eV B 41.4 eV C 105 eV D 10400 eV


19

35 Suppose Fuzzy, a quantum-mechanical duck of mass 2.00 kg, lives in a world in which
h, the Planck constant, is 2 J s.

Fuzzy is initially known to be within a pond 1.00 m wide.

What is the minimum uncertainty in the component of his velocity parallel to the width of
the pond?

A 0.250 m s1 B 0.500 m s1 C 1.00 m s1 D 2.50 m s1

36 Which of the following energy band diagrams is a possible representation of a p-type


semiconductor?

A B

Conduction band Conduction band

Donor level Acceptor level

Valence band Valence band

C D

Conduction band Conduction band

Donor level Acceptor level

Valence band Valence band


20

37 Which of the following energy level diagrams can be used to build a laser?

A B

E2 E3
(metastable state)
Fast decay

E2
Optical (metastable state)
pumping Laser transition Optical
pumping

Laser transition

E1
E1

C D

E4
E3 (metastable state)
(metastable state)
Laser transition
Laser transition E3

E2
Optical
pumping Optical Fast Decay
pumping
Fast decay
E2

E1
Fast Decay

E1
21

38 The variation with thickness x of the number n of -particles penetrating a material is


shown below.

0 x
0

What can be deduced from the graph about the -particles?

A The -particles have approximately the same initial energy.


B The range is independent of the initial energy.
C The -particles produce high levels of ionisation.
D The -particles have a large mass.

39 One reaction of a Beryllium nuclide and a neutron is as shown.


9
4 Be 01n 2 42 He 2X
What is particle X?

A an -particle
B an electron
C a neutron
D a proton

40 Uranium-238 decay by alpha-emission and has a half-life of 1.42 1017 s.

What mass of this nuclide will give an emission of one alpha-particle per second?

A 8.10 105 g
B 3.40 102 g
C 2.38 102 g
D 1.42 1019 g

End of Paper
Centre Number Index Number Name Class

RAFFLES INSTITUTION
2013 Preliminary Examination
PHYSICS
Higher 2
9646 / 02
16 September 2013
Paper 2
1 hour 45 minutes
Candidates answer on the Question Paper.
No Additional Materials are required.

READ THESE INSTRUCTIONS FIRST

Write your Centre number, index number, name and class in the spaces provided at the top
of this page.
Write in dark blue or black pen.
You may use a soft pencil for any diagrams, graphs or rough working.
Do not use staples, paper clips, highlighters, glue or correction fluid.

Answer all questions.

Write your answers in the spaces provided in this booklet.


The number of marks is given in brackets [ ] at the end of each question or part question.

For Examiners Use


1 / 10
2 /7
3 /9
4 /9
5 /7
6 /18
7 /12
Total /72

This booklet consists of 18 printed pages including the cover page.


2

Data

speed of light in free space, c = 3.00 x 108 m s1

permeability of free space, 0 = 4 x 107 H m1

permittivity of free space, 0 = 8.85 x 1012 F m1


(1 / (36 )) x 109 F m1

elementary charge, e = 1.60 x 1019 C

the Planck constant, h = 6.63 x 1034 J s

unified atomic mass constant, u = 1.66 x 1027 kg

rest mass of electron, me = 9.11 x 1031 kg

rest mass of proton, mp = 1.67 x 1027 kg

molar gas constant, R = 8.31 J K1 mol1

the Avogadro constant, NA = 6.02 x 1023 mol1

the Boltzmann constant, k = 1.38 x 1023 J K1

gravitational constant, G = 6.67 x 1011 N m2 kg2

acceleration of free fall, g = 9.81 m s2


3

Formulae

uniformly accelerated motion, s = ut 21 at 2

v2 = u 2 2as

work done on/by a gas, W = pV

hydrostatic pressure, p = gh

Gm
gravitational potential, =
r

displacement of particle in s.h.m., x = x0 sin t

velocity of particle in s.h.m., v = v0 cos t

= x
2
0 x2
resistors in series, R = R1 + R2 +

resistors in parallel, 1/R = 1/R1 + 1/R2 +

Q
electric potential, V =
4 r

alternating current/voltage, x = x0 sin t

transmission coefficient, T exp(2kd)

8 2 m U E
where k =
h2

radioactive decay, x = x0 exp (t)

0.693
decay constant, =
t1
2
4

1 (a) State the two conditions necessary for a body to be in equilibrium.

1.

2.

[2]

(b) Fig. 1.1 shows a uniform beam AB of weight 2700 N and length 6.0 m which has been
hoisted into the air by a crane. The lengths of the ropes AC and BC are both 6.0 m. The
tension in AC is T1 and that in BC is T2.

A worker, of weight 900 N, was sitting on the beam when it was hoisted and now finds
himself hanging onto the beam in mid-air. The worker is at point W where AW = 4.0 m and
BW = 2.0 m.
C

A M
W
G B

Fig. 1.1

When the beam and the worker are stationary, the beam makes an angle to the
horizontal. The point M is the mid-point of the beam and the point G on the beam is the
position of the centre of gravity of the beam and the worker.

(i) Explain why the point G must lie directly below C.

[2]
5

(ii) Calculate the distances MG and WG.

MG = m

WG = m [3]

(iii) If the angle is 2.8, determine the magnitudes of the tensions T1 and T2.

T1 = N

T2 = N [3]
6

2 (a) A transverse progressive wave travels from left to right. The variation with distance x of
the displacement y of the transverse wave is shown in Fig. 2.1.

Fig. 2.1
The frequency of the wave is 15 Hz.
For this wave, use Fig. 2.1 to determine
(i) the phase difference between the points labelled A and B,

phase difference = rad [1]


(ii) the speed of the wave.

speed = m s1 [2]

(b) The period of vibration of the wave is T. The wave moves forward from the position shown
in Fig 2.1 for a time 0.25T. On Fig. 2.1, sketch the new position of the wave. [2]

(c) A stretched string is used to form a stationary wave. Part of this wave, at a particular
instant, is shown in Fig. 2.2.

Fig. 2.2

The points on the string are at their maximum displacement.


(i) State the phase difference between the particles labelled X and Y.

phase difference = rad [1]


(ii) The period of vibration of this wave is . On Fig. 2.2, sketch the stationary
wave 0.25 after the instant shown in Fig. 2.2. [1]
7

3 Two spheres Q and R, carrying charges of 3.8 nC and +7.6 nC respectively, are placed
0.060 m apart from each other, as shown in Fig. 3.1. The diameter of the spheres is negligible
compared to the distance between them.

Q R
charge 3.8 nC charge +7.6 nC

0.060 m

Fig. 3.1 (not drawn to scale)

(a) On Fig. 3.1, draw electric field lines to represent the electric field in the region between Q
and R. [2]

(b) Deduce an expression for the potential V at an arbitrary point a distance x from Q along
the horizontal line joining Q and R.

[1]

(c) On Fig. 3.2, sketch the variation with x of the potential V.

0
0 0.01 0.02 0.03 0.04 0.05 0.06 x/m

Fig. 3.2 [2]


8

(d) A model for the hydrogen atom consists of a proton, considered to be a point charge +e,
at the centre of a spherical electron cloud of radius R. The charge density of the cloud is
uniform, and its total charge is e. The charge density is the charge divided by the volume
of the space occupied by the charge.

(i) On Fig. 3.3 below, sketch graphs to show the variation with distance r from
the proton, the electric fields

1. Ep due to the proton,

2. Ee due to the electron cloud.

Indicate R on the horizontal axis. Label your graphs. [3]

Fig. 3.3

(ii) Explain why a point charge outside the hydrogen atom experiences no electric
force.

[1]
9

4 (a) A uniform wire XY of radius 0.50 mm has a resistivity of 1.0 106 m. The total
resistance of the wire is 1.5 . Show that the length of wire XY is 1.2 m.

[2]

(b) The wire XY in part (a) is connected to the circuit shown in Fig. 4.1.
Cell A has an e.m.f of 6.0 V and an internal resistance of 1.0 .
Cell B has an e.m.f. of 1.5 V and an internal resistance of 1.0 .
R1 and R2 have resistances 2.0 and 5.0 respectively.
The thermistor has a resistance of 50 at room temperature.

cell A
R1

S
X Y

cell B thermistor

R2 switch K

Fig. 4.1

(i) Determine the potential difference VXY across wire XY when there is no current in
the galvanometer.

VXY = V [1]
10

(ii) Determine the length of XS such that there is no current in the galvanometer when
1. switch K is open.

length of XS = m [1]

2. switch K is closed.

length of XS = m [3]

(iii) State and explain the change (if any) in your answer to (b)(ii)2 when the room
temperature is increased.

[2]
11

5 (a) State what is meant by the binding energy of a nucleus.

[1]

(b) On Fig. 5.1 below, sketch the variation with nucleon number of the binding energy (B.E.)
per nucleon.
B.E. per nucleon

0
nucleon number
Fig. 5.1 [1]

(c) Using your sketch in Fig. 5.1, explain why fusion of nuclei having high nucleon numbers is
not associated with a release of energy.

[2]

(d) (i) Calculate the binding energy per nucleon, in MeV, of 35


17 Cl, using the following data:

Rest mass of 35
17 Cl nuclide = 34.96885 u
Rest mass of proton = 1.00728 u
Rest mass of neutron = 1.00866 u

B.E. per nucleon of 35


17 Cl = MeV [2]

(ii) The binding energy per nucleon of another isotope of chlorine, 37


17 Cl, is 8.35 MeV.
State and explain which of these two isotopes of chlorine is more stable.

[1]
12

6 The hull is the watertight body of a boat. The load waterline length LWL refers to the horizontal
length of a hull at the waters surface when a boat is carrying a normal load. It is a significant
factor in establishing how fast a boat can go.

A displacement hull refers to a hull that travels through the water rather than on top of it. A
displacement hull displaces significant amounts of water as it moves along, creating two series
of waves in so doing one at the bow and another at the stern. Hull speed vhull is the speed at
which the wavelength of the bow wave of the boat is equal to its load waterline length.

As boat speed increases, the size of the bow wave increases, and therefore so does its
wavelength. When hull speed is reached, a boat will appear trapped in the trough of its bow
wave. If the boat maintains its hull speed, its bow and stern are well supported by the crests of
their respective waves. It can continue moving forward efficiently as its motion is horizontal.

(a) (i) On Fig 6.1, sketch the waveform of the wave along the hull of a boat as the boat
moves through a calm sea at hull speed.

vhull
bow stern

Fig. 6.1 [1]

(ii) Deduce why it is inefficient for the boat to move above its hull speed.

[2]
13

(b) The variation with LWL of vhull for a typical sailboat is shown in Fig. 6.2.

5.0

4.5

vhull / m s1

4.0

3.5

3.0

2.5

2.0
4.0 6.0 8.0 10.0 12.0 14.0 16.0
LWL / m
Fig. 6.2

The relationship between vhull and LWL is thought to follow the expression

vhull = k LWLn

where k and n are constants.

Data from Fig. 6.2 are used to obtain values of lg vhull and lg LWL. These are plotted on the
graph of Fig. 6.3.
14

lg (vhull / m s1)
0.70

0.65

0.60

0.55

0.50

0.45

0.40

0.35
0.6 0.7 0.8 0.9 1.0 1.1 1.2
lg (LWL / m)
Fig. 6.3

(i) Use Fig. 6.2 to determine lg vhull for LWL = 12.0 m.

lg vhull = [1]

(ii) On Fig. 6.3,


1. plot the point corresponding to LWL = 12.0 m,
2. draw the best fit line for all the points. [2]

(iii) Determine the gradient of the line drawn in (b)(ii)2.

gradient = [2]
15

(iv) Using your answer in (b)(iii), determine the magnitude of k.

magnitude of k = [3]

(v) Determine the SI base units for k.

SI base units = [2]

(c) The Formidable-class frigates are the latest warships to enter into service with the
Republic of Singapore Navy. They have a load waterline length of 110 m and a maximum
speed of 15 m s1.
(i) Use your answers in part (b) to determine the hull speed of a Formidable-class
frigate.

hull speed = m s1 [2]

(ii) Suggest why the Formidable-class warship is designed to achieve a higher


maximum speed than its hull speed.

[1]

(d) As the bow wave increases the drag on a ship, large ships such as cargo ships and oil
tankers often have a protruding bulb at the bow of the ship just below the waterline, known
as a bulbous bow. A bulbous bow is shown in Fig. 6.4.

bow of ship

bulbous bow

Fig. 6.4

Suggest how bulbous bows help ships to achieve better fuel efficiency than similar
vessels without them.

[2]
16

7 A spark plug is a device for delivering electric current from an ignition system to the combustion
chamber of a spark-ignition engine to ignite the compressed fuel/air mixture by an electric
spark. Gap adjustment is crucial in proper engine operation. A narrow gap might prevent a
spark from developing fully to ignite the fuel-air mixture. A gap that is too wide might prevent a
spark from firing at all. A student wishes to investigate how a spark may be produced under
different conditions.

Fig. 7.1 shows a typical spark plug.

air gap

Fig. 7.1

You are required to design an experiment in the laboratory to investigate how the pressure of
the air affects the minimum voltage required to generate a spark across a gap between two
electrodes.

The following equipment is available: a pressure chamber, a pressure gauge, an air pump, a
variable high DC voltage supply, two electrodes, a voltmeter and any other equipment normally
available in a school laboratory.

You should draw a labelled diagram to show the arrangement of your apparatus. In your
account you should pay particular attention to
(a) the identification and control of variables,
(b) the equipment you would use,
(c) the procedure to be followed,
(d) how the pressure of the air and minimum voltage required would be determined,
(e) any precautions that would be taken to improve the accuracy and safety of the
experiment.

Diagram
17
18

[12]

END OF PAPER
Centre Number Index Number Name Class

RAFFLES INSTITUTION
2013 Preliminary Examination
PHYSICS
Higher 2
9646 / 03
Paper 3 23 September 2013
2 hours
Candidates answer on the Question Paper.
No Additional Materials are required.

READ THESE INSTRUCTIONS FIRST

Write your Centre number, index number, name and class in the spaces provided at the top
of this page.
Write in dark blue or black pen.
You may use a soft pencil for any diagrams, graphs or rough working.
Do not use staples, paper clips, highlighters, glue or correction fluid.

Section A
Answer all questions.

Section B
Answer any two questions.

You are advised to spend about one hour on each section.


Write your answers in the spaces provided in this booklet.
At the end of the examination, enter the numbers of the Section B questions you have
answered in the grid below.
The number of marks is given in brackets [ ] at the end of each question or part question.

For Examiners Use


1 / 11
2 / 6
Section A 3 / 7
4 / 6
5 /10
/20
Section B
/20
Total /80

This booklet consists of 19 printed pages including the cover page.


2

Data

speed of light in free space, c = 3.00 x 108 m s1

permeability of free space, 0 = 4 x 107 H m1

permittivity of free space, 0 = 8.85 x 1012 F m1


(1 / (36 )) x 109 F m1

elementary charge, e = 1.60 x 1019 C

the Planck constant, h = 6.63 x 1034 J s

unified atomic mass constant, u = 1.66 x 1027 kg

rest mass of electron, me = 9.11 x 1031 kg

rest mass of proton, mp = 1.67 x 1027 kg

molar gas constant, R = 8.31 J K1 mol1

the Avogadro constant, NA = 6.02 x 1023 mol1

the Boltzmann constant, k = 1.38 x 1023 J K1

gravitational constant, G = 6.67 x 1011 N m2 kg2

acceleration of free fall, g = 9.81 m s2


3

Formulae

uniformly accelerated motion, s = ut 21 at 2

v2 = u 2 2as

work done on/by a gas, W = pV

hydrostatic pressure, p = gh

Gm
gravitational potential, =
r

displacement of particle in s.h.m., x = x0 sin t

velocity of particle in s.h.m., v = v0 cos t

= x
2
0 x2
resistors in series, R = R1 + R2 +

resistors in parallel, 1/R = 1/R1 + 1/R2 +

Q
electric potential, V =
4 r

alternating current/voltage, x = x0 sin t

transmission coefficient, T exp(2kd)

8 2 m U E
where k =
h2

radioactive decay, x = x0 exp (t)

0.693
decay constant, =
t1
2
4

Section A

Answer all the questions in this Section.


It is recommended that you spend about one hour on this Section.

1 A certain planet has a radius of 1150 km. Fig. 1.1 below shows the variation with the distance r
from the centre of this planet, of the gravitational potential near it.
r / 103 km
0 1.0 2.0 3.0 4.0 5.0 6.0
0

0.20

/MJ kg1 0.40

0.60

0.80

Fig. 1.1

(a) Explain why gravitational potential has a negative value.

[2]

(b) (i) On Fig. 1.1, draw a tangent to the graph at r = 2500 km. [1]

(ii) The gradient of this tangent represents the magnitude of a particular vector
quantity. State what this physical quantity is.

[1]

(iii) Calculate the gradient of this tangent and hence state the magnitude of the
physical quantity that you have identified in (b)(ii), together with its S.I. unit.

magnitude and unit of physical quantity = [3]


5

(c) Use the graph of Fig. 1.1 to determine the escape velocity for an object at the surface of
the planet.

escape velocity = m s1 [2]

(d) An object of mass 20.0 kg is projected from the surface of the planet and reaches a point
3500 km from the centre of the planet at the highest point in its trajectory. Use the graph
of Fig. 1.1 to determine the total energy (gravitational potential energy plus kinetic
energy) of the object.

total energy = J [2]


6

2 Fig. 2.1 below shows the variation of force F with displacement x from a fixed point of a body
moving horizontally in the absence of frictional forces.

F/N

+ 18.4

+0.350 x/m
0.350 0

18.4

Fig 2.1

(a) Explain how it may be deduced that the motion is simple harmonic.

[1]

(b) Calculate the total energy of the body.

total energy = J [1]

(c) Determine the displacement of the body when its kinetic energy is equal to its potential
energy.

displacement = m [2]

(d) If frictional forces cannot be ignored, sketch on Fig 2.1 the variation with x of F for one
cycle of the motion starting from x = +0.350 m. [2]
7

3 (a) Explain what is meant by an ideal gas.

[1]

(b) State in words how the temperature of an ideal gas is related to the energy of the
molecules of the gas.

[1]

(c) A plasma is a mixture of gas atoms, gas ions and electrons, all in thermal equilibrium. In a
certain hydrogen plasma, the hydrogen atoms, hydrogen ions (protons) and electrons can
be assumed to behave like the molecules of a mixture of three ideal gases. The
root-mean-square speed of the hydrogen ions in the plasma is found to be 6.0 104 m s1.

(i) Determine the root-mean-square speeds of


1. the hydrogen atoms,
2. the electrons,
in the plasma.

r.m.s. speed of hydrogen atoms = m s1

r.m.s. speed of electrons = m s1 [3]

(ii) Determine the temperature of the plasma.

temperature of plasma = K [2]


8

4 (a) It is possible to use two separate oscillators feeding two loudspeakers to demonstrate
interference of sound. It is not possible to use two filament lamps, however similar, to
produce interference of light. Explain this difference.

[1]

(b) Two identical loudspeakers are driven by the same oscillator of frequency 200 Hz. The
loudspeakers are located on a vertical pole a distance of 4.00 m from each other.
A man walks straight towards the lower loudspeaker in a direction perpendicular to the
pole as shown in Fig. 4.1.

Loudspeaker

4.00 m

9.30 m Loudspeaker

Fig. 4.1

(i) Determine whether the man will hear a minimum or a maximum in sound intensity
when he is 9.30 m from the lower speaker.
(Take the speed of sound to be 330 m s1 and ignore any sound reflection from the
ground.)

[3]

(ii) State two changes that can be made to the set-up in Fig 4.1 in order to increase the
number of intensity fluctuations detected by the man as he walks towards the pole.

1.

2.

[2]
9

5 Fig. 5.1 shows a U-shaped magnet of length L whose magnetic field between its poles can be
assumed to be a horizontal uniform field.

Fig. 5.1
A current balance consisting of a rectangular loop PQRS is used to measure the flux density of
the magnetic field between the poles of the U-shaped magnet. The top view of the loop and
magnet is shown in Fig. 5.2.
The arm PQ of the loop is placed between the poles of the magnet and perpendicular to the
magnetic field of the magnet. The loop is pivoted horizontally so that it is initially in equilibrium.
A current is sent through one side of the loop, entering and leaving through leads connected to
the pivots.
I

Q R
I
pivot

L paper insulator
strips

I
pivot

P I S

I
Fig. 5.2 Plan (Top) View
The current I in PQ results in a force F that pushes it downwards into the page. Arm RS is then
loaded with small paper strips to restore equilibrium.

(a) On Fig. 5.2, label the poles of the magnet N or S to indicate the direction of the field
needed to produce a downward force on arm PQ. [1]

(b) The paper used for the strips has a mass per unit area of 80 g m 2. Each strip is 3.0 cm
long and 4.0 mm wide. Calculate the total mass of the five strips.

total mass of the paper = g [1]


10

(c) If L = 5.0 cm, I = 1.2 A and the distance of the pivots from the arm PQ is double that of
their distance from the arm RS, determine the magnetic flux density of the magnet.

magnetic flux density = T [3]

(d) The current balance is removed. A flat circular coil, of diameter 1.5 cm and 25 turns, is
now placed between the poles of the magnet, with its plane perpendicular to the magnetic
field. It is then moved away from the magnet to a region of negligible magnetic field in a
time of 0.040 s.

(i) Use Faradays law of electromagnetic induction to explain why an e.m.f. is induced
in the coil during this movement of the coil.

[2]

(ii) Calculate the average value of this induced e.m.f.

average induced e.m.f. = V [3]


11

Section B

Answer two questions from this section. Each question carries 20 marks.
It is recommended that you spend about one hour on this section.

6 (a) (i) Define acceleration.

[1]

(ii) State Newtons second law of motion.

[2]

(b) The variation with time t of vertical speed v of a parachutist falling from an aircraft is
shown in Fig. 6.1. The mass of the parachutist is 95 kg.

Fig. 6.1

(i) Calculate the distance travelled by the parachutist in the first 3.0 s of the motion.

distance = m [2]
12

(ii) Explain the variation of the resultant force acting on the parachutist from t = 0 (point
A) to t = 15 s (point C).

[3]

(iii) Describe the changes to the frictional force on the parachutist

1. at t = 15 s (point C),

[1]

2. between t = 15 s (point C) and t = 22 s (point E).

[2]

(iv) Calculate, for the parachutist between t = 15 s (point C) and t = 17 s (point D),

1. the average acceleration,

acceleration = m s2 [2]

2. the average frictional force.

frictional force = N [3]


13

(v) The frictional force on an object falling through air at high speeds v is given by Dv2,
where D is a constant. The value of D depends on the characteristics of the object
and on the density of the air.

1. Calculate the change in the value of D when the parachutist is at t = 14 s and


at t = 24 s.

change in D = kg m1 [3]

2. Explain why there is a change in the value of D.

[1]
14

7 (a) Define

(i) charge,

[1]

(ii) the coulomb,

[1]

(iii) potential difference,

[1]

(iv) the volt.

[1]

(b) A student sets up a circuit as shown in Fig. 7.1. Both batteries in the circuit have an
internal resistance of 0.30 . The light bulbs are identical and are marked 6.0 V, 10 W.
Switch S

A
0.40

E, 6.0 V,
0.30 0.30

Fig. 7.1

The switch S is closed for 5.0 minutes during which both light bulbs are lit and working
normally. The ammeter reads 0 A.

(i) Calculate
1. the resistance of each light bulb,

resistance = [1]
15

2. the current in each light bulb,

current = A [1]

3. the total amount of charge that passes through the two light bulbs during the
5.0 minutes,

charge = C [2]

4. the total energy supplied to the two light bulbs during the 5.0 minutes,

energy = J [1]

5. the current in the battery E.

current = A [1]

(ii) Show that the e.m.f. of the battery E is 8.3 V.

[2]

(iii) The student opens switch S. Determine the reading on the ammeter when the
switch is open.

current = A [1]
16

(c) The student modifies the circuit by adding a voltmeter to the circuit and placing the light
bulbs in series with one another as shown in Fig. 7.2.
Switch S

0.40 A

8.3 V, 6.0 V,
0.30 0.30

Fig. 7.2

The voltmeter reads 6.5 V when switch S is closed.

(i) Indicate clearly on Fig. 7.2. the direction of current flow through the 6.0 V
battery. [1]

(ii) Hence determine the current

1. through the 6.0 V battery,

current = A [2]

2. through the light bulbs.

current = A [1]

(d) One of the two circuits in Fig. 7.1 and Fig. 7.2. is in fact a simplified circuit for car
headlights. Suggest which circuit is likely to be the one. Give two reasons for your
answer.

[2]

(e) Suggest a reason why a two power source circuit is more advantageous than having a
single power source in the application of using them as car headlights.

[1]
17

8 (a) (i) State what is meant by the photoelectric effect.

[1]

(ii) Describe two experimental observations associated with this effect and explain
how each deviates from predictions of the classical wave theory.

[4]

(b) In a photoelectric emission experiment, light of wavelength 420 nm was shone on a metal
surface of work function energy of 2.0 eV so that an area of 25 mm 2 was illuminated. A
saturated photocurrent of 4.8 x 1010 A was observed.
Determine

(i) the threshold frequency,

threshold frequency = Hz [1]

(ii) the stopping potential,

stopping potential = V [2]


18

(iii) the rate of emission of photoelectrons,

rate of emission of photoelectrons = s1 [1]

(iv) the intensity of the light source, assuming that 1 in 2500 photons succeeds in
ejecting an electron from the surface.

intensity = W m2 [3]

(c) The scanning tunnelling microscope (STM) is an instrument which makes use of quantum
tunnelling to detect changes in the surface structure on the atomic scale.

(i) Explain what is meant by


1. potential barrier,

[1]

2. quantum tunnelling.

[1]

(ii) Briefly describe how the STM operating in the constant height mode is used to
obtain atomic-scale images of surfaces.

[2]
19

(iii) When the tip of a STM probe is set at a small distance from the sample, its
tunnelling current is proportional to the transmission coefficient. The work function
energy (U E) of the sample is 4.0 eV. Determine the ratio of the current when the
STM tip is 0.50 nm above a surface to the current when it is 0.75 nm above the
surface.

ratio = [4]

************ END OF PAPER ************


Raffles Institution 2013 H2 Physics Preliminary Examination
Paper 1 Answer Key

Qn Ans Qn Ans
1 C 21 D
2 C 22 B
3 B 23 B
4 D 24 A
5 C 25 C
6 D 26 C
7 D 27 B
8 A 28 C
9 C 29 B
10 A 30 A
11 D 31 B
12 D 32 B
13 B 33 A
14 B 34 C
15 D 35 A
16 D 36 D
17 A 37 B
18 A 38 A
19 C 39 C
20 B 40 A

Paper 2 Suggested Solutions:


1 (a) The two conditions necessary for a body to be in equilibrium are:
1. The resultant force on the object is zero, i.e., F = 0.
2. The resultant torque on the object about any axis is zero, i.e., = 0.

(b) (i) The combined weight of the beam and the worker can be considered to be acting at
G. Considering moments about C, moments due to T1 and T2 about C is zero as
their lines of action pass through C. Hence for the resultant moments about C to be
zero, the line of action of the combined weight of the beam and worker must pass
through C as well so that the moment of the combined weight about C is zero.
Hence the vertical line through G must pass through C.

(ii) Considering moments due to the weight of beam, Wb, and that of the worker, Ww ,
about G,
Wb (MG) = Ww (WG)
(2700)(MG) = (900)(WG)
WG = 3 MG
MW = AW AM = 4.0 3.0 = 1.0 m
MW = MG + GW = MG + 3 MG = 1.0 m
MG = 0.25 m
WG = 0.75 m
2

(iii) Method 1:
For horizontal equilibrium,
T1 cos (60.0o 2.8o) = T2 cos (60.0o + 2.8o)
cos 62.8o
T1 o 2
T
cos 57.2
T1 0.8438 T2 (1)
For vertical equilibrium,
T1 sin 57.2o + T2 sin 62.8o = 2700 + 900 (2)

Substituting equation (1) into equation (2) and solving for T2 ,


T2 = 2250 N (3 s.f.)
T1 = 0.8438 2250 = 1900 N (3 s.f.)

Method 2:
Taking moments about point B,
(component of T1 perpendicular to beam)(AB)
= (combined weight of beam and worker)(horizontal distance of G from B)
(T1 sin 60.0o)(6.0) = (3600)(2.75 cos 2.8o)
3600 2.75 cos 2.8o
T1 1900 N (3 s.f.)
6.0 sin 60o
Taking moments about point A,
(component of T2 perpendicular to beam)(AB)
= (combined weight of beam and worker)(horizontal distance of G from A)
(T2 sin 60.0o)(6.0) = (3600)(3.25 cos 2.8o)
3600 3.25 cos 2.8o
T2 2250 N (3 s.f.)
6.0 sin 60o

xAB 0.30 m 2 3 rad or 0.75 rad or 2.36 rad


2 (a) (i) 2
0.80 m 4

(ii) v f 15 Hz 0.80 m 12 m s1

(b)

Fig. 2.1
Correct sketch with peak moved to the right; wave profile moved by the correct distance
of 0.20 m.
3

(c) (i) Zero rad. All points between adjacent nodes oscillate in phase with each other;
they are rad out of phase with all points in the next half-wavelength section.

(ii)
t = t0

t = t0 + 0.25

Fig. 2.2

At the time 0.25 after the instant shown in Fig. 2.2, the individual traveling waves
are rad out of phase, and each element of the medium is passing through the
equilibrium position in its simple harmonic motion. The result is zero displacement
for elements at all values of x; that is, the wave pattern is a straight line.

3 (a)

q1 is 3.8 nC, q2 is +7.6 nC


Fig. 3.1

3.8 109 7.6 109


(b) V
4o x 4o 0.060 x
4

(c)
V

0
0 0.01 0.02 0.03 0.04 0.05 0.06 x/m

Fig. 3.2

(d) (i) E

e
4o R 2
Ep

r
R

Ee
e

4o R 2

Fig. 3.3

(ii) The hydrogen atom is electrically neutral hence it does not exert any electric force
on an external point charge.
OR: Outside of the hydrogen atom, the electric fields due to the proton and electron
cloud cancel each other, as they are equal in magnitude and opposite in directions.

L
4 (a) R
A

1.5
1.0 10 L 6

0.50 10
2
3

L 1.178 1.2 m (2 s.f.) (shown)

1.5
(b) (i) VXY 6.0
1.5 1.0 2.0
2.0 V
5

(ii) 1. VXS 1.5 V


XS VXS

XY VXY
XS 1.5

1.2 2.0
XS 0.90 m

2. VXS 1.5 i 1.0 i 55


i 0.026786
VXS 1.4732
XS VXS

XY VXY
XS 0.884 m (3 s.f.)

(iii) When the room temperature increases, resistance of thermistor decreases. The
effective resistance of the bottom circuit (where cell B is in) decreases. Hence
current flowing in same circuit increases and terminal potential difference across
cell B decreases. Therefore, the value of XS should be smaller.

5 (a) The binding energy of a nucleus is the work which must be done on the nucleus to
separate it completely into its constituent nucleons.

(b)
B.E. per nucleon

0
56 nucleon number
Fig. 5.1 [1]

(c) For nuclei having high nucleon numbers, the binding energy per nucleon decreases with
larger nucleon numbers. When two such nuclei fuse together, they will produce a
daughter nucleus which has an even larger nucleon number and smaller binding energy
per nucleon. This means that the daughter nucleus is less stable than the parent nuclei.
The total binding energy of the products is less than that of the initial nuclei, hence there is
an increase in the total mass of the system, and energy has to be supplied for such a
reaction to take place.

(d) (i) B.E. per nucleon


2
1.00728 17 (1.00866 18) 34.96885 1.66 1027 3 108

35 1.6 1013
8.29 MeV

(ii) 37
17 Cl is more stable, as it has a larger binding energy per nucleon.
6

6 (a) (i) vhull


bow stern

Fig. 6.1
(ii) Above hull speed, the wavelength of the bow increases such that the trough is at
the stern. The bow will be tilted upwards, resulting in a larger drag. The driving
force of the boat engine is also no longer horizontal, leading to inefficiency.

(b) (i) vhull = 4.35 m s1


lg vhull = 0.638

(ii)
lg (vhull / m s1)
0.70
(1.180, 0.690)

0.65

0.60

0.55

0.50

0.45

(0.640, 0.420)
0.40

0.35
0.6 0.7 0.8 0.9 1.0 1.1 1.2
lg (LWL / m)
Fig. 6.3
7

(iii) Gradient = 0.690 0.420 0.500


1.180 0.640

(iv) lg vhull = n lg LWL + lg k


Using (0.640, 0.420),
0.420 = 0.500 (0.640) + lg k
lg k = 0.100
k = 1.26

1
(v) Base units of k = m s = m0.500
m0.500

(c) (i) vhull = k LWLn = 1.26 1100.500 = 13.2 m s1

(ii) Warships are designed for combat operations. Having the highest possible
maximum speed has priority over moving efficiently.

(d) At cruising speed, the bulbous bow generates a wave that superposes destructively with
the bow wave and reducing the amplitude of the bow wave. The drag due to the bow
wave is reduced, increasing fuel efficiency.

7 Diagram
pressure
gauge

pressure
chamber

variable V electrodes
D.C. Supply

air
pump
8

1. Problem Definition
To investigate how the pressure of the air affects the minimum voltage required to generate a
spark across a gap between two electrodes.
Independent Variable:pressure of the air
Dependent Variable: minimum voltage required to generate a spark across a gap between two
electrodes
Controlled Variables: air gap between the electrodes, type of gas, material and shape of the
electrodes.

2. Methods of Data Collection


Procedure:
1. Set up the apparatus as shown in the diagram above.
2. Adjust the air pressure of the pressure chamber to a suitable pressure using the air pump.
3. Measure and record the pressure P of the air in the chamber, using a pressure gauge.
4. Switch on the variable d.c supply.
5. While keeping the pressure constant, slowly increase the variable d.c. supply until a spark is
seen.
6. Measure and record the voltage V applied to the electrodes, by reading off the reading on
the voltmeter.
7. Repeat the steps 2-6 for different pressures, P, by pumping more air into the pressure
chamber with the use of an air pump.
8. Distance between electrodes is kept constant by measuring the distance between the
electrodes using a ruler.
Type of gas is kept constant by using the same type of air through the air pump.
Material and shape of the electrodes are kept constant by using the same apparatus
throughout the entire experiment.

3. Method of Data Analysis


Assume that
V = k Pn
where V is the minimum voltage required to generate a spark across a gap between two
electrodes, P is pressure of the air, and k and n are constants
Taking lg to both sides, lg V = n lg P + lg k
Plot a graph of lg V against lg P with n as the gradient and lg k as the y-intercept.

4. Safety Precautions
1. Use goggles to view spark {possible danger of uv radiation}.
2. Use safety screen around pressure chamber {in case of explosion/implosion}.
3. Wear gloves to protect against high voltage d.c. supply

5. Additional Details
1. Perform experiment in the dark {hence enhance visibility of spark}
2. Repeat measurement of voltage V, and taking the average
3. If the gap is too wide or too small to see the spark, adjust the distance between the
electrodes accordingly to ensure that sparks can be seen.
9

Paper 3 Suggested Solutions:

1 (a) Gravitational potential at infinity is defined as being zero. Work done by an external force
is negative as the direction of the force is opposite to the direction of displacement of the
small mass from infinity to that point.

(b) (i)
r / 103 km
0 1.0 2.0 3.0 4.0 5.0 6.0
0

0.20

/MJ kg1 0.40

0.60

0.80

Fig. 1.1

(b) (ii) Gravitational field strength

[ 0.04 ( 0.66)] 106


(iii) Gradient = 3 3
0.133 N kg1
[4.70 0.04] 10 10

(c) (Ep + Ek)initial = (Ep + Ek)


1
m mv 2 0 0
2
v 2 2
At the surface of the planet, = 0.77 106 J kg1
v = 1240 m s1

(d) Total energy (TE)


= (Ep + Ek)3500km
= 0.26 106 20 0 = 5.2 106 J
10

2 (a) The graph shows that F x , F (constant) x which agrees with the defining
equation of simple harmonic motion (SHM)

(b) The total energy in the SHM is the area under the F-x graph from x = 0 to x = x0
1 1
Therefore, the total energy is Fmax . x0 18.4 0.350 3.22 J
2 2
1 2
(c) The potential energy of the body at any point in the SHM is kx , and the maximum
2
1
potential energy is also the total energy, which is kx02 .
2

When both the kinetic energy and the potential energy are equal, each must be half the
total energy. Hence
1 2 1 1
kx kx0 2
2 22
1 x 0.350
x 2 x0 2 or x 0 0.248 m
2 2 2
(d)
F/N

+ 18.4

+0.350 x/m
0.350 0

18.4

Fig 2.1

3 (a) An ideal gas is a hypothetical gas which obeys the ideal gas equation pV = nRT for all
pressures, temperatures and volumes.

(b) The mean kinetic energy of the molecules in an ideal gas is directly proportional to the
thermodynamic temperature.

(c) (i) Since all three are in thermal equilibrium,


1 1 1 3
mH cH2 mH cH2 me ce2 kT
2 2 2 2
1. Since mH mH , thus cH 6.0 10 4 m s 1
1
6.0 10
2
2. For proton, its kinetic energy is 1.67 1027 4
3.006 1018 J
2
1
3.006 1018
2

9.11 10 31 ce2

ce 2.6 106 m s1
11

3
(ii) 3.006 1018 kT
2
3

3.006 1018 1.38 10 23 T
2
5
T 1.5 10 K

4 (a) To obtain an observable interference pattern, the sources must be coherent. The two
loudspeakers can be made coherent by connecting them to the same signal generator,
but the two filament lamps can never be coherent sources since the light is produced in a
random manner and there is no way to control the phase of the light waves emitted.

(i) The distance of the upper loudspeaker from the man is 9.302 4.002 10.124 m
the path difference between the waves reaching the man is
10.124 9.30 0.8237 m
330
The wavelength 1.65 m,
200
0.8237
the path difference is 0.5
1.65
This means that the waves interfere destructively and the man will hence detect a
minimum of sound intensity.

(ii) Two changes to the set-up that can be made are:


1. Increase the frequency of the sound
2. Increase the separation between the two loudspeakers

5 (a)
I

Q R
I
pivot

L N S paper insulator
strips

I
pivot

P I S
Fig. 5.2 Plan (Top) View
I

(b) Total mass of paper = 5 (3.0 102) (4.0 103) 80 = 0.048 g


12

(c) Let x be the distance of arm RS from the pivots.


Taking moments about the pivots,
Fm (2x) = mgx
2BIL sin 90o = mg
mg (0.048 103 ) 9.81
B 3.924 103 T 3.9 103 T (2 s.f.)
2I L 2 1.2 (5.0 102 )

(d) (i) When the coil is placed between the poles of the magnet with its plane
perpendicular to the magnetic field, the magnetic field lines pass through the plane
of the coil and there is magnetic flux through the coil (and hence there is magnetic
flux linkage with the coil). This magnetic flux (and hence flux linkage) reduces to
zero when the coil is removed to a region of negligible magnetic field. According to
Faradays Law, an e.m.f. will be induced in the coil whenever there is a change in
the magnetic flux linkage in a circuit or coil and the magnitude of this induced e.m.f.
is proportional to the rate of change of flux linkage.

(NBA cos )
(ii) Induced e.m.f. E
t

2
25 (3.924 10 3 cos 0o 0) 1.5 10
2


0.040 4

4.3 10 4 V (2 s.f.)

6 (a) (i) Acceleration is the rate of change of velocity.

(ii) Newtons second law of motion states that the rate of change of momentum of a
body is proportional to the resultant force acting on it and occurs in the direction of
the force.

(b) (i) distance is represented by the area under graph


distance = () (29.5) (3.0) = 44.3 m
(ii) resultant force = (weight frictional force)
Since frictional force increases with speed (or speed2 for high speeds), resultant
force decreases.
At the start, the resultant force is constant as frictional force is negligible.
At the end, the resultant force is zero as frictional force = weight.
(iii) 1. The frictional force increases (drastically).

2. Frictional force is constant between points C and D and then decreases


between point D and E.

(iv) 1. (v 2 v1 ) 20.0 50.0


a 15 m s1
(t2 t1 ) (17.0 15.0)

2. downward direction as positive <f>


W <f> = m<a>
W = (95)(9.81) N
<f> = (95)(9.81) 95 (15) = 2400 N (2360 N) (2357 N) <a>
W
13

mg
(v) 1. mg Dv 2 D
v2
v 50.0 m s1 at t = 14 s
v 8.0 m s1 at t = 24 s
1 1 1 1
change in D mg 2
2
95 9.81 2
2
14 kg m 1
8.0 50.0 8.0 50.0

2. An increase in the cross-sectional area measured in a plane perpendicular to


its velocity, after the parachute opens or a change in shape after the
parachute opens.

7 (a) (i) Fundamental property of matter that can be either positive or negative and gives
rise to electrical force.

(ii) The quantity of electric charge that passes through a given point in a circuit when a
current of 1 ampere flows for 1 second.

(iii) Potential difference between two points in a circuit is defined as the energy
converted from electrical energy to other forms of energy when unit charge passes
from one point to the other.

(iv) One volt is defined as the potential difference between two points in a circuit in
which one joule of energy is converted when one coulomb of charge passes from
one point to the other.

V2
(b) (i) 1. P
R
6.0
2

R 3.6
10

2. P IV
10
I 1.67 A
6.0

3. Q I t 1.6667 5.0 60 2 1000 C

4. Total Energy QV
1000 6.0
6000 J

5. I 1.6667 2 3.33 A

(ii) Terminal p.d. of a battery = E I r


E I 0.40 0.30 6.0
E 8.3 V (shown)

(iii) 8.3 6.0 I 0.30 0.40 0.30


I 2.3 A
14

(c)
Switch S

0.40 A

(c)(i) V
6.0 V,
0.30

Fig. 7.2

(ii) 1. 6.0 I 0.30 6.5


I 1.67 A

2. V IR
6.5
I 2 0.903 A
3.6

(d) Fig. 7.1. because:


1. The bulbs should be connected in parallel so that if one bulb is faulty, the other will
still continue work.
2. The current through each bulb is larger, hence the bulbs will be brighter when it is
turned on.

(e) The second power source acts as a backup in case the first power source is not working.

8 (a) (i) Photoelectric effect is the emission of electrons from the surface of a metal when it
is exposed to electromagnetic radiation of sufficiently high frequency.
(ii) Any two of the following observations:
1. For every surface irradiated, there exists a threshold frequency below which no
photoemission occurs regardless of the intensity of the electromagnetic
radiation. This contradicts the classical wave theory, which predicts that
photoelectrons should be emitted for all frequencies.

2. The maximum kinetic energy of the photoelectron is independent of the light


intensity, but dependent on the frequency of the light. This is contrary to the
classical wave theory which predicted that increasing the light intensity should
lead to ejection of photoelectrons with greater speed and hence kinetic energy.

3. There is no time lag between illuminating the metal plate and the emission of
photoelectrons, even at very low intensities. This deviates from the classical
wave theory which predicts that there should be a measurable time lag since
energy is arriving in a continuous manner and a certain amount of time is
needed for the electron to gather enough energy before it gets ejected.
15

(b) (i) hfthreshold



fthreshold
h
2.0 (1.6 10 19 )
34
4.83 1014 Hz
6.63 10

(ii) hf eVs
hf
Vs
e
3 108
(6.63 10 34 )( 9
) 2.0 (1.6 10 19 )
420 10
1.60 10 19
0.960V

(iii) Current, I e ne e ,where ne is the number of photoelectrons per second


Ie 4.8 1010
ne 19
3.0 109 s1
e 1.60 10

(iv) n p 2500ne ,

where n p is the number of photons per second.

P
Intensity, I
A
n p hf n p hc

A A
9 34 8
(2500)(3.0 10 )(6.63 10 )(3 10 )
6 9
(25 10 )(420 10 )
2
0.142 W m

(c) (i) 1. Potential barrier refers to a potential energy distribution of which the energy
height is higher than the incident particles energy.

2. Quantum tunnelling refers to the phenomenon where a particle has a non-zero


probability of existing outside the potential barrier.

(ii) Electrons can tunnel through the empty space barrier between the tip of the STM
and the sample surface. The tunnelling current I decreases exponentially with the
tip-to-surface distance d, so a small change in d will cause a large change in I. In
constant-height mode, the tip travels in a fixed vertical position above the sample
and I varies depending on the local surface electron density of the sample. The
tunnelling current measured at each location on the sample surface constitute the
data set which can be mapped into an atomic-scale image of the surface.
16

(iii) 8 2 m(U E ) 8 2 (9.11 10 31 )(4.0)(1.6 10 19 )


k 1.02336 1010 m1
h2 (6.63 10 34 )2
I T e 2kd
I exp( 2kd1 )
d 0.50 nm
I d 0.75 nm exp( 2kd 2 )
exp[( 2)(1.02336 1010 )(0.50 10 9 )]

exp[( 2)(1.02336 1010 )(0.75 10 9 )]
167
Name Class Index Number

PIONEER JUNIOR COLLEGE


JC2 Preliminary Examination

PHYSICS 8866/01
Higher 1

Paper 1 Multiple Choice


27 September 2013

1 hour

Additional Material: Multiple Choice Answer Sheet

READ THESE INSTRUCTIONS FIRST

Write in soft pencil.


Do not use staples, paper clips, highlighters, glue or correction fluid.
Write your name, class and index number on the Answer Sheet in the spaces provided.

There are Thirty questions on this paper. Answer all questions. For each question there
are four possible answers A, B, C and D.
Choose the one you consider correct and record your choice in soft pencil on the
separate Answer Sheet.

Read the instructions on the Answer Sheet very carefully.

Each correct answer will score one mark. A mark will not be deducted for a wrong
answer.

Any rough working should be done in this booklet.

This document consists of 14 printed pages.

2013/PJC/PHYSICS/8866 [Turn over


2

Data

speed of light in free space, c 3.00 10 8 m s1

elementary charge, e 1.60 10 19 C

the Planck constant, h 6.63 10 34 J s

unified atomic mass constant, u 1.66 10 27 kg

rest mass of electron, me 9.11 10 31 kg

rest mass of proton, m p 1.67 10 27 kg

acceleration of free fall, g 9.81 m s2

Formulae

1 2
uniformly accelerated motion, s ut at
2
v 2 u 2 2as

work done on/by a gas, W pV

hydrostatic pressure, p gh

resistors in series, R R1 R2 ...

resistors in parallel, 1/ R 1/ R1 1/ R2 ...

2013/PJC/PHYSICS/8866
3

1 What are the SI base units of magnetic flux density?

A kg m s2 A1

B kg m2 s2 A1

C kg s2 A1 rad1

D kg s2 A1

2 A boat changes its velocity from 5 m s1 due East to 7 m s1 due South.

What is its change in velocity?

A 9 m s1 at a direction of 54 South of East

B 9 m s1 at a direction of 54 South of West

C 2 m s1 at a direction of 36 East of South

D 2 m s1 at a direction of 36 West of South

3 Four students each made a series of measurements of the acceleration of free fall g. The
table shows the results obtained.

Which student obtained a set of results that could be described as accurate but not
precise?

student results, g / m s2
A 9.81 9.79 9.83 9.82
B 9.73 10.24 9.98 9.25
C 9.45 9.21 8.99 8.76
D 8.45 8.46 8.50 8.41

2013/PJC/PHYSICS/8866 [Turn over


4

4 The acceleration-time graph of an object moving in a straight line is as shown.

acceleration
P

Q S time
0

If the object starts its motion from rest, at which point is the object moving with the
largest speed?

A P B Q C R D S

5 An athlete throws a javelin at an angle of 60 to the horizontal. The javelin leaves the
athletes hand with an initial kinetic energy, E.

Neglecting air resistance, the javelins kinetic energy at its highest point of motion is

A zero.

1
B E.
4

3
C E.
4

D E.

6 A stone is launched at different angles above the horizontal.

If the initial vertical component of the velocity is kept constant, which of the following will
be affected by changes in the horizontal component of the velocity of the stone? You
may ignore the effects of air resistance.

A range only

B maximum height only

C range and maximum height only

D range and time of flight

2013/PJC/PHYSICS/8866
5

7 Which body is in equilibrium?

A A cyclist accelerating along a road.

B A car rolling down a frictionless inclined plane.

C An apple falling freely towards the surface of Earth.

D A block sliding at constant velocity across a tabletop.

8 A mass of 2.0 kg is at rest on a smooth floor. A horizontal stream of water, travelling at


speed 8.0 m s1, strikes it at a rate of 1.0 kg s1 for a duration of 50 s without splashing.

What is the initial acceleration of the mass?

A 0.080 m s2

B 0.16 m s2

C 4.0 m s2

D 8.0 m s2

9 Newtons third law is about the forces of interaction between two bodies.

Which of the following statements relating to the third law is false?

A The two forces act on different bodies.

B The two forces are in opposite direction.

C The two forces are equal in magnitude.

D The two forces are equal in magnitude and opposite in direction. Hence the bodies
are in equilibrium as the forces cancels out.

2013/PJC/PHYSICS/8866 [Turn over


6

10 A heavy uniform beam of length 3L is supported by two vertical cords as shown.

T1 T2

2L L
T2
What is the ratio of the tensions in these cords?
T1
1
A
3

1
B
2

2
C
1

3
D
1

11 A spring obeying Hookes law has an unstretched length of 60 mm and a spring


constant of 500 N m1.

What is the tension in the spring when its overall length is 90 mm?

A 15 N

B 45 N

C 150 N

D 450 N

12 An incompressible liquid of density is contained in a vessel of uniform cross-sectional


area A. The atmospheric pressure is P.

What is the force acting on a horizontal plane of area a situated at a depth d in the liquid?

A AP adg

P dg
B
A a

P dg
C
a

D a P dg

2013/PJC/PHYSICS/8866
7

13 A bullet moving with a speed of 150 m s1 strikes a wooden plank. After passing through
the plank, its speed becomes 125 m s1. Another identical bullet strikes the plank with a
speed of 90 m s1.

What is its speed after passing through the plank if the bullets experience the same
constant retarding force in both cases?

A 25 m s1 B 35 m s1 C 50 m s1 D 70 m s1

14 A force F of 80 N pulls an object of weight 40 N from rest up an incline of 30 through


3.0 m as shown below. The frictional force present is 30 N.

3.0 m

30

Which of the following correctly gives the heat generated due to friction and the gain in
kinetic energy of the mass?

gain in kinetic
heat / J
energy / J

A 210 0

B 90 90

C 90 30

D 210 240

15 The potential energy of a body when it is at point P, a distance x from a reference point
O, is given by U = kx2, where k is a constant.

What is the force acting on the body when it is at P?

A kx in the direction OP

B kx in the direction PO

C 2kx in the direction OP

D 2kx in the direction PO

2013/PJC/PHYSICS/8866 [Turn over


8

16 A sound wave of frequency 1500 Hz propagates from left to right through a gas. The
diagram below shows the positions of some gas molecules at a particular instant of time
where the distance between P and Q is 0.500 m.

P Q

What is the speed of sound in this gas?

A 188 ms1

B 320 ms1

C 375 ms1

D 750 ms1

17 A beam of unpolarised light travelling in the direction shown below falls onto two
polarisers P and Q, arranged so that polarising directions of P and Q are parallel and
vertical. The emergent beam is of maximum intensity I.

Keeping P fixed, Q is then rotated clockwise in its own plane about the direction of the
incident beam.

unpolarised
incident beam emergent beam

P Q

I
If the emergent beam now has an intensity , what is the angle of rotation ?
4

A 27

B 30

C 45

D 60

2013/PJC/PHYSICS/8866
9

18 A small pipe opened at both ends is partly submerged in water as shown below. A tuning
fork vibrating at 850 Hz is placed over the top of the pipe. The pipe is slowly lifted until
the first loud sound is heard when the length of pipe above water is L. The experiment is
repeated with another tuning fork of unknown frequency f and the first loud sound is
heard when the length of pipe above water is 2L.

Given that the speed of sound is 340 m s1 , what is the value of f?

A 213 Hz

B 425 Hz

C 1700 Hz

D 2550 Hz

19 In a Youngs double slit experiment to demonstrate interference of light, light emerging


from one of the slits is reduced in intensity to half that of the other slit.

Which of the following statements is true?

A The fringe pattern will vanish completely.

B The dark lines will increase in intensity and the bright lines will decrease in intensity.

C The bright lines will increase in intensity and the dark lines will decrease in intensity.

D Both bright and dark lines will decrease in intensity.

2013/PJC/PHYSICS/8866 [Turn over


10

20 Four statements about potential difference or electromotive force are listed.

1 It involves changing electrical energy into other forms.

2 It involves changing other energy forms into electrical energy.

3 It is the energy per unit charge to move charge round a circuit.

4 It is the work done per unit charge by the charge moving from one point to
another.

Which of the following statements apply to potential difference and electromotive force?

potential difference electromotive force


A 1 and 3 2 and 4
B 1 and 4 2 and 3
C 2 and 3 1 and 4
D 2 and 4 1 and 3

21 The graph shows the variation with length l of resistance R for two wires X and Y made
from the same material.

R /

l/m

Which of the following statements describes what the graph shows?

A when equal lengths of X and Y are connected in series to a battery,


the power in X is twice the power of Y

B resistivity of X is twice the resistivity of Y

C when equal lengths of X and Y are connected in parallel to a battery,


current in X is twice the current of Y

D cross-sectional area of X is twice the cross-sectional area of Y

2013/PJC/PHYSICS/8866
11

22 In each arrangement of resistors below, the ammeter has a resistance of 2 .

Which arrangement gives the largest reading on the ammeter when the same potential
difference is applied between points P and Q?

A B
1 2 1
A
P Q A
P Q
2

C D
1
1 2 A
A P Q
P Q
2

23 A light-dependent resistor R has resistance of about 1 M in the dark and about 1 k


when illuminated. It is connected in series with a 5 k resistor to a 1.5 V cell of negligible
internal resistance.

1.5 V

5 k

flashing light

The light-dependent resistor is illuminated in a dark room by a flashing light.

Which graph best shows the variation with time t of potential difference V across R?

A B C D

V/V V/V V/V V/V

1.5 1.5 1.5 1.5

0 0 0 0
t t t t

2013/PJC/PHYSICS/8866 [Turn over


12

24 The diagram shows a flat surface with lines OX and OY at right angles to each other.

O X
P

Which current in a straight conductor will produce a magnetic field at O in the direction
OX?

A at P into the plane of the diagram

B at P out of the plane of the diagram

C at Q into the plane of the diagram

D at Q out of the plane of the diagram

25 In a cathode-ray oscilloscope tube, the electron beam passes through a region where
there are electric and magnetic fields directed horizontally rightwards as shown.

front view of screen

The deflections of the spot from the centre of the screen produced by the electric field E
and magnetic field B separately are equal in magnitude.

Which diagram shows a possible position of the spot on the screen when both fields are
operating together?

A B C D

2013/PJC/PHYSICS/8866
13

26 Two identical long straight aluminium wires P and Q carry the same current but in
opposite directions. They are suspended by identical fine nylon threads from a fixed
point X. It is found that, in equilibrium, the angle a is the same as the angle b.

b
a

P x Q

If the current in P is increased to twice its original value, which one of the following
statements about the angles a and b is correct?

1
A a b
2

B ab

C a 2b

D a 2b

27 A metal surface in an evacuated tube is illuminated with monochromatic light causing the
emission of photoelectrons which are collected at an adjacent electrode. The experiment
is to be repeated with light of half the intensity but the same wavelength.

How will the photocurrent I and stopping potential V be affected?

A I unchanged and V unchanged

B I unchanged and V halved

C I halved and V unchanged

D I halved and V halved

2013/PJC/PHYSICS/8866 [Turn over


14

28 What is the de Broglie wavelength of a particle of mass m and kinetic energy E?

A h 2mE

h
B
2mE

h
C
mE

2mE
D
h

29 Transitions between three energy levels in a particular atom give rise to three spectral
lines of wavelengths, in order of increasing magnitude 1 , 2 and 3 .

Which of the following equations correctly relates 1 , 2 and 3 ?

1 1 1
A
1 2 3

1 1 1
B
1 3 2

C 1 2 3

D 1 3 2

30 A photon of light enters a block of glass after travelling through a vacuum. The energy of
the photon on entering the glass block

A increases because its associated wavelength decreases.

B stays the same because the speed of the radiation and the associated wavelength do
not change.

C stays the same because the frequency of the radiation does not change.

D decreases because the speed of the radiation decreases.

End of paper
2013/PJC/PHYSICS/8866
Answers to 2013 JC2 Preliminary Examination Paper 1 (H1 Physics)

1 D 6 A 11 A 16 C 21 A 26 B
2 B 7 D 12 D 17 D 22 C 27 C
3 B 8 C 13 B 18 B 23 D 28 B
4 B 9 D 14 B 19 B 24 C 29 A
5 B 10 D 15 D 20 B 25 D 30 C

F
1 Since B ,
IL

F
units of B = units of
IL
kg m s2
=
Am
= kg s2 A1

Answer: D

2 v v f v i

v v f v i

v 2 v f 2 v i 2
v 7 5
2 2

v 9 m s1

vi
tan
vf
5
tan
7
36

Answer: B

3 For option B, the average value of g is 9.8 m s2, which is accurate. However, the values
are imprecise as there is a large scattering of values.

Option A: accurate and precise


Option C: inaccurate and imprecise
Option D: inaccurate but precise

Answer: B
4 The acceleration-time graph is obtained from the velocity-time graph. Point Q is the
turning point of the velocity-time graph and so is the maximum value.

Answer: B

5 Let the initial speed of the javelin be u.

1
At the initial point, K.E. mu 2 E
2

1
m u cos 60
2
At the maximum height, K.E.
2
1 1
mu 2
42
1
E
4
Answer: B

6 The horizontal component of velocity only affects the range. ( s x u x t )

Answer: A

7 Only D is the correct answer because it is moving at constant velocity. Resultant force
zero.

Answer: D

m
8 Rate of change of momentum of the water = v =1.0 8.0 = 8.0 kg m s1.
t
From Newtons second law of motion, force on the water = 8.0 N
From Newtons third law, force on the notice-board = 8.0 N.
F 8 .0
Hence magnitude of acceleration of the notice-board = = = 4.0 m s2.
m 2 .0

Answer: C

9 The two forces do not cancel out as they act on different bodies.

Answer: D

10 Taking moments about the cg,

3L L T 3
T1 = T2 ; 2 =
2 2 T1 1

Answer: D
11 Extension, e = 90 60 = 30 mm
Tension = k e = 500 0.030 = 15 N

Answer: A

12 Total pressure is P dg
Force on area a is a(P dg)

Answer: D

13 lost in kinetic energy = work done against resistive force in the plank
1
m[(150)2 (125)2 ] work done against friction
2
For the second bullet, the work done against friction stays the same when it passes
through the plank.
1 1
m[(150)2 (125)2 ] m[(90)2 (v )2 ]
2 2
v 35 m s1

Answer: B

14 Work done (WD) by applied force = (80)(3) = 240 J


WD against frictional force = friction distance moved = (30)(3) = 90 J = heat generated.
Gained in gravitational potential energy = mgh = (40)(3.0 sin 30)= 60 J
Using conservation of energy,
Gained in K.E. = 240 90 60 = 90 J

Answer: B

dV
15 As force F F 2kx
dx
and since force is in the direction of decreasing potential, it is in the direction PO.

Answer: D

16 v f
1500(0.250)
375m s1

Answer: C
17 I kA2
I
I' k A '2
4
kA2
k A '2
4
A
A'
2
A ' A cos
cos 0.5
Hence, 60

Answer: D

18 For the first tuning fork:



L
4
340 850
340 850(4L)
L 0.100m

For the second tuning fork:


second
2L
4
second 8L
340 f second
340 f (8L)
340 f (0.8)
f 425Hz

Answer: B

19 Since the intensity of the light from one slit is now half that of the light from the other slit,
the amplitudes of waves from both slits are no longer the same.

By Principle of superposition, when the waves are in antiphase with each other, there is
no complete cancellation and the resultant amplitude cannot be zero. As a result, there
would not be complete destructive interference and dark fringes appear brighter than
before. Hence intensity of dark fringes increases.
On the other hand, when the waves are in phase with each other, the resultant amplitude
has decreased. Therefore, the bright fringes appear less bright than previously. Hence
intensity of bright fringes decreases.

Answer: B
20 By definition, potential difference involves changing electrical energy into other forms,
which is the work done per unit charge by the charge moving from one point to another
in the circuit.
By definition, electromotive force involves changing other energy forms into electrical
energy, which comes from the energy per unit charge to move charge right round a
circuit.

Answer: B

21 Option A: Correct. Because when connected in series to a battery, a common current


passes through both X and Y, power = I 2R , so Px 2 Py .
Option B: Incorrect. Since wires X and Y are made from the same material, they have
the same resistivity.
Option C: Incorrect. Because when connected in parallel to a battery, a common p.d. is
1
experienced by both X and Y, current in X = current of Y.
2
l 1
Option D: Incorrect. Since R , cross-sectional area of X = cross-sectional area
A 2
of Y, for the same length of the two wires.

Answer: A

22 The largest reading on the ammeter means largest current passing through it. Ammeter
has a resistance of 2 .

Consider p.d. across PQ = V

V
Option A: Incorrect. Current passing through ammeter, I .
5
V
Option B: Incorrect. Current passing through ammeter, I .
2.67
V
Option C: Correct. Current passing through ammeter, I , because ammeter is in
2
parallel connection with the 1 and 2 resistors.
V
Option D: Incorrect. Current passing through ammeter, I .
3

Answer: C

23 When flashing light is not illuminating on the LDR, the p.d. across the LDR is 1.5 V (since
1 M >> 5 k).
When flashing light is illuminating on the LDR, the p.d. across the LDR is
1
1.5 0.25 V .
1 5

Answer: D

24 Using the right-hand grip rule, the current should be at Q into the page.

Answer: C
25 The electric field will cause the electrons to experience a force towards the left. The
magnetic field will cause the electrons to experience a force downwards. Since the
magnitude of the two forces is the same, the beam of electrons will be deflected as
shown in option D.

Answer: D

26 When the current in P increases, the magnetic forces of repulsion increase, but are the
same on both P and Q. Given that the masses are the same, the angle of deflection will
be the same for both, with only an increase in magnitude.

Answer: B

27 Having the wavelength and frequency constant would mean that the energy of the
photon remains constant (E = hf). The most energetic electron would have the same KE
as before; hence the stopping potential is unchanged. However, halving the intensity of
radiation will halved the photocurrent as the rate of emission of photoelectron is halved.

Answer: C

h
28 Using de Broglies equation,
mv
1
given that E mv 2 ,
2
2E 2
hence v
m
2E 2m 2E
v => mv 2mE
m m
h h

mv 2mE

Answer: B
3
29
1
2

For the three energy levels, the transition between them will release energy equivalent of
hc
E . The higher amount of energy release will correspond to radiation of a lower

wavelength.

The relationship between the three energy levels is E 1 E 2 E 3


hc hc hc

1 2 3

1 1 1

1 2 3

Answer: A

30 As photon passes through the glass, its speed decreases. This decrease is due to its
wavelength decrease, while the frequency remains the same. Since the energy of the
photon depends on frequency, energy remains constant.

Answer: C
Name Class Index Number

PIONEER JUNIOR COLLEGE


JC2 Preliminary Examination

PHYSICS 8866/02
Higher 1

Paper 2 Structured Questions


20 September 2013

2 hours
Candidates answer on the Question Paper.
No Additional Materials are required.

READ THESE INSTRUCTIONS FIRST

Write your name, class and index number on all the work you hand in.
Write in dark blue or black pen.
You may use a soft pencil for any diagrams, graphs or rough working.
Do not use staples, paper clips, highlighters, glue or correction fluid.

Section A
Answer all questions.

Section B For Examiners Use


Answer any two questions.
Section A
At the end of the examination, fasten all your work 1 / 8
securely together.
The number of marks is given in brackets [ ] at the end of 2 / 8
each question or part question. 3 / 6
4 / 8
5 / 10
Section B
6 / 20
7 / 20
8 / 20
Total / 80

This document consists of 25 printed pages.

2013/PJC/PHYSICS/8866 [Turn over


2

Data

speed of light in free space, c 3.00 10 8 m s1

elementary charge, e 1.60 10 19 C

the Planck constant, h 6.63 10 34 J s

unified atomic mass constant, u 1.66 10 27 kg

rest mass of electron, me 9.11 10 31 kg

rest mass of proton, m p 1.67 10 27 kg

acceleration of free fall, g 9.81 m s2

Formulae

1 2
uniformly accelerated motion, s ut at
2
v 2 u 2 2as

work done on/by a gas, W pV

hydrostatic pressure, p gh

resistors in series, R R1 R2 ...

resistors in parallel, 1/ R 1/ R1 1/ R2 ...

2013/PJC/PHYSICS/8866
3

Section A

Answer all the questions in this section.

1 A student times the fall of a small metal ball in vacuum. The data for the time t taken for
the ball to fall a vertical distance s from rest are given below.

t 860 10 ms
s 359 1 cm

(a) Determine the acceleration of free fall g, together with its uncertainty.

g = .................... .................... m s2 [3]

(b) The accuracy of the experiment to determine g can be improved by collecting several
pairs of values for s and t. Fig. 1.1 shows the variation with distance s of t.

t / ms

1000 x
x
800
x
600
x
400 x

200 x
1
0 s / cm 2
0 4 8 12 16 20 24

Fig. 1.1

2013/PJC/PHYSICS/8866 [Turn over


4

(i) Suggest why drawing a best fit line helps in reducing the random errors.

..................................................................................................................................

........................................................................................................................... [1]

(ii) Explain what feature of Fig. 1.1 suggests the presence of systematic errors.

..................................................................................................................................

........................................................................................................................... [1]

(iii) Explain how the value of g can be determined, if a graph of lgs is plotted against
lgt instead.

..................................................................................................................................

..................................................................................................................................

........................................................................................................................... [1]

(c) On Fig. 1.1, sketch a possible graph if the experiment in carried out in air instead.
Explain clearly why this is so.

.......................................................................................................................................

.......................................................................................................................................

.......................................................................................................................................

................................................................................................................................. [2]

2013/PJC/PHYSICS/8866
5

2 In a nuclear reactor, a fast moving neutron with initial speed u1 makes a head-on elastic
collision with a stationary nucleus of carbon-12 which has a mass 12 times that of the
neutron. The speeds of the neutron and the carbon nucleus after the collision are v1 and
v2 respectively.

(a) What is meant by a head-on elastic collision?

.......................................................................................................................................

.......................................................................................................................................

.......................................................................................................................................

................................................................................................................................. [2]

(b) (i) State the principle of conservation of linear momentum.

..................................................................................................................................

..................................................................................................................................

............................................................................................................................ [1]

(ii) Determine the ratio of the final speed of the neutron v1 to its initial speed u1.

ratio = ........................................ [3]

2013/PJC/PHYSICS/8866 [Turn over


6

(c) Hence determine the fraction of the kinetic energy of the neutron that is transferred to
the carbon nucleus.

fraction = ........................................ [2]

2013/PJC/PHYSICS/8866
7

3 A catapult consists of two strands of rubber that obeys Hookes law. Each strand has
original length of 0.200 m and each will stretch by 0.100 m when under a tension of 50 N.
A mass of 0.060 kg is projected vertically upwards from the catapult after each strand
has been extended to a length of 0.350 m.

(a) Calculate the energy stored in the stretched catapult.

energy stored = ........................................ J [3]

(b) Determine the maximum height attained by the stone if air resistance is negligible.

maximum height = ........................................ m [2]

(c) Explain, by reference to energy conservation, whether the presence of air resistance
will result in an increase or decrease of the maximum height attained.

........................................................................................................................................

........................................................................................................................................

................................................................................................................................. [1]

2013/PJC/PHYSICS/8866 [Turn over


8

4 (a) A single-turn square-shaped coil PQRS of sides 25 cm is suspended from a


well-insulated spring. The coil is placed half-way in a region of uniform magnetic field
of flux density 2.0 T, as shown in Fig. 4.1.

P Q

8.0 A

S R

magnetic field

Fig. 4.1

A current of 8.0 A flows through the coil and the spring extends by 10 mm.

(i) State the direction of the magnetic field.

........................................................................................................................... [1]

(ii) Calculate the spring constant.

spring constant = ........................................ N m1 [2]

(iii) The Earths magnetic field is in the same direction as the applied magnetic field
of 2.0 T. Suggest a reason why the Earths field does not affect your answer in
(a)(ii).

..................................................................................................................................

........................................................................................................................... [1]

2013/PJC/PHYSICS/8866
9

(b) The same magnetic field is now applied towards the right, with coil PQRS fully in the
region of the uniform field, as shown in Fig. 4.2.

P Q

8.0 A 2.0 T

S R

Fig. 4.2

(i) Calculate the torque experienced by the coil.

torque = ........................................ N m [2]

(ii) The coil is then tilted about the vertical axis and makes an angle to the
horizontal magnetic field lines as shown in Fig. 4.3.

x Q
2.0 T
P

Fig. 4.3

Discuss qualitatively how your answer to (b)(i) changes.

..................................................................................................................................

..................................................................................................................................

........................................................................................................................... [2]

2013/PJC/PHYSICS/8866 [Turn over


10

5 The resistance R of a small semiconductor device X varies with temperature / o C as


shown in Fig. 5.1.

R/

/ oC
Fig. 5.1

2013/PJC/PHYSICS/8866
11

(a) (i) The resistance and temperature of device X are related by the expression

B
R AeT

where A and B are constants and T represents the thermodynamic temperature.

Determine the values for A and B. Express the answers in the appropriate units.

A = ........................................ [2]

B = ........................................ [2]

(ii) On Fig. 5.2, sketch a graph of the variation of the current I with the potential
difference V through device X.

0 V

Fig. 5.2

[1]

2013/PJC/PHYSICS/8866 [Turn over


12

(b) Device X is now connected to a fixed resistor of resistance 40.0 as shown in


Fig. 5.3.
ideal
voltmeter
V

X 40.0

6.0 V
Fig. 5.3

(i) Calculate an estimated value for the voltmeter reading when device X is
immersed in water at temperature 30.0 o C .

V = ........................................ V [3]

(ii) If the temperature of the water is raised, would the voltmeter reading increase or
decrease? Explain.

..................................................................................................................................

..................................................................................................................................

........................................................................................................................... [2]

2013/PJC/PHYSICS/8866
13

Section B

Answer two of the questions in this section.

6 (a) State the principle of superposition.

.......................................................................................................................................

.......................................................................................................................................

................................................................................................................................. [2]

(b) Explain what is meant by constructive interference.

.......................................................................................................................................

.......................................................................................................................................

................................................................................................................................. [1]

(c) Sound produced by the loudspeaker shown in Fig. 6.1 has a frequency of
4.0 103 Hz. The sound waves arrive at the microphone M via two different paths,
LXM and LYM. The left-tube is fixed in position, while the right-tube is a
sliding-section. At position M, the sound waves from the two paths interfere.

loudspeaker

sliding-section
L which can be
moved horizontally

X Y

microphone

Fig. 6.1

Initially, the lengths of paths LXM and LYM are equal. The sliding-section is then
pulled out horizontally to the right by a distance of 0.020m, and the loudness at
microphone M changes from a maximum to a minimum.

2013/PJC/PHYSICS/8866 [Turn over


14

(i) Determine the path difference between the two waves after the sliding-section is
pulled out.

path difference = ........................................ m [1]

(ii) Hence, or otherwise, calculate the wavelength of sound produced by the speaker.

wavelength = ........................................ m [1]

(iii) When the opening at M is sealed, explain why a standing wave can be set up in
the tube.

..................................................................................................................................

..................................................................................................................................

..................................................................................................................................

..................................................................................................................................

........................................................................................................................... [3]

(iv) A good loudspeaker should be able to diffract sound over a large area. Estimate
the optimal diameter of the loudspeaker in order to achieve the maximum
spreading of sound waves. Explain your answer.

..................................................................................................................................

..................................................................................................................................

........................................................................................................................... [2]

2013/PJC/PHYSICS/8866
15

(d) A parallel beam of light is incident normally on two slits S1 and S2 of equal width as
shown in Fig. 6.2.

S1
parallel beam of light X E
S2

double slit
lens

Fig. 6.2

Coherent wavetrains of light emerging from the double slit are brought to superpose
at point E which is equidistant from S1 and S2.

(i) What do you understand by coherent wavetrains of light.

..................................................................................................................................

........................................................................................................................... [1]

(ii) Separate wavetrains of light from S1 and S2 reach E with amplitude A each. The
resultant intensity at E is I. Determine the new intensity of light at E when S1 is
covered with a black cardboard. Express your answer in terms of I.

new intensity = ........................................ [3]

2013/PJC/PHYSICS/8866 [Turn over


16

The lens and the black cardboard covering S1 are removed. A screen is placed at E
instead at a distance of 2.50 m from the double slit. The slit separation is 0.800 mm.
The double slit setup, as shown in Fig. 6.3, is then illuminated with light of
wavelength 589 nm.
2.50 m

S1
parallel beam of light 0.800 mm
S2

double slit

Fig. 6.3

(iii) Calculate the distance between the centre of a bright fringe and the centre of the
adjacent dark fringe on the screen.

distance = ........................................ m [2]

(iv) State and explain briefly the change, if any, that would be observed in the pattern
of fringes if the following adjustments are made in the experimental arrangement.
In each case, only one adjustment is made and all the other arrangements are
unchanged.

1. The coherent light of wavelength 589 nm is replaced with coherent


monochromatic red light.

.............................................................................................................................

.............................................................................................................................

....................................................................................................................... [2]

2013/PJC/PHYSICS/8866
17

2. The screen is rotated through 45 as shown in Fig. 6.4.

2.50 m

45
S1
parallel beam of light 0.800 mm
S2

double slit

Fig. 6.4

............................................................................................................................

.............................................................................................................................

....................................................................................................................... [2]

2013/PJC/PHYSICS/8866 [Turn over


18

7 (a) Distinguish between the electromotive force of a cell and the potential difference
between its terminals.

......

......

......

.............................................................................................................................. [2]

(b) Fig. 7.1 shows how the resistance of a light-dependent resistor (LDR) varies with the
intensity of the light incident on it.

resistance / k

incident light intensity / W m2

Fig. 7.1

Fig. 7.2 shows a light-sensing potential divider circuit used in a lamp where the
potential difference across the LDR can be used to control the brightness of the lamp
in a room.

1.2 k

9.0 V
p.d. to control
brightness of
lamp

Fig. 7.2

The battery has an e.m.f. of 9.0 V and negligible internal resistance. The
1.2 k resistor is made of carbon. When the room is in a low-light condition, the
potential difference across the LDR reaches 7.0 V.

2013/PJC/PHYSICS/8866
19

(i) State the potential difference across the 1.2 k resistor, when the room is in a
low-light condition.

potential difference = ........................................ V [1]

(ii) Hence, calculate the resistance R of the LDR.

R = ........................................ k [2]

(iii) Use Fig. 7.1 to determine the light intensity when the p.d. across the LDR is 7.0 V.

light intensity = ........................................ W m2 [1]

(iv) Fig. 7.3 shows a close-up of the LDR device used in the circuit in Fig. 7.2. The
LDR consists of a uniform strip of semiconductor whose resistance is dependent
on the intensity of the light incident on it. The cross-sectional area of the strip is
5.0 107 m2.

10.0 mm
LDR strip
5.0 mm

Fig. 7.3

Use your answer in (ii) to determine the resistivity of the LDR.

resistivity = ........................................ m [2]

2013/PJC/PHYSICS/8866 [Turn over


20

(v) Use the evidence provided by Fig. 7.1 to explain the sensitivity of this LDR for the
light-sensing circuit used in controlling the brightness of the lamp.

..................................................................................................................................

..................................................................................................................................

..................................................................................................................................

........................................................................................................................... [2]

(c) Fig. 7.4 shows a circuit containing five identical lamps A, B, C, D and E. The circuit
also contains three switches S1, S2 and S3.

S1 S3

A B
E

S2

C D

Y
Fig. 7.4

One of the lamps is faulty. In order to detect the fault, an ohm-meter (a meter that
measures resistance) is connected between terminals X and Y. When measuring
resistance, the ohm-meter causes negligible current in the circuit.

Fig. 7.5 shows the readings of the ohm-meter for different switch positions.

switch metre reading


S1 S2 S3 /
open open open
closed open open 30.0
closed closed open 22.5
closed closed closed 15.0

Fig. 7.5

2013/PJC/PHYSICS/8866
21

(i) Identify the faulty lamp, and the nature of the fault.

faulty lamp: ..............................................................................................................

nature of fault: .................................................................................................... [2]

(ii) Suggest why it is advisable to test the circuit using an ohm-meter that causes
negligible current rather than with a power supply.

..................................................................................................................................

..................................................................................................................................

........................................................................................................................... [1]

(iii) State the resistance of one of the non-faulty lamps, as measured using the
ohm-meter.

resistance = ........................................ [1]

(iv) After replacing the faulty lamp in the circuit in Fig. 7.4 with a similar working lamp,
the ohm-meter is connected between terminals X and Y.

On Fig. 7.6, complete the readings of the ohm-meter for different switch positions.

switch metre reading


S1 S2 S3 /
open open open
closed open open
closed closed open
closed closed closed

Fig. 7.6 [2]

(v) Each lamp is marked 12.0 V, 0.40 A.

Calculate, for one of the lamps operating at normal brightness,

1. its resistance,

resistance = ........................................ [1]

2013/PJC/PHYSICS/8866 [Turn over


22

2. its power dissipation.

power dissipation = ........................................ W [1]

(vi) Comment on your answers to (iii) and (v)1.

..................................................................................................................................

..................................................................................................................................

..................................................................................................................................

........................................................................................................................... [2]

2013/PJC/PHYSICS/8866
23

8 (a) In a photoelectric experiment, radiation of wavelength 254 nm and intensity


250 W m2, was incident on a copper surface in an evacuated tube to illuminate an
area of 15 mm2. A photocurrent of 5.20 10 10 A was collected on an adjacent
electrode.

(i) Explain what is meant by photoelectric emission.

..................................................................................................................................

..................................................................................................................................

........................................................................................................................... [2]

(ii) What was the rate of incidence of photons on the copper surface?

rate of incidence = ........................................ s1 [2]

(iii) What was the rate of emission of electrons?

rate of emission = ........................................ s1 [2]

(iv) The photoelectric quantum yield is defined as the ratio

number of photoelectrons emitted per second


.
number of photons incident per second

Calculate the quantum yield of this copper surface at the wavelength of 254 nm.

quantum yield = ........................................ [2]

2013/PJC/PHYSICS/8866 [Turn over


24

(v) The current I in the circuit is measured for various values of the applied potential
difference V between the copper and electrode.

photoelectric current, I / nA

5.2

-2 0 2 4 potential
difference, V / V
Fig. 8.1

1. Determine the maximum kinetic energy of the photoelectrons.

kinetic energy = ........................................ J [2]

2. Deduce the work function of copper.

work function = ........................................ J [2]

3. Explain why the photocurrent reaches a maximum value no matter how large
V is.

............................................................................................................................

............................................................................................................................

..................................................................................................................... [2]

2013/PJC/PHYSICS/8866
25

(b) Fig. 8.2 shows four energy levels A, B, C and D within an atom and an electron
transition from level A to level C which results in the emission of a photon of light.

A 0
B 0.43 x 1019 J

C 3.33 x 1019 J

D 14.72 x 1019 J

Fig. 8.2

(i) On Fig. 8.2, draw all the other possible transitions between these four levels
which result in photon emission. [2]

(ii) Calculate the wavelength of the visible light emitted as a result of the electron
transition from A to C.

wavelength = ........................................ m [2]

(iii) Which other transition which may result in visible light being emitted?

transition = ........................................ [1]

(iv) When a transition takes place from level A, B or C to level D, in which part of the
electromagnetic spectrum will the radiation occur?

........................................................................................................................... [1]

End of paper
2013/PJC/PHYSICS/8866 [Turn over
1

Answers to 2013 JC2 Preliminary Examination Paper 2 (H1 Physics)

Suggested Solutions:

No. Solution
1(a) Take downwards as positive.
1
s gt 2 (since ball is released from rest)
2
1
3.59 g 0.860 2
2
g 9.71 m s2

2s
g
t2
g s t
2
g s t
g 1 10
2
9.7080 359 860
g 0.3 m s2

Therefore, g 9.7 0.3 m s2.

1(b)(i) Drawing a best fit line minimises the effects of random


errors since the best fit line drawn has a weighted mean,
where the variation from the actual value may cancel out.

1(b)(ii) The line did not pass through the origin, such that s 0 at
t 0 . This resulted in all the measurements of s being
larger than the actual values, and hence suggesting the
presence of systematic error in the experiment.

1(b)(iii) By taking lg on both sides of the equation, we have


1
lg s lg g 2lg t . Since this equation is that of a straight
2
1
line, where lg g is the vertical intercept, the value of g
2
can therefore be determined.
2

1(c)
t / ms

1000 x
x
800
x
600
x
400 x

200 x
1
0 s / cm 2

0 4 8 12 16 20 24

As the ball falls, it gains speed but there is drag force acting
against its motion. Hence, the downwards acceleration is
lesser than when it is in vacuum, and the graph should show
2
a steeper gradient as time increases, where t s.
a

2(a) A head-on collision takes place along the line joining the
centres of the colliding bodies.

An elastic collision is one in which the kinetic energy is


conserved.

2(b)(i) The total momentum of a system is constant, provided no


external resultant force acts on it.

2(b)(ii) By conservation of linear momentum,

mu1 = mv1 12mv2


==> u1 v1 = 12v2 .(1)

velocity of approach = velocity of separation

u1 0 = v2 v1
u1 v1 = v2 (2)

2
(1) (2) 2u1 = 13 v2 ==> v2 = u1
13

2 11
From (2) v1 = v2 u1 = u1 u1 = u1
13 13
v1 11
Thus ratio =
u1 13
3

2(c) 1
m(u12 v12 ) v
Required fraction 2 1 [ 1 ]2
1 u1
mu12
2
11
1 [ ]2
13
0.28

3(a) Applying Hookes Law,


F = ke
F 50
k 500 N m1
e 0.100
Elastic potential energy stored in both strands
1
2 ke2 ke2 (500)(0.350 0.200)2 =11.3 J
2

3(b) From conservation of energy,


gained in G.P.E. = lost in E.P.E.
mgh = 11.3
h = 19.1 m

3(c) With presence of air resistance, there will be energy lost as


work done against air resistance. Thus less E.P.E. is
converted to G.P.E. resulting in a decrease in maximum
height attained.

4(a)(i) The magnetic field is pointing into the page.

4(a)(ii) BIl ke
2 8.0 0.25 k 10 103
k 400 N m1

The spring constant is 400 N m1.

4(a)(iii) The Earths magnetic flux density is of the order of 10 5 T,


and is negligible compared to 2 T.

4(b)(i) The torque on the coil is due to the magnetic force FB acting
on the sides PS and QR only.

Torque experienced by the coil,


FB l
2 8.0 0.25 0.25
1.0 N m
4

4(b)(ii) When the magnetic field is applied at an angle to the


plane area of the coil, the magnitude of the magnetic force
on sides PS and QR of the coil remains the same.
However, the perpendicular distance to the magnetic force
is smaller than before. Hence, the torque experienced by the
coil will decrease.

5(a)(i) B
R Ae T
B
ln R ln A
T
Taking temperatures at 50o C(323.15K)and
80o C(353.15K) , R= 110 and 50 respectively.

B
ln110 ln A (1)
323.15
B
ln50 ln A (2)
353.15

(1) (2),

B B
ln110 ln50
323.15 353.15

B 3.0 103 K
A 1.02 102
5(a)(ii)
I

5(b)(i) At 30.0 oC, the resistance of X is approximately 188 .

By potential divider,

40
V 6 1.05 V
40 188
5(b)(ii) The voltmeter reading should increase.

From Fig. 5.1, as the temperature of the water is raised, the


resistance of Device X decreases. Using the potential
divider principle, the p.d. across the 40 will increase.
5

6(a) The principle of superposition states that when two or more


travelling waves of the same type meet at a point in space,
the resultant displacement at that point is the vector sum of
the displacements that the waves would separately produce
at that point.

6(b) When 2 or more waves of same phase meet, they


superpose with each other to give a resultant wave of
maximum amplitude. This is known as constructive
interference.

6(c)(i) Path difference 2 0.020


0.040 m

6(c)(ii)
Path difference
2
2 0.040
0.080 m
6(c)(iii) The sound waves from path LXM and LYM meet in the
opposite directions.
Since both waves are of equal amplitude, frequency and
speed, they superpose and interfere to form a stationary
wave.

6(c)(iv) Optimal diameter is approximately 0.080 m.


When the diameter of the loudspeaker is comparable to the
wavelength of the sound it produces, significant diffraction
occurs.
6(d)(i) It means the wavetrains of light have constant phase
difference.

6(d)(ii) I k(A A)2


4kA2
When S1 is covered,
New intensity kA2
I

4

6(d)(iii) D
fringe separation
a
589 109 (2.50)

0.800 103
1.8406 10 3 m

1.8406 103
distance required
2
9.20 104 m
6

6(d)(iv) Wavelength of red light (approximately 700 nm) is larger


1. D
than 589 nm. Since fringe separation , this means that
a
the fringe separation is increased.

6(d)(iv) D
2. Since fringe separation and the lower half of the
a
screen is nearer the double slit, fringe separation at the
lower half of the screen is thus reduced. Upper half of the
screen is further from the double slit. Hence fringe
separation at the upper half is increased.

7(a) E.m.f. of the cell is the energy converted into electrical


energy from non-electrical energy when unit charge passes
through it.
The potential difference (p.d.) between the terminals is the
energy converted from electrical energy to other forms of
energy when unit charge passes from one point to the other.

7(b)(i) potential difference across the 1.2 k resistor = 2.0 V

7(b)(ii) Using V IR
2.0 = (1.2 103) I
I = 1.67 x 103 A
V 7.0
R= = 4200 or 4.2 k
I 1.67 10 3

7(b)(iii) From Fig. 7.1, light intensity = 24 W m2

7(b)(iv)
Length of strip, 10 5.0 10 3 10.0 10 3 0.060 m


Using R ,
A
RA 4200 5.0 107
3.5 102 m
0.060

7(b)(v) Larger changes in R at low light intensities, resulting in


larger changes in the p.d. across the LDR.
Hence greater sensitivity of the LDR at low light conditions,
which is used to control the brightness of the lamp.

7(c)(i) nature of fault: lamp is shorted


faulty lamp: lamp E

7(c)(ii) Shorted lamp could cause excessive current to flow in the


circuit that could cause damage to the power supply / other
lamps / blow fuse in power supply.
7

7(c)(iii) Resistance of one non-faulty lamp = 30.0 / 2 = 15.0

7(c)(iv)
switch metre reading
S1 S2 S3 /
open open open
closed open open 30.0
closed closed open 25.0
closed closed closed 15.0
7(c)(v) Using V = I R
1. R=V/I
= 12.0 / 0.40
= 30.0

7(c)(v) Using P = V I or I 2 R or V2 / R
2. P = 12.0 0.40
= 4.8 W

7(c)(vi) Lamp filament is cold and has lower resistance when


measuring with ohm-meter in (iii).

Resistance of filament rises as temperature rises when lamp


operates at normal brightness in (v)1.

8(a)(i) The liberation of electrons from a metal surface when it is


exposed to electromagnetic radiation of sufficiently high
frequency.

8(a)(ii) power of radiation incident on surface


= 250 W m2 0.000015 m2
= 0.00375 W
Nhf
P
t
N P

t hc
N 0.00375(254 109 )

t (6.63 1034 )(3.00 108 )
N
4.79 1015 s1
t
8

8a(iii) Q
I 5.20 1010
t
Q Ne
I
t t
N I

t e
I 5.20 1010
19
3.25 109 s1
e 1.6 10

8a(iv) quantum yield


number of photoelectrons emitted per second
=
number of photons incident per second
3.25 109
=
4.79 1015
= 6.78 107

8a(v)1. stopping potential = 2 V

qV = KEmax
KEmax = ( 1.6 1019 ) ( 2)
KEmax = 3.2 1019 J

8a(v)2. hf KEmax

hc (6.63 1034 )(3.00 108 )


hf 9
7.83 1019 J
(254 10 )

KEmax = 3.2 1019 J


hf KEmax 7.83 1019 3.2 1019

4.63 1019 J

8a(v)3. Increasing the p.d. between the electrodes cannot increase


the current beyond a certain maximum level. The maximum
current is limited by the rate at which photoelectrons are
emitted, and that this controlled by the intensity of the
radiation, not the potential difference.
9

8b(i)

A 0
B
0.43 x 1019 J

C
3.33 x 1019 J

14.72 x 1019 J
D

8b(ii) For transition from A to C


hf 3.33 1019 J
hc
3.33 1019 J

5.97 107 m
(visible light)

8b(iii) Transition from A to C emits photon of visible light.


Hence transition from B to C may emit visible light.

hc
check: 2.9 1019 J

6.86 107 m

Ans: transition B to C

8b(iv) Any transition place from level A, B or C to level D will emit


UV radiation
Name Class Index Number

PIONEER JUNIOR COLLEGE


JC2 Preliminary Examination

PHYSICS 9646/01
Higher 2

Paper 1 Multiple Choice


27 September 2013

1 hour 15 minutes

Additional Material: Multiple Choice Answer Sheet

READ THESE INSTRUCTIONS FIRST

Write in soft pencil.


Do not use staples, paper clips, highlighters, glue or correction fluid.
Write your name, class and index number on the Answer Sheet in the spaces provided.

There are forty questions on this paper. Answer all questions. For each question there
are four possible answers A, B, C and D.
Choose the one you consider correct and record your choice in soft pencil on the
separate Answer Sheet.

Read the instructions on the Answer Sheet very carefully.

Each correct answer will score one mark. A mark will not be deducted for a wrong
answer.

Any rough working should be done in this booklet.

This document consists of 18 printed pages.

2013/PJC/PHYSICS/9646 [Turn over


2

Data

speed of light in free space, c 3.00 10 8 m s1

permeability of free space, 0 4 10 7 H m1

permittivity of free space, 0 8.85 10 12 F m1

1 36 10 9 F m1

elementary charge, e 1.60 10 19 C

the Planck constant, h 6.63 10 34 J s

unified atomic mass constant, u 1.66 10 27 kg

rest mass of electron, me 9.11 10 31 kg

rest mass of proton, m p 1.67 10 27 kg

molar gas constant, R 8.31 J K1 mol1

the Avogadro constant, N A 6.02 10 23 mol1

the Boltzmann constant, k 1.38 10 23 J K1

gravitational constant, G 6.67 10 11 N m2 kg2

acceleration of free fall, g 9.81 m s2

2013/PJC/PHYSICS/9646
3

Formulae

1 2
uniformly accelerated motion, s ut at
2
v 2 u 2 2as

work done on/by a gas, W pV

hydrostatic pressure, p gh

Gm
gravitational potential,
r

displacement of particle in s.h.m., x x 0 sin t

velocity of particle in s.h.m., v v 0 cos t


2
x 0 x 2

3
mean kinetic energy of a molecule E kT
2
of an ideal gas,

resistors in series, R R1 R2 ...

resistors in parallel, 1/ R 1/ R1 1/ R2 ...

Q
electric potential, V
4 0 r

alternating current/voltage, x x 0 sin t

8 2 mU E
transmission coefficient, T exp 2kd where k
h2

radioactive decay, x x 0 exp(t )

0.693
decay constant,
t1
2

2013/PJC/PHYSICS/9646 [Turn over


4

1 What are the SI base units of magnetic flux density?

A kg m s2 A1

B kg m2 s2 A1

C kg s2 A1 rad1

D kg s2 A1

2 A boat changes its velocity from 5 m s1 due East to 7 m s1 due South.

What is its change in velocity?

A 9 m s1 at a direction of 54 South of East

B 9 m s1 at a direction of 54 South of West

C 2 m s1 at a direction of 36 East of South

D 2 m s1 at a direction of 36 West of South

3 The acceleration-time graph of an object moving in a straight line is as shown.

acceleration
P

Q S time
0

If the object starts its motion from rest, at which point is the object moving with the
largest speed?

A P B Q C R D S

4 An athlete throws a javelin at an angle of 60 to the horizontal. The javelin leaves the
athletes hand with an initial kinetic energy, E.

Neglecting air resistance, the javelins kinetic energy at its highest point of motion is

1 3
A zero. B E. C E. D E.
4 4

2013/PJC/PHYSICS/9646
5

5 Which body is in equilibrium?

A A satellite moving around the Earth in a circular orbit.

B A car rolling down a frictionless inclined plane.

C An apple falling freely towards the surface of Earth.

D A block sliding at constant velocity across a tabletop.

6 A mass of 2.0 kg is at rest on a smooth floor. A horizontal stream of water, travelling at


speed 8.0 m s1, strikes it at a rate of 1.0 kg s1 for a duration of 50 s without splashing.

What is the initial acceleration of the mass?

A 0.080 m s2

B 0.16 m s2

C 4.0 m s2

D 8.0 m s2

7 A barge is floating in seawater of density 1030 kg m3. The area of the horizontal cross-
section of barge is 97 m2 and the sides of the barge are vertical. The flat-bottom of the
barge is 70 cm under seawater when it is loaded with 2.0 104 kg of cargo.

How deep is the bottom of the barge below the water surface when the cargo is
unloaded off the barge?

A 20 cm

B 50 cm

C 70 cm

D 90 cm

8 A spring obeying Hookes law has an unstretched length of 60 mm and a spring constant
of 500 N m1.

What is the tension in the spring when its overall length is 90 mm?

A 15 N

B 45 N

C 150 N

D 450 N

2013/PJC/PHYSICS/9646 [Turn over


6

9 A bullet moving with a speed of 150 m s1 strikes a wooden plank. After passing through
the plank, its speed becomes 125 m s1. Another identical bullet strikes the plank with a
speed of 90 m s1.

What is its speed after passing through the plank if the bullets experience the same
constant retarding force in both cases?

A 25 m s1 B 35 m s1 C 50 m s1 D 70 m s1

10 A force F of 80 N pulls an object of weight 40 N from rest up an incline of 30 through


3.0 m as shown below. The frictional force present is 30 N.

3.0 m

30

Which of the following correctly gives the heat generated due to friction and the gain in
kinetic energy of the mass?

gain in kinetic
heat / J
energy / J

A 210 0

B 90 90

C 90 30

D 210 240

11 Two identical particles P and Q are set to travel in a circular path of the same radius. P
moves in a vertical circle and Q moves in a horizontal circle. Both move with the same
uniform speed.

Which of the following statements concerning the magnitude of the net force acting on P
and Q towards the centre of the circular path is true?

A Both net forces on P and Q vary with time and are never equal in magnitude.

B Both net forces on P and Q vary with time and are equal in magnitude periodically.

C The net forces on P and Q are always equal in magnitude.

D The magnitude of the net force on P is always larger than that on Q.

2013/PJC/PHYSICS/9646
7

12 A mass m1 is attached to one end of an elastic string of an unstretched length L. When


the mass is rotating with a linear speed v on a smooth table in a horizontal circle, an
extension e is obtained.

Which of the following shows the correct expression for mass m2, if it is rotated with the
same linear speed v but rotates at twice the radius as that produced by m1?

2m1(L 2e )
A m2
e

2m1(L e)
B m2
e

2m1(2L e)
C m2
e

2m1(2L 2e)
D m2
e

13 The gravitational force on an object at the top of Mount Fuji, with an elevation of 3776 m,
is 40 N.

What would be the approximate gravitational force on the same object if it were taken to
another mountain with twice the elevation?

A 10 N

B 20 N

C 30 N

D 40 N

14 The planet Venus has a mass 0.8 times that of Earth and a diameter 0.9 times that of
Earth.

Taking the acceleration due to gravity at the Earths surface to be 9.81 m s2 , what is the
acceleration due to gravity at the surface of Venus?

A 8.7 m s2

B 9.7 m s2

C 17 m s2

D 39 m s2

2013/PJC/PHYSICS/9646 [Turn over


8

15 Which one of the following statements about a simple harmonic oscillator is incorrect?

A The total energy of the oscillator is constant throughout the motion.

B The kinetic energy of the oscillator is maximum when it is at equilibrium position.

C The potential energy of the oscillator is zero when the oscillator is momentarily at rest.

D When the kinetic energy of the oscillator is equal to its potential energy, the oscillator
is neither at the rest position nor at the maximum displacement positions.

16 In order to check the speed of a camera shutter, the camera was used to photograph the
bob of a simple pendulum moving in front of a horizontal scale. The extreme positions of
the bob were at 600 mm and 700 mm marks. The photograph showed that while the
shutter was opened, the bob moved from 650 mm to 675 mm mark.

If the period of the pendulum was 2 s, how long does the shutter remain closed during
the first 2 s of its motion?

1
A s
6

1
B s
2

1
C 1 s
2

5
D 1 s
6

17 An aluminium calorimeter of mass 0.05 kg contains 0.095 kg of a mixture of water and


ice at 0 C. When a 0.100 kg of aluminium block heated to 100.0 C is dropped into the
mixture, the temperature rises to 5.0 C. The specific heat capacity of aluminium is
924 J kg1 K1, specific heat capacity of water is 4200 J kg1 K1 and specific latent heat
of fusion of ice is 3.36 x 105 J kg1.

What is the mass of ice originally present in the mixture?

A 0.0195 kg

B 0.0202 kg

C 0.0254 kg

D 0.0261 kg

2013/PJC/PHYSICS/9646
9

18 A cylinder contains a mixture of helium and argon gas in equilibrium at a temperature T.

Which of the following statements is correct about the mixture?

A Each gas molecule has the same translational kinetic energy.

B The gas molecules have the same rootmeansquare speed.

C The argon gas molecules have greater kinetic energy compared with the helium gas
molecules.

D Both types of gas molecules have the same mean translational kinetic energy.

19 Which of the following statements about the absolute scale of temperature is incorrect?

A It is based on two fixed points, namely the absolute zero of temperature and the
triple point of water.

B The triple point of water is the one and only temperature at which water can exist in
all three phases in equilibrium at the same time.

C The relationship between temperatures on the absolute temperature scale and


temperature on the Celsius scale is given by T / K = / C + 273.15.

D It depends on the property of the particular substance used to set it up.

2013/PJC/PHYSICS/9646 [Turn over


10

20 A small pipe opened at both ends is partly submerged in water as shown below. A tuning
fork vibrating at 850 Hz is placed over the top of the pipe. The pipe is slowly lifted until
the first loud sound is heard when the length of pipe above water is L. The experiment is
repeated with another tuning fork of unknown frequency f and the first loud sound is
heard when the length of pipe above water is 2L.

Given that the speed of sound is 340 m s1 , what is the value of f?

A 213 Hz

B 425 Hz

C 1700 Hz

D 2550 Hz

21 A narrow beam of monochromatic light falls at normal incidence on a diffraction grating.


The angle between the third order diffracted beam and the direction of the incident light
is 40.

What is the highest order of diffracted beam produced by this grating?

A 3rd

B 4th

C 5th

D 6th

2013/PJC/PHYSICS/9646
11

22 An ion carrying a charge of 4.8 10 19 C travels horizontally at a speed of


8.0 105 m s1, as shown below. It enters a uniform vertical electric field of strength
4200 V m1, which is directed downwards and acts over a distance of 0.16 m.

positive ion uniform


electric field
4200 V m1

0.16 m

Which of the following statements is not correct?

A The ion passes through the field in 2.0 10 7 s.

B The force on the ion acts vertically downwards at all times in the field.

C The horizontal component of the velocity of the ion is unaffected by the electric field.

D The magnitude of the force exerted on the ion by the field is 1.6 10 9 N.

23 A positive charge of 2.6 10 8 C is placed in an electric field of constant field strength


300 kV m1.

How much work must be done on the charge in order to move it a distance of 4.0 mm in
the same direction as that of the field?

A 3.1 10 5 J

B 3.1 10 5 J

C 3.1 10 2 J

D 3.1 10 2 J

2013/PJC/PHYSICS/9646 [Turn over


12

24 The graph shows the variation with length l of resistance R for two wires X and Y made
from the same material.

R /

l/m

Which of the following statements describes what the graph shows?

A when equal lengths of X and Y are connected in series to a battery,


the power in X is twice the power of Y

B resistivity of X is twice the resistivity of Y

C when equal lengths of X and Y are connected in parallel to a battery,


current in X is twice the current of Y

D cross-sectional area of X is twice the cross-sectional area of Y

25 A network is constructed using eight resistors, each of resistance R, and three switches
S1, S2 and S3.

R R

R R R
X S1 S2 S3 Y
R R

Which switch combination will give rise to the maximum total resistance between points
X and Y?

S1 S2 S3
A closed closed closed
B closed open closed
C open closed closed
D open open open

2013/PJC/PHYSICS/9646
13

26 In each arrangement of resistors below, the ammeter has a resistance of 2 .

Which arrangement gives the largest reading on the ammeter when the same potential
difference is applied between points P and Q?

A B

A A
P Q P Q

C D

A A
P Q P Q

27 In a cathode-ray oscilloscope tube, the electron beam passes through a region where
there are electric and magnetic fields directed horizontally rightwards as shown.

front view of screen

The deflections of the spot from the centre of the screen produced by the electric field E
and magnetic field B separately are equal in magnitude.

Which diagram shows a possible position of the spot on the screen when both fields are
operating together?

A B C D

2013/PJC/PHYSICS/9646 [Turn over


14

28 Two identical long straight aluminium wires P and Q carry the same current but in
opposite directions. They are suspended by identical fine nylon threads from a fixed
point X. It is found that, in equilibrium, the angle a is the same as the angle b.

b
a

P x Q

If the current in P is increased to twice its original value, which one of the following
statements about the angles a and b is correct?

1
A a b
2

B ab

C a 2b

D a 2b

29 A solenoid is connected in series with a battery and a switch as shown. A light, copper
ring is held near to the solenoid and coaxial with it.

solenoid copper ring

What will happen to the copper ring immediately after the switch is closed?

A The ring remains stationary.

B The ring swings directly away from the solenoid.

C The ring swings directly towards the solenoid.

D The ring rotates about its vertical axis.

2013/PJC/PHYSICS/9646
15

30 When a small cylindrical magnet is released from rest above a copper pipe, the
magnet falls slowly through the pipe.

magnet

copper pipe

Which of the following can be made so that the magnet falls slower through the same
length of the pipe?

A Place the magnet nearer to the top of the pipe before releasing it from rest.

B Use a pipe of material with higher resistivity.

C Use a weaker magnet of the same mass.

D Use a lighter magnet.

31 A 100 resistor conducts a current with changing direction and magnitude, as shown.

current / A

time

What is the mean power dissipated in the resistor?

A 100 W

B 150 W

C 250 W

D 400 W

2013/PJC/PHYSICS/9646 [Turn over


16

32 A transformer has 1200 turns on the primary coil and 500 turns on the secondary coil.
The primary coil draws a current of 0.25 A from a 240 V ac supply.

If the efficiency of the transformer is 83 %, what is the current in the secondary coil?

A 0.10 A

B 0.21 A

C 0.50 A

D 0.60 A

33 A metal surface in an evacuated tube is illuminated with monochromatic light causing the
emission of photoelectrons which are collected at an adjacent electrode. The experiment
is to be repeated with light of half the intensity but the same wavelength.

How will the photocurrent I and stopping potential V be affected?

A I unchanged and V unchanged

B I unchanged and V halved

C I halved and V unchanged

D I halved and V halved

34 A photon of light enters a block of glass after travelling through a vacuum. The energy of
the photon on entering the glass block

A increases because its associated wavelength decreases.

B stays the same because the speed of the radiation and the associated wavelength do
not change.

C stays the same because the frequency of the radiation does not change.

D decreases because the speed of the radiation decreases.

35 The accelerating potential difference in an X-ray tube is 20 kV.

What is the shortest wavelength of the X-ray photon emitted from the X-ray tube?

A 6.22 10 11 m

B 6.22 10 10 m

C 1.61 10 11 m

D 1.61 10 10 m

2013/PJC/PHYSICS/9646
17

36 Energy levels in low-pressure gases are represented as lines whereas in solids, the
levels are shown as bands.

What is responsible for the formation of bands?

A Solids are better electrical conductors than gases.

B Solids are not fluids but gases are fluids.

C Atoms in solids are much denser than those in gases.

D Atoms in solids are much closer together than those in gases.

37 Which statement about the energy bands in an ideal intrinsic semiconductor is correct?

A The conduction band lies just below the valence band.

B The number of electrons in the conduction band equals the number of holes in the
valence band.

C There is an energy gap of 5 eV to 10 eV between the valence and conduction band.

D There is a small overlap between the valence and conduction band.

38 Which of the following statements about laser is false?

A An external energy source is needed to create population inversion.

B The laser beam produced is coherent and of a single wavelength.

C By using a partially reflecting mirror which reflects light more efficiently, the intensity
of the laser beam produced is increased.

D Laser systems produce beams that would not spread out.

2013/PJC/PHYSICS/9646 [Turn over


18

39 The equation 235 1 121 113 1


92 U + 0 n 45 Rh + 47 Ag + 2 0 n shows the fission of a Uranium-235
nuclide by a slow moving neutron into a Rhodium-121 nuclide, a Silver-113 nuclide and
two neutrons.
binding energy per nucleon of 23592 U 7.59 MeV
121
binding energy per nucleon of 45 Rh 8.26 MeV
113
binding energy per nucleon of 47 Ag 8.52 MeV

What is the energy released during this fission process?

A 9.19 MeV

B 24.4 MeV

C 73.9 MeV

D 179 MeV

40 A parent nucleus, initially at rest, decays into two particles of masses m1 and m2, moving
away from each other in opposite directions.

If E is the total energy of the two particles, what is the energy associated with the particle
of mass m1?

m1
A E
m2

m2
B E
m1

m2
C E
m1 m2

m1
D E
m1 m 2

End of paper
2013/PJC/PHYSICS/9646
Answers to 2013 JC2 Preliminary Examination Paper 1 (H2 Physics)

1 D 6 C 11 C 16 D 21 B 26 A 31 C 36 D
2 B 7 B 12 A 17 A 22 D 27 D 32 C 37 B
3 B 8 A 13 D 18 D 23 B 28 B 33 C 38 D
4 B 9 B 14 B 19 D 24 A 29 B 34 C 39 D
5 D 10 B 15 C 20 B 25 D 30 D 35 A 40 C

F
1 Since B ,
IL

F
units of B = units of
IL
kg m s2
=
Am
= kg s2 A1

Answer: D

2 v v f v i

v v f v i

v 2 v f 2 v i 2
v 7 5
2 2

v 9 m s1

vi
tan
vf
5
tan
7
36

Answer: B

3 The acceleration-time graph is obtained from the velocity-time graph. Point Q is the
turning point of the velocity-time graph and so is the maximum value.

Answer: B
4 Let the initial speed of the javelin be u.

1
At the initial point, K.E. mu 2 E
2

1
m u cos 60
2
At the maximum height, K.E.
2
1 1
mu 2
42
1
E
4

Answer: B

5 Only D is the correct answer because it is moving at constant velocity. Resultant force
zero.

Answer: D

m
6 Rate of change of momentum of the water = v =1.0 8.0 = 8.0 kg m s1.
t
From Newtons second law of motion, force on the water = 8.0 N
From Newtons third law, force on the notice-board = 8.0 N.
F 8 .0
Hence magnitude of acceleration of the notice-board = = = 4.0 m s2.
m 2 .0

Answer: C

7 Upthrust on cargo = weight of sea- water displaced due to cargo = weight of cargo
Depth sank due to cargo(h) cross-section of barge(A) density of sea-water() g =
mg
m 2.0 10 4
h = = = 0.20 m = 20 cm
A 97 1030

Depth of bottom below surface is 70 20 = 50 cm

Answer: B

8 Extension, e = 90 60 = 30 mm
Tension = k e = 500 0.030 = 15 N

Answer: A
9 lost in kinetic energy = work done against resistive force in the plank
1
m[(150)2 (125)2 ] work done against friction
2
For the second bullet, the work done against friction stays the same when it passes
through the plank.
1 1
m[(150)2 (125)2 ] m[(90)2 (v )2 ]
2 2
v 35 m s1

Answer: B

10 Work done (WD) by applied force = (80)(3) = 240 J


WD against frictional force = friction distance moved = (30)(3) = 90 J = heat generated.
Gained in gravitational potential energy = mgh = (40)(3.0 sin 30)= 60 J
Using conservation of energy,
Gained in K.E. = 240 90 60 = 90 J

Answer: B

11 Since mass, radius and speed is the same, then the magnitude of the centripetal force
on P and Q is the same, regardless of orientation. Centripetal force is provided by the
net force towards the centre of the circular path and thus the net forces of P and Q are
equal in magnitude.

Answer: C

12 For mass m1:


m1v 2
ke
Le
ke(L e )
v2
m1
For mass m2:
m2v 2
k (2L 2e L )
2(L e )
m2v 2
k (L 2e )
2(L e )
2k (L 2e )(L e )
m2
v2
2k (L 2e )(L e )m1

ke(L e )
2m1(L 2e )

e

Answer: A
GMm
13 F
(R h)2
where R radius of Earth
h height of mountain

The height of the mountain is much smaller compared to radius of Earth. Even if the
object is taken to another mountain with twice the original elevation, this will not
significantly reduce the gravitational force acting on it.

Answer: D

GME
14 gE 9.81 Nkg1
rE 2
GMV
gV
rV 2
G(0.8)ME

(0.9rE )2
9.7 Nkg1

Answer: B

15 The potential energy of the oscillator is maximum when the oscillator is momentarily at
rest.

Answer: C

2 t
16 25 50 sin
2
1
t= s
6

5
Hence it remained closed for 1 s .
6

Answer: D

17 Applying conservation of energy,


heat lost by aluminium block = heat gained by ice heat gained by water and melted ice
heat gained by calorimeter
(0.100)(924)(95) = m(3.36 105) (0.095)(4200)(5) (0.05)(924)(5)
m 0.0195 kg

Answer: A

3
18 mean translational k.e. per gas molecule = kT and is independent of mass.
2
Answer: D
19 Absolute temperature scale does not depend on the property of any particular
substance.

Answer: D

20 For the first tuning fork:



L
4
340 850
340 850(4L)
L 0.100m

For the second tuning fork:


second
2L
4
second 8L
340 f second
340 f (8L)
340 f (0.8)
f 425Hz

Answer: B

21 d sin n
d sin 40 3
d 3

sin 40
d sin
Since n

d
Maximum value for n

3

sin 40
4

Answer: B

22 All statements are correct except D.


The magnitude of the force exerted on the ion, FE qE
FE 4.8 10 19 4200 2.0 10 15 N

Answer: D
23 When a positive charge is moved in the direction as the electric field, it will lose electric
potential energy. Hence negative work is done on the charge.

W.D. on charge = electric force distance


= qE distance
= 2.6 10 8 300000 4.0 10 3
= 3.1 105 J

Answer: B

24 Option A: Correct. Because when connected in series to a battery, a common current


passes through both X and Y, power = I 2R , so Px 2 Py .
Option B: Incorrect. Since wires X and Y are made from the same material, they have
the same resistivity.
Option C: Incorrect. Because when connected in parallel to a battery, a common p.d. is
1
experienced by both X and Y, current in X = current of Y.
2
l 1
Option D: Incorrect. Since R , cross-sectional area of X = cross-sectional area of
A 2
Y, for the same length of the two wires.

Answer: A

25 Option A: Incorrect. RXY = 0.85R.


Option B: Incorrect. RXY = 0.92R.
Option C: Incorrect. RXY = 1.25R.
Option D: Correct. RXY = 1.33R.

Answer: D

26 The largest reading on the ammeter means largest current passing through it. Ammeter
has a resistance of 2 .

Consider p.d. across PQ = V

V
Option A: Correct. Current passing through ammeter, I , because ammeter is in
2
parallel connection with the 1 and 2 resistors.
V
Option B: Incorrect. Current passing through ammeter, I .
2.67
V
Option C: Incorrect. Current passing through ammeter, I .
5
V
Option D: Incorrect. Current passing through ammeter, I .
3

Answer: A
27 The electric field will cause the electrons to experience a force towards the left. The
magnetic field will cause the electrons to experience a force downwards. Since the
magnitude of the two forces is the same, the beam of electrons will be deflected as
shown in option D.

Answer: D

28 When the current in P increases, the magnetic forces of repulsion increase, but are the
same on both P and Q. Given that the masses are the same, the angle of deflection will
be the same for both, with only an increase in magnitude.

Answer: B

29 When the switch is closed, there is a change in magnetic flux through the ring. By
Faradays law, an e.m.f. is induced in the ring. By Lenzs law, the direction of induced
current is such that it opposes the change in magnetic flux causing it. Hence, the
induced current in the ring causes the ring to move away from the solenoid.

Answer: B

30 For the magnet to fall slower, the induced upward magnetic force on the magnet has to
increase, or the resultant downward force on the magnet has to decrease.

Option A: Releasing the magnet from a smaller height will cause the rate of change of
magnetic flux through the pipe to decrease since the magnet enters the pipe
with a lower speed. Hence, the induced e.m.f. in the pipe will be of a smaller
value, which will result in a smaller current and induced magnetic force.
Option B: A pipe with a higher resistivity will cause the induced current to be of a
smaller value, which will result in a smaller induced magnetic force.
Option C: A weaker magnet will cause the induced e.m.f. to be of a smaller value, since
the rate of change of magnetic flux through the pipe will decrease.
Option D: The induced magnetic force is the same and the resultant downward force will
decrease due to the smaller weight of the magnet.

Answer: D

T T
22 12
31 IRMS 2 2 1.58 A
T
2
P IRMS R 1.582 100 250 W

Answer: C
32 For 100 % efficiency,
Is N p

I p Ns
Np 1200
Is Ip 0.25 0.60 A
Ns 500
For 83 % efficiency,
Is 0.83 0.60 0.50 A

Answer: C

33 Having the wavelength and frequency constant would mean that the energy of the
photon remains constant (E = hf). The most energetic electron would have the same KE
as before; hence the stopping potential is unchanged. However, halving the intensity of
radiation will halved the photocurrent as the rate of emission of photoelectron is halved.

Answer: C

34 As photon passes through the glass, its speed decreases. This decrease is due to its
wavelength decrease, while the frequency remains the same. Since the energy of the
photon depends on frequency, energy remains constant.

Answer: C

35 The potential energy gained by the electron = qV = 3.20 1015 J


hc
3.20 1015 J

6.22 1011 m

Answer: A

36 When atoms are close together, there are mutual interactions amongst the atoms and
bands are formed.

Answer: D

37 The atoms in an intrinsic semiconductor are neutral. When electrons move from the
valence band to the conduction band, the number of holes created is equal to the number
of electrons.

Answer: B

38 There would be diffraction since the slits through which the laser come out are narrow.

Answer: D

39 Energy released (121 8.26) (113 8.52) (235 7.59) 179 MeV

Answer: D
40 By conservation of momentum, m1v1 m2v 2

v1 m2
. (1)
v 2 m1
1
Energy of m1 , E1 m1v12 . (2)
2

1
Energy of m2 , E2 m2v 22 . (3)
2
E E1 E2

(2) E1 m1v12 m2
:
(3) E2 m2v 22 m1
E1 m
2
E E1 m1
m2
E1 E
m1 m2

Answer: C
Name Class Index Number

PIONEER JUNIOR COLLEGE


JC2 Preliminary Examination

PHYSICS 9646/02
Higher 2

Paper 2 Structured Questions


20 September 2013

1 hour 45 minutes
Candidates answer on the Question Paper.
No Additional Materials are required.

READ THESE INSTRUCTIONS FIRST

Write your name, class and index number on all the work you hand in.
Write in dark blue or black pen.
You may use a soft pencil for any diagrams, graphs or rough working.
Do not use staples, paper clips, highlighters, glue or correction fluid.

Answer all questions.

At the end of the examination, fasten all your work securely together.
The number of marks is given in brackets [ ] at the end of each question or part question.

For Examiners Use


1 / 10
2 / 8
3 / 7
4 / 10
5 / 9
6 / 16
7 / 12
Total / 72

This document consists of 21 printed pages.

2013/PJC/PHYSICS/9646 [Turn over


2

Data

speed of light in free space, c 3.00 10 8 m s1

permeability of free space, 0 4 10 7 H m1

permittivity of free space, 0 8.85 10 12 F m1

1 36 10 9 F m1

elementary charge, e 1.60 10 19 C

the Planck constant, h 6.63 10 34 J s

unified atomic mass constant, u 1.66 10 27 kg

rest mass of electron, me 9.11 10 31 kg

rest mass of proton, m p 1.67 10 27 kg

molar gas constant, R 8.31 J K1 mol1

the Avogadro constant, N A 6.02 10 23 mol1

the Boltzmann constant, k 1.38 10 23 J K1

gravitational constant, G 6.67 10 11 N m2 kg2

acceleration of free fall, g 9.81 m s2

2013/PJC/PHYSICS/9646
3

Formulae

1 2
uniformly accelerated motion, s ut at
2
v 2 u 2 2as

work done on/by a gas, W pV

hydrostatic pressure, p gh

Gm
gravitational potential,
r

displacement of particle in s.h.m., x x 0 sin t

velocity of particle in s.h.m., v v 0 cos t


2
x 0 x 2

3
mean kinetic energy of a molecule E kT
2
of an ideal gas,

resistors in series, R R1 R2 ...

resistors in parallel, 1/ R 1/ R1 1/ R2 ...

Q
electric potential, V
4 0 r

alternating current/voltage, x x 0 sin t

8 2 mU E
transmission coefficient, T exp 2kd where k
h2

radioactive decay, x x 0 exp(t )

0.693
decay constant,
t1
2

2013/PJC/PHYSICS/9646 [Turn over


4

1 In a nuclear reactor, a fast moving neutron with initial speed u1 makes a head-on elastic
collision with a stationary nucleus of carbon-12 which has a mass 12 times that of the
neutron. The speeds of the neutron and the carbon nucleus after the collision are v1 and
v2 respectively.

(a) What is meant by a head-on elastic collision?

.......................................................................................................................................

.......................................................................................................................................

.......................................................................................................................................

................................................................................................................................. [2]

(b) (i) State the principle of conservation of linear momentum.

..................................................................................................................................

..................................................................................................................................

............................................................................................................................ [1]

(ii) Determine the ratio of the final speed of the neutron v1 to its initial speed u1.

ratio = ........................................ [3]

2013/PJC/PHYSICS/9646
5

(c) Hence determine the fraction of the kinetic energy of the neutron that is transferred to
the carbon nucleus.

fraction = ........................................ [2]

(d) In nuclear engineering, a neutron moderator is a medium that reduces the speed of
fast neutrons. Explain which would make a better neutron moderator, carbon-12 or
neutron.

.......................................................................................................................................

................................................................................................................................. [2]

2013/PJC/PHYSICS/9646 [Turn over


6

2 (a) Starting from the definition of work, deduce the change in the gravitational potential
energy of a mass m, when moved a distance h upwards against a gravitational field
of field strength g.

[3]

(b) A catapult consists of two strands of rubber that obeys Hookes law. Each strand has
original length of 0.200 m and each will stretch by 0.100 m when under a tension of
50 N. A mass of 0.060 kg is projected vertically upwards from the catapult after each
strand has been extended to a length of 0.350 m.

(i) Calculate the energy stored in the stretched catapult.

energy stored = ........................................ J [3]

(ii) Determine the maximum height attained by the stone if air resistance is negligible.

maximum height = ........................................ m [2]

2013/PJC/PHYSICS/9646
7

3 A vertical peg is fixed to the rim of a horizontal turntable of radius r = 15.0 cm, rotating
with a constant angular speed = 4.0 rad s1, as shown in Fig. 3.1.

turntable
peg

parallel light
P

screen
Fig. 3.1

Parallel light is incident on the turntable so that the shadow of the peg is observed on a
screen, which is normal to the incident light. At time t = 0, = 0 and the shadow of the
peg is seen at P.

At some time t, the shadow is seen at Q.

(a) (i) Write down an expression for x in terms of , r and t, where x is the distance PQ.

[1]

2013/PJC/PHYSICS/9646 [Turn over


8

(ii) Hence, prove that the shadow on the screen executes simple harmonic motion.

[2]

(b) Calculate the speed of the shadow on the screen at a displacement of 7.5 cm above
P.

speed = ........................................ m s1 [2]

(c) Calculate the maximum acceleration of the shadow.

maximum acceleration = ........................................ m s2 [2]

2013/PJC/PHYSICS/9646
9

4 Two small charged metal spheres A and B are situated in a vacuum. The distance
between the centres of the spheres is 12.0 cm, as shown in Fig. 4.1.

12.0 cm

sphere A P sphere B

x
Fig. 4.1
The charge on each sphere may be assumed to be a point charge at the centre of the
sphere. Point P is a movable point that lies on the line joining the centres of the spheres
and is distance x from the centre of sphere A.

The variation with distance x of the electric field strength E at point P is shown in Fig. 4.2.

E / 106 N C1

x / cm

Fig. 4.2

2013/PJC/PHYSICS/9646 [Turn over


10

(a) State the evidence provided by Fig. 4.2 that the spheres are conductors.

....................................................................................................................................

........................................................................................................................... [1]

(b) (i) Comment, with explanation, on the polarity of the two charges.

..................................................................................................................................

..................................................................................................................................

..................................................................................................................................

............................................................................................................................ [2]

charge on sphere A
(ii) Use Fig. 4.2 to determine the ratio .
charge on sphere B

ratio = ........................................ [2]

(iii) Hence, on Fig. 4.3, sketch the electric field lines due to these two charges.

sphere A sphere B

Fig. 4.3

[2]

2013/PJC/PHYSICS/9646
11

(c) (i) State the relation between electric field strength E and potential V.

............................................................................................................................ [1]

(ii) A negative point charge of 0.20 C is moved by an external force from the point
where x = 2.0 cm to the point where x = 8.0 cm, along the line joining the centres
of the spheres.

Use Fig. 4.2 to determine the magnitude of the net work done by the external
force in this process.

net work done = ........................................ J [2]

2013/PJC/PHYSICS/9646 [Turn over


12

5 (a) A junction is formed between slices of p-type and n-type semiconductor material, as
shown in Fig. 5.1.

p-type material n-type material

Fig. 5.1

(i) On Fig. 5.1, draw an arrow to show the direction of movement of electrons as the
two slices are brought into contact. [1]

(ii) Describe the origin of the depletion region at the junction.

..................................................................................................................................

..................................................................................................................................

..................................................................................................................................

..................................................................................................................................

..................................................................................................................................

........................................................................................................................... [3]

(iii) On Fig. 5.1, draw the symbol for a battery, connected so as to increase the width
of the depletion region. [1]

(b) By reference to the band theory of conduction, explain why the electrical resistance
of an intrinsic semiconductor material decrease as its temperature rises.

.......................................................................................................................................

.......................................................................................................................................

.......................................................................................................................................

.......................................................................................................................................

.......................................................................................................................................

.......................................................................................................................................

................................................................................................................................. [4]

2013/PJC/PHYSICS/9646
13

6 The resistance R of a small semiconductor device X varies with temperature / o C as


shown in Fig. 6.1.

R/

/ oC
Fig. 6.1

2013/PJC/PHYSICS/9646 [Turn over


14

(a) (i) The resistance and temperature of device X are related by the expression

B
R AeT

where A and B are constants and T represents the thermodynamic temperature.

Determine the values for A and B. Express the answers in the appropriate units.

A = ........................................ [2]

B = ........................................ [2]

(ii) On Fig. 6.2, sketch a graph of the variation of the current I with the potential
difference V through device X.

0 V

Fig. 6.2

[1]

2013/PJC/PHYSICS/9646
15

(b) Device X is now connected to a fixed resistor of resistance 40.0 as shown in


Fig. 6.3.
ideal
voltmeter
V

X 40.0

6.0 V
Fig. 6.3

(i) Calculate an estimated value for the voltmeter reading when device X is
immersed in water at temperature 30.0 o C .

V = ........................................ V [2]

(ii) If the temperature of the water is raised, would the voltmeter reading increase or
decrease? Explain.

..................................................................................................................................

..................................................................................................................................

........................................................................................................................... [2]

2013/PJC/PHYSICS/9646 [Turn over


16

(c) Another common semiconductor device is a diode. A diode conducts a significant


amount of current in one direction only and it is commonly used to rectify an a.c.
voltage.

(i) Fig. 6.4 shows a simple half-wave rectifier circuit. Briefly explain how your circuit
works, assuming that the diode is ideal.

a.c.
resistor
source

Fig. 6.4

..................................................................................................................................

..................................................................................................................................

..................................................................................................................................

..................................................................................................................................

........................................................................................................................... [2]

(ii) The half-wave rectifier circuit in (c)(i) is used to rectify an a.c. source that has a
peak voltage of 6 V. Calculate the r.m.s. voltage of the rectified a.c. source.

r.m.s. voltage = ........................................ V [2]

2013/PJC/PHYSICS/9646
17

(iii) Fig. 6.5 shows a diode testing circuit.

LED1

limiting a.c.
resistor LED2 source

diode X to
be tested

Fig. 6.5

The switch is closed.

State which of the two light-emitting diodes (LEDs) will light up when the diode X
to be tested

1. is working,

..................................................................................................................... [1]

2. fails due to a short-circuit in X,

..................................................................................................................... [1]

3. fails due to an open-circuit in X.

..................................................................................................................... [1]

2013/PJC/PHYSICS/9646 [Turn over


18

7 Capacitors are devices that are able to store charges, similar in a way to batteries. They
store electric energy and release it when necessary. Once connected to a battery, the
capacitor will be charged to the same potential as the e.m.f. of the battery. One plate
accumulates positive charge and the other plate accumulates negative charge.

A fully charged capacitor connected to a closed circuit will discharge the charges that it
stored previously. As the charges flow, the current in the circuit decreases as the amount
of charges stored decreases.

A simple capacitor consist of two metallic plates brought in close proximity and isolated
from one another with a dielectric substance, such as air, paper, mica, and other
insulators. The type of material used as dielectric depends on the application.

parallel plates
dielectric material

A company wishes to install a new dielectric material in its capacitors. Design an


experiment to determine the relationship between the amount of charges stored in a
capacitor and the thickness of the dielectric material used in the capacitor.

The following equipment are available: connecting wires, resistors, voltmeter,


milliammeter, batteries, galvanometer, cathode ray oscilloscope and any other
equipment normally available in a school laboratory.

You should draw a labelled diagram to show the arrangement of your apparatus. In your
account you should pay particular attention to

(a) the identification and control of variables,

(b) the equipment you would use for the investigation,

(c) the procedure to be followed,

(d) how the amount of charges stored in the capacitor would be determined,

(e) any precautions that you would take to improve the accuracy and safety of the
experiment.

2013/PJC/PHYSICS/9646
19

Diagram

....................................................................................................................................................

....................................................................................................................................................

....................................................................................................................................................

....................................................................................................................................................

....................................................................................................................................................

....................................................................................................................................................

....................................................................................................................................................

....................................................................................................................................................

....................................................................................................................................................

....................................................................................................................................................

....................................................................................................................................................

....................................................................................................................................................

....................................................................................................................................................

....................................................................................................................................................

2013/PJC/PHYSICS/9646 [Turn over


20

....................................................................................................................................................

....................................................................................................................................................

....................................................................................................................................................

....................................................................................................................................................

....................................................................................................................................................

....................................................................................................................................................

....................................................................................................................................................

....................................................................................................................................................

....................................................................................................................................................

....................................................................................................................................................

....................................................................................................................................................

....................................................................................................................................................

....................................................................................................................................................

....................................................................................................................................................

....................................................................................................................................................

....................................................................................................................................................

....................................................................................................................................................

....................................................................................................................................................

....................................................................................................................................................

....................................................................................................................................................

....................................................................................................................................................

....................................................................................................................................................

....................................................................................................................................................

....................................................................................................................................................

....................................................................................................................................................

....................................................................................................................................................

....................................................................................................................................................

2013/PJC/PHYSICS/9646
21

....................................................................................................................................................

....................................................................................................................................................

....................................................................................................................................................

....................................................................................................................................................

....................................................................................................................................................

....................................................................................................................................................

....................................................................................................................................................

....................................................................................................................................................

....................................................................................................................................................

....................................................................................................................................................

....................................................................................................................................................

....................................................................................................................................................

....................................................................................................................................................

....................................................................................................................................................

....................................................................................................................................................

....................................................................................................................................................

....................................................................................................................................................

....................................................................................................................................................

....................................................................................................................................................

....................................................................................................................................................

....................................................................................................................................................

....................................................................................................................................................

....................................................................................................................................................

....................................................................................................................................................

....................................................................................................................................................

....................................................................................................................................................

............................................................................................................................................ [12]

End of paper
2013/PJC/PHYSICS/9646 [Turn over
1

Answers to 2013 JC2 Preliminary Examination Paper 2 (H2 Physics)

Suggested Solutions:

No. Solution
1(a) A head-on collision takes place along the line joining the
centres of the colliding bodies.

An elastic collision is one in which the kinetic energy is


conserved.

1(b)(i) The total momentum of a system is constant, provided no


external resultant force acts on it.

1(b)(ii) By conservation of linear momentum,

mu1 = mv1 12mv2


==> u1 v1 = 12v2 .(1)

velocity of approach = -velocity of separation

u1 0 = v2 v1
u1 v1 = v2 (2)

2
(1) (2) 2u1 = 13 v2 ==> v2 = u1
13

2 11
From (2) v1 = v2 u1 = u1 u1 = u1
13 13
v1 11
Thus ratio =
u1 13

1(c) 1
m(u12 v12 ) v
Required fraction 2 1 [ 1 ]2
1 u1
mu12
2
11 2
1 [ ]
13
0.28

1(d) Carbon-12 atom is more massive and slows down the


neutrons while the neutron has the same mass and will not
slow down the incoming neutron.
2

2(a)
Fext

Direction of
gravitational
s=h force due to
Fext uniform field

mass m

Work done is W = Fs, where s is the displacement in the


direction of the force.
To move the mass through a vertical height h without
acceleration, an external force is needed to overcome the
weight of the mass.
Thus, Fext = mg
Work done = Fext s = mgh = change in gravitational potential
energy.

2(b)(i) Applying Hookes law,


F = ke
F 50
k 500 N m1
e 0.100
Elastic potential energy stored in both strands
1
2 ke2 ke2 (500)(0.350 0.200)2 =11.3 J
2

2(b)(ii) From conservation of energy,


gained in G.P.E. = lost in E.P.E.
mgh = 11.3
h = 19.1 m

3(a)(i) x = rsint

3(a)(ii) Differentiating x with respect to time t, we have

v = rcost

Differentiating v with respect to time t, we have

a = 2rsint

Replacing rsint by x, we have

a = 2x

Hence the shadow on the screen undergoes simple


harmonic motion.
3

3(b) v = [xo2x2]
= 4.0[0.1502 0.0752]
= 0.52 m s1
3(c) a = 2xo
= 4.02 0.15
= 2.4 m s2
4(a) Zero electric field strengths in
sphere A (between x = 0 and x = 1.4 cm) and in
sphere B (between x = 11.4 and x = 12.0 cm)

4(b)(i) The charges on the spheres are both positive because the
field strength is zero at a point between the spheres or the
electric fields are in opposite directions.

4(b)(ii) At x = 0.08 m, the electric field strength due to sphere A


cancels out the electric field strength due to sphere B.

Electric field strength due to sphere A


QA
E A --- (1)
4 o x 2
Electric field strength due to sphere B
QB
E B --- (2)
4 o 0.12 x
2

EA = EB
QA QB

4 o 0.12 x
2 2
4 o x
QA QB

4 o 0.08 4 o 0.04
2 2

2
QA 0.08
4
QB 0.04

Allow estimation from graph, 7.8 cm < x < 8.2 cm

Alternatively:

Electric field strength at surface of sphere A,


QA QB
E A --- (1)
4 o rA 4 o 0.12 rA 2
2

Electric field strength at surface of sphere B,


QB QA
E B --- (2)
4 o rB 4 o 0.12 rB 2
2

(1) / (2)
1 QA QB
2 2
4 0 rA 0.12 rA E A 115
0.676
1 QB QA EB 170

4 0 rB2 0.12 rB 2
4

QA QB Q QA
0.676 2B 2
rA 0.12 rA
2 2
rB 0.12 rB
1 0.676 0.676 1
QA 2 2
QB 2 2
rA 0.12 rB rB 0.12 rB
0.676 1 0.676 1

QA

2
rB 0.12 rB 0.6 12 1.42 3.66
2 2

QB 1 0.676 1 0.676

rA 0.12 rB
2 2
1.4 12 0.62
2

4(b)(iii) Diagram

Deduct 1 mark if any of the following is not shown.


Correct field line direction and shape.
At least more field lines radiating out of sphere A than
sphere B.
Location of neutral point nearer to sphere B.

4(c)(i) The field strength is negative of the potential gradient, i.e.


dV V
E (not ).
dx x
4(c)(ii) x 8cm
V
x 2cm
E dx
Hence, change in potential from x = 2.0 cm to x = 8.0 cm,
V = area under Ex graph (from x = 2.0 cm to x = 8.0 cm)

Counting the number of squares, estimated about


3 (1 cm 25 106 N C1) squares or
75 (2 mm 5 106 N C1) squares

V = 3 (0.01 25 106 ) 7.5 105 V


(accept any logical estimation of area under Ex graph)

Magnitude of W.D. by external force, W = q V


W.D. = 0.20 7.5 x 105 = 1.5 105 J
(accept 10 % deviation)
5

5(a)(i)
p n

Fig. 5.1

5(a)(ii) When p-type and n-type materials are placed together, free
electrons, from n-type material, diffuse across junction to fill
up holes in the p-type material producing negative ions in p-
type material leaving positively charges ions in n-type
material.

This process continues until an electric field is set up to


prevent any further diffusion of electron through the p-n
junction.

This leads to the formation of a layer depleted of any mobile


charges at the junction and this layer is called the depletion
region.

5(a)(iii)

p n

Fig. 5.1

5(b) The energy between valence band and conduction band is


narrow at 1 eV.

At 0K, there are no electrons in the conduction band and the


valence band is fully filled.

At temperatures > 0K, a significant number of electrons


become thermally excited and move into the conduction
band, leaving holes behind in the valence band.

As temperature rises, more electrons-holes pairs are


produced resulting in more charge carriers and thus
reducing the resistance.

6(a)(i) B
R Ae T

B
ln R ln A
T
Taking temperatures at 50o C(323.15K)and
80o C(353.15K) , R= 110 and 50 respectively.
6

B
ln110 ln A (1)
323.15
B
ln50 ln A (2)
353.15

(1) (2),

B B
ln110 ln50
323.15 353.15

B 3.0 103 K
A 1.02 102

6(a)(ii)

6(b)(i) At 30.0 oC, the resistance of X is approximately 188 .

By potential divider principle,

40
V 6 1.05 V
40 188

6(b)(ii) The voltmeter reading should increase.

From Fig. 6.1, as the temperature of the water is raised, the


resistance of Device X decreases. Using the potential
divider principle, the p.d. across the 40 will increase.

6(c)(i)

On the positive cycle, the diode is forward biased. The diode


conducts. On the negative cycle, the diode is reversed
biased. The diode does not conduct. Hence, the AC input is
7

half-wave rectified.

6(c)(ii)
1 T 1
Vrms 62 3V
2 2 T

6(c)(iii) 1. LED1 will be flashing but appears lighted up throughout


due to the high frequency of flashing.

2. The LEDs will light up alternately, but the human eye will
not be able to differentiate the rapid flashing, resulting in
both LEDs being seemed to be lighted up at the same
time.

3. Neither LED will light up.


8

7. Suggested solution:

Diagram (equipment to be used is shown in the diagram)

resistor R1

switch 1

parallel plate with


V
dielectric material stop watch

resistor R2 switch 2

Fig. 7.1

Aim: To investigate the relationship between the thickness of the dielectric material in a
capacitor and the amount of charge stored.

Independent variable: thickness of the dielectric material

Dependent variable: amount of charge stored in the capacitor, by determining the area
under the current-time graph as the capacitor discharges.

Controlled variables : - the e.m.f. of the battery used to charge the capacitors,
- type of dielectric material

Procedure:

(a) Select a capacitor with a certain dielectric material of thickness t and connect the
capacitor to the circuit shown in Fig. 7.1.
(b) Close switch 1, leaving switch 2 open to charge the capacitor. The capacitor is fully
charged when the voltmeter connected across the capacitor reaches a maximum
reading.
(c) With the circuit connected, record the p.d. V of the fully charged capacitor.
(d) After the capacitor is fully charged, open switch 1 to disconnect the fully charged
capacitor from the battery.
(e) Close switch 2 and immediately start the stop watch. The capacitor is discharged
through the circuit containing resistor R2.
(f) Take readings of the current flowing in the circuit every 5 s interval.
(g) Continue to obtain the value of current as the capacitor discharges until the current
reduces to a very low value (near to zero).
(h) Plot the graph of current against time and determine the area below the curve, A and
record the reading.
(i) Replace the capacitor with another dielectric thickness and repeat steps (a) to (h).
(j) Plot a graph of the area A against the thickness t of dielectric material.
9

Suggested Marks Allocation


Listing the equipment and the diagram [3]
battery connected in series to charge the capacitor
voltmeter connected across the capacitor so that the potential
difference across the capacitor be measured.
another circuit connected to discharge the capacitor
procedure [2]
Able to provide instructions for charging and discharging the capacitor
(either separate circuit or same circuit with switches)
Repeat readings with capacitor of different dielectric thickness
how the amount of charge stored in the capacitor can be measured [2]
measuring the current flowing from the capacitor during discharging
(to obtain the current vs time curve)
Integral of discharge current vs time curve to obtain the amount of
charge stored in capacitor
Identifying and control of the variables [3]
- thickness of the dielectric material (independent variable)
- amount of charge stored in the capacitor (dependent variable)
- the e.m.f. of the battery used to charge the capacitors are to be kept
constant,
- resistance of resistor R1 and R2 in the circuit to be kept constant,

precautions that would improve the accuracy [2]

Allow sufficient time during charging of the capacitor so that the


capacitor is fully charged before disconnecting from the battery.
The resistor R1 and R2 may get too hot after a while and its resistance
may vary with increasing temperature.
Handle the capacitor with insulating gloves as static charge could
discharge the stored charges easily or short circuit the capacitor.
Name Class Index Number

PIONEER JUNIOR COLLEGE


JC2 Preliminary Examination

PHYSICS 9646/03
Higher 2

Paper 3 Longer Structured Questions


24 September 2013

2 hours
Candidates answer on the Question Paper.
No Additional Materials are required.

READ THESE INSTRUCTIONS FIRST

Write your name, class and index number on all the work you hand in.
Write in dark blue or black pen.
You may use a soft pencil for any diagrams, graphs or rough working.
Do not use staples, paper clips, highlighters, glue or correction fluid.

Section A
Answer all questions.

Section B
Answer any two questions.

You are advised to spend about one hour on each section.


For Examiners Use
At the end of the examination, fasten all your work securely
1 / 8
together.
The number of marks is given in brackets [ ] at the end of 2 / 8
each question or part question.
3 / 8
4 / 8
5 / 8
6 / 20
7 / 20
8 / 20
Total / 80

This document consists of 23 printed pages.

2013/PJC/PHYSICS/9646 [Turn over


2

Data

speed of light in free space, c 3.00 10 8 m s1

permeability of free space, 0 4 10 7 H m1

permittivity of free space, 0 8.85 10 12 F m1

1 36 10 9 F m1

elementary charge, e 1.60 10 19 C

the Planck constant, h 6.63 10 34 J s

unified atomic mass constant, u 1.66 10 27 kg

rest mass of electron, me 9.11 10 31 kg

rest mass of proton, m p 1.67 10 27 kg

molar gas constant, R 8.31 J K1 mol1

the Avogadro constant, N A 6.02 10 23 mol1

the Boltzmann constant, k 1.38 10 23 J K1

gravitational constant, G 6.67 10 11 N m2 kg2

acceleration of free fall, g 9.81 m s2

2013/PJC/PHYSICS/9646
3

Formulae

1 2
uniformly accelerated motion, s ut at
2
v 2 u 2 2as

work done on/by a gas, W pV

hydrostatic pressure, p gh

Gm
gravitational potential,
r

displacement of particle in s.h.m., x x 0 sin t

velocity of particle in s.h.m., v v 0 cos t


2
x 0 x 2

3
mean kinetic energy of a molecule E kT
2
of an ideal gas,

resistors in series, R R1 R2 ...

resistors in parallel, 1/ R 1/ R1 1/ R2 ...

Q
electric potential, V
4 0 r

alternating current/voltage, x x 0 sin t

8 2 mU E
transmission coefficient, T exp 2kd where k
h2

radioactive decay, x x 0 exp(t )

0.693
decay constant,
t1
2

2013/PJC/PHYSICS/9646 [Turn over


4

Section A

Answer all questions in this section.

1 A ball is thrown vertically upwards with a velocity of 25 m s1 from ground level and then
falls back to its starting point. Fig. 1.1 shows the variation with time of the velocity of the
ball.

velocity / m s1

25

t/s
0

25

Fig. 1.1

(a) Use Fig. 1.1 to determine

(i) the time taken by the ball to reach the maximum height,

time = ........................................ s [2]

(ii) the maximum height reached by the ball.

height = ........................................ m [1]

2013/PJC/PHYSICS/9646
5

(b) (i) On Fig. 1.1, sketch a graph of the motion of the ball if air resistance is not
negligible. [2]

(ii) Explain clearly how your answers in (a) will change.

..................................................................................................................................

..................................................................................................................................

..................................................................................................................................

........................................................................................................................... [3]

2013/PJC/PHYSICS/9646 [Turn over


6

2 (a) State the principle of superposition.

........................................................................................................................................

........................................................................................................................................

................................................................................................................................. [2]

(b) Sound produced by the loudspeaker shown in Fig. 2.1 has a frequency of
4.0 103 Hz. The sound waves arrive at the microphone M via two different paths,
LXM and LYM. The left-tube is fixed in position, while the right-tube is a
sliding-section. At position M, the sound waves from the two paths interfere.

loudspeaker

sliding-section
L which can be
moved horizontally

X Y

microphone

Fig. 2.1

Initially, the lengths of paths LXM and LYM are equal. The sliding-section is then
pulled out horizontally to the right by a distance of 0.020 m, and the loudness at
microphone M changes from a maximum to a minimum.

(i) Determine the path difference between the two waves after the sliding-section is
pulled out.

path difference = ........................................ m [1]

2013/PJC/PHYSICS/9646
7

(ii) Calculate the speed at which sound travels through the tubes.

speed = ........................................ m s1 [2]

(iii) When the opening at M is sealed, explain why a standing wave can be set up in
the tube.

..................................................................................................................................

..................................................................................................................................

..................................................................................................................................

..................................................................................................................................

........................................................................................................................... [3]

2013/PJC/PHYSICS/9646 [Turn over


8

3 Fig. 3.1 shows how the resistance of a light-dependent resistor (LDR) varies with the
intensity of the light incident on it.

resistance / k

incident light intensity / W m2


Fig. 3.1

Fig. 3.2 shows a light-sensing potential divider circuit used in a lamp where the potential
difference across the LDR can be used to control the brightness of the lamp in a room.

1.2 k

9.0 V
p.d. to control
brightness of
lamp

Fig. 3.2

The battery has an e.m.f. of 9.0 V and negligible internal resistance. The
1.2 k resistor is made of carbon. When the room is in a low-light condition, the potential
difference across the LDR reaches 7.0 V.

(a) State the potential difference across the 1.2 k resistor, when the room is in a
low-light condition.

potential difference = ........................................ V [1]

2013/PJC/PHYSICS/9646
9

(b) Hence, calculate the resistance R of the LDR.

R = ........................................ k [2]

(c) Use Fig. 3.1 to determine the light intensity when the p.d. across the LDR is 7.0 V.

light intensity = ........................................ W m2 [1]

(d) Fig. 3.3 shows a close-up of the LDR device used in the circuit in Fig. 3.2. The LDR
consists of a uniform strip of semiconductor whose resistance is dependent on the
intensity of the light incident on it. The cross-sectional area of the strip is
5.0 107 m2.

10.0 mm
LDR strip
5.0 mm

Fig. 3.3

Use your answer in (b) to determine the resistivity of the LDR.

resistivity = ........................................ m [2]

2013/PJC/PHYSICS/9646 [Turn over


10

(e) Use the evidence provided by Fig. 3.1 to explain the sensitivity of this LDR for the
light-sensing circuit used in controlling the brightness of the lamp.

........................................................................................................................................

........................................................................................................................................

........................................................................................................................................

................................................................................................................................. [2]

2013/PJC/PHYSICS/9646
11

4 A coil with 100 turns is placed in a uniform magnetic field of flux density 0.35 T. The area
of the coil perpendicular to the field is 2.5 102 m2, as shown in Fig. 4.1.

axis of rotation

magnetic field

coil with 100 turns


A B

Fig. 4.1

The coil is rotated at 50 revolutions per second, starting from the position shown in
Fig. 4.1, where t 0 s.

(a) On Fig. 4.2, sketch a graph to show the variation with time of the e.m.f. induced in
the coil. Label the values on the time axis clearly.

e.m.f. / V

Emax.

t/s
0

Emax.

Fig. 4.2
[2]

2013/PJC/PHYSICS/9646 [Turn over


12

(b) For the coil rotating through the first quarter of a revolution,

(i) calculate the average e.m.f. induced across the coil AB,

average e.m.f. induced = ........................................ V [2]

(ii) explain clearly which point, A or B, is at a higher potential.

..................................................................................................................................

..................................................................................................................................

..................................................................................................................................

........................................................................................................................... [2]

(c) The maximum current flowing through the coil is 1.2 A. Hence, calculate the
maximum torque experienced by the coil due to the current.

torque = ........................................ N m [2]

2013/PJC/PHYSICS/9646
13

5 (a) What is meant by the term threshold frequency as applied to the photoelectric effect?

........................................................................................................................................

................................................................................................................................. [1]

(b) In a typical set-up of the photoelectric experiment, a metal surface is illuminated with
radiation of wavelength 450 nm, causing the emission of photoelectrons which are
collected at an adjacent electrode.

(i) Calculate the energy of a photon incident on the surface.

energy = ........................................ J [2]

(ii) The intensity of the incident radiation is 2.7 103 W m2 and the area of the metal
surface is 3.0 cm2. Calculate the number of photons incident per second on the
surface.

number per second = ........................................ [2]

2013/PJC/PHYSICS/9646 [Turn over


14

(iii) Fig. 5.1 shows a graph of how the photoelectric current, I varies with the potential
difference, V between the electrodes.

I / A

V/V
1.6 0

Fig. 5.1

Calculate the threshold wavelength of the metal.

wavelength = ........................................ m [3]

2013/PJC/PHYSICS/9646
15

Section B

Answer two questions from this section.

6 (a) Define the term gravitational field strength.

........................................................................................................................................

................................................................................................................................. [1]

(b) The acceleration of free fall g on the surface of the Earth at the North Pole is slightly
higher than that at the Equator. The Earth is assumed to be a uniform sphere.
Account for the difference in the value of g at the North Pole and the Equator.

........................................................................................................................................

........................................................................................................................................

........................................................................................................................................

................................................................................................................................. [2]

(c) A geostationary satellite, of mass 1500 kg, orbits the Earth in a circle of radius r with
period T. It is known that the two quantities are related by the equation T Ar n . The
Earth has a mass of 6.0 1024 kg and a radius of 6.4 106 m.

(i) Determine the values of n and A. Include appropriate units, if any, with your
values.

n = ........................................ [2]

A = ........................................ [2]

2013/PJC/PHYSICS/9646 [Turn over


16

(ii) Calculate the distance of the orbit from the surface of the Earth.

distance = ........................................ m [2]

(iii) Calculate the total energy of the satellite in orbit.

total energy = ........................................ J [2]

(iv) Hence, or otherwise, determine the energy required to put this satellite into orbit.

energy = ........................................ J [2]

2013/PJC/PHYSICS/9646
17

(v) Explain why the satellites orbit must lie in the plane of the equator.

..................................................................................................................................

..................................................................................................................................

..................................................................................................................................

........................................................................................................................... [2]

(vi) The satellite carries rechargeable batteries and solar cells for the reception and
transmission of data. Explain why both are necessary.

..................................................................................................................................

..................................................................................................................................

..................................................................................................................................

........................................................................................................................... [2]

(vii)As the satellite orbits the Earth, it gradually loses energy because of air
resistance.

1. State whether the total energy of the satellite becomes more or less negative.

..................................................................................................................... [1]

2. Hence, state and explain the effect of this change on the radius of the orbit.

............................................................................................................................

............................................................................................................................

..................................................................................................................... [2]

2013/PJC/PHYSICS/9646 [Turn over


18

7 (a) In a continuous flow experiment to measure specific heat capacity, water flows at a
rate of 0.150 kg min1 through a tube and is heated by a 25.2 W heater. The steady
inflow and outflow temperatures are 15.2 C and 17.4 C respectively. The rate of
flow is then increased to 0.232 kg min1 and the rate of heating to 37.8 W with the
steady inflow and outflow temperatures unchanged.

(i) State what is meant by specific heat capacity.

..................................................................................................................................

..................................................................................................................................

........................................................................................................................... [2]

(ii) Explain why it is essential to repeat the experiment with different flow rate and
heater power but maintaining the same steady state of inflow and outflow
temperatures of water.

..................................................................................................................................

..................................................................................................................................

........................................................................................................................... [1]

(iii) Determine the specific heat capacity of water.

specific heat capacity of water = ........................................ J kg1 K1 [3]

2013/PJC/PHYSICS/9646
19

(b) By considering the expansion of an ideal gas contained in a cylinder and enclosed by
a piston, show that the work done in a small expansion is equal to the product of
pressure and volume change.

[2]

(c) A cylinder of nitrogen gas, at a temperature of 20.0 C and pressure of 1.01 105 Pa,
occupies a volume of 1000 cm3. It then undergoes a two-stage change.

(i) In stage A, the gas expands at constant pressure to a volume of 1500 cm3.
Assuming that the nitrogen gas is ideal and given that the specific heat capacity
at constant pressure of nitrogen is 1.03 kJ kg1 K1 and mass of 1 mole of nitrogen
gas is 28.0 g, calculate

1. the final temperature of the gas,

final temperature = ........................................ K [2]

2. the work done by the gas in expanding,

work done = ........................................ J [1]

3. the number of moles of gas,

number of moles = ........................................ [1]

2013/PJC/PHYSICS/9646 [Turn over


20

4. the quantity of heat supplied.

heat supplied = ........................................ J [2]

(ii) In stage B, the gas is compressed isothermally to its original volume.

1. State and explain the change in internal energy in stage B.

............................................................................................................................

............................................................................................................................

..................................................................................................................... [2]

2. Sketch a labelled p V diagram showing stages A and B.

[2]

(iii) Determine the change in internal energy of the gas at the end of its two-stage
change.

change in internal energy = ........................................ J [2]

2013/PJC/PHYSICS/9646
21

8 (a) Slow moving neutrons bombard stationary Uranium-235 nuclei to produce energy in
a nuclear reactor. One possible nuclear fission reaction is

235 1
92 U 0 n 139 1
57 La X 2 0 n

(i) Explain what is meant by nuclear fission.

..................................................................................................................................

..................................................................................................................................

..................................................................................................................................

........................................................................................................................... [2]

(ii) State the number of protons and the number of neutrons in nucleus X .

number of protons = ........................................ [1]

number of neutrons = ........................................ [1]

(b) Energy released in the reaction in (a) is 200 MeV. Part of this energy is carried away
as kinetic energy of the fission products.

(i) Suggest one other mechanism by which energy is released in the fission reaction.

........................................................................................................................... [1]

(ii) Suggest one reason why a slow moving neutron is used instead of a fast moving
neutron in bombarding the Uranium nucleus.

..................................................................................................................................

........................................................................................................................... [1]

(iii) Express 200 MeV in joules.

200 MeV = ........................................ J [1]

2013/PJC/PHYSICS/9646 [Turn over


22

(iv) The reaction in (a) is used by a nuclear plant to generate thermal power of
3065 MW.

1. Calculate the number of fission processes taking place each second in the
plant.

number of fission processes = ........................................ [2]

2. The nuclear plant has an overall efficiency of 30%. Calculate the waste heat
produced in each second.

waste heat produced in each second = ........................................ J [1]

(v) 97% of the waste heat is removed using water pumped from a river and the rest
is ejected into the atmosphere. A safety regulation mandates that the water which
is used to cool the nuclear plant cannot be heated by more than 3.5 C when it is
discharged back into the river. Determine the minimum mass of water needed in
each second to cool the nuclear plant if the regulation is to be met. The specific
heat capacity of water is 4.2 103 J kg1 K1.

minimum mass of water = ........................................ kg [3]

2013/PJC/PHYSICS/9646
23

(c) In Star Trek, the Enterprise is a super long range exploratory spacecraft designed to
fly to far fringes of the Solar system. A radioisotope thermoelectric generator (RTG)
is used to provide electrical power to Enterprise. The RTG is an electrical generator
which obtains its power from the radioactive decay of Plutonium-238. Plutonium-238
has a half-life of 87.7 years.

(i) Define decay constant of a radioactive nuclide.

..................................................................................................................................

........................................................................................................................... [1]

(ii) Calculate the decay constant of Plutonium-238.

decay constant = ........................................ s1 [2]

(iii) When the RTG was manufactured, it contained 1.74 1025 atoms of
Plutonium-238. Show that the activity of the Plutonium-238 in the RTG when it
was first manufactured was 4.36 1015 Bq.

[2]

(iv) Calculate the activity of Plutonium-238 in the RTG 10 years after manufacture.

activity = ........................................ Bq [2]

End of paper
2013/PJC/PHYSICS/9646
1

Answers to 2013 JC2 Preliminary Examination Paper 3 (H2 Physics)

Suggested Solutions:

No. Solution
1(a)(i) Let t be the time taken for the ball to reach the maximum
height.

The gradient of the vt graph gives the acceleration, which


is 9.81 m s2 because the ball is falling freely.

0 25
9.81
t 0
t 2.5 s

The time taken is 2.5 s.

1(a)(ii) The area under the vt graph gives the displacement.

1
s 2.5484 25
2
s 32 m

The maximum height is 32 m.

1(b)(i)
velocity / m s1

25

t/s
0

25

1(b)(ii) The ball will experience air resistance and weight in the
same direction. Therefore, the net downward acceleration is
larger and the time taken will be shorter to reach a smaller
maximum height.

2(a) The principle of superposition states that when two or more


travelling waves of the same type meet at a point in space,
the resultant displacement at that point is the vector sum of
the displacements that the waves would separately produce
at that point.
2

2(b)(i) Path difference 2 0.020


0.040 m

2(b)(ii)
Path difference =
2
2 0.040
0.080 m

v f
4000 (0.080)
320 m s1

2(b)(iii) The sound waves from path LXM and LYM travel in the
opposite directions and meet.
Since both waves are of equal amplitude, frequency and
speed, they superpose and interfere to form a stationary
wave.

3(a) Potential difference = 2.0 V

3(b) Using V I R ,
2.0 = (1.2 103) I
I = 1.67 103 A

V 7.0
R= = 4200 or 4.2 k
I 1.67 10 3

3(c) From Fig. 3.1, light intensity = 24 W m2

3(d)
Length of strip, 10 5.0 10 3 10.0 10 3 0.060 m


Using R ,
A
RA 4200 5.0 107
3.5 102 m
0.060

3(e) Larger changes in R at low light intensities, resulting in


larger changes in the p.d. across the LDR.
Hence greater sensitivity of the LDR at low light conditions,
which is used to control the brightness of the lamp.
3

4(a)
e.m.f. / V

Emax.

t/s
0 0.01 0.02

Emax.

4(b)(i) d
E
dt
0 NBA cos

t



0 100 0.35 2.5 102
0.005
175 V

The average e.m.f. induced is 175 V.

4(b)(ii) As the coil turns, the flux linkage through the coil decreases.
Hence, the induced magnetic field should be in the same
direction as the external field.

Using right-hand grip rule, the direction of the induced


current is clockwise, which means that B is at a higher
potential than A.

4(c) The maximum torque on the coil occurs when the plane of
the coil is vertical.

Maximum torque experienced by the coil,


FB d
NB I L d
NB I A
100 0.35 1.2 2.5 102
1.05 N m
5(a) Threshold frequency fo , refers to the minimum frequency of
the illuminating source that will cause a photoelectron to be
ejected.

5(b)(i) Energy of a photon,

E
hc


6.63 1034 3 108
4.4 1019 J
450 109
4

5(b)(ii) Power incident on metal,


P = (2.7 103)(3.0 104) = 0.81 W

N
P E
t
N P 0.81
1.8 1018 s1
t E 4.4 1019

5(b)(iii) Max. K.E. = eVs = (1.6 1019)(1.6) = 2.6 1019 J

Applying Einstein Photoelectric equation,

Work function,
hf max. K.E. 4.4 1019 2.6 1019 1.8 1019 J

Threshold wavelength,


hc


6.63 1034 3 108
1.1 106 m
1.8 109

6(a) The gravitational field strength g at a point is defined as the


gravitational force per unit mass acting at that point.

6(b) At the North Pole, gravitational force produces the


acceleration due to free fall.

On the equator, since the Earth is rotating, part of the


gravitational force on a mass supplies the centripetal force
for the mass to move in circular motion.

As such, the acceleration due to free fall at the equator is


slightly lower than that at the North Pole.
5

6(c)(i) For circular motion, centripetal force = gravitational force


Fc Fg
GMm
mr 2
r2
4 2 GM
3
T2 r
4 2 r 3
T2
GM
4 2 r 3
T2
6.67 10 11(6.0 1024 )
3
T 3.1 10 7 r 2
3

7
A 3.1 10 s m 2

3
n
2
6(c)(ii) 2
T 3
r
A
2
24 3600 3
7
3.1408 10
4.23 107 m
Distance from surface of Earth 4.23 107 6.4 106
3.6 107 m

6(c)(iii) Total energy of satellite GPE KE


1 GMm
mv 2
2 r
2
1 GM GMm
m
2 r r
GMm

2r
6.67 10 11(6.0 1024 )(1500)

2(4.2287 107 )
7.096 109 J
7.1 109 J

6(c)(iv) Energy at surface of Earth GPE KE


GMm
0
rEarth
6.67 1011(6.0 1024 )(1500)

(6.4 106 )
9.3797 1010 J
6

Energy required 7.096 109 (9.3797 1010 )


8.6701 1010 J
8.7 1010 J

6(c)(v) The force of attraction to the Earth is towards its centre so


the circular orbit must be centred on the Earths centre.

Any orbiting satellite would satisfy this condition but would


have varying latitude and will not be geostationary unless it
is over the Equator.

6(c)(vi) When the satellite is under the sun, the solar cells are used
to power the equipment as well as to recharge the batteries.

When the satellite is not in the sunlight, the rechargeable


batteries are used instead.

6(c)(vii) More negative.


1.
6(c)(vii) GMm
2. Total Energy
2r
Since total energy is more negative (i.e. magnitude has
increased) and is inversely proportional to the radius of orbit,
this means the radius of orbit has decreased.

7(a)(i) The specific heat capacity c of a substance is defined as the


heat (thermal energy) per unit mass required to raise the
temperature of the substance by one unit of temperature.

7(a)(ii) The same steady inflow and outflow temperatures are


maintained for both experiments so that the rate of heat lost
is the same in both experiments so that it can be taken into
account in the conservation of energy equations when
calculating the specific heat capacity.

7(a)(iii) m1 H
V1 I1 = c 1 -------------- (1)
t1 t1
m H
V2 I2 = 2 c 2 -------------- (2)
t2 t2
H H
Since 1 = 2 ,
t1 t2
(V2 I 2 V1I1 ) (37.8 25.2)(60)
c= =
m2 m1 (0.232 0.150)(17.4 15.2)
( )
t2 t1
= 4190 J kg1 K1
7

7(b)

Work done
= force distance moved
= (pressure cross-sectional area) distance moved
= pressure change in Volume

7(c)(i)1. Using pV = nRT


Since pressure is constant, V is proportional to T for a fixed
mass of gas.

V1 / V2 = T1 / T2
T2 = (1500/1000) (273.15 20) = 440 K

7(c)(i)2. Work done by gas = p V


= 1.01 105 (0.0015 0.0010)
= 51 J
7(c)(i)3. No. of moles = PV/RT
= 1.01 105 0.0010 / 8.31 293.15
= 0.0415 moles

7(c)(i)4. Heat supplied = mc


= (no. of moles molar mass) 1030 (440 293.15)
= 0.0415 0.028 1030 (440 293.15)
= 176 J
7(c)(ii) Since the change is isothermal, i.e. no change in
1. temperature, then there is no change in internal energy in
stage B. This is because internal energy of an ideal gas is
dependent on temperature only.

7(c)(ii)
2. p / 105 Pa

440 K

Stage B

440 K
1.01
293.15 K Stage A

0 V/cm3
0 1000 1500
7(c)(iii) Since there is no change in internal energy in Stage B, the
change in internal energy at the end of the 2 stage change
is = change in internal energy in Stage A.
By the first law of thermodynamics, the change in internal
8

energy in Stage A is given by


U = Q W
= 176 ( 51)
= 125 J

8(a)(i) Nuclear fission is the splitting of a large nucleus into two or


more smaller nuclei, with the emission of a few neutrons
and/or other radiations.

8(a)(ii) Number of protons 92 57 35

Number of neutrons 236 2 139 35 60


8(b)(i) Gamma radiation
8(b)(ii) A slow moving neutron can be captured by the Uranium
nucleus as compared to a fast moving neutron and so this
will enable the fission to occur.

8(b)(iii) 200MeV 200 106 1.6 1019


3.2 1011J

8(b)(iv) total power generated number of processes per unit time


1. energy in each process
N
3065 106 3.2 10 11
t
N
9.58 1019
t
8(b)(iv) waste heat produced in 1 s 0.7 3065 106
2.
2.15 109 J

8(b)(v)
0.97 (waste heat in 1 s) (mass of water in 1 s) c
0.97 2146 106 m 4200 3.5
m 1.416 105 1.42 105 kg

8(c)(i) The decay constant of a radioactive material is the


probability of decay of a nucleus per unit time.

8(c)(ii) ln 2

87.7 365 24 60 60
2.51 1010 s1
8(c)(iii) Ao No 2.51 1010 1.74 1025
4.36 1015 Bq

3
8(c)(iv) A Ao e t 4.36 1015 e 7.9010 10

4.03 1015 Bq
NANYANG JUNIOR COLLEGE
Science Department
JC 2 PRELIMINARY EXAMINATION
Higher 1

PHYSICS 8866/01
Paper 1 Multiple Choice 18 September 2013
1 hour
Additional Materials : Multiple Choice Answer Sheet

READ THESE INSTRUCTIONS FIRST

Write in soft pencil.


Do not use staples, paper clips, highlighters, glue or correction fluid.
Write your name, Centre number and index number on the Answer Sheet in the spaces
provided unless this has been done for you.

There are thirty questions on this paper. Answer all questions. For each question there are
four possible answers A, B, C and D.
Choose the one you consider correct and record your choice in soft pencil on the separate
Answer Sheet.

Read the instructions on the Answer Sheet very carefully.

Each correct answer will score one mark. A mark will not be deducted for a wrong answer.
Any rough working should be done in this booklet.

This document consists of 18 printed pages

Nanyang Junior College

NYJC 2013 JC2/Prelim/H1/8866/01 [Turn over


2

Data

speed of light in free space, c = 3.00 x 108 m s-1


elementary charge, e = 1.60 x 10-19 C
the Planck constant, h = 6.63 x 10-34 J s
unified atomic mass constant, u = 1.66 x 10-27 kg
rest mass of electron, me = 9.11 x 10-31 kg
rest mass of proton, mp = 1.67 x 10-27 kg
acceleration of free fall, g = 9.81 m s-2

Formulae

uniformly accelerated motion, s = ut + at2


v2 = u2 + 2as
work done on/by a gas, W = pV
hydrostatic pressure, p = gh
resistors in series, R = R1 + R2 +
resistors in parallel, 1/R = 1/R1 + 1/R2 +

NYJC 2013 JC2/Prelim/H1/8866/01


3

1 The e.m.f. induced in an inductor E is the product of its inductance L and the time rate
dI
of change in its current .
dt

Which of the following gives the dimension of L in base SI units?

A kg m2 s3 B kg m2 s2 A2 C kg m s2 A2 D kg m2 s2 A2

2 The e.m.f. of a cell is calculated with measurements obtained from a potentiometer


circuit by using the formula

x
E
L

where x is the balance length = (0.432 0.001) m,


L is the length of the potentiometer = (1.000 0.004) m, and
E is the e.m.f. of the 3 V driver cell.

What is the maximum uncertainty in the measurement of E such that can be


determined to an uncertainty of no more than 1%?

A 0.004 V B 0.01 V C 0.05 V D 0.4 V

3 Fig. 3 shows the acceleration of an elevator, initially at rest on the 11th floor, over a
period of time.

acceleration (upward)

A B C D
time

Fig. 3

At which of the labeled instances is the elevator most likely back on the 11th floor?

4 A 2013 Honda Accord Sports Sedan is able to accelerate from 0 to 100 km h1 in 6.5 s.
What is the minimum distance the car requires to reach a speed of 100 km h1?

A 15 m B 25 m C 90 m D 330 m

NYJC 2013 JC2/Prelim/H1/8866/01 [Turn over


4

5 A ball is projected from the top of a cliff with a velocity of 30 m s 1 at an angle of 20o
above the horizontal. It strikes the ground below with a velocity of 52 m s 1 at an angle
of 70o below the horizontal.

30 m s1
20o

o
70
1
52 m s

Which of the following gives the change in velocity in the ball during its flight?

A B

o o
40 80

1 1
12 m s 60 m s

C D

o o
90 40
1
1 42 m s
12 m s

NYJC 2013 JC2/Prelim/H1/8866/01


5

6 A raptor of mass 5.0 kg attacks its prey which is at rest on the ground with a velocity of
20 m s-1, 30o below the horizontal. After snatching its prey of a mass 2.0 kg, it flies off
with a velocity of 15 m s-1, 10o above the horizontal.

-1 o
15 m s 30 1
o 20 m s
10
Prey
(2.0 kg) Raptor
(5.0 kg)

Which of the followings gives the correct change in momentum for the prey, change of
momentum for the raptor and the type of collision correctly?

Change in momentum of the Change in momentum of the


Collision is
prey / kg m s-1 raptor / kg m s-1
A 0 25 elastic
B 30 64 elastic
C 30 25 inelastic
D 30 64 inelastic

7 A man of mass 80 kg is standing on a weighing machine which is placed inside a lift.


The lift is moving from the first storey to the highest storey of a high rise building. The
reading on the machine is found to fluctuate about 80 kg during the journey up. Which
of the following statements is definitely correct in explaining the above phenomenon?

A The reaction on the man by the weighing machine is equal to the mans
weight.
B The reaction on the man by the weighing machine is always bigger than the
mans weight. Otherwise the man cannot move up.
C The reaction on the man by the weighing machine is always equal to the force
the man acting on the weighing machine.
D Because the gravitational field strength (g) depends on the location of the
man, the weight of the man (mg) changes when the lift rises.

NYJC 2013 JC2/Prelim/H1/8866/01 [Turn over


6

8 Water flowing along a river drives a water-wheel of radius 1.50 m as shown in the
diagram. When the water approaching the wheel has a speed of 5.20 m s-1 and that
leaving it moves at 1.80 m s-1, the wheel rotates at a uniform rate of 5.00 rad s-1. Under
these conditions the through put of water is 980 kg s-1. The net torque on the wheel
and the friction torque at the axle of the wheel are given by

1.50 m

5.20 m s-1 1.80 m s-1

Net torque / N m Frictional torque / N m


A 10 000 5000
B 5000 5000
C 0 5000
D 0 2500

NYJC 2013 JC2/Prelim/H1/8866/01


7

9 A force F is required to raise the bucket of water from a well with a constant velocity of
0.20 m s-1 as shown in the figure below. If the weight of the bucket of water is 96 N, the
radius of the axle is 10.0 cm and the length of the handle is 30.0 cm, ignoring all
frictional forces, which of the following gives the force F and the work done in 5.0 s?

30.0 cm

10.0 cm

0.20 m s-1

96N

F/N Work done in 5.0 s / J


A 32 96
B 32 32
C 96 96
D 96 32

NYJC 2013 JC2/Prelim/H1/8866/01 [Turn over


8

10 A rod is leaning against a cylinder which is fixed to the ground. The rod is in a plane
which contains a circular cross-section of the cylinder. All surfaces are rough. Which of
the following diagrams best shows the forces acting on the rod?

A B

A
B

C D

C D

11 The diagram shows two identical vessels X and Y connected by a short pipe with a
tap.

Initially, X is filled with water of mass m to a depth h and Y is empty. When the tap is
opened, water flows from X to Y until the depths of water in both vessels are equal.

How much potential energy is lost by the water during this process?

mgh mgh
A 0 B C D mgh
4 2

NYJC 2013 JC2/Prelim/H1/8866/01


9

12 Air in a bicycle pump is forced through a valve at a constant pressure P. In one stroke
of the pump the volume of air in the pump chamber is reduced from V1 to V2.

What is the work done on this air in one stroke of the pump?

A P(V1 + V2) B P(V1 + V2) C P(V1 V2) D PV1

13 A constant force F acting on a car of mass m moves the car up the slope through a
distance s at a constant velocity v. The angle of the slope to the horizontal is .

Which expression gives the efficiency of the process?

mgs sin mv mv 2 mg sin


A B C D
Fv Fs 2Fs F

NYJC 2013 JC2/Prelim/H1/8866/01 [Turn over


10

14 The diagram below shows how the displacement of a point changes when two waves
X and Y pass through it separately.
Due to
Displacement
wave X
Due to
wave Y

Time
-0.3 1.7 3.7 5.7 7.7

Wave Y has amplitude 8 cm and frequency 150 Hz.

What are the possible amplitude and frequency of wave X?

amplitude / cm frequency / Hz

A 4 50
B 4 450
C 16 50
D 16 450

NYJC 2013 JC2/Prelim/H1/8866/01


11

15 Which phenomenon cannot be shown using sound waves?

A Interference
B Polarisation
C Refraction
D Total internal reflection

16 The diagram below shows the displacement profiles of two coherent waves. X and Y
are points on the respective waves.
Displacement velocity

X
Distance

Displacement
velocity

Y Distance

What is the phase difference between point X and point Y?

A 0 B 450 C 900 D 1800

NYJC 2013 JC2/Prelim/H1/8866/01 [Turn over


12

17 Two pipes Y and Z are of the same length. Pipe Y is closed at one end and pipe Z is
open at both ends.

Pipe Y

Pipe Z

The fundamental frequency of the closed pipe Y is 240 Hz. What is the best estimate
for the fundamental frequency of the open pipe Z?

A 60 Hz B 120 Hz C 480 Hz D 960 Hz

18 A loudspeaker is placed in front of a reflector and a microphone is placed at a point


where the sound detected has maximum amplitude.

microphone
loudspeaker

reflector
As the reflector is moved to the left, the amplitude of the sound detected by the
microphone initially decreases, then increases. After a distance of x, the amplitude of
the sound detected is maximum again.

Microphone
Loudspeake
r
Reflector

Which of the following gives the value of x in terms of the wavelength of the sound?


A B C D
4 2

NYJC 2013 JC2/Prelim/H1/8866/01


13

19 P and Q are two wires of the same length but of different material. The diameter and
resistivity of P are half those of Q. P and Q are connected in parallel to a battery.

What fraction of the total current in the circuit is through P?

A 0.20 B 0.25 C 0.33 D 0.50

20 What is the definition of resistance?

A Resistance is the ratio of potential difference to current.


B Resistance is the gradient of the graph of potential difference against current.
C Resistance is the voltage required for a current of 1 A.
l
Resistance is defined by the equation R , where is the resistivity of the
D A
material, l is the length of the wire, and A is its cross-sectional area.

21 In the circuit diagram below, F is an ideal diode. The voltage supply has negligible
internal resistance and the voltmeter reads 12 V.

2.0

V
2.0

2.0

If the connections to the terminals of the voltage supply are reversed, the voltmeter
reading would be

A 6.0 V B 9.0 V C 16 V D 24 V

NYJC 2013 JC2/Prelim/H1/8866/01 [Turn over


14

22 Five resistors of equal resistance are connected as shown.

S Q

Which two points would give the largest resistance?

A PQ B PR C PS D QS

23 In the circuit shown below, resistors X and Y, of resistances 4R and 2R respectively,


are connected to a 8.0 V battery of negligible internal resistance. When a voltmeter is
connected across resistor X, it gives a reading of 4.0 V.

8.0 V

X Y

What is the reading of the voltmeter when it is connected across resistor Y?

A 1.6 V B 2.0 V C 2.7 V D 4.0 V

NYJC 2013 JC2/Prelim/H1/8866/01


15

24 The diagram shows a circuit with 4 voltmeters V, V1, V2, and V3.

Which equation must be correct?

A V = V1 + V2 + V3
B V + V1 = V2 + V3
C V3 = 2 V2
D V V1 = V3

NYJC 2013 JC2/Prelim/H1/8866/01 [Turn over


16

25 The diagram shows two long straight wires which are attached to two identical springs
at a distance x apart. When a current is supplied to each of the wires, the magnetic
forces produced on the wires extend the spring by x.

no current

wire 1 wire 2 wire 1 wire 2

x 2x

Assuming that both springs obey Hookes law, what is the new equilibrium distance
between the wires when the current in wire 2 is increase to 2?

A 2x
B between 2x and 3x
C 3x
D 4x

26 Which of the following correctly describes the change, if any, in the kinetic energy and
the momentum of the charged particle when it enters a region of magnetic field at right
angles to the field?

kinetic energy momentum


A remains the same remains the same
B remains the same change
C change remains the same
D change change

NYJC 2013 JC2/Prelim/H1/8866/01


17

27 A straight conductor of length 0.56 m carrying a current is placed perpendicularly in a


0.22 m wide region of magnetic field B of flux density 0.65 T.

0.56 m

B = 0.65 T

conductor

0.22 m

What is the magnitude and direction of the current that produces a 1.6 N force which is
directed out of the plane of the paper on the conductor?

Magnitude of current / A Direction of current


A 4.4 Right
B 4.4 Left
C 11 Right
D 11 Left

28 White light falls on a photoelectric surface causing the release of photoelectrons, many
of which are moving when released.

The threshold frequency for this surface is 5.3 x 1014 Hz, which is the frequency of
yellow light.

Which statement is correct?

A If a red filter is interposed, the number of photoelectrons released per second


will decrease to zero.
B If a violet filter is interposed, the number of photoelectrons released per
second will increase.
C If the light is focused onto a small patch, the electrons released will, on
average, be moving more quickly.
D It takes a little while for enough energy to accumulate on the surface before
any photoelectrons are released.

NYJC 2013 JC2/Prelim/H1/8866/01 [Turn over


18

29 Electromagnetic radiation of wavelength and intensity , when incident on a metal


surface, causes n electrons to be ejected per unit time. The maximum kinetic energy of
the electrons is Emax.

State the effect on Emax and n when the wavelength is reduced but the intensity is
not changed.

Emax n
A the same the same
B the same decrease
C increase the same
D increase decrease

30 Some of the energy levels of the hydrogen atoms are shown on the below.

-0.54 eV
-0.85 eV
-1.50 eV

-3.40 eV

-13.6 eV

Which of the following shows the emission line spectrum to be observed if electrons of
energy 12.84 eV are incident onto the hydrogen atom?

NYJC 2013 JC2/Prelim/H1/8866/01


NANYANG JUNIOR COLLEGE
Science Department
JC 2 PRELIMINARY EXAMINATION
Higher 1

PHYSICS 8866/01
Paper 1 Multiple Choice 18 September 2013
1 hour
Additional Materials : Multiple Choice Answer Sheet

READ THESE INSTRUCTIONS FIRST

Write in soft pencil.


Do not use staples, paper clips, highlighters, glue or correction fluid.
Write your name, Centre number and index number on the Answer Sheet in the spaces
provided unless this has been done for you.

There are thirty questions on this paper. Answer all questions. For each question there are
four possible answers A, B, C and D.
Choose the one you consider correct and record your choice in soft pencil on the separate
Answer Sheet.

Read the instructions on the Answer Sheet very carefully.

Each correct answer will score one mark. A mark will not be deducted for a wrong answer.
Any rough working should be done in this booklet.

This document consists of 17 printed pages

Nanyang Junior College

NYJC 2013 JC2/PRELIM/H1/8866/01 [Turn over


2

Data

speed of light in free space, c = 3.00 x 108 m s-1


elementary charge, e = 1.60 x 10-19 C
the Planck constant, h = 6.63 x 10-34 J s
unified atomic mass constant, u = 1.66 x 10-27 kg
rest mass of electron, me = 9.11 x 10-31 kg
rest mass of proton, mp = 1.67 x 10-27 kg
acceleration of free fall, g = 9.81 m s-2

Formulae

uniformly accelerated motion, s = ut + at2


v2 = u2 + 2as
work done on/by a gas, W = pV
hydrostatic pressure, p = gh
resistors in series, R = R1 + R2 +
resistors in parallel, 1/R = 1/R1 + 1/R2 +

NYJC 2013 JC2/PRELIM/H1/8866/01


3

1 The e.m.f. induced in an inductor E is the product of its inductance L and the time rate
dI
of change in its current .
dt

Which of the following gives the dimension of L in base SI units?

A kg m2 s3 B kg m2 s2 A2 C kg m s2 A2 D kg m2 s2 A2
Ans: B

[E] = [L] ([I] / [t])


[L] = [E] [t] / [I] = [W/Q] [t] / [I] = [W] [t] / [I]2 [t]
= kg m2 s2 A2
2 The e.m.f. of a cell is calculated with measurements obtained from a potentiometer
circuit by using the formula

x
E
L

where x is the balance length = (0.432 0.001) m,


L is the length of the potentiometer = (1.000 0.004) m, and
E is the e.m.f. of the 3 V driver cell.

What is the maximum uncertainty in the measurement of E such that can be


determined to an uncertainty of no more than 1%?

A 0.004 V B 0.01 V C 0.05 V D 0.4 V


Ans: B

/ = x / x + L / L + E / E
E / E = 0.01 0.001 / 0.432 0.004 / 1.000 = 0.0037
E = 0.0037 3 = 0.01 V

3 Fig. 3 shows the acceleration of an elevator, initially at rest on the 11th floor, over a
period of time.

acceleration (upward)

A B C D
time

Fig. 3

NYJC 2013 JC2/MidYr/H2/9646/01 [Turn over


4

At which of the labeled instances is the elevator most likely back on the 11th floor?
Ans: C

At A, elevator is moving upwards with constant velocity.


At B, elevator is at rest at a higher floor.
At C, elevator is moving downwards with constant velocity.
At D, elevator is at rest at a lower floor because the time it spent moving
downwards is longer than that moving upwards.

4 A 2013 Honda Accord Sports Sedan is able to accelerate from 0 to 100 km h1 in 6.5 s.
What is the minimum distance the car requires to reach a speed of 100 km h1?

A 15 m B 25 m C 90 m D 330 m
Ans: C

u = 0 ; v = 100 km h1 = 27.8 m s1 ; t = 6.5 s


Using v = u + a t : 27.8 = a 6.5 a = 4.27 m s2
Using s = u + a t2 : s = 4.27 6.52 = 90 m

5 A ball is projected from the top of a cliff with a velocity of 30 m s1 at an angle of 20o
above the horizontal. It strikes the ground below with a velocity of 52 m s 1 at an angle
of 70o below the horizontal.

30 m s1
20o

o
70
1
52 m s

Which of the following gives the change in velocity in the ball during its flight?

A B

NYJC 2013 JC2/PRELIM/H1/8866/01


5

o
o
40 80

1 1
12 m s 60 m s

C D

o o
90 40
1
1 42 m s
12 m s

Ans: B
1
52 m s
v

1
30 m s

6 A raptor of mass 5.0 kg attacks its prey which is at rest on the ground with a velocity of
20 m s-1, 30o below the horizontal. After snatching its prey of a mass 2.0 kg, it flies off
with a velocity of 15 m s-1, 10o above the horizontal.

Raptor

Which of the followings gives the correct change in momentum for the prey, change of
momentum for the raptor and the type of collision correctly?

Change in momentum of the Change in momentum of the


Collision is
prey / kg m s-1 raptor / kg m s-1
A 0 25 elastic
B 30 64 elastic
C 30 25 inelastic
D 30 64 inelastic

NYJC 2013 JC2/MidYr/H2/9646/01 [Turn over


6

Ans: D.

The change in momentum of prey = pf pi = (2) (15 0) = 30 kg m s-1


The change in momentum of the raptor =
mp m( pf pi ) need to use vector diagram.
5.0(30 25) 25 kg m s1 It is algebric change and not vector vector change.
After collision both stick together, so it is inelastic collision.
7 A man of mass 80 kg is standing on a weighing machine which is placed inside a lift.
The lift is moving from the first storey to the highest storey of a high rise building. The
reading on the machine is found to fluctuate about 80 kg during the journey up. Which
of the following statements is definitely correct in explaining the above phenomenon?

A The reaction on the man by the weighing machine is equal to the mans
weight.
B The reaction on the man by the weighing machine is always bigger than the
mans weight. Otherwise the man cannot move up.
C The reaction on the man by the weighing machine is always equal to the force
the man acting on the weighing machine.
D Because the gravitational field strength (g) depends on the location of the
man, the weight of the man (mg) changes when the lift rises.
Ans: C

By Newtons 3rd law


8 Water flowing along a river drives a water-wheel of radius 1.50 m as shown in the
diagram. When the water approaching the wheel has a speed of 5.20 m s-1 and that
leaving it moves at 1.80 m s-1, the wheel rotates at a uniform rate of 5.00 rad s-1. Under
these conditions the through put of water is 980 kg s-1. The net torque on the wheel
and the friction torque at the axle of the wheel are given by

NYJC 2013 JC2/PRELIM/H1/8866/01


7

1.50 m

5.20 m s-1 1.80 m s-1

Net torque / N m Frictional torque / N m


A 10 000 5000
B 5000 5000
C 0 5000
D 0 2500
Ans: C

v u 5.20 1.80
F m 980 3330 N ,
t 1.00
0 (the wheel is turning with a consta nt speed)

torque by water = 3330 x 1.50 = 5000 N water - friction = 0

5000 - friction = 0

friction = 5000 N
9 A force F is required to raise the bucket of water from a well with a constant velocity of
0.20 m s-1 as shown in the figure below. If the weight of the bucket of water is 96 N, the
radius of the axle is 10.0 cm and the length of the handle is 30.0 cm, ignoring all
frictional forces, which of the following gives the force F and the work done in 5.0 s?

NYJC 2013 JC2/MidYr/H2/9646/01 [Turn over


8

30.0 cm

10.0 cm

0.20 m s-1

96N

F/N Work done in 5.0 s / J


A 32 96
B 32 32
C 96 96
D 96 32
Ans: C

Clockwise moment = anti-clockwise moment


F x 30.0 = 96 x 10.0
F = 32 N

In 5.0 s, increase in P.E. = mgh = 96 x 0.20 x 5.0 = 96 J

NYJC 2013 JC2/PRELIM/H1/8866/01


9

10 A rod is leaning against a cylinder which is fixed to the ground. The rod is in a plane
which contains a circular cross-section of the cylinder. All surfaces are rough. Which of
the following diagrams best shows the forces acting on the rod?

A B

A
B

C D

C D

Ans: B

Three forces in equilibrium must intersect at the same point. The rod will tend to
slide down towards the right when first put. Hence the frictions will act against the
motion at the two contact points, Hence answer B.
11 The diagram shows two identical vessels X and Y connected by a short pipe with a
tap.

Initially, X is filled with water of mass m to a depth h and Y is empty. When the tap is
opened, water flows from X to Y until the depths of water in both vessels are equal.

How much potential energy is lost by the water during this process?

NYJC 2013 JC2/MidYr/H2/9646/01 [Turn over


10

mgh mgh
A 0 B C D mgh
4 2
Ans: B

Centre of mass of water at start is at .


Depth of water in both vessels at equilibrium is , hence its centre of mass is at .
Thus the loss in gravitational potential energy is .
12 Air in a bicycle pump is forced through a valve at a constant pressure P. In one stroke
of the pump the volume of air in the pump chamber is reduced from V1 to V2.

What is the work done on this air in one stroke of the pump?

A P(V1 + V2) B P(V1 + V2) C P(V1 V2) D PV1


Ans: C

Work done on the air = PV


= P(V1 V2)
13 A constant force F acting on a car of mass m moves the car up the slope through a
distance s at a constant velocity v. The angle of the slope to the horizontal is .

Which expression gives the efficiency of the process?

mgs sin mv mv 2 mg sin


A B C D
Fv Fs 2Fs F

NYJC 2013 JC2/PRELIM/H1/8866/01


11

Ans: D

14 The diagram below shows how the displacement of a point changes when two waves
X and Y pass through it separately.

Due to
Displacement
wave X
Due to
wave Y

Time
-0.3 1.7 3.7 5.7 7.7

Wave Y has amplitude 8 cm and frequency 150 Hz.

What are the possible amplitude and frequency of wave X?

amplitude / cm frequency / Hz

A 4 50
B 4 450
C 16 50
D 16 450

NYJC 2013 JC2/MidYr/H2/9646/01 [Turn over


12

Ans: C

It can be seen quite clearly that wave X has a larger amplitude than wave Y.

1
It can be seen that X has a larger period than Y. Since f , X must have a
T
smaller frequency than Y.

If you want to do it mathematically, the working is below.

Let TX be the period of X and TY be the period of Y.

TX 3TY
1 1 1

TX 3 TY
1
fX fY
3
1
(150)
3
50 Hz
15 Which phenomenon cannot be shown using sound waves?

A Interference
B Polarisation
C Refraction
D Total internal reflection
Ans: B

Since sound waves are longitudinal waves, they cannot be polarisied.


16 The diagram below shows the profile of 2 different waves. X and Y are points on the 2
different waves.

NYJC 2013 JC2/PRELIM/H1/8866/01


13

Displacement velocity

X
Distance

Displacement
velocity

Y Distance

What is the phase difference between point X and point Y?

A 0 B 450 C 900 D 1800


Ans: C

Since point X is on a sine curve and point Y is on a cosine curve and they are at
the same distance, their phase difference is 900.
17 Two pipes Y and Z are of the same length. Pipe Y is closed at one end and pipe Z is
opened at both ends.

Pipe Y

Pipe Z

The fundamental frequency of the closed pipe Y is 240 Hz. What is the best estimate
for the fundamental frequency of the open pipe Z?

A 60 Hz B 120 Hz C 480 Hz D 960 Hz

NYJC 2013 JC2/MidYr/H2/9646/01 [Turn over


14

Ans: C

Closed pipe
L = 1 ; 4L= 1
Speed of wave, v= f1 1 = 240 (4L) = 960L

Open pipe
L= 2 ; 2L= 2
Using same speed of wave, v= f2 2
960L = f2 (2L)
f2= 480 Hz
18 A loudspeaker is placed in front of a reflector and a microphone is placed at a point
where there is a maximum amplitude.

microphone
loudspeaker

reflector

As the reflector is moved to the left, the amplitude of the sound detected by the
microphone initially decreases, then increases. After a distance of x, the amplitude of
the sound detected is maximum again.

Microphone
Loudspeake
r
Reflector

Which of the following is the value of x in terms of the wavelength of the sound?


A B C D
4 2

NYJC 2013 JC2/PRELIM/H1/8866/01


15

Ans: A

To change from a maximum to a minimum, the path difference changes from n1to
(n2+1/2)The change in the path difference is
1 1 1
(n2 ) n1 [(n2 n1 ) ] (n )
2 2 2
where n is an integer. This must be equal to the change in the path difference,
which is 2x.
1
(n ) 2 x
2
n 1
x ( )
2 4
1 3 5
, , ........
4 4 4
The option which is available is A.
19 Two wires P and Q, each of the same length and different material, are connected in
parallel to a battery. The diameter and resistivity of P is half that of Q.

What fraction of the total current passes through P?

A 0.20 B 0.25 C 0.33 D 0.50


Ans: C

l l
R 2
A d

2

R
d2
2
RP 1 2 2

RQ 2 1 1
IP 1

IQ 2
1
Hence, the fraction of total current through is or 0.33.
3
20 What is the definition of resistance?

A Resistance is the potential difference per unit current.


B Resistance is the gradient of the graph of potential difference against current.
C Resistance is the voltage required for a current of 1 A.
l
Resistance is defined by the equation R , where is the resistivity of the
D A
material, l is the length of the wire, and A is its cross-sectional area.

21 In the circuit diagram below, F is an ideal diode. The voltage supply has negligible
internal resistance and the voltmeter reads 12 V.

NYJC 2013 JC2/MidYr/H2/9646/01 [Turn over


16

2.0

V
2.0

2.0

If the connections to the terminals of the voltage supply are reversed, the voltmeter
reading would be

A 6.0 V B 9.0 V C 16 V D 24 V
2.0
X
F

V
Y 2.0
Z 2.0

Ans: C

Before voltage supply is reversed, diode is reversed-bias.


Hence, p.d. across resistor Y = 12 V
Hence, p.d. across voltage supply = 12 + 12 = 24 V

After voltage supply is reversed, diode is forward-bias.

Effective resistance of Y and Z = 1.0

p.d. across X is 2/3(24) = 16 V

22 Five resistors of equal resistance are connected as shown.

NYJC 2013 JC2/PRELIM/H1/8866/01


17

S Q

Which two points would give the largest resistance?

A PQ B PR C PS D QS

23 In the circuit shown below, resistors X and Y, of resistances 4R and 2R respectively,


are connected to a 8.0 V battery of negligible internal resistance. When a voltmeter is
connected across X, it gives a reading of 4.0 V.

8.0 V

X Y

What is the reading of the voltmeter when it is connected across Y?

A 1.6 V B 2.0 V C 2.7 V D 4.0 V

24 The diagram shows a circuit with 4 voltmeters V, V1, V2, and V3.

NYJC 2013 JC2/MidYr/H2/9646/01 [Turn over


18

Which equation must be correct?

A V = V1 + V2 + V3
B V + V1 = V2 + V3
C V3 = 2 V2
D V V1 = V3

25 The diagram shows two long straight wires wire 1 and wire 2 which is attached to two
identical springs at a distance x apart. When a current is supplied to both wires, a
magnetic force is produced on each wire which extend the spring by x. Assume both
springs obeys Hook law.

NYJC 2013 JC2/PRELIM/H1/8866/01


19

no current

wire 1 wire 2 wire 1 wire 2

x 2x

Determine the new distance between wire 1 and wire 2 when the current in wire 2 is
increase to 2.

A 2x
B between 2x and 3x
C 3x
D 4x

26 Which of the following correctly describes the change, if any, in the kinetic energy and
the momentum of the charged particle when it is moving perpendicular into a region of
magnetic field?

kinetic energy momentum


A remains the same remains the same
B remains the same change
C change remains the same
D change change

27 A long conductor carrying a current is placed perpendicularly in a rectangular region of


magnetic field B of flux density 0.65 T.

NYJC 2013 JC2/MidYr/H2/9646/01 [Turn over


20

0.56 m

B = 0.65 T

conductor

0.22 m

What is the magnitude and direction of the current that produces a 1.6 N force on the
wire directed out of the plane of the paper?

Magnitude of current / A Direction of current


A 4.4 Right
B 4.4 Left
C 11 Right
D 11 Left

28 White light falls on a photoelectric surface causing the release of photoelectrons, many
of which are moving when released.

The threshold frequency for this surface is 5.3 x 1014 Hz (yellow light).

Which statement is correct?

A If a red filter is interposed, the number of photoelectrons released per second


will decrease to zero.
B If a violet filter is interposed, the number of photoelectrons released per
second will increase.
C If the light is focused onto a small patch, the electrons released will, on
average, be moving more quickly.
D It takes a little while for enough energy to accumulate on the surface before
any photoelectrons are released.

29 Electromagnetic radiation of wavelength and intensity , when incident on a metal


surface, causes n electrons to be ejected per unit time. The maximum kinetic energy of
the electrons is Emax.

State the effect on Emax and n when the wavelength is reduced but the intensity is
not changed.

Emax n

NYJC 2013 JC2/PRELIM/H1/8866/01


21

A the same the same


B the same decrease
C increase the same
D increase decrease

30 Some of the energy levels of the hydrogen atoms are shown on the below.

-13.6 eV

Which of the following shows the emission line spectrum to be observed if electrons of
energy 12.84 eV are incident onto the hydrogen atom?

NYJC 2013 JC2/MidYr/H2/9646/01 [Turn over


NANYANG JUNIOR COLLEGE
JC 2 PRELIMINARY EXAMINATION
Higher 1

CANDIDATE
NAME

TUTORS
CLASS
NAME

PHYSICS 8866/02
Paper 2 Structured Questions 18 September 2013
2 hours
Candidates answer on the Question Paper.
No Additional Materials are required

READ THESE INSTRUCTIONS FIRST


Write your name and class on all the work you hand in.
Write in dark blue or black pen on both sides of the paper.
You may use a soft pencil for any diagrams, graphs or rough working.
Do not use staples, paper clips, highlighters, glue or correction fluid.

Section A
For Examiners Use
Answer all questions.
Section A
Section B
Answer any two questions. Circle the two questions answered in the table on the 1
right of this cover page.
2
At the end of the examination, fasten all your work securely together.
The number of marks is given in brackets [ ] at the end of each question or part 3
question.
4

Section B

Total

This document consists of 23 printed pages

Nanyang Junior College

NYJC 2013 JC2/Prelim/H1/8866/02 [Turn over


2
DATA AND FORMULAE

Data

8 1
speed of light in free space c = 3.00 x 10 m s
19
elementary charge e = 1.60 x 10 C
34
the Planck constant h = 6.63 x 10 Js
27
unified atomic mass constant u = 1.66 x 10 kg
31
rest mass of electron me = 9.11 x 10 kg
27
rest mass of proton mp = 1.67 x 10 kg
2
acceleration of free fall g = 9.81 m s

Formulae

uniformly accelerated motion s = ut + at2


v2 = u2 + 2as
work done on/by a gas W = pV
hydrostatic pressure p = gh
resistors in series R = R1 + R2 + ....
resistors in parallel 1/R = 1/R1 + 1/R2 + ....

NYJC 2013 JC2/Prelim/H1/8866/02


Section A

Answer all the questions in this section.

1 (a) The critical flow speed vc is related to the width a of the obstacle, the density of the
liquid and its viscosity by the equation:

A is a dimensionless constant.

(i) Explain what is meant by dimensionless constant.

[1]

(ii) Use the method of homogeneity to determine the base units of .

base units of = [2]

(b) The equation governing the volume flow rate, V / t of a fluid under streamline conditions
through a horizontal pipe of length L and radius r is

where p is the pressure difference across the pipe and X is a property of the fluid.

In an experiment to find X for water, a student quotes the following results:

X = 1.137 x 10-3 kg m-1 s-1

V / t = 3%

p = 2%

L = 0.5%

r = 5%

NYJC 2013 JC2/Prelim/H1/8866/02 [Turn over


For
4 Ex mine s
Use

Express the result of X with its associated uncertainty to the appropriate number of
significant figures.

X =( ) x 10-3 kg m-1 s-1 [4]

2 A stationary nucleus of mass 220u undergoes radioactive decay to produce a nucleus D of


mass 216u and an -particle of mass 4u, as illustrated in Fig. 2.1.

Fig. 2.1

The initial kinetic energy of the -particle is 1.0 10-12 J.

(a) (i) State the law of conservation of linear momentum.

[1]

(ii) Explain why the initial velocities of the nucleus D and the -particle after decay
must be in opposite directions.

[2]

NYJC 2013 JC2/Prelim/H1/8866/02


For
5 Ex mine s
Use

(b) (i) Calculate the initial speed of the -particle after decay.

speed of -particle = m s-1 [2]

(ii) Calculate the speed of nucleus D immediately after decay.

speed of nucleus D = m s-1 [2]

3 (a) A student is tasked to arrange identical 100 resistors to give loads of different
resistances. Draw diagrams in each case to show the arrangement of the resistors to
give a total resistance of

(i) 150

[1]

(ii) 40 .

[2]

NYJC 2013 JC2/Prelim/H1/8866/02 [Turn over


For
6 Ex mine s
Use

(b) The graphs on Fig. 3.1 show how the resistance of a metallic conductor R and that of a
thermistor T vary with temperature.
resistance /
200

T
150
R

100

50

0
0 50 100 150 200 250 300
temperature / C
Fig. 3.1

The metallic conductor R and the thermistor T are connected in series as shown in Fig.
3.2 together with a battery of negligible internal resistance. R and T are kept at the
same temperature.

e.m.f.

A
T R


Fig. 3.2

(i) When the resistance of R is twice that of T, the ammeter reads 24 mA. Calculate
the e.m.f. of the battery.

e.m.f. = V [2]

NYJC 2013 JC2/Prelim/H1/8866/02


For
7 Ex mine s
Use

(ii) Describe the change in the ammeter reading as the temperature increases from
0C to 100C. Explain your answer clearly.

[2]

4 (a) Define the tesla.

..

....[2]

(b) A large horseshoe magnet produces a uniform magnetic field of flux density B in the
region between its poles. The flux density outside this region is negligible.
The magnet is placed on a top-pan balance and a stiff wire XY is placed with 9.0 cm of
its length between the poles of the magnet, as shown in Fig. 4.1

9.0 cm

Fig 4.1

The wire XY is horizontal and normal to the magnetic field. A direct current of 2.4 A is
passed through the wire in the direction from Y to X. The reading on the top-pan
balance decreases by 2.1 g.

NYJC 2013 JC2/Prelim/H1/8866/02 [Turn over


For
8 Ex mine s
Use

(i) State the polarity of pole P of the magnet. Explain your answer with reference to
the forces on the magnet and wire.

..

..[3]

(ii) Calculate the flux density between the poles.

flux density = .T [2]

NYJC 2013 JC2/Prelim/H1/8866/02


For
9 Ex mine s
Use

5 As part of a road-safety campaign, the braking distances of a car at different speeds were
measured.

A driver in a particular car was instructed to travel along a straight road at a constant speed v.
A signal was given to the driver to stop and he applied the brakes to bring the car to rest in
the shortest possible distance.

The total distance D travelled by the car after the signal was given was measured for
corresponding values of v. A sketch-graph of the results is shown in Fig 5.1

Fig. 5.1

(a) Explain why the sketch graph suggests that D and v are not related by an expression
of the form
D = mv + c

where m and c are constants.

[1]

(b) It is suggested that D and v may be related by an expression of the form

D = av + bv 2

where a and b are constants.

In order to test this suggestion, the data shown below are used. The uncertainties in
the measurements of D and v are not shown.

NYJC 2013 JC2/Prelim/H1/8866/02 [Turn over


For
10 Ex mine s
Use

v / m s-1 D/m D
v

10.0 14.0 1.40

13.5 22.7 1.68

18.0 36.9 2.05

22.5 52.9

27.0 74.0 2.74

31.5 97.7 3.10

(i) State the unit of the values of given in the table above
V

unit = [1]

(ii) Calculate the value of , to an appropriate number of significant digits, for v =


1
V
22.5 m s .

D
= [2]
v

D
(c) Data from the table are used to plot a graph of (y-axis) against v (x-axis). Some of
v
the data points are shown plotted below.

NYJC 2013 JC2/Prelim/H1/8866/02


For
11 Ex mine s
Use

On the graph above,

(i) plot the data points for speeds corresponding to 22.5 m s1 and 31.5 m s1.

[1]

(ii) draw the best-fit line for all the data points.

[1]

(d) Use your graph in (c) to determine

(i) the total stopping distance D for a speed of 35 m s-1

stopping distance = m [1]

NYJC 2013 JC2/Prelim/H1/8866/02 [Turn over


For
12 Ex mine s
Use

(ii) the numerical values of a and b based on the equation given in (b).

a = ___

b = [3]

(e) Each of the data points in (b) was obtained by taking the average of several values of
D for each value of v. Suggest what effect, if any, the taking of averages will have on
the uncertainties in the data points.

[2]

NYJC 2013 JC2/Prelim/H1/8866/02


For
13 Ex mine s
Use

Section B

Answer two of the questions in this section.

6 (a) Starting with the definition of work, derive the expression for change in the gravitational
potential energy of a mass m, when moved a distance h upwards against a uniform
gravitational field of field strength g. [3]

(b) An object of mass 1.5 kg is released from a stationary hot air balloon. Fig. 6.1 shows
how the velocity of the object varies with time.

Fig. 6.1

NYJC 2013 JC2/Prelim/H1/8866/02 [Turn over


For
14 Ex mine s
Use

(i) By considering the forces acting on the falling object, explain the shape of the
graph in the regions marked A and B.

[4]

(ii) Using Fig. 6.1, calculate


1. the acceleration of the object 6.0 s after it was released.

acceleration = m s-2 [2]

2. the distance fallen in the 16 s after it was released.

distance fallen = m [2]

(iii) Using your answer from (ii) 2., calculate the change in gravitational potential
energy Ep of the object that occurred during the 16 s after it was released.

Ep = J [2]

NYJC 2013 JC2/Prelim/H1/8866/02


For
15 Ex mine s
Use

(iv) Calculate the change in kinetic energy Ek of the object during the same period.

Ek = J [2]

(v) Explain why Ep and Ek are not equal to one another.

[1]

(vi) The mass strikes the ground 16 s after it was released and penetrates 0.50 m into
the ground. Calculate the resistive force acting on the mass as it penetrates the
ground, assuming that it is a constant force.

resistive force = N [4]

NYJC 2013 JC2/Prelim/H1/8866/02 [Turn over


For
16 Ex mine s
Use

7 (a) Waves on water are usually produced by wind blowing across the surface.
Under certain conditions, standing waves called seiches can be produced on a shallow
lake.
Antinodes occur at opposite ends of the lake.
Fig. 7.1 shows the cross-section of a lake where a seiche is occurring, at equal
intervals of time.

t=0s

t = 24 s

t = 48 s

Fig. 7.1

The standing waves shown in Fig. 7.1 occur in a small lake, 800 m long. They have a
period of 96 s and amplitude of 1 m.

(i) Describe how an observer viewing the lake might be aware that there were
standing waves on the lake.

..[2]

(ii) For the standing wave, label each antinode A and each node N on the bottom
diagram of Fig. 7.1. [1]

(iii) Explain why the wavelength of the water waves is 1600 m.

.[1]

NYJC 2013 JC2/Prelim/H1/8866/02


For
17 Ex mine s
Use

(iv) Given that the period of the waves is 96 s, calculate the speed of the water
waves in the lake.

speed = .. m s-1 [3]

(b) Fig. 7.2 shows a double slit experiment using a monochromatic light source.

screen

single slit double slit

0.40 mm O
light source

2.5 m

Fig. 7.2

(i) Point P is the position of the dark fringe closest to the central bright fringe at O.
State the phase difference of the two light waves arriving at P.

phase difference = .. [1]

(ii) The source emits light of wavelength 600 nm. Fig. 7.3 shows the interference
pattern on the screen.

Fig. 7.3

NYJC 2013 JC2/Prelim/H1/8866/02 [Turn over


For
18 Ex mine s
Use

Calculate the value of x.

x = . m [3]

(iii) The light source is changed to a filament lamp that emits white light. Describe the
appearance of the central fringe on the screen.

..

..

..[2]

(c) Fig. 7.4 shows two coherent sources S1 and S2 and a detector D. S1 and D are fixed
while S2 is movable. The detector D detects the waves produced by both sources.

S1 D

S2

Fig. 7.4

NYJC 2013 JC2/Prelim/H1/8866/02


For
19 Ex mine s
Use

S2 is moved to different positions and the distance S2D is recorded.


Fig. 7.5 shows how S2D varies with the path difference of the waves detected by D.

S2D / m

10

0
0 1 2 3 4 5 6 7

|S1D S2D| / m
Fig. 7.5

(i) State the distance S1D.

S1D = . m [1]

(ii) When S2 is moved to certain positions, D detects a high intensity signal. When S2
is moved to other positions, D detects a low intensity signal. Give an explanation
for this.

..

..

..

..

..

..[4]

NYJC 2013 JC2/Prelim/H1/8866/02 [Turn over


For
20 Ex mine s
Use

(iii) The sources are in phase with each other. The wave produced has a wavelength
of 2 m. Determine S2D when the waves arriving at D has a phase difference of

S2D = .. m [2]

NYJC 2013 JC2/Prelim/H1/8866/02


For
21 Ex mine s
Use

8 (a) Radio station Symphony 92.4FM in Singapore broadcasts at 92.4 MHz with a radiated
power of 43.0 kW.

(i) Calculate the magnitude of the momentum of each photon.

momentum = kg m s1 [2]

(ii) Calculate the rate of emission of photons from the station.

rate of emission = s-1 [3]

(iii) Assuming that the emitter of the radio wave radiates uniformly in all directions,
calculate the number of photons per unit area per unit time reaching a radio
positioned at a location 1.00 km away.

number per unit area per unit time = m-2 s-1 [2]

NYJC 2013 JC2/Prelim/H1/8866/02 [Turn over


For
22 Ex mine s
Use

(iv) The radio has an antenna disc of effective area 2.0 x 10-2 m2 perpendicularly
facing the incident radio wave. Calculate the force exerted on the disc, assuming
the incident radio waves are completely absorbed by the antenna.

force = N [3]

(b) A hypothetical atom has only three energy levels: the ground level, 3.21 x 10-19 J above
the ground level and 4.81 x 10-19 J above the ground level.

(i) Sketch the three energy levels, labeled with the respective values of energy.

[2]

(ii) Calculate the maximum and minimum frequencies of the spectral lines that this
atom can emit when excited.

minimum frequency = Hz___

maximum frequency = Hz [2]

NYJC 2013 JC2/Prelim/H1/8866/02


23
(iii) An electron of kinetic energy 4.65 x 10-19 J collides with a stationary atom. The atom
moves away with 8.01 x 10-21 J of kinetic energy after the collision. Calculate the kinetic
energy of the incident electron after the collision if

1. the excitation is successful,

kinetic energy = J [2]

2. the excitation is unsuccessful.

kinetic energy = J [2]

(iv) Would it be possible to excite the atom with a photon of energy 4.65 x 10-19 J? Explain.

[2]

NYJC 2013 JC2/Prelim/H1/8866/02


NANYANG JUNIOR COLLEGE
JC 2 PRELIMINARY EXAMINATION
Higher 1

CANDIDATE
NAME

TUTORS
CLASS
NAME

PHYSICS 8866/02
Paper 2 Structured Questions 18 September 2013
2 hours
Candidates answer on the Question Paper.
No Additional Materials are required

READ THESE INSTRUCTIONS FIRST


Write your name and class on all the work you hand in.
Write in dark blue or black pen on both sides of the paper.
You may use a soft pencil for any diagrams, graphs or rough working.
Do not use staples, paper clips, highlighters, glue or correction fluid.

Section A
For Examiners Use
Answer all questions.
Section A
Section B
Answer any two questions. Circle the two questions answered in the table on the 1
right of this cover page.
2
At the end of the examination, fasten all your work securely together.
The number of marks is given in brackets [ ] at the end of each question or part 3
question.
4

Section B

Total

This document consists of 23 printed pages

Nanyang Junior College

NYJC 2013 JC2/Prelim/H1/8866/02 [Turn over


2
DATA AND FORMULAE

Data

8 1
speed of light in free space c = 3.00 x 10 m s
19
elementary charge e = 1.60 x 10 C
34
the Planck constant h = 6.63 x 10 Js
27
unified atomic mass constant u = 1.66 x 10 kg
31
rest mass of electron me = 9.11 x 10 kg
27
rest mass of proton mp = 1.67 x 10 kg
2
acceleration of free fall g = 9.81 m s

Formulae

uniformly accelerated motion s = ut + at2


v2 = u2 + 2as
work done on/by a gas W = pV
hydrostatic pressure p = gh
resistors in series R = R1 + R2 + ....
resistors in parallel 1/R = 1/R1 + 1/R2 + ....

NYJC 2013 JC2/Prelim/H1/8866/02


23
Section A

Answer all the questions in this section.

1 (a) The critical flow speed vc is related to the width a of the obstacle, the density of the
liquid and its viscosity by the equation:

A is a dimensionless constant.

(i) Explain what is meant by dimensionless constant.

[1]

A dimensionless constant is a number without any units whose presence in the

equation makes the equation numerically correct.

(ii) Use the method of homogeneity to determine the base units of .

= avc / A
[ ] = [avc / A ] [1]
= m . kg m-3. m s-1 = kg m-1 s-1

base units of = [2]

(b) The equation governing the volume flow rate, V / t of a fluid under streamline conditions
through a horizontal pipe of length L and radius r is

where p is the pressure difference across the pipe and X is a property of the fluid.

In an experiment to find X for water, a student quotes the following results:

X = 1.137 x 10-3 kg m-1 s-1

V / t = 3%

p = 2%

L = 0.5%

r = 5%

Express the result of X with its associated uncertainty to the appropriate number of
significant figures.

NYJC 2013 JC2/Prelim/H1/8866/02


4

V/t = (pr4)/(8LX)
X = (pr4)/(8L V/t )
(X/X)% = (p/p)% + 4(r/r)% + (L/L)% + (V/t / V/t )%
X = [0.02 + 4(0.05) + 0.005 + 0.03] x 1.137 x 10-3
= 0.3 x 10-3 kg m-1 s-1
X = (1.1 0.3) x 10-3 kg m-1 s-1

X =( ) x 10-3 kg m-1 s-1 [4]

2 A stationary nucleus of mass 220u undergoes radioactive decay to produce a nucleus D of


mass 216u and an -particle of mass 4u, as illustrated in Fig. 2.1.

Fig. 2.1

The initial kinetic energy of the -particle is 1.0 10-12 J.

(a) (i) State the law of conservation of linear momentum.


The total momentum of a system remains constant when no net external force
acts on the system . [1]

(ii) Explain why the initial velocities of the nucleus D and the -particle after decay must
be in opposite directions.
Since the momentum of the system before decay is zero and there is no net
external force acting on the system , the total momentum of the system after
decay must also be zero, a condition which can only be satisfied when nucleus
[2]
D and the -particle
must have momenta in opposite directions [A0].
(b) (i) Determine the initial speed of the -particle after decay.
EK = m v2
= (4 x 1.66 x 10-27) v2
v = 1.74 x 107 m s-1
speed of -particle = _______________ m s-1 [2]

(ii) Calculate the speed of nucleus D immediately after decay.

NYJC 2013 JC2/Prelim/H1/8866/02


5
By the principle of conservation of momentum,

( )

speed of nucleus D = _______________ m s-1 [2]

3 (a) A student has some resistors of 100 and he is tasked to arrange them to form
circuits. Draw the circuit diagrams in each case to show how the resistors can be
connected to form a total resistance of
(i) 150 [1]

(ii) 40 [2]

(b) The graphs on Fig. 3.1 shows how the resistance of a metal resistor R and a thermistor
T varies when the temperature changes.

NYJC 2013 8866/02/PRELIM/13 [Turn over


6

200

resistance /

150 T
R

100

50

0
0 50 100 150 200 250 300
temperature / C
Fig. 3.1

The metal resistor R and the thermistor T are connected in series as shown in Fig. 3.2
together with a battery of negligible internal resistance. R and T are kept at the same
temperature as each other.
e.m.f.

A
T R

Fig. 3.2

(i) Calculate the e.m.f. of the circuit when the resistance of R is twice that of T and the
ammeter reading is 24 mA.
From the graph, R = 80, T = 40,
E = I RT = (24 x 10-3)(80+40) = 2.88 V

e.m.f. = V [2]
(ii) Describe the change in the ammeter reading as the temperature increases from
0C to 100C. Explain your answer clearly.
When the temperature increases from 0oC to 75oC, the resistance of R
increases slowly compared to resistance of T decreasing more rapidly,
thus, the overall resistance decreases from drastically to gradually. Since
the emf is constant, the ammeter reading will increase from rapidly to
slowly. When the temperature increases from 75oC to 100oC, the change in
overall resistance is decreasing very slowly as the increase in resistance of

NYJC 2013 JC2/Prelim/H1/8866/02


7
R and the decrease in resistance of T is about the same and the ammeter
reading will increase very slowly. [2]

4 (a) Define the tesla.

One tesla is defined as the strength of a magnetic field in which a force per unit length of
one newton per metre must act on a wire carrying one ampere of current in a direction
perpendicular to the field

(b) A large horseshoe magnet produces a uniform magnetic field of flux density B between its
poles. Outside the region of the poles, the flux density is zero.
The magnet is placed on a top-pan balance and a stiff wire XY of 9.0 cm is situated between
its poles as shown in Fig. 4.1

Fig 4.1
The wire XY is horizontal and normal to the magnetic field. A direct current of 2.4 A is
passed through the wire in the direction from Y to X. The reading on the top-pan balance
decreases by 2.1 g.
i) State and explain the polarity of the pole P of the magnet using Newtons law of motion.

The polarity of P is north.


When the reading on the top pan balance decreases, there must be a force
exerted on the magnet by the wire in the upwards direction. By Newtons third
law, the force exerted on the wire by the magnetic field will be downwards. Since
the current is flowing from Y to X, the direction of the magnetic field is from
moving away from P, thus, P must be north.
ii) Calculate the flux density between the poles.

Change in weight = F = BIL


(0.0021)(9.81) = B (2.4)(0.090)
B = 0.0954 T
flux density = .T [2]

NYJC 2013 8866/02/PRELIM/13 [Turn over


8
5 As part of a road-safety campaign, the braking distances of a car at different speeds were
measured.

A driver in a particular car was instructed to travel along a straight road at a constant speed v.
A signal was given to the driver to stop and he applied the brakes to bring the car to rest in
the shortest possible distance.

The total distance D travelled by the car after the signal was given was measured for
corresponding values of v. A sketch-graph of the results is shown in Fig 5.1

Fig. 5.1

(a) Explain why the sketch graph suggests that D and v are not related by an expression
of the form
D = mv + c

where m and c are constants.

Since the graph shown is not a straight line, hence the relation
between D and V is non linear. [1]

(b) It is suggested that D and v may be related by an expression of the form

D = av + bv 2

where a and b are constants.

In order to test this suggestion, the data shown below are used. The uncertainties in
the measurements of D and v are not shown.

NYJC 2013 JC2/Prelim/H1/8866/02


9

v / m s-1 D/m D
v

10.0 14.0 1.40

13.5 22.7 1.68

18.0 36.9 2.05

22.5 52.9

27.0 74.0 2.74

31.5 97.7 3.10

(i) State the unit of the values of given in the table above
V
second
unit = [1]

(ii) Calculate the value of , to an appropriate number of significant digits, for v =


1
V
22.5 m s .

D/v = 52.9/22.5 = 2.35

D 2.35 s
= [2]
v

D
(c) Data from the table are used to plot a graph of (y-axis) against v (x-axis). Some of
v
the data points are shown plotted below.

NYJC 2013 8866/02/PRELIM/13 [Turn over


10

On the graph above,

(i) plot the data points for speeds corresponding to 22.5 m s1 and 31.5 m s1.

[1]

(ii) draw the best-fit line for all the data points.

[1]

(d) Use your graph in (c) to determine

(i) the total stopping distance D for a speed of 35 m s-1

From the graph, D/v = 3.38


D = 118 m

stopping distance = m [1]

NYJC 2013 JC2/Prelim/H1/8866/02


11

(ii) the numerical values of a and b based on the equation given in (b).

D = av + bv2
D/v = bv + a
Hence gradient = b and y intercept = b

Gradient = (3.38 1.50)/(35.0 11.5) = 0.080

b = 0.60.

a = ___

b = [3]

(e) Each of the data points in (b) was obtained by taking the average of several values of
D for each value of v. Suggest what effect, if any, the taking of averages will have on
the uncertainties in the data points.

The human error when the driver react to the signal to stop is mainly a
random error. Plotting a best fit line is the best approach to minimize
the random error of the data point. Hence by taking average of of
several values may not necessary reduce the random error further. [2]

Furthermore, taking more reading may actually increase the fatigue of


the driver.

Section B

NYJC 2013 8866/02/PRELIM/13 [Turn over


12

Answer two of the questions in this section.

6 (a) Starting with the definition of work, derive the expression for change in the gravitational
potential energy of a mass m, when moved a distance h upwards against a uniform
gravitational field of field strength g. [3]

Considering the mass being raised at a constant speed, the force F must be equal to the
weight of the mass.
Therefore, the change in gravitational potential energy
= work done by force F
= F displacement in the direction of the force
= mgh

(b) An object of mass 1.5 kg is released from a stationary hot air balloon. Fig. 6.1 shows how
the velocity of the object varies with time.

Fig. 6.1

NYJC 2013 JC2/Prelim/H1/8866/02


13
(i) By considering the forces acting on the falling object, explain the shape of the graph
in the regions marked A and B.
A : As the object is moving at low velocity, it experiences negligible air resistance
and the only force acting on the object would be its weight.
Hence it has a constant acceleration and the graph has a constant gradient.

B: As the speed of the object increases, air resistance increases.


Hence acceleration of the object decreases resulting in a graph with a
decreasing gradient. ]

[4]

(ii) Using Fig. 6.1, determine


1. the acceleration of the object 6.0 s after it was released.

acceleration = gradient of the tangent to the graph at 6.0 s


= 1.4 m s-2 ( 0.2 m s-2)

acceleration = _______________ m s-2 [2]

2. the distance fallen in the 16 s after it was released.

distance fallen = area under graph


= 450 m (acceptable range: 425 m to 460 m)

distance fallen = _______________ m [2]

(iii) Using your answer from (ii)2., calculate the change in gravitational potential energy
Ep of the object that occurred during the 16 s after it was released.

Ep = mgh
= 1.5 (9.81)(450)
= 6620 J
Ep = _______________ J [2]

(iv) Calculate the change in kinetic energy Ek of the object during the same period.

EK = mv2 - mu2
= (1.5)(35.0)2 0
= 919 J [A1]

Ek = _______________ J [2]

NYJC 2013 8866/02/PRELIM/13 [Turn over


14
(v) Explain why Ep and Ek are not equal to one another.
Work is done against air resistance.

[1]

(vi) The mass strikes the ground 16 s after it was released and penetrates 0.50 m into the
ground. Determine the average resistive force acting on the mass as it penetrates the
ground.

Work done against resistive force = Loss of EK (and EP


F(0.50) = 919 (+ 1.59.810.50)
F = 1840 N (or 1850 N)

average resistive force = _______________ N [4]

7 (a) Waves on water are usually produced by wind blowing across the surface.
Under certain conditions, standing waves called seiches can be produced on a shallow
lake.
Antinodes occur at opposite ends of the lake.
Fig. 7.1 shows the cross-section of a lake where a seiche is occurring, at equal
intervals of time.

t=0s

t = 24 s

t = 48 s

Fig. 7.1

The standing waves shown in Fig. 7.1 occur in a small lake, 800 m long. They have a
period of 96 s and amplitude of 1 m.

(i) Describe how an observer viewing the lake might be aware that there were
standing waves on the lake.

NYJC 2013 JC2/Prelim/H1/8866/02


15

The observer will notice that at some positions on the lake, the surface of the
lake is always calm / has zero displacement. This implies that a node exists
at that position and there were standing waves on the lake since progressive
waves do not have nodes.
.

..[2]

(ii) For the standing wave, label each antinode A and each node N on the bottom
diagram of Fig. 7.1. [1]
Node at the centre, antinodes at both ends of the bottom diagram.

(iii) Explain why the wavelength of the water waves is 1600 m.


The distance between 2 adjacent antinodes is half the wavelength of the
standing water wave. Thus the wavelength is 1600 m.

.[1]

(iv) Given that the period of the waves is 96 s, calculate the speed of the water
waves in the lake.
v f
1

T
1
1600
96
17 m s1

speed = .. m s-1 [3]

(b) Fig. 7.2 shows a double slit experiment using a monochromatic light source.

NYJC 2013 8866/02/PRELIM/13 [Turn over


16

screen

single slit double slit

0.40 mm O
light source

2.5 m

Fig. 7.2

(i) Point P is the position of the dark fringe closest to the central bright fringe at O.
State the phase difference of the two light waves arriving at P.

phase difference = .. [1]

(ii) The source emits light of wavelength 600 nm. Fig. 7.3 shows the interference
pattern on the screen.

Fig. 7.3

Calculate the value of x.


x
a( )
5
D
5D
x
a
5 2.5 600 10 9

0.40 10 3
0.019 m

NYJC 2013 JC2/Prelim/H1/8866/02


17

x = . m [3]

(iii) The light source is changed to a filament lamp that emits white light. Describe the
appearance of the central fringe on the screen.
A white fringe is seen at the centre. Away from the centre, the fringes will
partly overlap so that the first order fringes will appear to be white but tinged
with colour at the edges.

..

..

..[2]

(c) Fig. 7.4 shows two coherent sources S1 and S2 and a detector D. S1 and D are fixed
while S2 is movable. The detector D detects the waves produced by both sources.

S1 D

S2

Fig. 7.4

S2 is moved to different positions and the distance S2D is recorded.

NYJC 2013 8866/02/PRELIM/13 [Turn over


18
Fig. 7.5 shows how S2D varies with the path difference of the waves detected by D.

S2D / m

10

0
0 1 2 3 4 5 6 7

|S1D S2D| / m
Fig. 7.5

(i) State the distance S1D.


S1D = S2D when path difference = 0 m. From Fig. 8.5, S1D = 3 m

S1D = . m [1]

(ii) When S2 is moved to certain positions, D detects a high intensity signal. When S2
is moved to other positions, D detects a low intensity signal. Give an explanation
for this.
When S2 is moved to different positions, S2D changes while the distance S1D
remains the same. The path difference of the 2 waves thus changes, leading
to changes in the phase difference of the 2 waves meeting at D.
When the waves meet in phase, constructive interference takes place and a
high intensity signal is detected.
When the waves meet out of phase, destructive interference takes place
and a low intensity signal is detected.

..

..

NYJC 2013 JC2/Prelim/H1/8866/02


19
..

..

..

..[4]
(iii) The sources are in phase with each other. The wave produced has a wavelength
of 2 m. Determine S2D when the waves arriving at D has a phase difference of
phase difference of path difference of = 1 m
From the graph, S2D = 4 m or 2 m

S2D = .. m [2]

NYJC 2013 8866/02/PRELIM/13 [Turn over


20

NYJC 2013 JC2/Prelim/H1/8866/02


21

8 (a) Radio station Symphony 92.4FM in Singapore broadcasts at 92.4 MHz with a radiated
power of 43.0 kW.

(i) Determine the magnitude of the momentum of each photon. [2]

h hf 6.63 x1034 x 92.4 x106


p 2.04 x1034 kg m s1
c 3.00 x10 8

Momentum = .......................[2]

(ii) Calculate the number of photons emitted per second from the station.

E Nhf
power
t t
N power 43.0 x103
7.02 x1029
t hf 6.63 x1034 x 92.4 x106

Number = .s-1 [3]

(iii) Assuming that the emitter of the radio wave is a point source, calculate the
number of photons per unit area per unit time reaching a radio positioned at a
location 1.00 km away.

rate of photons emitted rate of photons arriving per unit area at 1.00 km away
N N 7.02x1029
= 7.02 x1029 5.59 x1022
t tA 4 (1.00 x10 )
3 2

Number = ...m-2 s-1 [2]

(iv) If the radio has an antenna disc of effective area 2.0 x 10-2 m2 facing
perpendicularly to the incident radio wave, calculate the force exerting on the disc,
assuming all the radio waves is total absorbed by the antenna.

N rate of photons hitting the antenna


5.59 x1022 N
tA 5.59 x1022 x(2.0 x102 ) 1.12 x1021
t

NYJC 2013 8866/02/PRELIM/13 [Turn over


22

Newton's 2nd law,


N ( pf pi )
Force on photons : F 1.12x1021( 0 -2.04x10 -34 )= - 2.28x10-13 N
t
Newton's 3rd law,
Force on antenna disc = 2.28x10-13 N

Force= .N [3]

(b) A hypothetical atom has only three energy levels: the ground level, 3.21 x 10-19 J above
the ground level and 4.81 x 10-19 J above the ground level.

(i) Sketch the three energy levels, labeled with the respective values of energy.

-1.60 x 10-19 J

-4.81 x 10-19 J

[2]

(ii) Calculate the maximum and minimum frequencies of the spectral lines that this
atom can emit when excited.

(0-(-4.81 x 10-19 )
fmax = 7.26 x 1017Hz
h
(0-(-1.6 x 10-19 )
fmin = 2.4 x 1017Hz
h

Maximum frequency =....Hz

Minimum frequency =.Hz [2]

NYJC 2013 JC2/Prelim/H1/8866/02


23

(iii) An electron of kinetic energy 4.65 x 10-19 J collides with a stationary atom. The atom
moves away with 8.01 x 10-21 J of kinetic energy after the collision. Calculate the kinetic
energy of the incident electron after the collision if

1. the excitation is successful,

Initial total k.e. = final total k.e. + excitation energy


4.65 x 10-19 = 8.01 x 10-21 J + k.e. of the electron
+ excitation energy
4.48 x 10-19 = k.e. of the electron + 3.21 x 10-19
k.e. of the electron = 1.27 x 10-19 J

k.e.= ..J [2]

2. the excitation is unsuccessful.

Initial total k.e. = final total k.e.


4.65 x 10-19 = 8.01 x 10-21 J + k.e. of the electron

k.e. of the electron = 4.57 x 10-19 J


k.e. = . J [2]

(d) Would it be possible to excite the atom with a 4.65 x 10-19 J photon? Explain.

No excitation. The photon energy is too much for the atom to go to the next level and is too
little for it.
to go the 0 level. It will be scattered elastically.

..

.[2]

NYJC 2013 8866/02/PRELIM/13 [Turn over


NANYANG JUNIOR COLLEGE
Science Department
JC 2 PRELIMINARY EXAMINATION
Higher 2

PHYSICS 9646/01
Paper 1 Multiple Choice 24 September 2013

1 hour 15 minutes

Additional Materials : Multiple Choice Answer Sheet

READ THESE INSTRUCTIONS FIRST

Write in soft pencil. Do not use staples, paper clips, highlighters, glue or correction fluid.
Write your name, Centre number and index number on the Answer Sheet in the spaces
provided unless this has been done for you.

There are forty questions on this paper. Answer all questions. For each question there are
four possible answers A. B. C and D. Choose the one you consider correct and record your
choice in soft pencil on the separate Answer Sheet.

Read the instructions on the Answer Sheet very carefully.

Each correct answer will score one mark. A mark will not be deducted for a wrong answer.
Any rough working should be done in this booklet.

This document consists of 16 printed pages

Nanyang Junior College

NYJC 2013 JC2/Prelim/H2/9646/01


2

Data
speed of light in free space, c = 3.00 x 108 m s-1
permeability of free space, o = 4 x 10-7 H m-1
permittivity of free space, o = 8.85 x 10-12 Fm-1
(1 / (36 )) x 10-9 Fm-1
elementary charge, e = 1.60 x 10-19 C
the Planck constant, h = 6.63 x 10-34 J s
unified atomic mass constant, u = 1.66 x 10-27 kg
rest mass of electron, me = 9.11 x 10-31 kg
rest mass of proton, mp = 1.67 x 10-27 kg
molar gas constant, R = 8.31 J K-1 mol-1
the Avogadro constant, NA = 6.02 x 1023 mol-1
the Boltzmann constant, k = 1.38 x 10-23 J K-1
gravitational constant, G = 6.67 x 10-11 N m2 kg-2
acceleration of free fall, g = 9.81 m s-2

Formulae
uniformly accelerated motion, s = ut + at2
v2 = u2 + 2as
work done on/by a gas, W = pV
hydrostatic pressure, p = gh
gravitational potential, = Gm / r
displacement of particle in s.h.m. x = xo sin t
velocity of particle in s.h.m. v = vo cos t

= x o
2
x2
mean kinetic energy of a molecule of an ideal gas 3
E = kT
2
resistors in series, R = R1 + R2 +
resistors in parallel, 1/R = 1/R1 + 1/R2 +
electric potential, V = Q / 4or
alternating current/voltage, x = xo sin t
transmission coefficient, T exp(-2kd)

8 2 m U E
where k =
h2
radioactive decay, x = xo exp (-t)
decay constant 0.693
= t1
2

NYJC 2013 JC2/Prelim/H2/9646/01


3
1 One form of Bernoullis principle for incompressible fluid flows states that throughout the flow,
a certain quantity X remains constant.
In the following expressions, m is mass, v is velocity, P is pressure, and is density. Which
of the expressions could possibly be correct?
1 1 p 1 2 1 2 p
A X mv 2 p B X mv 2 C X v p D X v
2 2 2 2

2 Trucks P, Q, and R are at rest side by side. They begin accelerating at the same time. The
graph of their acceleration over time is shown below.

At time 2t1, which truck(s) will have covered the largest distance?

A Truck P
B Truck Q
C Truck R
D Trucks P and Q

3 A hunter aligns his rifle sight with a target. At a signal, the target falls from rest, and the
hunter fires at the same time. The bullet takes 0.8 s from the time of the signal to hit the
target.

What is the initial speed of the bullet?

A 12.5 m s1 B 25.4 m s1 C 28.0 m s1 D 29.9 m s1

NYJC 2013 JC2/Prelim/H2/9646/01


4
4 Which of the following statements is true?

A An object can only be in equilibrium if both its velocity and acceleration are zero.
B An object at rest with average acceleration a always travels a distance of at2 in time t.
C An object can be moving with constant speed but with non-zero acceleration.
D In projectile motion, an object is at rest at the peak of its trajectory because its resultant
acceleration is zero.

5 A projectile of mass m is fired from the ground with a velocity v at angle of 45o from the
horizontal.

Neglecting air resistance, what is the direction and magnitude of the change in momentum of
the projectile from the instant after it is fired to the instant just before it reaches the ground
again?

direction magnitude

A zero

B mv

C 2 mv

D 2 mv

6 A box of mass 8.0 kg rests on a horizontal, rough surface. A string attached to the box passes
over a smooth pulley and supports a 2.0 kg mass at its other end.

When the box begins to move, a frictional force of 6.0 N acts on it.

What is the acceleration of the box?

A 1.4 m s-2 B 1.7 m s-2 C 2.0 m s-2 D 2.5 m s-2

NYJC 2013 JC2/Prelim/H2/9646/01


5
7 A cyclist is moving with constant velocity as shown in the Fig. 7. The rear wheel of the
bicycle is powered by cyclist through a chain.

Fig. 7

The number of The direction of the


Resultant torque on
torque on frictional force on the rear
the rear wheel
the rear wheel wheel by the ground

A 1 left clockwise

B 2 right zero

C 2 left zero

D 1 right anticlockwise

8 A uniform beam is mounted horizontally by a hinge on the wall and a rope at the other end.
The weight of the beam is 120 N and the force of the rope is 120 N at an angle 30 o as shown.
Which of the followings gives the correct values of the reaction R on the beam by the hinge
and the angle between R and the wall?

120 N

o
30

120 N

R/N
A 60 0o
B 104 90o
C 120 60o
D 164 30o

NYJC 2013 JC2/Prelim/H2/9646/01


6
9 Fig. 9 shows a diving board held in position by two supports. A diver of weight W is hanging
stationary at the end of the board and his body displaces water of weight 0.30W. Point G is
the centre of gravity of the diving board of weight 1.4W.

Fig. 9

L L
4L

Which of the following is the correct value of the vertical force acting on the bottom support
by the diving board?

A 2.1 W B 4.2 W C 5.4 W D 6.3 W

10 A body of mass 2.0 kg falls with a terminal velocity of 20 m s-1. Which of the following gives
the correct values of the rates of work done by the weight and air resistance, and the rate of
change in total energy (kinetic energy + gravitational potential energy) of the body? (take g =
10 m s-2)

Rate of work done by Rate of work done Rate of total energy change
weight / W by air resistance / W of body / W
A +400 -400 0
B -400 +400 0
C +400 -400 -400
D zero -400 -400

NYJC 2013 JC2/Prelim/H2/9646/01


7
11 The diameter of the Singapore flyer is D and the maximum mass of each capsule with
passengers is M. One day the flyer is operating with a linear speed v with maximum
passengers in all the N capsules. Ignore all frictional forces, the total work done in moving
the whole flyer one full round with constant speed is

A 0
1
B NMgD
2
1
C NMgD NMv 2
2
1
D 2MgD NMv 2
2

12 A steel ball is placed at the top of a smooth and frictionless ramp and released. The ramp is
curved in a circular arc from X to Y.

X Y

The direction of acceleration of the ball at position Q is best represented by the vector

A B C D

13 A stone with mass m is attached to light rod. The stone is rotated in a vertical circle of radius
r with a constant angular speed as shown. The acceleration of free fall is g.

What is the difference between the maximum and minimum magnitude of the tension in the
light rod during one revolution of the stone?

A zero B 2mg C mr2 D 2mr2


rfdrfggtr

NYJC 2013 JC2/Prelim/H2/9646/01


8
14 Fig. 14 shows the variation of the resultant gravitational potential of two isolated planets X
and Y with the distance r from planet X.

X
P Y

r
2

A rocket of mass m is launched from the surface of planet X to the surface of planet Y,
Assume that the fuel of the rocket is completely consumed in a negligible distance compared
to the distance between the two planets. Which of the following statement is correct?

A In order to reach planet Y, the minimum initial kinetic energy of the rocket is given by
m(1 3).
B The minimum kinetic energy of the rocket when arriving planet Y is m(2 3).

C The mass of planet X is larger than that of planet Y.

D The gradient of the tangent of the graph at point P is the gravitational field strength
due to planet Y.

15 There are two isolated planets P and Q of masses MP and MQ respectively.


Their centres are of distance L apart and they rotate with a uniform angular velocity about
an axis S which intersects the line joining their centres perpendicularly as shown in Fig. 15.
If the distance of planet P from the centre S is R, which of the following expressions does not
give the expression for the centripetal force on planet Q?

P
L
Q Fig. 15
R S

NYJC 2013 JC2/Prelim/H2/9646/01


9

GMP MQ GMP MQ
A B C MP R 2 D MQ (L - R) 2
L R
2 2
L

16 A point mass moves with simple harmonic motion. Which of the following statements is true?

A The maximum kinetic energy of the mass is independent of the frequency of the oscillation.

B The time taken for the system to change from maximum kinetic energy to maximum
potential energy is half of the period of the oscillation.

C An oscillation system with larger amplitude will have a larger maximum potential energy.
D The period of an oscillation system is dependent on its amplitude.

17 An object undergoes simple harmonic motion with an amplitude A, and its total energy is E.
What is the displacement of the object when its kinetic energy is E/4?

3 A 3A A
A A B C D
2 4 4 2

18 A gas tank springs a leak, and gas begins leaking out quickly. Which of the following options
correctly shows the changes occurring in the gas inside the tank?

Pressure Average speed


A Increases Increases
B Increases Decreases
C Decreases Increases
D Decreases Decreases

19 The First Law of Thermodynamics can be applied to the pumping of a bicycle tyre. In this
process, air is quickly forced into the tyre by mechanical action.
According to such an analysis, how would the following quantities in the tyre change?

Internal Energy Heat supplied Work done on system


A Increases Negligible Positive
B No change Positive Negative
C Increases Negligible Zero
D No change Negative Positive

NYJC 2013 JC2/Prelim/H2/9646/01


10
20 A point source of sound emits energy equally in all directions at a constant rate and a
person 8 m from the source listens. After a while, the power of the source is halved. If the
person wishes the sound to seem as loud as before, how far should he be now from the
source?

A 2m B 22 C 42 D 82

21 The graph shows the shape at a particular instant of part of a transverse wave travelling
along a string.

P S

R
Which statement about the motion of the elements of the string is correct?

A The kinetic energy of the element at P is a maximum.

B The displacement of the element at Q is always zero, it is a node.

C The total kinetic and potential energy at the element R is a minimum.

D The acceleration of the element at S is a maximum.

22 An pipe of effective length 0.60 m is alternated opened at both ends and closed at one end.
Given that the speed of sound in air in 300 m s-1, the possible resonant frequencies are

Opened at both ends Closed at one end

A 125 Hz, 750 Hz 500 Hz, 750 Hz

B 125 Hz, 375 Hz 250 Hz, 500 Hz

C 250 Hz, 500 Hz 125 Hz, 500 Hz

D 250 Hz, 750 Hz 375 Hz, 625 Hz

23 In a diffraction grating experiment, the first order image of the 435.8 nm blue light from a
commercial mercury vapour discharge lamp occurred at an angle of 15.8o. A first order red
line was also observed at 23.7o, thought to be produced by an impurity in the mercury.

The wavelengths of red lines of various elements are listed. Which element is the impurity in
the mercury lamp?

NYJC 2013 JC2/Prelim/H2/9646/01


11
Element Wavelength/ nm

A Zinc 636.0

B Cadmium 643.3

C Hydrogen 656.3

D Neon 670.8

24 Points Q, R, S, and T are equally spaced, as shown below. A positive charge +q is placed at
R, and a negative charge q is placed at S. The work done in bringing an electron to point Q
is W.

If another positive charge +q is placed at T, what is the work done in bringing an electron to
point Q?

A 1/3 W B 2/3 W C 5/3 W D 22/9 W

25 Which of the following graphs shows the electric potential along a straight line between two
charges +Q and Q?

A B

C D

NYJC 2013 JC2/Prelim/H2/9646/01


12
26 Two oppositely charged plates are placed next to
each other, as shown.

A positively charged particle is initially at rest, at


position P. Which of the paths A, B, C, or D, is it
most likely to follow subsequently?

27 Three conductors X, Y and Z are connected using conducting plates P, Q and R of negligible
resistance in the arrangement shown below.

Y
+5V 5V
Z

Q
P R
conducting plates

The three conductors are of the same length and cross-sectional area, but the resistivities of
the materials they are made of X, Y and Z are such that X > Y > Z.
Which of the following graphs best shows the variation of potential along axis of the
conductors when a potential difference of 10 V is applied across the arrangement? The
dotted lines show the position of plates Q and R.

A B
potential potential

Z Z
distance distance
X P Q R
P Q R X
Y Y

D
C
potential
potential
X, Z
X, Z distance
distance
P Q R
P Q R Y
Y

NYJC 2013 JC2/Prelim/H2/9646/01


13

28 A cell of e.m.f. 3.20 V is connected to a 6.00 resistor in the circuit


shown below. 3.20 V

If the power dissipated by the resistor is 1.40 W, how much power is 6.00
lost in the cell?

A 0.15 W B 0.25 W C 0.43 W D 0.65 W

29 A cell of e.m.f. 2.0 V and negligible internal resistance is connected to the network of
resistors shown below. P and Q are points in the circuit.

Calculate the potential difference between P and Q.

A 0V B 0.20 V C 0.60 V D 1.0 V

30 A cell of e.m.f. 5.0 V and negligible internal resistance is connected to four similar resistors
and a variable resistor T, as shown.

V
T

5.0 V

Resistance of each resistor is 1.0 k and resistance of T is 5.0 k.

What is the reading of the ideal voltmeter?

A 0V B 2.0 V C 3.0 V D 5.0 V

NYJC 2013 JC2/Prelim/H2/9646/01


14
31 In an electric motor, a rectangular coil WXYZ has 20 turns and is in a uniform magnetic field
of flux density 0.83 T.

The lengths of sides XY and ZW are 0.17 m and of sides WX and YZ are 0.11 m. The current
in the coil is 4.5 A.

What is the maximum torque provided by the motor?

A 0.070 N m B 0.63 N m C 1.4 N m D 2.8 N m

32 A neutral sub-atomic particle is at rest in a magnetic field of flux density B. It spontaneously


decays into two particles of mass m each. One of the particles has a negative charge q.
The particles move with velocities perpendicular to the magnetic field. After what time will
the particles collide?

qB m 2m
A B C D
m qB qB

33 A circular coil has resistance R and area A. The coil is placed in a uniform magnetic field of
strength B such that its plane is perpendicular to the magnetic field lines. The plane of the
coil is now rotated through 90. The magnitude of the charge that flows in the coil is

BA R
A zero B C D BAR
R BA

34 A rectifier is connected in series with load P and an alternating voltage supply as shown in
the figure below.
Vin / V
Vin
Vo

t 2t 3t 4t 5t 6t t/s

P -Vo

Which is a possible value of the r.m.s. voltage across load P?

A 0.18 Vo B 0.50 Vo C 0.71 Vo D 1.0 Vo

NYJC 2013 JC2/Prelim/H2/9646/01


15

35 In an ideal transformer, the most important function of the soft-iron core is

A to reduce eddy-currents.
B to improve the flux-linkage between the primary and secondary coils.
C to dissipate the heat generated by the two coils.
D to produce a uniform magnetic field in the two coils

36 Fig. 36 below shows a typical X-ray spectrum produced when electrons are accelerated
through a potential difference and are then stopped in a metal target.

Fig. 36
Intensity

min 1 2 Wavelength

Which of the following is not correct?

Feature(s) on the graph Is dependent on

A The wavelengths (1, 2) of the peaks the target material.


on the graph

B The minimum wavelength min the target material


of the X-rays produced

C The minimum wavelength min the potential difference used to accelerate


of the X-rays produced the bombarding electrons.

D The maximum intensity of the X-rays the potential difference used to accelerate
produced. the bombarding electrons.

NYJC 2013 JC2/Prelim/H2/9646/01


16
37 Which statement describes correctly how scanning tunnelling microscopes work in obtaining
an atomic-scale image of a surface?
A The position of the tip of the microscope in the Z-axis must be adjusted accordingly to
keep the tunnelling current a constant during the scan on a surface lying in the X-Y
plane.
B The width of the gap between the tip of the microscope and the surface must be varied
continuously to keep the tunnelling current a constant during the scan.

C The voltage applied between the tip of the microscope and the surface must be varied
continuously in order to keep the tunnelling current a constant during the scan.

D The voltage applied between the tip of the microscope and the surface must be high
enough in order to cause a discharge in the gap and thus allowed a tunnelling current to
flow.

38 Stimulated emission occurs in the laser when a photon P causes the emission of a photon Q.
Which of the following statements is true of the photons?

A They have different frequencies but are in phase and traveling in the same direction.
B They have the same frequency, are rad out of phase and traveling in the same
direction.
C They have the same frequency, are in phase and traveling in the different directions.
D They have the same frequency, are in phase and traveling in the same direction.

39 The resistivity of the intrinsic semiconductor silicon decreases from 400 m at 20oC to 40
m at 60oC.

Which statement, using band theory, best explains the change in resistivity of silicon with an
increase in temperature?

A Conduction band electrons which carry current move faster.


B More electrons can move through the valence band taking part in conduction.
C More valence band electrons can be promoted to the conduction band.
D The energy gap between the valence and conduction bands decreases

40 What may not be conserved in nuclear processes?

A charge
B momentum
C the total number of neutrons
D the total number of nucleons

NYJC 2013 JC2/Prelim/H2/9646/01


NANYANG JUNIOR COLLEGE
Science Department
JC 2 PRELIMINARY EXAMINATION
Higher 2

PHYSICS 9646/01
Paper 1 Multiple Choice 9 July 2013

1 hour 15 minutes

Additional Materials : Multiple Choice Answer Sheet

READ THESE INSTRUCTIONS FIRST

Write in soft pencil. Do not use staples, paper clips, highlighters, glue or correction fluid. Write your
name, Centre number and index number on the Answer Sheet in the spaces provided unless this has
been done for you.

There are TWENTY-TWO questions on this paper. Answer all questions. For each question there are
four possible answers A. B. C and D. Choose the one you consider correct and record your choice in
soft pencil on the separate Answer Sheet.

Read the instructions on the Answer Sheet very carefully.

Each correct answer will score one mark. A mark will not be deducted for a wrong answer. Any
rough working should be done in this booklet.

This document consists of 9 printed pages

Nanyang Junior College


2

Data
speed of light in free space, c = 3.00 x 108 m s-1
permeability of free space, o = 4 x 10-7 H m-1
permittivity of free space, o = 8.85 x 10-12 Fm-1
(1 / (36 )) x 10-9 Fm-1
elementary charge, e = 1.60 x 10-19 C
the Planck constant, h = 6.63 x 10-34 J s
unified atomic mass constant, u = 1.66 x 10-27 kg
rest mass of electron, me = 9.11 x 10-31 kg
rest mass of proton, mp = 1.67 x 10-27 kg
molar gas constant, R = 8.31 J K-1 mol-1
the Avogadro constant, NA = 6.02 x 1023 mol-1
the Boltzmann constant, k = 1.38 x 10-23 J K-1
gravitational constant, G = 6.67 x 10-11 N m2 kg-2
acceleration of free fall, g = 9.81 m s-2

Formulae
uniformly accelerated motion, s = ut + at2
v2 = u2 + 2as
work done on/by a gas, W = pV
hydrostatic pressure, p = gh
gravitational potential, = Gm / r
displacement of particle in s.h.m. x = xo sin t
velocity of particle in s.h.m. v = vo cos t

= x o
2
x2
mean kinetic energy of a molecule of an ideal gas 3
E = kT
2
resistors in series, R = R1 + R2 +
resistors in parallel, 1/R = 1/R1 + 1/R2 +
electric potential, V = Q / 4or
alternating current/voltage, x = xo sin t
transmission coefficient, T exp(-2kd)

8 2 m U E
where k =
h2
radioactive decay, x = xo exp (-t)
decay constant 0.693
= t1
2
3
1 One form of Bernoullis principle for incompressible fluid flows states that throughout the flow,
a certain quantity X remains constant.
In the following expressions, m is mass, v is velocity, P is pressure, and is density. Which
of the expressions could possibly be correct?
1 1 p 1 2 1 2 p
A X mv 2 p B X mv 2 C X v p D X v
2 2 2 2

2 Trucks P, Q, and R are at rest side by side. They begin accelerating at the same time. The
graph of their acceleration over time is shown below.

At time 2t1, which truck(s) will have covered the largest distance?

A Truck P
B Truck Q
C Truck R
D Trucks P and Q

3 A hunter aligns his rifle sight with a target. At a signal, the target falls, and the hunter fires at
the same time. The bullet takes 0.8 s from the time of the signal to hit the target.

What is the initial speed of the bullet?

A 12.5 m s2 (u_y only)


4
B 25.4 m s2 (s_y - 0.5at2)
C 28.0 m s2
D 29.9 m s2 (s_y + 0.5at2)

4 Which of the following statements is true?

A An object can only be in equilibrium if both its velocity and acceleration are zero.
B An object at rest with average acceleration a always travels a distance of at2 in time t.
C An object can be moving with constant speed but with non-zero acceleration.
D In projectile motion, an object is at rest at the peak of its trajectory because its resultant
acceleration is zero.

5 A projectile of mass m is fired from the ground with a velocity v at angle of 45o from the
horizontal.

Neglecting air resistance, what is the direction and magnitude of the change in momentum of
the projectile from the instant after it is fired to the instant just before it reaches the ground
again?

magnitude
direction

zero
A
45o vf
mv
B v 90o

2 mv 45o
C -vi
D 2 mv

The only force acting on the projectile while it is in flight is its weight. Hence the direction of the
change in momentum, which must be the same as that of the net force acting on the projectile,
is downwards.

6 A box of mass 8.0 kg rests on a horizontal, rough surface. A string attached to the box passes
over a smooth pulley and supports a 2.0 kg mass at its other end.

When the box begins to move, a frictional force of 6.0 N acts on it.
5
What is the acceleration of the box?

A 1.4 m s-2 B 1.7 m s-2 C 2.0 m s-2 D 2.5 m s-2

Net force acting on the system = msystem asystem


(2.0)(9.81) 6.0 = (8.0 + 2.0) asystem
asystem = 1.4 m s-2 = a8.0kg

7 A cyclist is moving with constant velocity as shown in the following figure. The rear wheel of
the bicycle is powered by cyclist through a chain.

The number of The direction of the


Resultant torque on
torque on frictional force on the rear
the rear wheel
the rear wheel wheel by the ground

A 1 left clockwise

B 2 right zero

C 2 left zero

D 1 right anticlockwise

Ans: B. Two torques on the rear wheel: Clockwise driving torque from the chain and the
anticlockwise torque from the frictional force from the ground. Direction of the fictional force on
the rear wheel is forward (Newtons 3rd law, wheel pushes ground backwards, ground pushes
wheel forwards. Resultant torque on the rear wheel is zero because the wheel is rotating with
constant with constant motion.
6

8 A uniform beam is mounted horizontally by a hinge on the wall and a rope at the other end.
The weight of the beam is 120 N and the force of the rope is 120 N at an angle 30 o as shown.
Which of the followings gives the correct values of the reaction R on the beam by the hinge
and the angle between R and the wall?

R
120 N

o
30

120 N

R/N
A 60 0o
B 104 90o
C 120 60o
D 164 30o

Ans; C
Fx = 0 Rx 120 cos 30o = 0 Rx = 104 N

Fy = 0 Ry + 120 sin 30o - 120 = 0 Ry = 60 N

R= (1042 + 602)1/2 = 120 N

= tan-1 (140/60) = 60o

9 Fig. 9 shows a diving-board held in position by two supports. A diver of weight W is hanging
stationary at the end of the board with his body displaces water of weight 0.30W. Point G is
the centre of gravity of the diving board of weight 1.4W. Which of the followings is the correct
value of the vertical force acting on the bottom support by the dive board?
7

Fig. 9

L L
4L

A 2.1 W B 4.2 W C 5.4 W D 6.3 W

Ans: B. Let the force on the board at the bottom support be R.


Take moment about the left support.
R x L = 1.4 W x 2L + (W 0.3W) x 5L (At right end, upthrust on man = 0.30 W)
R = 2.8 W + 3.5 W
R = 6.3 W

10 A body of mass 2.0 kg falls with a terminal velocity of 20 m s-1. Which of the following gives
the correct values of the rates of work done by the weight and air resistance, and the rate of
change in total energy (kinetic energy + gravitational potential energy) of the body? (take g =
10 m s-2)

Rate of work done Rate of work done Rate of total energy change
by weight / W by air resistance / W of body / W
A +400 -400 0
B -400 +400 0
C +400 -400 -400
D zero -400 -400

Ans: C. Rate of W.D. by weight = Wv = mg v =2.0 x 10 x 20 = 400 W


Rate of W.D. by air resistance = R v cos 180o = - mg v = - 2.0 x 10 x 20 = - 400 W
Rate of change in total energy = rate of change of p.e. + rate of change in k.e.
= -400 W + 0 = -400 W
8

11 The diameter of the Singapore flyer is D and the


maximum mass of each capsule and passengers is in
average M. One day the flyer is operating with an
average linear speed v with maximum passengers in
all the N capsules. Ignore all frictional forces, the total
work done in moving the whole flyer one full round with
constant angular velocity is

A 0
1
B NMgD
2
1
C NMgD NMv 2
2
1
D 2MgD NMv 2
2

Ans: A. No change in gravitational p.e. and k.e. for all the capsules, so no W.D.

12 A steel ball is placed at the top of a smooth and frictionless ramp and released. The ramp is
curved in a circular arc from X to Y.

X Y

The acceleration of the ball at position Q is best represented by the vector

A B C D
9
13 A stone with mass m is attached to light rod. The stone is rotated in a vertical circle of radius r
with a constant angular speed as shown.

The acceleration of free fall is g.

What is the difference between the maximum and minimum magnitude of the tension in the
light rod during one revolution of the stone?

A zero B 2mg C mr2 D 2mr2

At the top of the circular motion:

At the bottom of the circular motion:

14 Fig. 14 shows the variation of the resultant gravitational potential of two isolated planets X
and Y with the distance r from planet A.

X
P Y

r
2

3
10

A rocket of mass m is launched from the surface of planet X to the surface of planet Y,
Assume that the fuel of the rocket is completely consumed in a negligible distance compared
to the distance between the two planets. Which of the following statement is correct?

A In order to reach planet Y, the minimum initial kinetic energy of the rocket is given by
m(1 3).
B The minimum kinetic energy of the rocket when arriving planet Y is m(2 3).

C The mass of planet X is larger than that of planet Y.

D The gradient of the tangent of the graph at point P is the gravitational field strength
due to planet Y.

Ans: B. In order to reach Y, the rocket just have to clear the point with highest potential, Hence
initial k.e. = m(2 1). If the rocket just clear the point with zero k.e., it will fall into B and
convert the p.e. m(2 3) into k.e. Mass Y > Mass X as the potential graph is skewed to the left
and gradient of the tangent is the total gravitational field strength due to both planets.

R = 4.2 W

15 There are two isolated planets P and Q of masses MP and MQ respectively.


Their centres are of distance L apart and they rotate with a uniform angular velocity about
an axis S which intersects the line joining their centres perpendicularly as shown in Fig. 15.
If the distance of planet P from the centre S is R, which of the following expressions does not
give the expression for the centripetal force on planet Q?

A
L
B Fig. 15
R S

GMP MQ GMP MQ
A B C MP R 2 D MQ (L - R) 2
L R
2
L2

Ans: B. Newtons law of gravitation. Distance between the two planets should be D and not D-
R.

16 A point mass moves with simple harmonic motion. Which of the following statements is true?
A The maximum kinetic energy of the mass is independent on the frequency of the
oscillation.
11
B The time taken for the system to change from maximum kinetic energy to maximum
potential energy is a half of the period of the oscillation.
C An oscillation system with larger amplitude will have a greater potential energy.
D The period of an oscillation system is dependent to its amplitude..

1 1
m 2 x 0 m2f x 0 and thus KEmax. depends on the frequency of
2 2 2
16 A: KE max .
2 2
the oscillation.
B: Since the max KE to max PE displacement is the from the equilibrium point to
the amplitude point, that is quarter of a period.
C: v max x 0 and thus maximum velocity is directly proportional to amplitude.
Since max velocity gives a larger KE, it also gives a larger potential energy.
D: The period of oscillation is independent of the amplitude of the oscillation.
Answer: C

17 An object undergoes simple harmonic motion with an amplitude A, and its total energy
is E. What is the displacement of the object from the amplitude position when its
kinetic energy is E/4 ?

3 A 3A A
A A B C D
2 4 4 2

1 E 1 1 v2 v o2
KEmax mv o2 E mv x2 m o v x2
2 4 2 2 4 4
x2 v o2 x2
v x02 x 2 x0 1 v o2 1 2
x02 4 xo
1 x2 3x 0 3A
1 2 x
4 x0 2 2

18 A gas tank springs a leak, and gas begins leaking out quickly. Which of the following options
correctly shows the changes occurring in the gas inside the tank?

Pressure Average speed


A Increases Increases
B Increases Decreases
C Decreases Increases
D Decreases Decreases

Answer: D
As particles of the gas escape, the gas occupies a larger volume. To do so, it must expand
against atmospheric pressure, hence work done on gas is negative. Since the gas tank is
initially in thermal equilibrium with the surroundings, initial net exchange of heat between the
escaped gas particles and the surroundings is zero. By the First Law of Thermodynamics,
internal energy thus decreases, hence average speed of particles also decreases.
12

19 The First Law of Thermodynamics can be applied to the pumping of a bicycle tyre. In this
process, air is quickly forced into the tyre by mechanical action.
According to such an analysis, how would the following quantities in the tyre change?

Internal Energy Heat supplied Work done on system


A Increases Negligible Positive
B No change Positive Negative
C Increases Negligible Zero
D No change Negative Positive

Answer: A
Work is done on the pump-tyre system for it to undergo compression, hence work done on
tyre is positive. Since there is no heat loss in tyre pumping, Q should be negligible, and by
First Law of Thermodynamics U should increase.

20 A point source of sound emits energy equally in all directions at a constant rate and a
person 8 m from the source listens. After a while, the power of the source is halved. If the
person wishes the sound to seem as loud as before, how far should he be now from the
source?

A 2m B 22 C 42 D 82

Ans : C

I = P / 4r2 = P / 4.82

When intensity is halved; I/2 = (P/2) / 4.82

For sound to seem as loud as before; P / 4.82 = (P/2) / 4.r2

2 r2 = 82 => r = 42

21 The graph shows the shape at a particular instant of part of a transverse wave travelling
along a string.

P S

R
13
Which statement about the motion of the elements of the string is correct?

A The kinetic energy of the element at P is a maximum.

B The displacement of the element at Q is always zero, it is a node.

C The total kinetic and potential energy at the element R is a minimum.

D The acceleration of the element at S is a maximum.

Ans : D

All elements in the string is in simple harmonic motion. The element at S has the greatest

amplitude; hence the greatest acceleration.

22 An pipe of effective length 0.60 m is alternated opened at both ends and closed at one end.
Given that the speed of sound in air in 300 m s-1, the possible resonant frequencies are

Opened at both ends Closed at one end

A 125 Hz, 750 Hz 500 Hz, 750 Hz

B 125 Hz, 375 Hz 250 Hz, 500 Hz

C 250 Hz, 500 Hz 125 Hz, 500 Hz

D 250 Hz, 750 Hz 375 Hz, 625 Hz

Ans : D

Open pipe :- fo = v / 2L = 250 Hz f = fo, 2fo, 3fo etc

Closed pipe :- fo = v / 4L = 125 Hz f = fo, 3fo, 5fo etc

23 In a diffraction grating experiment, the first order image of the 435.8 nm blue light from a
commercial mercury vapour discharge lamp occurred at an angle of 15.8o. A first order red
line was also observed at 23.7o, thought to be produced by an impurity in the mercury.

The wavelengths of red lines of various elements are listed. Which element is the impurity
in the mercury lamp?

Element Wavelength/ nm
14
A Zinc 636.0

B Cadmium 643.3

C Hydrogen 656.3

D Neon 670.8

Ans : B

d sin = n

Using blue light : d = n B / sin B

Using red light : (n B / sin B) sin R = n R

R = 643.3 nm

24 Points Q, R, S, and T are equally spaced, as shown below. A positive charge +q is placed at
R, and a negative charge q is placed at S. The work done in bringing an electron to point Q
is w.

If another positive charge +q is placed at T, what is the work done in bringing an electron to
point Q?

A 1/3 w
B 2/3 w
C 5/3 w
D 22/9 w

Answer: C
PE at Q due to R+S = 1/40 (1/x - 1/2x) = 1/40 (1/2x) = w
PE at Q due to R+ST+ = 1/40 (1/x - 1/2x + 1/3x) = 1/40 (5/6x) = 5/3 w

25 Which of the following graphs shows the electric potential along a straight line between two
charges +Q and Q?
15

A B

C D

Answer: D
Potential should be positive near positive charge, negative near negative charge. Zero line is
not indicated.

26 Two oppositely charged plates are placed next to each other, as shown.
A positively charged particle is initially at rest, at position P. Which of the paths A, B, C, or D,
is it most likely to follow subsequently?

Answer: C
16
Answer: C
Electric field between the two plates is as shown.
Most likely path of particle would follow the electric field lines,
as the force acting on it is directed along the lines.
17
27 Three conductors X, Y and Z are connected using conducting plates of negligible resistance in
the arrangement shown below.

Y
+5V 5V
Z

conducting plates

The three conductors are of the same length and cross-sectional area, but the resistivities of
the materials they are made of X, Y and Z are such that X > Y > Z.
Which of the following graphs best shows the variation of potential along axis of the
conductors when a potential difference of 10 V is applied across the arrangement?

potential potential

Y Y
distance distance
X
X
Z Z

A B

potential potential
X, Y
X, Y distance distance

Z
Z

C D

RX > RY > RZ RY > RZ > R(X//Z)


VY > V(X//Z) (C)
18

28 A cell of e.m.f. 3.20 V is connected to a 6.00 resistor in the circuit shown below.

3.20 V

6.00

If the power dissipated by the resistor is 1.40 W, how much power is lost in the cell?

A 0.15 W B 0.25 W C 0.43 W D 0.65 W

current in resistor = ( P / R )1/2 = (1.40 / 6.00)1/2 = 0.483 A


= current in cell
power of cell = I V = 0.483 3.20 = 1.55 W
power loss = 1.55 1.40 = 0.15 W (A)

29 A cell of e.m.f. 2.0 V and negligible internal resistance is connected to the network of
resistors shown below. P and Q are points in the circuit.

Calculate the potential difference between P and Q.

A 0V B 0.20 V C 0.60 V D 1.0 V


19

Ans: A

Using the potential divider method,


3.0 103
Potential at P 2.0
3.0 103 2.0 103
1.2 V
3.0 103
Potential at Q 2.0
3.0 103 2.0 103
1.20 V
Potential difference 1.2 1.20
0 V

30 A cell of e.m.f. 5.0 V and negligible internal resistance is connected to four similar resistors
and a variable resistor T, as shown.

V
T

5.0 V

Resistance of each resistor is 1.0 k and resistance of T is 5.0 k.

What is the reading of the ideal voltmeter?

A 0V B 2.0 V C 3.0 V D 5.0 V

Ans: C

Redrawing the diagram,

5.0 V

V
20

1 1 1
( ) 1500
Total resistance of parallel arrangement 6000 2000
1500
V 5.0 3.0 V
2500

31 In an electric motor, a rectangular coil WXYZ has 20 turns and is in a uniform magnetic field of
flux density 0.83T.

The lengths of sides XY and ZW are 0.17 m and of sides WX and YZ are 0.11 m. The current
in the coil is 4.5 A.

What is the maximum torque provided by the motor?

A 0.070 Nm B 0.63 Nm C 1.4 Nm D 2.8 Nm

Ans: C
Torque = Fd
= (NBIL)d
= (20x0.83x4.5x0.17)(0.11)
= 1.4 Nm

32 A neutral sub-atomic particle at rest in a magnetic field of flux density B. It spontaneously


decays into two particles of mass m each. One of the particles has a negative charge q. The
particles move with velocities perpendicular to the magnetic field. After what time will the
particles collide?

A B C D

By conservation of momentum, both particles move off with the same speed.

Since they collide after a revolution, time for collision is


21

33 A circular coil has resistance R and area A. The coil is placed in a uniform magnetic field of
strength B such that its plane is perpendicular to the magnetic field lines. The plane of the
coil is now rotated through 90. The magnitude of the charge that flows in the coil is

BA R
A Zero B C D BAR
R BA

Ans: B

0 - BA BA
emf induced | | | - |
t t
BA
Current current I
R tR
BA BA
Amount of charge flowing Q It t
tR R

34 A rectifier is connected in series with load P and an alternating voltage supply as shown in
the figure below.

Vin / V
Vin
Vo

t 2t 3t 4t 5t t/s
6t

P -Vo

What is the value of the r.m.s. voltage across load P?

A 0.18 Vo B 0.50 Vo C 0.71 Vo D 1.0 Vo

Ans: B
The formula for finding r.m.s values is
Area under the V 2 vs t curve for 1 period
Period
Observations:
This ac source has a period of 2t.
Vot
0.5Vo
Using the formula, Vr.m.s 2t
22

35 In an ideal transformer, the most important function of the soft-iron core is

A to reduce eddy-currents.
B to improve the flux-linkage between the primary and secondary coils.
C to dissipate the heat generated by the two coils.
D to produce a uniform magnetic field in the two coils

Ans: B

36 Fig. 36 below shows a typical X-ray spectrum produced when electrons are accelerated
through a potential difference and are then stopped in a metal target.

Fig. 36
Intensity

min 1 2 Wavelength

Which of the following is not correct?

Feature(s) on the graph Is dependent on

A The wavelengths (1, 2) of the peaks the target material.


on the graph

B The minimum wavelength min the target material


of the X-rays produced

C The minimum wavelength min the potential difference used to accelerate


of the X-rays produced the bombarding electrons.

D The maximum intensity of the X-rays the potential difference used to accelerate
produced. the bombarding electrons.

Ans: B
23

37 Which statement describes correctly how scanning tunnelling microscopes work in


obtaining an atomic-scale image of a surface?
A The position of the tip of the microscope in the Z-axis must be adjusted accordingly to
keep the tunnelling current a constant during the scan on a surface lying in the X-Y
plane.
B The width of the gap between the tip of the microscope and the surface must be
varied continuously to keep the tunnelling current a constant during the scan.
C The voltage applied between the tip of the microscope and the surface must be varied
continuously in order to keep the tunnelling current a constant during the scan.
D The voltage applied between the tip of the microscope and the surface must be high
enough in order to cause a discharge in the gap and thus allowed a tunnelling current
to flow.

Ans: A

38 Stimulated emission occurs in the laser when a photon P causes the emission of a photon Q.
Which of the following statements is true of the photons?

A They have different frequencies but are in phase and traveling in the same
direction.
B They have the same frequency, are rad out of phase and traveling in the same
direction.
C They have the same frequency, are in phase and traveling in the different
directions.
D They have the same frequency, are in phase and traveling in the same direction.

Ans: D

39 The resistivity of the intrinsic semiconductor silicon decreases from 400 m at 20oC to 40 m
at 60oC.

Which statement, using band theory, best explains the change in resistivity of silicon with an
increase in temperature?

A Conduction band electrons which carry current move faster.


B More electrons can move through the valence band taking part in conduction.
C More valence band electrons can be promoted to the conduction band.
D The energy gap between the valence and conduction bands decreases
Ans: C
24
40 What may not be conserved in nuclear processes?

A charge
B momentum
C the total number of neutrons
D the total number of nucleons

Ans: C

1 D 11 A 21 D 31 C
2 A 12 D 22 D 32 C
3 C 13 B 23 B 33 B
4 C 14 B 24 C 34 B
5 D 15 B 25 D 35 B
6 A 16 A 26 C 36 B
7 B 17 A 27 C 37 A
8 C 18 D 28 A 38 D
9 D 19 A 29 A 39 C
10 C 20 C 30 C 40 C

1 D 11 A 21 D 31 C
2 A 12 D 22 D 32 C
3 C 13 B 23 B 33 B
4 C 14 B 24 C 34 B
5 D 15 B 25 D 35 B
6 A 16 A 26 C 36 B
7 B 17 A 27 C 37 A
8 C 18 D 28 A 38 D
9 D 19 A 29 A 39 C
10 C 20 C 30 C 40 C
NANYANG JUNIOR COLLEGE
Science Department
JC 2 PRELIMINARY EXAMINATION
Higher 2

Candidate
Name

Tutor
Class
Name

PHYSICS 9646/02
Paper 2 Structured Questions 24 September 2013
1 hour 45 minutes

Candidates answer on the Question Paper.


No Additional Materials are required.

READ THESE INSTRUCTIONS FIRST

Write your name, class and tutor name on all the work you hand in.
Write in dark blue or black pen on both sides of the paper.
You may use a soft pencil for any diagrams, graphs or rough working. For Examiners Use
Do not use staples, paper clips, highlighters, glue or correction fluid.
1
Answer all questions.
2
At the end of the examination, fasten all your work securely together.
The number of marks is given in brackets [ ] at the end of each question or 3
part question.
4

Total

This document consists of 20 printed pages

Nanyang Junior College

NYJC 2013 JC2/Prelim/H2/9646/02 [Turn over


2

Data
speed of light in free space, c = 3.00 x 108 m s-1
permeability of free space, o = 4 x 10-7 H m-1
permittivity of free space, o = 8.85 x 10-12 Fm-1
(1 / (36 )) x 10-9 Fm-1
elementary charge, e = 1.60 x 10-19 C
the Planck constant, h = 6.63 x 10-34 J s
unified atomic mass constant, u = 1.66 x 10-27 kg
rest mass of electron, me = 9.11 x 10-31 kg
rest mass of proton, mp = 1.67 x 10-27 kg
molar gas constant, R = 8.31 J K-1 mol-1
the Avogadro constant, NA = 6.02 x 1023 mol-1
the Boltzmann constant, k = 1.38 x 10-23 J K-1
gravitational constant, G = 6.67 x 10-11 N m2 kg-2
acceleration of free fall, g = 9.81 m s-2

Formulae
uniformly accelerated motion, s = ut + at2
v2 = u2 + 2as
work done on/by a gas, W = pV
hydrostatic pressure, p = gh
gravitational potential, = Gm / r
displacement of particle in s.h.m. x = xo sin t
velocity of particle in s.h.m. v = vo cos t

= x o
2
x2
mean kinetic energy of a molecule of an ideal gas 3
E = kT
2
resistors in series, R = R1 + R2 +
resistors in parallel, 1/R = 1/R1 + 1/R2 +
electric potential, V = Q / 4or
alternating current/voltage, x = xo sin t
transmission coefficient, T exp(-2kd)

8 2 m U E
where k =
h2
radioactive decay, x = xo exp (-t)
decay constant 0.693
= t1
2

NYJC 2013 JC2/Prelim/H2/9646/02


3

1 (a) State the relation between force and momentum. For


Examiners
Use
.. [1]

(b) A rigid bar of mass 450 g is held horizontally by two supports A and B, as shown in
Fig. 1.1.

Fig. 1.1

The support A is 45 cm from the centre of gravity C of the bar and the support B is
25 cm from C.

A ball of mass 140 g falls vertically onto the bar such that it hits the bar at a
distance of 50 cm from C, as shown in Fig. 1.1. The variation with time of the
velocity of the ball before, during and after hitting the bar is shown in Fig. 1.2.

Fig. 1.2

NYJC 2013 JC2/Prelim/H2/9646/02 [Turn over


4

For the time that the ball is in contact with the bar, use Fig. 1.2 to determine For
Examiners
(i) the magnitude of the change in momentum of the ball, Use

change in momentum = .. kg m s-1 [2]

(ii) the magnitude of the force exerted by the ball on the bar.

force by ball = .. N [2]

(c) Hence, calculate the magnitude of the force exerted on the bar by support A for the
time that the ball is in contact with the bar.

force by support A = .. N [2]

NYJC 2013 JC2/Prelim/H2/9646/02


5

2 An unpowered artificial satellite of mass m has been placed in a stable orbit around the For
Examiners
Sun in the same direction as that of the Earth. It is at a distance of 0.99R from the Sun, Use
where R is the orbital radius of the Earth as shown in Fig. 2.1.

0.99R
Earth
Sun
satellite
R

Fig. 2.1

(a) Ignore the very small force the satellite acts on the Earth. Show that the period of
the Earth round the Sun TE is given by

4 2 3/2
TE R
GMS

where MS is the mass of the Sun.

[2]

GMS m
(b) Show that the resultant force on the satellite is given by 0.99 , given that the
R2
mass of the Sun is 3.33 x 105 times the mass of Earth.

[2]

NYJC 2013 JC2/Prelim/H2/9646/02 [Turn over


6

(c) Hence determine the period of the satellite round the Sun in terms of the period of For
Examiners
the Earth TE. Use

Period of satellite = [2]

(d) Since the satellite is going round the Sun in a stable orbit, it is in stable equilibrium.
Comment on the statement.

.. [1]

3 (a) State the principle of superposition.

... [1]

(b) Figure 3.1 shows a double slit S1 and S2 emitting waves of amplitude A and of
wavelength 590 nm. They are placed 0.800 mm apart and at a distance of 2.70 m
from a line XY. Point O is in the center of the fringe pattern. Two polarizers P 1 and
P2 are placed in front of S1 and S2 respectively. The polarizers are rotated such that
a fringe pattern is observed along the line XY.

0.800 mm S2
S1

P1 P2

Fig. 3.1
2.70 m

X Y
O

NYJC 2013 JC2/Prelim/H2/9646/02


7

(i) Show that the fringe separation along the line XY is 2.00 mm. For
Examiners
Use

[1]

(ii) On Fig. 3.2, sketch the variation of intensity along the line XY. [2]

intensity

mm
X 4 3 2 1 0 1 2 3 4 Y

Fig. 3.2

(c) The polarizer P1 is rotated 90o along its plane.


(i) Calculate the resultant amplitude at point O on the line XY in terms of A.

amplitude = [1]

NYJC 2013 JC2/Prelim/H2/9646/02 [Turn over


8

(ii) Calculate the resultant amplitude at a point 1.00 mm from point O along the line For
Examiners
XY in terms of A. Use

amplitude = [1]

(iii) Describe the appearance of the fringe pattern.

... [1]

4 An ideal transformer has 5000 turns on its primary coil. It is used to convert a main
supply of root mean square value of 230 V to an alternating voltage having a peak value
of 12.0 V.

(a) (i) Explain what is meant by root mean square value of 230 V.

..

..

... [2]

(ii) Calculate the number of turns on the secondary coil.

number of turns = .. [2]

NYJC 2013 JC2/Prelim/H2/9646/02


9

(b) The secondary coil is connected in series with a resistor R. The variation with time t, For
Examiners
in seconds, of the potential difference at the secondary coil is given by the Use
expression
V = 12.0 sin(380t)
(i) Determine the frequency of the supply.

frequency = .. Hz [1]

(ii) To prevent overheating, the mean power dissipated in R must not exceed
300W. Calculate the minimum resistance of R.

resistance = .. [2]

5 (a) A uniform magnetic field has constant flux density B. A straight wire of fixed length
carries a current I at an angle to the magnetic field as shown in Fig. 5.1.

Fig. 5.1

(i) Define the term magnetic flux density.

... [1]

NYJC 2013 JC2/Prelim/H2/9646/02 [Turn over


10

(ii) The current I in the wire is changed, keeping the angle constant. For
Examiners
Use
On Fig. 5.2, sketch a graph to show the variation with the current I of the force
F on the wire. [1]

Fig. 5.2

(iii) The angle between the wire and the magnetic field is now varied. The current
I is kept constant.

On Fig. 5.3, sketch a graph to show the variation with angle of the force F on
the wire. [1]

Fig. 5.3

NYJC 2013 JC2/Prelim/H2/9646/02


11

(b) Negative ions are travelling through a vacuum in a narrow beam. The ions enter a For
Examiners
region of uniform magnetic field of flux density B and are deflected in a semi-circular Use
arc, as shown in Fig. 5.4.

detector
uniform magnetic
field

beam of
negative ions

Fig. 5.4

The ions, travelling with speed 1.40 105 m s-1, are detected at a fixed detector
when the diameter of the arc in the magnetic field is 12.8 cm.

(i) By reference to Fig. 5.4, state the direction of the magnetic field.

... [1]

(ii) The ions have mass 20u and charge 1.6 10-19 C. Show that the magnetic
flux density is 0.454 T. Explain your working.

[2]

(iii) Ions of a larger mass with the same charge and speed as those in (b)(ii) are
also present in the beam. On Fig. 5.4, sketch the path of these ions in the
magnetic field of magnetic flux density 0.454 T. [1]

NYJC 2013 JC2/Prelim/H2/9646/02 [Turn over


12

6 Fig. 6.1 shows a simple circuit. The resistance of the lamp is 20 and it requires a For
minimum of 60 V to light up. Examiners
Use
240 V

20

30 10

Fig. 6.1
Fig. 6.2 shows how the current I through the light dependent resistor varies with the
potential difference V across it when different intensities of light fall onto it.

I/A 10000 W m-2


10

5000 W m-2
8

6
2500 W m-2

4 100 W m-2

2 20 W m-2

0 V/V
-200 -150 -100 -50 0 50 100 150 200

-2

-4

-6

-8

-10
Fig. 6.2

NYJC 2013 JC2/Prelim/H2/9646/02


13

(a) Calculate the current through the 30 resistor when the potential difference across For
Examiners
the lamp is 40 V. Use

current = A [2]

(b) Explain how the above circuit can be used as a warning system for an environment
which requires low intensity light.

.. [2]

(c) Using Fig. 6.2, determine the intensity of light which will produce a potential
difference of 60 V across the lamp.

intensity = .. W m-2 [3]

NYJC 2013 JC2/Prelim/H2/9646/02 [Turn over


14

7 Wind power can be used for the generation of electric power. Fig 7.1 and Fig 7.2 For
illustrate a particular type of wind turbine. Examiners
Use

gearbox drive
shaft
drive
hub shaft

generator
generator
housing

Fig 7.2
Fig 7.1

Table 7.3 shows some information provided by the manufacturer.

Table 7.3
Height of tower (ground to hub) 80 m
Blade length 45 m
Number of blades 3
Rated power 3 MW
Voltage 650 V
Frequency 50 Hz

NYJC 2013 JC2/Prelim/H2/9646/02


15

Fig 7.4 shows the wind turbine power curve provided by the manufacturer. For
Examiners
Use

Fig 7.4
(a) Using the information provided in Table 7.3, calculate

(i) the height of the lowest point of the rotor above the ground,

height = . m [1]

(ii) the area swept by the rotor blades

area = . m2 [1]

(iii) the period of revolution of the rotor when the wind speed is 10 m s1, given that
the ratio of the speed of the blade tip to the wind speed is 7.

period = . s [2]

NYJC 2013 JC2/Prelim/H2/9646/02 [Turn over


16

(b) (i) Discuss, with reasons, if the rated power of 3 MW is a fair value. For
Examiners
Use

.. [2]

(ii) The average monthly electrical energy consumption per household in


Singapore is 470 kW h. Calculate the number of homes one wind turbine can
serve when operating at the rated power.

number of homes = . [2]

(c) (i) Using the information provided in Fig 7.4, obtain values for
1. the maximum power output,
maximum power = . MW [1]

2. the wind speed for this power.


wind speed = . m s1 [1]

(ii) The incident wind power E, which is the kinetic energy of the air incident on the
rotor to turn the blades per unit time, is given by

E = k L2 v3

where L is the blade length of the turbine,


v is the incident wind speed, and
k is a constant of value 1.96 kg m3

Calculate, for the turbine operating at maximum output power, the incident wind
power.

incident wind power = . W [1]

NYJC 2013 JC2/Prelim/H2/9646/02


17

(iii) According to Betz Law, which is derived from the principles of conservation of For
Examiners
mass and momentum, the maximum amount of the incident wind kinetic energy Use
that can be captured by a wind turbine is 59.3%.

Suggest one evidence that not all of the incident wind energy can be captured.

... [1]

(iv) Calculate the efficiency of the wind turbine in converting the accessible kinetic
energy to electrical energy when operating under the conditions stated in (i).

efficiency = . % [2]

(d) The wind turbine, like most others, has a cut-out speed. This means that at high
wind speeds, the gearbox disengages the generator from the rotor and the
generator is no longer turned by the rotor.

(i) Use Fig 7.4 to determine the cut-out speed.


cut-out speed = . m s1 [1]

(ii) Suggest one reason why it is necessary to have a cut-out speed.

... [1]

(e) Wind turbines are usually erected in wide open spaces. As such, they are
vulnerable to (i) strong winds which may cause the rotor to rotate too fast and be
damaged, and (ii) lightning which may strike the rotor, causing damage.

For each of the hazards mentioned, suggest how the risk of damage to the rotor
may be minimized.
(i) Strong winds

... [1]

(ii) Lightning

... [1]

NYJC 2013 JC2/Prelim/H2/9646/02 [Turn over


18

8. Fig. 8.1 shows a coil (coil X). For


Examiners
Use

Fig. 8.1

A student winds another coil (coil Y) tightly around coil X.

A changing e.m.f. in coil X induces an e.m.f. in coil Y.

The student wishes to investigate how the e.m.f. V in coil Y depends on the frequency f
of the current in coil X.

It is suggested that V is directly proportional to f.

Design a laboratory experiment to investigate the suggested relationship. You should


draw a diagram, in the space provided below, showing the arrangement of your
equipment. In your account you should pay particular attention to:

(a) the procedure to be followed,


(b) the measurements to be taken,
(c) the control of variables,
(d) the analysis of the data,
(e) the safety precautions to be taken.

Diagram

NYJC 2013 JC2/Prelim/H2/9646/02


19

For
Examiners
Use

...

...

...

...

...

...

NYJC 2013 JC2/Prelim/H2/9646/02 [Turn over


20

For
Examiners
Use

...

...

...

...

[12]

NYJC 2013 JC2/Prelim/H2/9646/02


NANYANG JUNIOR COLLEGE
Science Department
JC 2 PRELIMINARY EXAMINATION
Higher 2

Candidate
Name

Tutor
Class
Name

PHYSICS 9646/02
Paper 2 Structured Questions 24 September 2013
1 hour 45 minutes

Candidates answer on the Question Paper.


No Additional Materials are required.

READ THESE INSTRUCTIONS FIRST

Write your name, class and tutor name on all the work you hand in.
Write in dark blue or black pen on both sides of the paper.
You may use a soft pencil for any diagrams, graphs or rough working. For Examiners Use
Do not use staples, paper clips, highlighters, glue or correction fluid.
1
Answer all questions.
2
At the end of the examination, fasten all your work securely together.
The number of marks is given in brackets [ ] at the end of each question or 3
part question.
4

Total

This document consists of 20 printed pages

Nanyang Junior College

NYJC 2013 JC2/Prelim/H2/9646/02 [Turn over


2

Data
speed of light in free space, c = 3.00 x 108 m s-1
permeability of free space, o = 4 x 10-7 H m-1
permittivity of free space, o = 8.85 x 10-12 Fm-1
(1 / (36 )) x 10-9 Fm-1
elementary charge, e = 1.60 x 10-19 C
the Planck constant, h = 6.63 x 10-34 J s
unified atomic mass constant, u = 1.66 x 10-27 kg
rest mass of electron, me = 9.11 x 10-31 kg
rest mass of proton, mp = 1.67 x 10-27 kg
molar gas constant, R = 8.31 J K-1 mol-1
the Avogadro constant, NA = 6.02 x 1023 mol-1
the Boltzmann constant, k = 1.38 x 10-23 J K-1
gravitational constant, G = 6.67 x 10-11 N m2 kg-2
acceleration of free fall, g = 9.81 m s-2

Formulae
uniformly accelerated motion, s = ut + at2
v2 = u2 + 2as
work done on/by a gas, W = pV
hydrostatic pressure, p = gh
gravitational potential, = Gm / r
displacement of particle in s.h.m. x = xo sin t
velocity of particle in s.h.m. v = vo cos t

= x o
2
x2
mean kinetic energy of a molecule of an ideal gas 3
E = kT
2
resistors in series, R = R1 + R2 +
resistors in parallel, 1/R = 1/R1 + 1/R2 +
electric potential, V = Q / 4or
alternating current/voltage, x = xo sin t
transmission coefficient, T exp(-2kd)

8 2 m U E
where k =
h2
radioactive decay, x = xo exp (-t)
decay constant 0.693
= t1
2

NYJC 2013 JC2/Prelim/H2/9646/02


3

1 (a) State the relation between force and momentum. For


Examiners
Use
Force is proportional (or equal) to the rate of change of momentum.
.. [1]

(b) A rigid bar of mass 450 g is held horizontally by two supports A and B, as shown in
Fig. 1.1.

Fig. 1.1

The support A is 45 cm from the centre of gravity C of the bar and the support B is
25 cm from C.

A ball of mass 140 g falls vertically onto the bar such that it hits the bar at a
distance of 50 cm from C, as shown in Fig. 1.1. The variation with time of the
velocity of the ball before, during and after hitting the bar is shown in Fig. 1.2.

Fig. 1.2

NYJC 2013 JC2/Prelim/H2/9646/02 [Turn over


4

For the time that the ball is in contact with the bar, use Fig. 1.2 to determine For
Examiners
(i) the magnitude of the change in momentum of the ball, Use

p = 140 x 10-3 [5.4 ( 4.0)]


= 1.32 kg m s-1

change in momentum = .. kg m s-1 [2]

(ii) the magnitude of the force exerted by the ball on the bar.

( )( )

( )( ) N

force by ball = .. N [2]

(c) Hence, calculate the magnitude of the force exerted on the bar by support A for the
time that the ball is in contact with the bar.
Taking moments about B,
( ) ( )( ) ( )

force by support A = .. N [2]

NYJC 2013 JC2/Prelim/H2/9646/02


5

2 An unpowered artificial satellite of mass m has been placed in a stable orbit around the For
Examiners
Sun in the same direction as that of the Earth. It is at a distance of 0.99R from the Sun, Use
where R is the orbital radius of the Earth as shown in Fig. 2.1.

0.99R
Earth
Sun
satellite
R

Fig. 2.1

(a) Ignore the very small force the satellite acts on the Earth. Show that the period of
the Earth round the Sun TE is given by

4 2 3/2
TE R
GMS

where MS is the mass of the Sun.

GMS ME 2
2
ME R( )2
R TE
GMS 2
2
R( )2
R TE
4 2 3/2
TE R
GMS

[2]

GMS m
(b) Show that the resultant force on the satellite is given by 0.99 , given that the
R2
mass of the Sun is 3.33 x 105 times the mass of Earth.

MS
G( )m
GMS m GME m GMS m 3.33 x105
F 2
2
1.02 10000
(0.99R ) (0.01 R ) R2 R2
GMS m GMS m GMS m
1.02 2
0.03 2
0.99
R R R2

[2]

NYJC 2013 JC2/Prelim/H2/9646/02 [Turn over


6

(c) Hence determine the period of the satellite round the Sun in terms of the period of For
Examiners
the Earth TE.
GMS m 2 Use
0.99 m(0.99R )( )2
R2 T
GMS 2
R( )2
R2 T
4 2 2
T R TE
GMS
Period of satellite = [2]

(d) Si c h i i gi g d h S i i ,i i i q i i i m.
Comment on the statement.

The satellite is not in equilibrium because it has centripetal acceleration. Hence the

resultant force is not zero.
.. [1]

3 (a) State the principle of superposition.

... [1]

(b) Figure 3.1 shows a double slit S1 and S2 emitting waves of amplitude A and of
wavelength 590 nm. They are placed 0.800 mm apart and at a distance of 2.70 m
from a line XY. Point O is in the center of the fringe pattern. Two polarizers P 1 and
P2 are placed in front of S1 and S2 respectively. The polarizers are rotated such that
a fringe pattern is observed along the line XY.

0.800 mm S2
S1

P1 P2

Fig. 3.1
2.70 m

X Y
O

NYJC 2013 JC2/Prelim/H2/9646/02


7

(i) Show that the fringe separation along the line XY is 2.00 mm. For
Examiners
Use

[1]

(ii) On Fig. 3.2, ignoring diffraction effects, sketch the variation of intensity along
the line XY. Label your values clearly on the axes. [2]

intensity

mm
X 4 3 2 1 0 1 2 3 4 Y

Fig. 3.2

(c) The polarizer P1 is rotated 90o along its plane.


(i) Calculate the resultant amplitude at point O on the line XY in terms of A.

amplitude = [1]

NYJC 2013 JC2/Prelim/H2/9646/02 [Turn over


8

(ii) Calculate the resultant amplitude at a point 1.00 mm from point O along the line For
Examiners
XY in terms of A. Use

amplitude = [1]

(iii) Describe the appearance of the fringe pattern.

... [1]

4 An ideal transformer has 5000 turns on its primary coil. It is used to convert a main
supply of root mean square value of 230 V to an alternating voltage having a peak value
of 12.0 V.

(a) (i) Explain what is meant by root mean square value of 230 V.

..

..

... [2]

(ii) Calculate the number of turns on the secondary coil.

number of turns = .. [2]

NYJC 2013 JC2/Prelim/H2/9646/02


9

(b) The secondary coil is connected in series with a resistor R. The variation with time t, For
Examiners
in seconds, of the potential difference at the secondary coil is given by the Use
expression
V = 12.0 sin(380t)
(i) Determine the frequency of the supply.

frequency = .. Hz [1]

(ii) To prevent overheating, the mean power dissipated in R must not exceed
300W. Calculate the minimum resistance of R.

resistance = .. [2]

5 (a) A uniform magnetic field has constant flux density B. A straight wire of fixed length
carries a current I at an angle to the magnetic field as shown in Fig. 5.1.

Fig. 5.1

(i) Define the term magnetic flux density.

It is the force per unit length experienced by a straight conductor carrying unit

current when the conductor is placed at right angles to the magnetic field.
... [1]

NYJC 2013 JC2/Prelim/H2/9646/02 [Turn over


10

(ii) The current I in the wire is changed, keeping the angle constant. For
Examiners
Use
On Fig. 5.2, sketch a graph to show the variation with the current I of the force
F on the wire. [1]

Fig. 5.2

(iii) The angle between the wire and the magnetic field is now varied. The current
I is kept constant.

On Fig. 5.3, sketch a graph to show the variation with angle of the force F on
the wire. [1]

Fig. 5.3

NYJC 2013 JC2/Prelim/H2/9646/02


11

(b) Negative ions are travelling through a vacuum in a narrow beam. The ions enter a For
Examiners
region of uniform magnetic field of flux density B and are deflected in a semi-circular Use
arc, as shown in Fig. 5.4.

detector
uniform magnetic
field

beam of
negative ions

Fig. 5.4

The ions, travelling with speed 1.40 105 m s-1, are detected at a fixed detector
when the diameter of the arc in the magnetic field is 12.8 cm.

(i) By reference to Fig. 5.4, state the direction of the magnetic field.

Out of the plane of the paper.


... [1]

(ii) The ions have mass 20u and charge 1.6 10-19 C. Show that the magnetic
flux density is 0.454 T. Explain your working.
Since the magnetic force on the ions provide for its centripetal force,

( )( )
( )( )

[2]

(iii) Ions of a larger mass with the same charge and speed as those in (b)(ii) are
also present in the beam. On Fig. 5.4, sketch the path of these ions in the
magnetic field of magnetic flux density 0.454 T. [1]

NYJC 2013 JC2/Prelim/H2/9646/02 [Turn over


12

6 Fig. 6.1 shows a simple circuit. The resistance of the lamp is 20 and it requires a For
minimum of 60 V to light up. Examiners
Use
240 V

20

30 10

Fig. 6.1
Fig. 6.2 shows how the current I through the light dependent resistor varies with the
potential difference V across it when different intensities of light fall onto it.

I/A 10000 W m-2


10

5000 W m-2
8

6
2500 W m-2

4 100 W m-2

2 20 W m-2

0 V/V
-200 -150 -100 -50 0 50 100 150 200

-2

-4

-6

-8

-10
Fig. 6.2

NYJC 2013 JC2/Prelim/H2/9646/02


13

(a) Calculate the current through the 30 resistor when the potential difference across For
Examiners
the lamp is 40 V. Use
V
I
R
40

30 10
1.0 A

current = A [2]

(b) Explain how the above circuit can be used as a warning system for an environment
which requires low intensity light.

If the intensity of the light in the environment is high, the resistance of the light

dependent resistor will be low. This will cause the potential difference across the

lamp to be more than 50 V, causing the lamp to light up, indicating that the[2]
..
intensity of the light in the environment is high.

NYJC 2013 JC2/Prelim/H2/9646/02 [Turn over


14

(c) Using Fig. 6.2, determine the intensity of light which will produce a potential For
Examiners
difference of 60 V across the lamp. Use
1 1 1
Total resistance of parallel arrangement ( )1
40 20 20
8

8
Resistance of L.D.R: 240 60
8R
R 24

intensity of 5000 W m-2.

OR

If the p.d across the lamp is 60 V, the p.d. across the L.D.R will be 240 60 = 180
V. If we have the current flowing through the L.D.R when its p.d. is 180 V, we can
look for the point on Fig. 6.2 and determine which graph and which ntensity.

1 1 1
Total resistance of parallel arrangement ( )1
40 20 20
8

60
Current flowing through the L.D.R: I 7.5 A
8

From the graph, the graph which shows a current of 7.5 A at 180 V is the one with
an intensity of 5000 W m-2.

intensity = .. W m-2 [3]

NYJC 2013 JC2/Prelim/H2/9646/02


15

7 Wind power can be used for the generation of electric power. Fig 7.1 and Fig 7.2 For
illustrate a particular type of wind turbine. Examiners
Use

gearbox drive
shaft
drive
hub shaft

generator
generator
housing

Fig 7.2
Fig 7.1

Table 7.3 shows some information provided by the manufacturer.

Table 7.3
Height of tower (ground to hub) 80 m
Blade length 45 m
Number of blades 3
Rated power 3 MW
Voltage 650 V
Frequency 50 Hz

NYJC 2013 JC2/Prelim/H2/9646/02 [Turn over


16

Fig 7.4 shows the wind turbine power curve provided by the manufacturer. For
Examiners
Use

Fig 7.4
(a) Using the information provided in Table 7.3, calculate

(i) the height of the lowest point of the rotor above the ground,
Height = 80 45 = 35 m

height = . m [1]

(ii) the area swept by the rotor blades


Sweep area = 452 = 6.36103 m2

area = . m2 [1]

(iii) the period of revolution of the rotor when the wind speed is 10 m s1, given that
the ratio of the speed of the blade tip to the wind speed is 7.
Blade tip speed = 7 10 = 70 m s1

Period = (2 45) / 70 = 4.0 s

period = . s [2]

NYJC 2013 JC2/Prelim/H2/9646/02


17

(b) (i) Discuss, with reasons, if the rated power of 3 MW is a fair value. For
Examiners
Use
The turbine is able to produce more than the rated power for wind speeds

between 12 m s1 and 25 m s1.

For sites with wind speeds averaging above 12 m s1, the rated power will be

a fair value.
.. [2]

(ii) The average monthly electrical energy consumption per household in


Singapore is 470 kW h. Calculate the number of homes one wind turbine can
serve when operating at the rated power.
Power consumption per household = 470 (30 24)
= 0.65 kW
No of homes = 3000 0.65 = 4600

number of homes = . [2]

(c) (i) Using the information provided in Fig 7.4, obtain values for
1. the maximum power output,
maximum power = . MW [1]

2. the wind speed for this power.


wind speed = . m s1 [1]

(ii) The incident wind power E, which is the kinetic energy of the air incident on the
rotor to turn the blades per unit time, is given by

E = k L2 v3

where L is the blade length of the turbine,


v is the incident wind speed, and
k is a constant of value 1.96 kg m3

Calculate, for the turbine operating at maximum output power, the incident wind
power.
E = 1.96 452 153
= 1.34107 J s-1

incident wind power = . W [1]

NYJC 2013 JC2/Prelim/H2/9646/02 [Turn over


18

(iii) Acc di g z L w, which i d iv d f m h i ci f c v i f For


Examiners
mass and momentum, the maximum amount of the incident wind kinetic energy Use
that can be captured by a wind turbine is 59.3%.

Suggest one evidence that not all of the incident wind energy can be captured.
The wind will still be moving, albeit at a lower speed, after passing through the

rotor. Thus it could not have lost all of its kinetic energy.
... [1]

(iv) Calculate the efficiency of the wind turbine in converting the accessible kinetic
energy to electrical energy when operating under the conditions stated in (i).
Accessible wind power = 0.593 1.34107
= 7.94106 W
Efficiency = 3.0106 / 7.94106
= 38%
efficiency = . % [2]

(d) The wind turbine, like most others, has a cut-out speed. This means that at high
wind speeds, the gearbox disengages the generator from the rotor and the
generator is no longer turned by the rotor.

(i) Use Fig 7.4 to determine the cut-out speed.


cut-out speed = . m s1 [1]

(ii) Suggest one reason why it is necessary to have a cut-out speed.


The generators are designed to take a certain maximum electrical load.

Having a cut-out speed will prevent the circuit from being overloaded and [1]
...
damaged.
(e) Wind turbines are usually erected in wide open spaces. As such, they are
vulnerable to (i) strong winds which may cause the rotor to rotate too fast and be
damaged, and (ii) lightning which may strike the rotor, causing damage.

For each of the hazards mentioned, suggest how the risk of damage to the rotor
may be minimized.
(i) Strong winds
Install brakes to resist rotation.

Turn the blades to face away from the wind.
... [1]

(ii) Lightning

Use poor electrical conductor such as fiberglass for blades.



Install lightning rods taller than the turbine nearby.
... [1]

NYJC 2013 JC2/Prelim/H2/9646/02


19

8. Fig. 8.1 shows a coil (coil X). For


Examiners
Use

Fig. 8.1

A student winds another coil (coil Y) tightly around coil X.

A changing e.m.f. in coil X induces an e.m.f. in coil Y.

The student wishes to investigate how the e.m.f. V in coil Y depends on the frequency f
of the current in coil X.

It is suggested that V is directly proportional to f.

Design a laboratory experiment to investigate the suggested relationship. You should


draw a diagram, in the space provided below, showing the arrangement of your
equipment. In your account you should pay particular attention to:

(a) the procedure to be followed,


(b) the measurements to be taken,
(c) the control of variables,
(d) the analysis of the data,
(e) the safety precautions to be taken.

Diagram

NYJC 2013 JC2/Prelim/H2/9646/02 [Turn over


20

For
Examiners
Use

...

...

...

...

...

...

NYJC 2013 JC2/Prelim/H2/9646/02


21

For
Examiners
Use

...

...

...

...

[12]

NYJC 2013 JC2/Prelim/H2/9646/02 [Turn over


NANYANG JUNIOR COLLEGE
Science Department
JC 2 PRELIMINARY EXAMINATION
Higher 2
Candidate
Name

Tutor
Class
Name

PHYSICS 9646/03
Paper 3 Longer Structured Questions 18 September 2013

2 hours
Candidates answer on the Question Paper.
No Additional Materials are required.

READ THESE INSTRUCTIONS FIRST

Write your name, class and tutor name on all the work you hand in.
Write in dark blue or black pen on both sides of the paper.
You may use a soft pencil for any diagrams, graphs or rough
working. For examiners use
Do not use staples, paper clips, highlighters, glue or correction
fluid. Section A

Section A 1
Answer all questions.
2
Section B 3
Answer any two questions.
4
At the end of the examination, fasten all your work securely
together. 5
The number of marks is given in brackets [ ] at the end of
each question or part question. Section B

Total

This document consists of 22 printed pages

Nanyang Junior College

NYJC 2013 JC2/Prelim/H2/9646/03 [Turn over


2

Data
speed of light in free space, c = 3.00 108 m s1
permeability of free space, 0 = 4 107 H m1
permittivity of free space, 0 = 8.85 1012 Fm1
(1 / (36)) 109 Fm1
elementary charge, e = 1.60 1019 C
the Planck constant, h = 6.63 1034 J s
unified atomic mass constant, u = 1.66 1027 kg
rest mass of electron, me = 9.11 1031 kg
rest mass of proton, mp = 1.67 1027 kg
molar gas constant, R = 8.31 J K1 mol1
the Avogadro constant, NA = 6.02 1023 mol1
the Boltzmann constant, k = 1.38 1023 J K1
gravitational constant, G = 6.67 1011 N m2 kg2
acceleration of free fall, g = 9.81 m s2

Formulae
uniformly accelerated motion, s = ut + at2
v2 = u2 + 2as
work done on/by a gas, W = pV
hydrostatic pressure, p = gh
gravitational potential, = Gm / r
displacement of particle in s.h.m. x = x0 sin t
velocity of particle in s.h.m. v = v0 cos t

x
o
2
x2

3
mean kinetic energy of a molecule of an ideal gas E = kT
2
resistors in series, R = R1 + R2 +
resistors in parallel, 1/R = 1/R1 + 1/R2 +
electric potential, V = Q / 40r
alternating current/voltage, x = x0 sin t
transmission coefficient, T exp(2kd)

0.693 8 2 m U E
where k =
t1 h2
2

radioactive decay, x = x0 exp (t)


0.693
decay constant =
t1
2

NYJC JC2/Prelim/H2/9646/03
3
Section A
For
Answer all the questions in this section. Examiners
Use

1 Three similarly sized balls A, B, and C, of masses 0.40 kg, 0.20 kg, and 0.10 kg respectively,
are connected by strings such that their centre-to-centre distances are as shown in Fig. 1.1
below. The setup is swung in a horizontal circle on a frictionless table about O. The balls and
strings maintain a straight line, with the outermost ball having a speed of 6.0 m s1.

1.5 m 1.0 m 0.5 m



O
A B C
Fig. 1.1: Top View
Calculate
(a) the angular velocity of C.

= .............................. rad s1 [1]

(b) the tangential speed of ball B.

tangential speed = .............................. m s1 [2]

(c) the tensions in


(i) string BC and

tension in BC = .............................. N [2]

NYJC 2013 JC2/Prelim/H2/9646/03 [Turn over


4
(ii) string AB. For
Examiners
Use

tension in AB = .............................. N [3]

2 (a) State the First Law of Thermodynamics.

. [1]

(b) The variation with volume of pressure in the internal combustion engine of a car at
maximum power output is shown in Fig. 2.1. The engine goes through 4 distinct stages
A to D as shown.

Fig. 2.1

Complete the rest of the table below to show how the First Law of Thermodynamics
applies to the gas in the engine between each of the stages. [3]

Process U / J Q/J W/J

AB 0

BC 720

CD 760 0

DA 1600 1600

NYJC 2013 JC2/Prelim/H2/9646/03


5

(c) (i) Calculate the net work done by the gas per cycle when the engine goes through For
stages A to D. Examiners
Use

work done = .............................. J [1]

(ii) The car, travelling at a velocity of 30 m s1 on a level road, experiences a total


resistive force of 1.0 kN. The engine operates at a rate of 50 cycles per second.
1. Calculate the rate of net work done by the engine.

rate of work done = .............................. W [1]


2. Calculate the coefficient P, given by

coefficient P = .............................. [2]

3 (a) (i) Explain what is meant by a longitudinal wave.

... [1]

NYJC 2013 JC2/Prelim/H2/9646/03 [Turn over


6
(ii) With the aid of a diagram, explain the formation of compression and rarefraction For
points along a longitudinal wave. Examiners
Use

. [2]

(b) A constant-frequency siren vibrates with displacement y, where y = A sin 200t.


This sound causes vibrations of the diaphragm of an ear drum in an observer 500 m
away. The speed of sound is 335 m s1.
(i) Calculate the frequency of the sound.

f = .............................. Hz [1]

(ii) Show that the phase difference between the motion of the siren and the eardrum
is /2. [2]

NYJC 2013 JC2/Prelim/H2/9646/03


7
(c) (i) The siren can be considered as a point source. Fig. 3.1 shows the variation with
time of displacement y of the siren. For
Examiners
Sketch a graph on Fig. 3.1 to illustrate the variation with time of displacement y of Use
the eardrum at a distance of 500 m. [2]

Displacement y

Time

Fig. 3.1

4 A conducting block made of material X has dimensions 1.0 cm 1.0 cm 4.5 cm, as shown
below.

4.5 cm

1.0 cm

1.0 cm

When a potential difference of 9.0 V is applied across the square faces of the block, the
electrical resistance of the block is 6.0 .
(a) Define the term electrical resistance and the unit .

. [2]

(b) Calculate the electrical resistivity of material X.

resistivity = .............................. m [2]

NYJC 2013 JC2/Prelim/H2/9646/03 [Turn over


8
(c) Show that the strength of the electric field between the square faces of the block is For
200 N C1, assuming that it is uniform. [1] Examiners
Use

(d) Hence, or otherwise, calculate the work done by the electric field on an electron
passing through the block. State clearly whether the work done is positive or negative.

work done = .............................. J [2]

(e) At steady state, the electrons move through the block with negligible change in their
kinetic energies, despite the work done on them as calculated in (d). Suggest why this
is so.

. [1]

5 Fig. 5.1 shows a beam of electrons, moving with speed 6.0 105 m s1 in the x-direction,
passing through a single slit of width 1 nm.

px x

Fig. 5.1

NYJC 2013 JC2/Prelim/H2/9646/03


9
(a) Calculate the momentum of one of these electrons in the x-direction.
For
Examiners
Use

momentum = .............................. kg m s1 [1]

(b) Determine the de Broglie wavelength of the electron.

wavelength = ........................... m [1]

(c) In terms of classical wave theory, explain why the electron diffraction will be prominent
in this situation.

. [1]

(d) In Fig. 5.1, an electron can go through anywhere within the slit, hence the uncertainty
y of the y-position can be as big as the width of the slit,
Calculate the uncertainty of the momentum in the y-direction (py) of one electron that
is passing through the slit.

uncertainty = .............................. N s [2]

(e) On Fig. 5.1, add in two more vectors to show the relation between px, py and the
resultant momentum p of the electron. [1]

NYJC 2013 JC2/Prelim/H2/9646/03 [Turn over


10
(f) Fig. 5.2 shows the diffraction pattern on a screen formed by a beam of laser after For
passing through a small slit. The width of the centre maximum is found to be increasing Examiners
when the slit width is being reduced. Such a phenomenon in light optics is called single Use

slit diffraction.

Fig. 5.2

Discuss the consistency between the electron diffraction and light diffraction from your
working in (d), (e), and the description in (f).

. [2]

NYJC 2013 JC2/Prelim/H2/9646/03


11
Section B
For
Answer two questions in this section. Examiners
Use

6 (a) A ball is held between two fixed points A and B by means of two stretched springs as
shown in Fig. 6.1.

ball

A B

Fig. 6.1

The ball is free to oscillate horizontally along the line AB on the smooth plane.
The variation of the acceleration a of the ball with its displacement x from its equilibrium
position is shown in Fig. 6.2.

a / m s2
15

10

0 x / cm
3 2 1 1 2 3

10

15
Fig. 6.2

NYJC 2013 JC2/Prelim/H2/9646/03 [Turn over


12
(i) State and explain the features of Fig. 6.2 which indicate that the motion of the ball For
is simple harmonic. Examiners
Use

... [3]

(ii) On Fig. 6.3, sketch the velocity-displacement graph of the simple harmonic
motion which was illustrated in Fig. 6.2. [3]

Fig. 6.3

(iii) State an assumption made about the spring for the motion of ball to be in simple
harmonic motion, and explain your answer.

... [3]

NYJC 2013 JC2/Prelim/H2/9646/03


13
(b) A student removes the ball and attaches it to the apparatus illustrated in Fig. 6.4 in
order to investigate the oscillation vertically. For
Examiners
Use
pulley variable frequency
oscillator

thread
spring

ball

Fig. 6.4

The amplitude of the vibrations produced by the oscillator is constant. The variation
with frequency of the amplitude of the oscillations of the ball is shown in Fig. 6.5. The
mass of the ball is given to be 150 g, and its oscillations may be assumed to be simple
harmonic.

amplitude / mm

20

15

10

0
2 4 6 8 10 f / Hz

Fig. 6.5

NYJC 2013 JC2/Prelim/H2/9646/03 [Turn over


14
(i) State the phenomenon illustrated in Fig. 6.5. For
Examiners
.. Use

... [1]

(ii) For the maximum amplitude of vibration, state the magnitudes of the amplitude
and the frequency.

amplitude = .............................. mm

frequency = .............................. Hz [2]

Determine
(iii) the maximum acceleration of the ball.

acceleration = .............................. m s2 [2]

(iv) the maximum tension in the spring.

tension = .............................. N [3]

(v) the maximum kinetic energy of the ball.

maximum kinetic energy = .............................. J [1]

Some very light feathers are attached to the surface of the ball so that the feathers
extend outwards. The investigation is now repeated.
(vi) On Fig. 6.5, draw a line to show the new variation with frequency of the amplitude
of variation for frequencies between 2 Hz and 10 Hz. [2]

NYJC 2013 JC2/Prelim/H2/9646/03


15
7 (a) Mobile phones are required to be charged after they have been used for some time.
Very often, this involves connecting the mobile phone to a charging device via a wire. For
This meth h . Examiners
Use
Recently, a new way of charging called inductive h ( h )
being used. The mobile phone is placed on top of the charging device during charging.
A simplified setup is shown in Fig. 7.1.

Battery of the
mobile phone
Unknown
component

Part of the
mobile phone
secondary coil

primary coil Part of the


~ charging device

Fig. 7.1

(i) Explain how an e.m.f. can be induced in the secondary coil.

.. [4]

(ii) The charging device is connected to a 240 V power source. The number of turns
in the primary coil is 100. If the number of turns in the secondary coil is 5,
calculate the e.m.f. induced in the secondary coil.
Assume that the transformer is ideal.

e.m.f. induced in secondary coil = .............................. V [1]

NYJC 2013 JC2/Prelim/H2/9646/03 [Turn over


16
(iii) In order to charge the mobile phone, several conditions need to be met. Some of For
the conditions are: Examiners
Use
1. The e.m.f. induced in the secondary coil must be larger than 9 V.
2. Mobile phones can only be charged using direct current.
3. The current through the battery of the mobile phone must be in the direction
shown in Fig. 7.2.

Fig. 7.2

Draw an appropriate component in the dotted box in Fig. 7.1 which satisfies the
above conditions. [2]
(iv) Describe and explain one possible disadvantage of inductive charging over wired
charging.

.. [2]

(b) A bar magnet is removed from the centre of a coil in the direction shown in Fig. 7.3.

N S

Fig. 7.3
Draw an arrow on the coil in Fig. 7.3 to show the direction of the current. [1]

NYJC 2013 JC2/Prelim/H2/9646/03


17
(c) In Fig. 7.4, a circular loop of flexible wire is placed perpendicularly to a magnetic field
between the poles of an electromagnet. A resistor is connected to the ends of the coil. For
Examiners
Use
S

Loop of
flexible
wire

N
resistor

Fig. 7.4

Fig. 7.5 shows how the magnetic flux density B varies with time t.

B / 10-2 T
3

0
0 0.01 0.02 0.03 0.04 0.05 t /s

-1

-2

-3
Fig. 7.5

(i) Using Fig. 7.5, calculate the maximum e.m.f. induced in the loop when its area is
120 cm2.

maximum e.m.f. induced = .............................. V [3]

NYJC 2013 JC2/Prelim/H2/9646/03 [Turn over


18
(ii) The loop is observed to repeatedly expand and contract. Give an explanation of For
this phenomenon. Examiners
Use

... [4]

(iii) The total resistance of the circuit is 5.0 Calculate the maximum rate of flow of
electrons in the coil.

maximum rate of flow of electrons = .............................. s1 [3]

8 (a) (i) Explain what is meant by the binding energy of a nucleus.

.................................................................................................................................

............................................................................................................................. [1]

The variation of the binding energy per nucleon with nucleon number of all the different
nuclei is shown in Fig. 8.1.

binding energy
per nucleon fission process

0
0 M nucleon number
Fig. 8.1

NYJC 2013 JC2/Prelim/H2/9646/03


19
(ii) State the value of M and explain your answer clearly.
For
Examiners
Use
value of M = ..............................

Explanation:

... [2]

(b) In a fission process, a neutron with speed of 2000 m s1 collides with a Uranium-235
nucleus and causes a nuclear reaction summarised in the following equation.
1
0 n + 235
92 U 236
92 U 143
54 Xe + 90
38 Sr + 3 01n + energy

(i) Using the data from Table 8.2, show that the binding energy per nucleon for
Strontium-90 is 8.73 MeV.
Rest mass / u
90
Strontium, 38 Sr 89.9077 u
Proton, p 1
1 1.0078 u
1
Neutron, n 0 1.0087 u

Table 8.2

[2]

(ii) Hence calculate the total energy released during the nuclear fission reaction.
Data for binding energies per nucleon are shown in Table 8.3.

Isotope Binding energy per nucleon/MeV


Uranium-235 7.59
Xenon-143 8.41
Table 8.3

energy released = .............................. J [2]

NYJC 2013 JC2/Prelim/H2/9646/03 [Turn over


20
(iii) Explain quantitatively why the kinetic energy of the neutron directed at U-235 is For
neglected. Examiners
Use

... [1]

(iv) Give a reason why the total kinetic energy of the fission products and neutrons is
less than the value calculated in (b)(ii).

..

... [1]

(c) Fig. 8.4 shows the possible directions of Sr-90, Xe-143, and neutrons when nuclear
fission takes place. Assume that this is an isolated system.

Xe-143

U-235
slow-moving
neutron
Sr-90

Fig. 8.4

(i) Explain why it is unlikely for the two fission products to move in the same
direction after the fission process.

... [1]

NYJC 2013 JC2/Prelim/H2/9646/03


21
(ii) Explain why the total momentum of two fission products and neutrons after
reaction is not zero even though the total momentum of U-235 and slow moving For
neutron may be taken to be zero. Examiners
Use

... [2]

(d) Neutrons emitted from a nuclear fission may hit another U-235 nucleus, causing a
chain reaction.
(i) To control the reaction, neutrons emitted from a fission reaction may be slowed
down by Carbon-12 nuclei, 126 C . Show that when a neutron collides head-on with
a Carbon-12 nucleus as shown, the speed is reduced by about 15%.

at rest

C-12
neutron

[3]

(ii) On average, the neutron speed after each collision is 0.93 of its speed before the
collision.
Suggest why this speed reduction is different from what is stated in (ii) for a
nuclear reactor.

... [1]

NYJC 2013 JC2/Prelim/H2/9646/03 [Turn over


22
(iii) Suggest why a slow neutron has a higher chance of being captured by U-235 to For
cause a fission reaction compared to a fast neutron. Examiners
Use

... [1]

(e) The fission products are usually radioactive and give rise to a series of radioactive
decay products. Each decay product has its own half life, but eventually a stable
nuclide is reached.
One such fission product with its decay products and half life is shown below.


0.511 s 1.79 s

(i) Suggest how the number of Cs-143 nuclei inside the nuclear reactor may remain
constant even when it decays to form Ba-143.

... [1]

(ii) Explain why the Xenon-143 produced in the later part of the chain reaction may
not necessarily decay at a later time than those produced in the earlier part of the
chain reaction.

... [2]

NYJC 2013 JC2/Prelim/H2/9646/03


NANYANG JUNIOR COLLEGE
Science Department
JC 2 PRELIMINARY EXAMINATION
Higher 2
Candidate
Name

Tutor
Class
Name

PHYSICS 9646/03
Paper 3 Longer Structured Questions 18 September 2013

2 hours
Candidates answer on the Question Paper.
No Additional Materials are required.

READ THESE INSTRUCTIONS FIRST

Write your name, class and tutor name on all the work you hand in.
Write in dark blue or black pen on both sides of the paper.
You may use a soft pencil for any diagrams, graphs or rough
working. For examiners use
Do not use staples, paper clips, highlighters, glue or correction
fluid. Section A

Section A 1
Answer all questions.
2
Section B 3
Answer any two questions.
4
At the end of the examination, fasten all your work securely
together. 5
The number of marks is given in brackets [ ] at the end of
each question or part question. Section B

Total

This document consists of 22 printed pages

Nanyang Junior College

NYJC 2013 JC2/Prelim/H2/9646/03 [Turn over


2

Data
speed of light in free space, c = 3.00 108 m s1
permeability of free space, 0 = 4 107 H m1
permittivity of free space, 0 = 8.85 1012 Fm1
(1 / (36)) 109 Fm1
elementary charge, e = 1.60 1019 C
the Planck constant, h = 6.63 1034 J s
unified atomic mass constant, u = 1.66 1027 kg
rest mass of electron, me = 9.11 1031 kg
rest mass of proton, mp = 1.67 1027 kg
molar gas constant, R = 8.31 J K1 mol1
the Avogadro constant, NA = 6.02 1023 mol1
the Boltzmann constant, k = 1.38 1023 J K1
gravitational constant, G = 6.67 1011 N m2 kg2
acceleration of free fall, g = 9.81 m s2

Formulae
uniformly accelerated motion, s = ut + at2
v2 = u2 + 2as
work done on/by a gas, W = pV
hydrostatic pressure, p = gh
gravitational potential, = Gm / r
displacement of particle in s.h.m. x = x0 sin t
velocity of particle in s.h.m. v = v0 cos t

x
o
2
x2

3
mean kinetic energy of a molecule of an ideal gas E = kT
2
resistors in series, R = R1 + R2 +
resistors in parallel, 1/R = 1/R1 + 1/R2 +
electric potential, V = Q / 40r
alternating current/voltage, x = x0 sin t
transmission coefficient, T exp(2kd)

0.693 8 2 m U E
where k =
t1 h2
2

radioactive decay, x = x0 exp (t)


0.693
decay constant =
t1
2

NYJC JC2/Prelim/H2/9646/03
3
Section A
For
Answer all the questions in this section. Examiners
Use

1 Three similarly sized balls A, B, and C, of masses 0.40 kg, 0.20 kg, and 0.10 kg respectively,
are connected by strings such that their centre-to-centre distances are as shown in Fig. 1.1
below. The setup is swung in a horizontal circle on a frictionless table about O. The balls and
strings maintain a straight line, with the outermost ball having a speed of 6.0 m s1.

1.5 m 1.0 m 0.5 m



O
A B C
Fig. 1.1: Top View
Calculate
(a) the angular velocity of C.

rad s-1

= .............................. rad s1 [1]

(b) the tangential speed of ball B.

Since the balls and strings maintain a straight line, the angular velocity of
ball B must be the same as that of ball C.

( ) m s-1

tangential speed = .............................. m s1 [2]

(c) the tensions in


(i) string BC and

Considering the free body of ball C,


TBC provides for the centripetal force for circular motion of C

( )( ) ( )
N

tension in BC = .............................. N [2]

NYJC 2013 JC2/Prelim/H2/9646/03 [Turn over


4
(ii) string AB. For
Examiners
Use
Considering the free body of ball B,
TAB - TBC provides for the centripetal force for circular motion of B

( )( ) ( )
N [A1]

tension in AB = .............................. N [3]

2 (a) State the First Law of Thermodynamics.


[The First Law of Thermodynamics states that] the increase in the internal energy of a system is

the sum of the work done on the system and the heat supplied to the system.
. [1]

(b) The variation with volume of pressure in the internal combustion engine of a car at
maximum power output is shown in Fig. 2.1. The engine goes through 4 distinct stages
A to D as shown.

Fig. 2.1

Complete the rest of the table below to show how the First Law of Thermodynamics
applies to the gas in the engine between each of the stages. [3]

Process U / J Q/J W/J

AB 1640 0 1640

BC 720 720 0

CD 760 0 760

DA 1600 1600 0

NYJC 2013 JC2/Prelim/H2/9646/03


5

(c) (i) Calculate the net work done by the gas per cycle when the engine goes through For
stages A to D. Examiners
Use

Work done on gas = (1640 + 760) J = 880 J


Work done by gas = 880 J

work done = .............................. J [1]

(ii) The car, travelling at a velocity of 30 m s1 on a level road, experiences a total


resistive force of 1.0 kN. The engine operates at a rate of 50 cycles per second.
1. Calculate the rate of net work done by the engine.

Work done by gas per second


= 88050 = 44 kJ
Therefore rate of net work done = 44 kW = 4.4 104 W

rate of work done = .............................. W [1]


2. Calculate the coefficient P, given by

Engine power = 44 kW
at 30 m s1, P = Fv = 100030 = 30 kW
P = 30/44 = 0.68

coefficient P = .............................. [2]

3 (a) (i) Explain what is meant by a longitudinal wave.


The direction of propagation of the wave is parallel to the direction
of vibration/displacement of the particles.

... [1]

NYJC 2013 JC2/Prelim/H2/9646/03 [Turn over


6
(ii) With the aid of a diagram, explain the formation of compression and rarefraction For
points along a longitudinal wave. Examiners
Use

Points of compression occur when the particles on both sides of a



particle are displaced towards it.
Points of rarefraction occur when the particles on both sides of a
. [2]
particle are displaced away from it.

(b) A constant-frequency siren vibrates with displacement y, where y = A sin 200t.


This sound causes vibrations of the diaphragm of an ear drum in an observer 500 m
away. The speed of sound is 335 m s1.
(i) Calculate the frequency of the sound.

= 2f = 200
f = 100 Hz

f = .............................. Hz [1]

(ii) Show that the phase difference between the motion of the siren and the eardrum
is /2. [2]

= v/f = 335/100 = 3.35 m


Hence, calculate the phase difference, = (x/) 2 = 149.25 x 2 =
298.5 = /2

NYJC 2013 JC2/Prelim/H2/9646/03


7
(c) (i) The siren can be considered as a point source. Fig. 3.1 shows the variation with
time of displacement y of the siren. For
Examiners
Sketch a graph on Fig. 3.1 to illustrate the variation with time of displacement y of Use
the eardrum at a distance of 500 m. [2]

Displacement y Smaller amplitude, same period


Negative cosine graph

Time

Fig. 3.1

4 A conducting block made of material X has dimensions 1.0 cm 1.0 cm 4.5 cm, as shown
below.

4.5 cm

1.0 cm

1.0 cm

When a potential difference of 9.0 V is applied across the square faces of the block, the
electrical resistance of the block is 6.0 .
(a) Define the term electrical resistance and the unit .

Electrical resistance is the ratio of the potential difference across a conductor to



the current in it.
1 is the resistance of a conductor whose ratio of potential difference to current is

1 V to 1 A.
. [2]

(b) Calculate the electrical resistivity of material X.

= R A / L = 6.0 0.0102 / 0.045


= 1.3102 m [1]

resistivity = .............................. m [2]

NYJC 2013 JC2/Prelim/H2/9646/03 [Turn over


8
(c) Show that the strength of the electric field between the square faces of the block is For
200 N C1, assuming that it is uniform. [1] Examiners
Use

E = V / d = 9.0 / 0.045
= 200 N C1

(d) Hence, or otherwise, calculate the work done by the electric field on an electron
passing through the block. State clearly whether the work done is positive or negative.

F = E q = 200 1.61019 = 3.21017 N


W = F s = 3.21017 0.045 = + 1.441018 J
or
W = change in electric PE = q V = (1.61019 +9.0) = + 1.441018 J

work done = .............................. J [2]

(e) At steady state, the electrons move through the block with negligible change in their
kinetic energies, despite the work done on them as calculated in (d). Suggest why this
is so.
The work done by the field was converted into internal energy of the conductor (heat)

due to collisions with particles of the conductor.
. [1]

5 Fig. 5.1 shows a beam of electrons, moving with speed 6.0 105 m s1 in the x-direction,
passing through a single slit of width 1 nm.

p
py

px x

Fig. 5.1

NYJC 2013 JC2/Prelim/H2/9646/03


9
(a) Calculate the momentum of one of these electrons in the x-direction.
For
Examiners
p = mv = 9.11 10-31 6.0 105 = 5.5 10-25 kg m s-1 Use

momentum = .............................. kg m s1 [1]

(b) Determine the de Broglie wavelength of the electron.

h 6.63 x1034
25
1.2x109 m
p 5.5 x 10

wavelength = ........................... m [1]

(c) In terms of classical wave theory, explain why the electron diffraction will be prominent
in this situation.

The slit width and the wavelength of the electron are in



the order (10-9 m)
. [1]

(d) In Fig. 5.1, an electron can go through anywhere within the slit, hence the uncertainty
y of the y-position can be as big as the width of the slit,
Calculate the uncertainty of the momentum in the y-direction (py) of one electron that
is passing through the slit.

h
py y
4
h
py (1x10 9 )
4
h
py
4 (1x10 9 )
py 5 x10 26 N s

uncertainty = .............................. N s [2]

(e) On Fig. 5.1, add in two more vectors to show the relation between px, py and the
resultant momentum p of the electron. [1]

NYJC 2013 JC2/Prelim/H2/9646/03 [Turn over


10
(f) Fig. 5.2 shows the diffraction pattern on a screen formed by a beam of laser after For
passing through a small slit. The width of the centre maximum is found to be increasing Examiners
when the slit width is being reduced. Such a phenomenon in light optics is called single Use

slit diffraction.

Fig. 5.2

Discuss the consistency between the electron diffraction and light diffraction from your
working in (d), (e), and the description in (f).

In (f), the narrower the slit, the greater the diffraction of light. In (d) and (e), the

smaller the slit, the smaller is y, and by uncertainty principle, the larger is py.
Hence the spread of the electrons will be larger.

. [2]

NYJC 2013 JC2/Prelim/H2/9646/03


11
Section B
For
Answer two questions in this section. Examiners
Use

6 (a) A ball is held between two fixed points A and B by means of two stretched springs as
shown in Fig. 6.1.

ball

A B

Fig. 6.1

The ball is free to oscillate horizontally along the line AB on the smooth plane.
The variation of the acceleration a of the ball with its displacement x from its equilibrium
position is shown in Fig. 6.2.

a / m s2
15

1
0

0 x / cm
3 2 1 1 2 3

10

15
Fig. 6.2

NYJC 2013 JC2/Prelim/H2/9646/03 [Turn over


12
(i) State and explain the features of Fig. 6.2 which indicate that the motion of the ball For
is simple harmonic. Examiners
Use

... [3]

(ii) On Fig. 6.3, sketch the velocity-displacement graph of the simple harmonic
motion which was illustrated in Fig. 6.2. [3]

Fig. 6.3

(iii) State an assumption made about the spring for the motion of ball to be in simple
harmonic motion, and explain your answer.

... [3]

NYJC 2013 JC2/Prelim/H2/9646/03


13
(b) A student removes the ball and attaches it to the apparatus illustrated in Fig. 6.4 in
order to investigate the oscillation vertically. For
Examiners
Use
pulley variable frequency
oscillator

thread
spring

ball

Fig. 6.4

The amplitude of the vibrations produced by the oscillator is constant. The variation
with frequency of the amplitude of the oscillations of the ball is shown in Fig. 6.5. The
mass of the ball is given to be 150 g, and its oscillations may be assumed to be simple
harmonic.

amplitude / mm

20

15

10

0
2 4 6 8 10 f / Hz

Fig. 6.5

NYJC 2013 JC2/Prelim/H2/9646/03 [Turn over


14
(i) State the phenomenon illustrated in Fig. 6.5. For
Examiners
.. Use

... [1]

(ii) For the maximum amplitude of vibration, state the magnitudes of the amplitude
and the frequency.

amplitude = .............................. mm

frequency = .............................. Hz [2]

Determine
(iii) the maximum acceleration of the ball.

acceleration = .............................. m s2 [2]

(iv) the maximum tension in the spring.

tension = .............................. N [3]

(v) the maximum kinetic energy of the ball.

maximum kinetic energy = .............................. J [1]

Some very light feathers are attached to the surface of the ball so that the feathers
extend outwards. The investigation is now repeated.
(vi) On Fig. 6.5, draw a line to show the new variation with frequency of the amplitude
of variation for frequencies between 2 Hz and 10 Hz. [2]

NYJC 2013 JC2/Prelim/H2/9646/03


15
7 (a) Mobile phones are required to be charged after they have been used for some time.
Very often, this involves connecting the mobile phone to a charging device via a wire. For
This meth h . Examiners
Use
Recently, a new way of charging called inductive h ( h )
being used. The mobile phone is placed on top of the charging device during charging.
A simplified setup is shown in Fig. 7.1.

Battery of the
mobile phone
Unknown
component

Part of the
mobile phone
secondary coil

primary coil Part of the


~ charging device

Fig. 7.1

(i) Explain how an e.m.f. can be induced in the secondary coil.

The alternating current flowing in the primary coil produces a changing


.
magnetic field. This changing magnetic field is linked to the secondary coil
and there is a changing magnetic flux linkage in the secondary coil. By
.
F L , . m. . h .
.

.. [4]

(ii) The charging device is connected to a 240 V power source. The number of turns
in the primary coil is 100. If the number of turns in the secondary coil is 5,
calculate the e.m.f. induced in the secondary coil.
Assume that the transformer is ideal.
Vs N
s
Vp N p
5
Vs 240
100
12 V
e.m.f. induced in secondary coil = .............................. V [1]

NYJC 2013 JC2/Prelim/H2/9646/03 [Turn over


16
(iii) In order to charge the mobile phone, several conditions need to be met. Some of For
the conditions are: Examiners
Use
1. The e.m.f. induced in the secondary coil must be larger than 9 V.
2. Mobile phones can only be charged using direct current.
3. The current through the battery of the mobile phone must be in the direction
shown in Fig. 7.2.

Fig. 7.2

Draw an appropriate component in the dotted box in Fig. 7.1 which satisfies the
above conditions. [2]
(iv) Describe and explain one possible disadvantage of inductive charging over wired
charging.
1. Lower efficiency for inductive charging due to flux in the primary coil not 100%

linked to the secondary coil while there is no similar type of problem for wired
charging.

2. The maximum distance the mobile phone can be placed away from the charging

device for inductive charging is fixed and cannot be changed while the maximum
distance for wired charging depends on the length of the cable which can be

changed.
3. Any other reasonable answer.

.. [2]

(b) A bar magnet is removed from the centre of a coil in the direction shown in Fig. 7.3.

N S

Fig. 7.3
Draw an arrow on the coil in Fig. 7.3 to show the direction of the current. [1]

NYJC 2013 JC2/Prelim/H2/9646/03


17
(c) In Fig. 7.4, a circular loop of flexible wire is placed perpendicularly to a magnetic field
between the poles of an electromagnet. A resistor is connected to the ends of the coil. For
Examiners
Use
S

Fig. 7.4

Fig. 7.5 shows how the magnetic flux density B varies with time t.

B / 10-2 T
3

0
0 0.01 0.02 0.03 0.04 0.05 t /s

-1

-2

-3
Fig. 7.5

(i) Using Fig. 7.5, calculate the maximum e.m.f. induced in the loop when its area is
120 cm2.
Considering the magnitude of the induced e.m.f.,
d ( BA)
N
dt
dB
NA
dt
From Fig. 7.4,
dB 2.4 ( 2.4)

dt 0.0490 0.0310
2.4 102 ( 2.4 102 )
1 120 10 4

0.0490 0.0310
2
3.2 10 V maximum e.m.f. induced = .............................. V [3]

NYJC 2013 JC2/Prelim/H2/9646/03 [Turn over


18
(ii) The loop is observed to repeatedly expand and contract. Give an explanation of For
this phenomenon. Examiners
Use
Since there is a closed circuit, a current flows through the loop. This will

induce a magnetic force that acts along the radius of the loop since the loop
is in a magnetic field. The induced e.m.f. in the loop will change direction due

to the alternating nature of the magnetic field. This will cause the direction of
the current to change, leading to a magnetic force which changes its radial

direction. This causes the expansion and contraction of the loop.

... [4]

(iii) The total resistance of the circuit is 5.0 Calculate the maximum rate of flow of
electrons in the coil.
I
R
ne

t R
n

t eR
3.2 10 2

1.60 10 19 5.0
4.01016 s1
maximum rate of flow of electrons = .............................. s1 [3]

8 (a) (i) Explain what is meant by the binding energy of a nucleus.

.................................................................................................................................

............................................................................................................................. [1]

The variation of the binding energy per nucleon with nucleon number of all the different
nuclei is shown in Fig. 8.1.

binding energy
per nucleon fission process

0
0 M nucleon number
Fig. 8.1

NYJC 2013 JC2/Prelim/H2/9646/03


19
(ii) State the value of M and explain your answer clearly.
For
Examiners
Use
value of M = ..............................

Explanation:

... [2]

(b) In a fission process, a neutron with speed of 2000 m s1 collides with a Uranium-235
nucleus and causes a nuclear reaction summarised in the following equation.
1
0 n + 235
92 U 236
92 U 143
54 Xe + 90
38 Sr + 3 01n + energy

(i) Using the data from Table 8.2, show that the binding energy per nucleon for
Strontium-90 is 8.73 MeV.
Rest mass / u
90
Strontium, 38 Sr 89.9077 u
Proton, 11p 1.0078 u
1
Neutron, n 0 1.0087 u

Table 8.2

[2]

(ii) Hence calculate the total energy released during the nuclear fission reaction.
Data for binding energies per nucleon are shown in Table 8.3.

Isotope Binding energy per nucleon/MeV


Uranium-235 7.59
Xenon-143 8.41
Table 8.3

energy released = .............................. J [2]

NYJC 2013 JC2/Prelim/H2/9646/03 [Turn over


20
(iii) Explain quantitatively why the kinetic energy of the neutron directed at U-235 is For
neglected. Examiners
Use

... [1]

(iv) Give a reason why the total kinetic energy of the fission products and neutrons is
less than the value calculated in (b)(ii).

..

... [1]

(c) Fig. 8.4 shows the possible directions of Sr-90, Xe-143, and neutrons when nuclear
fission takes place. Assume that this is an isolated system.

Xe-143

U-235
slow-moving
neutron
Sr-90

Fig. 8.4

(i) Explain why it is unlikely for the two fission products to move in the same
direction after the fission process.

... [1]

NYJC 2013 JC2/Prelim/H2/9646/03


21
(ii) Explain why the total momentum of two fission products and neutrons after
reaction is not zero even though the total momentum of U-235 and slow moving For
neutron may be taken to be zero. Examiners
Use

... [2]

(d) Neutrons emitted from a nuclear fission may hit another U-235 nucleus, causing a
chain reaction.
(i) To control the reaction, neutrons emitted from a fission reaction may be slowed
down by Carbon-12 nuclei, 126 C . Show that when a neutron collides head-on with
a Carbon-12 nucleus as shown, the speed is reduced by about 15%.

at rest

C-12
neutron

[3]

(ii) On average, the neutron speed after each collision is 0.93 of its speed before the
collision.
Suggest why this speed reduction is different from what is stated in (ii) for a
nuclear reactor.

... [1]

NYJC 2013 JC2/Prelim/H2/9646/03 [Turn over


22
(iii) Suggest why a slow neutron has a higher chance of being captured by U-235 to For
cause a fission reaction compared to a fast neutron. Examiners
Use

... [1]

(e) The fission products are usually radioactive and give rise to a series of radioactive
decay products. Each decay product has its own half life, but eventually a stable
nuclide is reached.
One such fission product with its decay products and half life is shown below.


0.511 s 1.79 s

(i) Suggest how the number of Cs-143 nuclei inside the nuclear reactor may remain
constant even when it decays to form Ba-143.

... [1]

(ii) Explain why the Xenon-143 produced in the later part of the chain reaction may
not necessarily decay at a later time than those produced in the earlier part of the
chain reaction.

... [2]

NYJC 2013 JC2/Prelim/H2/9646/03

Vous aimerez peut-être aussi